Sie sind auf Seite 1von 587

General Adult Psychiatry I

Etiology
1
Marks: 0/1
Structural brain imaging studies have been used to elucidate the neurobiology of mood disorders.
Which ONE of the following abnormalities has NOT been shown in MRI studies of patients with major
depression?
Choose one answer.
a. Decreased grey matter volume of the subgenual pre-frontal cortex
b. Decreased volume of the basal ganglia structures
c. Decreased amygdala volume
d. Enlarged lateral ventricles
e. Decreased hippocampal volume
The size of the amygdala is increased in depression and not decreased. Enlarged lateral ventricles have
been found predominantly in the elderly with late-onset depression. Reduced hippocamapal volume is
seen mainly in those with unipolar depression. Decreased volume of the basal ganglia structures is seen
in those with unipolar depression. Decreased grey matter volume of the subgenual prefrontal cortex is
seen in those with both unipolar and bipolar depression.
Shorter Oxford Textbook of Psychiatry, 5th edition, eds Gelder M, Harrison P, Cowen P, p242
Incorrect
Marks for this submission: 0/1.
Question 2
Marks: 0/1
Many risk factors have been identified for developing depressive disorders. Which ONE of the
following statements regarding risk of developing depression is NOT true?
Choose one answer.
a. Risk is more increased in lower social classes than middle social classes following a life event
b. Risk is increased by having poor social support
c. Risk is increased if there is a first degree relative with bipolar affective disorder
d. Risk is increased in female heterosexuals compared to male homosexuals.
e. Risk in single women doubles in the presence of poverty
Research has documented consistently that persons of low-socioeconomic status (SES) are more
strongly affected emotionally by undesirable life events than those of higher status. Studies examining
sexual orientation and mental health have found a higher prevalence of anxiety, mood disorders,
substance misuse and suicide in homosexual compared with heterosexual populations. Single mothers
have twice the risk of developing depression and are twice as likely to have financial hardship. Having
a first degree relative with bipolar disorder also increases your risk of developing unipolar depression.
McLeod, J.D. and Kessler, R. C. (1990) Socioeconomic Status Differences in Vulnerability to
Undesirable Life Events. Journal of Health and Social Behaviour, 31, 2, 162-172.
Brown, B. W. and Moran, P. M. (1997) Single mothers, poverty and depression, Psychol Med, 27(1),
21-33.
Paykel, E.S. et al (1996) Life events, social support and marital relationships in the outcome of severe
depression. Psychol Med, 26(1), 121-33
Incorrect
Marks for this submission: 0/1.
Question 3
Marks: 0/1
Which ONE of the following anatomical regions has NOT been shown to have altered cerebral blood
flow and metabolism in studies of depressed patients using single-photon emission tomography
(SPECT) and positron emission tomography (PET)?
Choose one answer.
a. Dorsolateral prefrontal cortex
b. Anterior cingulate
c. Amygdala
d. Hippocampus
e. Basal ganglia
No abnormalities of the hippocampus have been found on PET/SPECT scans of patients with
depression. The dorsolateral prefrontal cortex is associated with cognitive dysfunction. The anterior
cingulate is associated with impaired attention and abnormal emotional processing. The amygdala is
associated with abnormal emotional processing. The basal ganglia are associated with impaired
incentive behaviour and psychomotor disturbances.
Shorter Oxford Textbook of Psychiatry, 5th edition, eds Gelder M, Harrison P, Cowen P, p243
Incorrect
Marks for this submission: 0/1.
Question 4
Marks: 0/1
Early enviromental influences might be involved in the aetiology of a depressive episode. Which ONE
of the following has NOT been shown to increase the risk of developing depression in later life?
Choose one answer.
a. Parental death in childhood
b. Maternal post-natal depression
c. Sexual abuse
d. Parental divorce
e. Non-caring and over-protective parenting
Epidemiological studies do not suggest that the death of a parent in childhood increases the risk of
depressive disorders in adult life. There is evidence that non-caring and overprotective parenting styles
are associated with non-melancholic depression in adult life. Depressive disorders in later life are
associated with parental separation, and especially divorce. There is a clear link between childhood
sexual abuse and adult depression. Post-natal depression can lead to a parenting style characterised by
neglect and emotional indifference, thus increasing the risk of depression in the next generation.
Shorter Oxford Textbook of Psychiatry, 5th edition, eds Gelder M, Harrison P, Cowen P, p233
Incorrect
Marks for this submission: 0/1.
Question 5
Marks: 0/1
A variety of neuro-endocrine abnormalities have been found in depressed subjects. Which of the
following endocrine abnormalities has NOT been found in depression?
Choose one answer.
a. Reduced free T3
b. Decreased levels of free thyroxine
c. Raised ACTH
d. Blunted TSH response to i.v. TRH
e. Blunted response of Growth Hormone to Clonidine
Free thyroxine levels are usually normal, but Clonidine-induced Growth Hormone response is blunted
in depression. Elevated cortisol, GH and CRH are also seen in depression. A blunt TSH response is
seen in one quarter of patients with depression, while T3 may be decreased.
Shorter Oxford Textbook of Psychiatry, 5th edition, eds Gelder M, Harrison P, Cowen P, p241.
New Oxford Textbook of Psychiatry, eds Gelder, Lopez-Ibor Jr, Andreason, 2000, p714
Incorrect
Marks for this submission: 0/1.
Question 6
Marks: 0/1
Life events (LE) have been implicated in the aetiology of many psychiatric disorders. Which ONE of
the following statements about life events is NOT true?
Choose one answer.
a. LE are more likely to occur prior to relapse of mania
b. LE are more likely to cause depression when there is lack of a confidant
c. LE are less important before first episode of depression than in relapse
d. LE more likely to occur before depression than schizophrenia
e. LE are more likely to cause depression when they are exit rather than entry life events
The relative risk of depression following life events is 6 compared with a relative risk of only 2 in
schizophrenia. In general, the importance of life events in the onset of a depressive episode decreases
as the number of episodes increases. The lack of a confidant was a vulnerability factor discovered by
Brown and Harris (1978). Loss events are more common in depression. There is a small excess of life
events prior to the relapse of mania.
Paykel, E.S. (1978) Contribution of life events to causation of psychiatric illness. Psychol Med, 8(2),
245-53.
Shorter Oxford Textbook of Psychiatry, 5th edition, eds Gelder M, Harrison P, Cowen P, p234-235
Incorrect
Marks for this submission: 0/1.
Question 7
Marks: 0/1
The dexamethasone suppression test (DST) suppresses cortisol via inhibition of ACTH at the pituitary
level. Which of the following statements about DST is true?
Choose one answer.
a. Non-suppression is seen in patients with schizophrenia
b. Non-suppression is less likely in those with melancholia
c. DST has been used to provide evidence to support the monoamine theory of depression
d. It can be used to diagnose depression
e. There is non-suppression in over 90% of depressed subjects
The DST is not sensitive or specific enough to be used diagnostically in depression. There is non-
suppression in only 50% of depressed patients - it is not a sensitive test. DST non-suppression is more
frequently seen in patients with melancholia. DST is used to show evidence of HPA axis involvement
and non-suppression is also found in mania and dementia.
Shorter Oxford Textbook of Psychiatry, 5th edition, eds Gelder M, Harrison P, Cowen P, p241
Incorrect
Marks for this submission: 0/1.
Question 8
Marks: 0/1
Genetic influences are acknowledged in the aetiology of mood disorders. Which ONE of the following
has NOT been shown in studies examining the genetics of mood disorder?
Choose one answer.
a. Individuals with the short allele of the serotonin transporter gene polymorphism are more likely
to become depressed following life events
b. The heritability of bipolar disorder is 85%
c. In bipolar disorder there is a susceptibility gene on chromosome 18 in some families
d. Relatives of unipolar probands have no increased risk of developing bipolar disorder
e. The risk of mood disorders in first degree relatives of those with bipolar disorder is increased 5-
fold
Twin studies at the Maudsley Hospital have revealed an 85% heritability for bipolar affective disorder.
There is an approximately 2-fold increased risk of mood disorders in first degree relatives of those with
bipolar disorder. Relatives of patients with unipolar disorder do not have increased rates of bipolar or
schizoaffective disorders. A susceptibility gene on Chr 18 has been reported for bipolar disorder in
genetic linkage studies. Association with the short allele of the 5HT transporter gene provides evidence
of the gene-environment link in the aetiology of depression.
Seminars in General Adult Psychiatry, 2nd edition, eds Stein G and Wilkinson G, p52.
Shorter Oxford Textbook of Psychiatry, 5th edition, eds Gelder M, Harrison P, Cowen P, p231-232
Incorrect
Marks for this submission: 0/1.
Question 9
Marks: 0/1
The monoamine theory of depression suggests that depressive disorder is due to abnormality in the
monoamine neurotransmitter system at one of more sites in the brain. Which ONE of the following
neurochemical findings in depression supports this theory?
Choose one answer.
a. Low CSF 5-HIAA
b. High levels of CSF homovanillic acid
c. Decreased platelet 5HT binding
d. Induction of depressive symptoms on administration of ?-methyl-para-tyrosine (AMPT) in
healthy subjects with a family history of depression
e. Increased plasma tryptophan
There is reduced plasma tryprophan (the precursor amino acid to serotonin) and decreased platelet-5HT
binding has been a finding in studies of the pathophysiology of depression. Low levels of 5-HIAA in
CSF is a finding that is more specifically linked with impulsive or aggressive behaviour. AMPT is used
to deplete noradrenaline; it has been shown to induce depressive symptoms only in patients with a past
history of depression. Low levels of homovanillic acid, a dopamine metabolite are found in depression.
Owens, M.J. and Nemeroff, C.B. (1994) Role of serotonin in the pathophysiology of depression: focus
on the serotonin transporter. Clinical Chemistry, 40, 288-295.
Shorter Oxford Textbook of Psychiatry, 5th edition, eds Gelder M, Harrison P, Cowen P, p239
Incorrect
Marks for this submission: 0/1.

Clinical Features
1
Marks: 0/1
Patients with depression show poor performance on several measures of neuropsychological function.
Which ONE of the following domains has NOT been shown to be impaired in depression?
Choose one answer.
a. Attention
b. Executive function
c. Retrograde memory
d. Language
e. Anterograde memory
There is no language impairment in depression. There is however impairment of executive function,
attention and memory (both retrograde and anterograde).
Shorter Oxford Textbook of Psychiatry, 5th edition, eds Gelder M, Harrison P, Cowen P, p244
Incorrect
Marks for this submission: 0/1.
Question 2
Marks: 0/1
Beck described a number of illogical ways of thinking in depression, which he referred to as 'cognitive
distortions'. Which ONE of the following distortions is NOT seen in depression?
Choose one answer.
a. Repression
b. Personalization
c. Selective abstraction
d. Arbitrary inference
e. Overgeneralization
Repression is a defence mechanism described in psychoanalytic theory, and is not a cognitive distortion
associated with depression. The others are examples of cognitive distortions seen in depression:
Arbitrary inference is the phenomenon of drawing a conclusion when there is no evidence for it and
some against it; Personalisation is the relating of external events to oneself in an unwarranted way;
Selective abstraction implies focusing on the detail and ignoring more important features of a situation;
Overgeneralization is the drawing of a general conclusion on the basis of a single incident.
Shorter Oxford Textbook of Psychiatry, 5th edition, eds Gelder M, Harrison P, Cowen P, p236
Incorrect
Marks for this submission: 0/1.
Question 3
Marks: 0/1
Which of the following is a recognised symptom of severe depression?
Choose one answer.
a. Night terrors
b. Parasomnias
c. Hypersomnia
d. Narcolepsy
e. Somnambulism
Most depressed patients experience difficulty in falling asleep, frequent awakenings during the night or
awakening very early in the morning. The opposite, excessive sleeping, occurs in a minority of severely
depressed patients. It can occur along with symptoms of 'atypical depression' which include increased
appetite and weight gain. Atypical depression (reversed biological symptoms) often occurs along with
excessive sensitivity to interpersonal rejection. Some authors claim that the diagnosis can be missed in
favour of a personality disorder. There are also some claims that atypical depression responds better to
MAOIs than to tricyclic antidepressants.
Companion to Psychiatric Studies, 7th edition, Churchill Livingstone.
Incorrect
Marks for this submission: 0/1.
Question 4
Marks: 0/1
Normal bereavement is most likely to include which ONE of the following symptoms?
Choose one answer.
a. Delusional beliefs that the deceased is still alive
b. Significant weight loss
c. Worthlessness
d. Persistent ideas of self-blame
e. Suicidal ideation
It is important to point out that no single feature is pathognomic of abnormal grief and it is the intensity
and duration of the features which determine the clinical picture. However, the presence of delusions
would be abnormal; ideas of worthlessness and persistent guilt are also more likely to be associated
with abnormal grief. In normal grief reaction the person loses appetite rather than significant weight.
Anger is mostly directed towards the deceased or the world rather than the self.
Oxford textbook of Pyschiatry, 4th edition, 2001, p 1143
Incorrect
Marks for this submission: 0/1.
Question 5
Marks: 0/1
Which ONE of the following symptoms is NOT concordant with a diagnosis of mania?
Choose one answer.
a. Clouded consciousness
b. Persecutory delusions
c. Stupor
d. Passivity
e. Periods of sadness
Clouding of consciousness suggests an organic cause. However, manic stupor and brief periods of
depressed mood are recognised phenomenon in mania. Persecutory delusions are also well recognised
in mania. First-rank symptoms have been reported in 10-20% of manic patients.
Shorter Oxford Textbook of Psychiatry, 5th edition, eds Gelder M, Harrison P, Cowen P, p223-230
Incorrect
Marks for this submission: 0/1.
Question 6
Marks: 0/1
The phenomenon of 'rapid cycling' has been described in bipolar affective disorder. Which ONE of the
following statements about rapid cycling bipolar disorder is true?
Choose one answer.
a. It occurs more frequently in men
b. It commonly occurs earlier in the course of the illness
c. It occurs in 5% of patients
d. It is associated with hypothyroidism
e. It is defined as at least six distinct episodes a year
Concomitant hypothyroidism with rapidly cycling bipolar disorder is common. Four episodes in 1 year
is the accepted definition. Rapid cycling occurs later in the course of illness and is seen at some stage
of illness in 15% of patients. It is more common in females.
Seminars in General Adult Psychiatry, 2nd edition, eds Stein G and Wilkinson G, p29
Incorrect
Marks for this submission: 0/1.
Question 7
Marks: 0/1
Regarding the course of mood disorders, which ONE of the following statements is true?
Choose one answer.
a. Bipolar I has a better outcome than bipolar II
b. 65% of depressed patients will have further episodes
c. Depressive episodes are shorter than Manic episodes
d. 30% of patients with depression develop a chronic, unremitting course
e. 90% of patients with mania have further episodes
Both manic and depressive episodes (treated and untreated) last about 6 months. 80% will have further
episodes, but only 10-20% develop a chronic, unremitting illness. The prognosis for mania is poorer
than for unipolar depression, and bipolar II has a better outcome than bipolar I.
Shorter Oxford Textbook of Psychiatry, 5th edition, eds Gelder M, Harrison P, Cowen P, p245
Incorrect
Marks for this submission: 0/1.
Question 8
Marks: 0/1
Sleep changes are noted in depression. Which ONE of the following changes in sleep architecture has
NOT been described in depression?
Choose one answer.
a. Decreased Stage 3 sleep
b. Decreased latency of rapid eye movement (REM) sleep
c. Decreased Stage 2 sleep
d. Reduction in sleep duration
e. Increase in proportion of REM sleep in the early part of the night
There is no reduction in fast-wave sleep (stage 2 sleep) in depression, but there is a reduction of slow
wave sleep (sleep stages 3 and 4). There is impairment of both sleep continuity and duration. REM
latency is shortened in depression and there is an increased proportion of REM sleep in the early part of
the night.
Shorter Oxford Textbook of Psychiatry, 5th edition, eds Gelder M, Harrison P, Cowen P, p242
Incorrect
Marks for this submission: 0/1.
Question 9
Marks: 0/1
Both DSM-IV and ICD-10 classify mood disorders. Which ONE of the following statements regarding
the classification of mood disorders is NOT correct?
Choose one answer.
a. Bipolar II disorder requires the presence of at least one hypomanic episode
b. In DSM-IV, manic episodes precipitated by antidepressants are not counted in the diagnosis of
bipolar disorder
c. In ICD-10, the presence of a single episode of mania or hypomania is enough to meet the
criteria for bipolar disorder
d. Depression is categorised according to severity in ICD-10
e. In DSM-IV, presence of a mixed affective state excludes bipolar II
There must be at least two episodes of mood disturbance for the diagnosis of Bipolar disorder
according to ICD-10. The diagnosis of Bipolar II disorder requires at least one hypomanic episode.
There is some argument as to whether mania/hypomania induced by antidepressants should be taken as
indicating an underlying bipolar disorder; in DSM-IV, mania or hypomania precipitated by
antidepressants does NOT count towards a diagnosis of bipolar disorder. The presence of a manic
episode or mixed affective state excludes bipolar II. Depression is categorised according to severity in
both ICD-10 and DSM-IV.
Shorter Oxford Textbook of Psychiatry, 5th edition, eds Gelder M, Harrison P, Cowen P, p227
Incorrect
Marks for this submission: 0/1.
Question 10
Marks: 0/1
Which of the following is used as a screening tool in psychiatry?
Choose one answer.
a. Simpson Angus Scale
b. Edinburgh Post Natal Depression Scale
c. MADRS
d. BDI
e. SANS
The Beck Depression Inventory (BDI) consists of 21 items which can be scored from 0-3. It is used to
assess the intensity of depression in clinical and normal patients and has good sensitivity and specificity
as a screening tool. The cut-off scores are: 0-13= minimal depression, 14-19= mild depression, 20-28=
moderate depression, 29-63 severe depression.
Snijders et al. J Neurol Neurosurg Psychiatry.2006; 77: 787-789
Incorrect
Marks for this submission: 0/1.

Epidemiology

1
Marks: 0/1
Epidemiological studies of Bipolar Disorder have shown which ONE of the following?
Choose one answer.
a. There is an increase of manic episodes in summer months
b. Female patients have proportionally more manic episodes than men
c. The lifetime risk is 2.5%
d. There is a female preponderance
e. Age of onset is later than in unipolar disorder

There is an increase of manic episodes in the summer months, although there is rarely a seasonal
pattern of episodes within individual patients. Studies have shown that women tend to have fewer
manic episodes, but prevalence is equal in males and females. The mean age of onset is earlier than of
unipolar disorders, at 17 years. The lifetime risk is between 0.3-1.5%.

Shorter Oxford Textbook of Psychiatry, 5th edition, eds Gelder M, Harrison P, Cowen P, p230

Incorrect
Marks for this submission: 0/1.
Question 2
Marks: 0/1
Epidemiological studies of depression have shown which ONE of the following?
Choose one answer.
a. The gender difference becomes more exaggerated with increasing age
b. Increased prevalence in Afro-Caribbean individuals
c. Age of onset is usually in late adolescence
d. Risk of major depression has increased over recent decades
e. Increased rates in Japan
Studies have shown increased risk of depression over the last few decades. A recent WHO
commissioned study reported in the Lancet in March 2008 reporst that depression will be the 2nd
leading cause of morbidity worldwide by 2030. The gender difference in depression persists across the
age range. The mean age of onset is 27 years. When socio-economic and education variables are
controlled for, there is minimal evidence of racial or ethnic differences in the prevalence of depression.
The ECA study showed lifetime prevalence of only 3%. New Oxford Textbook of Psychiatry, eds
Gelder, Lopez-Ibor Jr, Andreason, 2000, p698. Seminars in General Adult Psychiatry, 2nd edition, eds
Stein G and Wilkinson G, p49-51
Incorrect
Marks for this submission: 0/1.

Management

1
Marks: 0/1
The following are recommended in the treatment of mild depression except:
Choose one answer.
a. Exercise
b. Computerised CBT
c. Individual CBT
d. Antidepressants
e. Guided self help

Antidepressants are not recommnded mild depression as the risk benefit ratio is poor.

Depression: management of depression in primary and secondary care - NICE guidance,2007 (Click
here)

Incorrect
Marks for this submission: 0/1.
Question 2
Marks: 0/1
Which rating scale would you use to assess the response to antidepressants in a woman who has
recently given birth and appears depressed ?
Choose one answer.
a. MADRS
b. BPRS
c. EPDS
d. EAT
e. HAMD

The Edinburgh Postnatal Depression Scale (EPDS) is a 10-item self-rating post-natal depression scale
which has seen widespread use in epidemiological and clinical studies.

Detection of postnatal depression: development of the 10- item Edinburgh Postnatal Depression Scale.
British Journal of Psychiatry, 150,782-876

Incorrect
Marks for this submission: 0/1.
Question 3
Marks: 0/1
A range of psychotropics are used in the prophylactic treatment of bipolar disorder. Which ONE of the
following statements about prophylactic treatment in bipolar disorder is NOT true?
Choose one answer.
a. Lithium is associated with reduced risk of suicide
b. Cognitive therapy has proven efficacy in the prevention of relapse
c. Carbamazepine and lithium combined are more effective than either drug alone
d. Olanzapine prevents more manic than depressive relapses
e. Carbamazepine is as effective as lithium

Lithium has superior efficacy to carbamazepine. Olanzapine is more effective in preventing manic
episodes. There is a link between lithium treatment and reduced suicide risk. There is evidence that
cognitive therapy in bipolar disorder is effective in relapse prevention. The combination of these
lithium and carbamazepine has superior efficacy than when they are used alone.

Goodwin, G.M. et al (2003) Evidence-based guidelines for treating bipolar disorder: Recommendations
from the British Association for Psychopharmacology. Journal of Psychopharmacology. 17(2):149-173
Incorrect
Marks for this submission: 0/1.
Question 4
Marks: 0/1
Patients differ in their response to lithium therapy. Factors which predict a good response to lithium
include which ONE of the following?
Choose one answer.
a. First episode of depression as opposed to mania
b. Poor inter-episode functioning
c. Neurological signs
d. Rapid cycling
e. Family history of bipolar disorder

A family history of bipolar disorder is strongly predictive of prophylactic efficacy. Neurological signs
predict a poor response to lithium. Cases where the first episode was manic do better on lithium. Rapid
cyclers respond poorly to lithium. Good inter-episode functioning is predictive of a good response.

Seminars in General Adult Psychiatry, 2nd edition, eds Stein G and Wilkinson G, p40

Incorrect
Marks for this submission: 0/1.
Question 5
Marks: 0/1
Which of the following options can be considered in the initial treatment of severe depression:
Choose one answer.
a. Antidepressants alone
b. Antidepressants + individual CBT
c. None of the above
d. CBT alone
e. Augmentation

The combination of antidepressanats and individual CBT should be considered in the initial
management as it is more cost effective than either on its own.

Depression: management of depression in primary and secondary care - NICE guidance, 2007. Click
here for the link to the guideline

Incorrect
Marks for this submission: 0/1.
Question 6
Marks: 0/1
A patient on Clozapine complains of abdominal pain and appears confused. The most likely
explanation for his symptoms would be:
Choose one answer.
a. Hyperglycemia
b. Hypercalcaemia
c. Hypoglycemia
d. Hypokalemia
e. Hyponatremia

Although upto 30% of patients on Clozapine may develop diabetes over 5 years, severe hyperglycemia,
sometimes leading to ketoacidosis is rare. It has been reported in a number of case reports during
Clozapine treatment in patients with no prior history of hyperglycemia.

Use of drugs in psychiatry, Cookson, Gaskell, 2002

www.clozaril.com

Incorrect
Marks for this submission: 0/1.
Question 7
Marks: 0/1
Electroconvulsive therapy (ECT) has been associated with cognitive side effects. Which ONE of the
following statements is NOT true about the effects of ECT on cognition?
Choose one answer.
a. Cognitive effects are increased with high doses of anaesthetic agents
b. Bilateral electrode placement produces greater impairment than unilateral
c. Anterograde amnesia is associated with more difficulties over the long-term than retrograde
amnesia
d. Sine waveform is associated with increased cognitive impairment
e. Cognitive impairment is dose-dependent

Retrograde amnesia is thought to cause more difficulties over the long-term than anterograde amnesia.
The other statements are true: high-dose ECT produces greater impairment than low-dose; bilateral
ECT produces greater impairment than unilateral ECT; sine waves are associated with increased
cognitive impairment; high doses of anaesthetic are associated with adverse cognitive effects in ECT.

Seminars in General Adult Psychiatry, 2nd edition, eds Stein G and Wilkinson G, p119

Incorrect
Marks for this submission: 0/1.
Question 8
Marks: 0/1
The following drugs are recommended for post-stroke depression except:
Choose one answer.
a. Mianserin
b. Fluoxetine
c. Nortriptylline
d. Citalopram
e. Imipramine

Mianserin seems ineffective, and SSRIs & Nrotriptyline are recommended antidepressants for
treatment of post stroke depression.
Maudsley guidelines 2007, 9th edition,p-213

Incorrect
Marks for this submission: 0/1.
Question 9
Marks: 0/1
Seasonal Affective Disorder (SAD) has been shown to respond to phototherapy. Which ONE of the
following statements about phototherapy is true?
Choose one answer.
a. Early-morning treatment is superior
b. Response is independent of duration
c. 70% of patients with SAD will respond to phototherapy
d. It is important that some skin is exposed to the light
e. Response is independent of the intensity of light

Early morning phototherapy is superior but leads to increased side effects. About a half of patients have
a clinically significant response to the treatment. Response is dependent on the intensity of the light. It
is important for the eyes to be exposed as skin absorption does not alter circadian rhythms and is
ineffective in SAD. The longer the duration of phototherapy, the better the response.

Seminars in General Adult Psychiatry, 2nd edition, eds Stein G and Wilkinson G, p13

Incorrect
Marks for this submission: 0/1.
Question 10
Marks: 0/1
The antidepressant of choice in a patient with recent history of MI or unstable angina is:
Choose one answer.
a. None of the above
b. Venlafxaine
c. Sertraline
d. Fluoxetine
e. Mirtrazepine

The best evidence for antidepressant use in patients with cardiovascular disease is avalable for
Sertraline.

Depression: management of depression in primary and secondary care - NICE guidance, 2007. (Click
here)

Incorrect
Marks for this submission: 0/1.
Question 11
Marks: 0/1
If depression has not responded to 2 adequate trials of different antidepressants the following can be
considered
Choose one answer.
a. None of the above
b. Augmenting antidepressnat with antidepressant
c. Venlafaxine
d. All of the above
e. Lithium augmentation
All the above are options in the routine management of treatment ressitant depression
Nice guidelines for depression ,2004
Incorrect
Marks for this submission: 0/1.
Question 12
Marks: 0/1
A 40 year old woman with diagnosis of Bipolar Affective Disorder has had over the last few years four
episodes of depression and one episode of hypomania. Which of the following drugs should be used as
maintenance?
Choose one answer.
a. Lamotrigine
b. Valproate
c. Lithium
d. Carbamazepine
e. Olanzapine

The anticonvulsant Lamotrigine, recently approved as maintenance therapy for bipolar depression, has
undergone evaluation as acute and maintenance therapy for bipolar disorder in several controlled
clinical trials. NICE guidelines advocate the use of Lithium, Olanzapine, and valproate for prophylaxis
of Bipolar disorder; Lamotrigine is recommended to be used in Bipolar II disorder and recurrent
depression. Depression mood stabilisers, which stabilise mood from below the mood baseline
(euthymia), without inducing switch into mania or episode acceleration, are useful for the treatment of
bipolar depression.

European Neuropsychopharmacology, 2004 May;14 Suppl 2:S89-93. NICE guidelines on Bipolar


disorder, 2006.

Incorrect
Marks for this submission: 0/1.
Question 13
Marks: 0/1
A man would like to take a herbal antidepressant. Which herbal antidepressant would you recommend?
Choose one answer.
a. Ginsavena
b. Angus Castus
c. Would not recommend
d. Avena Sativa
e. Hypericum perforatum
Although there is evidence that extracts of hypericum are more effective than placebo for the treatment
of mild to moderately severe depressive disorders (see reference), these herbs have not been the subject
of any rigorous trial and there are significant problems with interactions with other medications like
warfarin and oral contraceptives. Serotonin syndrome is a possibility should the patient continue to take
the prepartion along with a prescribed SSRI. These drugs are best avoided.

Linde, BMJ 1996;313:253-258

Incorrect
Marks for this submission: 0/1.
Question 14
Marks: 0/1
The duration of maintence treatment with antidepressants in patients with more than two depressive
episodes is:
Choose one answer.
a. 6 months
b. 2years
c. 5 years
d. none
e. 1 year

Patients with more than 2 depressive episodes should be advised to continue antidepressants for 2years.

Depression: management of depression in primary and secondary care - NICE guidance, 2007 (Click
here)

Incorrect
Marks for this submission: 0/1.
Question 15
Marks: 0/1
Many studies have examined the efficacy of antidepressant treatment in different situations. Which
ONE of the following findings relating to antidepressant efficacy is NOT true?
Choose one answer.
a. Antidepressants are not effective in acute mild depression
b. Response to antidepressants is unrelated to prior life events
c. Tricyclics are more effective than SSRIs in hospitalised patients
d. Antidepressants are not effective in the treatment of dysthymia
e. Venlafaxine at doses of over 150mg/day is more effective than SSRIs in treating severe
depression

Antidepressants are effective in the treatment of dysthymia. Tricyclic antidepressants are more
effective in hospitalised patients. There is some evidence that Venlafaxine is more effective at higher
doses in treating moderate to severe depression. Antidepressants show no effect in acute mild
depression. Response to antidepressants is unrelated to prior life events but response to placebo
appears to be higher in those with prior life events.
Anderson, et al (2000). Evidence-based guidelines for treating depressive disorders with
antidepressants: a revision of the 1993 British Association for Psychopharmacology guidelines. Journal
of Psychopharmacology 14(1):3-20

Incorrect
Marks for this submission: 0/1.
Question 16
Marks: 0/1
Which ONE of the following strategies is NOT advocated in the treatment of bipolar depression?
Choose one answer.
a. Tricyclic antidepressant
b. CBT
c. Lithium
d. Lamotrigine
e. IPT
Tricyclics are not recommended because of the risk of switching to mania. There is good evidence to
support the use of lamotrigine in bipolar depression. CBT is effective and safe in bipolar depression.
IPT is effective and safe in bipolar depression. Lithium can be used in the treatment of bipolar
depression.
Goodwin, G.M. et al (2003) Evidence-based guidelines for treating bipolar disorder: Recommendations
from the British Association for Psychopharmacology. Journal of Psychopharmacology. 17(2):149-173
Incorrect
Marks for this submission: 0/1.
Question 17
Marks: 0/1
A man comes to your clinic nine months after the death of his mother. He occasionally hears her voice
calling him. You find features of a moderate depressive illness. Your best course of action is to:
Choose one answer.
a. Start an antipsychotic and follow him up
b. Do nothing and reassure him it will all go away
c. Add a mood stabiliser
d. Start an antidepressant and follow him up.
e. Bereavement counselling

The differentiation between a normal grief reaction and depression/abnormal grief does not depend on
any specific symptom. The symptoms of depression and abnormal grief are similar. It is the intensity
and duration of the symptoms which are important in making the clinical differentiation. In this case it
appears that the man is suffering from a prolonged grief reaction (moderate derpessive symptoms at 9
months post bereavement) and might benefit from antidepressant treatment.

Oxford textbook of Pyschiatry, 4th edition, 2001,p1143

Incorrect
Marks for this submission: 0/1.
Question 18
Marks: 0/1
The first line of treatment for moderate-to-severe depression in the UK is
Choose one answer.
a. None of the above
b. TCA's
c. Mirtrazepine
d. Venlafaxine
e. SSRI's

SSRI's are as potent as TCA's and not likely to be discontinued

Depression: management of depression in primary and secondary care - NICE guidance. Click here for
the guidance

Incorrect
Marks for this submission: 0/1.
Question 19
Marks: 0/1
Cognitive behavioural therapy (CBT) is increasingly being used in the treatment of mood disorders.
Which ONE of the following statements regarding the efficacy of CBT is TRUE?
Choose one answer.
a. There is no neurobiological basis for the efficacy of CBT
b. CBT is more effective than interpersonal therapy in the treatment of mild-moderate depression

c. Cognitive therapy is more effective than behavioural therapy


d. CBT is as effective as antidepressants in severe depression
e. When treatment is withdrawn, relapse rates are lower for CBT than antidepressants

Relapse rates are less after withdrawal of CBT rather than antidepressants. CBT and IPT have been
shown to have equal efficacy for mild-moderate depression. CBT is less effective than antidepressants
in severe depression. PET studies have shown significant metabolic changes that are thought to
underlie the effect of CBT. Cognitive therapy and behavioural therapy are equally effective.

Goldapple, K (2004) Modulation of Cortical-Limbic Pathways in Major Depression: Treatment-


Specific Effects of Cognitive Behaviour Therapy. Arch Gen Psych. 61:34-41.

Seminars in General Adult Psychiatry, 2nd edition, eds Stein G and Wilkinson G, p133

Incorrect
Marks for this submission: 0/1.

Aetiology
1
Marks: 0/1
In relation to cannabis use and public health, which ONE of the following statements is TRUE?
Choose one answer.
a. Recent evidence suggests that cannabis use probably causes more harm to public health than
tobacco or alcohol use.
b. Cannabis use appears to have dramatically increased since the recent reclassification of
cannabis under the UK Misuse of Drugs Act.
c. Preventing cannabis use may lead to reductions in population rates of psychosis.
d. If cannabis use does not cause psychosis then there is no public health justification for
preventing cannabis use.
e. There is good evidence that the prohibition of cannabis use is an effective strategy to reduce
use among young people.
Evidence of an association between cannabis use and psychosis has recently emerged. By normal
epidemiological conventions, evidence that this association has a causal basis is currently not strong.
However, cannabis may cause psychosis and this possibility presents a means to effectively reduce the
population burden of illnesses such as schizophrenia.
McGrath & Saha, 2007. Cannabis use and psychosis: the origins and implications of an association.
Advances in Psychiatric Treatment, 13: 400-411.
Incorrect
Marks for this submission: 0/1.
Question 2
Marks: 0/1
Ketamine and phenylcyclidine can induce schizophrenia-like psychosis in humans. This provides
evidence for the role of which neurotransmitter in the aetiology of schizophrenia?
Choose one answer.
a. Acetylcholine
b. Dopamine
c. Gamma aminobutyric acid (GABA)
d. Serotonin
e. Glutamate
Ketamine and phenylcyclidine act as antagonists at the NMDA glutamate receptor. GABA,
acetylcholine or dopamine transmission is not affected.
Shorter Oxford Textbook of Psychiatry, 5th edition, eds Gelder M, Harrison P, Cowen P, p289
Incorrect
Marks for this submission: 0/1.
Question 3
Marks: 0/1
With regard to cannabis use and psychotic illness in the UK population over the past 30 years, which
ONE of the following statements is TRUE?
Choose one answer.
a. Only a small minority of young people today are likely to have used cannabis.
b. Rates of psychosis have clearly increased.
c. Rates of psychosis have clearly fallen.
d. Use of cannabis by young people has increased.
e. If cannabis use causes psychosis, increasing rates of cannabis use should have led to increasing
rates of psychosis.

In the UK around half of adolescents will use cannabis at least once and about a fifth of them will use it
regularly (monthly or more frequently) in young adulthood. Cannabis is currently a Category C drug,
however the home secretary has advised that is should be reclassified as a Category B drug from 2009.
The reclassification reflects the market for cannabis in the UK being dominated by a more potent form
of cannabis, skunk.
McGrath & Saha, 2007. Cannabis use and psychosis: the origins and implications of an association.
Advances in Psychiatric Treatment, 13: 400-411.

Incorrect
Marks for this submission: 0/1.
Question 4
Marks: 0/1
Functional and structural neuroimaging has revealed a variety of differences in the brains of
schizophrenics compared with healthy controls. Which ONE of the following brain abnormalities has
been found in schizophrenia?
Choose one answer.
a. Increased volume of the hippocampus and parahippocampal gyrus
b. Heavier brains than healthy individuals
c. Reduced numbers of glutamate receptors in the frontal areas of the brain
d. Increased cerebral blood flow in the prefrontal cortex
e. Decreased expression of 5HT2A receptors in the prefrontal cortex
The decreased expression of 5HT2A receptors in prefrontal cortex gives weight to the serotonin
hypothesis of schizophrenia. Structural imaging studies consistently show reduced volume of
hippocampus and parahippocampal structures. Glutamate receptors are increased in frontal areas and
reduced in the medial temporal lobe. There is reduced blood flow in the prefrontal cortex indicating
'hypofrontality'. There is decreased brain weight compared with healthy individuals.
Shorter Oxford Textbook of Psychiatry, 5th edition, eds Gelder M, Harrison P, Cowen P, p286-289.
Incorrect
Marks for this submission: 0/1.
Question 5
Marks: 0/1
An increased rate of schizophrenia-like psychosis has been observed in individuals with
velocardiofacial syndrome (VCFS). Which candidate gene in the aetiology of schizophrenia has been
implicated in VCFS?
Choose one answer.
a. D-amino acid oxidase activator (G72)
b. Disrupted in schizophrenia 1 (DISC-1)
c. Catechol-O-methyl transferase (COMT)
d. Dopamine D3 receptor (DRD3)
e. Neuregulin (NRG-1)

Velocardiofacial syndrome involves a microdeletion in chromosome 22 (deletion of 22q11); the COMT


gene is found on 22q11.

Shorter Oxford Textbook of Psychiatry, 5th edition, eds Gelder M, Harrison P, Cowen P, p284

Incorrect
Marks for this submission: 0/1.
Question 6
Marks: 0/1
Individuals with Schizoptypy:
Choose one answer.
a. Often show poor social interactions
b. will eventually develop schizophrenia
c. Do not experience psychotic symptoms
d. Are not considered as having a schizophrenia-spectrum disorder
e. Do not share common genetic vulnerabilities with schizophrenia

Schizotypy is a concept which describes a continumm of personality characteristics and experiences


related to psychosis, in particular schizophrenia. The characteristics include poor social interactions,
unusual experiences (hallucinations, magical thinking, delusions), cognitive disorginsation, & non-
conformity. Though this schizophrenia spectrum disorder is considered to share common genetic
vulnerabilities, the relationship with schizophrenia is complex and not well understood.

Davies E, Developmental aspects of schizophrenia and related disorders: possible implications for
treatment strategies. Advances in Psychiatric Treatment (2007) 13: 384-391

Incorrect
Marks for this submission: 0/1.
Question 7
Marks: 0/1
Which ONE of the following conditions has been associated with catatonia?
Choose one answer.
a. Dopamine overactivity
b. Serotonin deficiency
c. GABA deficiency
d. Glutamate deficiency
e. Cholinergic deficiency

Various neurotransmitter abnormailities have been reported in Catatonia, including reduced GABA &
Dopamine, and increased Glutamate, Acetylcholine and Serotonin.

Rajagopal, S. Catatonia. Advances in Psychiatric Treatment (2007) 13: 51-59

Incorrect
Marks for this submission: 0/1.
Question 8
Marks: 0/1
Which of the following is true regarding the genetics of schizophrenia?
Choose one answer.
a. Genetic evidence is inconsistent with the neurodevelopment hypothesis of schizophrenia
b. Genes involved in synaptic function determine susceptibility to schizophrenia.
c. Schizophrenia has a small genetic component.
d. Genetic studies do not support the concept of schizotaxia.
e. Genetic predisposition always leads to schizotypy or schizophrenia.
Schizophrenia and other schizophrenia-spectrum disorders are neurodevelopmental disorders which
may share genetic susceptibility factors and represent different expressions of an underlying
vulnerability. The relationship between spectrum disorders and schizophrenia is complex, with not all
individuals who display characteristics of a prodrome going on to develop schizophrenia.
Davies, E. Developmental aspects of schizophrenia and related disorders: possible implications for
treatment strategies. Advances in Psychiatric Treatment (2007) 13: 384-391
Incorrect
Marks for this submission: 0/1.
Question 9
Marks: 0/1
Genetic studies like linkage analysis and association studies have attempted to identify genes
specifically associated with schizophrenia. Which ONE of the following has been strongly
associated with susceptibility to schizophrenia?
Choose one answer.
a. Serotonin transporter
b. Dysbindin
c. HNF4-alpha
d. CHEK-2
e. BRCA-1

Dysbindin (also known as dystrobrevin binding protein 1) is postulated to be a schizophrenia


susceptibility gene. Located on chromosome 6p, it encodes a neural protein that binds to beta-
dystrobrevin, which is located at pre-synaptic terminals and participates in signal transduction. BRCA-
1 is a susceptibility gene for breast cancer. HNF4-alpha is a susceptibility gene for diabetes, and
CHEK-2 is associated with predisposition to prostate and breast cancers.

Companion to Psychiatric Studies, 7th edition, eds Johnstone, Cunningham, Owens, Lawrie, Sharpe,
Freeman, p400.

Bray, N.J. et al (2005). Haplotypes at the dystrobrevin binding protein 1 (DTNBP1) gene locus mediate
risk for schizophrenia through reduced DTNBP1 expression. Human Molecular Genetics, 14(14),1947-
1954.

Incorrect
Marks for this submission: 0/1.
Question 10
Marks: 0/1
Research into schizophrenia has examined the unaffected relatives (who share some genetic material
with the patient) in order to elucidate the aetiology of the disorder. Which ONE of the following has
NOT been found in first degree relatives of schizophrenic patients?
Choose one answer.
a. A 10-fold increase in their risk of developing schizophrenia.
b. Smooth-pursuit eye-tracking abnormalities.
c. An increased risk of bipolar affective disorder.
d. Reduced volumes of hippocampal structures.
e. Evoked potential abnormalities.

There is no increased risk of bipolar disorder in relatives of schizophrenia patients. There is a 10-20
fold increased risk of developing schizophrenia in siblings and offspring of patients, but the risk is
lower in parents of schizophrenics. High-risk subjects had significantly reduced mean volumes of the
left and right amygdala, hippocampus and thalamus when compared to healthy controls. The amplitude
of the p300 wave (a measure of auditory information processing) is reduced in patients and a
proportion of their first-degree relatives. 25% of first-degree relatives have defective performance in
tests of eye tracking.

Shorter Oxford Textbook of Psychiatry, 5th edition, eds Gelder M, Harrison P, Cowen P, p282-289.

Lawrie, S. et al (2001) Brain structure, genetic liability and psychotic symptoms in subjects at high risk
of developing schizophrenia. Biol Psychiatry. 5;49(10):811-23

Incorrect
Marks for this submission: 0/1.
Question 11
Marks: 0/1
Cognitive impairments in schizophrenia have been increasingly recognised using neuropsychological
methods. They are associated with poor functional outcome. Which ONE of the following statements
about neuropsychological abnormalities in schizophrenia is TRUE?
Choose one answer.
a. There are abnormalities in face recognition
b. There is impaired implicit memory
c. They are not present premorbidly
d. They show no resolution with treatment of the acute episode
e. There is increased prepulse inhibition

Abnormalities in face recognition in schizophrenia may be related to dysfuction in the frontal cortex
and amygdala. Schizophrenic patients are known to suffer from information processing deficits which
can be detected in deficits of prepulse inhibition. Implicit memory is preserved. Cognitive impairments
can partially resolve after resolution of the acute episode. Schizophrenic patients have been shown to
have a number of cognitive deficits premorbidly, including lower premorbid IQ.

Perry, W. et al (2002) Information Processing Deficits in Acutely Psychotic Schizophrenia Patients


Medicated and Unmedicated at the Time of Admission. Am J Psychiatry, 159, 1375-1381.

Shorter Oxford Textbook of Psychiatry, 5th edition, eds Gelder M, Harrison P, Cowen P, p291,272.

Incorrect
Marks for this submission: 0/1.
Question 12
Marks: 0/1
If both parents of a child have schizophrenia, the risk of the child developing schizophrenia is?
Choose one answer.
a. 40-50%
b. 80%
c. 20-30%
d. 10-20%
e. 30-40%
Risk of developing schizophrenia in children whose biological parents both have the disorder is 46.3%.
If a single parent is affected the risk to the child is 6%. An indentical twin with the illness also confers
a 46% chance whereas a sibling or dizygotic twin with the disorder confers a 10% risk of developing
the illness for the proband. A sibling and a parent being affected brings the risk to 17%. Having a child
with schizophrenia confers a 13% risk on the parent. And finally, having an uncle, aunt, nephew, or
niece with the illness confers a 3-4% lifetime risk.

Examination Notes in Psychiatry,Basic Sciences, 2nd Edition, Gin Malhi & Saj Malhi (2006), Hodder
Arnold, London.

Incorrect
Marks for this submission: 0/1.
Question 13
Marks: 0/1
With regards to schizophrenia in childhood which ONE of the following statements is TRUE?
Choose one answer.
a. Outcome of childhood-onset schizophrenia is better than that of adolescent onset schizophrenia

b. A diagnosis of schizophrenia does not have predictive validity


c. Schizotypal features cannot occur
d. Interventions do not have any effect on measures of schizotypy in adult life.
e. Younger age correlates with higher thought disorder scores

Schizophrenia and other schizophrenia-spectrum disorders are neurodevelopmental disorders which


may share genetic susceptibility factors and represent differential expressions of an underlying
vulnerability. The identification of prodrome (which includes features of spectrum disorders), could
have impact of early interventions.The outcome of childhood onset schizophrenia is worse than
adolescent onset schizophrenia.

Davies E, Developmental aspects of schizophrenia and related disorders: possible implications for
treatment strategies. Advances in Psychiatric Treatment (2007) 13: 384-391

Incorrect
Marks for this submission: 0/1.
Question 14
Marks: 0/1
Regarding the evidence that cannabis use may cause psychotic illness, which ONE of the following
statements is TRUE?
Choose one answer.
a. An increased frequency of reported cannabis use has been associated with increased reporting
of unusual thoughts and perceptions.
b. Confounding occurs when people exaggerate both their use of cannabis and their experience of
psychotic symptoms.
c. Cannabis use is associated with increased diagnosis of schizophrenia.
d. It is likely that the association between cannabis use and psychotic symptoms seen in several
studies has arisen by chance.
e. The fact that there are plausible neurophysiological mechanisms through which cannabis use
might cause psychosis is probably the strongest evidence that it does.
Since the late 1960s cannabis use has increased substantially in most high-income countries (Hickman
et al, 2007). The increase may now be levelling off, but this is hardly a reason for complacency.
Cannabis is now well established as the third most widely used psychoactive drug (after alcohol and
tobacco) in Europe, the USA and Australasia (Advisory Council on the Misuse of Drugs, 2006). In the
UK around half of adolescents will use cannabis at least once and about a fifth of them will use it
regularly (monthly or more frequently) in young adulthood.

McGrath & Saha, 2007. Cannabis use and psychosis: the origins and implications of an association.
Advances in Psychiatric Treatment, 13: 400-411.

Incorrect
Marks for this submission: 0/1.
Question 15
Marks: 0/1
Development factors associated with schizophrenia ...
Choose one answer.
a. Do not include educational difficulties.
b. Do not include abnormal motor development.
c. Do not include poor social adjustment.
d. May involve late neurodevelopmental influences.
e. Are specific to schizophrenia

The prevalence of schizophrenia in childhood is low, but children who develop schizophrenia in adult
life may show subtle and non-specific developmental abnormalities, consistent with the
neurodevelopmental hypothesis. These include motor, social, and cognitive (educational) difficulties.
The neurodevelopmental influences can be early (peirnatal) or later (in childhood).

Davies E, Developmental aspects of schizophrenia and related disorders: possible implications for
treatment strategies. Advances in Psychiatric Treatment (2007) 13: 384-391

Incorrect
Marks for this submission: 0/1.
Question 16
Marks: 0/1
Many psychosocial factors have been associated with the development of schizophrenia. Which ONE
of the following psychosocial factors has NOT been shown to be associated with an increased risk of
developing schizophrenia?
Choose one answer.
a. Low IQ
b. In adopted subjects whose biological parents have a personality disorder
c. Being born between July to September
d. Maternal food deprivation
e. Those who exhibit less social play as children

Increased rates of schizophrenia have been found in those born in January to March. Increased rates of
schizophrenia have been found in the offspring of women who suffered severe food deprivation during
the first trimester. Low IQ is associated with birth trauma and neurodevelopmental impairment. It was
found by Kety in the Danish adoption study that biological relatives had higher rates of DSM-III
schizotypal personality disorder. Jones (1994) found that children who eventually developed
schizophrenia showed delayed developmental milestones, speech problems, lower educational test
scores and less social play.

Companion to Psychiatric Studies, 7th edition, eds Johnstone, Cunningham Owens, Lawrie, Sharpe,
Freeman, p400.

Kety ,S.S. et al (1988) Schizophrenic illness in the families of schizophrenic adoptees: findings from
the Danish national sample. Schizophr Bull. 14(2), 217-22.

Incorrect
Marks for this submission: 0/1.
Question 17
Marks: 0/1
Many studies have attempted to explain the social correlates of schizophrenia. Which ONE of the
following social factors has NOT been associated with increased risk of schizophrenia?
Choose one answer.
a. Childhood sexual abuse
b. Urban birth
c. Being born into a socially deprived household
d. Critical comments from family members
e. Migration to developed countries
There is a link between childhood sexual abuse and psychotic symptoms in later life, but a link with
schizophrenia per se is not convincing. The first admission rate for schizophrenia is generally higher in
urban than rural areas, and much higher from the central areas of large cities than from the surrounding
suburbs. The relative risk for migrants is 2.7. Patients with schizophrenia are more likely to have been
born into socially deprived households. Individuals who live in households where there is a high level
expressed emotion from relatives (including critical comments) have a 2- to 3-fold increased risk of
schizophrenia (as well as increased risk of relapse in those already with a diagnosis).
Shorter Oxford Textbook of Psychiatry, 5th edition, eds Gelder M, Harrison P, Cowen P, p290.
Spataro, J and Mullen, P (2004) Impact of childhood sexual abuse on mental health. Prospective study
in males and females. The British Journal of Psychiatry. 184: 416-421
Incorrect
Marks for this submission: 0/1.

Clinical Features

1
Marks: 0/1
Which ONE of the following statements about primary negative symptoms in schizophrenia is NOT
true?
Choose one answer.
a. They occur later in the illness.
b. They can be due to neuroleptic medication.
c. They are associated with structural abnormalities in the prefrontal cortex.
d. Blunting of affect is seen.
e. They are associated with poor prognosis.

Secondary, but not primary negative symptoms may be related to medication. Primary negative
symptoms occur late in the natural history of the illness. Structural abnormalities in the parts of the
brain responsible for executive function are associated with negative symptoms; they indicate a poor
prognosis. Affective blunting is one of the core features of the negative syndrome.

Shorter Oxford Textbook of Psychiatry, 5th edition, eds Gelder M, Harrison P, Cowen P, p268-270,295

Incorrect
Marks for this submission: 0/1.
Question 2
Marks: 0/1
According to ICD-10, which ONE of the following disorders is a recognised subtype of schizophrenia?
Choose one answer.
a. Undifferentiated
b. Paraphrenia
c. Deficit syndrome
d. Disorganized
e. Bouffee delirante

Undifferentiated schizophrenia is a term used in ICD-10 to describe a syndrome which does not fit
readily into any of the defined subtypes of schizophrenia. There is no category for paraphrenia or late-
onset schizophrenia. Bouffee Delirante is now classified as an acute and transient psychotic disorder
and hence resolves rapidly. Disorganised is an equivalent term for hebephrenic schizophrenia, which is
used in DSM-IV. Deficit syndrome describes a syndrome with early severe and persistent negative
symptoms.

Shorter Oxford Textbook of Psychiatry, 5th edition, eds Gelder M, Harrison P, Cowen P, p270.

Incorrect
Marks for this submission: 0/1.
Question 3
Marks: 0/1
Cannabis has been highlighted, albeit controversially, as a potential causative factor in schizophrenia.
Which ONE of the following has NOT been found in studies examining the link between cannabis use
and schizophrenia?
Choose one answer.
a. If cannabis were eliminated, this could prevent approximately 25% of cases of schizophrenia.

b. It is more likely to cause psychosis in those who start using cannabis early.
c. There is a dose-response relationship to risk of developing psychosis.
d. Cannabis increases the relative risk of developing schizophrenia by up to 6 times.
e. It increases the release of dopamine.
Assuming a causal relationship, the elimination of cannabis use at the population level would reduce
the incidence of schizophrenia by about 8%. The Swedish Conscript Study by Andreasson et al (1987)
showed that the relative risk of developing schizophrenia is increased up to 6-fold in those with a
history of heavy use of cannabis. This strengthens the argument for cannabis being a causative factor.
Early onset of cannabis use confers the greatest risk. Elevation of dopamine levels is thought to be the
mechanism by which cannabis can produce or exacerbate psychotic symptoms.

Arseneault, L. et al, 2004. Causal association between cannabis and psychosis: examination of the
evidence. British Journal of Psychiatry. 184:110-117.

Shorter Oxford Textbook of Psychiatry, 5th edition, eds Gelder M, Harrison P, Cowen P, p286.

Incorrect
Marks for this submission: 0/1.
Question 4
Marks: 0/1
With regards to the differential diagnosis of catatonia, which ONE of the following statements is
TRUE?
Choose one answer.
a. A functional or organic cause is always identifiable.
b. Increasing age is a risk factor for catatonic depression.
c. Catatonia associated with mood disorders is less common than catatonia associated with
schizophrenia.
d. Idiopthic catatonia is more common in males.
e. Deep-vein thrombosis is a recognised cause of catatonia.

Increasing age is a risk factor for catatonic depression. Catatonia is more commonly associated with
mood disorders than with schizophrenia. Various causes of catatonia include infections, metabolic
abnormality, cocaine, ciprofloxacilin, withdrawal of clozapine etc.

Rajagopal, S. Catatonia. Advances in Psychiatric Treatment (2007) 13: 51-59

Incorrect
Marks for this submission: 0/1.
Question 5
Marks: 0/1
Liddle (1987) defined 3 overlapping clinical syndromes in schizophrenia, which he linked to patterns of
neuropsychological deficit and to regional cerebral blood blood flow. Liddle's clinical syndromes of
schizophrenia included which ONE of the following?
Choose one answer.
a. Autism
b. Avolition
c. Ambivalence
d. Loosening of associations
e. Disorganisation

Liddle examined the pattern of correlation between schizophrenic symptoms in a group of patients with
illnesses of similar chronicity and found that the symptoms segregated into three distinguishable
syndromes. Psychomotor poverty (poverty of speech, flat affect, decreased spontaneous movement),
disorganisation (disorder of form of thought and inappropriate affect), and reality distortion (delusions
and hallucinations).

Companion to Psychiatric Studies, 7th edition, eds Johnstone, Cunningham Owens, Lawrie, Sharpe,
Freeman, p396.

Incorrect
Marks for this submission: 0/1.
Question 6
Marks: 0/1
Kurt Schneider identified a group of experiences that might provide a reliable basis for the diagnosis of
schizophrenia, known as 'first rank symptoms'. Schneider's first rank symptoms of schizophrenia:
(choose ONE of the following)
Choose one answer.
a. Include persecutory delusions.
b. Include thought blocking.
c. Include emotional blunting.
d. Include thought insertion.
e. Predict outcome of schizophrenia at 5 years.

In the absence of cerebral pathology, the presence of any of Schneider's first-rank symptoms is
indicative of, though not pathognomonic of, schizophrenia. The first-rank symptoms do not have
prognostic significance. The first-rank symptoms are : auditory hallucinations (3rd person or thought
echo or running commentary), thought alienation (thought insertion, broadcasting or withdrawal),
somatic passivity, delusional perception, made feelings, impulses or actions.

Textbook of Psychiatry, 2nd edition eds Puri, Laking, Treasaden, p155.

Incorrect
Marks for this submission: 0/1.
Question 7
Marks: 0/1
The presence of high levels of which ONE of the following is most strongly linked to transition to
psychosis?
Choose one answer.
a. Anxiety
b. Functioning
c. Mania
d. Positive symptoms
e. Negative symptoms

The factors that predict the transition to psychosis of at risk individuals include positive symptoms, low
GAF scores (poor functioning), and depression. Mania, anxiety or negative symptoms were not
different between the group who made the transition to psychosis and the group which did not.

Parker, S & Lewis, S. Identification of yound people at risk of developing psychosis . Advances in
Psychiatric Treatment (2006) 12: 249-256
Incorrect
Marks for this submission: 0/1.
Question 8
Marks: 0/1
The outcome of a case of schizophrenia is remarkably heterogeneous and unpredictable. However,
there are some constitutional and illness factors that have some predictive value. Features associated
with a worse prognosis in schizophrenia include which ONE of the following?
Choose one answer.
a. A stressful precipitating event.
b. Enlarged lateral ventricles.
c. Acute onset.
d. Prominent affective symptoms.
e. Living in developing country.

Structural brain abnormalities have been linked with worse outcomes in schizophrenia. Prominent
affective symptoms are associated with a better prognosis. Interestingly, research shows that the
outcome is better in those living in developing countries. A clear precipitant is linked with a better
outcome. Acute onset is linked with a better prognosis.

Shorter Oxford Textbook of Psychiatry, 5th edition, eds Gelder M, Harrison P, Cowen P, p295 .

Incorrect
Marks for this submission: 0/1.
Question 9
Marks: 0/1
People are thought to be at risk of developing psychosis if: (choose ONE of the following)
Choose one answer.
a. Their ability to function has deterioriated over the previous 2 months.
b. They experience frank psychotic symptoms, which remit within a week.
c. They experience subclinical symptoms, which remit within a week.
d. They have a relative with a diagnosis of a psychotic disorder.
e. They meet the criteria for schizothypal personality disorder.

Some highly significant predictors of psychosis were found in a study by Yung et al 2003 fo predicting
who develop psychosis in a hgh risk group. The predictors include : long duration of prodromal
symptoms, poor functioning at intake, low-grade psychotic symptoms, depression and disorganization.

Yung et al., 2003. Psychosis prediction: 12-month follow up of a high-risk ("prodromal") group.
Schizophrenia Research, 60(1):21-32.

Incorrect
Marks for this submission: 0/1.
Question 10
Marks: 0/1
Which of the following features is NOT CONSISTENT with a diagnosis of persistent delusional
disorder?
Choose one answer.
a. Thought insertion.
b. Hearing impairment.
c. Transitory hallucinations.
d. Onset in middle age.
e. Presence of symptoms for at least 3 months.

The presence of a first-rank symptom (thought insertion) would suggest schizophrenia. According to
ICD-10, in order to make a diagnosis of persistent delusional disorder, the symptoms must have been
present for at least 3 months. Transitory (not persistent) hallucinations can occur in delusional
disorders. There is evidence that deafness (but not blindness) is associated with delusional disorder.
The mean age of onset for delusional disorders is 42 years.

Shorter Oxford Textbook of Psychiatry, 5th edition, eds Gelder M, Harrison P, Cowen P, p313,309.

Incorrect
Marks for this submission: 0/1.
Question 11
Marks: 0/1
Which ONE of the following statements about suicide in schizophrenia is NOT true?
Choose one answer.
a. It is associated with high premorbid functioning.
b. It is most likely to occur later in the illness.
c. It can be reduced by clozapine treatment.
d. It accounts for up to 40% of the excess mortality in schizophrenia.
e. There is a lifetime risk of 7%.

Suicide usually occurs earlier in the illness. Risk of death from suicide is increased 10-fold in
schizophrenia. The lifetime risk is approx 7%. High premorbid functioning is associated with increased
suicide risk in schizophrenia. An RCT by Meltzer et al (2003) showed clozapine treatment significantly
reduced suicide risk.

Shorter Oxford Textbook of Psychiatry, 5th edition, eds Gelder M, Harrison P, Cowen P, p294, 205,
410.

Incorrect
Marks for this submission: 0/1.

Epidemiology

1
Marks: 0/1
There are important gender differences in several aspects of schizophrenia and its treatment. Which
ONE of the following statements is INCORRRECT?
Choose one answer.
a. Schizophrenia in women is associated with later onset.
b. Schizophrenia in women shows a bimodal peak of incidence.
c. Schizophrenia in women has a slightly lower incidence than in men.
d. Schizophrenia in women is associated with more structural brain abnormalities than men.
e. Schizophrenia in women is associated with lower mortality compared with men.

Schizophrenia in men is associated with more structural brain abnormalities. The incidence of
schizophrenia is slightly higher in men than in women and male preponderance is higher in more
severe forms. There is a later mean age of onset (5 years later). There is a second peak around 50 years.
The mortality rate in men with schizophrenia is twice that of women.

Shorter Oxford Textbook of Psychiatry, 5th edition, eds Gelder M, Harrison P, Cowen P, p280, 305.

Incorrect
Marks for this submission: 0/1.
Question 2
Marks: 0/1
Many epidemiological studies have examined the patterns of prevalence of schizophrenia. Which ONE
of the following statements about the prevalence of schizophrenia is accurate?
Choose one answer.
a. The prevalence of schizophrenia is 2.3% during a lifetime, according to the Epidemiologic
Catchment Area Study.
b. The prevalence of schizophrenia is similar across different populations.
c. The prevalence of schizophrenia is low, relative to its incidence.
d. The prevalence is higher in Barbados than in the UK.
e. Prevalence is higher in developing than in developed countries.

Schizophrenia has a low incidence but relatively high prevalence. Prevalence rates are broadly similar
across populations; for example in Africa, India and Western countries. Incidence rates for
schizophrenia in the Caribbean are similar, hence the concern over higher rates in Afro-Caribbean
populations in England. Low prevalence of schizophrenia has been reported in the Hutterites and the
Anabaptist sect in the USA. The ECA is a popular subject for factual statements; the lifetime
prevalence of schizophrenia was found to be 1.3%.

New Oxford Textbook of Psychiatry, eds Gelder, Lopez-Ibor Jr, Andreason, 2000, p 588-589.

Shorter Oxford Textbook of Psychiatry, 5th edition, eds Gelder M, Harrison P, Cowen P, p280.

Robins, L.N. and Reiger, D.A. (1991). Psychiatric disorders in America. The Epidemiologic Catchment
Area Study. Free Press, New York.

Incorrect
Marks for this submission: 0/1.

Management
1
Marks: 0/1
Which ONE of the following has been shown to be effective in reducing the risk of developing
psychosis in at-risk mental states?
Choose one answer.
a. Monitoring alone.
b. Low-dose atypical antipsychotics.
c. Usual dose antipsychotics.
d. Antidepressant drug treatment.
e. Mood stabilisers.

Either low dose antipsychotics or a combination of low dose antipsychotics with cognitive therapy is
advoicated by some for management of at-risk individuals; ethical issues however need to be
considered since not all of the at-risk population go on to develop psychosis.

Parker S& Lewis S , Identification of yound people at risk of developing psychosis . Advances in
Psychiatric Treatment (2006) 12: 249-256.

Incorrect
Marks for this submission: 0/1.
Question 2
Marks: 0/1
Which ONE of the following is TRUE regarding the treatment of catatonia?
Choose one answer.
a. NMDA agonists have role in treatment.
b. Carbamazepine is contraindicated in the acute catatonic phase.
c. Clinically significant improvement typically begins to occur about 24 hours after starting
medications.
d. ECT is indicated in both functional and organic catatonia.
e. Antipsychotics are the first line treatment.

Benzodiazepines are the first line treatment, and ECT is used if BZD treatment fails. Other treatments
that can be used are Mood stabilisers (Carbamazepine), NMDA antagonists, Dopamine agonists.
Antipsychotics are at best avoided during the acute catatonic phase. Improvement, though dramatic in
few, takes up to 48 hours to be seen during treatment with benzodiapines.

Rajagopal, S. Catatonia. Advances in Psychiatric Treatment (2007) 13: 51-59.

Incorrect
Marks for this submission: 0/1.
Question 3
Marks: 0/1
Clozapine is the only antipsychotic licensed for use in treatment resistant schizopnrenia. Clozapine
treatment: (choose ONE of the following)
Choose one answer.
a. Would be expected to cause little postural hypotension, judging by its receptor biding profile.
b. Causes more akathisia at any clinical dose than risperisdone.
c. Causes hypersalivation by interaction with muscarinic M3 receptors.
d. May be used with carbamazepine for those experiencing clozapine-induced seizures.
e. Improves tardive dyskinesia.

Clozapine is the only antipsychotic that is associated with alleviation of TD. It induces less akathisia
than risperidone. It is thought that clozapine may produce hypersalivation though M4 agonism, alpha2
antagonism or inhibition of the swallowing reflex. Significant postural hypotension is a common side-
effect of clozapine hence the very slow dose titration on initiation. The combination of clozapine and
carbamazepine is dangerous due to the potential for blood dyscrasias.

Maudsley Prescribing Guidelines 2007, eds Taylor, Paton and Kerwin, 9th Edition, p61.

New Oxford Textbook of Psychiatry, eds Gelder, Lopez-Ibor Jr, Andreason, 2000, p 1322.

Incorrect
Marks for this submission: 0/1.
Question 4
Marks: 0/1
After one episode of schizophrenia, antipsychotics should be continued for: (choose ONE of the
following)
Choose one answer.
a. Less than 1 year.
b. None of the above.
c. 6 months.
d. Less than 6 months.
e. 1-2 years.

Reccommended for 1-2 years.

NICE guidelines for schizophrenia, 2006; 1.3.3.7

Incorrect
Marks for this submission: 0/1.
Question 5
Marks: 0/1
A proportion of schizophrenic patients do not respond adequately to conventional treatment and are
defined as 'treatment resistant'. Which ONE of the following statements regarding treatment-resistant
schizophrenia is TRUE?
Choose one answer.
a. It is a situation where antipsychotic polypharmacy may be recommended.
b. It is more likely in those with predominantly positive psychotic symptoms.
c. It is defined as lack of clinical improvement despite the sequential use of the recommended
doses for 6 to 8 weeks of at least 3 antipsychotics.
d. It responds to clozapine in approximately 60% of patients.
e. It occurs in approximately 1/4 of patients.
Although antipsychotic polypharmacy should generally be avoided, TRS is one situation where it can
be used with some benefit. The NICE Guidelines state that treatment-resistance occurs after adequate
trial with only 2 antipsychotics, one of which should have been an atypical. Approximately 30% of
patients do not respond to antipsychotics. It is more likely in those with more negative symptoms.
Studies have shown that approximately 30% of patients with TRS will respond to clozapine.

Seminars in General Adult Psychiatry, 2nd edition, eds Stein G and Wilkinson G, p279-280.

Incorrect
Marks for this submission: 0/1.
Question 6
Marks: 0/1
Regarding psychosocial interventions in schizophrenia, which ONE of the following statements is
TRUE?
Choose one answer.
a. Cognitive remediation is a treatment for positive psychotic symptoms.
b. CBT is effective in the treatment of delusions.
c. Presence of high expressed emotion within families predict the outcome of most psychiatric
disorders.
d. Compliance therapy has not yet been shown to improve the outcome of patients prescribed
neuroleptic medication.
e. Intensive case management has been shown to be more effective than other therapies in
preventing relapse in psychoses.

The efficacy of CBT in schizophrenia is well established. The benefits of compliance therapy were
reported by Kemp in 1996. The UK700 trial did not find an advantage for intensive case management
except in some specific subgroups of patients. Cognitive remediation is a treatment for cognitive
impairment in schizophrenia. Expressed emotion has primarily been implicated in the relapse of
schizophrenia.

Kemp, R. et al (1996) Compliance therapy in psychotic patients: randomised controlled trial. BMJ, 10,
312(7027), 345-9.

Burns, T. (2002) The UK700 trial of Intensive Case Management: an overview and discussion. World
Psychiatry. 1(3), 175-178.

Zimmerman, G. et al (2005) The effect of cognitive behavioral treatment on the positive symptoms of
schizophrenia spectrum disorders: a meta-analysis. Schizophr Res. 1, 77(1), 1-9.

Incorrect
Marks for this submission: 0/1.
Question 7
Marks: 0/1
Neuropleptic malignant syndrome is a rare but fatal complication of antipsychotic treatment. Which
ONE of the following statements about NMS is NOT true?
Choose one answer.
a. It is more likely to occur in those where physical restraint has been used.
b. The risk of recurrence is 30 times greater following one episode.
c. It is dose-related.
d. NMS leads to a leukocytosis.
e. NMS can be treated with bromocriptine.

Neuroleptic Malignant Syndrome can occur at any dose. Physical restraint is a risk factor for NMS.
Leukocytosis is commonly found on routine blood investigations. The finding that risk of recurrence is
30 times greater following one episode of NMS was demonstrated by Gurrera (2002). Treatment can
include administration of a dopamine agonist.

Seminars in General Adult Psychiatry, 2nd edition, eds Stein G and Wilkinson G, p265-266.

Incorrect
Marks for this submission: 0/1.
Question 8
Marks: 0/1
The following are options for augmenting clozapine in a patient who has shown partial response to
clozapine treatment, EXCEPT:
Choose one answer.
a. Sulpiride
b. Pimozide
c. Risperidone
d. Lamotrigine
e. Omega 3 triglycerides

Pimozide is not recommended due to cardiotoxicity.

Maudsley Prescribing Guidelines 2007, eds Taylor, Paton and Kerwin, 9th Edition, p64.

Incorrect
Marks for this submission: 0/1.
Question 9
Marks: 0/1
A female schizophrenia patient has been put on Olanzapine and Quetiapine, but with poor effect. Your
next step is to try: (choose ONE of the following)
Choose one answer.
a. Haloperidol
b. Aripiprazole
c. Lithium
d. Amisulpiride
e. Clozapine

After a trial of two antipsychotics, of which at least one must be a second generation antipsychotic, it is
reasonable to switch to clozapine, if possible. This guidance from NICE is also presented as an
algorithim in the Maudsley Guidelines.

The Maudsley prescribing guidelines, 9th Edition, 2007, Informa Healthcare, p29.

Incorrect
Marks for this submission: 0/1.
Question 10
Marks: 0/1
Which ONE of the following statements about Clozapine is INCORRECT?
Choose one answer.
a. Reduces suicidal thoughts.
b. Is used for first-line treatment of Schizophrenia.
c. Is effcetive in refractory cases.
d. Reduces aggression.
e. Is very effective in actively and floridly psychotic patients.

Clozapine is recommended after adequate trials of two different antipsychotics.

Maudsley Prescribing Guidelines 2007, eds Taylor, Paton and Kerwin, 9th Edition, p61.

Incorrect
Marks for this submission: 0/1.
Question 11
Marks: 0/1
Atypical antipsychotic medications are used in the first-line treatment of schizophrenia. Olanzapine is
one example of an atypical antipsychotic drug. Which ONE of the following statements about
olanzapine is TRUE?
Choose one answer.
a. It exerts its sedative effects through its 5HT2A receptor activity.
b. It only causes diabetes mellitus at doses above 20mg.
c. It has superior efficacy compared with haloperidol.
d. It is less likely to cause hyperprolactinaemia than risperidone.
e. Its association with more weight gain and metabolic effects when compared with other
atypicals is not supported by research.

The atypicals risperidone, amisulpiride and zotepine are more potent D2 antagonists and therefore have
the highest risk of inducing hyperprolactinaemia. In the CATIE study, olanzapine was associated with
greater weight gain and increases in measures of glucose and lipid metabolism. The available data do
not indicate a clear difference in efficacy between typicals and atypicals, apart from clozapine.
Olanzapine does not require high doses to be associated with diabetes mellitus. The sedative effects of
olanzapine are because of its potent antihistaminergic effects.

Liebermann, J.A. et al (2005) Effectiveness of antipsychotic drugs in patients with chronic


schizophrenia. NEJM, 353(12), 1209-23.

Haupt, D.W. (2006) Differential metabolic effects of antipsychotic treatments. European


Neuropsychopharmacology, 16, Suppl 3:S149-55

Incorrect
Marks for this submission: 0/1.
Question 12
Marks: 0/1
There is growing interest in the idea that early treatment of schizophrenia may improve the long-term
outcome. Which ONE of the following statements about early intervention in psychosis is TRUE?
Choose one answer.
a. The mean DUP (duration of untreated psychosis) is 1-2 years.
b. Prolonged DUP correlates with neurobiological markers of illness severity.
c. Advocates use of high-dose antipsychotics early in the illness.
d. Out of those found 'at risk' of developing psychosis in the PACE study, 80% became psychotic
within one year.
e. The 'critical period' refers to the period of brain development during childhood when
environmental damage can lead to psychotic symptoms in adulthood.

The mean duration of untreated psychosis is 1-2 years, and is true even in countries with highly
developed systems of healthcare. The 'critical period' refers to the 3-5 year period following illness
onset when there may be a decline in social and cognitive function. Only 40% developed psychosis in
the first year. Prolonged duration of untreated psychosis does not correlate with neurobiological
markers of illness severity, which may indicate that prolonged DUP may not be a direct causal link
with poor outcome. Low-dose atypicals are used.

Seminars in General Adult Psychiatry, 2nd edition, eds Stein G and Wilkinson G, p301.

Incorrect
Marks for this submission: 0/1.
Question 13
Marks: 0/1
CBT for delusions is contraindicated if the patient: (choose ONE of the following)
Choose one answer.
a. Has persistent symptoms.
b. Has poor insight.
c. Has cognitive deficits.
d. Is taking antipsychotic medication.
e. Is not distressed by their paranoia.

CBT is recommended for people with distressing symptoms as it enables individuals to engage with the
colloborative goal of reducing distress; hence it is not recommended for people who are not distressed
by their symptoms. CBT has been shown to be effective in treatment of persistent symptoms,
particularly delusions. Poor insight, cognitive deficits, taking medications are not contraindications.

Freeman D, Garety P. Helping patients with paranoid & suspicious thoughts: a cognitive behavioural
approach. Advances in Psychiatric treatment (2006), 404 -415.

Incorrect
Marks for this submission: 0/1.
Question 14
Marks: 0/1
Tardive dyskinesia refers to involuntary, choreoathetoid movements of (typically) the orofacial
structures. It is related to chronic antipsychotic treatment and can be permanent. Which ONE of the
following findings related to tardive dyskinesia is NOT true?
Choose one answer.
a. May increase on cessation of antipsychotic treatment.
b. Is more likely to occur in older patients.
c. Is more likely to occur in those with a family history of Parkinson's disease.
d. Is associated with Gly/Gly allele of the DRD3 gene.
e. May occur through activation of D3 receptors.

It is thought that TD occurs due to dopamine D2 receptor hypersensitivity (and not D3). Stopping
antipsychotics sometimes worsens the movement disorder. A family history of Parkinson's disease is a
known risk factor for developing TD. Increasing age is a risk factor for TD. Studies have shown
increased allelic frequency of the serine-glycine substitution in the DRD3 gene that codes for the D3
receptor.

Seminars in General Adult Psychiatry, 2nd edition, eds Stein G and Wilkinson G, p270.

Casey, D. E., 2004. Pathophysiology of antipsychotic drug-induced movement disorders. J Clin


Psychiatry. 65 Suppl 9, 25-8.

Incorrect
Marks for this submission: 0/1.
Question 15
Marks: 0/1
The antipsychotic which is best supported for use in patients with Tardive Dysknesia is:
Choose one answer.
a. Olanzapine
b. Quetiapine
c. Aripiprazole
d. Clozapine
e. Risperidone

Clozapine has the best evidence for use in Tardive dyskinesia; however there is also evidence emerging
for Quetiapine and Olanzapine.

Maudsley Prescribing Guidelines 2007, eds Taylor, Paton and Kerwin, 9th Edition, p99.

Incorrect
Marks for this submission: 0/1.
Question 16
Marks: 0/1
Which of the following causes symptoms of inappropriate ADH secretion?
Choose one answer.
a. Olanzapine
b. Haloperidol
c. Quetiapine
d. All of these
e. Risperidone
Hyponatremia can occur due to water intoxication, hyperlipidemia or due to drug induced SIADH. The
synptoms of SIADH include confusion and lethargy and if this progresses can lead to seizures and then
coma. All the antidepressants have been implicated as have aripiprazloe, olanzapine,risperidone,
phenothiazines, quetiapine and clozapine. Bromocriptine, Narcotics and opiate derivatives have also
been implicated. Regular sodium level monitoring is desirbale for patients on these medications and is
essential for those who show signs of lethargy and confusion. if the sodium levels are less than 125
mmol/L, stop the offending agent. Although a deterioation in the clinical presentation might make
evaluation difficult.

The Maudsley prescribing guidelines, 9th Edition, 2007, Informa Healthcare,p 138.

Incorrect
Marks for this submission: 0/1.
Question 17
Marks: 0/1
A male schizophrenia patient on clozapine has gained 20kg. Your next step is to: (choose ONE of the
following)
Choose one answer.
a. Switch to Risperidone.
b. Switch to Olanzapine.
c. Switch to Quetiapine.
d. Add sodium valproate.
e. Switch to aripiprazole.

Aripriprazole seems to have least effect on 5HT receptors, and hence limits weight gain. Clozapine and
Olanzapine have the higest risk, followed by Zotepine, Chlorpromazine, Quetiapine and Risperidone
(moderate risk) and Amisulpiride, Aripiprazole, Haloperidol, Trifluoperazine and Ziprasidone (low
risk).

The Maudsley prescribing guidelines, 9th Edition, 2007, Informa Healthcare, p110.

Incorrect
Marks for this submission: 0/1.

EMIs
1
Marks: 0/3
DIAGNOSIS – PSYCHOTIC DISORDERS

A. Catatonic schizophrenia
B. Delusional disorder
C. Depression with psychotic features
D. Hebephrenic schizophrenia
E. Paranoid schizophrenia
F. Post psychotic depression
G. Residual schizophrenia
H. Schizoaffective disorder – depressive
I. Schizotypal disorder
J. Simple schizophrenia

For each of the following patients, select the most likely diagnosis.
A 22-year-old university student presents with persecutory ideas, sadness and inability to concentrate.
His academic performance had deteriorated recently which he put to a plot hatched by the aliens and
his tutors at the university to stem the growth of the genius in him. He also mentioned hearing people
laughing at him when alone.
After six months of treatment with antipsychotic medication, he recovered well and started to go back
to the university, although not totally convinced about his tutors. He developed a low mood, difficulty
in concentration, increasing tiredness and inability to sleep. He lost weight and had recurrent worries
about death.
A 27-year-old man believes that his suffering is the greatest in the world. He has been feeling low for
6 weeks and lost a lot of weight from not eating, as he believes that all his intestines were rotten and
he couldn‘t digest any food even if he tried to eat. He has lost all interest and motivation to do
anything.
1. E – This is paranoid schizophrenia, characterised by a prodrome and presence of persecutory
delusions and auditory hallucinations.
2. F – This is post-psychotic depression. Note that this can only be diagnosed once the psychotic
symptoms are absent or present minimally.
3. C – The coexistence of depressive symptoms and mood congruent delusions suggests psychotic
depression
Incorrect
Marks for this submission: 0/3.
Question 2
Marks: 0/3
MEDICATION FOR PSYCHOTIC DISORDERS

A. Chlorpromazine
B. Clonazepam
C. Clozapine
D. Diazepam
E. Flupentixol depot
F. Flupentixol oral
G. Haloperidol
H. Olanzapine
I. Quetiapine
J. Risperidone

Choose a treatment for each of the following clinical presentations

An older schizophrenic woman has developed diabetes mellitus. She is agreeing to take medication
but has irregular compliance with tablets and is currently psychotic. When her son reminds her to take
her tablets they usually end up in a quarrel.
An elderly man with persistent psychotic symptoms, he is not tolerating oral flupentixol. He has
previously failed to respond to sulpiride and olanzapine
A woman with Parkinson‘s disease, troubled by hallucinations. She is not currently on an anti-
parkinsonian drug and is antipsychotic naïve.
1. E. In this scenario a depot antipsychotic would be most appropriate.
2. C. This describes treatment resistance. In this case, clozapine would be the medication of choice
3. I. According to the Maudsley Prescribing guidelines, quetiapine at lower doses can be used to treat
psychotic symptoms in Parkinson‘s disease.
Incorrect
Marks for this submission: 0/3.
Question 3
Marks: 0/3
RISK FACTORS

A. Female sex
B. Living alone
C. Living in Spain
D. Past psychiatric history
E. Recent discharge from hospital
F. Visual impairment
G. High intellect
H. Unemployment.
I. Divorce.
J. Moving House.

Which THREE of the above are risk factors for:

Suicide

Depression

Late onset schizophrenia

1. B, D, E - Living alone, past psychiatric history and recent discharge from psychiatric hospital are
risk factors for suicide.
2. A, B, D - Female sex, living alone (in the context of poor social support) and past history are risk
factors for depression.
3. A, B, F - There is a preponderance of females with late-onset schizophrenia (M:F = 7:1),
premorbidly patients tend to have poor adjustment, social isolation and lower marriage and child-
bearing rates and sensory impairment is also over-represented.
Incorrect
Marks for this submission: 0/3.
Question 4
Marks: 0/3

A. Alcohol withdrawal

B. Alcoholic hallucinosis

C. Complex partial seizures

D. Delirium tremens

E. Fahr‘s syndrome

F. Metachromatic leucodystorphy

G. Neuroacanthosis

H. Post Herpatic encephalopathy


I. Systemic Lupus Erythematosis

A 44 year old homeless man is brought into A&E. He is agitated, psychotic and depressed. The ECG
shows decreased alpha activity and his gait is disturbed. MRI shows hypointensity of the striatum.
A 25 year old man recently returned from America shows aggressive behaviour. He recently has
periods of decreased consciousness associated with making funny movements with his hands and
sounds from his lips. His full blood count and liver functions are normal
A 30 year old woman has one sided facial nerve palsy and circumscribed lesions on both legs.

1. D. DT can present as vivid hallucinations, delusions, profound confusion and inattention, agitation
and restlessness, sleeplessness, autonomic overactivity and fearful affect. Gait can be disturbed, EEG
might show reduced alpha activity and MRI could reflect changes in striatum.

2. C. Complex partial seizures cause impaired consciousness and arise from a single brain region.
Impaired consciousness implies decreased responsiveness and awareness of self and surroundings.
During a complex partial seizure, the patient may not communicate, respond to commands, or
remember events that occurred. Epilepsy of prolonged duration and brain dysfunction involving a
broad area including the temporal and frontal lobes may be associated with the occurrence of subacute
postictal aggression.

3. H. Shingles is an acute infection caused by reactivation of latent varicella-zoster virus. After


primary chickenpox infection, the virus lies dormant in the dorsal root ganglia of the spinal cord. When
reactivated, it travels along the sensory nerve to affect one or more dermatomes, causing the
characteristic shingles rash. It can lead to complications like Ramsay Hunt syndrome characterised by
unilateral facial palsy associated with pain and herpetic blisters in the external auditory meatus due to
involvement of the geniculate ganglion.

Incorrect
Marks for this submission: 0/3.
Question 5
Marks: 0/3
INVESTIGATIONS

A. Beck‘s depression inventory


B. Blood gas analysis
C. Blood screen for analgesics
D. Full blood count
E. Information from informant
F. Liver function tests
G. Mini mental state examination
H. Narcotic drug screen
I. Serum antimanic drug levels
J. Thyroid function tests

For each of the following scenarios, identify the most indicated investigations.

1. A 30 year old woman has had two episodes of depression and one episode of mania in the last eight
months. She has been compliant with her mood stabiliser medication, which she has been on for the last
ten years. She is currently depressed and has taken an overdose of her tablets two days ago.
2. A 73 year-old lady was brought by her son. She had a six-month history of being withdrawn, reduced
appetite and self-care. She had occasionally, mentioned about suicidal ideas to her son and all her
symptoms had worsened over six weeks, after the death of her husband. She has been on
antihypertensive medication for over 10 years.

3. A 47-year-old man with a long history of alcohol and opioid dependence was admitted after he had
attempted to hang himself. On assessment he was found to be more depressed subjectively rather than
objectively.
1. I , J– A lithium level would be useful to check for potential toxicity, possibly related to overdose and
also to check whether the levels are within therapeutic range, as low levels may indicate non-
compliance and possible relapse. The rapid cycling nature of her disorder warrants a thyroid function
test.
2. E, G, J – Informant history is vital to assess this lady. MMSE would screen for a dementia and TFTs
would rule out underlying hypothyroidism.
3. B, E, H – Blood gases are important to check for hypoxia and acid-base disturbance. An informant
history would also be useful as would a narcotic drug screen, given the history of opioid dependence.
Incorrect
Marks for this submission: 0/3.
Question 6
Marks: 0/3

DIAGNOSIS

A. Capgras Syndrome

B. Charles Bonnet Syndrome

C. Cotard‘s Syndrome

D. Couvade Syndrome

E. De Clerambault‘s Syndrome

F. Di George Syndrome

G. Ekbom‘s syndrome

H. Fregoli‘s Syndrome

I. Gansers syndrome

J. Othello‘s Syndrome

For each of the following select the most appropriate diagnosis.

An elderly woman has insomnia, low mood and weight loss. She tells you there is no point in her
eating anything as her insides are rotting.
A young man whose wife is pregnant complains of morning sickness and an inflated abdomen.
A mother of a 20 year-old boy says that lately her son has been saying that her male friend has been
replaced by an imposter. She keeps telling him this is not the case but he does not believe her. She has
had to ask her son to leave her home because of this.

1. C - Cotards syndrome is used to describe nihilistic delusions occurring within a depressive episode.
These delusions characteristically involve ideas of the bowels rotting away, although other ideas such
as ‗being dead‘, part of the body being ‗rotten‘ can also be seen. It is often, although not exclusively,
seen in older people with severe depression. It can also occur in schizophrenia, bipolar disorder &
organic conditions involving lesions in the non-dominant temporoparietal cortex.

2. D - Couvade Syndrome is the term used to describe a man who is experiencing symptoms of his
partner‘s pregnancy. Various theories have been suggested to explain this although none are
conclusive. They include anthropological theories such as ‗minimization of gender differences in
preparation for child rearing‘, hormonal theories such as ‘reduction in testosterone‘ and psychodynamic
theories such as ‗an expression of somatised anxiety‘.

3. A - Capgras syndrome comprises of delusions where the person believes that a familiar person
usually family members have been replaced by imposters with physical appearance identical to the
original person. The person can consciously recognise the face but no longer experiences the emotional
response associated with evaluation of the face. It is common in schizophrenia and frontal lobe lesions.

Incorrect
Marks for this submission: 0/3.
Question 7
Marks: 0/3

Theme - Investigations

A. 24 hour urinary free cortisol.


B. Dexamethasone suppression test
C. Elevated TSH.
D. Elevated serum cholesterol.
E. Hypernatraemia
F. Hypokalaemia
G. Hyponatraemia
H. Low TSH
I. Ultrasound.
J. No rise in plasma cortisol on short synacthen test

Choose TWO of these would you expect to find for each of these clinical pictures:

1. A 21-year-old woman with low mood, tiredness, intolerance of cold and menorrhagia.
2. A 30-year-old woman pigmentation of her skin, low mood, general weakness, weight loss and
syncope.
3. A 30 year old woman with increased body weight, thin arms and legs, excess hair growth and
emotional lability.
1. C, D. The scenario describes hypothyroidism so you would therefore expect a raised TSH
and raised cholesterol. (TSH levels are normal in pituitary failure and raised in primary
thyroid failure).
2. G, J. The scenario describes Addison‘s disease. Hyponataemia and lack of cortisol response
on the synacthen test are characteristic findings.
3. A, B. The scenario describes Cushing‘s syndrome, which is due to increased exposure to
steroid hormones. 24 hour urinary free cotisol levels and DST are tests for Cushing‘s
disease.
Incorrect
Marks for this submission: 0/3.
Question 8
Marks: 0/3

DIAGNOSIS

A. Addisons Disease

B. Cardiomyopathy

C. Cushings Syndrome

D. Delirium Tremens

E. Diabetes Insipidis

F. Diabetes Mellitus

G. Diabetic keto Acidosis

H. Lithium toxicity

I. Neuroleptic malignant syndrome

J. Serotonin Syndrome

Choose the most appropriate diagnosis for the following:

A young man on Clozapine complains of breathlessness and has a persistent tachycardia.


A woman on lithium for many years who recently complains of feeling weak. Her serum sodium is
raised and she looks dehydrated.
A young man started on an antipsychotic 2 weeks ago now complains of stiff muscles, fever , labile
BP, and confusion.

1. B. Risk of cardiomyopathy and cardiomyositis peaks in the first two months of Clozapine initiation.
Obvious signs are hyperventilation, edema and tachycardia. Needs medical intervention and
consideration of drugs that are less interfering with cardiac functions.

2. E. Lithium induced diabetes insipidis should be considered in patients with unexplained hypernatremia
and dehydration. This is reversible on stopping Lithium.
3. I. NMS occurs due to excessive blockade of dopamine receptors leading to acute deficiency of dopamine
in the brain in response of prescription of a bigger starting dose in a drug naive patient or rapid
escalation of an existing dose. Commonly seen with typicals and considered to manifest in patients who
are sensitive to medication effects. It is a medical emergency and patient should be transferred to a
medical ward for treatment. CK levels can be checked. They are usually between 1000-2000. If
untreated can result in renal failure, respiratory depression, coma and death.
Incorrect
Marks for this submission: 0/3.
Question 9
Marks: 0/3
INVESTIGATIONS

A. Clozapine levels
B. Creatinine kinase
C. CT scan of the brain
D. ECG
E. EEG
F. Full blood count
G. Lithium levels
H. Lumbar puncture
I. Renal function tests
J. Thyroid function tests

For each of the following patients, select the most appropriate investigation.

1. A man who has recently been started on depot antipsychotic medication. He has developed muscular
rigidity; tachycardia, pyrexia and unstable blood pressure

2. A woman on clozapine treatment who has developed pyrexia of unknown origin

3. A woman on lithium treatment who has coarse tremor, drowsiness and diarrhoea
Incorrect
Marks for this submission: 0/3.
Question 10
Marks: 0/3

SIDE EFFECTS – METABOLIC ABNORMALITY:


A. Hypercalcemia
B. Hyperglycaemia
C. Hyperkalemia
D. Hypernatremia
E. Hyperthyroidism.
F. Hypocalcemia.
G. Hypoglycaemia
H. Hypokalemia
I. Hyponatremia
J. Hypothyroidism
Choose ONE metabolic abnormality associated with each of the following:
A 78 year old gentleman, diagnosed with depression was started on Sertraline. A few days later he
was found to have muscle weakness and drowsiness.
A 45 year old man with history of alcohol dependence went on a drinking spree on a Friday night. He
was found to be anxious, sweating profusely and confused by the next morning.
A 40 year old man, with a diagnosis of resistant schizophrenia was started on clozapine. Three weeks
later he was found to be confused. His carers mentioned that he was complaining of tiredness,
increased thirst and increased urinary frequency.

1. H. Hyponatremia, a side effect with SSRIs and Carbamazepine leads to confusion, drowsiness, muscle
weakness, and seizures
2. G. Hypoglycemia, due to prolonged drinking and no eating adequately are frequently encountered in
patients with alcohol binges.
3. B. Clozapine induced hyperglycaemia; part of the metabolic syndrome is being tested.

Incorrect
Marks for this submission: 0/3.
Question 11
Marks: 0/3
AETIOLOGY

A. Alcohol
B. Childhood sexual abuse
C. Diabetes
D. Having given birth 6 months previously
E. Impaired hearing
F. Loss of mother before age 14
G. Peri-natal hypoxia
H. Reduced visual acuity
I. Regular use of NSAIDs
J. Smoking

Which of the above risk factors is most strongly associated with the following?
A young woman with mood-congruent delusions, auditory hallucinations and depressed mood.
An older woman with first onset of both somatic and auditory hallucinations and delusions.
A cognitively intact older man with visual hallucinations.
1. D. Recent birth is the most likely risk factor in this lady with severe depression.
2. E. Hearing impairment is a risk factor for psychosis in later life
3. H This refers to Charles de Bonnet syndrome.
Incorrect
Marks for this submission: 0/3.
Question 12
Marks: 0/3
DIAGNOSIS OF MOOD DISORDERS

A. Acute stress reaction.


B. Adjustment disorder.
C. Bipolar affective disorder, current episode severe depression without psychotic symptoms.
D. Bipolar affective disorder, current episode severe depression with psychotic symptoms.
E. Mild depressive disorder.
F. Mixed anxiety and depression
G. Moderate depressive disorder.
H. Organic depressive disorder.
I. Severe depressive disorder with psychotic symptoms.

Choose the ONE most appropriate diagnosis for each of the following
A 25 year old lady presents 3 months after losing her job, with low mood, lack of energy and
enjoyment, loss of appetite, disturbance in sleep and poor concentration with some suicidal ideas
A 75 year lady is brought to A&E by her daughter one month after she had been treated for UTI by
her GP. She has some somatic complaints, psychomotor retardation and is hearing two voices, one of
which is of her dead husband asking her to join him.
A 45 year old man acting has been acting bizarrely at home; he has inability to sleep and loss of
weight. He thinks the neighbours are poisoning his food and is found one night searching the house
for intruders. 8 weeks ago he bought a new car out of the blue, which was out of character for him.
1. G - This scenario best fits the ICD-10 criteria for a moderate depressive episode as three of the core
symptoms are present (depressed mood, anhedonia, loss of energy) and four additional symptoms.

2. I - This scenario best fits a depressive psychosis as auditory hallucinations are present. Although she
has had a UTI, this is unlikely to be the cause for her depression.

3. D- The history of excessive spending alludes to a previous hypomanic episode followed by a severe
depressive episode with psychotic features
Incorrect
Marks for this submission: 0/3.
Question 13
Marks: 0/3
AETIOLOGY

A. Conductive deafness
B. Divorced
C. Female sex
D. High intellect
E. Past psychiatric history
F. Poor premorbid adjustment
G. Recent discharge from hospital
H. Unemployment

For each of the following, select three commonly associated risk factors.

1. Depression

2. Late Paraphrenia

3. Suicide

1. B, C, E – Being divorced, female and with a past psychiatric history are risk factors for depression.
2. A, C, F – Sensory impairment, female gender and poor premorbid adjustment/personality are risk
factors for late paraphrenia.
3. B, G, H – Being divorced, female and unemployed are risk factors for suicide.
Incorrect
Marks for this submission: 0/3.
Question 14
Marks: 0/3
A. AIDS dementia complex

B. Amyotropic Lateral Sclerosis

C. Guillian-Barre syndrome

D. Huntington‘s disease

E. Neurocysticercosis

F. Neurosarcoidosis

G. Picks Disease

H. Pseudobulbar palsy

I. Systemic lupus eythematosus

J. Turner syndrome

Choose ONE option for the following:

A 40-year-old man presented with impulsivity and recent change of personality. His wife has
mentioned that his father and grandfather also developed the same condition.
A 50-year-old man develops nodules on the shin and also respiratory problems.
A man known to have Multiple Sclerosis develops difficulties with his speech. His wife describes his
mood as ‗all over the place‘.
Answers:

1. D. Huntington's disease is a genetic disorder (inherited due to a faulty gene 4) which usually affects
people in their 40s and 50s. It primarily affects the brain, with a gradual loss of control of movement,
memory and mental ability. HD is also associated with personality changes and depression as well as
other mental illnesses. Strong genetic component – autosomal dominance with full penetrance.

2. F. Neurosarcoidosis is a serious and devastating manifestation of sarcoidosis in the nervous system.


Sarcoidosis is a chronic inflammatory disorder that typically occurs in adults between 20 and 40 years
of age and primarily affects the lungs, but can also impact almost every other organ and system in the
body. Histologically, neurosarcoidosis is characterized by formation of granulomas in the central
nervous system. The lesion consists of lymphocytes and mononuclear phagocytes surrounding a
noncaseating epithelioid cell granuloma. These granulomas represent an autoimmune response to
central nervous system tissues.

3.H Pseudobulbar palsy results from disease to both corticobulbar tracts. It can be caused by bilateral
internal capsule infarcts, myelination disorders, motor neuron disease, high brain stem lesions,
syringobulbia, guillain barre syndrome, head injury or polio. The patient might have tongue paralysis
leading to difficulties with speech (Donald Duck speech), dysphagia and dysphonia. Emotional lability
may occur. Reflexes (jaw jerk) might be exaggerated as well.
Incorrect
Marks for this submission: 0/3.
Question 15
Marks: 0/3
INVESTIGATIONS

A. Acanthocytosis
B. Cortical atrophy & hypodensities in basal ganglia
C. Hyperglycaemia
D. Hypokalaemia
E. Hyponatremia
F. Increased urinary catecholamines
G. Low plasma caeruloplasmin
H. Low serum B12 levels
I. Raised Mean Corpuscular Volume
J. Short Synacthen test

For each of the following patients, select TWO findings that would be expected.

1. A 45-year-old man presents with a two-month history of anxiety, vomiting, panic attacks and
headaches. His BP is 170/110 mm hg, and he appears to be sweating profusely.

2. A 20-year-old lady with a history of hepatomegaly develops tremors, rigidity and dysarthria. She is
also noticed to have cognitive deficits.

3. A 24-year-old lady presents with irritable mood, lethargy, loss of appetite and fatigue. She has some
hyperpigmented areas on her skin.
1. D, F – These would support a diagnosis of phaeochromocytoma, as suggested by the clinical
features.
2. B, G – These investigations would suggest Wilson‘s disease.
3. E, J – These investigations would suggest Addison‘s disease.
Incorrect
Marks for this submission: 0/3.
Question 16
Marks: 0/3
DIAGNOSIS OF PSYCHOTIC DISORDERS

A. Behavioural and personality changes


B. Blunting of affect
C. Catatonic symptoms
D. Emotional disturbance
E. Later life onset
F. Pre-morbid personality shy and withdrawn
G. Prominent delusions and hallucinations
H. Symptoms last for at least 3 months
I. Symptoms last for only 2 weeks
J. Young adult or adolescent

For each of the following choose the most characteristic symptoms.

1. Paranoid schizophrenia
2. Hebephrenic schizophrenia
3. Persistent delusional disorder
1. G – Prominent delusions and hallucinations are the core feature of paranoid schizophrenia.
2. D, F, J – Hebephrenic schizophrenia is characterised by emotional disturbance, abnormal premorbid
personality and younger age of onset.
3. E, H – Persistent delusional disorder is characterised by later age of onset and symptoms must be
present for more than 3 months for a diagnosis to be made.
Incorrect
Marks for this submission: 0/3.
Question 17
Marks: 0/3
A. Chlorpromazine
B. Clozapine
C. Flupentixol depot
D. Haloperidol
E. Levomopromazine
F. Lorazepam
G. Olanzapine
H. Quetiapine
I. Risperidone
J. Stop antipsychotics

Choose ONE most suitable medication for the following patients:

An 80 year old woman with Diabetes Mellitus and psychosis agrees to take medication but has
repeated relapses of her psychosis due to poor concordance. Whenever her son reminded her to take
her medication, it would end up in a quarrel.
A 66 year old psychotic elderly man has persistent psychotic symptoms, despite treatment with
adequate doses of sulpiride and olanzapine. He is currently on Flupenthixol, which he is not tolerating
that well.
A 74 year old man has been newly diagnosed to have Parkinson‘s disease, complains of severe,
distressing visual hallucinations. He is not on any drug treatment for his Parkinson‘s disease and has
never been tried on a neuroleptic medication.
A 68 year old man who presented with delusional disorder for the first time was treated with
Olanzapine leading to resolution of his delusions. He has however developed severe weight gain.

1. C - As atypical antipsychotics act on (block) 5-HT2 receptors causing more weight gain than typical
antipsychotics, the choice would be to go for a typical antipsychotic. As compliance seems to be
an issue, Flupenthixol depot would be the best option.

2. B - Failing the trial of two antipsychotics implies treatment resistance, and Clozapine would be the
drug of choice according to NICE guidelines, including in the elderly.

3. H - Quetiapine according to studies has proven to be the drug of choice in treating Parkinson‘s
psychosis. Quetiapine moderately reduces visual hallucinations and mildly reduces paranoia and
delusions in patients with Parkinson's disease with psychosis.

4. H - Weight gain in the first 10 weeks of prescribing an atypical antipsychotic is about 4.5 kgs with
Clozapine, 4 kgs with Olanzapine, 2 kgs with Risperidone and minimal with Quetiapine. Hence
the latter would be the choice of drug in an elderly patient.

Incorrect
Marks for this submission: 0/3.
Question 18
Marks: 0/3

INVESTIGATION
A. Brief Psychiatric Rating Scale

B. Edinburgh Postnatal Depression Scale

C. Geriatric Depression Scale

D. GHQ 12

E. Hamilton Depression Scale

F. Montgomery Asberg Depression Rating Scale

G. Morgan Russel scale

H. Scale for Negative symptoms in schizophrenia

I. Simpson Angus Scale

J. Zung scale

Select the rating scales that are useful for the scenarios described below:

1. To assess symptoms of depression in a 37 year old lady presenting for the first time to psychiatric
services.

2. A 22 year old lady, who has a history of obsessive-compulsive disorder, currently appears to be
depressed following the recent delivery of her boy. The screening tool, which can be used to assess her
depression.

3. To assess the extrapyramidal side effects in a lady who has been on haloperidol 20 mg per day for
her psychotic symptoms.

<object classid="clsid:38481807-CA0E-42D2-BF39-B33AF135CC4D" id="ieooui"></object >

1. E, F.The Hamilton Rating Scale for Depression (HAM-D, HRSD) is the most widely utilized rating
scale to assess symptoms of depression. The HAM-D is an observer-rated scale consisting of 17 to 21
items (including two 2-part items, weight and diurnal variation). Ratings are made on the basis of the
clinical interview, plus any additional available information, such as nursing or family member report.
The items are rated on either a 0 to 4 spectrum (0 = none/absent and 4 = most severe) or a 0 to 2
spectrum (0 = absent/none and 2 = severe). The HAM-D heavily emphasizes somatic symptoms of
depression and works best for the assessment of individuals with relatively severe illness.

The Montgomery Asberg Depression Rating Scale (MADRS) is a frequently used observer-rated
depression scale. The scale was developed more than 20 years ago but is still favourite among
researchers to measure the severity of depressive disorders and the changes of depressive symptoms
during therapy.

2. B. The EPDS has been developed as a screening tool for detecting postnatal depression. It consists of
reactions to 10 statements within a questionnaire which mothers are asked to complete at regular
intervals after their baby's birth.

3. I.Simpson Angus scale is a tool used to assess EPSEs.


Incorrect
Marks for this submission: 0/3.
Question 19
Marks: 0/3
Adverse effects:

A. Addison‘s disease
B. Cardiomyopathy
C. Cushings Syndrome
D. Delirium Tremens
E. Diabetes Insipidis
F. Diabetes Mellitus
G. Diabetic ketoacidosis
H. Lithium toxicity
I. Neuroleptic malignant syndrome
J. Phaeochromocytoma

A young man on Clozapine complains of breathlessness and has a persistent tachycardia


A woman on lithium for many years who recently complains of feeling weak. Her serum sodium is
raised and she looks dehydrated.
A young man started on an antipsychotic 2 weeks ago now complains of stiff muscles, fever, labile
BP, and confusion.

1. B - Risks of cardiomyopathy and cardiomyositis peak in the first two months of Clozapine
treatment. Obvious signs are hyperventilation, oedema and tachycardia. Clozapine causes a
transient tachycardia itself, so it is important to evaluate the persistence of any tachycardia and
associated signs such as exercise intolerance, breathlessness and chest pain. If any suspicion
stop the medication and seek a cardiology opinion. Needs medical intervention and
consideration of drugs that are less interfering with cardiac functions.

2. E - Lithium induced diabetes insipidis should be considered in patients with unexplained


hypernatremia and dehydration. This is reversible on stopping Lithium.

3. I - NMS occurs due to excessive blockade of dopamine receptors leading to acute deficiency of
dopamine in the brain in response of prescription of a bigger starting dose in a drug naive
patient or rapid escalation of an existing dose. Commonly seen with typicals and considered to
manifest in patients who are sensitive to medication effects, although increasing numbers of
case reports have entered the literature regarding atypical antipsychotic induced NMS. It is a
medical emergency and patient should be transferred to a medical ward for treatment. CK levels
can be checked. They are usually between 1000-2000. If untreated can result in renal failure,
respiratory depression, coma and death. Death from NMS is now very rare, around 0.01%-
0.12% , although in the past levels as high as 20% were reported. The reduction is largely due to
the early recognition and the decreased reduce of typicals.

Incorrect
Marks for this submission: 0/3.

General Adult Psychiatry II


Anxiety Disorders - Aetiology
1
Marks: 0/1
The autonomic aspects of the classic fear response are believed to be mediated by which brain
structure?
Choose one answer.
a. Frontal cortex
b. Amygdala
c. Hippocampus
d. Parietal lobe
e. Cerebellum

The amygdala is important in attaching emotional significance to external stimuli and is believed to be
involved in mediating the unconscious memories, for example autonomic aspects of the classic fear
response. The hippocampus and other areas of the temporal lobe are believed to mediate conscious
memories, e.g. details of a traumatic event. Neuroimaging studies of people with PTSD have shown
decreased activity in medial prefrontal and anterior cingulate areas to be correlated with increased
activity in the amygdale. The current hypothesis is that PTSD represents a failure of medial prefrontal
and/or anterior cingulated networks to regulate amygdala activity, resulting in hyperactivity to external
threat.

Bisson, I. J. Pharmacological treatment of post-traumatic stress disorder. Advances in Psychiatric


Treatment (2007), vol. 13, 119-126

Incorrect
Marks for this submission: 0/1.
Question 2
Marks: 0/1
Current neuroanatomical models of PTSD include all the following features EXCEPT:
Choose one answer.
a. The amygdala is involved in the processing of incoming information.
b. The hippocampus is involved in the mediation of conscious memory.
c. There are connections between frontal brain areas and hypothalamus.
d. There is increased activity in the frontal cortical areas.
e. There is hyperactivation of the amygdala.

Neuroimaging studies of people with PTSD have shown decreased activity in medial prefrontal and
anterior cingulate areas to be correlated with increased activity in the amygdala. The amygdala is
important in attaching emotional significance to external stimuli and is believed to be involved in
mediating the unconscious memories, for example autonomic aspects of the classic fear response. The
hippocampus and other areas of the temporal lobe are believed to mediate conscious memories, e.g.
details of a traumatic event. The current hypothesis is that PTSD represents a failure of medial
prefrontal and/or anterior cingulated networks to regulate amygdala activity, resulting in hyperactivity
to external threat.

Bisson, I. J. Pharmacological treatment of post-traumatic stress disorder. Advances in Psychiatric


Treatment (2007), vol. 13, 119-126
Incorrect
Marks for this submission: 0/1.
Question 3
Marks: 0/1
Fear and anxiety states activate noradrenergic neurons in which ONE of the following brain structures?
Choose one answer.
a. Hypothalamus
b. Hippocampus
c. Thalamus
d. Locus coeruleus
e. Amygdala

The locus coeruleus is the area in the brainstem from which all brain connections using noradrenaline
arise. Fear and anxiety states activate NE release in many stress related limbic structures like amygdala,
stria terminalis, medial prefrontal cortex and lateral septum.

Morilak, D. A., et al. Role of norepinephrine in the behavioural response to stress, Prog
Neuropsychopharmacol Biol Psychiatry, 2005, 1214-1224.

Incorrect
Marks for this submission: 0/1.
Question 4
Marks: 0/1
Which anxiety disorder is associated with increased activity in the cortical -basal ganglia - thalamic
pathway?
Choose one answer.
a. Social Phobia
b. Obsessive-compulsive disorder
c. PTSD
d. Generalised Anxiety disorder
e. Panic Disorder

Functional imaging studies suggest that in OCD patients, there is abnormal metabolic activity in the
orbitofrontal cortex, the anterior cingulate/caudal medial prefrontal cortex, and the caudate nucleus (the
anterior part of the striatum). Activity within this cortico-basal ganglia network (sometimes called the
'OCD circuit') is increased at rest relative to that in controls, is accentuated during provocation of
symptoms, and is attenuated following successful treatment. This neuroimaging evidence supports
earlier suggestions of basal ganglia involvement in OCD based on the fact that focal lesions in the
striatum or in its target structure, the pallidum, can produce striking OCD-like behaviours.

Graybiel A.M., Rauch S.L. Toward a neurobiology of obsessive-compulsive disorder Neuron, 28,
2000: 343-347

Incorrect
Marks for this submission: 0/1.
Question 5
Marks: 0/1
The vulnerability factors associated with PTSD include which ONE of the following?
Choose one answer.
a. Middle age
b. Male gender
c. Good impulse control
d. Internal locus of control
e. Personality disorders

The vulnerability factors associated with PTSD include demographic factors (Female gender, younger
or older age group, lower socio-economic status, single / divorced, unemployed, low educational level,
low social support), trauma related factors (previous traumatic events, including childhood unresolved
trauma, severity of exposure, negative appraisal of the event) and clinical factors (pre-existing
psychopathology including personality disorders) and other factors like lack of internal locus of
control.

Ahmed, A. S. Post-traumatic stress disorder, resilience and vulnerability. Advances in Psychiatric


Treatment (2007), vol. 13, 369-375.

Incorrect
Marks for this submission: 0/1.
Question 6
Marks: 0/1
Each of the following agents have been used to provoke panic attacks in experimental studies except:
Choose one answer.
a. Carbon dioxide
b. Disulfiram
c. Lactate
d. Cholecystokinin
e. Yohimbine
Carbon dioxide, sodium lactate, sodium bicarbonate, yohimbine, m-chlorophenylpiperazine (mCPP),
flumazenil, cholecystokinin & caffeine have all been used to induce panic attacks.
Incorrect
Marks for this submission: 0/1.

Anxiety Disorders - Diagnosis

1
Marks: 0/1
To satisfy criteria for the diagnosis of OCD, a patient would have which ONE of the following?
Choose one answer.
a. Egosyntonic intrusive thoughts.
b. Easy to resist intrusive thoughts.
c. Suicidal thoughts.
d. Egodystonic intrusive thoughts.
e. Transient obsessive thoughts.

ICD 10 criteria for OCD includes repititive, intrusive, egodystonic thoughts which are almost
impossible to resist and it has an impact on day to day functioning.

The ICD10 classification of mental and behavioural disorders: clinical descriptions and diagnostic
guidelines. 1992.

Incorrect
Marks for this submission: 0/1.
Question 2
Marks: 0/1
Social phobias are most associated with which ONE of the following?
Choose one answer.
a. Depression.
b. Obsessions.
c. Avoidance to reduce anxiety.
d. Generalising to other phobias.
e. Compulsions to reduce anxiety.

ICD 10 criteria for phobias includes marked fear, or an avoidance of the situation causing phobia.

The ICD10 classification of mental and behavioural disorders: clinical descriptions and diagnostic
guidelines. 1992.

Incorrect
Marks for this submission: 0/1.
Question 3
Marks: 0/1
Alcohol abuse is most commonly associated with which ONE of the following anxiety disorders?
Choose one answer.
a. Social phobia
b. Generalised anxiety
c. Needle phobia
d. OCD
e. Animal phobia

The two anxiety disorders closely tied to alcoholism are panic disorder and social phobia. Comorbid
anxiety also makes the outcome of alcoholism poor.

Schneider et al. Comorbid anxiety and affective disorder in alcohol-dependent patients seeking
treatment: the first Multicentre Study in Germany. Alcohol and Alcoholism Vol. 36, No. 3, pp. 219-
223, 2001.

Incorrect
Marks for this submission: 0/1.
Question 4
Marks: 0/1
Agorophobia is NOT considered to be a fear of:
Choose one answer.
a. Leaving home
b. Open empty spaces
c. Entering shops
d. Being in crowds and public places
e. Travelling alone in trains, buses or planes

It is a fairly well-defined cluster of phobias embracing fears of leaving home, entering shops, crowds
and public places, or travelling alone in trains, buses or planes.

The ICD10 classification of mental and behavioural disorders: clinical descriptions and diagnostic
guidelines. 1992. P135.

Incorrect
Marks for this submission: 0/1.
Question 5
Marks: 0/1
Obsessive compulsive disorder is associated with many neurological conditions including which ONE
of the following?
Choose one answer.
a. Motor neuron disease
b. Herpes encephalitis
c. Sydenham's chorea
d. Korsakoff syndrome
e. Vitamin B12 deficiency

OCD is also associated with Gilles de la Tourette's syndrome and encephalitis lethargica.

Cheyette SR, Cummings JL: Encephalitis lethargica: lessons for contemporary neuropsychiatry. J
Neuropsychiatry Clin Neurosci 1995; 7(2):125-134.

Incorrect
Marks for this submission: 0/1.
Question 6
Marks: 0/1
An exaggerated startle response is seen in which ONE of the following conditions?
Choose one answer.
a. Paraphilias
b. Depression
c. Mania
d. Phobic disorders
e. PTSD
Symptoms of hyperarousal seen in PTSD include hypervigilance, exaggerated startle response,
irritability, difficulty concentrating, and sleep problems. This cluster is also noted in acute stress
reaction.

Companion to Psychiatric Studies, Ed: Johnstone, Lawrie et al, 7th ed., p477.

Incorrect
Marks for this submission: 0/1.
Question 7
Marks: 0/1
Which ONE of the following is the most common psychiatric disorder in anankastic personality?
Choose one answer.
a. Somatisation syndrome
b. Depression
c. OCD
d. Dementia
e. Schizophrenia

Depression is more common in patients of anakastic personality disorder. OCD is associated in less
than a quarter patients.

New Oxford Textbook of Psychiatry, Ed: Gelder, Lopez-Ibor Jr & Andreasen, 2000 edition, p946.

Incorrect
Marks for this submission: 0/1.
Question 8
Marks: 0/1
Magical undoing refers to a psychodynamic defence mechanism. It is most commonly associated with
which ONE of the following?
Choose one answer.
a. Adolescent psychosis.
b. Obsessive-compulsive disorder.
c. Depression.
d. Autism.
e. Anankastic personality disorder.

It is a psychodynamic explanation for OCD symptoms. Magical undoing/thinking refers to


omnipotence of thought where persons believe that they can cause an event to occur merely by
thinking, leading to fear of having an aggressive thought. It also refers to the belief that certain rituals
can undo events that might happen in the external world.

Synopsis of Psychiatry, Kaplan & Sadock, 9th edition, p619.

Incorrect
Marks for this submission: 0/1.
Question 9
Marks: 0/1
During a final exam, a worried college student suddenly experiences perioral tingling, carpo-pedal
spasms, and feelings of derealization. The most likely diagnosis is:
Choose one answer.
a. Somatisation
b. Panic attacks
c. Conversion disorder
d. Factitious disorder
e. Malingering

Hyperventilation is a central feature of panic disorder and acute anxiety attacks. It causes hypocapnia
and respiratory alkalosis, which in turn lead to decreased cerebral blood flow and a decrease in ionized
serum calcium. Dizziness, derealization, and light-headedness are due to the cerebral vasoconstriction,
while circumoral tingling, carpopedal spasm, and paresthesias are symptoms of tetany.

The ICD10 classification of mental and behavioural disorders: clinical descriptions and diagnostic
guidelines. 1992. P135.

Incorrect
Marks for this submission: 0/1.
Question 10
Marks: 0/1
Social phobia is more likely to impair functioning in which ONE of the following settings?
Choose one answer.
a. Queuing for tube tickets.
b. Attending a public lecture.
c. A small group of acquaintances.
d. Singing in a chorus.
e. Supermarkets.

Presence of 3-4 people makes anxiety higher than a larger group as chances of scrutiny and
embarrassment are more in smaller groups.

The ICD10 classification of mental and behavioural disorders: clinical descriptions and diagnostic
guidelines. 1992. P136.

Incorrect
Marks for this submission: 0/1.
Question 11
Marks: 0/1
Apart from OCD, compulsive acts are also seen in which ONE of the following conditions?
Choose one answer.
a. Animal phobia
b. Frontotemporal dementia
c. Generalised Anxiety disorder
d. Panic disorder
e. PTSD

Frontal lesions can produce compulsive behaviour without obsessions. This supports the theory that
OCD has an organic basis.

Symptoms in the mind, SIMS, Third edition. p336.

Incorrect
Marks for this submission: 0/1.
Question 12
Marks: 0/1
A patient with recurrent panic attacks is LEAST likely to develop which ONE of the following?
Choose one answer.
a. Substance misuse
b. Depression
c. Panic disorder
d. Agoraphobia
e. OCD

OCD does not share the common feature of panic attacks seen in other anxiety disorders. Recurrent
panic attacks can lead to alcohol use, which can worsen the anxiety disorder further.

New Oxford Textbook of Psychiatry, Ed: Gelder, Lopez-Ibor Jr & Andreasen, 2000 edition, p811.

Synopsis of psychiatry, 9th ed., Ed: Kaplan & Sadock, p599.

Incorrect
Marks for this submission: 0/1.
Question 13
Marks: 0/1
A 28 yr. old typist pays 6 pounds-a-day for parking as she is terrified by the thought of being trapped in
the hospital underground parking. The most likely diagnosis is:
Choose one answer.
a. Performance anxiety
b. Generalized anxiety disorder
c. Agoraphobia
d. Social phobia
e. Specific phobia

This lady has a specific fear, avoidance and secondary impairment characteristic of phobias. Specific
phobias often have onset followed by notable precipitants. Freudian theory of displacement is not
widely accepted as an aetiology.

The ICD10 classification of mental and behavioural disorders: clinical descriptions and diagnostic
guidelines. 1992, p135.

Incorrect
Marks for this submission: 0/1.
Question 14
Marks: 0/1
Agoraphobia is more frequent while travelling in trains than cars. Which ONE of the
following statements explains this?
Choose one answer.
a. There are more people who can help in a train.
b. The inescapable nature of the situation.
c. It is easier to escape from a train.
d. Agoraphobics are from higher socioeconomic class.
e. Road traffic accidents can cause agoraphobia.

Agoraphobia depends on the inescapable nature of a situation - it is easier to escape from a car than
train.

The ICD10 classification of mental and behavioural disorders: clinical descriptions and diagnostic
guidelines. 1992. P135.

Incorrect
Marks for this submission: 0/1.

Anxiety Disorders - Epidemiology

1
Marks: 0/1
Which ONE of the following anxiety disorders has the earliest age of onset?
Choose one answer.
a. Generalised Anxiety disorder
b. Social Phobia
c. Post traumatic stress disorder
d. Panic Disorder
e. Agoraphobia

Social phobias often begin at adolescence. Panic disorders can present at any age but has two peaks of
onset, the larger at around 15-25, and a later peak at 45-54. 15-35 is the first peak for agoraphobia, with
45-54 the second peak (very similar to panic disorders).

New Oxford Textbook of Psychiatry, Ed: Gelder, Lopez-Ibor Jr & Andreasen, 2000 edition P811.

Synopsis of psychiatry, 9th ed., Ed: Kaplan & Sadock, p599.

Incorrect
Marks for this submission: 0/1.
Question 2
Marks: 0/1
Which ONE of the following anxiety disorders is LEAST common in women presenting to primary
care?
Choose one answer.
a. Generalised anxiety disorder
b. Agoraphobia
c. Specific phobias
d. Social Phobia
e. Panic Disorder

Most neurotic disorders, except social phobia, are more common in women. Although men and
women with social phobia seek treatment in relatively equal numbers, epidemiological studies suggest
that men are more likely than women to have social phobia. In agoraphobia, extensive avoidance is
more common in females, leading to disparity in clinical presentation - more men with agoraphobia do
not seek help. Absence of panic attacks could also reduce help-seeking in men.

Companion to Psychiatric Studies, Ed: Johnstone, Lawrie et al, Seventh edition, p471.

New Oxford Textbook of Psychiatry, Ed: Gelder, Lopez-Ibor Jr & Andreasen, 2000 edition, p796.

Incorrect
Marks for this submission: 0/1.
Question 3
Marks: 0/1
The anxiety disorder with the highest prevalence according to the National Psychiatric Morbidity
household survey is:
Choose one answer.
a. Agoraphobia
b. Phobic disorders
c. Panic disorder
d. Obsessive compulsive disorder
e. Generalised anxiety disorder

As per the NPM, phobic disorders are the most prevalent. Phobic disorders had a prevalence of 11.9,
GAD of 4.5, OCD of 1.6, and panic disorder of 1.0.

Companion to Psychiatric Studies, Ed: Johnstone, Lawrie et al, Seventh edition, p468.

Incorrect
Marks for this submission: 0/1.

Anxiety Disorders - Management

1
Marks: 0/1
In the treatment of OCD, NICE recommends AGAINST the use of all the following medications
EXCEPT:
Choose one answer.
a. Moclobomide
b. Venlafaxine
c. Imipramine
d. Desipramine
e. Fluvoxamine

NICE recommends the use of SSRIs in cases of moderate or severe OCD. NICE has also provided a list
of drugs not recommended for OCD, which include TCAs (except Clomipramine), SNRIs (including
venlafaxine), and anxiolytics.

NICE: Obsessive-compulsive disorder 2005.

Incorrect
Marks for this submission: 0/1.
Question 2
Marks: 0/1
Which ONE of the following interventions has no therapeutic value in PTSD?
Choose one answer.
a. Debriefing after trauma.
b. Trauma focussed CBT.
c. Antidepressant pharmacotherapy.
d. Short term benzodiazepines.
e. Eye movement desensitisation and Reprocessing.

A Cochrane review clearly states that routine debriefing after trauma is of no use. Psychological
debriefing is either equivalent to, or worse than control or educational interventions in preventing or
reducing the severity of PTSD, depression, anxiety and general psychological morbidity after trauma.
There is even some suggestion that it may increase the risk of PTSD and depression.

Click here for reference

Incorrect
Marks for this submission: 0/1.
Question 3
Marks: 0/1
In the treatment of PTSD, there is no RCT evidence regarding the efficacy of which ONE of the
following?
Choose one answer.
a. Carbamazepine
b. Venlafaxine
c. Mitrazapine
d. Olanzapine
e. Fluoxetine

There are RCTs which provides evidence for the efficacy of Mitrazapine, Paroxetine, Imipramine &
Phenelyzine, and RCTs that provide evidence for non-efficacy of Venlafaxine, Olanzapine, Fluoxetine
& Sertraline, but none for Carbamazepine.

Bisson I J. Pharmacological treatment of post-traumatic stress disorder. Advances in Psychiatric


Treatment (2007), vol. 13, 119-126.

Incorrect
Marks for this submission: 0/1.
Question 4
Marks: 0/1
The NICE guidelines for PTSD recommends Mirtazapine or Paroxetine in which ONE of the following
circumstances?
Choose one answer.
a. Third line treatment of PTSD.
b. Treatment to be initiated in secondary care.
c. For treatment of treatment resistant PTSD.
d. First-line treatment of PTSD.
e. Second-line treatment of PTSD.

Drug treatments (paroxetine or mirtazapine for general use, and amitriptyline or phenelzine for
initiation only by mental health specialists) should be considered for the treatment of PTSD in adults
who express a preference not to engage in trauma-focused psychological treatment. NICE recommends
that trauma-focussed interventions (trauma-focused cognitive behavioural therapy [CBT] or eye
movement desensitisation and reprocessing [EMDR]) should be first line treatments for PTSD.

NICE: Post-traumatic stress disorder (PTSD). The management of PTSD in adults and children in
primary and secondary care. March 2005.

Incorrect
Marks for this submission: 0/1.
Question 5
Marks: 0/1
Assessment for cognitive behavioural therapy in OCD involves all of the following EXCEPT:
Choose one answer.
a. Assessment of the patients' readiness to change.
b. Patients' degree of insight.
c. Knowledge of the degree of family involvement.
d. Context in which Obsessions and compulsions have developed.
e. Forensic assessment of intrusive thoughts & impulses.

The assessment for CBT includes: the context in which OCD has developed; the nature of the
obsession(s) - their content, the degree of insight, the frequency of their occurrence, the triggers, the
feared consequence, the patient's appraisal of the obsession; the main emotion(s) linked with the
obsession or intrusion; the avoidance behaviour; the degree of family involvement and the degree of
impact in the person's occupational, social and family life, goals and valued directions in life, readiness
to change and expectations of therapy, including previous experience of CBT for the disorder.

Veale D. Cognitive-behavioural therapy for obsessive-compulsive disorder. Advances in Psychiatric


Treatment (2007), vol. 13, 438-446.

Incorrect
Marks for this submission: 0/1.
Question 6
Marks: 0/1
A 36 year-old woman comes into your outpatient clinic. She is obsessed with dirt and contamination,
and has to wash her hands up to twenty times if she touches anything. The treatment you would are
most likely to recommend would be:
Choose one answer.
a. Cognitive Analytical therapy.
b. Psychodynamic therapy.
c. Exposure and response prevention.
d. Interpersonal therapy.
e. Cognitive behaviour therapy.

NICE guideline on the management of OCD suggest that CBT with exposure and response prevention
should be offered to patients with this disorder. If this is not effective or the disorder requires
medication as well, an SSRI should be used. For more resistant cases, Risperidone might need to be
added.

NICE guidelines on the management of OCD and body dysmorphic disorder, 2005.

Incorrect
Marks for this submission: 0/1.
Question 7
Marks: 0/1
There is no evidence for which of the following treatments for PTSD?
Choose one answer.
a. EMDR
b. Group psychotherapy
c. Trauma focused CBT
d. Hypnotherapy
e. Stress management

Although one four-arm RCT suggested that hypnotherapy reduced intrusive symptoms and avoidance
symptom scores significantly, a BMJ clinical evidence review concluded that there was insufficient
evidence for the use of hypnotherapy for PTSD.

Bisson (2007) Click here for reference

Incorrect
Marks for this submission: 0/1.
Question 8
Marks: 0/1
A 28 year-old lady with obsessive compulsive disorder is being treated with Sertraline (50 mg) for the
last two months. She reports no improvement in her symptoms. The next recommended step would be:
Choose one answer.
a. Increase the dose of sertraline.
b. Switch to venlafaxine.
c. Add an antipsychotic.
d. Change to a different antidepressant.
e. Add sodium valproate.

Sertraline is usually increased upto a maximum of 200mg/day before the trial is considered to have
failed. According to NICE guidelines, after failing trial of an antidepressant with adequate dosage and
duration one should try another SSRI/Clomipramine; if this also fails, combine antidepressant with
CBT; if this fails as well, progress to add an antipsychotic. The dosage of antidepressant medication
required for treatment of OCD is usually higher than required for depression.

NICE guidelines on the management of OCD and body dysmorphic disorder, 2005.

New Oxford Textbook of Psychiatry, Ed: Gelder, Lopez-Ibor Jr & Andreasen, 2000 edition, p827.

Incorrect
Marks for this submission: 0/1.
Question 9
Marks: 0/1
When using antidepressants to treat OCD, which ONE of the following may be necessary?
Choose one answer.
a. Lower dose for longer duration.
b. Higher dose for longer duration.
c. Higher dose for shorter duration.
d. Lower dose for shorter duration.
e. Same dose but longer duration.

Usually higher doses are needed for longer times, and relapse on withdrawal is almost inevitable in
severe illness. The effect is due to disturbed serotonergic mechanisms. Though more effective in
comorbid depressed patients, the effect is independent of antidepressant action.

New Oxford Textbook of Psychiatry, Ed: Gelder, Lopez-Ibor Jr & Andreasen, 2000 edition, p827.

Incorrect
Marks for this submission: 0/1.
Question 10
Marks: 0/1
The behavioural therapy of agoraphobia includes which ONE of the following?
Choose one answer.
a. Systematic desensitisation.
b. Exposure and response prevention.
c. Thought stopping.
d. Habit reversal.
e. Massed negative practice.

Relaxation, graded exposure and desensitisation constitute behavioural therapy for agoraphobia.

New Oxford Textbook of Psychiatry, Ed: Gelder, Lopez-Ibor Jr & Andreasen, 2000 edition, p815.

Incorrect
Marks for this submission: 0/1.

Dissociative & Somatoform Disorders

1
Marks: 0/1
Many anxiety disorders have a gender bias. What is the male:female ratio for Somatization?
Choose one answer.
a. 1:1
b. 2:1
c. 1:5
d. 1:2
e. 1:10

Somatisation is twice more common in women as compared to men.

Oxford textbook of Psychiatry, 4th edition, 2001.

Incorrect
Marks for this submission: 0/1.
Question 2
Marks: 0/1
Conversion disorder is rarely seen in clinical practice. It is associated with which ONE of the following
features?
Choose one answer.
a. Equal tendency for symptoms to present on left and right sides of the body.
b. More symptoms on the left side of the body.
c. Sexual symptoms.
d. Gastrointestinal symptoms.
e. More symptoms on the right side of the body.

It has been suggested that conversion symptoms are more common on the left side of the
body. However, a recent systematic review has suggested that this is not borne out by the literature and
may reflect bias in previous reporting (Stone et al 2002a). Conversion disorder appears to be
more common in women than in men.

Companion to Psychiatric Studies, Ed: Johnstone, Lawrie et al, Seventh edition, p481.

Incorrect
Marks for this submission: 0/1.
Question 3
Marks: 0/1
Dissociative fugue is characterized by which ONE of the following?
Choose one answer.
a. Hereditary aetiology.
b. The subject learning a new language.
c. The subject making a purposeful journey.
d. The subject retaining his first name only.
e. The subject deteriorating in memory.

In fugue, the relocation is different from wandering as it is apparently purposeful. There is no proven
inheritance risk for dissociation.

Companion to Psychiatric Studies, Ed: Johnstone, Lawrie et al, Seventh edition, p1093.

Incorrect
Marks for this submission: 0/1.
Question 4
Marks: 0/1
Psychatric patients in medical settings are UNLIKELY to present with which ONE of the following?
Choose one answer.
a. Induced medical symptoms.
b. Bodily pain symptoms.
c. Refusal to accept medical model of illness.
d. Poor compliance with medical treatment.
e. Anxiety symptoms.

Patients seen in medical services who require psychiatric attention are more likely to have presented
with somatic than with psychological symptoms; for example, the patient who meets criteria for a
depressive disorder may have presented with bodily pain. Alternatively, they may have presented with
a problem in medical management or with poor compliance with medical treatment, such as poorly
controlled insulin-dependent diabetes mellitus. Finally, they may present with a behaviourally induced
medical condition, for example, because of overdose, the effect of excess alcohol intake or because of
other forms of self-harm.

Companion to Psychiatric Studies, Ed: Johnstone, Lawrie et al, Seventh edition, p685.

Incorrect
Marks for this submission: 0/1.
Question 5
Marks: 0/1
The difference between somatisation and hypochondriasis is:
Choose one answer.
a. Hypochondriacal patient expects treatment approaches.
b. Hypochondriacal patient is delusional.
c. Somatising patient expects symptom relief.
d. Somatising patient expects prognostic advice.
e. Somatising patient expects diagnostic clarification.

Hypochondriasis is the fear of having physical disorder. In somatisation the preoccupation is with the
symptoms and not the disorder/diagnosis. Treatment is expected for alleviation of symptoms and not
investigation for confirmation of diagnosis.

New Oxford Textbook of Psychiatry, Ed: Gelder, Lopez-Ibor Jr & Andreasen, 2000 edition, p1080.

Incorrect
Marks for this submission: 0/1.
Question 6
Marks: 0/1
A malnourished man who has had numerous admissions to a local general hospital after fainting at
public places asks for admission as he is very dizzy. Physical examination was normal. The likely
diagnosis is:
Choose one answer.
a. Malingering
b. Factitious disorder
c. Dysthymia
d. Somatoform disorder
e. Hypochondriasis

This man is probably homeless and so has a motive in feigning illness, thus qualifying as a malingerer.

New Oxford Textbook of Psychiatry, Ed: Gelder, Lopez-Ibor Jr & Andreasen, 2000 edition, p1126.

Incorrect
Marks for this submission: 0/1.
Question 7
Marks: 0/1
Which ONE of the following conditions is usually transient and does not have strong evidence-based
treatment?
Choose one answer.
a. Depersonalisation-derealisation syndrome
b. Adolescent Psychosis
c. Depression
d. OCD
e. Somatoform disorder
Reassurance and explanation, antidepressant drugs, exercise and relaxation have all been tried in
depersonalisation syndrome. There is however, little in the way of clinical trial evidence to guide
treatment.

Companion to Psychiatric Studies, Ed: Johnstone, Lawrie et al, Seventh edition, p483

Incorrect
Marks for this submission: 0/1.
Question 8
Marks: 0/1
Which ONE of the following in ICD-10 is the diagnosis closest to Chronic Fatigue Syndrome?
Choose one answer.
a. Psychasthenia
b. Adjustment disorder
c. Neurasthenia
d. Mixed anxiety and depression
e. Thrombasthenia

CFS was not recognised when ICD-10 was drafted. Neurasthenia is the code with close agreement.

The ICD-10 classification of mental and behavioural disorders: clinical descriptions and diagnostic
guidelines. 1992, p7

Incorrect
Marks for this submission: 0/1.
Question 9
Marks: 0/1
What is the motivation behind a factitious disorder?
Choose one answer.
a. Seeking diagnostic clarification.
b. Resolution of unconscious conflict.
c. Monetary gain without sick role.
d. Sick role but no monetary gain.
e. Sick role and monetary gain.

Illness is feigned for no obvious reason in factitious disorder.

New Oxford Textbook of Psychiatry, Ed: Gelder, Lopez-Ibor Jr & Andreasen, 2000 edition, p1126.

Incorrect
Marks for this submission: 0/1.
Question 10
Marks: 0/1
Which ONE of the following is regarded as a medical functional syndrome?
Choose one answer.
a. Fibromyalgia
b. Hypertension
c. Cruetzfeld Jacob disease
d. Bronchial asthma
e. Multiple sclerosis

An American population survey found that a quarter to a third of the population had been severely
troubled with symptoms such as pain and fatigue at some time in their lives and that in one-third of
these cases the symptoms remained unexplained by somatic pathology. They could therefore be
regarded as functional symptoms (Kroenke & Price 1993). In primary care, one in five new
consultations are functional somatic symptoms (Goldberg et al 1988).

Kroenke K., Price, R. K. (1993). Symptoms in the community: prevalence, classification, and
psychiatric comorbidity. Archives of Internal Medicine 153:2474-2480.

Incorrect
Marks for this submission: 0/1.
Question 11
Marks: 0/1
'Briquet syndrome' or 'St. Louis hysteria' is an eponym for multiple somatisation disorder. Which ONE
of the following is NOT associated with this condition?
Choose one answer.
a. Analgesic misuse.
b. Repeated contact with physicians.
c. Response to antidepressants.
d. Response to behavioural therapy.
e. Age of onset after 40.

Criteria for the diagnosis include: a history beginning before the age of 30, multiple physical symptoms
(including pain, gastrointestinal symptoms, sexual symptoms, and psuedoneurological symptoms)
severe enough to interfere with the patient's life and lead to contact with physicians, and a lack of any
medical explanation despite appropriate investigations.

Companion to Psychiatric Studies, Ed: Johnstone, Lawrie et al, Seventh edition, p805.

Incorrect
Marks for this submission: 0/1.

Suicide & DSH


1
Marks: 0/1
Suicide is one of the most common causes of death in which ONE of the following groups of patients?
Choose one answer.
a. Dementia
b. Autism
c. Schizophrenia
d. Downs syndrome
e. Postpartum mothers

The national confidential enquiry on maternal death (Oates, 2001) showed the high prevalence of
suicide in post-partum women. Cardiovascular mortality is more common in schizophrenia and
dementia.

Companion to Psychiatric Studies, Ed: Johnstone, Lawrie et al, Seventh edition, p663.

Incorrect
Marks for this submission: 0/1.
Question 2
Marks: 0/1
Factors predicting suicide after deliberate self-harm include which ONE of the following?
Choose one answer.
a. Unemployment
b. Female gender
c. Being married
d. Suburban neighbourhood
e. Anxiety disorders

The following are some of the risk factors for suicide: Male gender (3 times more likely than women),
Advancing age, Unemployment, Concurrent mental disorder, Previous suicide attempt, Alcohol and
drug abuse, Low socio-economic status, Previous psychiatric treatment, Certain professions (eg.
doctors, students), Lack of social support, Living alone.

Companion to Psychiatric Studies, Ed: Johnstone, Lawrie et al, Seventh edition, p661.

Incorrect
Marks for this submission: 0/1.
Question 3
Marks: 0/1
The risk of self harm is increased when which ONE of the following phenomena is noted on mental
state examination?
Choose one answer.
a. Psychomotor retardation
b. Dystonic reactions
c. Somatic syndrome
d. Command hallucinations
e. Obsessions

OCD in the absence of depression is associated with lower risk of suicide than many other psychiatric
disorders.

Companion to Psychiatric Studies, Ed: Johnstone, Lawrie et al, Seventh edition, p664.
Incorrect
Marks for this submission: 0/1.
Question 4
Marks: 0/1
Which ONE of the following neurological illnesses is independently associated with elevated suicide
risk, irrespective of depression?
Choose one answer.
a. Cerebellar ataxia
b. Parkinsons disease
c. Myopathy
d. Epilepsy
e. Stroke

Individuals with epilepsy have a higher risk of suicide taking into account coexisting psychiatric
disease, demographic differences, and socioeconomic factors.

Christensen, J., et al (2007). Epilepsy and risk of suicide: a population-based case-control study. The
Lancet Neurology, 6(8): 693-698.

Incorrect
Marks for this submission: 0/1.
Question 5
Marks: 0/1
Which of the following is the most common method of attempting suicide in the UK?
Choose one answer.
a. Hypnotic overdose
b. Antipsychotic overdose
c. Analgesic overdose
d. Opioid overdose
e. Insulin overdose

The most commonly used method in suicide attempts, by both men and women, is self-poisoning.
There has been an enormous rise in the use of paracetamol over the past 20 years; it is now the most
common drug used for suicidal attempts, involved in nearly half of all adolescent overdoses and 70% of
overdoses by children.

Companion to Psychiatric Studies, Ed: Johnstone, Lawrie et al, Seventh edition, p663.

Incorrect
Marks for this submission: 0/1.
Question 6
Marks: 0/1
Deliberate self harm (DSH) is a known risk factor for completed suicide. What is the risk of completed
suicide in the year after an episode of DSH?
Choose one answer.
a. 1 in 100.
b. 1 in 1000.
c. 1 in 10.
d. 1 in 10000.
e. 1 in 5.

Suicide in baseline population occurs in 1 in 10,000 people a year; the incidence in those committing
recent non-fatal self harm increases to around 1 in 100 (around 0.6% - 0.9% ), which was confirmed in
a large scale follow-up study in Oxford.

Hawton, K and Zahl, D. Suicide following deliberate self-harm: long-term follow-up of patients who
presented to a general hospital. The British Journal of Psychiatry (2003) 182: 537-542.

Incorrect
Marks for this submission: 0/1.

Eating Disorders
1
Marks: 0/1
Anorexia nervosa is suggested to have a genetic basis. The hereditary association is strong for which
subtype of the disease?
Choose one answer.
a. Atypical anorexia
b. Restricting type
c. Late onset type
d. Purging type
e. Bingeing type

Concordance rates for monozygotic twins are higher than dizygotic twins in anorexia nervosa. The rate
is 3:1, suggesting a genetic aetiology. Restricting anorexia nervosa shows 65% MZ and 32% DZ
concordance.

Companion to Psychiatric Studies, Ed: Johnstone, Lawrie et al, Seventh edition, p493.

Incorrect
Marks for this submission: 0/1.
Question 2
Marks: 0/1
Which ONE of the following personality traits is most commonly noted in anorexia patients?
Choose one answer.
a. Poor impulse control
b. Perfectionism
c. Daydreaming
d. Dependence
e. Argumentativeness
Anankastic traits are noted in anorexics. High rates of perfectionism is found in non-affected dizygotic
twins of anorexia probands. A general 'neuroticism' inheritance is noted in bulimics, with the family
showing increased alcohol use, mood disorders and obesity.

Strober, M., Lampert, C., Morrell, W., Burroughs, J., and Jacobs, C. (1990). A controlled family study
of anorexia nervosa: evidence of familial aggregation and lack of shared transmission with affective
disorders. International Journal of Eating Disorders, 9, 239-53.

Companion to Psychiatric Studies, Ed: Johnstone, Lawrie et al, Seventh edition, p493.

Incorrect
Marks for this submission: 0/1.
Question 3
Marks: 0/1
The mortality in patients with Anorexia nervosa is higher than in the general population. What is the
increased risk of mortality?
Choose one answer.
a. 6%
b. 15%
c. 20%
d. 10%
e. 2%

Emborg (1999) calculated a SMR in anorexia of 6.7%. Anorexia is one of the most lethal of psychiatric
disorders. It has a 5-10% mortality over ten years. Two-thirds of the deaths are attributable to the direct
effects of the disorder and one third are related to suicide.

Emborg (1999). International Journal of Eating disorders, 25(3): 243-251.

Companion to psychiatric studies, 7th edition, 2004, p493.

Incorrect
Marks for this submission: 0/1.
Question 4
Marks: 0/1
Which of the following is a component of behavioral therapy used in the treatment of obesity?
Choose one answer.
a. Habit reversal
b. Paradoxical injunction
c. Self monitoring
d. Force feeding
e. Negative practice
The simple technique of keeping a diary has been shown to be effective in many cases. The most
consistent self-monitors lose more weight than the least consistent self-monitors. Click here for
reference
Incorrect
Marks for this submission: 0/1.
Question 5
Marks: 0/1
Which ONE of the following findings are most likely to be seen in patients with Anorexia Nervosa?
Choose one answer.
a. High oestrogen
b. Hyperkalaemia
c. Low triiodothyronine
d. Low cortisol
e. High white cell count

The following metabolic and endocrine changes are noted in anorexia: low oestrogen or sex hormones;
hypokalemia; high cortisol; leucopenia with relative lymphocytosis; low LHRH, LH, FSH; low
oestrogen & progesterone; low T3; high cortisol; high fasting growth harmone levels.

Companion to Psychiatric Studies, Ed: Johnstone, Lawrie et al, Seventh edition, p492.

Incorrect
Marks for this submission: 0/1.
Question 6
Marks: 0/1
The babies of anorexic mothers are most likely to be associated with which ONE of the following?
Choose one answer.
a. Lower APGAR scores
b. Pre-term birth
c. Low birth weight
d. Larger head circumference
e. Large-for-date babies

A large scale study found that women with Anorexia Nervosa tend to deliver low birth weight babies,
though the risk of pre-term birth is similar to controls. Apgar scores were within normal limits. Women
with a history of bulimia nervosa (with or without a history of anorexia nervosa) had an increased rate
of lifetime miscarriages, as did women with a history of other psychiatric disorders. Foetal loss is more
common in anorexic women and they are more likely to require caesarean deliveries. Finally the risk of
depression is greater in women with anorexia post partum.

Micali, N. et al (2007). Risk of major adverse perinatal outcomes in women with eating disorders. Br J
Psychiatry, 190:255-9.

Incorrect
Marks for this submission: 0/1.
Question 7
Marks: 0/1
Which ONE of the following is LEAST commonly associated with Bulimia Nervosa?
Choose one answer.
a. Chest pain
b. Dental decay
c. Peptic ulcer
d. Parotid gland enlargement
e. Oesophageal tears

Oesphagitis, oesophageal tears, dental decay are common in Bullimia, however peptic ulcers are not
described in the physical complications of this syndrome. Chest pain occurs secondary to the
hypokalemia and is less common, although around 50% of bullmic patients have electrolyte
disturbances.

Companion to psychiatric studies, 7th edition, 2004, p497.

Incorrect
Marks for this submission: 0/1.
Question 8
Marks: 0/1
Family dynamics are often affected for patients with anorexia nervosa. Which ONE of the following
features is associated with such problems?
Choose one answer.
a. Authoritative parenting
b. Disengagement
c. Unprotected childhood
d. Enmeshment
e. Flexible parenting

Five main characteristics of family interaction were identified as detrimentally linked to anorexic
presentation: enmeshment (a tight web of family relationships with the members appearing to read each
other's minds), overprotectiveness, rigidity, involvement of the sick child in parental conflicts, lack of
resolution of conflicts.

Minuchin, S.,et al (1975). A conceptual model of psychosomatic illness in children: family


organization and family therapy. Archives of General Psychiatry, 32, 1031-8.

New Oxford Textbook of Psychiatry, Ed: Gelder, Lopez-Ibor Jr & Andreasen, 2000 edition, p838.

Incorrect
Marks for this submission: 0/1.

Liaison Psychiatry
1
Marks: 0/1
Which ONE of the following treatments is the most effective management for chronic fatigue
syndrome?
Choose one answer.
a. Antidepressants
b. Psychoeducation
c. Relaxation therapy
d. Self-help group
e. Cognitive behavioural therapy
The mainstay for the treatment of chronic fatigue syndrome is CBT. This involves graded exercise
programs, agreed in collaboration with the patient. The cognitive element may look at illness beliefs,
patient resistance to engaging in the program, self-esteem etc. The use of antidepressants has
conflicting evidence base - the two well conducted trials on this have been contradictory. Belonging to
a self-help group is associated with a poorer prognosis in chronic fatigue - possibly because the more
severely affected patient join such groups when it is clear that their illness has become refractory.
Sharpe et al (1997). General Hospital Psychiatry, 19:185-199.
Incorrect
Marks for this submission: 0/1.
Question 2
Marks: 0/1
Delusional parasitosis is often diagnosed by dermatologists. Which ONE of the following substances is
associated with it?
Choose one answer.
a. Amphetamines
b. Alcohol
c. Opiates
d. Cannabis
e. Cocaine
Delusions of parasitosis is the fixed belief that one is infested with living organisms despite a lack of
medical evidence of such infestation. Cocaine abuse is associated with tactile hallucinations, described
in many cases as crawling, biting, or stinging or other perceptual abnormalities such as auditory
hallucinations of buzzing or other sounds. Delusions are often of a cutaneous invasion, so patients
frequently present to dermatologists. Delusions of parasitosis appears to be uncommon, with an equal
sex distribution in patients under the age of 50 and a female to male ratio of 3 to 1 in patients aged over
50.
Click here for reference
Incorrect
Marks for this submission: 0/1.
Question 3
Marks: 0/1
The puerperium is a high-risk period for psychiatric illnesses. The incidence of puerperal psychosis is
Choose one answer.
a. 4 in 1000
b. 1 in 100
c. 1 in 10
d. 1-2 in 1000
e. 3 in 1000
The incidence of puerperal psychosis is variably quoted as 1 in 500 to 1000. This is constant across
various cultures.
Companion to Psychiatric Studies, Ed: Johnstone, Lawrie et al, Seventh edition, p747.
Incorrect
Marks for this submission: 0/1.
Question 4
Marks: 0/1
Which of the following would be the first-line pharmacological treatment for body dysmorphic
disorder?
Choose one answer.
a. Tricyclic antidepressants (TCA)
b. Selective serotonin uptake inhibitors (SSRI)
c. Monoamine oxidase uptake inhibitors (MAOI)
d. Serotonin-noradrenaline reuptake inhibitors (SNRI)
e. Selective noradrenaline reuptake inhibitors
Treatment trials have shown that SSRIs or tricyclics with serotonergic rather than noradrenergic
properties are the most helpful drugs. NICE guidelines recommend against the use of tricyclics (except
clomipramine) They also do not recommend use of MAOIs or SNRIs.
NICE guidance (2005). Core interventions in the treatment of obsessive-compulsive disorder and body
dysmorphic disorder.
Incorrect
Marks for this submission: 0/1.
Question 5
Marks: 0/1
Individuals with which ONE of the following disorders fake symptoms or disorders as part of a need to
maintain the sick role?
Choose one answer.
a. Malingering disorder
b. Hypochondriacal disorder
c. Factitious disorder by proxy
d. Factitious disorder
e. Malingering disorder by proxy
Patients with factitious disorder knowingly fake symptoms, but do so for psychological reasons and not
for monetary or other discrete objectives as in the case of Malingering. They usually prefer the sick role
and may move from hospital to hospital in order to receive care.
Eisendrath, S. J. & McNiel, D. E. (2004). Factitious physical disorders, litigation, and mortality.
Psychosomatics, 45(4): 350-3.
Incorrect
Marks for this submission: 0/1.
Question 6
Marks: 0/1
Which ONE of the following is associated with a psychogenic cause of impotence?
Choose one answer.
a. Loss of morning erections
b. Recent life event
c. Normal psychosexual history
d. Painful intercourse
e. Loss of masturbatory ability
Emotional factors affecting sex include both interpersonal problems (such as marital/relationship
problems, or lack of trust and open communication between partners) and psychological problems
within the individual (depression, sexual fears or guilt, past sexual trauma, sexual disorders). In organic
sexual dysfunction, masturbation, morning erections are lost, while desire may be preserved in early
stages.
McMahon & Carmita Abdo et al, 2994. Disorders of Orgasm and Ejaculation in Men. The Journal of
Sexual Medicine, 1:1 58.
Incorrect
Marks for this submission: 0/1.
Question 7
Marks: 0/1
A patient presenting to a plastic surgeon for breast reconstructive surgery most commonly suffers from
which ONE of the following?
Choose one answer.
a. Delusional disorder
b. No psychiatric illness
c. Dysmorphophobia
d. Hypochondriasis
e. Gender dysphoria
Most patients attending clinics for breast reconstruction are post-mastectomy patients. They are free
from psychiatric problems more often than not.
Click here for reference
Incorrect
Marks for this submission: 0/1.
Question 8
Marks: 0/1
Puerperal depression is most commonly identified by which ONE of the following?
Choose one answer.
a. Community health visitors
b. Perinatal psychiatric team
c. Partners / spouses
d. Community psychiatric nurse
e. Social workers
The majority of depressions occurring at puerperium are mild and do not require specific psychiatric
intervention. Most will be uncovered during routine postnatal screening by the general practitioner or
health visitor. The provision of extra support and non-directive counselling by suitably trained primary
healthcare workers (such as health visitors in the UK) has been shown in randomised trials to be
effective in reducing depression when compared with routine care (Holden et al 1989, Wickberg &
Hwang 1996).
Companion to Psychiatric Studies, Ed: Johnstone, Lawrie et al, Seventh edition, p749.
Incorrect
Marks for this submission: 0/1.
Question 9
Marks: 0/1
An 80-year old lady is admitted to hospital for evaluation after an 8-month preoccupation with having
colon cancer. She has no history of mental illness. On admission, she reported that her two brothers had
both had colon cancer. As evidence of a possible tumour, she reported having diffuse abdominal pain.
Because of her concern about having cancer, she had seen 11 physicians in the previous year, but each
in turn had been unable to reassure her that she did not have cancer. What is her likely diagnosis?
Choose one answer.
a. Somatoform autonomic dysfunction
b. Factitious disorder
c. Somatisation disorder
d. Hypochondriacal disorder
e. Malingering disorder
Hypochondriacal disorder is a persistent preoccupation with the possibility of having serious disease.
Attention is usually focused on one or two organ systems only. There is a persistent refusal to accept
the advice of several different doctors that there is no physical illness underlying the symptoms, and the
belief persists despite repeated investigations revealing no physical explanation.
Revision Notes in Psychiatry - Puri & Hall, 2nd Edition (2004), p423.
Incorrect
Marks for this submission: 0/1.
Question 10
Marks: 0/1
Which ONE of the following is a direct reason for the neuropsychiatric effects seen in HIV infection?
Choose one answer.
a. Cytomegalovirus
b. Toxoplasma
c. Cerebral vasculitis
d. Non-Hodgkin's lymphoma
e. Progressive multifocal leukoencephalopathy
HIV causes neuropsychiatric complications in a number of ways. Direct effects of HIV include
encephalitis, cerebral vasculitis, lymphocytic meningitis. Toxoplasma and cytomegalovirus are
opportunistic infections, and not direct effects. Progressive multifocal leukoencephalopathy is caused
by opportunistic infection by papova virus.
Dube et al, (2005). Journal of Psychiatry and Neuroscience, 30(4): 237-346.
Incorrect
Marks for this submission: 0/1.
Question 11
Marks: 0/1
Which ONE of the following is NOT a feature of chronic fatigue syndrome?
Choose one answer.
a. Fatigue after mild exertion.
b. Duration more than 6 months.
c. Disturbed sleep patterns.
d. Muscular aches and pains.
e. Feeling tired after rest, relaxation and enjoyment.
CFS is characterised by fatigue, post-exertional malaise, sleep dysfunction, pain, neurological/cognitive
manifestations, autonomic manifestations, neuroendocrine manifestations and immune manifestations.
Companion to Psychiatric Studies, Ed: Johnstone, Lawrie et al, Seventh edition, p482.
Incorrect
Marks for this submission: 0/1.
Question 12
Marks: 0/1
Which of the following is the most common psychiatric cause of A&E attendance?
Choose one answer.
a. Panic attacks
b. Alcohol use
c. Personality disorder
d. Psychosis
e. Phobic avoidance
Alcohol misuse is the most common substance misuse disorder. Alcohol problems are encountered
particularly frequently in accident and emergency departments. The problem is greater in males, and as
many as 20% of male medical admissions are problem drinkers (Orford et al, 1992).
Companion to Psychiatric Studies, Ed: Johnstone, Lawrie et al, Seventh edition, p697.
Incorrect
Marks for this submission: 0/1.
Question 13
Marks: 0/1
Which ONE of the following disorders involves the translation of unacceptable drives or troubling
conflicts into bodily motor or sensory symptoms that suggest a neurological or other kind of medical
condition?
Choose one answer.
a. Conversion disorder
b. Hypochondriacal disorder
c. Somatisation disorder
d. Body dysmorphic disorder
e. Adjustment disorder
Conversion disorders are presumed to be psychogenic in origin. They are associated with traumatic
events, insoluble problems or disturbed relationships. The unpleasant affect associated with these
conflicts is transformed (converted) into symptoms. There must be evidence for psychological
causation, and no evidence of physical disorder that may explain the symptoms.
Revision Notes in Psychiatry - Puri & Hall, 2nd Edition (2004) p418.
Incorrect
Marks for this submission: 0/1.

EMIs
1
Marks: 0/3
DIAGNOSIS – NEUROTIC AND STRESS RELATED DISORDERS

A. Acute stress reaction


B. Adjustment disorder.
C. Body Dysmorphophobia
D. Dissociative disorder
E. Factitious disorder
F. Hypochondriasis
G. Malingering
H. Persistent delusion disorder
I. Phobic disorder
J. Somatoform disorder.

For each of the following patients, select the most likely diagnosis
A woman in her twenties presents in the A&E with multiple skin rashes and abscesses. After much
investigation, no underlying cause is found. The nursing staff tell the doctor that this lady had
presented many times to the A&E with many symptoms including chest pain and abdominal pain, for
which all the investigations have come back as normal.
A 26-year-old man tells his GP that he has severe form of cancer of the nose, due which his
appearance has changed considerably. He says that he visited the surgery that day because he knew
that his regular GP was away, and that she had not understood the seriousness of his problems, and
dismissed his claims, with only two sets of investigations.
A 27-year-old man calls 999 with severe & distressing abdominal pain. He is brought to the A&E for
assessment, where the nurses recognise him from many previous admissions with similar complaints
of disabling abdominal pain, which dramatically improves with parenteral painkillers. He starts
shouting at the nursing staff for being so inhuman as to not give him pain relief, and walks away to
have a cigarette.
1. E. In factitious disorder repeated doctor seeking occurs without any underlying gain apart from
medical attention.
2. H. This qualifies as a hypochondriacal delusion of body dysmorphic type.
3. G. This is malingering for obtaining opioids.
Incorrect
Marks for this submission: 0/3.
Question 2
Marks: 0/3
DIAGNOSIS

A. Dissociative amnesia
B. Dissociative fugue
C. Dissociative motor disorder
D. Factitious disorder.
E. Hypochondriasis
F. Malingering
G. Persistent somatoform disorder
H. Post concussion syndrome
I. Post traumatic stress disorder
J. Retrograde amnesia

Choose ONE option that best fits each of the following scenarios
During a battle a soldier suddenly loses the use of his right arm. There are no other neurological signs
and he does not appear distressed by his ailment.
A 45 year old man attends his GP with back pain from a fall which he says occurred at work.
Neurological examination yields inconsistent findings with regards to the nature of the injuries. The
GP later receives a letter from the man‘s solicitor requesting a medical report for a compensation
claim
A man presents to A+E having wandered far away from his home address. He has no idea what his
name is or where he is from. On examination he able to register new information and there is no
impairment of recall.
1. C - The scenario best fits a dissociative (hysterical) motor disorder. The soldier has been subject to
psychological trauma following which he has developed a paralysis that makes him unable to fight. The
symptoms are accompanied by the characteristic ‗belle indifference‘.
2. F - This is most likely to be malingering. Clues to the diagnosis include the inconsistent symptoms
and the motive of getting a financial reward.
3. B- This describes dissociative fugue where the patient wanders far from home, typically with no
recollection of their personal identity
Incorrect
Marks for this submission: 0/3.
Question 3
Marks: 0/3
REACTIONS TO STRESS – DIAGNOSIS

A. Acute stress reaction


B. Adjustment disorder
C. Agoraphobia
D. Depressive episode
E. Generalised anxiety disorder
F. Mixed anxiety & depressive disorder
G. Normal adaptive reaction
H. Panic disorder
I. Post Traumatic stress disorder
J. Social Phobia

For each of the following patients, select the most likely diagnosis.
1. A 40-year-old man was made redundant one week ago after 15 years with the company. He was
found to be depressed, worrying about his future all the time and finding it increasingly difficult to
cope or make plans for the future. On a visit to his bank to explain his inability to pay his mortgage,
even though the staffs at the bank were helpful, he was abusive towards them unprovokedly and had
to be escorted out due to his highly agitated state.
2. A 56-year-old man was made redundant after repeated absences from work. He was considered a
commendable employee till about three months ago, when his secretary committed suicide by
jumping off the office building in front of him. Initially he was numb after the incident, but he was
detached from everybody at the office and home. He complained of not being able to concentrate in
the office, and had repeated flashbacks of the incident, which frightened him and made him lash out at
his partner.
3. A 34-year-old lady was presented to the A&E with low mood, palpitations and tremors. She was
admitted to the ward where her cardiovascular work up was normal. On interview by the liaison
psychiatrist, she told them that the previous day her partner had met with an accident and had suffered
a severe head injury. On detailed assessment it was found that she had a good premorbid adjustment
and was ready for discharge with complete resolution of her symptoms.
1. B – This best fits an adjustment disorder, a psychological reaction arising in relation to adapting to
new circumstances. Depressive features, anxiety and outbursts of emotion can be seen.
2. I – There are symptoms congruent with PTSD in this scenario, including emotional numbing,
flashbacks and irritability.
3. A – This is an acute stress reaction to stress which is used to describe immediate and brief
psychological responses to sudden, intense stressors.
Incorrect
Marks for this submission: 0/3.
Question 4
Marks: 0/3
NEUROTIC AND STRESS RELATED DISORDERS

A. Acute stress reaction


B. Adjustment Disorder
C. Briquets syndrome
D. Dissociative amnesia
E. Dissociative fugue
F. Gansers syndrome
G. Hypochondriasis
H. Possession disorder
I. Post Traumatic Stress Disorder
J. Somatization disorder

For each of the following patients, select the most likely diagnosis.
A 45-year-old housewife presents to her GP with pain in her tummy, arms and legs, and headaches for
over a year, in spite of them repeatedly giving reassurance that there is no obvious cause for them.
She has been previously investigated for the same and no obvious abnormality was found. She though
insists on more investigations and it is now affecting relations within her partner
A 29-year-old man presents to casualty following a brief loss of both a sense of personal identity and
awareness of his surroundings. On interview there is a limited but repeated set of movements and
utterances. His neurological examination in contradiction to his symptoms is unremarkable.
A 39-year-old mother is dazed and appears withdrawn on hearing that her daughter has been involved
in a life threatening accident. Shortly afterwards she becomes overactive and irritable.
1. J – This is somatisation disorder - multiple physical complaints for which no physical cause is found.
2. H – The best fit diagnosis is possession state, where there is a temporary loss of the sense of personal
identity and of full awareness of the person‘s surroundings.
3. A – This is an acute stress reaction. Clues include a clear precipitating stressful event and transient
increased arousal
Incorrect
Marks for this submission: 0/3.
Question 5
Marks: 0/3
DIAGNOSIS – ANXIETY DISORDERS

A. Agoraphobia
B. Body dysmorphic disorder
C. Generalised anxiety disorder
D. Hypochondriacal disorder
E. Obsessive compulsive disorder
F. Panic disorder
G. PTSD
H. Social phobia
I. Somatisation disorder
J. Specific phobia

For each of the following patients, select the most likely diagnosis.
1. A 23-year-old man admitted to the A&E after an accident was found to be anxious, fearful,
sweating profusely and pacing in the A&E. Even though his injuries were minor, while they were
waiting for an X-Ray and some blood tests, he self-discharged from the A&E. He had never been to
his GP in a long time, and had fainted when he had gone to get his vaccination prior to his holiday.
2. A man has come to the psychiatric outpatients‘ clinic complaining of episodes of tremor,
palpitations, dizziness and sweating every time he goes to the shops or gets on the bus. He has to take
his wife with him to do simple errands, as he wants to make sure that someone is around to help if he
collapses.
3. A woman in the outpatients‘ clinic, who appears extremely distressed. She says that she has been
having intrusive thoughts, which she tries to resist, that she will contaminate her children with germs.
In order to prevent this from happening she has to clean their rooms in a set pattern every morning
and does not allow anyone else into the house.
1. J – This is a type of simple phobia, where there is an irrational fear of an object/situation,
anticipatory anxiety and avoidance.
2. A - This is agoraphobia – a fear of being away from home, in crowds or in situations that they cannot
leave easily.
3. E – This describes the obsessive thoughts and compulsive rituals seen in OCD.
Incorrect
Marks for this submission: 0/3.
Question 6
Marks: 0/3
ANXIETY DISORDERS – DIAGNOSIS

A. Agoraphobia
B. Depressive episode
C. Generalised anxiety disorder
D. Mixed anxiety & depressive disorder
E. Obsessive compulsive disorder
F. Panic disorder
G. Post Traumatic stress disorder
H. Social Phobia
I. Somatisation disorder
J. Specific phobia

For each of the following patients, select the most likely diagnosis.
A 26-year-old lady lost her mother in a car accident. Nine months later she developed dizziness,
palpitations, nervousness and stomach upset for which no physical cause was found. She was on the
edge always and found it difficult to concentrate due to her worries about her future. Her nervousness
was worse when she travelled by car.
A 40-year-old woman, who was referred by her GP, was discharged, as she had not attended her two
new appointments. Due to her being referred a third time, a CPN went to see her, when she explained
her fears of going outside the house. She usually worries about any appointment for days prior to the
appointment and by the time of the appointment she was too worried about collapsing on the road
without anyone being there to help her, and hence doesn‘t keep appointments.
A 29-year-old student has been feeling very low due to inability to find a partner. He was constantly
worried about being alone all his life, and had decreased appetite and lost a lot of weight recently. He
found it difficult to interact with members of the opposite sex, as he was worried about having
hiccups in front of them, which might lead to them to think lowly of him.
1. C – This describes psychological and somatic symptoms of generalised (‗free-floating‘) anxiety.
Note that while this appears to have been precipitated by a car accident, she does not fulfil the criteria
for PTSD.
2. A – This is classic agoraphobia, with fear of leaving the house along with anticipatory anxiety and
avoidance.
3. H – This describes social phobia, where the core fear is embarrassing oneself in public
Incorrect
Marks for this submission: 0/3.

General Adult Psychiatry III

Perinatal Psychiatry
1
Marks: 0/1
A woman comes to see you in the out-patient clinic. She has a 6 month old son and, for the last three months,
has had recurring thoughts of harming him. She does not wish to harm him and these thoughts make her
tearful and anxious. The birth was uneventful but she perceives it to have been traumatic. What is the most
likely diagnosis?
Choose one answer.
a. PTSD
b. Baby Blues
c. Postnatal depression
d. Puerperal Psychosis
e. OCD
Although symptoms of depression are the same, the features of anxiety are more common in postnatal
depression, and many mothers have fleeting, repetitive and intrusive thoughts about harming the baby. These
thoughts are distressing but it is rare for mothers to act on such thoughts.
Oxford textbook of Psychiatry, 4th edition, 2001.
Incorrect
Marks for this submission: 0/1.
Question 2
Marks: 0/1
Which ONE is TRUE of the Edinburgh Postnatal Depression Scale?
Choose one answer.
a. It is a semi-structured interview conducted by a trainer clinician.
b. The scale can be used to detect anxiety neuroses, phobias and personality disorder.
c. It has been shown to be effective as a screening tool.
d. It is a simple instrument to use with women who cannot read or write.
e. It is used in secondary mental health services to diagnosis postnatal depression.
The Edinburgh Postnatal Depression Scale (EPDS) has been developed to assist primary care health
professionals to detect mothers suffering from postnatal depression. The EPDS, which has been widely tested,
has been utilized in 23 countries and carries a significant level of sensitivity (86%) and specificity (78%) in
identifying those at risk of or potentially suffering from either prenatal or postpartum depression. The EPDS is a
ten-item self-rating scale, requiring about five minutes to complete. In doubtful cases, it can be re-
administered after two weeks. The English version is available for download from this Web site; it is also
available in many languages, and has cross-cultural validity. The EPDS is more difficult to use with non-literate
women due to its format, and takes longer to administer. The scale will not detect mothers with anxiety
neuroses, phobias or personality disorder.
Cox J, Holden L (1994). Perinatal Psychiatry: Use and Misuse of the Edinburgh Postnatal Depression Scale.
London: Gaskell.
Incorrect
Marks for this submission: 0/1.
Question 3
Marks: 0/1
What is the relative risk to the baby of developing Ebstein's anomaly if mother is on lithium in the first
trimester of pregnancy?
Choose one answer.
a. 10
b. 4.5
c. 5.5
d. 2.5
e. 1.5
The relative risk for congenital defects overall is 7.5, and 10 for Ebstein's anomaly after lithium exposure.
NICE guidelines on Antenatal & Postnatal Mental Health (2007).
Incorrect
Marks for this submission: 0/1.
Question 4
Marks: 0/1
What is the treatment of choice in a patient with severe symptoms of premenstrual syndrome?
Choose one answer.
a. Fluoxetine
b. Citalopram
c. Sertraline
d. Venlafaxine
e. Paroxetine
Research shows SSRIs are very effective in relieving the emotional and physical PMS and PMDD symptoms for
most women. SSRI therapy usually brings relief within a few days of starting medication.
Kornstein, SG (2001). Selective serotonin-reuptake inhibitors reduce the symptoms of severe premenstrual
syndrome. Evidence-based Obstetrics & Gynecology, Vol. 3(1): 32-33.
Incorrect
Marks for this submission: 0/1.
Question 5
Marks: 0/1
Which of the following antidepressants is expressed in breast milk at the highest concentration?
Choose one answer.
a. Amitriptyline
b. Fluoxetine
c. Sertraline
d. Nortryptyline
e. Paroxetine
The prescription of psychotropic drugs during lactation is a clinically important but complex issue. Most of the
information available on the excretion of these drugs into breast milk and the impact that this has on the
breast-fed infant is based on single case reports. Although all psychotropic drugs that have been studied are
excreted into breast milk, there is limited knowledge on the practical impact of the, often very low,
concentrations found. In mothers receiving tricyclic antidepressants, it seems unwarranted to recommend that
breast feeding should be discontinued. The exception to this rule is in mothers receiving doxepin. Of the SSRIs,
fluoxetine and citalopram should probably be avoided during lactation as they are found at relatively higher
concentrations in breast milk. Treatment with other SSRIs (fluvoxamine, paroxetine or sertraline) seems to be
compatible with breast feeding, although this view should be considered as preliminary due to the lack of data.
Hale (2008) Medications in Mother's Milk - 13th Ed.
American Academy of Paediatrics Policy Statement (2001)
Incorrect
Marks for this submission: 0/1.
Question 6
Marks: 0/1
What is the risk of cardiac abnormality in a child born to a mother who was on Lithium in the first trimester of
her pregnancy?
Choose one answer.
a. 1/10
b. 1/5
c. 1/100
d. 1/1000
e. 1/10000
The risk of cardiac abnormality in a pregnant woman on Lithium in the first trimester is 60/1000 (compared to
8/1000 in the general population). The risk of Ebsteins anomaly is increased from 1 in 20,000 in the general
population to 10 in 20,000 in women taking Lithium.
NICE guidelines on Antenatal & Postnatal Mental Health (2007).
Incorrect
Marks for this submission: 0/1.
Question 7
Marks: 0/1
You have started a patient with postnatal depression on an antidepressant and wish to monitor changes in her
symptoms. Which of the following observer rating scales would you use?
Choose one answer.
a. Edinburgh Depression Scale
b. Zung
c. BDI
d. Morgan-Russel
e. MADRS
The Montgomery-Asberg Depression Rating Scale (MADRS) is an observer rated depression scale which is
designed to assess and measure the severity of symptoms of depression, including depression in patients with
comorbid medical conditions, and change in depressive state during treatment.
BDI is a self rated depression scale which is mainly used to screen patients for prescience of depression.
The Edinburgh postnatal depression rating scale is a 10 item self report scale to screen for postnatal depression
in the community and is less sensitive to monitor response to treatment.
The Morgan Russel scale is a widely used measure of outcome for anorexia nervosa consisting of two scores: an
average outcome score and a general outcome score (possible total of 12). The average outcome score is based
on the outcome in five areas: nutritional status, menstrual function, mental state, sexual adjustment, and
socioeconomic status.
The Zung Self-Rating Depression Scale is a 20-item self-report questionnaire that is widely used as a screening
tool, covering affective, psychological and somatic symptoms associated with depression. The questionnaire
takes about 10 minutes to complete, and items are framed in terms of positive and negative statements.
Montgomery SA, Asberg M (1979). A new depression scale designed to be sensitive to change. BJPsych
134:382-389.
Also see Click here for reference for more information
Incorrect
Marks for this submission: 0/1.
Question 8
Marks: 0/1
A patient with bipolar affective disorder came off her medication during her pregnancy. She delivered four
weeks ago and is currently breast feeding her baby. She feels that she may be relapsing and requests
medication which is safe for breast feeding. Which ONE of the following would you prescribe?
Choose one answer.
a. Sodium valproate
b. Lamotrigine
c. Lorazepam
d. Carbemazepine
e. Risperidone
NICE Guidelines on Antenatal & Postnatal Mental health advises use of antipsychotics if medication is required
during breast feeding.
NICE guidelines on Antenatal & Postnatal Mental Health (2007).
Incorrect
Marks for this submission: 0/1.
Question 9
Marks: 0/1
Which ONE of the following disorders is more common in women than men?
Choose one answer.
a. Schizophrenia
b. Completed suicide
c. Bipolar disorder
d. Alcohol dependence
e. Dysthymia
Psychiatric disorders that are more common in women than in men include : dementia of Alzheimer's type,
dysthymia, anxiety and phobic disorders, dissociative disorders, eating disorders, deliberate self-harm. Suicide
is more common in males. Alcohol dependence is around eight times more common in males than females.
Puri, Laking and Treasaden (1996). Textbook of Psychiatry. London: Churchill Livingtone, p232.
Incorrect
Marks for this submission: 0/1.
Question 10
Marks: 0/1
Which rating scale would you recommend to screen a health visitor for depression in women after delivery:
Choose one answer.
a. MADRS
b. Morgan-Russel
c. Edinburgh Depression Scale
d. Zung
e. BDI
The Edinburgh postnatal depression rating scale is a 10 item self report scale to screen for postnatal depression
in the community. The Montgomery-Asberg Depression Rating Scale (MADRS) is an observer rated depression
scale which is designed to assess and measure the severity of symptoms of depression, including depression in
patients with comorbid medical conditions, and change in depressive state during treatment. BDI is a self rated
depression scale which is mainly used to screen patients for presence of depression. The Morgan Russel scale is
a widely used measure of outcome for anorexia nervosa consisting of two scores: an average outcome score
and a general outcome score (possible total of 12). The average outcome score is based on the outcome in five
areas: nutritional status, menstrual function, mental state, sexual adjustment, and socioeconomic status. The
Zung Self-Rating Depression Scale is a 20-item self-report questionnaire that is widely used as a screening tool,
covering affective, psychological and somatic symptoms associated with depression. The questionnaire takes
about 10 minutes to complete, and items are framed in terms of positive and negative statements.
Cox JL, Holden JM, Sagovsky R (1987). Detection of postnatal depression. Development of the 10-item
Edinburgh Postnatal Depression Scale. BJPsych, 150:782-786.
Incorrect
Marks for this submission: 0/1.
Question 11
Marks: 0/1
Which of the following features are more frequent in the children of mothers suffering from postnatal
depression?
Choose one answer.
a. Affective sharing
b. Positive facial expressions
c. Secure attachment
d. Initial sociability
e. Behavioural problems
Children of mothers suffering postnatal depression have been shown to demonstrate behavioural problems,
frequent insecure attachment, difficulties in expressive language, fewer positive facial expressions, more
negative facial expressions, mild cognitive abnormalities, less affective sharing and less initial sociability.
Poobalan et al (2007). BJPsych 191:378-386
Incorrect
Marks for this submission: 0/1.
Question 12
Marks: 0/1
What is the incidence of fetal birth defects in a woman taking valproate?
Choose one answer.
a. 1/15
b. 5/100
c. 1/100
d. 1/10
e. 1/1000
The risk of fetal birth defects (particularly neural tube defects) in a woman taking sodium valproate is 100-
200/10,000 (1-2/100).
NICE guidelines on Antenatal & Postnatal Mental Health 2007.
Incorrect
Marks for this submission: 0/1.

Personality Disorder
1
Marks: 0/1
A young man is impulsive and has angry outbursts. He has been arrested several times for hitting his partner.
Which is the most likely diagnosis?
Choose one answer.
a. Paranoid PD
b. Narcissistic PD
c. Dissocial PD
d. Emotionally Unstable PD
e. Anakastic PD
ASPD is characterised by any 3 of the following:
a) callous unconcern for the feelings of others
b) gross and persistent attitude of irresponsibility and disregard for social norms, rules and obligations
c) incapacity to maintain enduring relationships, though having no difficulty in establishing them
d) very low tolerance to frustration and a low threshold for discharge of aggression, including violence
e) incapacity to experience guilt and to profit from experience, particularly punishment
f) marked proneness to blame others, or to offer plausible rationalizations, for the behaviour that has brought
the patient into conflict with society.
Oxford textbook of Psychiatry, 4th edition, 2001
Incorrect
Marks for this submission: 0/1.
Question 2
Marks: 0/1
Which of the following is a feature of dissocial personality disorder?
Choose one answer.
a. Persistent irresponsibility
b. Lack of desire for confiding relationships
c. Blames self excessively
d. Explosive in nature
e. Difficulty establishing relationships
The ICD-10 criteria for dissocial personality disorder are:
a) callous unconcern for feelings of others
b) gross and persistent irresponsibility and disregard for social norms, rules and obligations
c) incapacity to maintain enduring relationships
d) low tolerance to frustration, low threshold for aggression and violence
e) incapacity to experience guilt or to profit from experience
f) blames others
World Health Organization. International Classification of Mental and Behavioural Disorders. Diagnostic Criteria
for Research (10th edn). Geneva: World Health Organization, 1993.
Incorrect
Marks for this submission: 0/1.
Question 3
Marks: 0/1
Which of the following did Bateman and Fonagy propose to be effective for borderline personality disorder?
Choose one answer.
a. Dialectical Behaviour Therapy
b. Desensitisation
c. Partial Hospitalisation
d. Cognitive Behaviour Therapy
e. EMDR
Partial hospitalisation has been proposed by Bateman and Fonagy (1999) as being more effective for borderline
personality disorder than standard psychiatric care.
Bateman, A. & Fonagy, P. (1999) The effectiveness of partial hospitalisation in the treatment of borderline
personality disorder: a randomised controlled trial. American Journal of Psychiatry, 156:1563-1569.
Incorrect
Marks for this submission: 0/1.
Question 4
Marks: 0/1
Which of the following is a criteria for the ICD-10 diagnosis of anxious (avoidant) personality disorder?
Choose one answer.
a. Excessive sensitiveness to setbacks and rebuffs.
b. Feelings of excessive doubt and caution.
c. Preoccupation with being criticised or rejected in social situations.
d. Apparent indifference to praise or criticism.
e. Bears grudges persistently.
The ICD-10 criteria for anxious (avoidant) personality disorder are:
a) persistent, pervasive tension and apprehension
b) believe they are socially inept, unappealing or inferior to others
c) preoccupation with being criticised or rejected in social situations
d) unwillingness to become involved unless certain of being liked
e) restrictions in lifestyle because of need for security
f) avoidance of activities involving interpersonal contact because of fear of criticism, disapproval or rejection
World Health Organization. International Classification of Mental and Behavioural Disorders. Diagnostic Criteria
for Research (10th edn). Geneva: World Health Organization, 1993.
Incorrect
Marks for this submission: 0/1.
Question 5
Marks: 0/1
Anxious (avoidant) and dependent personality disorders are often comorbid with social phobia. Which of the
following classes of medication is the most effective treatment for social phobia in this context?
Choose one answer.
a. Mood stabilisers
b. Serotonin-noradrenaline reuptake inhibitors (SNRIs)
c. Atypical antipsychotics
d. Monoamine oxidase inhibitors (MAOIs)
e. Tricyclics (TCAs)
The under-recognition of social phobia remains an issue of concern. There is frequent comorbidity with other
psychiatric disorders, including other anxiety disorders, depression, alcohol abuse, and personality disorders.
Current conceptualizations of the aetiology of social phobia reflect psychodynamic theories and evidence from
family and genetic studies, neurobiological research, and neuroimaging. Drugs such as monoamine oxidase
inhibitors, selective serotonin-reuptake inhibitors, benzodiazepines, and beta-adrenergic blockers have proven
to be efficacious, as has cognitive-behavioural treatment, including group approaches. Comorbid social phobia
of avoidant and dependent personality disorder responds best to MAOIs.
Fones et al (1998). Social Phobia: An update. Harvard Review of Psychiatry 5(5):247-259.
Hori (1998). Pharmacotherapy for personality disorders. Psychiatry & Clinical Neurosciences, 52(1):13-19.
Incorrect
Marks for this submission: 0/1.
Question 6
Marks: 0/1
Which of the following personality disorders would be the most appropriate diagnosis in the following case
vignette?
A 56-year old single high-school graduate, living alone, who rarely socialises. He makes his living by fixing things
and is adept at this, but chooses not to open a shop. He seems indifferent to praise, advice or complaints from
customers. He appears aloof and barely acknowledges the greetings of others.
Choose one answer.
a. Histrionic
b. Schizotypal
c. Schizoid
d. Paranoid
e. Dissocial
The ICD-10 criteria for schizoid personality disorder are:
a) finds few activities pleasurable
b) emotional coldness, detachment or flattened affect
c) limited capacity to express feelings
d) apparent indifference to praise or criticism
e) little interest in sexual experiences with another person
f) preference for solitary activities
g) preoccupation with fantasy and introspection
h) lack of desire for close friends or confiding relationships
i) insensitivity to social norms and conventions
World Health Organization. International Classification of Mental and Behavioural Disorders. Diagnostic Criteria
for Research (10th edn). Geneva: World Health Organization, 1993.
Incorrect
Marks for this submission: 0/1.
Question 7
Marks: 0/1
Which of the following is true for patients with emotionally unstable personality disorder?
Choose one answer.
a. They are still likely to retain the diagnosis at age 50.
b. They are less likely to have children.
c. They have a normal rate of marriage.
d. They are unlikely to be employed.
e. They have a similar rate of substance misuse.
Long-term follow up of emotionally unstable PD has shown that marriage rates are half the average, odds of
having children one-quarter, severity of symptoms decreases with age, and only one-quarter fulfil diagnostic
criteria at age 50. Employment is fairly common.
Stone (1993). Long-term outcome in personality disorders. BJ Psych, 162: 299-313.
Incorrect
Marks for this submission: 0/1.
Question 8
Marks: 0/1
Which of the following medications has been shown to be effective in the management of aggressive
behaviour in patients with personality disorder?
Choose one answer.
a. Haloperidol
b. Lithium
c. Dothiepin
d. Risperidone
e. Carbamazepine
Pharmacological treatments show some promise in the treatment of individual symptoms rather than discrete
personality disorder categories. In particular, lithium, valproate and SSRIs have been found to be helpful in the
treatment of aggressive and assaultative behaviour.
Moleman et al. Psychopharmacological treatment of personality disorders: A review. Treatment of personality
disorders, 1999, p.207-227. Publisher: Kluwer Academic Publishers, Dordrecht, Netherlands.
Incorrect
Marks for this submission: 0/1.
Question 9
Marks: 0/1
What are people with emotionally unstable borderline personality disorder are stuck in?
Choose one answer.
a. Klein's paranoid-schizoid position
b. Klein's depressed position
c. Oedipal phase
d. Winnicott's 'false self'
e. Klein's paranoid position
The paranoid schizoid position, for Klein, is characterized by splitting, projective identification, idealization, and
omnipotence. Klein argues that the young infant possesses a rudimentary ego which "largely lacks cohesion"
and "has a tendency towards integration alternating with a tendency towards disintegration, a falling into bits."
Klein, 1946/1994, p. 140.
Click here for reference
Incorrect
Marks for this submission: 0/1.
Question 10
Marks: 0/1
Which of the following personality disorders would be the most appropriate diagnosis in the following case
vignette?
A middle-aged successful businessman who requests his doctor for therapy because "others have trouble
getting on with me". He has noticed this for many years but felt no need for change until the break-up of his
third marriage. Since childhood, he was obsessed with money and power, which he has achieved quite easily
because, in his words, "everyone else is weak or incompetent". He has extraordinary confidence in his own
ability to succeed, but not in a psychotic or manic way.
Choose one answer.
a. Schizotypal
b. Dissocial
c. Schizoid
d. Histrionic
e. Narcissistic
DSM-IV criteria for narcissistic personality disorder are:
a) grandiose sense of self-importance
b) fantasies of unlimited power, success, brilliance, beauty or ideal love
c) believes they are 'special'
d) requires excessive admiration
e) sense of entitlement
f) interpersonally exploitative
g) lacks empathy
h) often envious of others
i) arrogant, haughty behaviours or attitudes.
American Psychiatric Association. Diagnostic and Statistical Manual of Mental Disorders (DSM), Fourth Edition
(1994).
Incorrect
Marks for this submission: 0/1.
Question 11
Marks: 0/1
Which of the following personality disorders would be the most appropriate diagnosis for someone who allows
others to make important decisions for them, feels uncomfortable when alone and fears being abandoned?
Choose one answer.
a. Histrionic
b. Anxious (avoidant)
c. Dependent
d. Emotionally unstable
e. Anankastic
The ICD-10 criteria for dependent personality disorder are:
a) allow others to make important life decisions
b) subordination of own needs to those of others on whom dependent
c) unwillingness to make demands on people on whom dependent
d) uncomfortable or helpless when alone, fear inability to care for themselves
e) fear of being abandoned
f) unable to make decisions without excessive advice from others.
World Health Organization. International Classification of Mental and Behavioural Disorders. Diagnostic Criteria
for Research (10th edn). Geneva: World Health Organization, 1993.
Incorrect
Marks for this submission: 0/1.
Question 12
Marks: 0/1
One of the predictors of negative response to treatment in antisocial personality disorder is:
Choose one answer.
a. Axis 1 diagnosis of organic brain disorder.
b. Axis 1 diagnosis of psychosis.
c. Presence of anxiety.
d. Axis 1 diagnosis of depression.
e. Axis 1 diagnosis of bipolar affective disorder.
Predictors of negative response to treatment in antisocial personality are:
a) history of arrest for a felony
b) history of repeated lying, use of aliases, conning
c) unresolved legal situation on admission
d) history of conviction for a felony
e) hospitalisation as an alternative to imprisonment
f) history of violence to others
g) axis 1 diagnosis of organic brain impairment.
Fagin (2004). Management of personality disorders in acute in-patient settings. Part 2: Less-common
personality disorders. Advances in Psychiatric Treatment, 10:100-106
Incorrect
Marks for this submission: 0/1.
Question 13
Marks: 0/1
Which of the following personality disorders would be the most appropriate diagnosis in the following case
vignette?
A 25-year old female junior doctor has been reprimanded at work for wearing inappropriate clothing to work.
Her work colleagues mention that she always seeks to be the centre of attention and is overly flirtatious. She is
occasionally quite dramatic in displaying emotion.
Choose one answer.
a. Emotionally unstable
b. Narcissistic
c. Histrionic
d. Anankastic
e. Anxious (avoidant)
The ICD-10 criteria for histrionic personality disorder are:
a) self-dramatization, theatricality, exaggerated expression of emotions
b) suggestibility
c) shallow and labile affect
d) seeks excitement, centre of attention
e) inappropriate seductiveness
f) over-concern with physical attractiveness.
World Health Organization. International Classification of Mental and Behavioural Disorders. Diagnostic Criteria
for Research (10th edn). Geneva: World Health Organization, 1993.
Incorrect
Marks for this submission: 0/1.
Question 14
Marks: 0/1
In dissocial personality disorder, which of the following is a predictor of good response to therapy?
Choose one answer.
a. Presence of anxiety.
b. Unresolved legal situation.
c. History of arrest.
d. Use of aliases.
e. Organic brain impairment.
Predictors of good response to therapy in dissocial PD are: presence of anxiety, diagnosis of depression,
diagnosis of psychosis (other than organic conditions).

Fagin (2004). Management of personality disorders in acute in-patient settings. Part 2: Less-common
personality disorders. Advances in Psychiatric Treatment, 10:100-106.
Incorrect
Marks for this submission: 0/1.
Question 15
Marks: 0/1
A man comes to your out-patient clinic. He gives you a history of childhood conduct disorder. He recognises
that he has a problem with anger and often gets into fights with his partner. He wishes to prevent this from
happening. You interview him and find no evidence of mental illness or active symptoms. What do you decide?
Choose one answer.
a. To start no treatment but review him in three months.
b. Start an antipsychotic and review in 3 months.
c. Behavioural strategies and anger management.
d. Start an antidepressant and review in 3 months.
e. To discharge him.
Approaches that reinforce appropriate behaviours and attempt to make connections between the person's
actions and their feelings may be more beneficial.
Oxford textbook of Psychiatry, 4th edition, 2001
Incorrect
Marks for this submission: 0/1.
Question 16
Marks: 0/1
Which of the following is a diagnostic criterion for borderline personality disorder?
Choose one answer.
a. Self-harm
b. Obsessions
c. Depression
d. Attempts to avoid real or imagined abandonment
e. Suicide
According to ICD 10 criteria, self-harm would be associated with impulsive rather than a borderline type
personality.

Oxford textbook of Psychiatry, 4th edition, 2001.


Incorrect
Marks for this submission: 0/1.
Question 17
Marks: 0/1
Which of the following psychological therapies for personality disorder is an example of 'milieu therapy', in
which the social setting itself produces a therapeutic effect?
Choose one answer.
a. Rational emotive therapy
b. Cognitive Analytic Therapy
c. Therapeutic community
d. Dialectical behaviour therapy
e. Partial hospitalisation
Therapeutic communities, such as the Henderson or Cassel hospitals, are a specific example of milieu therapy.
In therapeutic communities, patients live and work together, with meetings between staff and patients to
discuss things such as their own and other patient's behaviour. The group takes a decision as a whole as to
whether certain behaviours are acceptable and what sanctions should be applied.
Dolan et al (1997). Change in borderline symptoms one year after therapeutic community treatment for severe
personality disorder. BJPsych, 171:274-279.
Incorrect
Marks for this submission: 0/1.
Question 18
Marks: 0/1
Which of the following is a characteristic finding in patients with personality disorder?
Choose one answer.
a. Less severe symptoms of Axis I psychiatric disorders.
b. Lower rates of Axis I psychiatric disorders.
c. Longer and costlier treatment for Axis I psychiatric disorders.
d. Lower mortality from suicide, accidents and violence.
e. Better prognosis for Axis I psychiatric disorders.
Common characteristics of patients with personality disorder include high rates of Axis I psychiatric disorders,
more severe Axis I symptoms, worse prognosis of Axis I disorders, longer and costlier treatments for Axis I
disorders, excess mortality from suicide, accidents and violence.
Tyrer et al (1990). Psychological Medicine 20:423-431.
Reich & Green (1991). Effect of Personality Disorders on Outcome of Treatment. Journal of Mental and Nervous
Diseases, 179:74-82.
Kent et al (1995). Heavy utilization of inpatient and outpatient services in a public mental health service.
Psychiatric Services, 46:1254-1257.
Incorrect
Marks for this submission: 0/1.
Question 19
Marks: 0/1
In antisocial personality disorder, which of the following is a predictor of positive response to treatment?
Choose one answer.
a. Presence of anxiety.
b. History of arrest for a felony.
c. History of violence.
d. Hospitalisation.
e. Organic brain impairment.
Predictors of positive response to treatment in antisocial personality are the presence of anxiety, Axis 1
diagnosis of depression or Axis 1 diagnosis of psychosis.

Fagin (2004). Management of personality disorders in acute in-patient settings. Part 2: Less-common
personality disorders. Advances in Psychiatric Treatment, 10:100-106.
Incorrect
Marks for this submission: 0/1.

Gender Identity Disorder & PMS

1
Marks: 0/1
Which of the following is the most likely diagnosis for a patient who describes persistent and intense distress
about their assigned sex, together with an insistence that they are of the opposite sex?
Choose one answer.
a. Transsexualism
b. Dual-role transsexualism
c. Sexual maturation disorder
d. Egodystonic sexual orientation
e. Gender Identity Disorder of childhood
Gender Identity Disorder is a persistent, intense distress about assigned sex, together with desire to be (or
insistence that one is) of the other sex, usually manifest during early childhood, and always before puberty. It is
relatively uncommon. There is a profound disturbance of the sense of maleness of femaleness.

World Health Organization. International Classification of Mental and Behavioural Disorders. Diagnostic Criteria
for Research (10th edn). Geneva: World Health Organization, 1993.
Incorrect
Marks for this submission: 0/1.
Question 2
Marks: 0/1
Which of the following has the best evidence of use in pre-menstrual dysphoria?
Choose one answer.
a. Progesterone
b. Vitamin B6
c. SSRIs
d. Evening primrose oil
e. Light therapy
Research shows SSRIs are very effective in relieving the emotional and physical PMS and PMDD symptoms for
most women. SSRI therapy usually brings relief within a few days of starting medication Selective serotonin-
reuptake inhibitors reduce the symptoms of severe premenstrual syndrome.

Kornstein (2002). Selective serotonin-reuptake inhibitors reduce the symptoms of severe premenstrual
syndrome. Evidence-based Obstetrics & Gynecology, Vol. 3(1):32 - 33.
Incorrect
Marks for this submission: 0/1.
Question 3
Marks: 0/1
Which ONE of the following is known to be an effective intervention for premenstrual syndrome?
Choose one answer.
a. Evening Primrose Oil
b. Sertraline
c. Cognitive analytic therapy
d. Nefazodone
e. Progesterone
The efficacy of SSRIs in the treatment of both physical and psychological symptoms of premenstrual syndrome
(Dimmock et al, 2000) has been confirmed and studies have shown that these drugs also reduce functional
impairment. The onset of improvement is more rapid than that in SSRI treatment of depressive disorder.
Treatment can be given throughout the menstrual cycle, but several studies now support dosing during the
luteal phase alone, as this appears to be equally efficacious (Freeman et al, 2004).

Henshaw (2007). PMS: diagnosis, aetiology, assessment and management. Adv Psych Treat, 13: 139-146.
Incorrect
Marks for this submission: 0/1.
Question 4
Marks: 0/1
Which ONE of the following is true for the functional impairment in premenstrual syndrome?
Choose one answer.
a. It is limited to interpersonal relationships.
b. It causes 20-30% of sufferers to miss work.
c. It is less severe than that found in dysthymia.
d. It is greater at home than in work settings.
e. It is not improved by treatment with SSRIs.
Pearlstein et al (2000) found that women with premenstrual dysphoric disorder scored worse than women with
dysthymia on the parental factor of the Social Adjustment Self Report Scale, a finding consistent with data from
a large telephone survey in which symptomatic women reported highest functional impairment at home (Hylan
et al, 1999). However, the impairment in occupational function is also significant. In Hylan et al's study, 8-16%
of symptomatic women had missed work in the preceding year because of premenstrual syndrome and 5-8% of
those who had ever missed work had been absent for more than 14 days in the past year. Even symptomatic
women who do not seek treatment for premenstrual syndrome appear to use more healthcare services in
general and missed work more often than asymptomatic women.

Henshaw (2007). PMS: diagnosis, aetiology, assessment and management. Adv Psych Treat, 13:139-146.
Incorrect
Marks for this submission: 0/1.
Question 5
Marks: 0/1
Select the TRUE statement regarding dual-role transvestism.
Choose one answer.
a. Sexual excitement accompanies cross-dressing.
b. There is persistent intense distress about assigned sex.
c. There is a strong desire to join the opposite sex.
d. Cross-dressing enables individuals to enjoy temporary membership of the opposite sex.
e. Individuals seek surgical reassignment to the opposite gender.
Dual-role transvestism includes the wearing of clothes of the opposite sex for part of the time to enjoy a
temporary experience of membership of the opposite sex, without the desire for a more permanent sex
change. No sexual excitement accompanies this cross-dressing, distinguishing it from fetishistic transvestism.

World Health Organization. International Classification of Mental and Behavioural Disorders. Diagnostic Criteria
for Research (10th edn). Geneva: World Health Organization, 1993.
Incorrect
Marks for this submission: 0/1.

EMIs

1
Marks: 0/3
DIAGNOSIS - SUBSTANCE MISUSE

A. Acute alcohol intoxication


B. Alcoholic hallucinosis
C. Delirium Tremens
D. HIV
E. Intracranial bleed
F. Korsakoff's psychosis
G. LSD intoxication
H. Opiate intoxication
I. Opiate withdrawal
J. Wernicke's encephalopathy

A 30-year-old man, dependent on alcohol, comes to the emergency department. He has a wound in his
head. He is homeless and becomes agitated while in the department.

Which ONE is the most likely condition he is suffering from if:


He experiences seeing squirrels crawling over the floor
He has 2 abscesses in groin and neck, he has small pupils and is bradycardic.
He doesn‘t recognise the left side of his body.
1. C – The visual hallucinations are most suggestive of delirium tremens.
2. H – These features are suggestive of opiate overdose intoxication.
3. E – This hemi-neglect is most likely to have been caused by an intracranial bleed.
Incorrect
Marks for this submission: 0/3.
Question 2
Marks: 0/3
PHARMACOLOGY OF PREGNANCY AND THE PUERPERIUM

A. Fluoxetine
B. Haloperidol
C. Lofepramine
D. Moclobemide
E. Olanzapine
F. Paroxetine
G. Sulpiride
H. Trifluoperazine
I. Zolpidem
J. Zopiclone

For each of the following scenarios, choose ONE of the medications above that is the most appropriate
treatment. Assume that the benefits of pharmacology outweigh the disadvantages.
A 23-year-old woman in the first trimester of pregnancy develops agitation, paranoid ideation and
third person auditory hallucinations. Her appetite, energy and sleep are all normal.
A 31-year-old woman in the second trimester of pregnancy develops low mood, hopelessness and
suicidal ideation. Her appetite, energy and sleep are all reduced.
A 31-year-old woman who is breast-feeding her one-month-old infant develops low mood,
hopelessness and suicidal ideation. Her appetite, energy and sleep are all reduced.
A 27-year-old woman who is breast-feeding her two-month-old infant develops severe sleep
disturbance but has no other symptoms of psychiatric illness.
1. H. Trifluoperazine and chlorpromazine are the agents whose use has been most reprted in pregnancy.
Both olanzapine and clozapine appear to be relatively safe, but experience with their use is more
limited.
2. A. While most experience is with tricyclics such as nortriptyline, amitriptyline and imipramine, there
is a reasonable amount of evidence suggesting that fluoxetine is relatively safe.
3. F. While both paroxetine and sertraline are excreted into breast milk, infant levels of these drugs
have been shown to be very low, and so they are probably safe to give to the breast-feeding mother.
Infant serum levels of fluoxetine tend to be a little higher in this situation.
4. I. Zolpidem is to be preferred over zopiclone as there is more evidence to suggest that excreted
amounts are minimal with no observed effects on nursed infants.
Incorrect
Marks for this submission: 0/3.
Question 3
Marks: 0/3
SUBSTANCE MISUSE – MANAGEMENT

A. Buprenorphine
B. Benzodiazepines
C. Bupropion
D. Diamorphine
E. Loperamide
F. Lofexidine
G. Methadone
H. Naloxone
I. Naltrexone
J. NSAID

For each of the following patients, select the most likely treatment.
A 32-year-old opiate dependent woman in the second trimester of her pregnancy, wants to stop using
heroin, but is scared of withdrawal symptoms.
A 45 year old man has recently finished detoxification from opiods. He is very motivated to stop
using any heroin, and seeks treatment to help him stop opiate use, but is not keen on any substitution
treatment
A 22 year old man who was recently started on outpatient methadone treatment is found unconscious.
He is tachycardic and has pin point pupils on examination.
1. G. Methadone is the only medication currently licensed for use in opioid detox or substitution
treatment in pregnancy.
2. I. Naltrexone which is an opioid antagonist has been used to prevent relapses in people who have
completed detox, and don‘t want to be on substitution treatment.
3. H. Naloxone reverses the effects of heroin.
Incorrect
Marks for this submission: 0/3.
Question 4
Marks: 0/3
SIDE EFFECTS – METABOLIC ABNORMALITY

A. Hypercalcemia
B. Hyperglycaemia
C. Hyperkalemia
D. Hypernatremia
E. Hyperthyroidism.
F. Hypocalcemia.
G. Hypoglycaemia
H. Hypokalemia
I. Hyponatremia
J. Hypothyroidism

Choose ONE metabolic abnormality associated with each of the following.


A 78 year old gentleman, diagnosed to have depression was started on Sertraline. Few days later he
was found to have muscle weakness and drowsiness.
A 45 year old man with history of alcohol dependence went on a drinking spree on a Friday night. He
was found to be anxious, sweating profusely and confused by the next morning.
A 40 year old man, with a diagnosis of resistant schizophrenia was started on clozapine. Three weeks
later he was found to be confused. His carers mentioned that he was complaining of tiredness,
increased thirst and increased urinary frequency.
1. H. Hyponatremia, a side effect with SSRIs and Carbamazepine leads to confusion, drowsiness,
muscle weakness, and seizures
2. G. Hypoglycemia, due to prolonged drinking and no eating adequately are frequently encountered in
patients with alcohol binges.
3. B. Clozapine induced hyperglycaemia; part of the metabolic syndrome is being tested.
Incorrect
Marks for this submission: 0/3.
Question 5
Marks: 0/3
MEDICATION SIDE EFFECTS - PREGNANCY

A. Amniotic Bands
B. Dental Enamel Staining
C. Down‘s Syndrome
D. Edwards syndrome
E. Irritability in the newborn
F. Neonatal haemorrhage
G. Neonatal withdrawal symptoms
H. Neural Tube Defects
I. Oral cleft.
J. Tricuspid valve abnormality.

For each of the following scenarios, choose ONE most appropriate option

A woman with bipolar disorder on lithium attending pre-conceptual counselling.


A pregnant woman taking sodium valproate for her epilepsy.
A woman with on diazepam for her anxiety disorder has discovered that she is pregnant.
1. J. Lithium is associated with the congenital malformation Ebstein‘s anomaly, which is abnormality
of tricuspid valve, wherein there could be an abnormal attachment of one or two leaflets of the tricuspid
valve.
2. H. Sodium valproate is associated with neural tube defects (2-8%). It is also associated with
craniofacial abnormalities.
3. I. Benzodiazepines given in the first trimester is associated with oral cleft.
Incorrect
Marks for this submission: 0/3.
Question 6
Marks: 0/3
DIAGNOSIS – SUBSTANCE MISUSE

A. Acute alcohol intoxication


B. Alcoholic hallucinosis
C. Amphetamine intoxication
D. Cannabis intoxication
E. Cocaine intoxication
F. Cocaine withdrawal
G. Delirium tremens
H. LSD intoxication
I. Opiate addiction
J. Opiate withdrawal

For each of the following patients, select ONE most appropriate diagnosis
An 18-year-old man presents with a two month history of panic attacks, anxiety and vivid flashbacks.
He has recently become more withdrawn, and at times shown inappropriate behaviour. On
examination he has a conjunctival injection and a cough.
A 45-year-old man of no fixed abode is an inpatient on the medical ward. The nursing staff state he is
confused and disorientated. He has been observed shouting and picking up invisible objects off the
floor. At times he appears terrified and is talking to imaginary people by his bed. He is restless,
tremulous and hypertensive.
A 19-year-old man with a history of polysubstance abuse, is sweating profusely, appears to be
agitated and tachycardic. He claims to see the colour red while listening to his CDs.
1. D – Social withdrawal, change of behaviour, respiratory symptoms and red eyes are suggestive of
cannabis use.
2. G – This acute confusion and visual hallucinations in a dishevelled man are suggestive of delirium
tremens.
3. H – The physiological arousal and synaesthetic features suggest LSD use.
Incorrect
Marks for this submission: 0/3.
Question 7
Marks: 0/3
DIAGNOSIS - PERSONALITY DISORDERS

A. Ananakstic personality disorder


B. Anxious-avoidant personality disorder
C. Borderline personality disorder
D. Dependant personality disorder
E. Dissocial personality disorder
F. Histrionic personality disorder
G. Narscissistic personality disorder
H. Paranoid personality disorder
I. Schizoid personality disorder
J. Schizotypal personality disorder.

Choose ONE of the above personality disorders which best fit each scenario:
A young girl with history of childhood sexual abuse, who has chronic suicidal ideation which she
often expresses by episodes of deliberate self-harm. She gets cross with some nurses who are treating
her.
A man with illusions, obsessive ruminations and suspiciousness, does not socialize properly and has
poor relations with people. He had an avid interest in studying psychology, and tends to like
collecting pebbles with a particular grain on them. There is no history of hallucinations.
A university student who gets angry and says on failing his thesis, "they are all jealous of me and
seem to be irritating me purposefully". His family says he is always mistrusting of people, and of a
complaining nature
1. C – This describes borderline personality disorder.
2. J – This describes features of schizotypal disorder.
3. H – This describes features of paranoid PD.
Incorrect
Marks for this submission: 0/3.
Question 8
Marks: 0/3
DIAGNOSIS - PERSONALITY DISORDERS

A. Ananakstic personality disorder


B. Anxious-avoidant personality disorder
C. Borderline personality disorder
D. Dependant personality disorder
E. Dissocial personality disorder
F. Histrionic personality disorder
G. Narscissistic personality disorder
H. Paranoid personality disorder
I. Schizoid personality disorder
J. Schizotypal personality disorder.

Choose ONE of the above personality disorders which best fit each scenario:
A woman is kept waiting for a couple of minutes by her CPN, who had to take a telephone call. She
smashes the nearby window and threatens to slash her wrists. She appears to have made similar
threats in the past.
A 28-year-old man has written a detailed complaint to the Chief Executive of the trust owing to ill
treatment by all the health professionals involved in his care. He was being treated for delusional
disorder, jealous type.
A 37-year-old charming man is seen in the outpatients for his alcohol dependence. He has a long
forensic history, which he forgets to mention. When the doctor questions him about this, he says that
most of the offences due to the fault of others.

1. F – The impulsive behaviour and threats of self-harm suggest emotionally unstable: impulsive PD.

2. H – The most likely diagnosis is paranoid PD who tend to be resentful, self-important and bear
grudges.

3. D – This is dissocial PD who do not adhere to social norms, fail to take responsibility for their
actions and display superficial charm.

Incorrect
Marks for this submission: 0/3.
Question 9
Marks: 0/3
Diagnosis – drug use

A. Acute alcohol intoxication


B. Alcoholic hallucinosis
C. Amphetamine intoxication
D. Delirium tremens
E. Opiate withdrawal
F. Opiate addiction
G. LSD intoxication
H. Cannabis intoxication
I. Cocaine withdrawal
J. Cocaine intoxication

Choose ONE diagnosis that best fits each of the clinical scenarios given below:
A 20-year-old man is brought to casualty in the early hours of the morning after suddenly developing
the belief that his friends are conspiring to kill him. His pupils are dilated and he is sweating, agitated
and tachycardic.
A 45-year-old man of no fixed abode is an inpatient on the medical ward. The nursing staff states that
he is confused and disorientated. He has been observed shouting and picking up invisible objects off
the floor. At times he appears terrified and is talking to imaginary people by his bed. He is restless,
tremulous and hypertensive.
A young woman has been brought to casualty by the police, having been arrested for shoplifting.
While in custody she became unwell, complaining of severe abdominal cramps and pain in her
muscles and joints. In the casualty department she appears restless; she is sweating profusely and has
‗goose bumps‘ on her skin.
1. People intoxicated with amphetamines can develop acute psychotic episodes, which are relatively
short lived. Another clue to the diagnosis is the characteristic physical symptoms.
2. Delirium tremens best fits this clinical picture. The core features are all present, including: clouding
of consciousness, bizarre affect, hallucinations, autonomic disturbance and tremor.
3. The clinical picture is one of opiate withdrawal. The woman will not have had access to drugs while
in custody and has developed withdrawal symptoms. Another clue is that she has been arrested for
shoplifting, which may be to finance her addiction.
Incorrect
Marks for this submission: 0/3.
Question 10
Marks: 0/3
PSYCHOTROPIC USE IN LACTATING MOTHERS

A. Chlorpromazine
B. Citalopram
C. Clomipramine
D. Diazepam
E. Doxepin
F. Fluoxetine
G. Haloperidol
H. Lorazepam
I. Nitrazepam
J. Paroxetine
K. Sulpiride

While it is best to avoid the use of psychotropics in the lactating mother, there are times when this is
necessary. For eah of the following situations, choose ONE drug from above that is being described.
Even though this selective serotonin reupdate inhibitor (SSRI) is excreted in the breast milk, it is
detected in infants‘ serum in low levels and no adverse effects have been reported.
No adverse side-effects have been reported with this tricyclic antidepressant in infants who were
exposed to it by breast-feeding.
This antipsychotic has improved lactation in lactating mothers and no adverse effects were noted in
the nursing infants.
This benzodiazepine is recommended for breast-feeding mothers.
1. J. From case reports, citalopram was associated with uneasy sleep in an infant and fluoxetine use
caused adverse effects in two infants.
2. C. Doxepin caused reversible adverse effects in two infants.
3. K. Some studies reported lethargy in infants whose mothers were on chlorpromazine.
4. H. Long-acting benzodiazepines cause lethargy, sedation and weight loss in infants.
Incorrect
Marks for this submission: 0/3.
Question 11
Marks: 0/3
DRUG INTERACTIONS

A. Aspirin
B. Cimetidine
C. Ibuprofen
D. Metaclopramide
E. Oral Contraceptive Pills.
F. Pindolol
G. Quetiapine
H. Ranitidine
I. Terfenadine
J. Warfarin
Choose ONE drug likely to be associated with each of the following scenarios:
A person on clozapine treatment developing orthostatic hypotension
A person developing acute dystonia on drinking grapefruit juice
A young man on lithium treatment develops confusion and tremors. Two days ago he had hurt himself
during a football match
1. B. Cimetidine is a CYP 450 enzyme inhibitor, which leads to increase in clozapine levels, and hence
the side effects.
2. D. Grapefruit juice is a CYP 450 enzyme inhibitor, which leads to excess levels of metaclopramide
and acute dystonia from metaclopramide, which is a D2 blocker..
3. C. Ibuprofen (NSAIDs) inhibits the renal clearance of lithium, leading to increased levels of lithium,
and subsequent toxicity
Incorrect
Marks for this submission: 0/3.
Question 12
Marks: 0/3
Investigations

A. 24 hour urinary free cortisol.


B. Dexamethasone suppression test
C. Elevated TSH.
D. Elevated serum cholesterol.
E. Hypernatraemia
F. Hypokalaemia
G. Hyponatraemia
H. Low TSH
I. Ultrasound.
J. No rise in plasma cortisol on short synacthen test

Choose TWO of these would you expect to find for each of these clinical scenarios:

1. A 21-year-old woman with low mood, tiredness, intolerance of cold and menorrhagia.

2. A 30-year-old woman pigmentation of her skin, low mood, general weakness, weight loss and
syncope.

3. A 30 year old woman with increased body weight, thin arms and legs, excess hair growth and
emotional lability.
1. C, D. The scenario describes hypothyroidism so you would therefore expect a raised TSH and raised
cholesterol. (TSH levels are normal in pituitary failure and raised in primary thyroid failure).
2. G, J. The scenario describes Addison‘s disease. Hyponataemia and lack of cortisol response on the
synacthen test are characteristic findings.
3. A, B. The scenario describes Cushing‘s syndrome, which is due to increased exposure to steroid
hormones. 24 hour urinary free cotisol levels and DST are tests for Cushing‘s disease.
Incorrect
Marks for this submission: 0/3.
Question 13
Marks: 0/3
DIAGNOSIS - PERSONALITY DISORDERS

A. Outbursts of anger or violence


B. Excessive sensitiveness to setbacks
C. Flattened affect
D. Impulsivity
E. Inability to plan ahead
F. Low Frustration tolerance
G. Marked insensitivity to social norms.
H. Over concern with physical attractiveness
I. Self-referential attitude
J. Shallow affect
K. Unstable affect

From the options above select,

1. Three characteristic features of Emotionally unstable personality disorder

2. Two characteristic features of Paranoid personality disorder

3. Two characteristic features of Histrionic personality disorder


Incorrect
Marks for this submission: 0/3.
Question 14
Marks: 0/3
SUBSTANCE MISUSE – MANAGEMENT

A. Buprenorphine
B. Benzodiazepines
C. Bupropion
D. Diamorphine
E. Loperamide
F. Lofexidine
G. Methadone
H. Naloxone
I. Naltrexone
J. NSAID

For each of the following patients, select the most likely treatment.
A 40-year-old opiate dependent woman who wants to become abstinent but cannot tolerate
withdrawal symptoms.
A man of 35 who wants to stop using IV heroin but does not want methadone
At the clinic a 22-year-old known patient is found to have collapsed in the toilets with pinpoint pupils
1. F. Lofexidine is a alpha 2 agonist like clonidine used in the management of opiod withdrawal.
2. A – Buprenorphine, is used in substitution treatment of opiod dependence.
3. H – naloxone is the treatment of opiod overdoses.
Incorrect
Marks for this submission: 0/3.

ADHD

1
Marks: 0/1
You see a 12 year old boy with Tourette's syndrome, who also has hyperkinetic disorder. The most effective
treatment for this boy would be
Choose one answer.
a. A TCA
b. A NARI
c. A beta blocker
d. A central alpha agonist
e. A stimulant
Methylphenidate and clonidine are individually effective for treating ADHD in children with tics and that the
two drugs help control different symptoms of ADHD and a combination of the drugs is more effective than
either drug alone. The Tourette's Syndrome Study Group. "Treatment of ADHD in children with tics."
Neurology, Vol. 58, No. 4, February 26, 2002, pp. 527-536
Incorrect
Marks for this submission: 0/1.
Question 2
Marks: 0/1
You see a 12 year old boy with Tourette's syndrome, who also has hyperkinetic disorder. The most effective
treatment for this boy would be
Choose one answer.
a. A NARI
b. Risperidone
c. A central a agonist
d. Pimoline
e. A TCA
NICE guidelines recommend the use of Methylphenidate or Atomoxetine (which is a NARI) in the treatment of
ADHD and comorbid Tics. Nice guidelines for ADHD,2008
Incorrect
Marks for this submission: 0/1.
Question 3
Marks: 0/1
You see a 8 year old child with hyperkinetic disorder and no other comorbidities. The first line of treatment
that you would consider taking into account the current evidence base is
Choose one answer.
a. Atomoxetine
b. Dexamphetamine
c. Clonidine
d. Methylphenidate
e. Imipramine
NICE guideline recommend the use of methylphenidate as a first line medical treatment for ADHD with no
other co-morbidities. Some patients who do not respond to methylphenidate will respond to dexamphetamine
and vice versa. Dexamphetamine is licensed for use in children over 3 years for narcolepsy and as adjunct in the
management of refractory hyperkinetic states in children (under specialist supervision). Nice guidelines for
ADHD,2008
Incorrect
Marks for this submission: 0/1.
Question 4
Marks: 0/1
The first line of treatment of an 8 year old child with hyperkinetic disorders without any co-morbidities but
severe impairment is
Choose one answer.
a. Atomoxetine
b. Clonidine
c. Cognitive behavioral therapy
d. Dexamphetamine
e. Methylphenidate
Methylphenidate for ADHD without significant comorbidity, methylphenidate for ADHD with comorbid conduct
disorder, methylphenidate or atomoxetine in the presence of tics, Tourettes syndrome, anxiety disorder,
stimulant misuse or risk of stimulant diversion, atomoxetine if methylphenidate has been tried and has been
ineffective at the maximum tolerated, dose, or the child or young person is intolerant to low or moderate
doses of methylphenidate. NICE Guidelines Sept 2008
Incorrect
Marks for this submission: 0/1.
Question 5
Marks: 0/1
The common side effect of clonidine does not include
Choose one answer.
a. Constipation
b. Hypotension
c. Depression
d. Sedation
e. Dry mouth
Uncommon CNS side effects of clonidine include insomnia,anxiety,depression,night mares and hallucinations.
Synopsis of Psychiatry,10 edn,p-999
Incorrect
Marks for this submission: 0/1.
Question 6
Marks: 0/1
The following is true regarding Atomoxetine
Choose one answer.
a. Inhibits noradrenalin uptake post-synaptically
b. Is a stimulant medication for ADHD
c. If used with methylphenidate, increases cardiovascular side-effects
d. Has a half life of 24 hours
e. Licensed for use in adults
Atomoxetine is a highly specific NARI, blocking pre-synaptic NA transporters . It is licensed for ADHD in children
and adolescents, and adults where treatment is a continuation of adolescent therapy. Half life varies from 5hrs
in rapid metabolisers - 21.6hrs in slow metabolisers. The longer duration of action allows for once-daily dosing.
Co administration with methylphenidate does not increase CVS effects beyond those seen with
methylphenidate alone. Click here for reference
Incorrect
Marks for this submission: 0/1.
Question 7
Marks: 0/1
In Attention-deficit hyperactivity disorder, combined behavioural therapy and pharmacological treatment is
considered to be as equally effective as
Choose one answer.
a. Intensive behavioural therapy in reducing hyperactivity
b. Intensive behavioural treatment in reducing oppositionality
c. Medication in reducing hyperactivity, impulsivity and inattention
d. Psychosocial treatment in improving social skills and peer relations
e. Medication in improving academic performance
Combination treatments as well as medication-management alone are both significantly superior to intensive
behavioural treatments and routine community treatments in reducing ADHD symptoms. In other areas of
functioning (specifically anxiety symptoms, academic performance, oppositionality, parent-child relations, and
social skills), the combined treatment approach was consistently superior to routine community care, whereas
the single treatments (medication-only or behavioural treatment only) were not. Click here for reference
Incorrect
Marks for this submission: 0/1.
Question 8
Marks: 0/1
You see a 8 year old child with hyperkinetic disorder and no other comorbidities. The first line of treatment
that you would consider taking into account the current evidence base is
Choose one answer.
a. Imipramine
b. Methylphenidate
c. Clonidine
d. Dexamphetamine
e. Atomoxetine
NICE guideline recommend the use of methylphenidate as a first line medical treatment for ADHD with no
other co-morbidities. Some patients who do not respond to methylphenidate will respond to dexamphetamine
and vice versa. Dexamphetamine is licensed for use in children over 3 years for narcolepsy and as adjunct in the
management of refractory hyperkinetic states in children (under specialist supervision). Nice guidelines for
ADHD,2008
Incorrect
Marks for this submission: 0/1.
Question 9
Marks: 0/1
The following is true in the treatment of ADHD
Choose one answer.
a. Atomoxetine is the first line in preschool children
b. Avoid using antipsychotics in the treatment of ADHD in children
c. Atomoxetine can be given only as second line
d. Comorbid symptoms should not change the prescription
e. Medication benefits only hyperactivity
Nice guidelines recommend to avoid the use of antipsychotics for ADHD in children and young people. Nice
guidelines for ADHD,2008
Incorrect
Marks for this submission: 0/1.
Autistic Spectrum Disorders

1
Marks: 0/1
Select the following that is commonly associated with Autism
Choose one answer.
a. Deprivation of face-to-face contact in first 6 months of life
b. Predisposition to schizophrenia in adulthood
c. Less social isolation after childhood
d. Decline in IQ in adolescence
e. Epilepsy in early childhood
Most children with autistic spectrum disorders do not show deficits in all these areas, and many gain
some social behaviours as they grow older. 25% develop seizures, peaking at 11-14 years of age.
Extreme and prolonged early deprivation in grossly inadequate institutions may result in autistic
features. It is doubtful that an autistic-spectrum disorder predisposes to schizophrenia Rutter, Michael;
Taylor, Eric, Child and Adolescent Psychiatry 2002, page 637; Howlin, P., Goode, S., Hutton, J., et al
(2004) Adult outcome for children with autism. Journal of Child Psychology and Psychiatry and Allied
Disciplines, 45, 212229.
Incorrect
Marks for this submission: 0/1.
Question 2
Marks: 0/1
The following is true with respect to Autism
Choose one answer.
a. Perinatal complications not associated with autism
b. Rates have been found to be higher in patients exposed to the
MMR vaccine
c. Monozygotic concordance rate less than 10%
d. Hereditability of over 90%
e. Not associated with Fragile X syndrome
Early studies of twins estimated the heritability of autism to be more than 90%; in other words, that
90% of the differences between autistic and non-autistic individuals is due to genetic effects.
[Monozygotic concordance is more than 90%. Autism is seen in around 25% of male patients with
fragile X and 6% of females. Perinatal complications are associated with autism (there is a higher
prevalence of complications, although note that this may not be causal).] Rutter (2000) Genetic Studies
of Autism: From the 1970s into the Millenium. Journal of Abnormal Child Psychology. 28(1) 3-14.
Abrahams BS, Geschwind DH (2008) Advances in autism genetics: on the threshold of a new
neurobiology. Nat Rev Genet 9 (5): 34155.
Incorrect
Marks for this submission: 0/1.
Question 3
Marks: 0/1
In which of the following would you see abnormal language development in?
Choose one answer.
a. Conduct disorder
b. Tourettes syndrome
c. Autism
d. ADHD
e. Aspergers syndrome
Deficits in language development and difficulty using language to communicate ideas are among the
principal criteria for diagnosing autistic disorder. Language is not affected in the other choices
mentioned. Synopsis of Psychiatry, 10th edn, pg 1194
Incorrect
Marks for this submission: 0/1.
Question 4
Marks: 0/1
Epidemiological studies have failed to account for autism in
Choose one answer.
a. Prenatal cytomegalovirus infection
b. Toxin exposure
c. Perinatal insults
d. Exposure to thimerosal (mercury) in vaccines
e. Congenital rubella
Epidemiologic studies indicate that environmental factors such as toxic exposures, teratogens, perinatal
insults, and prenatal infections such as rubella and cytomegalovirus account for a few cases. These
studies fail to confirm that immunizations with the measles-mumps-rubella vaccine are responsible for
the surge in autism. Lancet. 1999 Jun 12;353(9169):2026-9. MMR vaccination and autism. Wakefield
AJ. and Muhle R, Trentacoste SV, Rapin I. The genetics of autism. Pediatrics. 2004 May;113(5):e472-
86. Review.
Incorrect
Marks for this submission: 0/1.
Question 5
Marks: 0/1
Select the feature that is considered as a diagnostic feature of essential for the diagnosis of Autism
Choose one answer.
a. Lack of imitative play
b. Inability to make direct eye-to-eye
contact
c. Impairments in theory of mind
d. An IQ less than 70
e. Lack of interests and activities
Deficits occur in imitation, social play and spontaneous imaginative role play or play with dolls or
action figures (Harris 1993; Lewis & Boucher 1995). There are restricted, repetitive activities and
interests. There is unusual eye contact and may be gaze avoidant or stare inappropriately. Lack of
theory of mind is not a diagnostic criteria as per the ICD-10. World Health Organization (1992) The
ICD-10 Classification of Mental and Behavioural Disorders: Clinical Descriptions and Diagnostic
Guidelines. Geneva: WHO.
Incorrect
Marks for this submission: 0/1.
Question 6
Marks: 0/1
You are counselling a mother who has one child with autism and is expecting her second child. She
wishes to know the risk of autism in this second child. The most appropriate response would be:
Choose one answer.
a. 1 10%
b. 30 -40%
c. 0.1-1%
d. 40 50%
e. 10 20%
The recurrence rate in siblings of affected children is approximately 2% to 8%. Muhle R, Trentacoste
SV, Rapin I. The genetics of autism. Paediatrics. 2004 May;113(5):e472-86. Review. Case Studies in
Child and Adolescent Mental Health. MS Thambirajah. P76. Radcliffe Publishing.
Incorrect
Marks for this submission: 0/1.
Question 7
Marks: 0/1
Autism is most likely to be associated with which of the following:
Choose one answer.
a. ADHD
b. Gilles De La Tourette
c. Conduct disorder
d. Williams syndrome
e. Aspergers syndrome
Hyperactivity is common with autism Goodman & Scott, Child Psychiatry, 2002, pg 43
Incorrect
Marks for this submission: 0/1.
Question 8
Marks: 0/1
A child is diagnosed with infantile autism . The following is not a feature of this condition
Choose one answer.
a. Restricted interests
b. Late onset
c. Communication impairment
d. Early onset
e. Social impairment
Infantile autism is defined by the combination of four sets of features . Child Psychiatry,Goodman and
scott,2002,p-41
Incorrect
Marks for this submission: 0/1.
Question 9
Marks: 0/1
Aspergers syndrome differs from childhood autism in
Choose one answer.
a. Reciprocal social interactions
b. Language development
c. Motor milestones
d. Non-verbal communication
e. Restricted, repetitive interests
Unlike in autism, intellectual ability and syntactical speech are normal in Aspergers syndrome Tom
Berney. Asperger syndrome from childhood into adulthood. Advan. Psychiatr. Treat., Sep 2004; 10:
341 351
Incorrect
Marks for this submission: 0/1.
Question 10
Marks: 0/1
Which of the following features is not associated with Retts syndrome
Choose one answer.
a. Developmental delays are seen early in the course of the illness
b. Midline hand wringing
c. Non progressive
d. Purposeful hand movements lost
e. Normal development till 18 months of age
Retts syndrome is seen in girls, initially with normal development followed by regression and death by
30 yrs Goodman & Scott, Child Psychiatry,2002,p-45
Incorrect
Marks for this submission: 0/1.

Aspergers Syndrome
1
Marks: 0/1
The male to female ratio is aspersers disorder is
Choose one answer.
a. 6-8:1
b. 2-4:1
c. 4-6:1
d. 10-12:1
e. 8-10:1
The World Health Organisation ICD-10 classification gives the male to female ratio as 8:1 Tom
Berney. Asperger syndrome from childhood into adulthood. Advances in Psychiatric Treatment (2004)
10: 341-351
Incorrect
Marks for this submission: 0/1.
Question 2
Marks: 0/1
Aspergers syndrome differs from childhood autism in
Choose one answer.
a. Restricted, repetitive interests
b. Motor milestones
c. Reciprocal social interactions
d. Language development
e. Non-verbal communication
Unlike in autism, intellectual ability and syntactical speech are normal in Aspergers syndrome Tom
Berney. Asperger syndrome from childhood into adulthood. Advan. Psychiatr. Treat., Sep 2004; 10:
341 351
Incorrect
Marks for this submission: 0/1.

Conduct Disorder
1
Marks: 0/1
In a 12 month period around one in five British 11-15 year olds are more likely to use the following
Choose one answer.
a. Opiates
b. Volatile substances
c. Cannabis
d. Amphetamines
e. Cocaine
The National Centre for Social Research and the National Foundation for Educational Research, show that 12%
of pupils aged 12-15 had used drugs in the last month and 20% had used drugs in the last year. The usuage was
as follows: Cannabis (13%); Volatile substances (7%); Class A stimulants & opiates (4%) Click here for reference
Incorrect
Marks for this submission: 0/1.
Question 2
Marks: 0/1
A 10 year old girl presenting with antisocial behaviour and delinquency is more likely to develop the following
Choose one answer.
a. Depression
b. Criminal behaviour
c. Aggressive behaviour
d. Antisocial personality disorder
e. Substance misuse disorder
A homotypic outcome is common in males. There is aggression and violence, alcohol and drug use, crime and
antisocial personality disorder. A heterotypic course is more common in females, with less aggression and
crime and the development of emotional disorders or personality disorders Click here for reference
Incorrect
Marks for this submission: 0/1.
Question 3
Marks: 0/1
Absence from school without parental permission is most likely to be associated with
Choose one answer.
a. Conduct disorder
b. Attention-deficit disorder
c. Depression
d. Encopresis
e. Oppositional defiant disorder
Truancy occurs in group and is the most common reason for non-attendance in the last years of school. It is
often linked to conduct disorder Goodman & Scott. Child Psychiatry. Blackwell Publishing. Pg 76
Incorrect
Marks for this submission: 0/1.
Question 4
Marks: 0/1
Which of the following is less likely an outcome of childhood onset conduct disorder?
Choose one answer.
a. Substance misuse
b. Mental health problems
c. Violence against women and children
d. Have less number of children themselves
e. Psycopathic personality traits
In a follow up study,the childhood-onset delinquents were the most elevated on psychopathic personality
traits, mental-health problems, substance dependence, numbers of children, financial problems, work
problems, and drug-related and violent crime, including violence against women and children. The adolescent-
onset delinquents at 26 years were less extreme but elevated on impulsive personality traits, mental-health
problems, substance dependence, financial problems, and property offenses. A third group of men who had
been aggressive as children but not very delinquent as adolescents emerged as low-level chronic offenders who
were anxious, depressed, socially isolated, and had financial and work problems. Noffitt TE et al Males on the
life-course-persistent and adolescence-limited antisocial pathways: follow-up at age 26 years Dev
Psychopathol.2002 Winter;14(1):179-207.
Incorrect
Marks for this submission: 0/1.
Question 5
Marks: 0/1
The following is true regarding Conduct disorder
Choose one answer.
a. Predicted by temperament in the first 6 months of life
b. With peer rejection alone increases later antisocial behaviour, irrespective of the childs aggression
c. Without comorbid ADHD, has normal executive function
d. Associated with maternal smoking during pregnancy in a number of studies
e. In boys, usually gets approval from non-deviant peers
Maternal smoking during pregnancy has been associated in a number of prospective longitudinal studies with
an increased risk of conduct disorder in children and adolescents (Wakschlag et al. 1997; Fergusson et al. 1998;
Brennan et al. 1999; Rasanen et al. 1999; Weissman et al. 1999). Environmental risk factors include: poverty
and unemployment, smoking in pregnancy, parental psychopathology (especially maternal depression and
paternal antisocial personality disorder), parental alcohol or substance misuse, parental discord, poor
parenting, insecure attachment, teacher and peer rejection, delinquent peer affiliation, early onset of youth
drug and alcohol misuse. Toupin et al demonstrated that children with CD were significantly impaired in four of
five executive function measures after ADHD symptoms and socioeconomic status (SES) were controlled.
Infants with temperaments classified as difficult at 3 years of age are more likely to be referred for aggressive
problems later on and together with the trait of negative emotionality they have a clear modest effect in
predicting later antisocial behaviour of the early-onset type. Children who are aggressive or antisocial are often
rejected by their peers (Marshall & Watt, 1999). As Dishion et al (1991) show, peer group rejection is often a
prelude to deviant peer group membership, which reinforces deviant behaviours. Parent Training Programmes
for the Management of Young Children with Conduct Disorders: Findings from Research. Joanna Richardson
and Carol Joughin. Gaskell. Jun 2002 pg 7Unsocialised conduct disorder: associated with disturbed peer
relationships characterised by isolation, rejection and lack of lasting, empathic, reciprocal relationships with
peers. Aggressive children (whether rejected or not) have more conduct problems than non-aggressive
(whether rejected or not), but rejected show more diverse and severe conduct problems, along with greater
deficiencies in the domain of adaptability. Toupin J, Dry M, Pauz R, Mercier H & Fortin L (2000) Cognitive and
Familial Contributions to Conduct Disorder in Children. Journal of Child Psychology and Psychiatry Vol41 Iss3 Pg
333; Rutter, Michael; Taylor, Eric, Child and Adolescent Psychiatry 2002; Farrington D P (1999) Conduct
disorder and delinquency. In Risks and Outcomes in Developmental Psychopathology (eds H C Steinhausen & F
C Verhulst), pp.165192. Oxford: OUP; Thomas, A., Chess, S., and Birch, H. (1968). Temperament and behavior
disorders in children. New York University Press and Bierman, K.L., Smoot, D.L., and Aumiller, K. (1993).
Characteristics of aggressive, rejectively aggressive (non-rejected) and rejected (nonaggressive) boys. Child
Development, 64, 13951
Incorrect
Marks for this submission: 0/1.
Question 6
Marks: 0/1
A mother asks you what is the chances of her 12 year old son with conduct disorder going on to develop
antisocial personality disorder?
Choose one answer.
a. 55%
b. 22%
c. 11%
d. 44%
e. 33%
A study conducted on a nationally representative sample from the National Epidemiologic Survey on Alcohol
and Related Conditions (NESARC) found that Approximately 75% of those with Conduct disorder also met
criteria for Antisocial personality disorder, contradicting the views from previous studies that only about 40% of
conduct disorder go on to have ASPD. Gelhorn H L. et al. DSM-IV Conduct disorder criteria as predictors of
antisocial personality disorder. Compr Psychiatry. 2007; 48(6): 529538.
Incorrect
Marks for this submission: 0/1.
Question 7
Marks: 0/1
A 12 year old is brought to see you due to shoplifting. The diagnosis that is he is most likely to have is
Choose one answer.
a. Hyperkinetic disorder
b. Depression
c. Conduct disorder
d. Aspergers syndrome
e. Oppositional defiant disorder
Shoplifting is mostly not associated with mental disorder. More likely stealing for money for other pursuits. The
most likely diagnosis to be considered would be Conduct disorder. Goodman & Scott. Child Psychiatry. Juvenile
Delinquency. Chapter 7. Blackwell Publishing.
Incorrect
Marks for this submission: 0/1.

Emotional Disorders
1
Marks: 0/1
A 15 year old boy presents with a moderate depressive illness. He is otherwise fit and intelligent and
does not have suicidal ideas or plans. The treatment of choice for this boy would be
Choose one answer.
a. Venlafaxine
b. CBT & SSRI
c. SSRI
d. CBT
e. TCA
Major controversies about prescribing of antidepressants in children and adolescents. TCAs,
Venlafaxine, and Paroxetine should not be used, and limits on use of SSRIs NICE recommends: Mild
depression ? Antidepressant medication should not be used for the initial treatment of children and
young people with mild depression. Moderate to severe depression ? Children and young people with
moderate to severe depression should be offered, as a first-line treatment, a specific psychological
therapy (individual cognitive behavioural therapy [CBT], interpersonal therapy or shorter-term family
therapy; it is suggested that this should be of at least 3 months duration). ? Antidepressant medication
should not be offered to a child or young person with moderate to severe depression except in
combination with a concurrent psychological therapy. Specific arrangements must be made for careful
monitoring of adverse drug reactions, as well as for reviewing mental state and general progress; for
example, weekly contact with the child or young person and their parent(s) or carer(s) for the first 4
weeks of treatment. In the event that psychological therapies are declined, medication may still be
given, but as the young person will not be reviewed at psychological therapy sessions, the prescribing
doctor should closely monitor the child or young persons progress on a regular basis and focus
particularly on emergent adverse drug reactions. there is limited evidence for the treatment of
depression esp. antidepressants in children - except for fluoxetine December 2003 - the CSM advised
for fluoxetine only - the balance of risks and benefits favourable (However specialist may prescribe
other SSRIs) NICE guidance, 2005
Incorrect
Marks for this submission: 0/1.
Question 2
Marks: 0/1
The most common reason for self harm in adolescents is
Choose one answer.
a. Schoolwork
b. Eating disorder
c. Bereavement
d. Relationship problems with family
e. Consequences of childhood sexual abuse
The most common problem facing the young people involved relationship difficulties within the
family, followed by relationship with friends which was more common in girls. Keith Hawton and
Louise Harris, Deliberate self-harm by under-15-year-olds: characteristics, trends and outcome, The
Journal of child psychiatry and psychology, Volume 49, Issue 4, Pages 441-448
Incorrect
Marks for this submission: 0/1.
Question 3
Marks: 0/1
Select the following which is associated with Adolescent onset depression
Choose one answer.
a. Hypersecretion of salivary DHEA
b. The effect of more cortisol than DHEA
c. Velocardiofacial syndrome (i.e. Di George Syndrome)
d. Methyltetrahydrofolate reductase (MTHFR) gene polymorphism
e. The dystrobrevin gene
Sleep and cortisol studies do not show the abnormalities described in adults (these are less marked in
young adults too). A raised cortisol : DHEA ratio has been described. It predicts persistent as opposed
to remitting depression. It is not clear whether this is genetic, due to early adversity or the effect of
repeated depressive episodes. Only one study has reported salivary DHEA levels in young patients with
current MDD1. Hyposecretion of DHEA was significantly associated with MDD, with 31% of patients
with depression aged 8-16 years showing levels below the 20th centile of control subjects (equivalent
to less than 0.7 nmol/l) (Goodyer et al, 1996). This was not a function of age, nor was it associated with
any comorbid syndrome at presentation(2). Dystobrevin binding protein (DTNBP1) is a susceptibility
locus for schizophrenia (3) Methyltetrahydrofolate reductase (MTHFR) gene polymorphism (C677T
polymorphism) may be associated with late onset depression (4). Velocardiofacial syndrome is the
biggest known genetic risk factor for schizophrenia (5). chromosome 22q11. (1). I. M. GOODYER, R.
J. PARK, C. M. NETHERTON, and J. HERBERT. Possible role of cortisol and
dehydroepiandrosterone in human development and psychopathology Br. J. Psychiatry, September 1,
2001; 179(3): 243 - 249. (2) see weblink (3). see webslink (4). Lewis SJ, Lawlor DA, Davey Smith G,
Araya R, Timpson N, Day IN, Ebrahim S. The thermolabile variant of MTHFR is associated with
depression in the British Women's Heart and Health Study and a meta-analysis. Mol Psychiatry. 2006
Apr;11(4):352-60 . (5). Hollis C P (2000) Adolescent schizophrenia. Advances in Psychiatric
Treatment, 6, 8392.
Incorrect
Marks for this submission: 0/1.
Question 4
Marks: 0/1
The common depressive symptoms in pateints with learning disability does not include
Choose one answer.
a. Self injury
b. Suicidal thoughts
c. Hyperactivity
d. Weight gain
e. Irritability
Suicidal thoughts and acts occur mainly in those with mild LD and overall they are not frequently
reported . Companion to psychiatric studies, 8th edn,p-582
Incorrect
Marks for this submission: 0/1.
Question 5
Marks: 0/1
The antidepressant of choice in treating moderate depression in children is
Choose one answer.
a. Sertraline
b. Citalopram
c. Paroxetine
d. Fluoxetine
e. Mirtrazepine
Nice guidelines suggest that fluoxetine should be considered the first line in the treatment of depression
Nice guidleines,2005
Incorrect
Marks for this submission: 0/1.

Enuresis

1
Marks: 0/1
The following is indicated in an 8 year old boy with primary enuresis
Choose one answer.
a. Retention-control training
b. An intravenous urogram
c. IQ testing
d. EEG
e. Fluid restriction before bedtime
Nocturnal enuresis occurs in approximately 1520% of five-year-olds. Only 12% of children aged 15
years or over will occasionally suffer from nocturnal enuresis. By age 15-16, if there is a poor response
to therapy, a video-urodynamic study is indicated. Very rarely due to anatomical problems. Enuresis is
the result of an interaction between physiological and psychological factors (the strongest predictor of
the age at which a child attains nocturnal bladder control is the number of first-order relatives having a
known history of enuresis). It is not generally caused by emotional problems or psychological trauma
Common sense measures include restricting fluid before bedtime, waking to use the toilet (become less
popular) and star charts. There is no value in retention-control training. Blackwell C (1989) cited in
Boson S & Lynth N (2000) Nocturnal enuresis. In Clinical Evidence: A Compendium of the Best
Available Evidence for Effective Health Care. Issue 3. London: BMJ Publishing Group. Goodman &
Scott. Child Psychiatry. Blackwell Publishing. Pg 123
Incorrect
Marks for this submission: 0/1.
Question 2
Marks: 0/1
A 8 year old child is referred with enuresis .The main stay of treatment is
Choose one answer.
a. Amitryptilline
b. Alarms
c. Anticholinergics
d. Behaviour therapy
e. Imipramine
A simple monitoring and reward programme that includes a chart may be a n effective step before
looking at other options Text book of Psychiatry,Hales,3 rd edition,p-1356
Incorrect
Marks for this submission: 0/1.
Question 3
Marks: 0/1
Which of the following is a false statement regarding functional enuresis in children
Choose one answer.
a. Male to female ratio of 3:1
b. Part of Helman's triangle for predicting future violence
c. Maybe nocturnal or diurnal in nature
d. Parental attitude has no aetiological role
e. Cannot occur as a complication of a psychiatric illness
Besides genetic factors, rigid toilet training, negative or indifferent attitudes of parents and stressful
events leading to anxiety are etiological factors. Helmans triangle includes Bedwetting, firesetting and
cruelty to animals. Blackwell C (1989) cited in Boson S & Lynth N (2000) Nocturnal enuresis. In
Clinical Evidence: A Compendium of the Best Available Evidence for Effective Health Care. Issue 3.
London: BMJ Publishing Group.
Incorrect
Marks for this submission: 0/1.

PANDAS

1
Marks: 0/1
In a child with PANDAS you would expect to find raised
Choose one answer.
a. antiDNAse B
b. antiASO
c. antinuclear antibodies
d. IgA
e. ceruloplasmin
Two different strep. tests are commercially available: the antistrepolysin O (ASO) titer, which rises 3-6 weeks
after a strep. infection, and the antistreptococcal DNAase B (AntiDNAse-B) titer, which rises 6-8 weeks after a
strep. infection. Click here for reference
Incorrect
Marks for this submission: 0/1.
Question 2
Marks: 0/1
In children with PANDAs, the following symptom is least likely to be present
Choose one answer.
a. Obsessions
b. Anxiety
c. Depression
d. Choreiform movements
e. Auditory hallucinations
Children with PANDAS often experience one or more of the following symptoms in conjunction with their OCD
and/or tics: 1. ADHD symptoms (hyperactivity, inattention, fidgety) 2. Separation anxiety (Child is "clingy" and
has difficulty separating from his/her caregivers. For example, the child may not want to be in a different room
in the house from his/her parents.) 3. Mood changes (irritability, sadness, emotional lability) 4. Sleep
disturbance 5. Night- time bed wetting and/or day- time urinary frequency 6. Fine/gross motor changes (e.g.
changes in handwriting) 7. Joint pains SE Swedo, HL Leonard, JL Rapoport. Pediatric autoimmune
neuropsychiatric disorders associated with streptococcal infections (PANDAS) - Pediatrics. 2004
Apr;113(4):907-11.
Incorrect
Marks for this submission: 0/1.
Question 3
Marks: 0/1
Which of the following is less commonly seen in Pediatric Autoimmune Neuropsychiatric disorder associated
with Streptococcal infection
Choose one answer.
a. Motor tics
b. Separation anxiety
c. Hallucinations
d. Obsessions
e. Joint pains
PANDAS, is an abbreviation for Pediatric Autoimmune Neuropsychiatric Disorders Associated with
Streptococcal Infections. The term is used to describe a subset of children who have Obsessive Compulsive
Disorder (OCD) and/or tic disorders such as Tourette's Syndrome, and in whom symptoms worsen following
strep. infections such as "Strep throat" and Scarlet Fever. In conjunction they may have symptoms of ADHD,
Sleep disturbances, bed wetting ,gross motor disturbances. In addition to these symptoms, children may also
become moody, irritable or show concerns about separating from parents or loved ones. It has also been
described as a subtype of childhood-onset OCD. The children usually have dramatic, "overnight" onset of
symptoms, including motor or vocal tics, obsessions, and/or compulsions. In addition to these symptoms,
children may also become moody, irritable or show concerns about separating from parents or loved ones. This
abrupt onset is generally preceded by a Strep. throat infection. SE Swedo, HL Leonard, JL Rapoport. Pediatric
autoimmune neuropsychiatric disorders associated with streptococcal infections (PANDAS) - Pediatrics. 2004
Apr;113(4):907-11.
Incorrect
Marks for this submission: 0/1.

Tourette's Syndrome
1
Marks: 0/1
Which is true regarding Tourette's syndrome?
Choose one answer.
a. More common in females
b. Antipsychotic medication is to be avoided
c. 50% of children with this condition continue to have symptoms into adult life.
d. Rarely begins before adolescence
e. Coprolalia is essential for diagnosis
The condition usually starts in childhood, more common in males (1:4), has a strong genetic
component, strongly associated with obsessive compulsive disorder, depressive symptoms are
commonplace and antipsychotic medication (haloperidol) are the mainstay of pharmacological
treatment. The condition is chronic with 50 % continue to have symptoms into adult life. Companion to
Psychiatric Studies, 7th Edn, 2004, pp 354, 597.
Incorrect
Marks for this submission: 0/1.
Question 2
Marks: 0/1
Gilles de la Tourettes syndrome is strongly associated with which of the following?
Choose one answer.
a. OCD
b. Conduct Disorder
c. ODD
d. ADHD
e. Autism
Tics occur in between 17 and 40 percent of children with obsessive-compulsive disorder. Many of the
children with Tourette syndrome have obsessional symptoms. Depressive symptoms are commonplace,
and attention deficit hyperactivity disorder has been described. Shorter Oxford Textbook of Psychiatry,
5th Edn, 2006, p 685 & Companion to Psychiatric Studies, 7th Edn, 2004, p 354.
Incorrect
Marks for this submission: 0/1.
Question 3
Marks: 0/1
One of the most evidence based treatment for reducing Tics in Tourette Syndrome is
Choose one answer.
a. Clonidine
b. Buspirone
c. Clonidine
d. Haloperidol
e. Atomoxetine
Besides Haloperidol, Sulpiride and Ziprasidone are known to be effective in controlling tics in
Tourette. Evidence also exists for Risperidone, Pimozide and Olanzapine. Other pharmacological
treatments for tics include: -agonists such as clonidine and guanfacine; botulinum toxin; calcium
antagonists such as nifedipine, flunarizine and verapamil; nicotine; and the selective androgen receptor
antagonist, flutamide. All received some support in open use for decreasing tics Dave Coghill Current
issues in child and adolescent psychopharmacology. Part 2: Anxiety and obsessivecompulsive
disorders, autism, Tourettes and schizophrenia.Advances in Psychiatric Treatment (2003) 9: 289-299
Incorrect
Marks for this submission: 0/1.
Question 4
Marks: 0/1
Massed negative treatment is used in which of the following conditions
Choose one answer.
a. Enuresis
b. Conduct disorder
c. Emotional disorders
d. Tics in tourettes disorder
e. ADHD
Massed negative practice (MNP) is a proposed treatment for the tics of Tourette syndrome in which the
individual with Tourette's "practices" tics continuously until a conditioned level of fatigue is reached. It
is based upon the Hullian learning theory, which holds that tics are "maladaptive habits that are
strengthened by repetition and can be replaced by the strengthening of more adaptive habits (i.e., not
having tics) Woods DW, Himle MB, Conelea CA. Behavior therapy: other interventions for tic
disorders. Adv Neurol. 2006;99:23440.
Incorrect
Marks for this submission: 0/1.
Question 5
Marks: 0/1
You see a 12 year old boy with Tourette's syndrome, who also has hyperkinetic disorder. The most
effective treatment for this boy would be
Choose one answer.
a. Pimoline
b. A NARI
c. A TCA
d. Risperidone
e. A central a agonist
NICE guidelines recommend the use of Methylphenidate or Atomoxetine (which is a NARI) in the
treatment of ADHD and comorbid Tics. Nice guidelines for ADHD,2008
Incorrect
Marks for this submission: 0/1.

Child & Adolescent - EMIs

1
Marks: 0/3

MEDICATION IN CHILD AND ADOLESCENT PSYCHIATRY.


A.Atomoxetine
B.Carbamazepine
C.Clonidine
D.Desipramine
E.Diazepam
F.Fluoxetine
G.Melatonin
H.Methylphenidate
I.Sodium Valproate
J.Topiramate

Lead in: Choose appropriate treatment for each of the following clinical presentations

1.A child with is fidgety, hyperactive and has poor attention at school and home

2.A 3 year old child with mild learning disability, now presenting with intractable insomnia, persisting even
after behavioural intervention

3. Child with low mood, insomnia, loss of weight and loss of interest
1.A, C, H - Are the three drugs commonly used in treatment of ADHD. 1st line treatment is stimulants, 2nd line is
Atomoxetine, 3rd line is Imipramine and 4th line is an alpha agonist.
2.E, G - Administration of melatonin to children with severe sleep disorders, along with other severe disabilities
(including autism), has been reported to help resolve sleep problems. Diazepam is used as a short term
measure.
3.F - . The Cochrane review demonstrated that children and adolescents 'responded' to treatment with SSRIs.
But there was also evidence of an increased risk of suicidal ideation and behaviour for those prescribed
SSRIs (RR 1.80, 95% CI 1.19 to 2.72). Fluoxetine was the only SSRI where there was consistent evidence
from three trials that it was effective in reducing depression symptoms in both children and adolescents
Incorrect
Marks for this submission: 0/3.
Question 2
Marks: 0/3

MEDICATION IN CHILD AND ADOLESCENT PSYCHIATRY

A. Citalopram
B. Desipramine
C. Escitalopram
D. Fluoxetine
E. Haloperidol
F. Lithium
G. Quetiapine
H. Reboxetine
I. Risperidone
J. Sertraline

Lead in: Choose appropriate treatment for each of the following clinical presentations

1. Treatment in a 13 year old child with OCD who is being treated with CBT and develops depression

2. Two treatments to consider as add-ons if anti-obsessional drug has not worked for six months and the child
has motor tics

3. An antidepressant NOT recommended for OCD

1. D and J
In the USA, fluoxetine was authorised in 2003 for the treatment of depressive illness and obsessive compulsive
disorder in children over age 7. Sertraline is also authorised for obsessive compulsive disorder in children over
6 years old.

2. E and I

McDougle et al. were the first to note that patients with OCD and tics benefited from augmentation with
haloperidol. Overall, current evidence supports the use of risperidone or haloperidol for patients with OCD and
tics, because these medications have proven efficacy for both Tourette syndrome and refractory OCD.
3. B and H

The only antidepressants that have been consistently shown effective in treating OCD are those that are
serotonergic. Eg Of the tricyclic antidepressants, Clomipramine is serotenergic; but Desipramine, is ineffective
in treating obsessive-compulsive symptoms. There is no evidence for the use of Reboxetine in OCD.
Incorrect
Marks for this submission: 0/3.
Question 3
Marks: 0/3
MEDICATION IN CHILD AND ADOLESCENT PSYCHIATRY
A. Clozapine
B. Diazepam
C. Family therapy
D. Lithium
E. Logotherapy
F. Methylphenidate
G. Parent training
H. Paroxetine
I. Risperidone
J. Sodium Valproate

Lead in: Choose appropriate treatment for each of the following clinical presentations

A 4 year old child, disrespectful to teachers and mother


A 17 year old, aggressive in all circumstances and to the point that parents have a court injunction against
him, but no other symptoms
A 13 year old girl, aggressive, in trouble at school, diagnosed as hyperkinetic by psychologist
1. G - . NICE recommends group-based parent-training/education programmes in the management of children
with conduct disorders. Individual-based programmes are only recommended where the family’s needs are
too complex for a group-based programme
2. I - Risperidone is the most widely prescribed drug for CD, even though there is no licensed drug for the
treatment of CD. It should therefore only be used in extreme cases.
3. F - NICE recommends methylphenidate as the first line drug for ADHD
Incorrect
Marks for this submission: 0/3.

LD
Aetiology & Clinical Features
1
Marks: 0/1
The following is associated with Williams syndrome
Choose one answer.
a. social emotional repertoire is retained
b. There is deletion in chromosome 9
c. hyperactivity is rare
d. abstract and conceptual-relational vocabulary is good
e. hypocalcaemia is present in infancy
Williams syndrome: (Excessive friendliness, empathic and emotional). A random genetic mutation (deletion of a
small piece of ch 7) associated with Good concrete thinking, practical vocabulary, poor abstract or conceptual-
relational vocabulary. (Most older children and adults with WS speak fluently and use good grammar) Common
problems include significant cardiovascular disease (narrowed arteries) and elevated blood calcium levels in
infancy. More than 50% of children with WS have ADD or ADHD. Gelder, Michael G.; Lopez-Ibor, Juan Jose;
Andreasen, Nancy C., New Oxford Textbook of Psychiatry, Volume 2 2003, eTextbook p 1957
Incorrect
Marks for this submission: 0/1.
Question 2
Marks: 0/1
Which of the following is true regarding Lennox-Gastaut syndrome
Choose one answer.
a. Onset is rare in childhood
b. Seizures are easily controlled
c. 90% of cases make a full recovery
d. Intellectual impairment is uncommon
e. Valproate is first choice of treatment.
Onset is typically between 1 and 8 years of age, the syndrome is charecterised by multiple seizure types,
intellectual failureand/or behavioural disturbance and by EEG abnormalities comprising diffuse slow spike and
wave discharges set against an abnormal background with paroxysms of fast 10-12 Hz rhythms. Lishman's
Organic Psychiatry. 4th Edn. 2009. pp317-318.
Incorrect
Marks for this submission: 0/1.
Question 3
Marks: 0/1
In adults with Downs syndrome there is an increased risk of
Choose one answer.
a. hypomania
b. Leukaemia
c. Joint stiffness
d. hyperthyroidism
e. Vascular dementia
Downs syndrome is associated with increased risks of leukaemia, Alzheimers disease and hypothyroidism cf.
the general population. (1) Prone to joint instability, hyper-flexible and hypotonia. (2) The onset of mania in
Downs syndrome has been disputed. However, cases of mania have been reported later in life among those
having Down syndrome. Mania is very uncommon among women, whereas in the general population the male
: female ratio is equal. (3) (1) Mike Kerr, Improving the general health of people with learning disabilities.
Advan. Psychiatr. Treat., May 2004; 10: 200 - 206; (2) Gelder, Michael G.; Lopez-Ibor, Juan Jose; Andreasen,
Nancy C., New Oxford Textbook of Psychiatry, Volume 2 2003, eTextbook pg 1955. (3) Seminars in the
Psychiatry of Learning Disabilities (Second edition) Edited by William Fraser and Michael Kerr. Royal College of
Psychiatrists. Gaskell Aug 2003 Pg 160. And Gelder, Michael G.; Lopez-Ibor, Juan Jose; Andreasen, Nancy C.,
New Oxford Textbook of Psychiatry, Volume 2 2003, eTextbook pg 1976
Incorrect
Marks for this submission: 0/1.
Question 4
Marks: 0/1
The following are associated with Turners syndrome
Choose one answer.
a. Learning disability, as defined by ICD-10, is moderate
b. Oestrogen is given to improve fertility
c. Maternal age is not considered a risk factor
d. When the X chromosome originates from the father, the incidence of autism is similar to boys
e. Verbal IQ is more severely affected than performance IQ
Maternal or paternal age is not considered a risk factor for the condition (1). Overall intelligence is unimpaired
or there may be mild LD. Preserved verbal skills. Specific deficits in visuo-spatial tasks, visual memory, and
arithmetic. Turner girls with the paternal X chromosome had an easier time making and keeping friends, were
more aware of others' feelings, got along better with teachers and families and generally seemed happier, than
those whose X chromosomes were of maternal provenance (2) Oestrogen replacement therapy is usually
started at the time of normal puberty, starting breast development. Oestrogen and progesterone are given a
little later to induce a monthly 'period', Almost all women are infertile, but pregnancy with donor embryos may
be possible. (3)
Incorrect
Marks for this submission: 0/1.

Question 5
Marks: 0/1
An X-linked dominant cause of learning disability is
Choose one answer.

a. Lesch Nyhan syndrome


b. Metachromic leucodystrophy
c. Retts syndrome
d. Tay Sachs disease
e. Neurofibromatosis
Retts is an X-linked dominant disorder, reported only in girls. Lesch Nyhan is X-linked recessive, which almost
exclusively affects boys. X-linked inherited disorders: fragile X, Aicardi, Lesch-Nyhan, Lowe, Norrie, and Coffin-
Lowry syndromes, mucopolysaccharidosis II, Duchenne muscular dystrophy and a-thalassaemia-mental
retardation syndrome. Autosomal recessive: phenylketonuria, homocystinuria, maple syrup urine disease,
aspartylglucos-aminuria, mannosidosis, Salla disease, I-cell disease, mucopolysaccharidoses (except type II),
neuronal ceroid lipofuscinoses, Tay-Sachs disease, metachromatic leucodystrophy, Smith-Lemli-Opitz
syndrome, and Joubert syndrome. Autosomal dominant disorders: Neurofibromatosis and tuberous sclerosis
Gelder, Michael G.; Lopez-Ibor, Juan Jose; Andreasen, Nancy C.. 2003., New Oxford Textbook of Psychiatry,
Volume 2. New Edition. Oxford University Press.
Incorrect
Marks for this submission: 0/1.
Question 6
Marks: 0/1
The following is true regarding Fragile X syndrome
Choose one answer.
a. is a CCG trinucleotide repeat disease
b. Is due to a fragile gene revealed by cell-culture in a folate-thymidine-depleted medium
c. does not occur in females
d. represents half of all mental retardation cases in males
e. is due to triplet repeats on the short arm of chromosome X
Examination of the karyotype of affected individuals' lymphocytes, cultured in a folate-depleted and thymidine-
depleted medium, reveals a constriction followed by a thin strand of genetic material that extends beyond the
long arm at the highly conserved band Xq27.3 with greater than 200 CGG triplet repeats. Accounts for half of all
the X-linked mental retardation cases.70% females are unaffected carriers. Gelder, Michael G.; Lopez-Ibor, Juan
Jose; Andreasen, Nancy C., New Oxford Textbook of Psychiatry, Volume 2 2003, eTextbook p1953
Incorrect
Marks for this submission: 0/1.
Question 7
Marks: 0/1
Which of the following is the most common inherited cause of intellectual disability
Choose one answer.
a. Klinefelter's syndrome
b. Fragile X syndrome
c. Turners syndrome
d. Downs syndrome
e. Fetal Alcohol syndrome
The three most common causes are Fragile X, Downs syndrome and Fetal Alcohol syndrome with Fragile X
contributing nearly 30% of cases of the total in all. Jaffe, Jerome H., M.D. "Mental Retardation." In
Comprehensive Textbook of Psychiatry, edited by Benjamin J. Sadock, M.D. and Virginia A. Sadock, M.D. 7th
edition. Philadelphia, PA: Lippincott Williams and Wilkins, 2000
Incorrect
Marks for this submission: 0/1.

Question 8
Marks: 0/1

Select the phenotype associated with the XYY karyotype


Choose one answer.

a. occurs more often in families with a history of criminal behaviour


b. is associated with infertility
c. have rates of offending similar to XXY
d. are taller than average males
e. has higher rates of aggression
47,XYY syndrome have normal sexual development and are able to conceive children. Although not leading to
violent behaviour, XYY is associated with behavioural features that increase the likelihood of antisocial acts but
do not cause them directly. Petty crime, not severe aggression or violent crime. Affected individuals show
about three times the rate of criminal behaviour than controls or individuals with other chromosomal
anomalies such as XXY, even allowing for their reduced IQ. (1) Research has shown that XYY males were more
likely to be taller than average and of low intelligence, but failed to provide conclusive evidence about a link to
aggressive or violent behaviour.(2) In 1976 a paper was published which concluded that XYY males were more
likely to be imprisoned, but that this was due to their low intelligence and low socioeconomic status which
placed them at higher risk of being caught.(3) The current state of opinion on the XYY issue is that there is
insufficient evidence to establish any firm link between the particular genotype and an increased risk of
aggressive behaviour, although there does appear to be an increased risk of offending. (1) Gelder, Michael G.;
Lopez-Ibor, Juan Jose; Andreasen, Nancy C., New Oxford Textbook of Psychiatry, Volume 2 2003, eTextbook
p1755 and Click here for reference 2) Baker, D., Telfer M. A., Richardson, C. E. & Clark, G. R. (1970).
Chromosome errors in men with antisocial behavior: comparison of selected men with Klinefelter's syndrome
and XYY chromosome pattern. JAMA 214, 869-78; Jacobs, P. A., Price, W. H., Richmond, S. & Ratcliff, B. A. W.
(1971). Chromosome surveys in penal institutions and approved schools. J. Med. Genet. 8, 49-58; Schiavo, R. et
al. (1984), Sex, chromosome anomalies, hormones and aggressivity. Arch. Gen. Psychiatry 4, 93-9. (3) Witkin, H.
A. et al. (1976). XYY and XXY men: criminality and aggression. Science 193, 547-55.
Incorrect
Marks for this submission: 0/1.
Question 9
Marks: 0/1
Which of the following disorders is associated with tall stature?
Choose one answer.
a. Foetal Alcohol Syndrome
b. Cornelia De Lange Syndrome
c. XXY
d. Prader Willi Syndrome
e. Williams Syndrome
As XXY males enter puberty, they often dont make as much testosterone as other boys. This can lead to a taller,
less muscular body, less facial and body hair, and broader hips than other boys. Click here for reference
Incorrect
Marks for this submission: 0/1.
Question 10
Marks: 0/1
The following is true regarding Fragile X syndrome
Choose one answer.
a. does not affect females
b. is associated with a decline in cognitive functioning in adolescence
c. is associated with abnormal speech, but not language development
d. affected women show more severe mental retardation than affected men
e. is a major cause of early menapause in females
Mean IQ score in fragile X males seems to decline with increasing age. 80% of males have an IQless than70
(mild-mod), only 30% females have an IQ 50-69 (rarely lower). (1) "Litany" speech: cluttered, repetitive and
poor articulation. the delay in language development is somewhat proportional to their intellectual
development. There is perseveration of words and phrases, echolalia, cluttering, narrative speech (language
which is often devoid of content, yet the language structure remains intact), and speech sound substitution
difficulties. Syntactic competency (ability to put words together to form a sentence) and semantic concept
(knowledge of word meanings) remain intact in fragile X males, they often show problems in comprehending
sentences with high associate compounds and show productive semantic error and pragmatic competence
(normal social behaviour associated with language). (2) The risk of premature menopause to fragile X
permutation carriers may not be as great as that reported elsewhere (3) (1) World Psychiatric Association,
2000, Mental Health in Mental Retardation, Pavilion Publishing (2) Gelder, Michael G.; Lopez-Ibor, Juan Jose;
Andreasen, Nancy C., New Oxford Textbook of Psychiatry, Volume 2 2003, eTextbook p1954; (3) Fragile X
Permutations Are Not a Major Cause of Early Menopause. Aileen Kenneson, Daniel W. Cramer, and Stephen T.
Warren. The American Journal of Human Genetics, volume 61 (1997), pages 13621369.
Incorrect
Marks for this submission: 0/1.
Question 11
Marks: 0/1
Select the true statement regarding Angelman syndrome
Choose one answer.
a. Paternal disomy occurs
b. uniparental disomy is more common than in Prader-WIlli syndrome
c. the abnormal chromosome inherited is of paternal origin
d. EEG changes are present at birth
e. hyperphagia and cognitive impairment are common
About 70% of AS individuals have a de novo deletion of chromosome 15 bands q11-13 on the chromosome
inherited from their mother. Three other etiologic types of AS include paternal uniparental disomy 15 (2-5%),
imprinting mutations (2-5%) and AS cases that have no evidence of deletion, uniparental disomy, nor
imprinting mutations (20-25%). In this last group, mutations in the UBE3A/E6-AP gene have recently been
described (1) 25% of Prader-Willi is due to uniparental disomy compared to less than 20% of Angelmans.
Angelmans can be diagnosed by 6-12 months and the first sign can be detected on EEG. EEG changes are not
present at birth but develop during the 1st year of life. Prader-Willi is associated mild MR with insatiable
overeating. Angelman's is associated with severe MR with epilepsy (2) (1)- Knoll JH, Nicholis RD, Laiande M On
the parental origin of the deletion in Angelman syndrome. Hum Genet 1989; 83(2):205-207. And Kishino T,
Laiande M, Wagstaff J. UBE3A/E6-AP mutations cause Angelman Syndrome. Nature Genet 1997; 15:70-73. (2) -
Gelder, Michael G.; Lopez-Ibor, Juan Jose; Andreasen, Nancy C., New Oxford Textbook of Psychiatry, Volume 2
2003, eTextbook p1956-8
Incorrect
Marks for this submission: 0/1.
Question 12
Marks: 0/1
Which statement about Retts syndrome is INCORRECT?
Choose one answer.
a. a genetic cause is known
b. Complex stereotyped preoccupations or routines are rare
c. The child develops normally until 18 months
d. Mental deterioration precedes motor deterioration
e. There is often hyperventilation
Rett syndrome is caused by mutation MECP2 gene on the X chromosome. Occurs predominantly in girls.
Usually with an onset between 7 and 24 months of age, with mental and physical deterioration.
Hyperventilation and loss of purposive hand movements are particularly characteristic. In contrast to autism,
both deliberate self-injury and complex stereotyped preoccupations or routines are rare. Villard L, Cardoso AK,
Chelly PJ, Tardieu PM, Fontes M. Two affected boys in a Rett syndrome family: clinical and molecular findings.
Neurology 2000;55:118893;
Incorrect
Marks for this submission: 0/1.
Question 13
Marks: 0/1
Which of the following is commonly associated with microcephaly?
Choose one answer.
a. Foetal Alcohol Syndrome
b. Hydrocephalus
c. Sotos Syndrome
d. Fragile X Syndrome
e. Downs Syndrome
Children with might demonstrate learning and behavioral problems caused by prenatal exposure to alcohol.
Children with FAS are small for gestational age and have facial abnormalities. (1). Sotos syndrome (cerebral
gigantism) is a rare genetic disorder characterized by excessive physical growth during the first 2 to 3 years of
life. The disorder may be accompanied by mild mental retardation, delayed motor, cognitive, and social
development, hypotonia (low muscle tone), and speech impairments (2) Click here for reference
Incorrect
Marks for this submission: 0/1.
Question 14
Marks: 0/1
The following disorder results due to deletion of the chromosome of maternal origin
Choose one answer.
a. Patau syndrome
b. Retts syndrome
c. Angleman syndrome
d. Prader-willi syndrome
e. Tuberous sclerosis
Angelman syndrome is due to the micodeletion in the long arm of the maternally derived chromosome 15,
characterised by ataxia, epilepsy, paroxysm of laughter, absence of speech, microcephaly, severe LD, and
behavior problems. Oxford handbook of Psychiatry, 2005, Page 704
Incorrect
Marks for this submission: 0/1.
Question 15
Marks: 0/1
The most frequent cutaneous manifestation of Tuberous Sclerosis:
Choose one answer.
a. Shagreen patch
b. Hypomelanotic macules
c. Caf au lait spots
d. Butterfly rash
e. Axillary freckles
Hypomelanotic macules, commonly in an ash-leaf shape, are the most frequent cutaneous manifestation of
tuberous sclerosis and are best seen using a Woods light. The best-known cutaneous manifestation of tuberous
sclerosis is adenoma sebaceum in a butterfly distribution Anita Devlin. Paediatric neurological examination.
Advan. Psychiatr. Treat., Mar 2003; 9: 125 - 134
Incorrect
Marks for this submission: 0/1.
Question 16
Marks: 0/1
Which of the following increases the risk of Alzeihmers Disease in a person with Downs Syndrome?
Choose one answer.
a. Smoking
b. Mild LD
c. Moderate LD
d. Aluminium exposure
e. Positive family history
Various risk factors for Alzhiemers dementia in Downs syndrome has been reported, though the most
established ones are age and presence of APOE e4. Other risk factors include family history, maternal age when
child was born, female gender and atypical karotypes. Some studies have reported IQ and cognitive reserve
(i.e., how well the person is functioning) as a risk factor, though this is yet to be established as a risk factor.
Zigman WB, Lott IT, Alzheimer's disease in Down syndrome: neurobiology and risk. Mental Retardation &
Developmental Disabilities Research Reviews 2007; 13: 237 - 246.
Incorrect
Marks for this submission: 0/1.
Question 17
Marks: 0/1
The XXY karyotype is associated with
Choose one answer.
a. Moderate to severe learning difficulties
b. Behavioural problems in childhood
c. Hypogonadotrophic hypogonadism
d. Gynaecomastia
e. Higher rates of violent offending
In most cases, the physical and neurobehavioral characteristics of Klinefelter syndrome (XXY) are relatively
mild. KS is not usually associated with moderate or severe mental retardation. Generally gives rise to mild LD
and prone to emotional instability. Hypergonadotrophic hypogonadism- the pituitary compensates and
hypersecretes. Have small tests, impotence, infertility and gynaecomastia (1). As they get older, XXY males are
usually quieter, less self-confident, less active, and more helpful and obedient than other boys. As teens, they
tend to be quiet and shy. As XXY males enter puberty, they often dont make as much testosterone as other
boys. This can lead to a taller, less muscular body, less facial and body hair, and broader hips than other boys.
As teens, XXY males may have larger breasts, weaker bones, and a lower energy level than other boys. (2) (1)
Daniel H. Geschwind, Elisabeth Dykens (2004) Neurobehavioral and Psychosocial Issues in Klinefelter Syndrome
Learning Disabilities Research and Practice. Vol. 19 Issue 3 Page 166 .
Incorrect
Marks for this submission: 0/1.
Question 18
Marks: 0/1
Select the true statement regarding children with Downs syndrome
Choose one answer.
a. Are prone to high tone deafness
b. Are likely to have recurrent bronchitis
c. Have rates of OCD similar to normal children
d. Commonly have autism
e. Are likely to develop epilepsy
Sleep apnoea and repeated chest infections are common in children with Downs syndrome. Epilepsy is not as
common as in other causes of LD. Rates are 10% of adults with DS, 40% of those greater than 40y/o and rises to
80% of those who have Alzheimers. (1) Autism is rare. Usually lovable and easy going. Currently, estimates vary
between 1 and 10%. This is higher than is seen in the general population and less than other groups of children
with learning disabilities (20%) (2) OCD is more common in the DS population than the non-DS population.
Ordering and tidiness was the most common form of OCD found. The prevalence of obsessive-compulsive
disorder (OCD) is reported to be 1.65-2.5% in the general population. Myers and Pueschel (1991) found that 4
(1.7%) of 236 Down's syndrome individuals had OCD, and Prasher (1995) found OCD in 9 (4.5%) of 201 subjects.
Ordering is the most common type of action/obsession, along with ritualistic touching and cleaning (3). Low
tone deafness (conductive problems due to glue ear). High tone deafness is sensorineural. Hearing evaluation
of children with Down's syndrome revealed a high prevalence of conductive hearing loss. Auditory impairments
associated with conductive disturbances in Down's Syndrome are mainly due to serous otitis media. (4). (1) -
Gelder, Michael G.; Lopez-Ibor, Juan Jose; Andreasen, Nancy C., New Oxford Textbook of Psychiatry, Volume 2
2003, p1955. Also see weblinks
Incorrect
Marks for this submission: 0/1.
Question 19
Marks: 0/1
The following are causes of LD in the newborn, EXCEPT for
Choose one answer.
a. Transplacental transmission of protozoal infection
b. Maternal smoking in pregnancy
c. Recurrent urinary tract infections in pregnancy
d. Cytomegalovirus in pregnancy
e. Galactosemia
Urinary tract infections are common during pregnancy. These infections increase the risk of preterm labour and
premature rupture of the membranes containing the fetus. Neonatal outcomes that are associated with UTI
include sepsis and pneumonia. UTI increases the risk of low-birth-weight infants and prematurity. Click here for
reference
Incorrect
Marks for this submission: 0/1.

Epidemiology

1
Marks: 0/1
A subcultural explanation rather than a biological one in learning disabilities is most accounted for by:
Choose one answer.
a. An even spread of LD across different socio-economic groups of the population
b. Parents with a child with LD have lower IQ
c. Moderate LD
d. Facial dysmorphologies
e. Mild LD
IQ levels follow a normal distribution curve for IQ's above 70. For lower levels of IQ the curve is skewed due to
the addition of pathological (organic) causes of learning disability. 'Organic' LD has known Perinatal or
chromosomal causes, often mod-severe LD (IQ less than 50) e.g.. Down's syndrome. 'Sub-cultural' LD is the
normal variant on the lower end of the Gaussian distribution of intelligence. Click here for reference
Incorrect
Marks for this submission: 0/1.
Question 2
Marks: 0/1
The following is true regarding Moderate to profound mental retardation
Choose one answer.
a. Affects 5 in 1000 children
b. Represents 10% of all learning disability
c. Affects as many boys as girls
d. Is associated with low socioeconomic class
e. Is equivalent to an IQ less than 49
In the UK the total prevalence of LD is 20-30 per 1000, with 3-4 per 1000 having an IQ less than 50 Unlike mild
LD, moderate to profound LD is not associated with SEC or gender. However boys are overrepresented in
special schools and have higher rates of associated disorders. IQ less than 50 for moderate (12%), less than35
for severe (7%) and less than20 for profound (1%). Philip Graham. Child Psychiatry A developmental Approach.
Oxford Medical Publication. Chapter 2
Incorrect
Marks for this submission: 0/1.
Question 3
Marks: 0/1
The following is true in only a minority of those with an IQless than35
Choose one answer.
a. Eventually look after themselves under supervision
b. Will require institutional care
c. Benefit from habit training
d. Have autistic behaviours
e. Have single highly developed cognitive ability
A small number of those with severe LD are idiot savants. Many have repetitive stereotype behaviours. Many
can eventually look after themselves, however a third will require institutional care. Even for the individual
with profound mental retardation who is mute, for whom verbally based interventions are less practical,
behavioural procedures and modification of the environment can be helpful (Szymanski & King 1999). Gelder,
Psychiatry: Oxford Core Texts. Chapter 21: Learning disability.
Incorrect
Marks for this submission: 0/1.
Question 4
Marks: 0/1
Individuals with profound learning disability
Choose one answer.
a. Are unable to benefit from behaviour therapy
b. Have hyperkinesis as much as normal individuals
c. Will need nursing care in adulthood
d. Account for 0,5% of the LD population
e. Cannot be diagnosed with depression
Profound LD represents 1-2% of the LD population. Very few learn to care for themselves. Behavioural
procedures and modification of the environment can be helpful (Szymanski & King 1999). In those with more
severe forms of learning disability, somatic symptoms and their behavioural correlates are suggestive of
affective disorders.(1). Among children with mental retardation, rates of attention deficit disorder or attention
deficit with hyperactivity disorder ranged from 7 to 15%. (2) (1) - Seminars in the Psychiatry of Learning
Disabilities (Second edition) Edited by William Fraser and Michael Kerr. Royal College of Psychiatrists. Gaskell
Aug 2003. Pg 159. (2) - Rutter, Michael; Taylor, Eric, Child and Adolescent Psychiatry 2002
Incorrect
Marks for this submission: 0/1.
Question 5
Marks: 0/1
The following disorder is found only in girls.
Choose one answer.
a. Angelman syndrome
b. Friedrich Ataxia
c. Rett's syndrome
d. Beckwith-Wiedemann syndrome
e. Tuberous sclerosis
Retts syndrome (ICD10):apparently normal early development followed by partial or complete loss of speech
and of skills in locomotion and use of hands, together with deceleration in head growth, usually onset between
7 and 24 months of age. Characterstic include loss of purposive hand movements, hand wringing stereotypies
and hyperventilation. ICD-10, WHO
Incorrect
Marks for this submission: 0/1.
Question 6
Marks: 0/1
The scale used to screen for cognitive impairment in people with Downs syndrome is
Choose one answer.
a. DMR
b. Mental retardation scale
c. CAMCOG
d. MMSE
e. CAMDEX
Alzheimer's dementia is quite common in people with Downs syndrome over the age of 35, though the
traditional screening instruments used in adults without downs syndrome may be less useful in identifying this.
The three dementia screening instruments that are currently in use among people with intellectual disabilities,
namely the Dementia Scale for Down Syndrome (DSDS; Gedye, 1995) and the Dementia Questionnaire for
Persons with Mental Retardation (DMR; Evenhuis, 1992, 1996), and the Dementia Screening Questionnaire for
Individuals with Intellectual Disabilities (DSQIID) (deb et al., 2007) Deb et al. Dementia Screening Questionnaire
for Individuals with Intellectual Disabilities. The British Journal of Psychiatry (2007) 190: 440-444.
Incorrect
Marks for this submission: 0/1.
Question 7
Marks: 0/1
Select the true statement regarding Phenylketonuria
Choose one answer.
a. Is treated by increasing the amount of tryptophan in the diet
b. Is not detected by amniocentesis
c. Occurs in 1 in 20 000 births
d. Recurrence rates in a healthy couple is less than 1%
e. Is always detected by the Guthries test
Autosomal recessive inheritance (1 in 4 chance). Prevalence of 1:10 000-25 000 (0.05-0.2 / 1000 births). It
affects approximately 1 in 12000 people. Treatment with a phenylalanine-restricted diet must start within 20
days of birth. Amniocentesis or chorionic villus sampling with DNA analysis can be performed to determine
whether a fetus has the disorder. In the UK, all newborn babies (at 5-10 days old) are screened for both PKU
and congenital hypothyroidism using the Guthrie test. False positive and false negative results can occur in PKU
screening. Gelder, Michael G.; Lopez-Ibor, Juan Jose; Andreasen, Nancy C., New Oxford Textbook of Psychiatry,
Volume 2 2003 eTextbook p1959.
Incorrect
Marks for this submission: 0/1.
Question 8
Marks: 0/1
Primary prevention of learning disability includes
Choose one answer.
a. hair sample enzyme estimation
b. Guthrie testing
c. Diet modification
d. Vitamin K injection
e. Genetic counselling
Primary prevention - prevents the occurrence of LD. Secondary prevention, involves early screening and
treatment. Gelder, Psychiatry: Oxford Core Texts. Chapter 21: Learning disability
Incorrect
Marks for this submission: 0/1.

Management

1
Marks: 0/1
Which of the following is true of use of psychotropic medications in the learning disability population?
Choose one answer.
a. Naltrexone is an effective antidepressant
b. Levetiracetam is a first line treatment for epilepsy
c. Donepezil is of proven benefit for dementia associated with Downs syndrome
d. Ethosuxamide is a first line antiepileptic
e. Proprananol is an effective antimanic agent
Propranolol has been used for anxiety and aggression in LD. (1). Direct treatment of the dementia itself remains
controversial in Downs syndrome. There has been limited research showing that donepezil, an
acetylcholinesterase inhibitor, is effective in slowing the decline of functional ability in Alzheimers disease, but
at this stage it is uncertain whether similar gains could be achieved in Alzheimers disease associated with
Downs syndrome. (2) ; There has been some evidence that these agents may be useful in this group of patients
(Prasher et al, 2002; Kishnani et al, 2001), V.P. Prasher, A. Huxley The Down Syndrome Ageing Study Group and
M.S. Haque, A 24-week, double-blind, placebo-controlled trial of donepezil in patients with Downs syndrome
and Alzheimers dementia (3) Keppra / levetiracetam is an adjunctive treatment for a wide range of epilepsies
(4) Ethosuximide is a first line treatment for absence seizures Click here for reference
Incorrect
Marks for this submission: 0/1.
Question 2
Marks: 0/1
A 35 year old patient with learning disability complains of swallowing problem. This is a likely side effect of
Choose one answer.
a. Clonazepam
b. Fluoxetine
c. Sodium Valproate
d. Lithium
e. Naltrexone
Dysphagia can be related to pancreatitis which is associated with Valproate. Hinder & Perry, Sodium-Valproate-
Induced Pancreatitis in a Man with Profound Intellectual Disability: the Significance of Diagnostic Difficulties,
Journal of Applied Research in Intellectual Disabilities, Volume 13, Issue 4, Pages 292-297
Incorrect
Marks for this submission: 0/1.
Question 3
Marks: 0/1
A child with learning disability has persistant sleep disturbance, melatonin is being considered as a treatment
option. Which of the following are true?
Choose one answer.
a. Total sleep time is not affected
b. Useful in those with initial insomnia
c. It is contraindicated in this age group
d. Increases daytime sleep
e. There is no effect on sleep
All forms of insomnia are more common in children with learning difficulties, autism, ADHD and sensory
impairments (particularly visual). Although behavioural interventions should be the primary intervention and
have a robust evidence base, exogenous melatonin is now the first-line medication prescribed (off licence
prescription) for childhood insomnia. It reduces the time to falling asleep, increases total sleep time and
promotes normal sleep pattern. Maudsley Prescribing Guidelines, 10th Edn, p 279.
Incorrect
Marks for this submission: 0/1.
Question 4
Marks: 0/1
SIADH is more likely to be associated with
Choose one answer.
a. Lithium
b. Quetiapine
c. Lamotrigine
d. Olanzapine
e. Carbamazepine
Carbamazepine is associated with SIADH in addition to antidepressants and antipsychotics Chan, Drug-induced
syndrome of inappropriate antidiuretic hormone secretion. Causes, diagnosis and management, Drugs Aging.
1997 Jul;11(1):27-44.
Incorrect
Marks for this submission: 0/1.
Question 5
Marks: 0/1
Dietary restriction is useful in the following conditions
Choose one answer.
a. Lesch Nyhan syndrome
b. Prader Willi syndrome
c. Tay-Sachs disease
d. Sturge-Weber syndrome
e. Hurler's syndrome
PWS: Obesity can be caused by hyperphagia, and a decreased calorific requirement owing to low energy
expenditure levels. (Obesity is not normally a feature of those whose food intake is strictly controlled.) LN: The
overall prognosis for this condition remains poor and most affected subjects die in early adulthood. (1) TS: By
the time a child with Tay-Sachs disease is three or four-years old, the nervous system is so badly affected that
life itself cannot be supported. (2) HS: This is a metabolic neurodegenerative disorder - one of the
mucopolysaccahridoses. This disorder has been treated with allogeneic bone marrow transplantation with
good results. (1) Sturge-Weber syndrome (SWS), also called encephalotrigeminal angiomatosis Gelder, Michael
G.; Lopez-Ibor, Juan Jose; Andreasen, Nancy C., New Oxford Textbook of Psychiatry, Volume 2 2003,
Incorrect
Marks for this submission: 0/1.
Question 6
Marks: 0/1
A 35 year old man with LD has been treated with Carbamazepine for localised seizures with secondary
generalisation. The following electrolyte abnormality is a side effect of Carbamazepine
Choose one answer.
a. Hypokalemia
b. Hyperchloremia
c. Hyperkalemia
d. Hyponatremia
e. Hypernatremia
Hyponatremia is seen in 5-40% patients on Carbamazepine , more so with higher serum levels and older
individuals New Oxford textbook of psychiatry,Vol2,p-1328
Incorrect
Marks for this submission: 0/1.

Miscellaneous

1
Marks: 0/1
Which of the following is not true of self injurious behaviour in individuals with learning disabilities
Choose one answer.
a. more common in males
b. high in those with hearing and visual impairment
c. More in older adults
d. higher in those with co-morbid epilepsy
e. Inversely proportional to IQ
It is more common in young adults ,impairment with mobilty and communication and autism. Collacot
RA, Epidemiology of self injurious behaviour in adults with learning disabilites.BJPsych
1998;173:428-432
Incorrect
Marks for this submission: 0/1.
Question 2
Marks: 0/1
In the learning disabled population, normalisation usually refers to
Choose one answer.
a. Treat people with borderline LD like normal individuals
b. Maintain inclusion in mainstream school
c. Provide a pattern of life similar to other people
d. Recognise everyone, no matter what ability, has different needs
e. Reduce the stigma of learning disabilities
The normalisation philosophy originated in Scandinavia in the 1960s and one of the main aims was to
provide for people with mental retardation "an ordinary life" as possible. Philip Graham. Child
Psychiatry A developmental Approach. Oxford Medical Publication. Chapter 2
Incorrect
Marks for this submission: 0/1.
Question 3
Marks: 0/1
Empathy Skills are most likely to be delayed in
Choose one answer.
a. A hearing child adopted for 6 months by deaf parents
b. A deaf child of deaf parents
c. A hearing child of deaf parents
d. A hearing child with one deaf parent
e. A deaf child of hearing parents
The role of Language acquisition in the development of Theory of mind can be deciphered by studying
deaf children with deaf parents and deaf children with hearing parents. Deaf parents are more sensitive
to communicative efforts of their infants, and thus help in early development of language compared to
deaf children with hearing parents. Deaf children outperform both deaf children of hearing parents and
hearing children on false-belief tasks, probably due to early exposure to visual perspective-taking (sign-
language) and develop theory of mind earlier. This shows that language has an important role to play in
acquisition of Theory of mind. Candida C. Peterson & Michael Siegal ,Insights into Theory of Mind
from Deafness and Autism, Mind & Language Volume 15 Issue 1, Pages 123 - 145. AND Schick B., et
. Language and Theory of Mind: A Study of Deaf Children. Child Development, March/April 2007,
Volume 78, Number 2, Pages 376 396.
Incorrect
Marks for this submission: 0/1.
Question 4
Marks: 0/1
The following are associated with Offenders with learning disabilities
Choose one answer.
a. are better treated individually than in groups for sexual offending
b. have a mean age higher than that of normal IQ offenders
c. have similar rates of offending as normal IQ once social class controlled for
d. are over represented in homicides
e. sexual offend more often due to hypersexuality
It is generally considered that the prevalence rates for offenders with intellectual disability may be
higher than those in the general population. This is especially true for sexual offences and arson. Mild
LD has increased petty crime rates. The average age of offenders with LD is higher than for other
offenders. (1) . In non-LD offenders more impressive results are found with group work. In LD
population the programme consists of modifications of programmes used with normal IQ groups. (2).
Low intelligence is an important predictor of offending, and it can be measured very early in life. For
example, in a prospective longitudinal survey of about 120 Stockholm males, Stattin and Klackenberg-
Larsson (1993) reported that low intelligence measured at age 3 significantly predicted officially
recorded offending up to age 30. All of these results held up after controlling for social class. Also, in
the Providence (Rhode Island) site of the National Collaborative Perinatal project, Lipsitt et al. (1990)
showed that low IQ at age 4 predicted later juvenile delinquency. (3) (1) M. K. Simpson, J. Hogg
(2001) Patterns of offending among people with intellectual disability: a systematic review. Part II:
predisposing factors. Journal of Intellectual Disability Research 45 (5), 397406. (2) Day K. Male
mentally handicapped sex offenders. British Journal of Psychiatry 1994; 165: 630-9 Low intelligence is
an important predictor of offending, and it can be measured very early in life.
Incorrect
Marks for this submission: 0/1.

LD - EMIs

1
Marks: 0/3

GENETIC DISORDERS IN LEARNING DISABILITIES.

A. Cri-du Chat Syndrome


B. Down‘s Syndrome
C. Edward Syndrome
D. Fragile X
E. Hurler‘s Syndrome.
F. Lesch Nyhan
G. Patau syndrome
H. Prader Willi syndrome
I. Rett‘s syndrome
J. Turner‘s Syndrome

Lead in: Choose one disorder for each of the following clinical description
1. A child with LD presenting with Obsessional eating
2. A child with presenting with loss of previous learned behaviour and hand wringing movements.
3. Self harm in a 2 year old boy who is seen to be involved in biting himself.

1. H Prader Willi syndrome. Also known as HHHO (hypotonia, hypogonadism, hypomentia, obesity)
syndrome. Obesity is due to excessive appetite and overeating (hyperphagia)

2. I Retts syndrome. Apparently normal or near-normal early development is followed by partial or


complete loss of acquired hand skills and acquired fine motor manipulative. There are distinctive
stereotyped tortuous wringing or "hand-washing" movements, with the arms flexed in front of the chest
or chin;

3. F Lesch Nyhan syndrome. Behavioral disturbances emerge between 2-3 years of age. Persistent self-
injurious behavior (biting the fingers, hands, lips, and cheeks; banging the head or limbs) is a hallmark
of the disease

Incorrect
Marks for this submission: 0/3.
Question 2
Marks: 0/3

GENETIC DISORDERS IN LEARNING DISABILITIES

A. Cri-du Chat Syndrome

B. Down‘s Syndrome

C. Edward Syndrome

D. Fragile X

E. Hurler‘s Syndrome.

F. Lesch Nyhan

G. Patau syndrome

H. Prader Willi syndrome

I. Rett‘s syndrome

J. Turner‘s Syndrome

Lead in: Choose one disorder for each of the following clinical description

1. A boy who appeared normal at birth but developed coarse facies, hepatospenomegaly, and corneal
clourding over the first year of life.
2. A girl with mild LD and a verbal IQ greater than performance IQ
3. Severe mental retardation seen only in girls
1. E Hurlers syndrome.The clinical features of Hurlers syndrome include coarse facies, corneal
clouding, mental retardation, hernias, dysostosis multiplex, and hepatosplenomegaly. Children with
Hurler syndrome appear normal at birth and develop the characteristic appearance over the first years
of life

2. J Turners syndrome. Has a frequency of about 1 in 1,800 girls. In the UK, there are about 12,000
affected girls and women. Overall intelligence is unimpaired or there may be mild LD. Verbal skills are
preserved and there are specific deficits in visuo-spatial tasks, visual memory, and arithmetic. Hence
verbal IQ is greater than performance IQ.

3. I Rett syndrome. Rett syndrome occurs in one of every 10,000 to 15,000 live female births. It is
caused by mutations (structural alterations or defects) in the MECP2 gene, which is found on the X
chromosome. Boys with the MECP2 mutation only have one X chromosome, and no back-up copy that
could compensate for the defective one. Boys with such a defect die shortly after birth.

Incorrect
Marks for this submission: 0/3.

Forensic Psychiatry MCQs


1
Marks: 0/1
What percentage of people charged with sexual offences have no previous history of sexual offences?
Choose one answer.
a. 10%
b. 20%
c. 100%
d. 50%
e. 80%
Percentage of people charged with sexual offences having no previous history of sexual offence is around 40-
50%
Soothill BJCriminology1978 18 267-76
Incorrect
Marks for this submission: 0/1.
Question 2
Marks: 0/1
Of the people who have had a court appearance for indecent exposure, what percentage have no previous
convictions?
Choose one answer.
a. 5%
b. 10%
c. 80%
d. 50%
e. 40%
""
PREDICTING RECONVICTIONS FOR SEXUAL AND VIOLENT OFFENCES USING THE REVISED OFFENDER GROUP
RECONVICTION SCALE. Home Office Research, Development and Statistics Directorate. Indecent Exposure in
the West Midlands (From Sex Offenders in the Criminal Justice System, P 171-180, 1980, by D J West - See NCJ-
70970)
Incorrect
Marks for this submission: 0/1.
Question 3
Marks: 0/1
The court may be asked to determine an offender's fitness to plead. A finding of Not fit to plead
Choose one answer.
a. may result in a hospital disposal
b. is the outcome if the accused suffers amnesia for the offence
c. is determined by the MacNaughton rules
d. is commonly found in those with mental illness
e. only applies to those with schizophrenia
Once a defendant is found unfit to plead, a range of sentencing options are available including hospital orders.
See Criminal Procedure (Insanity and Unfitness to Plead) Act 1991
Click here for reference
Incorrect
Marks for this submission: 0/1.
Question 4
Marks: 0/1
Which ONE of the following statements is true when the court imposes a hospital order?
Choose one answer.
a. The hospital order can be given for a set period of time
b. The hospital order can only be given by a Crown Court judge
c. The hospital order may at any time be converted to a prison sentence
d. The hospital order may be used by the court as an alternative to a prison sentence
e. The hospital order necessitates admission to a medium-secure hospital
Hospital orders are alternatives to prison sentence or any other disposal. They can be given by Magistrates or
Crown Court. They are not time-limited, and do not necessarily require medium secure hospitals - they can be
made to general psychiatric hospitals in certain circumstances
""
Incorrect
Marks for this submission: 0/1.
Question 5
Marks: 0/1
Compared to male prisoners, female prisoners show a LOWER rate of
Choose one answer.
a. neurotic conditions
b. psychotic disorders
c. substance misuse
d. personality disorder
e. conviction for drug offences
Women in prison show higher rates of neurotic conditions, personality disorder and substance misuse than
men. They are less likely to be serving a sentence for violence or burglary but are more likely to have
committed a drug offence, theft, fraud or deception. Prison has become a receptacle for some very disturbed
women with severe personality disorders, many of whom have previously been rejected for admission to
secure hospital facilities.
Companion to Psychiatric Studies (Johnstone et al). Maden et al (1990). Gorsuch (1999)
Incorrect
Marks for this submission: 0/1.
Question 6
Marks: 0/1
Which personality disorder is most strongly associated with violent offending?
Choose one answer.
a. Borderline
b. Antisocial
c. Narcissistic
d. Dependent
e. Paranoid
Antisocial PD by definition encompasses those who do not conform to social norms and standards (such as
legal rules) and strongly associated with violent offending.
Click here for reference
Incorrect
Marks for this submission: 0/1.
Question 7
Marks: 0/1
Which ONE of the following groups of patients is most likely to cause disagreement between psychiatrists
regarding risk?
Choose one answer.
a. patients from minority ethnic communities
b. patients without mental capacity who refuse life-sustaining treatments
c. patients who are mentally ill but accepting treatment
d. patients presenting with psychosis
e. patients presenting with personality disorder
Risk assessment forms an integral part of clinical practice. Traditionally, risk has been portrayed as a binary
concept, and its assessment regarded as a test that can be correctly or incorrectly classified. Nowadays, risk
assessments are less straightforward than is commonly perceived and are often complicated by multiple forms
of uncertainty. These uncertainties arise where psychiatrists are unsure about their interpretation of
information, where information is missing, or where interpretation of the risk situation is open to challenge.
They centre on doubts about the accuracy and the defensibility of assessment of patients' risk status and the
need for risk containment.
Dixon & Oyebode (2007) Adv Psych Treat 13:70-78
Incorrect
Marks for this submission: 0/1.
Question 8
Marks: 0/1
Aggressive antisocial behaviour has been shown to be associated with
Choose one answer.
a. normal dopaminergic function
b. low levels of serotonin
c. high levels of testosterone
d. low levels of noradrenaline
e. high levels of cortisol
Recent studies in adolescence suggest that there is no evidence that physical aggression is related to high
testosterone levels. Instead, testosterone was is more related to social dominance, with the assumption that
that the behaviours associated with dominance vary according to social context. There is an inverse
relationship between serotonergic function and aggression. Ventral striatal damage affects dopaminergic
transmission and impairs the recognition of human signals of aggression. There is no published literature
showing a link between noradrenaline and aggression in humans. Recent research has found lower cortisol
levels in violent adults, and suggested that cortisol may moderate the relationship between testosterone and
aggression
Rowe (2004) Volavka (2002) Calder (2004) Poprna (2007)
Incorrect
Marks for this submission: 0/1.
Question 9
Marks: 0/1
The most common psychiatric diagnosis in prisoners is
Choose one answer.
a. antisocial personality disorder
b. borderline personality disorder
c. depression
d. schizophrenia
e. learning disability
A systematic review of 23000 prisoners in 12 countries showed 4% had psychotic illness, 10% had major
depressive disorder, 47% of male and 21% of female prisoners had an antisocial personality disorder
Fazel & Danesh (2002)
Incorrect
Marks for this submission: 0/1.
Question 10
Marks: 0/1
Exhibitionism or indecent exposure is a non-indictable sexual offence. Exhibitionism
Choose one answer.
a. is usually associated with mental illness
b. is common in older men
c. is the commonest sexual offence in men
d. usually proceeds to more serious forms of sexual offending
e. is equally prevalent in men and women
Exhibitionism is the commonest sexual offence in men, who are usually relatively young (25-35). Offenders do
not routinely proceed to more serious forms of offending. Women may be perpetrators.
Click here for reference
Incorrect
Marks for this submission: 0/1.
Question 11
Marks: 0/1
Which of the following is the most effective management for delusional jealousy?
Choose one answer.
a. marital therapy
b. cognitive behavioural therapy
c. day hospital treatment
d. separation of the couple
e. antipsychotic medication
Delusional jealousy has a high association with violence and homicide, and responds poorly to medication and
psychological therapy. Separation of the couple is often required, although this does not eliminate the risk of
future violence
Shepherd (1961). Companion to Psychiatric Studies (Johnstone et al)
Incorrect
Marks for this submission: 0/1.
Question 12
Marks: 0/1
Which of the following is the LEAST likely to be associated with development of antisocial behaviour?
Choose one answer.
a. being born to a teenage mother
b. low maternal IQ
c. being an only child
d. maternal mental health problems
e. prenatal maternal smoking
Children of teenage mothers are at risk for a range of negative outcomes, including unemployment, violent
offending, early parenthood and early school leaving. Prenatal maternal smoking is associated with antisocial
behaviour in the child, although there are doubts about the causal status of this association. Low maternal IQ
and maternal mental health problems are also linked aetiologically to antisocial behaviour.
Jafee (2001) for adverse outcomes for children of teenage mothers. Maughan (2004) for link between prenatal
smoking and child antisocial behaviour.
Incorrect
Marks for this submission: 0/1.
Question 13
Marks: 0/1
Suicides in prison are more likely to occur
Choose one answer.
a. in remand prisoners
b. towards the end of the prison term
c. in those suffering from personality disorder
d. in those with previous offending history
e. in sentenced prisoners
Suicide is most common in remand prisoners early in their prison stay. Violent means (e.g. hanging) are
commonly used. Often there is no warning of self-harm, but reception procedures are notoriously bad at
correctly identifying mental illness
Shaw et al, 'Suicide by prisoners', British Journal of Psychiatry, (2004) 184: 263-267
Incorrect
Marks for this submission: 0/1.
Question 14
Marks: 0/1
Which ONE of the following statements is true for offending behaviour In those with learning disability?
Choose one answer.
a. interpersonal violence is common and serious
b. there is no difference in offending as compared to the general population
c. homicide is frequently seen
d. offences against property are uncommon
e. sexual offending is more common
Sexual offending is overrepresented in those with learning disability, and is usually opportunistic with low
specificity in the age and sex of the victim. Homicide is rare and interpersonal violence usually minor, although
overall the conviction rate is higher in those with LD. Property offending predominates.
Gelder et al (2006). Shorter Oxford Textbook of Psychiatry. P744. Green et al (2002)
Incorrect
Marks for this submission: 0/1.
Question 15
Marks: 0/1
Violence perpetrated by those with major mental illness is
Choose one answer.
a. unrelated to substance misuse
b. likely to be targeted at strangers
c. becoming more common
d. less common in those with psychotic symptoms
e. more prevalent than in community controls
Taylor and Gunn have shown that the number of people with mental illness committing homicide has remained
remarkably consistent between 1957 and 1995. The MacArthur Violence Risk Assessment study showed that
violence amongst the mentally ill is related to substance misuse. Birth cohort studies have shown a significantly
greater rate of violent crime amongst those with mental disorder compared to controls. Violence is more likely
in those with current psychotic symptoms.
Hodgins et al (1996) Steadman et al (1998) Link et al (1992)
Incorrect
Marks for this submission: 0/1.
Question 16
Marks: 0/1
A restriction order may be imposed by the Crown Court
Choose one answer.
a. in order to ensure a patient's compliance with treatment
b. in order to protect the public from serious harm
c. in the most serious of offences only
d. whether or not the patient is made subject to a hospital order
e. in order to admit a patient to a high-secure hospital
A Crown Court may make a restriction order in order to restrict the patient's discharge, transfer or leave of
absence from hospital for a specified or unlimited period without the consent of the Justice Secretary. The
restriction order may only be made once a hospital order has been made. A restriction order has no existence
independently of the hospital order to which it relates. There is no direct relationship between the seriousness
of offence and the probability that a restriction order will be made.
Mental Health Act Manual (Jones)
Incorrect
Marks for this submission: 0/1.
Question 17
Marks: 0/1
Which of the following is the MOST likely causative factor in the development of juvenile delinquency
Choose one answer.
a. low levels of MAO-A activity
b. XYY syndrome
c. low intelligence
d. low mood
e. living in an inner city area
There is a long-established link between low intelligence and delinquency, which isn't necessarily due to
offenders with lower IQ being caught more often. Instead, low intelligence is probably related to general
neuropsychological deficits. An association reported in the 1960's of an extra Y chromosome in some offenders
has later been discredited. Boys with low levels of MAO-A were more likely to be antisocial, but only if they had
also been maltreated and abused as children.
Caspi et al (2002) on link between MOA-A activity and antisocial behaviour. Borgaonkar & Shah (1974) for
association between XYY and offending.
Incorrect
Marks for this submission: 0/1.
Question 18
Marks: 0/1
The delusional conviction of being loved by someone who is identified but unattainable is the basis of
Choose one answer.
a. Cotard's syndrome
b. Ekbom's syndrome
c. de Clerambault's syndrome
d. Fregoli syndrome
e. Othello syndrome
De Clerambault's syndrome is a manifestation of erotomania, where the subject, usually a woman, believes
that an identified person is in love with her. The supposed lover is usually inaccessible.
""
Incorrect
Marks for this submission: 0/1.
Question 19
Marks: 0/1
A mentally disordered offender may be considered to be unfit to plead due to
Choose one answer.
a. being intoxicated at the time of the offence
b. being unable to understand and follow the evidence of court procedure
c. being unable to give oral evidence in court
d. the fact that they were suffering from schizophrenia at the time of the offence
e. suffering from a personality disorder
At times a defendant because of a mental disorder may lack the capacity to defend himself and is said to be
unfit to plead. There are five tests to determine fitness to plead, arising mainly from R v Pritchard. A defendant
must be able to understand the charge, distinguish between a plea of guilty and not guilty, instruct counsel,
follow proceedings in court, challenge a juror.
R v Pritchard (1836)
Incorrect
Marks for this submission: 0/1.
Question 20
Marks: 0/1
The most common psychiatric diagnosis in violent offenders is
Choose one answer.
a. mania
b. substance misuse
c. learning disability
d. depression
e. schizophrenia
Recent methodologically sophisticated studies suggest that mental illness is a risk factor for violence. Although
this is statistically significant, the absolute risk is relatively small. Substance misuse is the commonest diagnosis.
Comorbid mental illness and substance misuse significantly increases the risk of violence. Antisocial personality
disorder and substance misuse disorders have the greatest association with offending.
MacArthur Foundation risk assessment study and the ECA survey
Incorrect
Marks for this submission: 0/1.
Question 21
Marks: 0/1
Which of the following is the LEAST likely to be implicated in the aetiology of criminal behaviour?
Choose one answer.
a. Hyperactivity in childhood
b. Childhood antisocial behaviour
c. Family criminality
d. Orbitofrontal cortex dysfunction
e. Chromosomal abnormalities
There is no good evidence to connect chromosomal abnormalities directly to criminal behaviour. Old studies on
populations of XYY males which demonstrated increased aggression have since been discredited.
New Oxford Textbook of Psychiatry, Ed: Gelder, Lopez-Ibor Jr & Andreasen, 2000 edition P932
Incorrect
Marks for this submission: 0/1.
Question 22
Marks: 0/1
Which ONE of the following statements is true regarding the relationship between Epilepsy and offending
behaviour? Epilepsy
Choose one answer.
a. is rarely connected with violence occurring during seizures
b. frequently results in offending during seizures
c. is generally accepted to be more prevalent in prisoners compared to the control population
d. can be used as justification for a hospital order
e. frequently allows for a defence of automatism
Early research suggested a preponderance of epilepsy in prisoners, but there were flaws in the methodology
and it is far from being generally accepted to be the case. Violence or other offending during seizures is rare.
Epilepsy is not a mental illness and so cannot be used for hospital disposal.
Click here for reference
Incorrect
Marks for this submission: 0/1.
Question 23
Marks: 0/1
The House of Lords
Choose one answer.
a. is the highest court in the UK at which cases may be heard
b. employs a jury to decide on complex cases
c. restricts itself to criminal cases
d. only deals with legislative matters
e. is subordinate to the High Court
The House of Lords is the highest UK court, and hears criminal or civil cases. It is the only court which can
overrule the Court of Appeal, and is composed of highly trained individuals from a range of backgrounds.
Click here for reference
Incorrect
Marks for this submission: 0/1.
Question 24
Marks: 0/1
The plea of Not guilty by reason of insanity
Choose one answer.
a. is determined by the Pritchard criteria
b. may no longer be used as a defence
c. only applies to homicide cases
d. carries an automatic hospital disposal
e. is a rare outcome in criminal cases
Not guilty by reason of insanity is rarely used, being determined by the MacNaughton rules. It can be used in
any offence, not only homicide. Homicide cases are usually dealt with by means of pleading guilty to
manslaughter on the grounds of diminished responsibility.
""
Incorrect
Marks for this submission: 0/1.
Question 25
Marks: 0/1
The UK 700 case management trial showed that
Choose one answer.
a. intensive case management of psychotic patients results in improvements in clinical functioning
b. intensive case management of psychotic patients is similar in cost to standard care
c. intensive case management of psychotic patients reduces overall hospital use
d. intensive case management of psychotic patients reduces violent incidents compared to standard
treatment
e. intensive case management of psychotic patients results in improvements in social functioning
""
Burns et al (1999)
Incorrect
Marks for this submission: 0/1.
Question 26
Marks: 0/1
Which of the following factors is most associated with reoffending in paedophiles?
Choose one answer.
a. Depressed mood
b. Sexual deviance
c. Comorbid substance misuse
d. Lack of victim empathy
e. Previous violent offending
A meta-analysis of 82 recidivism studies (1,620 findings from 29,450 sexual offenders) identified deviant sexual
preferences and antisocial orientation as the major predictors of sexual recidivism for both adult and
adolescent sexual offenders. Many of the variables commonly addressed in sex offender treatment programs
(e.g., psychological distress, denial of sex crime, victim empathy, stated motivation for treatment) had little or
no relationship with sexual or violent recidivism.
RK Hanson, MT Bussiere. The characteristics of persistent sexual offenders: a meta-analysis of recidivism
studies. JOURNAL OF CONSULTING AND CLINICAL PSYCHOLOGY, 2005 Dec;73(6):1154-63.
Incorrect
Marks for this submission: 0/1.
Question 27
Marks: 0/1
Which ONE of the following statements is true for the HCR-20 tool?
Choose one answer.
a. it is designed for use in the learning-disability population
b. it is designed for assessment of sexual violence
c. it produces a percentage probability for future violence
d. it is a structured professional judgement approach to risk assessment
e. it has a total of 60 items - 20 for each domain
The HCR-20 is composed of a total of 20 historical, clinical and risk items. It may be used by any discipline with
appropriate training. It offers a structured professional approach towards risk assessment and management,
and is not intended to be used as an actuarial tool to calculate likelihood of reoffending. The similar SVR-20 is
used for assessing sexual violence.
HCR-20 (Assessing Risk for Violence) Version 2 - User Manual
Incorrect
Marks for this submission: 0/1.
Question 28
Marks: 0/1
The prevalence of psychotic disorders in prisons compared to the general population is:
Choose one answer.
a. 10 times
b. 40 times
c. 50 times
d. 5 times
e. 20 times
72% of male and 70% of female sentenced prisoners suffer from two or more mental health disorders. One in
five prisoners have four of the five major mental health disorders. A significant number of prisoners suffer from
a psychotic disorder. 7% of male and 14% of female sentenced prisoners have a psychotic disorder; 14 and 23
times the level in the general population.
Mental health consultations in a prison population: a descriptive study. BMC Psychiatry 2006,
6:27doi:10.1186/1471-244X-6-27
Incorrect
Marks for this submission: 0/1.
Question 29
Marks: 0/1
Which of the following is an actuarial risk assessment tool?
Choose one answer.
a. Violence Risk Scale (VRS)
b. Sexual Violence Risk-20 (SVR-20)
c. Historical Clinical Risk-20 (HCR-20)
d. Violence Risk Appraisal Guide (VRAG)
e. Spousal assault Risk Assessment guide (SARA)
VRAG is an actuarial measure of violent recidivism, developed from a study of high-security psychiatric patients
in Canada. HCR-20, SARA and SVR-20 are structured clinical approaches to risk assessment. VRS is a dynamic
measure of change in risk.
""
Incorrect
Marks for this submission: 0/1.
Question 30
Marks: 0/1
What is the best predictor of future sexual offences?
Choose one answer.
a. Lack of victim empathy
b. Mental illness
c. Past psychiatric history
d. Previous non-sexual offences
e. History of illicit drug use
Good predictors of future offending include lack of victim empathy amongst various other factors
""
Incorrect
Marks for this submission: 0/1.
Question 31
Marks: 0/1
Which ONE of the following statements is true for the crime of arson?
Choose one answer.
a. arson is strongly associated with mental illness
b. arson is treated as a relatively minor offence
c. arson is always punished with a mandatory prison sentence or hospital order
d. arson has a high rate of re-offending
e. arson can be difficult to successfully prosecute
Arson can be difficult to prosecute as the evidence is often consumed by the fire, leaving little evidence of what
took place. Recidivism is around 10%. Arson does not have a strong association with any particular mental
illness, and overall it is regarded as a very serious offence by the Courts.
Gelder et al (2006). Shorter Oxford Textbook of Psychiatry. P744
Incorrect
Marks for this submission: 0/1.
Question 32
Marks: 0/1
Offending in learning disabilities
Choose one answer.
a. Sexual offending is related to hypersexuality
b. Offending is more likely in severe LD
c. Conviction for arson leads to a fixed prison sentence
d. It is over represented in borderline and mild LD
e. Fire setting is the most common offence
Crimes of personal violence are low in frequency, sex offences and arson are over represented in LD population
Hodgins Crim Justice and Behaviour 24 432-454
Incorrect
Marks for this submission: 0/1.
Question 33
Marks: 0/1
The association between schizophrenia and violent behaviour
Choose one answer.
a. should be calculated after allowing for the effects of
mediating influences
b. is statistically, but not clinically or socially, significant
c. may account for up to 10% of violent crime, including homicide
d. is primarily the result of active symptoms such as delusions
and hallucinations
e. is not statistically significant enough to merit serious attention by clinicians
There is a correlation between having a schizophrenic syndrome and increased rates of antisocial behaviour in
general and violence in particular. The evidence that such associations are not just statistically but clinically and
socially significant is now overwhelming. Studies suggest that in prisons throughout the Western world 5-10%
of those awaiting trial for murder will have a schizophrenic disorder. Conversely, clinicians may never see a
patient who has committed a homicide or serious act of interpersonal violence. Up to 10% of homicide
offenders may have schizophrenia, but the annual risk that a person with schizophrenia will commit a homicide
is in the region of 1 in 10 000 and that of acquiring conviction for violence is 1 in 150.
Mullen (2006) Adv Psych Treat 12:239-248
Incorrect
Marks for this submission: 0/1.
Question 34
Marks: 0/1
Which ONE of the following is a correct feature of Hare's Psychopathy Checklist-Revised (PCL-R)?
Choose one answer.
a. it consists of a checklist of traits that can be administered without specific training
b. it is a tool which is routinely used on homicide perpetrators
c. it is a clinical rating scale of 20 items to assess psychopathy
d. it has only been validated in adult male forensic populations
e. it produces a score for psychopathy which is required by the courts in determining disposal
The PCL-R is long and complex, and requires exhaustive training to administer and interpret. Its role is
controversial, and it is only helpful in a relatively small group of patients. It is not used routinely for any
particular offence. There now is enough empirical evidence to support the validated use of PCL-R with male,
female and adolescent offenders, as well as with sex offenders.
Click here for reference
Incorrect
Marks for this submission: 0/1.

Old Age Psychiatry

The Ageing Process

1
Marks: 0/1
Many theories have been postulated concerning the biology of normal aging. Which one of the
following statements explains the 'Hayflick Limit'?
Choose one answer.
a. Chemical and physical properties of macromolecules are distorted via cross-linkage with
increasing chronological age
b. There is an inverse correlation between rates of peroxidation and the longevity of mammalian
species
c. Human diploid cells exhibit a finite lifespan when cultured in the laboratory
d. Lipofuscin indicates an age-related decline in cellular catabolic function
e. There is an inverse relationship between basal metabolic rate and longevity
Links between rates of peroxidation and longevity refers to Stochastic theories concentrating on the
role of free radical damage in aging. The Hayflick limit refers to the phenomenon that cells undergo
irreversible cessation of mitosis and enter a non-dividing state known as replicative senescence. Rate-
of-living theory refers to relationship between basal metabolic rate and longevity. Accumulation
theories ascribe aging to the build up of waste products within cells. Lipofuscin is a highly insoluble
compound which may be an indication of poor cell function. This explains cross-linkage theory.
Psychiatry in the Elderly, eds Jacoby and Oppenheimer, 1996, p10-14
Incorrect
Marks for this submission: 0/1.
Question 2
Marks: 0/1
Which one of these pharmacokinetic changes is not seen with ageing?
Choose one answer.
a. Reduced GFR
b. Reduced total body mass
c. Increased rate of gastric emptying
d. Reduced body water
e. Increased gastric pH
In older people there is a decreased rate of gastric emptying and reduced secretion of gastric acid
leading to drugs being absorbed slowly and slower onset of action. The elderly have more body fat, less
body fat and less albumin than younger adults leading to increased volume of distribution, longer
duration of action and higher concentrations of some drugs. Though the liver size is reduced, there is no
reduction in metabolic capacity. Renal function is reduced (35% by age 65 and 50% by age 80) leading
to accumulation of drugs primarily excreted via the kidneys (lithium & Sulpiride) leading to toxicity
and side effects.
The Maudsley Prescribing guidelines, 9th edition, page 412, 2007
Incorrect
Marks for this submission: 0/1.
Question 3
Marks: 0/1
In an 80 year old patient compared to a 40 year old patient you would expect the half-life of temazepam
to be:
Choose one answer.
a. The same
b. Increased by 100%
c. Reduced by 50%
d. Increased by 200%
e. Increased by 50%
Half life of drugs is usually prolonged in older people because of increased rate of metabolism in liver
and decreased renal function. The half life in younger people is usually around 8-10 hours and this is
increased in the elderly to around 15 hours, (increase by around 50%).
""
Incorrect
Marks for this submission: 0/1.
Question 4
Marks: 0/1
In the normal aging process, which one of the following statements is correct?
Choose one answer.
a. There is a decrease in slow wave sleep
b. There is a relative decrease in body fat
c. Longevity is highly heritable
d. Increased theta activity and slowing of delta activity are observed in the EEG.
e. Drugs have shorter half lives
Normal ageing is associated with a reduction in total sleep time, reduced slow wave sleep and daytime
fatigue. The half life of drugs are often increased in the elderly, due to increased body fat and reduced
renal clearance. There is a relative increase in body fat. The EEG in normal aging shows increased
theta and delta activity, as well as slowing of alpha. Twin studies looking at longevity produced low
heritability estimates of 0.26 for males and 0.23 for females.
Breslau J et al (1989) Topographic EEG changes with normal aging and SDAT. Electroencephalogr
Clin Neurophysiol. 72(4), 281-9. Herskind et al (2005). The heritability of human longevity: A
population-based study of 2872 Danish twin pairs born 1870-1900. Human Genetics. 97(3), 319-323
Incorrect
Marks for this submission: 0/1.

Alzheimer's Disease
1
Marks: 0/1
Which ONE of the following radiological or histological features does not support a diagnosis of Alzheimer's
Disease?
Choose one answer.
a. Sparing of medial temporal structures
b. Generalised cortical atrophy
c. Hirano bodies
d. Neurofibrillary tangles
e. Proliferation of astrocytes (gliosis)
Medial temporal atrophy is the most consistent and recognised structural difference in AD compared with age
matched controls. Generalised cortical atrophy is seen in AD. Are intracellular crystalline deposits found in AD.
The cardinal features of AD are neurofibrillary tangles and amyloid plaques. In addition to NFTs and plaques,
gliosis and loss of synapses occur.
Shorter Oxford Textbook of Psychiatry, eds Gelder, Harrison, Cowen, 5th Edition, p336. Dementia, eds Burns,
O'Brien, Ames. 2005, 3rd Edition p87-88
Incorrect
Marks for this submission: 0/1.
Question 2
Marks: 0/1
In which ONE of the following scenarios does NICE advocate use of acetylcholinesterase inhibitors in dementia?
Choose one answer.
a. patient has mild or moderate Alzheimer's Disease
b. treatment of vascular dementia
c. diagnosis made by GP
d. patient has severe Alzheimer's disease
e. MMSE score between 10 and 20
Patients must have moderate Alzheimer's Disease only (MMSE between 10 and 20). Acetylcholinesterase
inhibitors are not licensed for the treatment of vascular dementia. This is true - Acetylcholinesterase inhibitors
are only licensed for Alzheimer's Disease. Alzheimer's Disease must be diagnosed in a specialist clinic(
psychiatrists, neurologists, physicians with a special interest in care of the elderly). The drug should be
continued only while the patient's MMSE score is above 10.
NICE Guidelines. November 2006
Incorrect
Marks for this submission: 0/1.
Question 3
Marks: 0/1
Apolipoprotein E (ApoE) is associated with an increased risk of developing late-onset Alzheimer's Disease.
Which one of the following is correct concerning ApoE E?
Choose one answer.
a. The ApoE e4 allele is commonest in East Asian individuals.
b. Genetic testing is useful for counselling relatives
c. The ApoE genotype e2/e2 is associated with an increased risk of AD
d. ApoE is located on Chromosome 19
e. The ApoE genotype e4/e4 is associated with a decreased risk of AD
Clinically the e4/e4 allele increases the risk of developing AD at any particular age. ApoE E is located on
Chromosome 19. ApoE status is not used in genetic counselling as it is not sensitive or specific enough to be of
use. The ApoE genotype e2/e2 may confer protection against the development of AD. Studies have shown the
frequency of ApoE e4 to be significantly lower in East Asian individuals.
Dementia, eds Burns, O'Brien, Ames. 2005, 3rd Edition, p431
Incorrect
Marks for this submission: 0/1.
Question 4
Marks: 0/1
Acetylcholinesterase (AchE) Inhibitors are the mainstay of treatment for Alzheimer's Disease. Which ONE of the
following is correct?
Choose one answer.
a. Donepezil and galantamine are both administered twice-daily
b. Rivastigmine is a pseudo-reversible inhibitor of acetylcholinesterase and butylcholinesterase
c. Donepezil has low specificity for acetylcholinesterase
d. Memantine is an antagonist at muscarinic cholinergic receptors
e. Galantamine exerts an effect on muscarinic cholinergic receptors
Memantine acts as an antagonist at N-Methyl-D-aspartate (NMDA) receptors. Donepezil has high specificity for
acetylcholinesterase. Rivastigmine is a pseudoreversible inhibitor of acetylcholinesterase and
butylcholinesterase (AChE and BuChE). Galantamine is a reversible inhibitor of AchE, and also modulates
nicotinic receptor functioning. Donepezil and galantamine are selective inhibitors of AchE. Donepezil is
administered once-daily.
Taylor, Paton and Kerwin. Maudsley Prescribing Guidelines 2006-2007, 9th Edition. Page 415.
Incorrect
Marks for this submission: 0/1.
Question 5
Marks: 0/1
How do you differentiate that a dementia is subcortical not cortical:
Choose one answer.
a. Calculation is preserved
b. Euthymic mood
c. Absence of dysarthria
d. Mild aphasia
e. Normal speed of cognitive processes
Subcortical dementias are characterised by: no aphasia, dysarthria, visuospatial abilities damaged, calculation
preserved till end, apathetic personality, depressed mood, co-ordination damaged, motor speed slowed +
chorea and tics
Organic Psychiatry,Lishman,3 rd Edition P 452
Incorrect
Marks for this submission: 0/1.
Question 6
Marks: 0/1
A histological finding in the brain of patients with Alzheimer's Disease are Neurofibrillary tangles (NFTs), which
play an important role in neurodegeneration. Which one of the following statement concerning NFTs is
correct?
Choose one answer.
a. NFTs are extracellular inclusion bodies
b. Tau is found in the unstable highly phosphorylated form.
c. Are pathognomonic of Alzheimer's Disease.
d. NFTs are commonly found in the medial temporal structures, but rarely occur in the hippocampus and
amygdala.
e. The degree of cognitive impairment correlates to the number of NFTs.
NFTs are intracellular inclusions composed of ubiquinated and phosphorylated tau. Tau is found in the highly
stable phosphorylated form.
There is a positive correlation concerning the number of NFTs and the severity of dementia. NFTs are common
in the medial temporal structures, in the hippocampus, amygdala and parahippocampal gyrus, as well as
occurring throughout the neocortex and the deep grey matter including the lentiform nucleus, mamillary
bodies and substantia nigra. NFTs occur in other conditions as well as in the normal aging process.
Organic Psychiatry, eds Lishman, Third edition, p442. Dementia, eds Burns, O'Brien, Ames. 2005, 3rd Edition,
p398
Incorrect
Marks for this submission: 0/1.
Question 7
Marks: 0/1
The risk of developing Alzheimer's disease is low with
Choose one answer.
a. Cigarette smoking
b. Chronic inflammation
c. Traumatic brain injury
d. Higher educational level
e. Female sex
A higher education level is a protective factor against AD, perhaps due to increased synaptic and/or dendritic
complexity attendant to learning demands. Low educational attainment has been associated with a doubling of
AD incidence.
Companion to Psychiatric Studies, 7th edition, eds Johnstone, Cunningham Owens, Lawrie, Sharpe, Freeman,
p631.
Incorrect
Marks for this submission: 0/1.
Question 8
Marks: 0/1
There is a well established relationship between Down's syndrome and Alzheimer's Disease. Which gene is
associated with the development of AD in Down's Syndrome?
Choose one answer.
a. Presenilin 1 gene
b. - amyloid precursor protein gene
c. Apolipoprotein E gene
d. ATP 7B gene
e. Presenilin 2 gene
Presenilin 1 gene is implicated in the development of early onset AD. ApoE gene is associated with an increased
risk of developing late-onset AD. APP gene is located on Chromosome 21. Patients with Down's Syndrome
(trisomy 21) develop dementia due to an extra copy of APP. Presenilin 2 gene is implicated in the development
of early onset AD. Wilson's Disease is caused by a mutation in the APT 7B gene located on Chromosome 13
(Autosomal recessive condition).
Shorter Oxford Textbook of Psychiatry. Gelder, Harrison, Cowen. 2007, 5th Edition, p 337. Dementia, eds Burns,
O'Brien, Ames. 2005, 3rd Edition, p799
Incorrect
Marks for this submission: 0/1.
Question 9
Marks: 0/1
Which ONE of the following is a feature of Alzheimer's disease?
Choose one answer.
a. triphasic spikes on EEG
b. localised temporal lobe atrophy
c. lacunar lesions
d. loss of dopaminergic neurones in substantia nigra
e. neuritic plaques
Lacunar lesions are seen in vascular dementia. Localised temporal lobe atrophy would be a sign of fronto-
temporal dementia. Triphasic spikes on EEG are a sign of CJD. Loss of dopaminergic neurones in the substantia
nigra is associated with Parkinson's disease, dementia of Parkinson's disease and Lewy Body dementia.
Organic Psychiatry,Lishman,3 rd Edition P 440
Incorrect
Marks for this submission: 0/1.
Question 10
Marks: 0/1
In the management of noncognitive symptoms of dementia in the elderly
Choose one answer.
a. Psychotherapy services are easily accessible
b. Both psychotropic medication and non pharmacological approaches are useful when used together
from first onset of symptoms
c. Psychotherapeutic approaches can reduce the need for psychotropic medications
d. Non pharmacological interventions should be used when medications are not effective
e. Cognitive behavioural therapies are less likely to be of help in controlling symptoms
NICE recommends the use of non pharmacological approaches in the management of non cognitive
behavioural symptoms in the first instance unless there is a risk of harm to self or others, or sever distress in
which case medication can be used.
NICE Guidelines. November 2006
Incorrect
Marks for this submission: 0/1.
Question 11
Marks: 0/1
Epidemiological studies of Alzheimer's Disease have shown which one of the following to be correct?
Choose one answer.
a. The incidence of Alzheimer's Disease and dementia is lower in North America and Europe compared
with East Asian Countries
b. The prevalence of Alzheimer's Disease differs in urban compared to rural settings
c. Alzheimer's Disease affects approximately 10% of the over 65s
d. The prevalence of Alzheimer's Disease is higher in females compared to males
e. The prevalence of Alzheimer's Disease will more than triple over the next 50 years
AD is apparently more common in urban settings. Prevalence rates for populations over 65 years ranges from
2-7% (moderately -severely affected people). Age-specific prevalence rates are 1% at 65. A meta-analysis of the
incidence of AD found that East Asian countries have a lower incidence of AD than Europe. The prevalence will
double in the next fifty years. The 2007 Dementia UK Report estimates that in 2006-7, approx 560,000 people
had dementia in the UK. It is predicted this will rise to 1.4 million by 2051. There is no association between AD
and sex (male: female ratio).
Baker et al. International Journal of Geriatric Psychiatry 1993; 8, 379-385. Jorm AF, Jolley D (1998). The
incidence of dementia: a meta-analysis. Neurology, 51(3), 728-33. Knapp et al (2007) Dementia UK:Report to
the Alzheimer's Society.
Incorrect
Marks for this submission: 0/1.
Question 12
Marks: 0/1
Which of the following is contraindicated in the elderly with psychosis and cognitive impairment?
Choose one answer.
a. Olanzapine
b. Sulpiride
c. Quetiapine
d. Haloperidol
e. Promazine
In March 2004, the Medicines and Healthcare products Regulatory Agency's Committee on Safety of Medicines
issued a safety warning about the atypical antipsychotic drugs risperidone and olanzapine, advising that these
drugs should not be used for the treatment of behavioural symptoms of dementia. Though, quetiapine was
considered as a risk factor by the MHRA, they felt that there was insufficient evidence to restrict its use in the
treatment of behavioural symptoms in dementia.
Duff G. Atypical antipsychotics and stroke. Http://mhra.gov.uk, 2004
Incorrect
Marks for this submission: 0/1.
Question 13
Marks: 0/1
Cardiac adverse effects of Acetylcholinesterase (ACHE) inhibitors may be caused by
Choose one answer.
a. Parasympathetic stimulation via the vagal nerve
b. increased noradrenaline levels
c. direct cardiotoxicity
d. Decreased acetylcholine levels
e. Cardiac ischemia
AChE inhibitors are known to cause cardiac side effects due to parasympathetic stimulation via the vagus
nerve.
Rowland J. et al. Cardiovascular monitoring with acetylcholinesterase inhibitors: a clinical protocol. Adv
Psychiatr Treatment, 2007 178-84
Incorrect
Marks for this submission: 0/1.
Question 14
Marks: 0/1
In terms of the clinical features of Alzheimer's Disease, which one of the following statements is correct?
Choose one answer.
a. Parietal lobe symptoms in Alzheimer's disease are associated with earlier mortality.
b. Urinary incontinence is more likely to occur in Alzheimer's disease rather than in vascular dementia
c. The amnesia is similar for all decades of life
d. Prosopagnosia means impaired recognition of objects
e. Delusions are frequently grandiose
Parietal lobe damage is associated with diminished survival. Amnesia is characteristically for recent memories
initially before remote memories are lost. Urinary incontinence is more suggestive of vascular dementia
(NINDS-AIREN criteria). Delusions are frequently persecutory in nature.
Prosopagnosia is the inability to recognise familiar faces.
New Oxford Textbook of Psychiatry, eds Gelder, Lopez-Ibor Jr, Andreason, 2000, p431. Seminars in Old Age
Psychiatry, eds Butler and Pitt, 1998, p50-61
Incorrect
Marks for this submission: 0/1.
Question 15
Marks: 0/1
What is not a diagnostic criterion for Alzheimer's dementia
Choose one answer.
a. Gradual memory loss
b. Emotional lability
c. General cerebral atrophy
d. Persevered awareness of the environment
e. Apathy
For diagnosis of dementia, Four features must be present: (1) Decline in memory & Cognitive abilities; (2)
Persevered awareness of the environment (absence of clouding of consciousness) (3) A decline in emotional
control or motivation, or a change in social behaviour, manifest as at least one of the following: emotional
lability; irritability; apathy; & coarsening of social behaviour. (4) Presence for at least six months. The other
criteria specific to Alzheimer's dementia are: Evidence of a very slow, gradual onset and progression (the rate
of the latter may be known only
retrospectively after a course of 3 years or more); (b) predominance of memory impairment, over intellectual
impairment

ICD - 10. WHO 1992.


Incorrect
Marks for this submission: 0/1.
Question 16
Marks: 0/1
Many factors are felt to increase the risk of Alzheimer's Disease. Which one of the following is the most
important risk factor for the development of Alzheimer's Disease?
Choose one answer.
a. Head Injury
b. Female Gender
c. Smoking
d. Aluminium
e. Family History
Some studies have shown smoking to be slightly protective. Incidence studies have not presented a consistent
picture concerning female gender to be a risk factor. Some research has supported the hypothesis that head
trauma is only a risk factor for individuals who carry the ApoE allele. First degree relatives of patients with late-
onset AD have a risk of three times that of the general population. There is inconclusive evidence concerning
Aluminium. Flatten (2001) looked at aluminium in drinking water.

Dementia, eds Burns, O'Brien, Ames. 2005, 3rd Edition, p369-372


Incorrect
Marks for this submission: 0/1.
Vascular Dementia
1
Marks: 0/1
Regarding the clinical presentation of vascular dementia, which one of the following is correct?
Choose one answer.
a. Apraxia is more suggestive of vascular dementia than Alzheimer's disease
b. Insight is rarely retained in the early stages of the disease process
c. A step wise progression is more suggestive of lewy body dementia than vascular dementia
d. Post stroke dementia is an independent predictor of mortality.
e. Personality change occurs later than in Alzheimer's disease
Emotional and personality changes often occur first, followed by impairment of memory and intellect.
Tatemichi et al (1994) found post-stroke dementia is an independent predictor of mortality. Insight is
often present until a late stage of illness. Apraxia is a core feature of AD. Stepwise course of
deteriorating cognitive function indicates vascular dementia, rather than DLB.
Practical Psychiatry of Old Age. Wattis, Curran, 3rd Edition p84. Dementia, eds Burns, O'Brien, Ames.
2005, 3rd Edition, p531
Incorrect
Marks for this submission: 0/1.
Question 2
Marks: 0/1
CADASIL appears to be the commonest genetic form of Vascular dementia. Which one of the
following statements regarding CADASIL is correct?
Choose one answer.
a. Epilepsy occurs in the majority of patients with CADASIL
b. CADASIL shows an X-linked dominant pattern of inheritance
c. CADASIL shows an autosomal dominant pattern of inheritance
d. CADASIL shows an autosomal recessive pattern of inheritance
e. CADASIL is caused by a deletion on chromosome 19.
CADASIL (cerebral autosomal dominant arteriopathy with subcortical infarcts and
leukoencephalopathy) shows an autosomal dominant pattern of inheritance. CADASIL is caused by a
point mutation in the Notch3 gene on chromosome 19. Diagnosis can be established by skin biopsy
with confirmation by immunostaining with notch3 monoclonal antibody. Stroke is a clinical
manifestation in 80% of patients with CADASIL. Epilepsy occurs in 10% of patients with CADASIL.
Dementia, eds Burns, O'Brien, Ames.2005, 3rd Edition, p549,579-580
Incorrect
Marks for this submission: 0/1.
Question 3
Marks: 0/1
The Hachinski Ischemic Scale (HIS) is widely used to help differentiate Vascular Dementia from
Alzheimer's Disease based on clinical features. Which ONE of the following is a component of the
HIS?
Choose one answer.
a. Deterioration of personality
b. Somatic complaints
c. Gait disturbance
d. Gradual onset of symptoms
e. Urinary incontinence
The Hachinski Ischemia Score includes points for abrupt onset (2), fluctuating course (2), history of
strokes (2), focal neurological symptoms (2), focal neurological signs (1), stepwise deterioration (1),
history of hypertension (1), evidence of associated atherosclerosis (1), relative preservation of
personality (1), depression (1), emotional incontinence (1), somatic complaints (1), nocturnal confusion
(1). A score of more than 6 indicates vascular dementia.
Hachinski et al. Cerebral blood flow in dementia. Arch Neurol 1975;32(9):632-7.
Incorrect
Marks for this submission: 0/1.
Question 4
Marks: 0/1
Epidemiological studies looking at Vascular Dementia (VaD) have shown which one of the following
to be correct?
Choose one answer.
a. The prevalence is higher in Japan compared with Europe and the United States
b. The prevalence has an equal sex ratio
c. Onset is usually in the sixth decade
d. The prevalence increases with age, doubling every 10 years.
e. Survival is longer than for Alzheimer's Disease
Onset is usually in the seventh or eight decade. In general, survival from VaD is around 5 years. This is
less than that of the general population or those with AD. The prevalence of VaD is higher with
increasing age and in males. Studies have shown a higher prevalence of VaD in China and Japan which
have a higher incidence of vascular disease. The prevalence of VaD increases with age, approximately
doubling every five years.
""
Incorrect
Marks for this submission: 0/1.
Question 5
Marks: 0/1
Which one of the following concerning the management of vascular dementia is correct?
Choose one answer.
a. Ginkgo biloba should be avoided due to increased risk of bleeding
b. Acetylcholinesterase inhibitors are licensed for the treatment of vascular dementia
c. Aspirin is of proven benefit
d. Nimodipine is recommended in mild-moderate cases
e. Donepezil is of benefit in vascular dementia
A Cochrane review concluded that there is no evidence that aspirin is effective in treating patients with
VaD. There is growing evidence to suggest cholinesterase inhibitors may be of benefit in VaD, but they
are not licensed for this use. Early evidence suggests nimodipine may be useful in mild-moderate cases
(A pilot open label trial of nimodipine in patients with small vessel VaD was positive). There is some
evidence to suggest that cholinesterase inhibitors may be of benefit in vascular dementia. Ginkgo
biloba is currently under investigation, and several trials have shown modest positive results.
Williams et al (2000), Aspirin for vascular dementia. Cochrane Database of Systematic Reviews 4.
NICE Guidelines. November 2006. Dementia, eds Burns, O'Brien, Ames. 2005, 3rd Edition, p584-587
Incorrect
Marks for this submission: 0/1.

Vascular Dementia
1
Marks: 0/1
Epidemiological studies looking at Vascular Dementia (VaD) have shown which one of the following
to be correct?
Choose one answer.
a. The prevalence has an equal sex ratio
b. Onset is usually in the sixth decade
c. The prevalence increases with age, doubling every 10 years.
d. Survival is longer than for Alzheimer's Disease
e. The prevalence is higher in Japan compared with Europe and the United States
Onset is usually in the seventh or eight decade. In general, survival from VaD is around 5 years. This is
less than that of the general population or those with AD. The prevalence of VaD is higher with
increasing age and in males. Studies have shown a higher prevalence of VaD in China and Japan which
have a higher incidence of vascular disease. The prevalence of VaD increases with age, approximately
doubling every five years.
""
Incorrect
Marks for this submission: 0/1.
Question 2
Marks: 0/1
Regarding the clinical presentation of vascular dementia, which one of the following is correct?
Choose one answer.
a. A step wise progression is more suggestive of lewy body dementia than vascular dementia
b. Post stroke dementia is an independent predictor of mortality.
c. Insight is rarely retained in the early stages of the disease process
d. Apraxia is more suggestive of vascular dementia than Alzheimer's disease
e. Personality change occurs later than in Alzheimer's disease
Emotional and personality changes often occur first, followed by impairment of memory and intellect.
Tatemichi et al (1994) found post-stroke dementia is an independent predictor of mortality. Insight is
often present until a late stage of illness. Apraxia is a core feature of AD. Stepwise course of
deteriorating cognitive function indicates vascular dementia, rather than DLB.
Practical Psychiatry of Old Age. Wattis, Curran, 3rd Edition p84. Dementia, eds Burns, O'Brien, Ames.
2005, 3rd Edition, p531
Incorrect
Marks for this submission: 0/1.
Question 3
Marks: 0/1
The Hachinski Ischemic Scale (HIS) is widely used to help differentiate Vascular Dementia from
Alzheimer's Disease based on clinical features. Which ONE of the following is a component of the
HIS?
Choose one answer.
a. Deterioration of personality
b. Gradual onset of symptoms
c. Somatic complaints
d. Urinary incontinence
e. Gait disturbance
The Hachinski Ischemia Score includes points for abrupt onset (2), fluctuating course (2), history of
strokes (2), focal neurological symptoms (2), focal neurological signs (1), stepwise deterioration (1),
history of hypertension (1), evidence of associated atherosclerosis (1), relative preservation of
personality (1), depression (1), emotional incontinence (1), somatic complaints (1), nocturnal confusion
(1). A score of more than 6 indicates vascular dementia.
Hachinski et al. Cerebral blood flow in dementia. Arch Neurol 1975;32(9):632-7.
Incorrect
Marks for this submission: 0/1.
Question 4
Marks: 0/1
Which one of the following concerning the management of vascular dementia is correct?
Choose one answer.
a. Aspirin is of proven benefit
b. Donepezil is of benefit in vascular dementia
c. Acetylcholinesterase inhibitors are licensed for the treatment of vascular dementia
d. Nimodipine is recommended in mild-moderate cases
e. Ginkgo biloba should be avoided due to increased risk of bleeding
A Cochrane review concluded that there is no evidence that aspirin is effective in treating patients with
VaD. There is growing evidence to suggest cholinesterase inhibitors may be of benefit in VaD, but they
are not licensed for this use. Early evidence suggests nimodipine may be useful in mild-moderate cases
(A pilot open label trial of nimodipine in patients with small vessel VaD was positive). There is some
evidence to suggest that cholinesterase inhibitors may be of benefit in vascular dementia. Ginkgo
biloba is currently under investigation, and several trials have shown modest positive results.
Williams et al (2000), Aspirin for vascular dementia. Cochrane Database of Systematic Reviews 4.
NICE Guidelines. November 2006. Dementia, eds Burns, O'Brien, Ames. 2005, 3rd Edition, p584-587
Incorrect
Marks for this submission: 0/1.
Question 5
Marks: 0/1
CADASIL appears to be the commonest genetic form of Vascular dementia. Which one of the
following statements regarding CADASIL is correct?
Choose one answer.
a. Epilepsy occurs in the majority of patients with CADASIL
b. CADASIL is caused by a deletion on chromosome 19.
c. CADASIL shows an X-linked dominant pattern of inheritance
d. CADASIL shows an autosomal recessive pattern of inheritance
e. CADASIL shows an autosomal dominant pattern of inheritance
CADASIL (cerebral autosomal dominant arteriopathy with subcortical infarcts and
leukoencephalopathy) shows an autosomal dominant pattern of inheritance. CADASIL is caused by a
point mutation in the Notch3 gene on chromosome 19. Diagnosis can be established by skin biopsy
with confirmation by immunostaining with notch3 monoclonal antibody. Stroke is a clinical
manifestation in 80% of patients with CADASIL. Epilepsy occurs in 10% of patients with CADASIL.
Dementia, eds Burns, O'Brien, Ames.2005, 3rd Edition, p549,579-580
Incorrect
Marks for this submission: 0/1.

Lewy Body Dementia

1
Marks: 0/1
Differentiating dementia of Lewy bodies (DLB) from Alzheimer's Disease (AD) is based on clinical
history. Which ONE of the following features would lead you to suspect AD rather than early DLB?
Choose one answer.
a. Impairment in problem solving
b. Cognitive impairment
c. Visuospatial dysfunction
d. Executive dysfunction
e. Attentional deficits
Attentional deficits, visuospatial dysfunction and executive dysfunction are prominent in early DLB.
Symptoms of persistent or prominent memory impairment are not always present early in the natural
history of DLB. Prominent deficits on tests of problem solving may be useful as a clinical discriminator
to identify cases of DLB.
Dementia, eds Burns, O'Brien, Ames. 2005, 3rd Edition, p607-609
Incorrect
Marks for this submission: 0/1.
Question 2
Marks: 0/1
Which ONE of the following are clinical features is NOT found in diffuse Lewy body disease?
Choose one answer.
a. Fluctuations in cognitive functioning
b. Apathy
c. Parkinsonism
d. Attentional impairment
e. Dementia
Apathy is not a clinical characteristic of diffuse Lewy body disease.
Text book of Neuropsychiatry,2 edition, Schiffer,Rao,Fogel,p-936
Incorrect
Marks for this submission: 0/1.
Question 3
Marks: 0/1
In Lewy body dementia
Choose one answer.
a. Neurofibrillary tangles are plenty
b. Lewy bodies are typically present in the hippocampus
c. Degree of cortical cell loss is more so compared to Alzheimer's
d. Senile plagues are absent
e. the distinctive pathological finding is the occurrence of numerous Lewy bodies in the cortex
Hippocampus is largely spared in Lewy body dementia
Organic Psychiatry,Lishman,3 rd Edition P 452
Incorrect
Marks for this submission: 0/1.
Question 4
Marks: 0/1
Which of the following symptoms would lead you to suspect a diagnosis of dementia with Lewy bodies
(DLB)?
Choose one answer.
a. Early memory impairment
b. Auditory hallucinations
c. Prominent affective features
d. Spontaneous motor features of parkinsonism
e. Urinary incontinence
Recurrent visual hallucinations are a core feature of DLB. Cognitive decline is required for a
diagnosis.. Neuroleptic sensitivity is a feature which supports the diagnosis. Spontaneous motor
features of parkinsonism are a core feature of DLB. Fluctuating cognition with pronounced variations
in attention and alertness is a core feature of DLB.
McKeith et al. Consensus guidelines for the clinical and pathologic diagnosis of dementia with Lewy
Bodies. Neurology 1996;47:113-24.
Incorrect
Marks for this submission: 0/1.
Question 5
Marks: 0/1
The dementia syndrome, characterized by fluctuating symptom severity, visual hallucinations,
attentional impairment, altered state of consciousness, and delusions, particularly difficult to
distinguish from delirium is
Choose one answer.
a. Alzheimer's dementia
b. Vascular dementia
c. Dementia due to HIV disease
d. Dementia due to hypothyroidism
e. Lewy body dementia
Lewy body dementia mimics delirium in its characteristic fluctuations in symptoms over time.
Organic Psychiatry,Lishman,3 rd Edition P 450-1
Incorrect
Marks for this submission: 0/1.
Question 6
Marks: 0/1
Neuropsychiatric symptoms are a common problem in Dementia of Lewy Bodies (DLB). Which ONE
of the following is NOT true regarding symptoms and treatment?
Choose one answer.
a. Anti-Parkinsonian medications can induce psychotic symptoms
b. Clozapine may be beneficial in treating psychotic symptoms
c. Antipsychotics are the first line of treatment for psychotic symptoms
d. Visual hallucinations are the most common psychotic symptom
e. Acetylcholinesterase inhibitors have been shown to be effective
Although studies have shown antipsychotics to be effective, neuroleptic sensitivity is can in cases be
life threatening. Other studies however report a worsening of cognition and behaviour with clozapine.
Neuropsychiatric symptoms may be improved with acetylcholinesterase inhibitors. Visual
hallucinations are common and troublesome in DLB. Reducing anti-parkinsonian medication is
recommended in certain patients.
Dementia, eds Burns, O'Brien, Ames. 2005, 3rd Edition, p651. Taylor, Paton and Kerwin. Maudsley
Prescribing Guidelines 2006-2007, 9th Edition.
Incorrect
Marks for this submission: 0/1.

Frontotemporal Dementia
1
Marks: 0/1
Fronto-temporal dementia, a relatively common form of early-onset dementia, arises from the
progressive degeneration of the frontal and temporal lobes. Which ONE of the following is a
characteristic finding in frontotemporal dementia (FTD)?
Choose one answer.
a. Mental rigidity
b. Prominent memory loss
c. Gait abnormalities
d. Relative preservation of personality
e. Visuospatial deficits
FTD is characterised by cognitive defects in attention, abstraction planning, judgement and
organisation. Overriding feature of FTD is an breakdown of personal and interpersonal conduct.
Perceptuospatial function is preserved. Repetitive and imitative behaviour are both part of the
behavioural characteristics of FTD. Mental rigidity and concreteness of thought are present in FTD.
Lund-Manchester consensus on clinical criteria. Dementia, eds Burns, O'Brien, Ames. 2005, 3rd
Edition, p675
Incorrect
Marks for this submission: 0/1.
Question 2
Marks: 0/1
Which of the following is a characteristic neuropathological finding in Frontotemporal dementia?
Choose one answer.
a. senile plaques
b. Cholinergic deficits
c. atrophy of the medial temporal cortex
d. spongiform changes
e. neurofibrillary tangles
Senile plaques and neurofibrillary tangles are generally not seen in cases of FTD. Unlike AD,
cholinergic deficits are not seen in FTD. Spongiform changes occur in the superficial layers of the
cortex. Principle sites of atrophy in FTD include the frontal and anterior temporal neocortices, the
amygdala and basal ganglia. 5HT receptors are lost from frontal and temporal cortex in FTD.
Shorter Oxford Textbook of Psychiatry, eds Gelder, Harrison, Cowen, 5th Edition p343. Dementia, eds
Burns, O'Brien, Ames. 2005, 3rd Edition, p674
Incorrect
Marks for this submission: 0/1.
Question 3
Marks: 0/1
Semantic Dementia arises from degeneration of the temporal lobes. It is a distinctive clinical syndrome.
Which ONE of the following is NOT a diagnostic feature of semantic dementia?
Choose one answer.
a. Prosopagnosia
b. Associative agnosia
c. Non-fluent speech
d. Preserved single word repetition
e. Semantic paraphasia
This is impaired recognition of identity of familiar faces. This is impaired recognition of object
identity. Speech is fluent, spontaneous, but empty of meaning. Semantic errors occur in speech, for
example a rabbit is referred to as a cat. Single word repetition is preserved.
Dementia, eds Burns, O'Brien, Ames. 2005, 3rd Edition p703
Incorrect
Marks for this submission: 0/1.
Question 4
Marks: 0/1
Features of Picks disease include all except
Choose one answer.
a. most commonly affects frontal and parietal lobes
b. characteristic knife blade appearance
c. absence of senile plaques and tangles
d. Usually parietal and Occipital lobes are affected
e. presence of pick bodies
Gross appearance of brain is characteristic
Organic Psychiatry,Lishman,3 rd Edition P 462
Incorrect
Marks for this submission: 0/1.
Other Dementias
1
Marks: 0/1
Auditory comprehension is abnormal in
Choose one answer.
a. Conduction
b. Wernicke's
c. Transcortical motor
d. Broca's
e. Anomia
Auditory comprehension is abnormal in Wernicke's aphasia, as well as in global aphasia and
transcortical sensory aphasia.
Organic Psychiatry,Lishman,3 rd Edition P 44
Incorrect
Marks for this submission: 0/1.
Question 2
Marks: 0/1
Which ONE of the following is normal in Broca's aphasia?
Choose one answer.
a. Writing comprehension
b. Repetition
c. Auditory comprehension
d. Fluency
e. Naming
In Broca's aphasia, auditory comprehension is relatively normal.
Organic Psychiatry,Lishman,3 rd Edition P 45
Incorrect
Marks for this submission: 0/1.
Question 3
Marks: 0/1
Which ONE of the following is an autosomal recessively inherited degenerative dementia?
Choose one answer.
a. Wilson's disease
b. Friedreich's ataxia
c. Frontotemporal dementia
d. Parkinson's disease
e. Huntington's disease
Wilson's disease is autosomal recessively inherited.
Organic Psychiatry,Lishman,3 rd Edition P 662
Incorrect
Marks for this submission: 0/1.
Question 4
Marks: 0/1
The psychiatric disorder most commonly found in patients with Parkinson's disease is
Choose one answer.
a. Psychosis
b. Generalized anxiety disorder
c. Intermittent explosive disorder
d. Obsessive-compulsive disorder
e. Affective disorder
Affective disorder is the most common psychiatric disturbance in Parkinson's disease, with an
estimated incidence of from 20% to 90%.
Text book of Neuropsychiatry,2 edition, Schiffer,Rao,Fogel,p-736
Incorrect
Marks for this submission: 0/1.

Delirium
1
Marks: 0/1
Delirium in the elderly is common. Which ONE of the following statements is NOT true?
Choose one answer.
a. patients with delirium have high mortality rates compared with non-delirious patients
b. patients with delirium are mis-diagnosed by medical staff in up to two thirds of cases
c. Patients with delirium make up 40% of elderly medical admissions
d. patients with delirium have longer hospital stays than non-delirious patients
e. patients with delirium are three times more likely to develop dementia
Patients with delirium are three times more likely to develop dementia, even if they have no prior
cognitive or functional impairment. Delirium may be unrecognised by doctors and nurses in up to two-
thirds of cases. Hospital mortality rates of patients with delirium range from 6% to 18% and are twice
that of matched controls. Patients with delirium make up 10-20% of medical admissions. This is higher
for surgical patients however with an estimated 10-50% of FNOF patients presenting with delirium.
Patients who develop delirium have high mortality, institutionalisation and complication rates and
longer lengths of hospital stays than non-delirious patients.
Rockwood et al. The risk of dementia and death after delirium. Age Ageing 1999;28:551-6. Foreman
M et al. Improving recognition of delirium in the elderly. Primary Psychiatry 2004;11:46-50. Bhat RS,
Rockwood K. The prognosis of delirium. Psycho geriatrics 2002;2:165-79. Levkoff S et al.
Epidemiology of delirium: an overview of research issues and findings. Int Psychogeriatr 1991;3:149-
67
Incorrect
Marks for this submission: 0/1.
Question 2
Marks: 0/1
Which of the following is the most common feature in delirium?
Choose one answer.
a. Delusions
b. Hallucinations
c. Disturbed sleep wake cycle
d. Labile mood
e. Increased motor activity
Common features of a delirious state are clouding of consciousness, altered sleep wake cycle,
agitation/overactive state or underactive/drowsy state and impaired cognition, with altered sleep-wake
cycle and disorientation being the most common features in elderly with delirium. Most of the studies
have also found the elderly to have a hypoactive delirium as opposed to a hyperactive type of delirium.
Gelder MG, Lopez-Ibor Jr, Andreasen N. New Textbook of Psychiatry. Oxford university press. Page.
1633
Incorrect
Marks for this submission: 0/1.
Question 3
Marks: 0/1
Risk factors for post head injury delirium include
Choose one answer.
a. occipital damage
b. female sex
c. Old age
d. left hemisphere lesions
e. smoking
Multiple conditions can contribute to the development of delirium in patients with traumatic brain
injury. These include structural brain damage, cerebral oedema, secondary brain hypoxia, seizures,
electrolyte imbalance, infections, and substance (e.g., barbiturates, opioids, steroids, alcohol)
withdrawal. Old age, coexistent severe medical disease, polypharmacy, and lesions of the basal ganglia
and right hemisphere have also been shown to be significant risk factors
Companion to Psychiatric Studies, Ed: Johnstone, Lawrie et al, Seventh edition 7 e P346
Incorrect
Marks for this submission: 0/1.
Question 4
Marks: 0/1
Which ONE of the following is the MOST likely to cause delirium in the elderly?
Choose one answer.
a. Tricyclic antidepressants
b. Isoniazid
c. Amoxicillin
d. Warfarin
e. Olanzapine
Although nearly all drugs have the potential to cause delirium in the elderly, tricyclic antidepressants
are a very common cause of delirium due to their anticholinergic activity. Among the neuroleptics the
phenothiazines have prominent anticholinergic properties that may cause delirium. Long-acting
sedatives, such as chlordiazepoxide and diazepam, and shorter-acting agents, such as triazolam,
temazepam and lorazepam, may cause delirium. Among the antihypertensives, the central alpha-
receptor blockers and the beta-receptor blockers are associated with the greatest risk of delirium.
Digitalis may cause delirium in the absence of cardiotoxic effects.
Seminars in Old Age Psychiatry, eds Butler and Pitt, 1998, p38
Incorrect
Marks for this submission: 0/1.
Question 5
Marks: 0/1
A 84 yr lady with dementia is admitted to a medical hospital with hip fracture. The relative risk of her
developing delirium is
Choose one answer.
a. 1
b. 9
c. 3
d. 7
e. 5
The factors associated with development of delirium in patients with hip fracture are age and dementia
(RR 5.53 (CIs: 3.56-8.58)). Many other factors including use of medications (psychotropic medications
and certain types of anaesthetic agents), severe medical illness, Visual impairment, Depression,
Abnormal sodium, Use of indwelling catheter, Use of physical restraints, and Adding three or more
medications during hospitalisation have been found to be important in some studies but have not been
found to have a significant association with delirium during systematic reviews.
Kalisvaart KJ et al. Risk factors and prediction of postoperative delirium in elderly hip-surgery
patients: implementation and validation of a medical risk factor model. J Am Geriatr Soc. 2006
May;54(5):817-22
Incorrect
Marks for this submission: 0/1.
Question 6
Marks: 0/1
A 80 year old lady was admitted to general hospital being severely unwell.Two days later she began
complaining to her relatives that staff are not treating her well and poisoning her food. She also reports
seeing spiders crawling under the bed, most possibly placed there by the Nurses. the most likely
diagnosis is:
Choose one answer.
a. Delirium
b. Dissociative state
c. Delusional disorder
d. Dementia
e. Schizophrenia
Increasing old age, dementia, visual impairment and severe medical illness are important risk factors
for delirium. An acute onset, elderly and concurrent severe medical illness suggests delirium in this
case.
The prevention, diagnosis and management of delirium in older people. National Guidelines (June
2006). Clinical Effectiveness and Evaluation Unit. Royal College of Physicians
Incorrect
Marks for this submission: 0/1.

Mood Disorders
1
Marks: 0/1
Regarding DSH and suicide in the elderly, which ONE of the following is correct?
Choose one answer.
a. Suicide rates in the elderly are similar for males and females
b. DSH is more common in elderly men than women
c. Suicide rates are similar in the elderly to other age groups
d. Physical illness is commonly associated with suicide
e. Self-harm in the elderly is more common than in teenagers.
Being a male is a risk factor for suicide in the elderly. Physical illness is one of the risk factors for
suicide in the elderly. DSH declines with age while completed suicide increases. There is roughly a
similar rate of DSH in elderly males and females. Suicide rates in the elderly are higher than in other
age groups.
New Oxford Textbook of Psychiatry, eds Gelder, Lopez-Ibor Jr, Andreason, 2000, p1659
Incorrect
Marks for this submission: 0/1.
Question 2
Marks: 0/1
Which ONE of the following statements regarding the treatment of late onset depression is correct?
Choose one answer.
a. Antidepressants are less effective in late-onset depression compared to younger onset.
b. Side effects of antidepressants occur at a similar rate in the elderly
c. Tricyclic antidepressants can be used in patients with subclinical dementia
d. Psychological interventions show reduced efficacy in the elderly
e. ECT is generally well tolerated in the elderly
Antidepressants do appear to be as effective as in younger patients, although there is little available
data. The central anticholinergic effects of TCA are undesirable in case there is a subclinical dementing
process, and can worsen cognitive impairment. ECT is appropriate for patients with distressing
agitation, suicidal ideas and behaviour, life threatening stupor and failure to respond to meds. In one
study, 39% of elderly patients had hyponatraemia on an SSRI compared with 10% of controls. The
elderly are more sensitive to side effects, have more comorbid physical illness and are more likely to be
on other medication which may interact. Psychological interventions have similar efficacy in the
elderly population, but are of limited availability.
Cunningham LA. Depression in the medically ill: choosing an antidepressant. J Clin Psychiatry 1994;
55(9, suppl A):90-7. Kirby D et al: Hyponatraemia in elderly psychiatric patients treated with SSRIs
and venlafaxine: a retrospective controlled study in an inpatient unit. Int J Geriatr Psychiatry 2002;
17:231-237. Shorter Oxford Textbook of Psychiatry, eds Gelder, Harrison, Cowen, 5th Edition, p513
Incorrect
Marks for this submission: 0/1.
Question 3
Marks: 0/1
There is no clear distinction between clinical presentation of late onset depression and that of early
onset. Which ONE of the following symptoms is more striking in the elderly?
Choose one answer.
a. Anhedonia
b. Somatic preoccupation
c. Loss of energy
d. Suicidal ideation
e. Diurnal variation
Can occur in up to 30% of depressed elderly patients. This is obviously a clinical feature of depression,
but is not more striking in late onset compared to early onset depression. Classical symptoms may not
be evident, with somatic, anxious and hypochondrical complaints occurring more frequently. A degree
of cognitive impairment has been detected in 70% of patients. Depressive delusions regarding poverty,
physical illness or nihilistic delusions are common.
Oxford Handbook of Psychiatry, Semple et al, First Edition, 2005. p478.
Incorrect
Marks for this submission: 0/1.
Question 4
Marks: 0/1
With regards to the course and prognosis of mood disorders in the elderly, which ONE of the following
is correct?
Choose one answer.
a. The majority of cases of late onset depression become chronic and unremitting
b. Late onset depression is a prodrome of dementia
c. Mania carries a similar mortality to that of depression
d. A depressive episode following an episode of late onset mania occurs less frequently than in
younger patients
e. Presence of organic central pathology is associated with a poor outcome in late life depression

20% of those with late onset depression develop chronic symptoms. Poor outcome is associated with
the severity of symptoms, presense of cerebral pathology and poor compliance with treatment. Despite
evidence that depression is a risk factor for dementia, there is no evidence that it is a prodrome for
dementia. Naturalistic studies of cognitively intact depressed patients have not found an increased rate
of dementia. However, patients with depression and cognitive impairment (who may present with a
pseudodementia picture) have a 4-fold increase in the risk of developing an irreversible dementia, and
require careful follow-up. Depressive episodes following mania occur more frequently in the elderly,
however the clinical presentation of an episode of mania is the same in younger and older patients.
Mania carried a higher mortality to that of depression and may indicate a more severe disruption of the
central nervous system.
Shorter Oxford Textbook of Psychiatry, eds Gelder, Harrison, Cowen, 5th Edition, p512. Baldwin 1986
BJPsych 149 : 574-583
Incorrect
Marks for this submission: 0/1.
Question 5
Marks: 0/1
Which ONE of the following has been demonstrated in patients with late-onset depression?
Choose one answer.
a. Good initial treatment response
b. normal size ventricles
c. White-matter signal hyperintensities
d. functional hyperperfusion of frontal and limbic areas
e. Twin studies show a high level of heritability in both men and women
There is strong evidence supporting vascular dementia - increased number and severity of white matter
hypertensities are frequently seen on MRI scans. A subgroup of elderly patients with affective disorders
show ventricular dilatation on CT. Heritability is lower than that of early onset, suggesting a slightly
smaller genetic contribution. Impairments of vision or hearing contribute to the likelihood of
depression. Physical illnesses are associated with between 60-75% of cases. A consistent finding has
been functional hypoperfusion in frontal and limbic areas. Current evidence base suggests deep white
matter lesions are associated with poor initial treatment response and poor long-term outcome.
Oxford Handbook of Psychiatry, Semple et al, First Edition, 2005. p478. Organic Psychiatry, eds
Lishman, third edition, p140. Shorter Oxford Textbook of Psychiatry, eds Gelder, Harrison, Cowen, 5th
Edition, p512
Incorrect
Marks for this submission: 0/1.
Question 6
Marks: 0/1
The term 'pseudodementia' is often used to describe memory impairment in patients with depressive
symptoms. Which ONE of the following statements concerning pseudodementia is NOT true?
Choose one answer.
a. Pseudo dementia is a risk factor for developing dementia
b. Sleep deprivation will improve cognitive functioning in those with pseudo dementia
c. Visuospatial functions can differentiate pseudo dementia from dementia
d. Serial sevens testing is useful in differentiating dementia from pseudo dementia
e. Apraxia and aphasia are usually absent in pseudo dementia
Visuo-spatial dysfunction is usually absent in pseudodementia. Sleep deprivation will improve
cognitive function in those with an affective disorder, but will worsen those with a dementing illness.
Serial sevens is a test of concentration, which is likely to be impaired in both conditions.
Higher cortical dysfunction such as apraxia and aphasia is usually absent in pseudo dementia. Those
who present with depression and cognitive impairment have a four-fold increase in the risk of
developing an irreversible dementia even though their initial cognitive deficit reverses with treatment.
New Oxford Textbook of Psychiatry, eds Gelder, Lopez-Ibor Jr, Andreason, 2000, p1645
Incorrect
Marks for this submission: 0/1.
Question 7
Marks: 0/1
Epidemiological studies looking at affective disorders in the elderly have found which of the following
to be correct?
Choose one answer.
a. The average age of onset for late onset mania is 65 years
b. GPs over diagnose depression in the elderly
c. Mania makes up 1/3rd of psychiatric admissions in the elderly
d. The gender difference for depression increases with age
e. The prevalence of late onset depression is 13%
Prevalence rates for depression in the elderly range from 3-15% in community samples. Higher rates
are found in those who attend general practitioners (15-20%), care home residents (15-20%) and
general hospital in-patients (approx. 20%). Mania represents 12% of all affective disorders treated on
specialised geriatric psychiatry units. The gender difference in depression persists with increasing age.
There is no evidence that it increases although some suggest that the prevalence rates in men and
women may diminish after menopause (around the age of 55).
GPs tend to under diagnose depression in the elderly. The average age of onset for late onset mania is
55 years.
Cairney J, Wade TJ (2002) The influence of age on gender differences in depression: further
population-based evidence on the relationship between menopause and the sex difference in depression.
Soc Psychiatry Psychiatr Epidemiol. 37(9), 401-8. Seminars in Old Age Psychiatry, eds Butler and Pitt,
1998, p127
Incorrect
Marks for this submission: 0/1.

Psychosis in Old Age


1
Marks: 0/1
The presentation of very late onset schizophrenia is similar to that of younger onset schizophrenia.
Which ONE of the following is more likely to be found in younger onset schizophrenia compared to
very late onset schizophrenia?
Choose one answer.
a. Partition delusions
b. Persecutory delusions
c. Third person auditory hallucinations
d. Formal thought disorder
e. Visual, tactile and olfactory hallucinations
Partition delusions (i.e. believing that a person or object can pass through a barrier) are more common
in very late onset schizophrenia. Persecutory delusions often predominate in very late onset
schizophrenia, although other delusions can commonly arise. Formal thought disorder, affective
flattening and blunting and negative symptoms are seen more commonly in younger onset
schizophrenia. Third person, running commentary and derogatory auditory hallucinations are seen more
often in very late onset schizophrenia.
Shorter Oxford Textbook of Psychiatry, eds Gelder, Harrison, Cowen, 5th Edition, p514
Incorrect
Marks for this submission: 0/1.
Question 2
Marks: 0/1
Many factors have been identified regarding the aetiology of very late onset schizophrenia. Which
ONE of the following factors has NOT been found to be associated with very late onset schizophrenia?
Choose one answer.
a. Association with sensory deficits
b. Female predominance
c. Schizoid personality traits
d. Social isolation
e. White matter hypertensities
There is an association with sensory deficits (impaired vision and hearing). Unlike late life depression,
white matter hypertensities are not increased. Schizoid and paranoid personality traits are common in
elderly who develop late-onset psychotic disorders. There is an association with social isolation. There
is a female preponderance which increases with age.
Shorter Oxford Textbook of Psychiatry, eds Gelder, Harrison, Cowen, 5th Edition, p514
Incorrect
Marks for this submission: 0/1.
Question 3
Marks: 0/1
Diogene's syndrome, or senile squalor syndrome, is characterised by severe neglect. Which one of the
following concerning Diogene's syndrome is true?
Choose one answer.
a. Syllogomania rarely occurs
b. Diogene's syndrome is not associated with end stage personality disorder
c. Diogene's syndrome can co-exist with dementia
d. Diogene's syndrome is associated with a high mortality after hospital admission
e. Diogene's syndrome is often caused by delusional thinking
Syllogomania, the hoarding of rubbish, occurs with Diogene's syndrome. 46% of patients have a 5 year
mortality rate. Hospitalisation has to be avoided whenever possible, due to a nigh mortality after
admission. Other causes of self neglect such as dementia, depression and psychosis must be excluded.
Other causes of self neglect such as dementia, depression and psychosis must be excluded. It has been
suggested that the syndrome represents the end-stage of a personality disorder or that there may be a
degree of frontal lobe impairment.
Hanon et al. Diogenes syndrome: a transnosographic approach. Encephal. 2004 Jul-Aug;30(4):315-22.
Shorter Oxford Textbook of Psychiatry, eds Gelder, Harrison, Cowen, 5th Edition, p515
Incorrect
Marks for this submission: 0/1.

Other topics
1
Marks: 0/1
Alcohol misuse is an important consideration in the elderly population. Which one of the following is
NOT true concerning alcohol misuse?
Choose one answer.
a. Can present as self-neglect and depression
b. Is more common in men compared to women
c. Acute alcohol withdrawal syndrome is more protracted and severe in elderly people
d. Alcohol misuse increases the risk of developing Alzheimer's disease
e. In patient treatment is recommended for those requiring detox
Presentation can be with falls, drowsiness, delirium, depression, self-neglect or poor control of co
morbid conditions such as hypertension or diabetes. It has been reported that there is increased
occurrence of all types of dementia except Alzheimer's disease in elderly people with alcohol use
disorders. Out-patient detoxification may not be appropriate for older adults. Acute alcohol withdrawal
syndrome is more protracted and severe in elderly people than in younger patients with drinking
problems of equal severity. However being female is a risk factor for alcohol misuse.
Thomas, V. S. & Rockwood, K. J. (2001) Alcohol abuse, cognitive impairment and mortality among
older people. Journal of the American Geriatric Society, 49, 415-420. Dar K, Alcohol use disorders in
elderly people: fact or fiction? APT(2006) 12: 173-181
Incorrect
Marks for this submission: 0/1.
Question 2
Marks: 0/1
With regards to anxiety disorders in the elderly, which ONE of the following statements is true?
Choose one answer.
a. Panic disorder is common
b. Obsessive-compulsive disorder accounts for most anxiety in late life
c. Agoraphobia may occur as a primary disorder in late life
d. Symptoms of generalized anxiety disorder are associated with a lower level of suicidality
e. The presence of a comorbid anxiety disorder with depression is associated with a lower level of
somatic symptoms
Generalised anxiety disorder and phobias account for most anxiety in late life. Panic disorder is unusual
in old age, and a first onset presentation should always prompt a search for an underlying physical or
depressive disorder. It is true that agoraphobia may occur as a primary disorder in old age - whereas
simple phobia, OCD in males, and panic disorder either persist from younger years or arise in the
context of another psychiatric or medical disorder. Elderly patients with comorbid depression and
anxiety have been found to have a higher rate of suicide. Comorbid anxiety with depression is
associated with poorer social function and a higher level of somatic symptoms.
Flint (1994) Epidemiology and co morbidity of anxiety disorders in the elderly. Am J Psychiatry
1994;151(5):640-9. Lenze et al. Co morbid anxiety disorders in depressed elderly patients. Am J
Psychiatry 2000;157(5):722-8.
Incorrect
Marks for this submission: 0/1.
Question 3
Marks: 0/1
Which ONE of the following statements is NOT true concerning personality disorders in the elderly
population?
Choose one answer.
a. Paranoid traits can be mistaken for delusional disorder
b. Obsessive-compulsive characteristics can become more prominent
c. There is a reduction in Cluster B personality disorders
d. The prevalence of personality disorders in the over 50s is approximately 3%
e. Personality disorder may be a risk factor for late-life depression
The prevalence is between 7-10%, lower than that of the younger population. Personality disorder may
be a risk factor for late life depression, and personality disorder as a comorbid condition appears to be
more common in patients hospitalised for depression than in those hospitalised for disorders of
cognition. Obsessive-compulsive and schizoid characteristics may become more prominent (Engles et
al 2003). There are a number of reasons postulated for the reduction in Cluster B personality disorders
in the elderly, including maturation of personality and reduction in problematic behaviours associated
with the personality disorder. Schizoid and paranoid traits may become more pronounced due to
isolation, and can be mistaken for delusional disorder or schizophrenia.
Shorter Oxford Textbook of Psychiatry, eds Gelder, Harrison, Cowen, 5th Edition, p515
Incorrect
Marks for this submission: 0/1.

EMIs - Old Age Psychiatry


1
Marks: 0/3
PSYCHOPHARMACOLOGICAL ACTION

A. Donepezil
B. Galantamine
C. Ginko Biloba
D. Melissa oil
E. Memantine
F. Memtase
G. Rivastigmine
H. Tacrine
I. Vitamin E

For each of the following statements, select the most appropriate medication
Action by inhibition of both acteylcholinesterase and butyrylcholinesterase.
A once daily cholinesterase inhibitor
Some research evidence for its utility in vascular dementia
1. G – Rivastigimine inhibits both acteylcholinesterase and butyrylcholinesterase.
2. A – Donepezil can be given once a day because of its long half-life.
3. E– There is some evidence to suggest its use in vascular dementia.
Incorrect
Marks for this submission: 0/3.
Question 2
Marks: 0/3
DEMENTIA – DIAGNOSIS

A. Alcohol Korsakoff‘s Syndrome


B. Alzheimer‘s disease
C. Chronic Subdural Haematoma
D. Dementia of Lewy Body type
E. Dementia Pugilistica
F. Depression
G. Frontotemporal Dementia
H. New variant CJD
I. Normal Pressure Hydrocephalus
J. Vascular Dementia

For each of the following patients, select the most likely diagnosis
A 51-year-old lady was brought by her husband due to rapidly progressive memory problems. On
examination she was ataxic and was found to have rigidity, tremors and myoclonus.
An 89-year-old woman was found wandering the streets at night. She is disoriented in time, place or
person. Generalised cortical atrophy was found on her CT scan
An 86-year-old woman presents with history of memory loss of 2-3 years. The onset of her memory
problems was due to multiple episodes of short lasting confusion with incomplete recovery. After
each episode, her memory has become worse. She also has history of Ischaemic heart disease,
diabetes and vasculopathy of both legs.
H – The rapid progression of cognitive decline and neurological symptoms in a relatively young person
are suggestive of CJD.

B – This is most suggestive of Alzheimer‘s disease.


J – The clues to the diagnosis here include the presence of cardiovascular disease and stepwise
progression.
Incorrect
Marks for this submission: 0/3.
Question 3
Marks: 0/3
DEMENTIA - DIAGNOSIS

A. Alcohol Korsakoff‘s Syndrome


B. Alzheimer‘s disease
C. Chronic subdural haematoma.
D. Dementia Of Lewy Body type
E. Depression
F. Frontotemporal Dementia
G. Normal Pressure Hydrocephalus
H. Persistent somatoform disorder
I. Complex partial seizures
J. Vascular Dementia

For each of the following patients, select the most likely diagnosis.
A 55-year-old man is brought in by his brother who is finding it difficult to manage him at home due
to memory problems, urinary incontinence and problems with his mobility.
A 58-year-old woman is brought by her husband as he had noticed her to be displaying bizarre
behaviour over the last 3 days. She has been complaining of a strange taste in the mouth and mentions
funny smells in the house. Her husband has noticed be her to be agitated and in stupor at times. He
also reports that she has stopped her medications a week ago.
A 78-year-old man, whose wife died 4 months ago, has been brought by his children as he has
stopped going out of the house. His children report that he has a flat affect, disturbed sleep and has
lost weight. They are concerned about him due to his self neglect
1. G – NPH is suggested by the presence of memory impairment, urinary incontinence and mobility
problems in a relatively young man.
2. I – The aura and loss of awareness along with altered behaviour suggests complex partial seizures
3. E – The most likely diagnosis is depression as there are core depressive features present.
Incorrect
Marks for this submission: 0/3.
Question 4
Marks: 0/3

Dementia

A. Carer married to patient


B. Higher physical dependency in the patient
C. History of poor relationship with carer
D. Living alone
E. Living in an institution
F. Living with another person
G. Female carer
H. Male carer
I. Patient with dementia

1. Non-accidental injury from carer


2. Carer distress
3. Depression of elderly
1. The British Geriatric society identified a number of potential risk factors that are associated with
physical/psychological abuse in a domestic situation. These were social isolation, poor previous
relationship between the abused and the abuser, a pattern of family violence being modelled,
dependence of the abused on the abuser for accommodation, finances etc, history of mental health
problems and substance misuse in the abuser.
2. Female carers and those who looked after patients with more severe depression or psychotic
symptoms or less severe cognitive impairments (higher scores on the MMSE reflect less
cognitive impairment) were at risk of experiencing high levels of distress. Patient depression was
the strongest predictor of distress, followed by carer gender, psychotic symptoms and cognitive
impairments (Donaldson et al., Determinants of carer stress in Alzheimer's disease. Int J Geriatr
Psychiatry. 1998, 13(4):248-56.). Other studies have identified poor previous relationship as a
risk factor for care distress. Carers who were not the patient's spouse and who looked after
patients with fewer physical signs of depression and/or more mood-related signs or behavioural
changes were at risk of experiencing higher levels of subjective burden
3. The risk factors for depression in the elderly include: Physical health problems, disability, Sensory
impairment, Adverse life events – particularly losses e.g. bereavement, Poor social support, Past
history of depression, Neuroticism, Caring for ill relatives, Being widowed/divorced, and Lack of
confidante.
Incorrect
Marks for this submission: 0/3.
Question 5
Marks: 0/3
DEMENTIA – NEUROBIOLOGY

A. Cerebellar atrophy
B. Circumscribed frontal atrophy.
C. Generalised cortical atrophy.
D. Lenticular atrophy.
E. Lewy bodies
F. Medial temporal structures sparing.
G. Neurofibrillary tangles
H. Parietotemporal atrophy
I. Senile Plaques
J. Striatonigral degeneration

For each of the following choose the most relevant abnormal finding.

1. Pick‘s Dementia

2. Lewy body dementia

3. Dementia of Alzheimer‘s type


1. B. Pick‘s dementia: Circumscribed frontal or fronto-temporal atrophy

2. E, F. Lewy body dementia will have lewy bodies and sparing of the temporal structures. These are
intraneuronal cytoplasmic inclusion bodies, consisting mainly of [alpha]-synuclein and ubiquitin,
which are one of the pathological hallmarks of Parkinson disease

3. C, G, I. ADT will have generalised cortical atrophy, senile plaques and neurofibrillary tangles.
Incorrect
Marks for this submission: 0/3.
Question 6
Marks: 0/3
DEMENTIA - CLINICAL FEATURES

A. Agitation
B. Auditory hallucinations
C. Disinhibition
D. Fluctuating cognitive impairment
E. Gradual onset
F. Incontinence
G. Memory impairment
H. Parkinsonism
I. Sudden onset
J. Visual hallucinations

For each of the following diagnosis, identify the characteristic clinical


features as required:

1. THREE Features of Lewy body dementia

2. TWO features of Alzheimer‘s disease

3. TWO features of vascular dementia.


1. D, H, J – Fluctuating cognitive impairment, parkinsonism and visual hallucinations are the hallmarks
of Lewy Body dementia.

2. E,G – In Alzheimer‘s disease there is a gradual onset of memory impairment along with apraxia,
agnosia and aphasia.

3. G, I – Vascular dementia is also characterised by memory impairment, but the onset is sudden and
there is step-wise decline
Incorrect
Marks for this submission: 0/3.
Question 7
Marks: 0/3

Psychosis - management

Options

A. Chlorpromazine
B. Clozapine
C. Flupentixol depot
D. Haloperidol
E. Levomopromazine
F. Lorazepam
G. Olanzapine
H. Quetiapine
I. Risperidone
J. Stop antipsychotics

Choose ONE most suitable medication for the following patients.


An 80 year old woman with Diabetes Mellitus and psychosis agrees to take medication but has
repeated relapses of her psychosis due to poor concordance. Whenever her son reminded her to take
her medication, it would end up in a quarrel.
A 66 year old psychotic elderly man has persistent psychotic symptoms, despite treatment with
adequate doses of sulpiride and olanzapine. He is currently on Flupenthixol, which he is not tolerating
that well.
A 74 year old man has been newly diagnosed to have Parkinson‘s disease, complains of severe,
distressing visual hallucinations. He is not on any drug treatment for his Parkinson‘s disease and has
never been tried on a neuroleptic medication.
A 68 year old man who presented with delusional disorder for the first time was treated with
Olanzapine leading to resolution of his delusions. He has however developed severe weight gain.

1. C - As atypicals act on 5T2 (block) causing more weight gain than typical antipsychotics the choice
would be to go for a typical antipsychotic. As compliance seems to be an issue, Flupenthixol
depot would be the best option.

2. B - Failing trial of two antipsychotics would mean treatment resistance and Clozapine would be the
drug of choice according to NICE guidelines, including in the elderly.

3. H - Quetiapine according to studies has proven to be the drug of choice in treating Parkinson‘s
psychosis. Quetiapine moderately reduces visual hallucinations and mildly reduces paranoia and
delusions in patients with Parkinson's disease with psychosis.

4. H - Weight gain in the first 10 weeks of prescribing an atypical is about 4.5 kgs with Clozapine, 4
kgs with Olanzapine, 2kgs with Risperidone and minimal with Quetiapine hence would be the
choice of drug in an elderly patient.

Incorrect
Marks for this submission: 0/3.

Psychotherapy
General Principles of Psychotherapy
1
Marks: 0/1
Identify the defence mechanism is being employed by a woman who says she is happy but the world and
everyone in it seems depressed
Choose one answer.
a. Projection
b. Externalisation
c. Projective identification
d. Overgeneralisation
e. Selective abstraction
The woman is using projection by attributing her subjective feelings onto an external force. This is different
from projective identification (Melanie Klien) which carries with it the added behavioural component of
reacting to the intolerable impulse projected outside self (provides a validation for the projection). Selective
abstraction is not a defense mechanism (instead, it is a CBT term). Externalisation and overgeneralisation are
types of cognitive distortion listed in the context of CBT (they are not listed by Freud or neo-Freudians as
'defense mechanisms').
Johnstone, E. C., Cunningham Owens, D. G., Lawrie, S. M., Sharpe, M., & Freeman, C. P. L. (2004). Companion to
psychiatric studies (7th ed., pp 311-314); Rycroft, C. (1995). A critical dictionary of psychoanalysis. London:
Penguin Books
Incorrect
Marks for this submission: 0/1.
Question 2
Marks: 0/1
Which of the following is generally considered to be the optimal number of psychotherapy sessions required to
produce a successful outcome in moderate to severe mental health problems?
Choose one answer.
a. At least sixteen
b. At least twelve
c. At least fourteen
d. At least ten
e. At least eight
Although the optimal number of sessions depends on the condition being treated (e.g., Linehan et al prescribed
twice weekly sessions for 12 months for patients with BPD), the general consensus is that a minimum of 16
sessions is required for symptomatic relief. Therapies of fewer than eight sessions are deemed unlikely to be
optimally effective for most moderate to severe mental health problems. Longer therapies may be required to
achieve lasting change in social and personality functioning. Specific phobias and uncomplicated panic disorder
(without agoraphobic symptoms) can respond to brief interventions.
Dept. of Health (2001) 'Treatment Choice in Psychological Therapies and Counselling -
Evidence Based Clinical Practice Guidelines'. London.
Incorrect
Marks for this submission: 0/1.
Question 3
Marks: 0/1
Which of the following is generally considered to be an important factor in deciding whether or not a patient
should receive psychotherapy?
Choose one answer.
a. The patient's intelligence
b. The patient's culture
c. The patient's "ego-integrity"
d. The patient's "ego strength"
e. The patient's age
Frustration tolerance and ego strength are essentially the ability to cope with the tensions arising from inner
conflict. These 2 characteristics can be measured using psychological assessment (e.g., 16 PF) and are
important determinants of the client's suitability for therapy. Ego integrity is used, but not defined, by Erikson
as part of the last of his stages of man. The patient's age, sex, social class or ethnic group are generally not
important factors in choice of therapy and should not determine access to therapy. Ethnic and cultural identity
should be respected by referral to culturally-sensitive therapists. General intelligence does not determine
suitability per se, but is an important factor in therapy selection (e.g. those with LD need to be provided with
modified versions of the prescribed therapy).
Dept. of Health (2001) 'Treatment Choice in Psychological Therapies and Counselling; -
Evidence Based Clinical Practice Guidelines'. London.
Incorrect
Marks for this submission: 0/1.
Question 4
Marks: 0/1
Towards the end of a therapy session, a patient suddenly blurts out 'I am abusing my children' before quickly
shifting the topic to other things. The appropriate thing for the therapist to do is:
Choose one answer.
a. To immediately clarify what they means by the term 'abusing'
b. to ensure boundaries are maintained by ending the session on time and waiting until the next session
to bring up the issue.
c. To reassure her that everything said in therapy is confidential and that she should discuss the use in
more detail.
d. Change the subject
e. To tell her that you have to report her to the authorities.
Ideally it is important for the therapist to ensure that boundaries are maintained and the session ends as per
schedule. However, in this case, as there is a potential risk of harm to others, and particularly for children, it is
the duty of the Therapist to clarify if there is cause for concern. There may be a need to make a possible
referral to Children protection agencies (at the therapist's discretion and after discussions with members of the
multi disciplinary team).
Zachary A. Individual Psychoanalytical Psychotherapy with perpetrators of sexual abuse. Advances in
Psychiatric Treatment 1998; 4:77-81 AND Norcross, J. C. & Guy, J. D. (2007). Leaving it at the office: A guide to
psychotherapist self care (pp. 93-113). London: Guilford Press.
Incorrect
Marks for this submission: 0/1.
Question 5
Marks: 0/1
Which of the following is true about the initial appraisal of an event:
Choose one answer.
a. Negative automatic thoughts are a part of secondary appraisal
b. Initial appraisals are driven by the id impulse
c. Perception of the magnitude of the threat is part of primary appraisal
d. Automatic thoughts form a part of primary appraisal
e. Perception of the magnitude of the threat is a part of secondary appraisal
Primary appraisal determines the significance of an event (can be based on emotional reactions, according to
Schachter and Singer) while secondary appraisal evaluates our potential to cope with that event. In GAD, panic
disorders, depression and other forms of 'neurosis', initial appraisals are distorted evaluations of the
magnitude of the threat or relevance to the person (which could be based on previous experiences that prime
the response). Negative automatic thoughts and dysfunctional elements that stem from the initial appraisal.
Gelder, M. G., Lopez-Ibor Jr, J., & Andreasen, N. C. (2000). New Oxford textbook of psychiatry (Vol. 2, pp. 1373-
1388). Oxford University Press.
Incorrect
Marks for this submission: 0/1.
Question 6
Marks: 0/1
Which of the following is true about psychotherapies in the learning disability population?
Choose one answer.
a. Guided mourning is not useful in mild LD.
b. Unmodified CBT is used in moderate LD.
c. Patients with LD who have been sexually abused are treated with brief psychodynamic therapy.
d. Gradual muscle relaxation is done in group psychodynamic therapy.
e. Anxiety management using behavioural techniques is usually effective with people with Learning
disability.
Psychological treatment in the form of behavioural therapies (including muscular relaxation, skills training,
chaining) have been found to be efficacious in patients with LD. Other forms of therapies including some CBT
techniques, grief work, psychoeducation, and eclectic psychotherapies can be used in their modified form (to
accommodate for the differences in intellectual functioning) with patients having LD. psychodynamic
approaches are not indicated in patients having LD, and is usually used as an exclusion criterion.
Royal College of Psychiatrists (1997) Meeting the Mental Health Needs of People with Learning Disability
(Council Report CR56). London: Royal College of Psychiatrists (c.f. Click here for reference
Incorrect
Marks for this submission: 0/1.

Psychodynamic psychotherapy

1
Marks: 0/1
You have been treating a 45 year old male with long-standing relationship problems using
psychotherapy. During supervision you reflect that you have felt like avoiding sessions with this man
as interacting with him evokes anger and irritation in you. What term best describes your feelings?
Choose one answer.
Error! Not a valid embedded object. a. Transference
Error! Not a valid embedded object. b. Counter transference
Error! Not a valid embedded object. c. Working through
Error! Not a valid embedded object. d. Acting out
Error! Not a valid embedded object. e. Negative therapeutic reaction
The feelings triggered in the therapist in response to the client are termed counter-transference.
Transference is a process in which feelings and ideas from previous relationships are transferred on to
the therapist and the client begins to relate to the therapist as if the therapist were someone from his
past. Negative Therapeutic Reaction refers to the worsening of a patient's symptoms in response to the
interpretations that were expected to improve them. Working through is the process by which a patient
in analysis gradually discovers the full implication of an interpretation or insight. Acting out refers to
the patient's behaviour that can be interpreted as a substitute for remembering what has happened in the
past.

Gelder, M. G., Lopez-Ibor Jr, J., & Andreasen, N. C. (2000). New Oxford textbook of psychiatry (Vol.
2, pp. 1421-1431). Oxford University Press; Rycroft, C. (1995). A critical dictionary of psychoanalysis.
London: Penguin Books
Incorrect
Marks for this submission: 0/1.
Question 2
Marks: 0/1
A 40 year old man with moderate depression is unwilling to take antidepressants but is willing to try
psychotherapy. The presence of which of the following of his characteristics/demographics make him
suitable for brief psychodynamic psychotherapy:
Choose one answer.
Error! Not a valid embedded object. a. Evidence of employment and stable source of income
Error! Not a valid embedded object. b. Above average intelligence
Error! Not a valid embedded object. c. Presence of at least one clearly identifiable stressor
Error! Not a valid embedded object. d. His age (less than 50 years old).
Error! Not a valid embedded object. e. Use of projection and denial as defense mechanisms
Brief dynamic therapy is indicated in those individuals of above average intelligence, who can
verbalise (and think in feeling terms), have a high motivation for recovery and have good ego strength.
Other characteristics for inclusion include the ability to engage, good response to trial interpretation,
identifiable goals and conflicts, at least one past meaningful relationship, and flexible defenses among
others.
Gelder, M. G., Lopez-Ibor Jr, J., & Andreasen, N. C. (2000). New Oxford textbook of psychiatry (Vol.
2, pp. 1421-1431). Oxford University Press.
Incorrect
Marks for this submission: 0/1.
Question 3
Marks: 0/1
In the course of a psychotherapy session a Woman, who is attracted to other Men, tells you she believes
her husband is having an affair. Which type of defence mechanism are you seeing?
Choose one answer.
Error! Not a valid embedded object. a. Projection
Error! Not a valid embedded object. b. Sublimation
Error! Not a valid embedded object. c. Denial
Error! Not a valid embedded object. d. Reaction formation
Error! Not a valid embedded object. e. Suppression
Projection is an immature defense mechanism whereby objectionable thoughts or emotions are
attributed to others. Denial (also an immature defence mechanism) is not accepting reality that is too
painful. Reaction formation is a neurotic defence mechanism in which one does the opposite of an
unacceptable impulse. Sublimation (satisfying socially objectionable impulses in an acceptable manner)
& suppression (purposely ignoring an unacceptable impulse or emotion in order to diminish discomfort
and accomplish a task) are mature defense mechanisms.
Johnstone, E. C., Cunningham Owens, D. G., Lawrie, S. M., Sharpe, M., & Freeman, C. P. L. (2004).
Companion to psychiatric studies (7th ed., pp 311-314). Churchill Livingstone.
Incorrect
Marks for this submission: 0/1.
Question 4
Marks: 0/1
A woman having individual psychodynamic psychotherapy with you comes in distressed saying she
has had a dream about her father sexually abusing her. As a therapist, you would:
Choose one answer.
Error! Not a valid embedded a. Acknowledge her distress and ask her to discuss her dream with
object. you.
Error! Not a valid embedded b. Tell her you can stop her father from abusing her by contacting the
object. authorities.
Error! Not a valid embedded
c. Reassure her that she has not been abused.
object.
Error! Not a valid embedded d. Acknowledge her distress and ask if she was abused by her father
object.
Error! Not a valid embedded
e. Explore the feelings it arouses in you
object.
In psychodynamic therapy, dreams are viewed as symbolised representations of the unconscious that
provide a means to access the id impulses. Manifest content of dreams need to be interpreted (dream
work) to identify the 'latent' content which would give clues to the unconscious conflict fuelling the id
impulses represented in the dream. in the scenario presented, the therapist would seek to explore in
detail the contents of the dream and ask the client to follow the steps involved in dreamwork (maintain
a record of dream, devote a section of the session to discuss dreams etc.)
Kaplan & Sadock Synopsis of Psychiatry 9th edition ( pp. 197). Lippincott Williams & Wilkins.
Incorrect
Marks for this submission: 0/1.
Question 5
Marks: 0/1
Which of the following is a technique used in psychodynamic psychotherapy?
Choose one answer.
Error! Not a valid embedded object. a. Exploration of counter-transference
Error! Not a valid embedded object. b. Dream analysis
Error! Not a valid embedded object. c. Acting out
Error! Not a valid embedded object. d. Free association
Error! Not a valid embedded object. e. Exploration of parapraxes
Dynamic therapy is a focussed approach that deals more with the present than past childhood
experiences. Most often defenses, and transference-counter transference are taken up for interpretation.
Free association (although used in modified forms can be part of dynamic techniques), dream analysis
and exploration of parapraxes are techniques used in psychoanalytic psychotherapy. Acting out is not a
technique, it refers to the pathological behaviour that a patient uses as a substitute for remembering
what has happened in the past. for example the patient may leave the therapy to go and get drunk
Gelder, M. G., Lopez-Ibor Jr, J., & Andreasen, N. C. (2000). New Oxford textbook of psychiatry (Vol.
2, pp. 1421-1431). Oxford University Press; Rycroft, C. (1995). A critical dictionary of psychoanalysis.
London: Penguin Books
Incorrect
Marks for this submission: 0/1.
Question 6
Marks: 0/1
Bion's concept of the 'container/contained function' when applied to the context of therapy refers to:
Choose one answer.
Error! Not a valid a. The process used by the therapist to advise the patient to contain the
embedded object. feelings without letting them out.
Error! Not a valid b. The therapist's identification of the projected transference and the
embedded object. resultant interpretation and safe re-projection onto the group
Error! Not a valid c. Returning the patients' difficult feelings back to them in a way they can
embedded object. cope with.
Error! Not a valid d. Patients' exploration of their feelings and relating them to the
embedded object. surroundings.
Error! Not a valid
e. The containment of therapy within a timed framework
embedded object.
Bion's concept of 'container-content' helps in understanding the Klienian defense mechanism 'projective
identification' in the context of the dynamics between transference-counter transference. It also helps in
understanding how thinking and communication develop mutually. 'Container' refers to a thinking
function that the projected element encounters (usually the group members' projection). The therapist
then uses the transference and the resulting counter-transference to understand more about the group's
needs and defenses. (thus, in the therapeutic context, it refers to the therapist-patient dynamics).
Billow, R.M. (2003). Relational Variations of The 'Container-Contained'. Contemp. Psychoanal.,
39:27-50.
Incorrect
Marks for this submission: 0/1.
Question 7
Marks: 0/1
Which of the following terms refer to a technique that might be used by a psychodynamic
psychotherapist in individual therapy?
Choose one answer.
Error! Not a valid embedded object. a. Working through
Error! Not a valid embedded object. b. Confrontation
Error! Not a valid embedded object. c. Understanding maladaptive schema
Error! Not a valid embedded object. d. Acting out
Error! Not a valid embedded object. e. Mirroring
Confrontation refers to when a therapist draws attention to a particular aspect of a patient's behaviour
without offering an explanation (e.g. the patient may always be late to appointments without realising
it). Working through and acting out refer to processes occurring in the patient. Mirroring is a technique
used in group therapy and Rogerian therapy. Exploring maladaptive schema occurs in CBT.
Gelder, M. G., Lopez-Ibor Jr, J., & Andreasen, N. C. (2000). New Oxford textbook of psychiatry (Vol.
2, pp. 1421-1431). Oxford University Press; Rycroft, C. (1995). A critical dictionary of psychoanalysis.
London: Penguin Books
Incorrect
Marks for this submission: 0/1.
Question 8
Marks: 0/1
During therapy intake, a patient asks you 'What is transference?' . The most appropriate answer would
be that it refers to:
Choose one answer.
Error! Not a valid embedded a. The process of interpreting the patients thoughts and feelings
object.
Error! Not a valid embedded
b. Bringing up past relationships in therapy
object.
Error! Not a valid embedded
c. Empathy displayed by the therapist
object.
Error! Not a valid embedded
d. The therapeutic alliance
object.
Error! Not a valid embedded
e. The therapist's response to the patient
object.
Transference refers to the unintentional redirection of feelings experienced in the past with another
person onto the therapist. It is the process by which the patient's unconscious feelings and conflicts that
were a part of a significant past relationship get 'transferred on' to the therapist during the course of
therapy. Working through transference is integral to the process of recovery.
Gelder, M. G., Lopez-Ibor Jr, J., & Andreasen, N. C. (2000). New Oxford textbook of psychiatry (Vol.
2, pp. 1437). Oxford University Press.
Incorrect
Marks for this submission: 0/1.
Question 9
Marks: 0/1
Which of the following patient characteristics is associated with successful psychodynamic
psychotherapy?
Choose one answer.
Error! Not a valid embedded
a. The patient must be able to set an agenda with the therapist
object.
Error! Not a valid embedded b. The patient must be able to tolerate frustration in relationships
object.
Error! Not a valid embedded
c. The patient must be able to take part in role play
object.
Error! Not a valid embedded
d. The patient must be able to complete homework tasks
object.
Error! Not a valid embedded e. The patient must be willing to be educated about his or her
object. symptoms
Besides having above average intelligence and high motivation, some of the patient characteristics that
are related to good outcome in psychodynamic psychotherapy include high ego strength, good
frustration tolerance, psychological mindedness, and flexible defenses. Homework tasks, agenda
setting, role play and education are key components of CBT.
Gelder, M. G., Lopez-Ibor Jr, J., & Andreasen, N. C. (2000). New Oxford textbook of psychiatry (Vol.
2, pp. 1421-1431). Oxford University Press.
Incorrect
Marks for this submission: 0/1.

CBT & other behavioural therapies

1
Marks: 0/1
You are asked to manage a 22 years old woman with a history of disordered eating. She gives a 5
months history of binge eating and self-induced vomiting. She does not appear to be depressed. You
calculate her body mass index as 22.5. Which of the following is the preferred treatment in the first
instance?
Choose one answer.
Error! Not a valid embedded object. a. Drug therapy
Error! Not a valid embedded object. b. Group therapy
Error! Not a valid embedded object. c. Self-help
Error! Not a valid embedded object. d. Cognitive behavioural therapy
Error! Not a valid embedded object. e. Interpersonal therapy
The history is suggestive of bulimia nervosa. CBT should be considered the preferred
psychotherapeutic treatment for bulimia nervosa. CBT is more effective than IPT. There is less
research to support the use of group therapy. CBT is more effective than antidepressants in the short
term.
Agras S, Walsh T, Fairburn C et al (2000) A multi-center comparison of cognitive-behavioural and
interpersonal therapy for bulimia nervosa. Arch. Gen. Psychiatry 57, 459-466
Incorrect
Marks for this submission: 0/1.
Question 2
Marks: 0/1
You are asked to treat a 25 year old women with psychotherapy. The patient is asking for therapy to
find solace and courage, and to introspect about her feelings and her life. Which of the following
therapies is not suited to her needs since she does not have any specific symptoms:
Choose one answer.
Error! Not a valid embedded object. a. Psychoanalysis
Error! Not a valid embedded object. b. Existential therapy
Error! Not a valid embedded object. c. Family therapy
Error! Not a valid embedded object. d. Humanist therapy
Error! Not a valid embedded object. e. Cognitive Behavioural Therapy
CBT combines behavioural, cognitive and experimental techniques to treat patients. The patient must
be seeking a change in his symptoms or a practical or specific target rather than self exploration.
Johnstone, E. C., Cunningham Owens, D. G., Lawrie, S. M., Sharpe, M., & Freeman, C. P. L. (2004).
Companion to psychiatric studies (7th ed., pp 310-327)
Incorrect
Marks for this submission: 0/1.
Question 3
Marks: 0/1
Your are referred a 37 year old women who has suffered from persistent, severe and distressing non-
localised abdominal pain for the past 7 years. Her pain cannot be fully explained by a physiological
process or physical disorder and her GP wonders if she would benefit from a trial of psychotherapy.
Which form of psychotherapy would you recommend?
Choose one answer.
Error! Not a valid embedded
a. Exposure techniques and humanist therapy
object.
Error! Not a valid embedded b. Family intervention therapy and behavioural skills training
object.
Error! Not a valid embedded
c. Cognitive therapy and behaviour therapy
object.
Error! Not a valid embedded
d. Psychodynamic therapy and humanist therapy
object.
Error! Not a valid embedded
e. Systematic desensitization and exposure techniques
object.
CBT is psychological treatment of choice for chronic pain and chronic fatigue syndrome
Gelder, M. G., Lopez-Ibor Jr, J., & Andreasen, N. C. (2000). New Oxford textbook of psychiatry (Vol.
2, pp. 1111). Oxford University Press.
Incorrect
Marks for this submission: 0/1.
Question 4
Marks: 0/1
Which of the following is the most effective treatment for Post Traumatic Stress Disorder?
Choose one answer.
Error! Not a valid embedded object. a. Debriefing
Error! Not a valid embedded object. b. Eye Movement Desensitization and Reprocessing
Error! Not a valid embedded object. c. Stress management
Error! Not a valid embedded object. d. Interpersonal therapy
Error! Not a valid embedded object. e. Cognitive analytic therapy
Individual Trauma Focussed CBT or EMDR should be the first line treatment in the management of
PTSD. Though both share similar principles, NICE guidelines has considered them separately as they
have difficult theoretical constructs. Drug treatments (paroxetine or mirtazapine for general use, and
amitriptyline or phenelzine for initiation only by mental health specialists) should be considered for the
treatment of PTSD in adults who express a preference not to engage in trauma-focused psychological
treatment. If one of the options would have been Cognitive behaviour therapy, then that would have the
correct option.
NICE Guidance The management of PTSD in adults and children in primary and secondary care (2005)
Incorrect
Marks for this submission: 0/1.
Question 5
Marks: 0/1
A 9 year old boy is referred to your team for an assessment of attention and behavioural difficulties.
His parents tell you that he has just finished a course of therapy for a prolonged episode of bed-wetting.
Your patient was most likely to have been previously treated by:
Choose one answer.
Error! Not a valid embedded object. a. A psychodynamic therapist
Error! Not a valid embedded object. b. A physical therapist
Error! Not a valid embedded object. c. A family intervention therapist
Error! Not a valid embedded object. d. A cognitive therapist
Error! Not a valid embedded object. e. A behaviour therapist
Behavioural methods are effective in the management of secondary enuresis (wetting after a period of
dryness). Star charts may be used for younger children; a pad-and-bell enuresis alarm may also be used;
the child may also be encouraged to help change the
Gelder, M. G., Lopez-Ibor Jr, J., & Andreasen, N. C. (2000). New Oxford textbook of psychiatry (Vol.
2, pp. 1791-1792). Oxford University Press.
Incorrect
Marks for this submission: 0/1.
Question 6
Marks: 0/1
You are managing a patient with agoraphobia. You decide to treat her by riding with her on a bus.
What is this treatment called?
Choose one answer.
Error! Not a valid embedded object. a. Flooding
Error! Not a valid embedded object. b. Object-relations
Error! Not a valid embedded object. c. Rational emotive behavioural therapy
Error! Not a valid embedded object. d. Systematic desensitization
Error! Not a valid embedded object. e. Transference
Flooding occurs when a patient is rapidly exposed to an anxiety-provoking stimulus in real life. The
patient is not allowed to withdraw from the stimulus until he or she feels calm and in control (this
process is called habituation). During the exposure
Gelder, M. G., Lopez-Ibor Jr, J., & Andreasen, N. C. (2000). New Oxford textbook of psychiatry (Vol.
2, pp. 1382-1384). Oxford University Press.
Incorrect
Marks for this submission: 0/1.
Question 7
Marks: 0/1
One of the post-therapy cognitive outcomes you would hope to achieve in someone with
hypochondriasis is:
Choose one answer.
Error! Not a valid embedded
a. Improved ability to deal with frustration
object.
Error! Not a valid embedded b. Better identification and reattribution of assumptions of
object. symptoms
Error! Not a valid embedded
c. The complete absence of thoughts of illness.
object.
Error! Not a valid embedded
d. Less distress at thoughts of illness
object.
Error! Not a valid embedded
e. Better role functioning
object.
Psychotherapy that is effective with patients with hypochondriasis mainly include modified versions of
CBT. Following the initial formulation (identification of triggers, extraction of relevance/meaning of
symptoms, and maintaining factors) treatment aims at behavioural modification (better self monitoring,
reduction in safety-seeking and reassurance-seeking behaviour), modification in cognitions (better
management of ruminations, beliefs/images, and reattribution of assumptions) and better anxiety
management. The distractors are behavioural/affective outcomes while the correct option is an example
of a desirable cognitive outcome.
Salkovsis, P. M., Warwick, H. M. C., & Dealse, A. C. (2003). Cognitive-behavioural treatment for
severe and persistent health anxiety (hypochondriasis). Brief Treatment and Crisis Intervention, 3:3,
353-367.
Incorrect
Marks for this submission: 0/1.
Question 8
Marks: 0/1
You are working as an ST4 in an addictions service. Your team ask you to prescribe disulfiram
(antabuse) for a patient who is recovering from alcohol dependency syndrome. What term might be
used to describe this treatment?
Choose one answer.
Error! Not a valid embedded object. a. Response prevention
Error! Not a valid embedded object. b. Aversion therapy
Error! Not a valid embedded object. c. Biofeedback
Error! Not a valid embedded object. d. Reciprocal inhibition
Error! Not a valid embedded object. e. Compliance therapy
Disulfiram-ethanol reaction causes symptoms such as flushing, headache, vomiting etc. which can
prove to be unpleasant. The use of Disulfiram thus amounts to the use of an aversive stimulus to help
the patient abstain from drinking. (In aversion therapy, the patient is given an unpleasant, negative,
stimulus when a specific behaviour is undesirable). The other options are incorrect as: Biofeedback =
Individuals receive information (feedback) about an aspect of their physiological state and then attempt
to alter that state. Reciprocal inhibition refers to when systematic desensitization is coupled with a
response that is incompatible with anxiety such as relaxation or eating. Reponses prevention = patients
abstain from performing rituals despite their strong urges to do so.
Gelder, M. G., Lopez-Ibor Jr, J., & Andreasen, N. C. (2000). New Oxford textbook of psychiatry (Vol.
1, pp. 500). Oxford University Press.
Incorrect
Marks for this submission: 0/1.
Question 9
Marks: 0/1
A 25 years old patient is referred to you with a history of repeated hand washing. You diagnose
Obsessive Compulsive Disorder with cleaning rituals and decide to treat the patient with
psychotherapy. Which of the following techniques would be the most appropriate to use?
Choose one answer.
Error! Not a valid embedded object. a. Psychodynamic psychotherapy
Error! Not a valid embedded object. b. Aversion therapy
Error! Not a valid embedded object. c. Exposure with Response Prevention
Error! Not a valid embedded object. d. Cognitive restructuring
Error! Not a valid embedded object. e. Thought stopping
The treatment of choice for obsessional rituals is a combination of response prevention with exposure
to any environmental cues that increase the symptoms. There is no good evidence that thought stopping
has a specific effect on its own, nor is there a strong evidence base for the use of psychodynamic
psychotherapy.
Gava I, Barbui C, Aguglia E, Carlino D, Churchill R, De Vanna M, McGuire HF. Psychological
treatments versus treatment as usual for obsessive compulsive disorder (OCD). Cochrane database of
systematic reviews 2007, Issue 2
Incorrect
Marks for this submission: 0/1.
Question 10
Marks: 0/1
A 44 year old man with a history of low mood and insomnia is referred to you by his GP. You assess
him in an outpatient clinic and diagnose depression. At the follow up appointment you decide to refer
him for cognitive-behavioural therapy (CBT). When you discuss this with the patient he asks you to tell
him about the evidence for using CBT. Which of the following statements about the evidence base for
CBT in treating adult depression is correct?
Choose one answer.
Error! Not a valid
a. CBT is less effective than pharmacotherapy in reducing later relapse
embedded object.
Error! Not a valid
b. CBT needs to continue for at least 16 weeks in order to be effective
embedded object.
Error! Not a valid c. There is good evidence that CBT when combined with medications is
embedded object. superior to either treatment alone
Error! Not a valid
d. CBT has a clear role in maintenance therapy
embedded object.
Error! Not a valid
e. CBT is as effective as psychodynamic psychotherapy
embedded object.
CBT is effective when used for at least 16/52 and is as effective as pharmacotherapy in reducing later
relapse. CBT may also be considered superior to other brief therapies. Its role in maintenance therapy is
unclear. There is no consensus as to whether CBT + medications is better than either modality.
Dept. of Health (2001) 'Treatment Choice in Psychological Therapies and Counselling -
Evidence Based Clinical Practice Guidelines'. London; Gaffan, EA, Tsaousis, I & Kemp Wheeler, SM
(1995). Researcher allegiance and meta-analysis: the case for cognitive therapy for depression. Journal
of Consulting and Clinical Psychology, 63, no. 6, 996-80
Incorrect
Marks for this submission: 0/1.
Question 11
Marks: 0/1
A patient who reaches a conclusion for which there is no evidence is experiencing which type of
automatic thought?
Choose one answer.
Error! Not a valid embedded object. a. All or nothing thinking
Error! Not a valid embedded object. b. Over-generalisation
Error! Not a valid embedded object. c. Arbitrary inference
Error! Not a valid embedded object. d. Selective abstraction
Error! Not a valid embedded object. e. Catastrophising
Arbitrary inference refers to the process of reaching a conclusion when there is no evidence for it and
even some against it; Selective abstraction - focusing on a specific detail and ignoring the more
important features of a situation;
All or nothing thinking - 'Black and White thinking'; the tendency to see events as completely good or
completely bad;
Personalisation - relating external events to oneself in an unwarranted fashion;
Catastrophising - viewing events as a total catastrophe;
Over-generalisation - reaching a general conclusion on the basis of a single event;
Gross, R. (2005). Psychology: The science and mind of behaviour (5th Ed. Pp. 826-827). Hodder
Arnold.
Incorrect
Marks for this submission: 0/1.

Evidence base for psychotherapy


1
Marks: 0/1
With regards to the evidence base for the use of CBT in bulimia nervosa which of the following
statements is true?
Choose one answer.
a. The efficacy of CBT in bulimia nervosa is unclear
b. CBT is more effective than IPT
c. CBT should ideally take place over a period of 5 months
d. CBT is effective in reducing binging
e. CBT, although useful, is less effective than antidepressants in the short-term
The efficacy of cognitive / cognitive-behavioural therapy has been established (Lewandowski et al,
1997); CBT should be intensive, occurring over 5 weeks, and involve diary keeping (self-monitoring).
It is most effective for attitudes to weight and shape. It is more effective than antidepressants in the
short-term. CBT combined with antidepressants may be better (Mitchell et al, 2001). Less research has
been carried out on other forms of therapy, but there are indications that family therapy (Sandberg et al,
1997) and interpersonal approaches may be effective. CBT is more effective than IPT (Agras et al,
2000)
Lewandowski LM, Gebing TA, Anthony JL, O'Brien WH (1997) Meta-analysis of cognitive-
behavioural treatment studies for bulimia. Clinical Psychology review, 17(7), 703-718; 27. Mitchell JE,
Peterson CB, Myers T, Wonderlich S (2001) Combining pharmacotherapy and psychotherapy in the
treatment of patients with eating disorders. Psychiatr. Clin. North. Am. 24, 315
Incorrect
Marks for this submission: 0/1.
Question 2
Marks: 0/1
Dialectical behaviour therapy for personality disorder:
Choose one answer.
a. Increases the use of medication
b. Has been shown to be effective in paranoid personality disorder
c. May be used effectively on inpatients
d. Is effective in reducing drug misuse
e. Is a watered down version of CBT
Linehan's DBT has been shown to be effective in the management of Borderline Personality Disorder.
There are 4 primary modes of treatment delivery: individual therapy, telephonic contact between
sessions, group sessions for skills training, and therapist consultation groups. DBT can be carried out
effectively in in-patient settings.
Bateman, A. W. & Tyrer, P. (2004). Psychological treatment for personality disorders. Advances in
Psychiatric Treatment, 10, 378-388
Incorrect
Marks for this submission: 0/1.
Question 3
Marks: 0/1
Existing psychotherapy research suggests which of the following?
Choose one answer.
a. Problem solving therapy does not appear to reduce deliberate self-harm
b. CBT and psychoanalytic psychotherapy are effective in increasing weight in anorexia
c. Supportive therapy is effective in generalised anxiety disorder
d. Psychoeducation is effective in hypochondriasis
e. There is a specific role for psychosocial interventions in the management of conversion
disorders
CBT, psychoanalytic psychotherapy and Family Therapy are effective in increasing weight (Dare et al,
2001)
Dare C, Eisler I, Russell G et al (2001) Psychological therapies for adults with anorexia nervosa:
randomised controlled trial of out-patient treatments. Br. J. Psychiatry 178, 216
Incorrect
Marks for this submission: 0/1.
Question 4
Marks: 0/1
Existing psychotherapy research suggests which of the following?
Choose one answer.
a. There is clear evidence for differential efficacy between the psychotherapies used in the
treatment of depression in childhood
b. Family intervention in schizophrenia increases relapse and readmission rates
c. Psychological therapy is efficacious in the treatment of bulimia
d. Family interventions are a useful adjunct in treating patients with bipolar disorder
e. Borderline personality disorder is untreatable
Psychological therapy is efficacious in the treatment of bulimia (Hartmann et al, 1992). The efficacy of
cognitive / cognitive-behavioural therapy has been established (Lewandowski et al, 1997)
Hartmann A, Herzog T, Drinkmann A (1992) Psychotherapy of bulimia nervosa: What is effective? A
meta-analysis. Journal of Psychosomatic Research, 36(2), 159-167
Incorrect
Marks for this submission: 0/1.
Question 5
Marks: 0/1
Your colleague asks you what has been shown to predict a good response in psychodynamic
psychotherapy. In response you would list all of the following options except:
Choose one answer.
a. Good response to trial interpretation
b. Above average intelligence
c. Previous response to therapy
d. Focality of the presenting complaint
e. Patient's ability to understand things in psychological terms
In the context of psychodynamic therapy, previous response to therapy has not been considered crucial
in determining good outcome. Instead, factors such as those listed as distractors (especially
'psychological mindedness') along with good frustration tolerance (ego strength), high motivation for
change, and presence of at least one past meaningful relationship have been identified as crucial in
facilitating recovery.
Charman, D. P. (2004). Core processes in brief psychodynamic psychotherapy: Advancing effective
practice (pp. 23-46). London: Lawrence Erlbaum Associates, Publishers. Also see Gelder, M. G.,
Lopez-Ibor Jr, J., & Andreasen, N. C. (2000). New Oxford textbook of psychiatry (Vol. 2, pp. 1421-
1431). Oxford University Press.
Incorrect
Marks for this submission: 0/1.
Question 6
Marks: 0/1
A 14 year old boy presents with a 4 week history of the core symptoms of depression. His parents have
recently divorced and he is experiencing frequent suicidal thoughts. Which of the following treatments
is the most appropriate?
Choose one answer.
a. SSRI and psychotherapy
b. Psychotherapy
c. Tricyclic antidepressant and psychotherapy
d. Tricyclic antidepressant
e. SSRI
For young people aged 12-18 years offer fluoxetine in addition to psychological therapy.
Depression in children and young people: identification and management in primary, community and
secondary care. NICE guidance (2005).
Incorrect
Marks for this submission: 0/1.

Other therapeutic approaches


1
Marks: 0/1
In the practice of supportive psychotherapy:
Choose one answer.
a. Fundamental change is a realistic goal
b. Patients' awareness of their clinical condition is not enhanced
c. Focus is on suppression and repression
d. Assets and achievements are minimised
e. Therapist support is a core component
Bloch (1979) gives a useful analysis of the key components of supportive psychotherapy.
Reassurance is critical. It is necessary to remove doubts and misconceptions, and focus on assets. If
reassurance is to be effective it must be realistic. The aim is to create a climate of hope and positive
expectation.
A thorough and detailed explanation of the illness should be given, focused on the here and now. The
emphasis is very much on the external reality the patient faces. The overriding aim is to improve the
individual's coping ability rather than enhance self-3) awareness. Repeated reality-testing is the key.
Direct advice is acceptable and indeed desirable. The ultimate aim is to produce transferable skills. The
individual should not only develop improved coping skills but also know when to seek help.
Suggestion by the clinician can result in change by influencing the patient both implicitly and
explicitly.
Encouragement can promote self-esteem, prevent feelings of inferiority and facilitate more appropriate
patterns of behaviour.
Environmental change may often be necessary in order to effect significant change.
Sympathetic active listening, unconditional acceptance and allowing catharsis ensure the patient a safe
and secure environment, thereby facilitating full and frank disclosure.
Junaid O and Hegde S (2007) Supportive psychotherapy in dementia. Advances in Psychiatric
Treatment 13: 17-23.
Incorrect
Marks for this submission: 0/1.
Question 2
Marks: 0/1
In systemic family therapy, which technique helps the therapist to maintain an objective approach and
clarify the gains and losses of each family member?
Choose one answer.
a. Circular Questioning
b. Relabeling
c. Neutrality
d. Hypothesising
e. Prescribing the symptom
Hypothesising before the session to ensure an objective approach, to organise questioning and to clarify
the gains and losses of each family member and neutrality (where the therapist consciously avoids
taking sides). Symptomatic behaviour is relabelled as positive in a process called positive connotation.
Gelder, M. G., Lopez-Ibor Jr, J., & Andreasen, N. C. (2000). New Oxford textbook of psychiatry (Vol.
2, pp. 1472-1481). Oxford University Press.
Incorrect
Marks for this submission: 0/1.
Question 3
Marks: 0/1
You are treating a 29 year old depressed male with cognitive analytic therapy. Your patient tells you
that either he controls others or he feels abused by them. Which term best describes your patients
pattern of thinking?
Choose one answer.
a. Dilemma
b. Negative automatic thought
c. Snag
d. Personalisation
e. Trap
Cognitive Analytic therapy is a time-limited psychotherapy that was developed by Ryle to treat
'neuroses'. Inspired by Vygotsky's ideas, It essentially integrates concepts of cognitive therapy and
psychodynamic approaches. Personalisation = relating external events to oneself in an unwarranted
fashion; 'Traps' = negative assumptions generate acts that produce consequences, which in turn
reinforce assumptions; 'Dilemmas' - a person acts as if available actions or possible roles are limited
and polarised (false dichotomies) and so resist change i.e. 'either/or' situations - closeness / separation;
control/passivity; control of feelings / emotional chaos
'Snags' - appropriate goals or roles are abandoned either because others would oppose them or they are
thought to be 'forbidden' or 'dangerous' in light of personal beliefs
Ryle, A., & Kerr, I. B. (2003). Cognitive Analytic Therapy. British Journal of Psychiatry. 2003, 83:79
Incorrect
Marks for this submission: 0/1.
Question 4
Marks: 0/1
Which of the following is a feature of interpersonal therapy?
Choose one answer.
a. Group skills training
b. Validation
c. Identifying traps and dilemmas
d. Consideration of role conflicts
e. Prioritizing a hierarchy of target behaviours
Forming a hierarchy of target behaviours, validation, and group skills training are components of DBT.
Identifying traps, snags and dilemmas forms part of CAT. IPT pays particular attention to
"interpersonal deficits", role conflicts, life transitions & losses.
Gelder, M. G., Lopez-Ibor Jr, J., & Andreasen, N. C. (2000). New Oxford textbook of psychiatry (Vol.
2, pp. 1411-1421). Oxford University Press; for DBT see Marsha Lineham (1993)
Incorrect
Marks for this submission: 0/1.
Question 5
Marks: 0/1
Which one of the following terms is a key principle in the work of therapeutic community?
Choose one answer.
a. Dependence
b. Reality confrontation
c. Basic assumption
d. Fight-flight
e. Pairing
Reality confrontation is integral to the working of TCs as it ensures that members are grounded to
'reality' by constant provision of feedback and interpretations of their behaviour (thus eliminating room
for denial and distortions). Rapaport (1960), an anthropologist who studied the Henderson Hospital,
described four principles that have defined the work of therapeutic communities - democracy, reality
confrontation, permissiveness and communality
Gelder, M. G., Lopez-Ibor Jr, J., & Andreasen, N. C. (2000). New Oxford textbook of psychiatry (Vol.
2, pp. 1483-1490). Oxford University Press.
Incorrect
Marks for this submission: 0/1.
Question 6
Marks: 0/1
Which of the following is a feature of systemic family therapy?
Choose one answer.
a. Relabeling
b. Externalising the problem
c. Circular questioning
d. Prescribing the symptom
e. The structure of the family is made explicit
Relabeling, externalising the problem, & prescribing the symptom are techniques used in Strategic
Family Therapy. Making the structure of the family explicit is a feature of structural family therapy.
Gelder, M. G., Lopez-Ibor Jr, J., & Andreasen, N. C. (2000). New Oxford textbook of psychiatry (Vol.
2, pp. 1472-1481). Oxford University Press.
Incorrect
Marks for this submission: 0/1.
Question 7
Marks: 0/1
With regards to the theories of structural family therapy, what is the likely effect of a family having
rigid boundaries?
Choose one answer.
a. Rigid boundaries will prevent family members from becoming autonomous
b. Rigid boundaries will promote engagement
c. Rigid boundaries prevent alignments
d. Rigid boundaries will encourage healthy relationships
e. Rigid boundaries will prevent communication
Rigid boundaries prevent communication and cause disengagement. Clear boundaries encourage
healthy relationships. Permeable boundaries prevent family members from becoming autonomous.
Alignments occur when two family members join together or are in opposition with each other.
Alignments can occur with all three types of boundary.
Gelder, M. G., Lopez-Ibor Jr, J., & Andreasen, N. C. (2000). New Oxford textbook of psychiatry (Vol.
2, pp. 1472-1481). Oxford University Press.
Incorrect
Marks for this submission: 0/1.
Question 8
Marks: 0/1
Which of the following terms is the key principle in the work of therapeutic community?
Choose one answer.
a. Basic assumption
b. Dependence
c. Pairing
d. Fight-flight
e. Reality confrontation
There are four defining principles that are associated with Therapeutic Communities: democratisation,
permissiveness, communalism, and reality confrontation. Reality confrontation refers to the constant
feedback given to the residents regarding others' interpretation of their behaviours and interaction in
order to help the resident stay grounded (and avoid denial/distortion).
Gelder, M. G., Lopez-Ibor Jr, J., & Andreasen, N. C. (2000). New Oxford textbook of psychiatry (Vol.
2, pp. 1483-1489). Oxford University Press. Also see the original work: Rapoport, R. N. (1960)
Community as doctor. Tavistock, London.
Incorrect
Marks for this submission: 0/1.

EMIs - Psychotherapy
1
Marks: 0/3

Psychological treatment in group settings:

A. Cohesiveness
B. Conditioning
C. Counter-dependence
D. Dependence
E. Fight-flight
F. Free floating discussion
G. Interpreting transference
H. Pairing
I. Universality
J. Vicarious learning

From the options above, choose:

1. Two curative factors in group therapy.

2. Three factors that hinder working in groups.

3. Two factors that are found in psychodynamic groups.

Answers:

1.A & I.Yalom identified 11 curative factors in groups including cohesiveness (i.e. a sense of
belongingness) and universality (a sense that symptoms/problems are not specific to one group
member, but shared by many others) the other factors are: Installation of hope, imparting of
information, interpersonal learning, imitation of adaptive behaviour, catharsis,corrective
recapitulation of the family group, altruism, guidance, existential factors, and social skills.
2.D, E, H. Bion identified several factors that hinder group work including: dependence and fight-
flight response & Pairing. A group holds one of three basic assumptions, which may differ from
the beliefs of the individual members. The basic assumption of dependence arises from the group's
anxious need to depend absolutely on someone (usually the therapist) to protect the members,
satisfy all their desires and solve their problems.This corresponds to Klein‘s depressive position.
Collective belief in an enemy who can be dealt with only by attack or retreat forms the basic
assumption of fight-flight. This corresponds to Klein‘s paranoid-schizoid position.The basic
assumption of pairing defines the notion that some future event or person will come from outside
to solve all problems. For example, their may be hope for the pairing of two individual members of
the group that could lead to the ―birth‖ of a new solution for the group‘s problems.This corresponds
to Klein and Freud‘s oedipal fantasy.
3.F, G.Free-floating discussions (group-analytic equivalent of free-association) and Interpreting
transference (along with interpretation of defences and resistances, archaic and primordial
experiences) are important in the context of psychodynamic groups. Floukes also listed ‗group-
specific processes‘ as important factors.
Incorrect
Marks for this submission: 0/3.
Question 2
Marks: 0/3
PSYCHODYNAMIC PSYCHOTHERAPY

A. Abreaction
B. Acting Out
C. Dream Interpretation
D. Free Association
E. Hypnosis
F. Projective Identification
G. Rationalisation
H. Resistance
I. Transference
J. Working Through
For each of the following, select the most appropriate technique:
A 31-year-old lady repeatedly turns up late for her therapy sessions that are well underway. Off late,
she has even cancelled a few appointments. In the sessions, the therapist has noticed that she abruptly
changes topics or resorts to lengthy silences.
The therapist asks her to maintain a diary of her dreams and goes through her notes during the session,
asking her for elaborations and clarifications. He later records his notes and observations
She gradually starts finding herself dreaming of the therapist. She is touched that he understands her
and cares for her.
1. H – This scenario suggests resistance to therapy.
2. C – This describes the process of dream interpretation.
3. I – This is transference – the feelings and emotions that are unconsciously invoked by the therapist in
the patient
Incorrect
Marks for this submission: 0/3.
Question 3
Marks: 0/3
PSYCHOLOGICAL TREATMENT

A. Communication Analysis
B. Diary keeping
C. Free association
D. Goodbye letter
E. Homework
F. Lying on a couch
G. Parapraxis
H. Procedural sequence model
I. Role transitions
J. Systematic review
K. Time limited

For each of the following therapies, choose the THREE most appropriate features:

Cognitive Analytic Therapy (CAT)

Psychodynamic psychotherapy

Interpersonal Therapy (IPT)

1. D, H, K. Goodbye letter, procedural sequence model and time limited approach are associated with
CAT

2. C, G, F. Free association, lying on a couch and parapraxis are associated with psychodynamic
therapy

3. A, I, K. Role transitions, and a time limited approach are associated with IPT.
Incorrect
Marks for this submission: 0/3.

Neuropsychiatry
Head Injury & Stroke
1
Marks: 0/1
One of the following is an indicator of severity of head injury
Choose one answer.
a. speed of collision
b. MMSE after trauma
c. force of blow
d. speed of driver
e. Length of post traumatic amnesia
The Traumatic Coma Data Bank analyzed 780 patients with head injuries and identified 5 factors that correlated
with a poor outcome, as follows: (1) age older than 60 years, (2) initial GCS score of less than 5, (3) presence of
a fixed dilated pupil, (4) prolonged hypotension or hypoxia early after injury, and (5) presence of a surgical
intracranial mass lesion. Length of PTA is correlated with time for recovery and rehabilitation success.

Click here for reference


Incorrect
Marks for this submission: 0/1.
Question 2
Marks: 0/1
Frontal injury can result in
Choose one answer.
a. Hallucinations
b. Akathisia
c. Capgras syndrome
d. Visual loss
e. Moria
Signs and symptoms of frontal lobe damage include personality change (loss of drive, apathy, disinhibition,
antisocial behaviour, hypersexuality, lack of insight, prankish joking known as 'Witzelsucht', and a childlike
interest in stimuli known as 'moria'); impaired concentration, attention and ability to carry out planned activity;
contralateral spastic paresis if encroaches on motor cortex; Broca's aphasia.
Companion to Psychiatric Studies, Ed: Johnstone, Lawrie et al, Seventh edition 7 e P345
Incorrect
Marks for this submission: 0/1.
Question 3
Marks: 0/1
Which ONE of the following is NOT true about dementia due to head injury?
Choose one answer.
a. progressive
b. may gradually ameliorate
c. non progressive
d. emotional lability seen
e. may persist indefinitely
This is usually non progressive, but a h/o head trauma is a risk factor for development of dementia due to
neurodegenerative disorders
NMS Psychiatry,3 Edn,P-134
Incorrect
Marks for this submission: 0/1.
Question 4
Marks: 0/1
A traumatic brain injury (TBI) patient has eye opening to speech (but not spontaneously),
decorticate motor response, and disoriented speech. His Glasgow Coma Scale will be
Choose one answer.
a. 3
b. 10
c. 12
d. 8
e. 9
The Glasgow Coma Scale is commonly used to grade the severity of traumatic brain injury. The scale gives a
quantitative estimate of level of consciousness and neurological status based on patterns of eye opening, as
well as best verbal and motor responses. Glasgow coma scale scores between 13 and 15 define mild brain
injury, scores between 9 and 12 define moderate brain injury, and scores between 3 and 8 define severe injury
Click here for reference
Incorrect
Marks for this submission: 0/1.
Question 5
Marks: 0/1
Frequency of seizures in closed head injury is
Choose one answer.
a. 25%
b. 75%
c. 15%
d. 5%
e. 50%
Upto 5% of closed head injury patients developed seizures. Early posttraumatic seizures are not related to the
presence of intracerebral parenchymal damage on CT scan. The occurrence of early seizures do not usually
affect the mortality and outcome of moderate closed head injury patients.

Click here for reference


Incorrect
Marks for this submission: 0/1.
Question 6
Marks: 0/1
Which ONE of the following neurotransmitter changes does NOT occur post-head injury?
Choose one answer.
a. traumatic brain injury is associated acutely with an increase in cholinergic transmission
b. Excitotoxicity following traumatic brain injury is mediated by glutamate and acetylcholine
c. Decreased levels of 5-hydroxyindoleacetic acid (5-HIAA) are found with frontotemporal contusions
d. Acetyl choline was the first to be clearly implicated
e. Increase in intracellular magnesium is an important finding
A fall in intracellular magnesium levels has been shown to correlate with outcome of head injury in
experimental models
Organic Psychiatry,Lishman,3 rd Edition P 164
Incorrect
Marks for this submission: 0/1.
Question 7
Marks: 0/1
In the treatment of post-head injury epilepsy, which ONE of the following has the LEAST effect on cognition?
Choose one answer.
a. Phenobarbitone
b. Topiramate
c. Lamotrigine
d. Carbamazepine
e. Phenytoin
Among the newer anticonvulsant medications, lamotrigine has demonstrated fewer adverse cognitive effects
than topiramate.
Text book of Neuropsychiatry,2 edition, Schiffer,Rao,Fogel,p-1112
Incorrect
Marks for this submission: 0/1.
Question 8
Marks: 0/1
Features of pseudobulbar palsy include all except
Choose one answer.
a. brisk jaw jerk
b. dysphagia
c. emotional lability
d. fasciculation of tongue
e. dysphonia
Pseudobulbar palsy is a set of clinical signs on examination, not a diagnosis. The features include slowed slurred
speech, difficulty with swallowing, weakness of face, tongue, and swallowing muscles, a tendency for
uncontrollable laughter or crying, and brisk jaw and gag reflexes. The commonest causes of pseudobulbar palsy
are multiple bilateral strokes, multiple sclerosis, amyotrophic lateral sclerosis, progressive supranuclear palsy,
some forms of static and progressive childhood diseases, and some others

Acute pseudobulbar palsy as the initial presentation of intravascular lymphomatosis.


European Journal of Internal Medicine, Volume 15, Issue 2, Pages 128-130
A. Arboix, I. Costa, C. Basses, J. Sans-Sabrafen
Incorrect
Marks for this submission: 0/1.
Question 9
Marks: 0/1
Most common cognitive difficulty after brain injury is
Choose one answer.
a. constructional apraxia
b. Dysexecutive deficits
c. Affective instability
d. Memory deficits
e. Visuospatial deficits
Memory deficits are the most frequent chronic cognitive disturbance reported by patients and relatives in
traumatic brain injury. Memory dysfunction is characterized by both anterograde and retrograde deficits, faulty
sequencing of events, and inefficient encoding and storage strategies.
Companion to Psychiatric Studies, Ed: Johnstone, Lawrie et al, Seventh edition 7 e P345
Incorrect
Marks for this submission: 0/1.
Question 10
Marks: 0/1
The neuroimaging modality that is most helpful in acute stroke is
Choose one answer.
a. MRI
b. CT scan
c. PET
d. X Ray
e. SPECT
CT scan can be done relatively quickly, cheaper and picks up hemorrhage faster than MRI.
Click here for reference
Incorrect
Marks for this submission: 0/1.
Question 11
Marks: 0/1
The single greatest risk factor or traumatic brain injury is
Choose one answer.
a. Bipolar disorder
b. Substance use
c. Under 18 years age
d. Speeding in motorway
e. Poor sleep
Half of all TBIs are due to transportation accidents involving automobiles, motorcycles, bicycles, and
pedestrians. These accidents are the major cause of TBI in people under age 75.For those 75 and older, falls
cause the majority of TBIs. Approximately 20% of TBIs are due to violence, such as firearm assaults and child
abuse, and about 3% are due to sports injuries. Fully half of TBI incidents involve alcohol use.

Companion to Psychiatric Studies, Ed: Johnstone, Lawrie et al, Seventh edition 7 e P345
Incorrect
Marks for this submission: 0/1.
Question 12
Marks: 0/1
Risk factors for post head injury delirium include
Choose one answer.
a. occipital damage
b. female sex
c. Old age
d. smoking
e. left hemisphere lesions
Multiple conditions can contribute to the development of delirium in patients with traumatic brain injury.
These include structural brain damage, cerebral edema, secondary brain hypoxia, seizures, electrolyte
imbalance, infections, and substance (e.g., barbiturates, opioids, steroids, alcohol) withdrawal. Old age,
coexistent severe medical disease, polypharmacy, and lesions of the basal ganglia and right hemisphere have
also been shown to be significant risk factors
Companion to Psychiatric Studies, Ed: Johnstone, Lawrie et al, Seventh edition 7 e P346
Incorrect
Marks for this submission: 0/1.
Question 13
Marks: 0/1
The LEAST likely to cause extrapyramidal symptoms when used to treat behavioural problems associated with
head injury is
Choose one answer.
a. Risperidone
b. Aripiprazole
c. Amisulpiride
d. Olanzepine
e. Quetiapine
Among the newer atypical antipsychotic medications, quetiapine produces the fewest EPS
Maudsley Guidelines 2007, edition 9, p141
Incorrect
Marks for this submission: 0/1.

Epilepsy
1
Marks: 0/1
Which ONE of the following is NOT true about Epileptic automatism?
Choose one answer.
Error! Not a valid embedded object. a. occurs during or after seizure
Error! Not a valid embedded object. b. continuous EEG disturbance
Error! Not a valid embedded object. c. actions can be controlled
Error! Not a valid embedded object. d. clouding of consciousness
Error! Not a valid embedded object. e. performs actions without being aware
Complex partial seizures (TLE) are characterised by three As - Auras, Absences and Automatisms, in that order.
Complex movements and actions are not under voluntary control.
Organic Psychiatry,Lishman,3 rd Edition P 253
Incorrect
Marks for this submission: 0/1.
Question 2
Marks: 0/1
Which ONE of the following is NOT true regarding episodic dyscontrol?
Choose one answer.
Error! Not a valid embedded a. Anticonvulsants may be beneficial in violent subjects even if the EEG is
object. normal
Error! Not a valid embedded
b. h/o of childhood febrile seizures present
object.
Error! Not a valid embedded
c. Soft neurological signs are present
object.
Error! Not a valid embedded d. An abnormal EEG may predict a favourable response to anticonvulsants in
object. violent subjects
Error! Not a valid embedded
e. EEG abnormalities are consistently observed
object.
EEG abnormalities have not been consistently observed
Text book of Neuropsychiatry,2 edition, Schiffer,Rao,Fogel,p-1108-9
Incorrect
Marks for this submission: 0/1.
Question 3
Marks: 0/1
Which ONE of the following is seen in Frontal Lobe epilepsy?
Choose one answer.
Error! Not a valid embedded object. a. psychotic features common
Error! Not a valid embedded object. b. none of the above
Error! Not a valid embedded object. c. normal speech
Error! Not a valid embedded object. d. All of the above
Error! Not a valid embedded object. e. prolactin levels remain same after F L seizures
Prolactin levels are often not elevated after frontal lobe seizures
Organic Psychiatry,Lishman,3 rd Edition P 249
Incorrect
Marks for this submission: 0/1.
Question 4
Marks: 0/1
The axis for aetiology in the proposed five axis diagnostic scheme to classify seizures by International League
Against Epilepsy (ILAE) is
Choose one answer.
Error! Not a valid embedded object. a. Axis II
Error! Not a valid embedded object. b. Axis V
Error! Not a valid embedded object. c. Axis I
Error! Not a valid embedded object. d. Axis IV
Error! Not a valid embedded object. e. Axis III
In the ILAE proposed diagnostic scheme (International League Against Epilepsy Commission Report 2001a,
2001b), Axis IV is used to classify etiology (when etiology is known).Axis I is used to record ictal phenomenology
(a description of the ictal events). Axis II is used to record seizure type. The physician specifies the type from
the ILAE List of Epileptic Seizures, identifying localization within the brain or precipitating stimuli when
appropriate. Axis III is used to record the type of epileptic syndrome (when a syndrome diagnosis is possible).
The physician specifies the syndrome type from the ILAE List of Epilepsy Syndromes. Axis V is used to record
World Health Organization impairment classifications.
Click here for reference
Incorrect
Marks for this submission: 0/1.
Question 5
Marks: 0/1
Which ONE of the following diagnostic clues may help differentiate psychosis due to a central nervous system
lesion from schizophrenic psychosis?
Choose one answer.
Error! Not a valid embedded
a. Poor premorbid social history
object.
Error! Not a valid embedded
b. Rapid fluctuations in mental status
object.
Error! Not a valid embedded
c. Responsiveness to usual biological or psychological interventions
object.
Error! Not a valid embedded d. Presentation that meets DSM-IV (American Psychiatric Association 1994)
object. criteria
Error! Not a valid embedded
e. Gradual change in personality, mood, or ability to function
object.
Rapid fluctuations in mental status are indicative of a lesion of the central nervous system or of seizures
Organic Psychiatry,Lishman,3 rd Edition P 295
Incorrect
Marks for this submission: 0/1.
Question 6
Marks: 0/1
Which ONE of the following is NOT true about pseudoseizures?
Choose one answer.
Error! Not a valid embedded object. a. Plantar flexor reflexes are preserved
Error! Not a valid embedded object. b. Sleep-related seizures are infrequent
Error! Not a valid embedded object. c. Injury is infrequent
Error! Not a valid embedded object. d. Restraint accentuates the seizure
Error! Not a valid embedded object. e. Postictal features are typical
Postictal features are not present in nonepileptic seizures, also known as pseudoseizures.
Organic Psychiatry,Lishman,3 rd Edition P 292-4
Incorrect
Marks for this submission: 0/1.
Question 7
Marks: 0/1
The major laboratory test used in the diagnostic workup of seizures to enable differentiation from
pseudoseizures is
Choose one answer.
Error! Not a valid embedded object. a. Serum calcium
Error! Not a valid embedded object. b. Prolactin level
Error! Not a valid embedded object. c. Serum albumin
Error! Not a valid embedded object. d. Serum potassium
Error! Not a valid embedded object. e. Creatine phosphokinase (CPK)
Prolactin level is the only major laboratory test used in the diagnostic workup of seizures. A hormone secreted
by the anterior pituitary gland, serum prolactin, is released by epileptic activity spreading from the temporal
lobe to the hypothalamic-pituitary axis.
Organic Psychiatry,Lishman,3 rd Edition P 290
Incorrect
Marks for this submission: 0/1.
Question 8
Marks: 0/1
Consciousness is preserved in this subtype of seizures
Choose one answer.
Error! Not a valid embedded object. a. Complex partial seizures
Error! Not a valid embedded object. b. Absence seizures
Error! Not a valid embedded object. c. Simple partial seizures
Error! Not a valid embedded object. d. Status Epilepticus
Error! Not a valid embedded object. e. Generalised Tonic clonic
In simple partial seizures , no alteration in consciousness is present.
Organic Psychiatry,Lishman,3 rd Edition P 240
Incorrect
Marks for this submission: 0/1.
Question 9
Marks: 0/1
Which ONE of the following is NOT a features of temporal lobe seizures?
Choose one answer.
Error! Not a valid embedded object. a. depersonalization
Error! Not a valid embedded object. b. Absence of Psychiatric symptomatology
Error! Not a valid embedded object. c. visual hallucinations
Error! Not a valid embedded object. d. Cognitive abnormalities
Error! Not a valid embedded object. e. Panoramic memory
Temporal lobe epilepsy often presents with neurotic and psychotic presentations
Organic Psychiatry,Lishman,3 rd Edition P 250
Incorrect
Marks for this submission: 0/1.
Question 10
Marks: 0/1
Which ONE Of the following is NOT a recommended treatment for seizure disorder with concomitant
psychiatric symptoms?
Choose one answer.
Error! Not a valid embedded a. Optimize the addition of psychotropic medication by targeting specific
object. psychiatric symptoms
Error! Not a valid embedded
b. Evaluate the need for adjustment of the anticonvulsant
object.
Error! Not a valid embedded c. Anticipate possible drug interactions between anticonvulsant and
object. psychotropic medications
Error! Not a valid embedded d. Start with higher doses of psychotropic medications in order to rapidly treat
object. symptoms
Error! Not a valid embedded
e. Preferably use anticonvulsant monotherapy
object.
The psychiatrist should start with lower, not higher, doses of psychotropic medication and should wait until
symptoms stabilize before changing doses.
Text book of Neuropsychiatry,2 edition, Schiffer,Rao,Fogel,p-1104-5
Incorrect
Marks for this submission: 0/1.

Sleep disorders
1
Marks: 0/1
The most common parasomnia is
Choose one answer.
a. Nocturnal seizures
b. Sleep terrors
c. Sleepwalking
d. Nightmares
e. Bruxism
According to data from the national cooperative study of patients seen in sleep disorders center
(Coleman et al. 1982), the most commonly encountered parasomnias are those secondary to nocturnal
seizure activity (33.7%).
Coleman RM, Roffwarg HP, Kennedy SJ, et al: Sleep-wake disorders based on a polysomnographic
diagnosis. A national cooperative study. JAMA 247:997-1003, 1982
Incorrect
Marks for this submission: 0/1.
Question 2
Marks: 0/1
Which ONE of the following is the most common cause of excessive daytime sleepiness?
Choose one answer.
a. Sleep Apnoea
b. Narcolepsy
c. Kleine-Levin syndrome
d. Posttraumatic hypersomnia
e. Idiopathic hypersomnia
Sleep apnoea accounts for 43.2% of disorders associated with excessive daytime sleepiness.
Organic Psychiatry,Lishman,3 rd Edition P 731
Incorrect
Marks for this submission: 0/1.
Question 3
Marks: 0/1
The antidepressant that does not suppress REM sleep is
Choose one answer.
a. Fluoxetine
b. Desipramine
c. MAOI's
d. Bupropion
e. Paroxetine
Bupropion does not suppress REM sleep, and neither does nefazodone.
Comprehensive text book of psychiatry,8 the edition,p-294
Incorrect
Marks for this submission: 0/1.
Question 4
Marks: 0/1
Sudden arousal from sleep with intense fearfulness, which may begin with scream or cry is associated
with
Choose one answer.
a. Sleepwalking
b. Sleep terror
c. REM sleep behaviour disorder
d. Nightmare
e. Rhythmic movement disorder
Sleep terror is characterized by sudden arousal with intense fearfulness, which may begin with a
piercing scream or cry.
Organic Psychiatry,Lishman,3 rd Edition P 737
Incorrect
Marks for this submission: 0/1.
Question 5
Marks: 0/1
Which ONE of the following is NOT a common feature of Narcolepsy?
Choose one answer.
a. Excessive daytime sleepiness
b. Hypnopompic hallucinations
c. Hypnagogic hallucinations
d. Cataplexy
e. Sleep paralysis
Hypnopompic hallucinations are not commonly seen in narcolepsy
Organic Psychiatry,Lishman,3 rd Edition P 723
Incorrect
Marks for this submission: 0/1.
Question 6
Marks: 0/1
Which ONE of the following does NOT predispose to Obstructive sleep apnoea?
Choose one answer.
a. Obesity
b. Hyperthyroidism and acromegaly
c. Micrognathia or retrognathia
d. Middle age or older
e. Male sex
Hypothyroidism and acromegaly are predisposing factors for obstructive sleep apnea.
Organic Psychiatry,Lishman,3 rd Edition P 732
Incorrect
Marks for this submission: 0/1.
Question 7
Marks: 0/1
Which ONE of the following sleep abnormalities is NOT seen in depression?
Choose one answer.
a. Increase in REM density
b. Generalized sleep disturbance
c. Lengthening of latency to REM sleep
d. Occurrence of REM sleep earlier in the night
e. Increase in slow-wave sleep in the first non-rapid eye movement (NREM)-rapid eye movement
(REM) cycle
REM latency is shortened, not lengthened, in sleep disturbances in depressed patients.
Text book of Neuropsychiatry,2 edition, Schiffer,Rao,Fogel,p-385
Incorrect
Marks for this submission: 0/1.

Psychiatry & Neurology


1
Marks: 0/1
A lady with multiple sclerosis presents with depression. The drug most likely to be the cause is:
Choose one answer.
a. Amantadine
b. Baclofen
c. Beta interferon
d. Oxygen therapy
e. Steroids
Use of interferon beta is associated with depression and suicidal thoughts; these conditions are also
more common with MS. Any symptoms of depression should be immediately reported - patients should
be made aware that depression and/or suicidal thoughts may occur with interferon beta. Caution is also
advised in patients with a history of epilepsy and those with heart problems.
Expert Opin Drug Saf. 2007 May;6(3):279-88.
Incorrect
Marks for this submission: 0/1.
Question 2
Marks: 0/1
Which ONE of the following symptoms is LESS likely to be associated with temporal lobe tumors than
Schizophrenia?
Choose one answer.
a. Olfactory hallucinations
b. Tactile hallucinations
c. Visual hallucinations
d. Flat or inappropriate affect
e. "Spells" and dreamlike episodes
Flat or inappropriate affect, a classic symptom of schizophrenia, is not seen in patients with temporal
lobe tumors; rather, these patients often exhibit a broad range of affect and appropriate interpersonal
behaviour.
Organic Psychiatry,Lishman,3 rd Edition P 279
Incorrect
Marks for this submission: 0/1.
Question 3
Marks: 0/1
Which ONE of the following is NOT true about memory deficits in schizophrenia?
Choose one answer.
a. Patients frequently exhibit disorganized or inefficient retrieval strategies
b. Procedural learning is usually intact with few or no deficits
c. Deficits are more pronounced in tests of recognition than in tests of recall
d. Patients usually have little or no deficits in nondeclarative memory tests
e. Patients have shallow or inefficient encoding of information
Declarative memory deficits in schizophrenia are usually more pronounced in tests of recall than in
tests of recognition.
Text book of Neuropsychiatry,2 edition, Schiffer,Rao,Fogel,p-779
Incorrect
Marks for this submission: 0/1.
Question 4
Marks: 0/1
Which of the following is most common in delirium?
Choose one answer.
a. Labile mood
b. Disturbed sleep wake cycle
c. Hallucinations
d. Increased motor activity
e. Delusions
Common features of a delirious state are clouding of consciousness, altered sleep wake cycle,
agitation/overactive state or underactive/drowsy state and impaired cognition
Textbook of Organic Psychiatry
Incorrect
Marks for this submission: 0/1.
Question 5
Marks: 0/1
Which ONE of the following is NOT a symptom of orbitofrontal syndrome?
Choose one answer.
a. Perseveration
b. Hyperactivity
c. Impulsivity
d. lability of mood
e. Disinhibition
Perseveration is not a characteristic of orbitofrontal syndrome, but is seen with injury to the
dorsolateral frontal cortex.
Core psychiatry,2 edition, Wright ,stern & Phelan,p-384
Incorrect
Marks for this submission: 0/1.
Question 6
Marks: 0/1
Which ONE of the following is a feature of dorsolateral prefrontal syndrome?
Choose one answer.
a. Apathy
b. Perseveration
c. Poor initiation of tasks
d. Intact memory
e. Psychomotor retardation
This is associated with poor memory recall as evidenced by fluency test
Core psychiatry,2 edition, Wright ,stern & Phelan,p-384
Incorrect
Marks for this submission: 0/1.
Question 7
Marks: 0/1
Which ONE of the following is NOT commonly observed in amnesic patients?
Choose one answer.
a. Confabulations
b. Retrograde amnesia
c. Intact intellectual function
d. Aphasia
e. Anterograde amnesia
Aphasia is not a clinical characteristic typically found in amnesic patients.
Organic Psychiatry,Lishman,3 rd Edition P 8
Incorrect
Marks for this submission: 0/1.
Question 8
Marks: 0/1
Depressed patients are more likely to have deficits in which of the following domains than
nondepressed patients?
Choose one answer.
a. Automatic processing
b. Procedural skills
c. Nondeclarative memory
d. Incidental learning
e. Effortful processing
In general, depressed patients are more likely to manifest deficits on tasks that require effort or greater
depth of processing, but not on tasks that can be completed automatically.
Text book of Neuropsychiatry,2 edition, Schiffer,Rao,Fogel,p-28
Incorrect
Marks for this submission: 0/1.
Question 9
Marks: 0/1
Which drug given prophylactically is shown to decrease delirium:
Choose one answer.
a. Risperidone
b. Quetiapine
c. Olanzapine
d. Haloperidol
e. Amisulpiride
""
Journal of the American Geriatrics Society, Volume 54,Number 5, May 2006 , pp. 860-861
Incorrect
Marks for this submission: 0/1.
Question 10
Marks: 0/1
Which of the following treatments for multiple sclerosis is most likely to lead to depression?
Choose one answer.
a. Amantadine
b. Glatiramer acetate
c. Steroids
d. Interferon-beta
e. Baclofen
Use of interferon beta is associated with depression and suicidal thoughts; these conditions are also
more common with MS. Any symptoms of depression should be immediately reported - patients should
be made aware that depression and/or suicidal thoughts may occur with interferon beta. Caution is also
advised in patients with a history of epilepsy and those with heart problems.
Expert Opin Drug Saf. 2007 May;6(3):279-88.
Incorrect
Marks for this submission: 0/1.
Question 11
Marks: 0/1
Which of the following has the best evidence base for its use in post-stroke depression?
Choose one answer.
a. Mirtazepine
b. Amitriptyline
c. Fluoxetine
d. Sertraline
e. Reboxetine
Randomized controlled trials have demonstrated the efficacy of sertraline, citalopram and nortriptyline
to treat post-stroke depression. Whether antidepressant medication may help to prevent post-stroke
depression and decrease post-stroke mortality will require further controlled studies.
Expert Opin Pharmacother. 2008 May;9(8):1291-8.
Incorrect
Marks for this submission: 0/1.
Question 12
Marks: 0/1
The recommended treatment for pathological crying is:
Choose one answer.
a. Citalopram
b. Fluoxetine
c. Sertraline
d. Venlafaxine
e. Paroxetine
References to a disturbance in central serotoninergic neurotransmission have become frequent in the
literature, implicating this as an important etiological factor for pathological crying. In the present
communication three cases of successful treatment of pathological crying using the SSRI citalopram
are reported
Pharmacopsychiatry. 2001 Nov;34(6):254-8
Incorrect
Marks for this submission: 0/1.

Movement Disorders

1
Marks: 0/1
The most common disorder associated with resting tremor is
Choose one answer.
a. Hereditary, essential tremor
b. Parkinson's disease
c. Medication induced (caffeine, lithium).
d. Metabolic encephalopathy
e. Anxiety
Parkinson's disease is the most common disorder associated with rest tremor.
Text book of Neuropsychiatry,2 edition, Schiffer,Rao,Fogel,p-939
Incorrect
Marks for this submission: 0/1.
Question 2
Marks: 0/1
Which ONE of the following is NOT a common comorbidity of Huntington's disease?
Choose one answer.
a. Obsessive-compulsive disorder
b. Intermittent explosive disorder
c. Schizophreniform disorder
d. Bipolar disorder
e. Major depression
Obsessive-compulsive disorder is infrequently comorbid with Huntington's disease.
Organic Psychiatry,Lishman,3 rd Edition P 470-2
Incorrect
Marks for this submission: 0/1.
Question 3
Marks: 0/1
The psychiatric disorder most commonly found in patients with Parkinson's disease is
Choose one answer.
a. Affective disorder
b. Obsessive-compulsive disorder
c. Intermittent explosive disorder
d. Generalized anxiety disorder
e. Psychosis
Affective disorder is the most common psychiatric disturbance in Parkinson's disease, with an
estimated incidence of from 20% to 90%.
Text book of Neuropsychiatry,2 edition, Schiffer,Rao,Fogel,p-736
Incorrect
Marks for this submission: 0/1.
Question 4
Marks: 0/1
Which ONE of the following are clinical features is NOT found in diffuse Lewy body disease?
Choose one answer.
a. Parkinsonism
b. Dementia
c. Attentional impairment
d. Fluctuations in cognitive functioning
e. Apathy
Apathy is not a clinical characteristic of diffuse Lewy body disease.
Text book of Neuropsychiatry,2 edition, Schiffer,Rao,Fogel,p-936
Incorrect
Marks for this submission: 0/1.
Question 5
Marks: 0/1
Which ONE of the following terms describes sudden jerks, sometimes simple, but sometimes as
complex as well-organized voluntary movements, that may be temporarily relieved during sleep or
when concentrating on an enjoyable task?
Choose one answer.
a. Tics
b. Chorea
c. Asterixis
d. Myoclonus
e. Stereotypies
Tics are sudden jerks which can be simple or as complex as organized voluntary movements, such as
touching an object or speaking a word.
Text book of Neuropsychiatry,2 edition, Schiffer,Rao,Fogel,p-949
Incorrect
Marks for this submission: 0/1.
Question 6
Marks: 0/1
The following clinical finding is helpful in distinguishing progressive supranuclear palsy (PSP) from
Parkinson's disease?
Choose one answer.
a. Dementia
b. Axial parkinsonism with tremor
c. Vertical eye movement abnormalities
d. Stable gait
e. Postural stability
PSP may be distinguished from Parkinson's disease and other progressively degenerative motor
disorders by the following constellation of clinical findings: axial parkinsonism without tremor,
unstable gait with postural instability, and vertical eye movement abnormalities without alien limb
movements
Text book of Neuropsychiatry,2 edition, Schiffer,Rao,Fogel,p-936
Incorrect
Marks for this submission: 0/1.
Question 7
Marks: 0/1
Wilson's disease is associated with abnormal liver function and degeneration of which ONE of the
following regions?
Choose one answer.
a. Hippocampus
b. Amygdala
c. Basal ganglia
d. Corpus callosum
e. Thalamus
In Wilson's disease, also called hepatolenticular degeneration, the basal ganglia degenerate in
association with abnormalities in liver function.
Organic Psychiatry,Lishman,3 rd Edition P 662
Incorrect
Marks for this submission: 0/1.
Question 8
Marks: 0/1
Which ONE of the following is NOT a lower motor neurone lesion?
Choose one answer.
a. Pseudobulbar palsy
b. bulbar palsy
c. MND
d. GBS
e. Syringomyelia
Pseudobulbar palsy is a Upper Motor Neurone Lesion
Organic Psychiatry,Lishman,3 rd Edition P 338
Incorrect
Marks for this submission: 0/1.

HIV & AIDS


1
Marks: 0/1
Which ONE of the following antidepressants should be avoided in depression secondary to HIV/AIDS?
Choose one answer.
a. Nefazodone
b. Bupropion
c. Citalopram
d. Escitalopram
e. Mirtazapine
Nefazodone should be avoided in the treatment of depression in HIV/AIDS patients, because of its
affinity for cytochrome P450 (leading to drug-drug interactions) and its propensity to cause
hepatotoxicity.
Text book of Neuropsychiatry,2 edition, Schiffer,Rao,Fogel,p-1025
Incorrect
Marks for this submission: 0/1.
Question 2
Marks: 0/1
Which ONE of the following is the commonest CNS neoplasm in patients with HIV/AIDS?
Choose one answer.
a. Primary non Hodgkins Lymphoma
b. Plasmocytomas
c. Lymphoid granulomatosis
d. Metastatic deposits from Kaposi's sarcoma
e. Secondaries from Lymphomas
Primary B cell non hodgkins is the commonest CNS neoplasm
Organic Psychiatry,Lishman,3 rd Edition P 320
Incorrect
Marks for this submission: 0/1.
Question 3
Marks: 0/1
The most common opportunistic infection in AIDS, which may present as a focal or diffuse cognitive
or affective disturbance, is
Choose one answer.
a. Cytomegalovirus
b. Progressive multifocal leukoencephalopathy
c. Cryptococcus neoformans
d. Syphilis
e. Toxoplasma gondii
Toxoplasma gondii is perhaps the most common opportunistic infection in AIDS and may present as a
focal or diffuse cognitive or affective disturbance. Clinically, toxoplasmosis symptoms include malaise,
confusion, lethargy, headache, fever, and focal deficits.
Organic Psychiatry,Lishman,3 rd Edition P 320
Incorrect
Marks for this submission: 0/1.

Neurophysiology
1
Marks: 0/1
Which ONE of the following stages of sleep does NOT correspond to their characteristic EEG feature?
Choose one answer.
a. stage 2-sleep spindles and K complexes
b. stage 3-delta activity
c. stage4-delta activity
d. stage1-theta activity
e. Stage0-beta activity
Alpha activity is characteristic feature of the EEG during stage 0 sleep - transition from awake state to
sleep state.
NMS Physiology, 2 edition P79
Incorrect
Marks for this submission: 0/1.
Question 2
Marks: 0/1
Which of the following statements regarding the EEG in cognitive disorders is true?
Choose one answer.
a. Mild diffuse slowing is pathological in individuals older than 75 years.
b. EEG slowing is correlated with the severity of dementia and the number of senile plaques in
Alzheimer's disease.
c. Focal delta activity in the anterior temporal areas is rare in patients older than 60 years.
d. A drop in background alpha of 1-2 Hz over a short period of time is not indicative of delirium
as long as the alpha rhythm remains in the normal range.
e. Low-voltage beta activity increases in adults up to age 60 years and remains stable thereafter.

EEG slowing has been found to be correlated with the severity of dementia and the number of senile
plaques in Alzheimer's disease.
Organic Psychiatry,Lishman,3 rd Edition P 439
Incorrect
Marks for this submission: 0/1.
Question 3
Marks: 0/1
Which ONE of the following EEG changes occurs with use of tricyclic antidepressants?
Choose one answer.
a. no effect on EEG
b. reduce theta activity
c. reduce beta activity
d. all the above
e. increase delta activity
TCA's increase delta, theta and beta rhythms
Revision notes in Psychiatry,P 144-5
Incorrect
Marks for this submission: 0/1.
Question 4
Marks: 0/1
Which ONE of the following EEG changes is associated with the use of benzodiapines?
Choose one answer.
a. doesn't affect EEG
b. reduce theta rhythm
c. reduce alpha rhythm
d. all the above
e. reduce beta rhythm
Benzodiapines reduce the alpha rhythm, increase beta and theta rhythm
Revision notes in Psychiatry,P 144-5
Incorrect
Marks for this submission: 0/1.
Question 5
Marks: 0/1
Which ONE of the following EEC changes is caused by antipsychotics?
Choose one answer.
a. reduce theta activity
b. no effect on EEG
c. increase beta activity
d. all the above
e. reduce delta activity
Antipsychotics increase beta ,theta and delta rhythms
Revision notes in Psychiatry,P 144-5
Incorrect
Marks for this submission: 0/1.
Question 6
Marks: 0/1
Which ONE of the following electroencephalography (EEG) frequencies is NOT correct?
Choose one answer.
a. Delta 1-4hz
b. Beta: 12-30 Hz
c. Gamma: more than 30 Hz
d. Theta less than 1hz
e. Alpha: 8-12 Hz
Theta range is 4-8 Hz
Organic Psychiatry,Lishman,3rd Edition P 127
Incorrect
Marks for this submission: 0/1.

Miscellaneous
1
Marks: 0/1
Diagnostic features of neurasthenia include the following EXCEPT
Choose one answer.
a. Duration of the disorder is at least 6 months
b. Muscular aches and pains
c. Poor sleep
d. Easy fatigability after minimal effort
e. Tension headaches
The duration of disorder must be at least 3 months
ICD-10
Incorrect
Marks for this submission: 0/1.
Question 2
Marks: 0/1
Which ONE of the following would LOWER your suspicions of brain tumor in a patient with
behavioural symptoms?
Choose one answer.
a. Seizures
b. Focal neurological signs
c. Nausea and vomiting
d. Nonfocal neurological signs
e. Headaches
Nonfocal neurological signs are not indicative of a brain tumor.
""
Incorrect
Marks for this submission: 0/1.
Question 3
Marks: 0/1
Trance-like states are more common with which ONE of the following?
Choose one answer.
a. Frontal lobe neoplasms
b. pituitary neoplasm
c. Occipital lobe neoplasms
d. Temporal lobe neoplasms
e. Pareital lobe neoplasms
Temporal lobe neoplasms are most commonly associated with trance like states and complex behaviour
disturbacnces
NMS Psychiatry,3 Edn,P-133,140
Incorrect
Marks for this submission: 0/1.
Question 4
Marks: 0/1
Which ONE of the following is true about working memory?
Choose one answer.
a. Is a form of declarative memory
b. Is similar to what in the past was called long-term memory
c. Is viewed as a single memory buffer
d. Is a form of nondeclarative memory
e. Has the central executive as its most important component
The central executive is presumably the most important component of working memory, although its
role is the least well defined. One of its functions is to coordinate information from the separate
subsystems.
Text book of Neuropsychiatry,2 edition, Schiffer,Rao,Fogel,p-33-34
Incorrect
Marks for this submission: 0/1.
Question 5
Marks: 0/1
The dementia syndrome, characterized by fluctuating symptom severity, visual hallucinations,
attentional impairment, altered state of consciousness, and delusions, particularly difficult to
distinguish from delirium is
Choose one answer.
a. Lewy body dementia
b. Alzheimer's dementia
c. Dementia due to hypothyroidism
d. Vascular dementia
e. Dementia due to HIV disease
Lewy body dementia mimics delirium in its characteristic fluctuations in symptoms over time.
Organic Psychiatry,Lishman,3 rd Edition P 450-1
Incorrect
Marks for this submission: 0/1.
Question 6
Marks: 0/1
Which ONE of the following is NOT a feature of Frontal Lobe Syndrome?
Choose one answer.
a. empty euphoria
b. disinhibition
c. hemispheric neglect
d. personality change
e. impaired judgement
Hemispheric neglect is seen in parietal lobe lesions
Organic Psychiatry,Lishman,3 rd Edition P 76-77
Incorrect
Marks for this submission: 0/1.
Question 7
Marks: 0/1
Abulia may be defined as
Choose one answer.
a. The patient's continuing into present activity the elements of previous actions
b. The patient's automatic repetition of his or her own word or phrase
c. The patient's incapacity to sustain activities that he or she was quite capable of beginning
d. The patient's lack of will, which in less severe form may include slowness, delayed response,
laconic speech, and reduced initiative and effort
e. The patient's incapacity to carry out a sequential or ordered set of actions toward a unitary goal
in the presence of the necessary objects
Abulia or lack of will is a term used to describe loss of spontaneity due to cerebral disease.
Organic Psychiatry,Lishman,3 rd Edition P 214
Incorrect
Marks for this submission: 0/1.
Question 8
Marks: 0/1
One of the following is not a feature of chronic fatigue syndrome
Choose one answer.
a. Muscle aches and pains
b. Duration more than 6 months
c. Feeling tired after rest, relaxation and enjoyment
d. Disturbed sleep pattern
e. Exercise makes person more tired
CFS is characterised by fatigue, post-exertional malaise, sleep dysfunction, pain, neurological/cognitive
manifestations, autonomic manifestations, neuroendocrine manifestations and immune manifestations
Oxford textbook of Psychiatry, 4th edition, 2001
Incorrect
Marks for this submission: 0/1.
Question 9
Marks: 0/1
Which ONE of the following is an autosomal recessively inherited degenerative dementia?
Choose one answer.
a. Huntington's disease
b. Parkinson's disease
c. Frontotemporal dementia
d. Friedreich's ataxia
e. Wilson's disease
Wilson's disease is autosomal recessively inherited.
Organic Psychiatry,Lishman,3 rd Edition P 662
Incorrect
Marks for this submission: 0/1.
Question 10
Marks: 0/1
Palilalia is
Choose one answer.
a. The patient's repetition of speech of another person
b. The patient's automatic repetition of his or her own word or phrase
c. Loss of the production or recognition of affective elements of speech
d. A disorder of fluency in which discourse, rather than purely articulation, is disturbed by a range
of deficits in speech pragmatics, motor control, and attention
e. The patient's speech marked by impulsive utterances of stereotyped or simple responses with no
aphasic or echolalic features
Palilalia is the automatic repetition of the patient's own words or phrases.
Text book of Neuropsychiatry,2 edition, Schiffer,Rao,Fogel,p-949
Incorrect
Marks for this submission: 0/1.
Question 11
Marks: 0/1
The Frontal Assessment Battery primarily assesses
Choose one answer.
a. Executive cognitive dysfunction
b. Dementia
c. Focal cognitive syndromes
d. Memory
e. HIV-related dementia
Dubois et al. (2000) devised the Frontal Assessment Battery to identify executive dysfunction at the
bedside.
Dubois B, Slachevsky A, Litvan I, et al: The FAB: a Frontal Assessment Battery at bedside. Neurology
55:1621-1626, 2000
Incorrect
Marks for this submission: 0/1.
Question 12
Marks: 0/1
Which ONE of the following is true regarding dissociative amnesia?
Choose one answer.
a. It usually interferes with social or occupational functioning
b. It is more common in the elderly than in adolescents or young adults.
c. It usually is transient
d. It is more common in males
e. It is frequently diagnosed, especially among patients with "hysterical" disorders.
Dissociative amnesia is usually transient; most cases show rapid recovery of memory.
Companion to Psychiatric Studies, 7th edition, eds Johnstone, Cunningham Owens, Lawrie, Sharpe,
Freeman, p481.
Incorrect
Marks for this submission: 0/1.
Question 13
Marks: 0/1
Tumour of which of the following areas causes Bitemporal hemianopsia, optic atrophy, and diabetes
insipidus?
Choose one answer.
a. Pituitary
b. Cerebellum
c. Basal ganglia
d. Occipital lobes
e. Pineal
Clinical signs and symptoms associated with pituitary tumors include bitemporal hemianopsia, optic
atrophy, hypopituitarism, and diabetes insipidus.
Organic Psychiatry,Lishman,3 rd Edition P 229-230
Incorrect
Marks for this submission: 0/1.
Question 14
Marks: 0/1
You have a female patient with hepatic encephalopathy now presenting with increased sexual drive and
carbohydrate craving. She has:
Choose one answer.
a. Kluver-Bucy
b. Hypothyroidism
c. Prader Willi syndrome
d. Diabetes mellitus
e. Cushing's
A condition where damage to the temporal brain lobes from any of a variety of causes (accident,
hypoglycemia, Alzheimer's and others) results in symptoms such as memory loss and abnormal
behaviour characterised by hyperorality and disinhibition
Click here for reference
Incorrect
Marks for this submission: 0/1.
Question 15
Marks: 0/1
Which ONE of the following is NOT a test of executive function?
Choose one answer.
a. None of the above
b. Trail making test
c. Winconsin card sorting test
d. stroop task
e. dual task
All these test executive functions
Text book of Neuropsychiatry,2 edition, Schiffer,Rao,Fogel,p-334
Incorrect
Marks for this submission: 0/1.
Question 16
Marks: 0/1
Huntington's disease shows which pattern of inheritance
Choose one answer.
a. X-linked recessive
b. X-linked dominant
c. Autosomal dominant with poor penetrance
d. Autosomal recessive with low penetrance
e. Autosomal dominant with high penetrance
Huntington' s disease is Autosomal dominant with high penetrance
Oxford textbook of Psychiatry, 4th edition, 2001
Incorrect
Marks for this submission: 0/1.
Question 17
Marks: 0/1
A 40 year old farmer comes to see you with general malaise, low mood, and a circumscribed 4cm red
lesion on his chest. You diagnose:
Choose one answer.
a. Chronic fatigue syndrome
b. Lyme disease
c. Viral Encephalitis
d. Huntingdon's disease
e. AIDS
The acute phase of Lyme disease infection is a characteristic reddish "bulls-eye" rash, with
accompanying fever, malaise, and musculoskeletal pain (arthralgia or myalgia).[1] The characteristic
reddish "bull's-eye" rash (known as erythema migrans) may be seen in about 80% of early stage Lyme
www.medicinenet.com/lyme_disease
Incorrect
Marks for this submission: 0/1.
Question 18
Marks: 0/1
Which ONE of the following classes of medication do NOT cause memory impairment?
Choose one answer.
a. Benzodiazepines
b. Heterocyclic antidepressants
c. Antiparkinsonian agents
d. Atypical antipsychotics
e. Neuroleptics
it appears that atypical antipsychotic agents have an ameliorative effect on cognitive deficits in
schizophrenia, with risperidone exerting stronger beneficial effects on working (short-term) memory
and olanzapine exerting stronger action on verbal learning and memory.
Organic Psychiatry,Lishman,3 rd Edition P 54-5
Incorrect
Marks for this submission: 0/1.
Question 19
Marks: 0/1
Cerebellar lesions can cause
Choose one answer.
a. hemiparesis
b. intention tremor
c. hypertonia
d. peripheral neuropathy
e. witzelsucht
Cerebellar lesions cause physical signs and impairment in higher order functioning - for example,
intention tremor, impaired alternating movements (dysdiadokinesis), personality change, ataxia.
Organic Psychiatry,Lishman,3 rd Edition P 230-1
Incorrect
Marks for this submission: 0/1.
Question 20
Marks: 0/1
In testing for which primitive reflex does the examiner bring the reflex hammer toward the patient's
mouth, then to the side?
Choose one answer.
a. Rooting
b. Suck.
c. Snout
d. Nuchocephalic
e. Avoidance
Rooting is assessed by bringing an object, such as a reflex hammer, toward a patient's mouth and then
to the side.
Macleod's clinical examination,11 edition,p-360
Incorrect
Marks for this submission: 0/1.
Question 21
Marks: 0/1
Lesions in the parietal lobe can cause
Choose one answer.
a. memory impairment
b. personality change
c. ataxia
d. aphasia
e. constructional apraxia
Apraxia occurs in parietal lobe lesions
Organic Psychiatry,Lishman,3 rd Edition P 17-18
Incorrect
Marks for this submission: 0/1.
Question 22
Marks: 0/1
Which ONE of the following is NOT a component of Gerstmann's syndrome?
Choose one answer.
a. Hemi-neglect
b. Finger agnosia
c. nondominant parietal lobe tumor
d. Acalculia
e. Dysgraphia
Hemi-neglect is not a feature of Gerstmann's syndrome.
Organic Psychiatry,Lishman,3 rd Edition P 65
Incorrect
Marks for this submission: 0/1.
Question 23
Marks: 0/1
What treatment has the most evidence for depression following a myocardial infarction:
Choose one answer.
a. Paroxetine
b. Venlafaxine
c. Sertraline
d. Fluoxetine
e. Citalopram
The Sertraline Anti-Depressant Heart Attack Trial (SADHAT) study was an investigator-initiated,
open-label, clinical trial conducted in US. This study concluded that Sertraline was an effective
treatment for post MI depression
Am Heart J 137(6):1100-1106, 1999. 1999
Incorrect
Marks for this submission: 0/1.

EMIs – Neuropsychiatry
1
Marks: 0/3
Investigations

A. 24 hour urinary free cortisol.


B. Dexamethasone suppression test
C. Elevated TSH.
D. Elevated serum cholesterol.
E. Hypernatraemia
F. Hypokalaemia
G. Hyponatraemia
H. Low TSH
I. Ultrasound.
J. No rise in plasma cortisol on short synacthen test

Choose TWO of these would you expect to find for each of these clinical pictures:

A 21-year-old woman with low mood, tiredness, intolerance of cold and menorrhagia.

A 30-year-old woman pigmentation of her skin, low mood, general weakness, weight loss and syncope.

A 30 year old woman with increased body weight, thin arms and legs, excess hair growth and
emotional lability.
1. C, D. The scenario describes hypothyroidism so you would therefore expect a raised TSH and raised
cholesterol. (TSH levels are normal in pituitary failure and raised in primary thyroid failure).
2. G, J. The scenario describes Addison‘s disease. Hyponataemia and lack of cortisol response on the
synacthen test are characteristic findings.
3. A, B. The scenario describes Cushing‘s syndrome, which is due to increased exposure to steroid
hormones. 24 hour urinary free cotisol levels and DST are tests for Cushing‘s disease.
Incorrect
Marks for this submission: 0/3.
Question 2
Marks: 0/3
GENETIC DISORDERS

A. Apolipoprotein (APP)
B. Apolipoprotein E (APO E)
C. ATP 7B gene
D. EPM 2A gene
E. FMRi gene
F. Frataxin gene
G. Huntington gene
H. Hypocretin gene.
I. No gene involved.
J. Presenilin I

Choose ONE gene which is linked to the following conditions.

1. Early-onset alzheimer's disease


2. Huntingtons disease
3. Wilsons disease
Early-onset Alzheimer's disease
Huntington's disease
Wilson's disease
1. J. Presenilin-I is linked to presenile dementia. The APP gene is linked with early onset AD while
ApoE is linked with risk of developing normal age of onset AD.

2. G. Huntington‘s disease is caused due to Huntington gene, leading to trinucleotide repeats of CAG

3. C. The disease gene responsible for Wilson disease, known as the ATP7B gene, is located on the
long arm (q) of chromosome 13 (13q14.3). The protein regulated by this gene plays a role in the
transport of copper (copper-transporting ATPase)
Incorrect
Marks for this submission: 0/3.
Question 3
Marks: 0/3
Lobe Functions

A. Ataxia
B. Broca‘s Aphasia
C. Cortical Blindness
D. Dyscalculia
E. Finger Agnosia
F. Perseveration
G. Pure word Deafness
H. Stereognosis
I. Verbal memory Deficits
J. Visual Hallucinations

For each of the following patients, select TWO most appropriate neurological signs.

1. A 70 year old man had a CVA which resulted in a large infarct in the left frontal area

2. A 56 year old lady who has a left parietal infarct

3. A 60 year lady with a large occipital lesion.


1. B, F – A Broca‘s aphasia and perseveration can be found in frontal lobe lesions.
2. D, E – A left parietal lesion can cause finger agnosia and dyscalculia.
3. C, J – Cortical blindness and visual hallucinations can be seen in lesions of the occipital lobe.
Incorrect
Marks for this submission: 0/3.
Question 4
Marks: 0/3
NEUROANATOMY

A. Arcuate fasciculus
B. Heschl‘s Gyrus
C. Superior colliculus
D. Lentiform nucleus
E. Mamillary bodies
F. Medial geniculate nucleus
G. Olivary nucleus
H. Red nucleus
I. Subiculum
J. Uncinate Fasciculus

For each of the following choose one most related neuroanatomical structure
Korsokoff syndrome
Primary Auditory cortex
Impaired repetition of spoken word
1. E. Mamillary bodies are affected commonly in Korsakoff syndrome.
2. B. Heschl's Gyrus, which is also known as the anterior transverse temporal gyrus, is the primary
auditory area. It is a small gyrus running transversely across the superior temporal gyrus (temporal
operculum) on the upper surface of each temporal lobe immediately in front of the planum temporale.
3. A. Arcuate fasciculus connects broca‘s area (Motor area) and Wernick‘es area (Sensory area).
Incorrect
Marks for this submission: 0/3.
Question 5
Marks: 0/3
Functions of Neurons

A. Astrocytes
B. Betz cells
C. Chandelier cells.
D. Double Bouquet cells.
E. Martinotti cells.
F. Microglia
G. Oligodendrocytes
H. Purkinje cells
I. Pyramidal Neuron
J. Schwann cells

Choose TWO from the above that:

1. Are involved in repair of myelin sheath of axons

2. Mainly use glutamate as their neurotransmitter.

3. Involved in removing dead tissue from nervous system


1. G, J. Oligodendrocytes and schwann cells are involved in myelin forming around axons.
2. B, I. Pyramidal cells and betz cells are glutamatergic. Excessive stimulation may cause excitotoxic
damage.
3. A, F. Microglia and Astrocytes are the scavenger cells
Incorrect
Marks for this submission: 0/3.
Question 6
Marks: 0/3
EEG ABNORMALITiES

A. Diffuse slowing of EEG


B. Fast beta activity
C. Flattened trace
D. Increased delta waves
E. Normal EEG
F. Periodic K Complexes
G. Posteriorly reduced alpha.
H. Reduced & slow Alpha waves
I. Spike and wave activity
J. Triphasic sharp waves
For each of the following scenarios choose the EEG abnormality that is most likely.

1. A 75 year old gentleman whose wife had died three months ago, presented with low mood, weight
loss, decreased sleep, and belief that he had cancer of his bowel despite being investigated and found
normal for the same.

2. A 68 year old man has been losing his way home frequently. His wife gives a nine months history of
gradual memory loss, impaired activities of daily living and social functioning.

3. A 68 year old man has been arrested for shop lifting frequently. He has a history of episodic
shoplifting and has been found to be confused and aggressive while being arrested. On examination
there is evidence for mild cognitive impairment.
1. E, H. Depression: can have normal EEG or age related changes.
2. H, D. Alzheimer‘s dementia: Reduced alpha and in later stages replaced by Theta and delta waves.
3. E, I. Complex partial epilepsy: Spike wave activity.
Incorrect
Marks for this submission: 0/3.
Question 7
Marks: 0/3
NEUROLOGICAL SIGNS

A. Anosagnosia
B. Brocas Aphasia
C. Buccofacial Apraxia
D. Impaired episodic memory
E. Initiation impairment
F. Prosopagnosia
G. Semantic Paraphasias
H. Wernicke‘s Aphasia

For each of the following abnormality, select ONE most appropriate sign:
Patient can describe and match a face but cannot recognise the person
They cannot drink through a straw
Their speech is telegraphic
1. F – This is prosopanosia – the inability to recognise faces.
2. C – This is buccofacial apraxia.
3. B – In Broca‘s aphasia, speech is characteristically telegraphic
Incorrect
Marks for this submission: 0/3.
Question 8
Marks: 0/3
Sleep disorders

A. Adjustment sleep disorder


B. Circadian rhythm disturbance
C. Idiopathic hypersomnia
D. Insufficient sleep syndrome.
E. Narcolepsy with cataplexy
F. Obstructive sleep apnoea
G. Periodic limb movement disorder
H. Recurrent hypersomnia
I. REM Sleep behaviour disorder
Choose ONE abnormality associated with the following scenarios:
A 25 year old air hostess has problems with her job due to daytime sleepiness. She has difficulty with
sleeping at night, but reports no other symptoms
A 40 year old man has trouble with excessive day time sleepiness. His wife has complained that she
has to sleep in another room due to his loud snoring at night. She has also heard him gasp for breath
on a few occasions
A 32 year old woman complains of excessive daytime sleepiness, feeling of unlocking of her knees,
which is brought about by laughter
1. B. Long-haul flights disrupt circadian rhythm.
2. F. The noisy breathing indicates obstructive sleep apnoea.
3. E. This describes narcolepsy with cataplexy (sudden loss of muscle tone secondary to emotional
excitement).
Incorrect
Marks for this submission: 0/3.
Question 9
Marks: 0/3
NEUROLOGICAL SIGNS

A. Alexia without Agraphia


B. Ataxia
C. Broca‘s aphasia
D. Finger agnosia
E. Hemiagnosia
F. Perseveration
G. Prosopagnosia
H. Pure Agraphia
I. Visual hallucination
J. Wernicke‘s aphasia

Identify one symptom / sign associated with:


Weakness of the dominant hand.
A lesion of the non-dominant parietal lobe
A lesion of the left posterior parietal lobe
1. C – A left frontal lesion, encroaching on the motor strip, would produce a Broca‘s aphasia and
contralateral weakness in the hand.
2. G – A non-dominant parietal lobe lesion would produce prosopagnosia (inability to recognise faces).
3. D – This is congruent with a dominant parietal lobe lesion.
Incorrect
Marks for this submission: 0/3.
Question 10
Marks: 0/3
NEUROLOGICAL SIGNS

A. Ataxia
B. Extensor plantar reflex
C. Flapping tremor
D. Intention tremor
E. Neck stiffness
F. Nystagmus
G. Optic atrophy
H. Perseveration
I. Petechial rash
J. Rigidity

For each of the following patients, select appropriate neurological signs.

A 35-year-old man with alcohol dependence syndrome develops Wernicke‘s encephalopathy.

An 18-year-old man develops fever and bacterial septicaemia.

A 60-year-old man with cirrhosis of liver secondary to alcohol dependence develops hepatic
encephalopathy.
1. A, F – Ataxia, nystagmus and ophthalmoplegia are the core features of Wernicke‘s encephalopathy.
2. I – A petechial rash is commonly observed in bacterial septicaemia.
3. C – A flapping tremor is characteristic of hepatic encephalopathy
Incorrect
Marks for this submission: 0/3.
Question 11
Marks: 0/3
Neurochemistry / neuroreceptors

A. Effect on meso-limbic D2 receptor


B. P450 enzyme inhibition
C. Effect on D2 tuberoinfundibular receptor
D. P450 enzyme induction
E. Effect on 5HT3 receptor
F. Effect on alpha 1 adrenoreceptor
G. Effect on nigro-striatal D2 receptor
H. Effect on NMDA receptor
I. Effect on 5HT2 receptor

Select ONE of the above that is the mechanism by which each of these scenarios occur:
A woman taking St John‘s Wort who becomes pregnant in spite of taking the oral contraceptive pill.
A man who is on a typical antipsychotic and develops EPSE
A woman taking fluoxetine who is commenced on amitriptyline. She becomes confused, agitated and
exhibits myoclonus.
1. D. St John‘s Wort is an enzyme inducer and thereby reduces plasma levels of drugs that are
metabolised by the P450 system.
2. G. Blockade of D2 receptors in the caudate-putamen in the nigro-striatal pathway, produces the
motor disturbances in EPSE.
3. I. This scenario describes serotonin syndrome. 5HT2 receptors may be involved in this. Serotonin
syndrome is a toxic state caused mainly by excess serotonin within the central nervous system. It
results in a variety of mental, autonomic and neuromuscular changes, which can range in severity from
mild to life-threatening. Most cases are self-limiting. Severe serotonin syndrome is nearly always
caused by a drug interaction involving two or more 'serotonergic' drugs, at least one of which is usually
a selective serotonin reuptake inhibitor or monoamine oxidase inhibitor.
Incorrect
Marks for this submission: 0/3.
Question 12
Marks: 0/3
NEUROLOGICAL SIGNS

A. Anosognosia
B. Dysdiadochokinesis
C. Hemisomatagnosia
D. Resting tremor
E. Right-left disorientation
F. Rigidity
G. Shuffling gait
H. Unilateral anosmia
I. Upper homonymous hemianopia
J. Visual hallucinations

Choose the symptoms listed above which are associated with each of the scenarios:

A right handed man with left-sided parietal lobe lesion

A woman started on haloperidol four weeks ago, is finding it difficult to move quickly, and has trouble
turning around corners

A woman with posterior cerebral artery occlusion

1. E – This is part of Gerstmann‘s syndrome which is finger agnosia, right-left disorientation,


dyscalculia, agraphia.

2. D, F, G – This appears to be the Parkinsonism due to antipsychotics, and three common symptoms
are resting tremors, rigidity and hypokinesia (here resulting in shuffling gait).

3. J – Posterior cerebral artery occlusion causes a contralateral hemianopia, visual hallucinations, visual
perseveration, visual agnosias and spatial disorientation

Incorrect
Marks for this submission: 0/3.
Question 13
Marks: 0/3
NEUROLOGICAL SIGNS

A. Anomia
B. Anterograde amnesia
C. Buccofacial Apraxia
D. Initiation impairment
E. Learning Difficulty
F. Prosopagnosia
G. Retrograde amnesia
H. Remote paraphasia
I. Semantic Paraphasias
J. Wernicke‘s aphasia

For each of the following abnormality, select ONE most appropriate sign:
They call an apple an orange
They have difficulty in learning how to use their new microwave
They call a pencil as a writing brush
1. I – Verbal paraphasia are those in which another word is substituted for the target word. If
substitution occurs within related category (Fruits), is is termed as semantic paraphasias, if it remotely
connected, it is termed as remote paraphasias.
2. B – In anterograde amnesia there is impairment of the ability to learn new material.
3. A – Here the patient gives his literal interpretation of the word, which suggest that he has anomia or
naming difficulty rather than paraphasias.
Incorrect
Marks for this submission: 0/3.
Question 14
Marks: 0/3
NEUROANATOMY

A. Abducent nerve palsy


B. Trochlear nerve palsy
C. Ipsilateral optic atrophy
D. Gertsmann‘s Syndrome
E. Anosognosia
F. Gait Ataxia
G. Constructional Apraxia
H. Loss of smell
I. Agraphia

Choose TWO symptoms that would be seen in the following:

Frontal meningioma

A right-handed person with right parietal lobe lesion.

An unkempt, dishevelled alcoholic


1. C, H. Ipsilateral optic nerve atrophy and anosmia are frontal lobe symptoms.
2. E, G. This is a non-dominant parietal lobe lesion. Anosognosia is a persistent denial of a neurological
deficit such as a hemiparesis. Visuospatial difficulties, neglect and constructional/dressing apraxias can
also occur.
3. A, F. The scenario is alluding to Wernicke-Korsakov‘s syndrome which results from thiamine
deficiency (often associated with chronic alcoholism). There is an acute neurological syndrome
comprising impaired consciousness and disorientation, truncal ataxia, papillary abnormalities,
ophthalmoplegia, nystagmus and a peripheral neuropathy. This is then followed by an amnesic
syndrome.
Incorrect
Marks for this submission: 0/3.
Question 15
Marks: 0/3
NEUROIMAGING

A. 11C carfentanil
B. 11C raclopride
C. 123I iomazenil
D. 18 F deoxyglucose
E. 99mTc HMPAO.
F. CT scan.
G. EEG
H. Functional MRI
I. T2 weighted MRI.
J. Volumetric MRI.

For each of the following research scenarios choose one most relevant investigation.

1. Measurement of neuronal loss in anterior Cingulate region.


2. Dopamine (D2) receptor binding by antipsychotics.
3. Measurement of cerebral glucose metabolism.
Measurement of neuronal loss in anterior Cingulate region.
Dopamine (D2) receptor binding by antipsychotics.
Measurement of cerebral glucose metabolism.
1. J. Volumetric MRI will be useful to determine neuronal loss.
2. B 11C raclopride is used to detect D2 receptors.
3. D SPECT & PET are useful for cerebral metabolism, cerebral blood flow and receptor studies. The
Radiotracers most commonly used are:
Radiotracer Application
PET:
C15O2 / H2 15O Blood Flow
18Fdeoxyglucose glucose metabolism
11C raclopride D2 receptor
11C Flumazenil Benzodiazepine receptor
11C Carfentanil Opiate receptor
18F altaserin; 18F septoperone 5-HT2 receptor
SPECT
99mTc HMPAO Blood flow
[123I]iodobenzamide D2 receptors
[123I]iomazenil BZD receptor.
Incorrect
Marks for this submission: 0/3.

Addiction
1
Marks: 0/1
The following is true regarding Acamprosate.
Choose one answer.
a. Is associated with abuse potential
b. Acts in a dose-dependent fashion.
c. Reduces craving for alcohol
d. act as an agonist at NMDA receptors
e. Can be used with controlled drinking
Acamprosate works by reducing the craving for alcohol. It seems to involve the restoration of NMDA
receptor tone in glutaminergic systems
Glutamatergic Neurotransmission in Alcoholism. J Biomed Sci 1998;5:309-320
Incorrect
Marks for this submission: 0/1.
Question 2
Marks: 0/1
Resilience is
Choose one answer.
a. different from protective factors, which increase the chances of a child being more resilient
b. a process that is rarely open to influence
c. a static trait, internal to the individual
d. a defence mechanism created by a child who feels worthless, unwanted and lacks confidence

e. extremely difficult to encourage if it is not there innately


There seem to be factors and processes that can minimise the negative impact of parental drug or
alcohol misuse, or protect against them. Thus, some children are resilient and develop no significant
problems related to their parents' substance misuse. Resilience should be conceptualised as a process,
rather than a static trait and/or something solely internal to the individual.
Velleman & Templeton (2007) Adv Psych Treat 13:79-89
Incorrect
Marks for this submission: 0/1.
Question 3
Marks: 0/1
Which of the following statements is true?
Choose one answer.
a. Cocaine acts primarily on the 5-HT system
b. Hallucinogens increase dopaminergic activity
c. Cannabinoids bind to the GABA receptor
d. MDMA is a 5-HT agonist
e. PCP acts on the glutamate receptor
Cocaine blocks dopamine update at the dopamine re-uptake site. Hallucinogens act at multiple sites,
but the effects are thought to be related to 5-HT antagonism. MDMA is a hallucinogenic amphetamine.
Cannabis has anticholinergic effects, and its action is particularly marked in the hippocampus.
Seeman et al (2005) Molecular Psychiatry 10(9): 877-883
Incorrect
Marks for this submission: 0/1.
Question 4
Marks: 0/1
Which of the following is NOT a complication of alcohol misuse?
Choose one answer.
a. Gastric carcinoma
b. Diabetes mellitus
c. Anaemia
d. Central pontine myelinolysis
e. Renal agenesis
Alcohol affects all systems of body and causes a multitude of physical, psychiatric and social
complications
Revision Notes in Psychiatry - Puri & Hall, 2nd Edition (2004) p336-340
Incorrect
Marks for this submission: 0/1.
Question 5
Marks: 0/1
In relation to the evidence that cannabis use may cause psychotic illness, which ONE of the following
statements is true?
Choose one answer.
a. in several studies, increased frequency of reported cannabis use has been associated with
increased reporting of unusual thoughts and perceptions
b. confounding occurs when people exaggerate both their use of cannabis and their experience of
psychotic symptoms
c. several large prospective studies among young people in the general population have shown
cannabis use to be associated with increased diagnosis of schizophrenia
d. it is likely that the association between cannabis use and psychotic symptoms seen in several
studies has arisen by chance.
e. the fact that there are plausible neurophysiological mechanisms through which cannabis use
might cause psychosis is probably the strongest evidence that it does
Since the late 1960s cannabis use has increased substantially in most high-income countries (Hickman
et al, 2007). The increase may now be levelling off, but this is hardly a reason for complacency.
Cannabis is now well established as the third most widely used psychoactive drug (after alcohol and
tobacco) in Europe, the USA and Australasia (Advisory Council on the Misuse of Drugs, 2006). In the
UK around half of adolescents will use cannabis at least once and about a fifth of them will use it
regularly (monthly or more frequently) in young adulthood.
Macleod (2007) Adv Psych Treat 13: 400-411
Incorrect
Marks for this submission: 0/1.
Question 6
Marks: 0/1
You see a 45 year old man in A&E who admits to drinking 8 pints of lager a day for the past 20 years.
He recognises that this may be causing him physical and psychological problems, but is not sure that he
wants to change his drinking pattern yet. What stage is he at on Prochaska and DiClemente's model of
Stages of Change?
Choose one answer.
a. Contemplation
b. Preparation
c. Precontemplation
d. Action
e. Maintenance
The Prochaska and DiClemente model, developed over the past 20 years, identifies 5 stages:
precontemplation, contemplation, preparation, action, and maintenance. It covers a cycle of attitudes
from denial to secure, solidly established commitment to change. In the precontemplation stage, people
are not thinking seriously about changing and are not interested in any kind of help. In the
contemplation stage people are more aware of the personal consequences of their bad habit and they
spend time thinking about their problem. Although they are able to consider the possibility of changing,
they tend to be ambivalent about it. In the preparation stage, people have made a commitment to make
a change. The action stage is is the stage where people believe they have the ability to change their
behavior and are actively involved in taking steps to change their bad behavior by using a variety of
different techniques. Maintenance involves being able to successfully avoid any temptations to return
to the bad habit. Along the way to permanent cessation or stable reduction of substance misuse, most
people experience relapse.
Prochaska JO, DiClemente CC, Norcross JC. In search of how people change. Am Psychol 1992;
47:1102-4.
Incorrect
Marks for this submission: 0/1.
Question 7
Marks: 0/1
You are called to A&E to see a man withdrawing from amphetamines. Which of the following is he
least likely to exhibit:
Choose one answer.
a. Insomnia
b. Decreased appetite
c. Seizures
d. Agitation
e. Hypersomnia
Amphetamine withdrawal is associated with agitation, depression, insomnia and loss of appetite called
crash.
Oxford textbook of Psychiatry, 4th edition, 2001
Incorrect
Marks for this submission: 0/1.
Question 8
Marks: 0/1
In relation to interventions to prevent drug use among young people, which ONE of the following is
true?
Choose one answer.
a. school-based educational interventions appear very effective in reducing young people's drug
use
b. school-based interventions may occasionally have unexpected adverse effects
c. there is no evidence that motivational interviewing is effective as an intervention to reduce drug
use
d. one of the most effective prevention strategies seems to be the prohibition of drugs such as
cannabis
e. reductions in drug use seen with school-based educational campaigns may be small but tend to
be sustained over several years.
Criminal justice-based interventions appear politically popular and should not be dismissed out of
hand. However, they seem likely to fail in relation to all these requirements. School-based educational
interventions are also popular, and may be more acceptable and humane. They can, however, still have
unexpected adverse effects (Aveyard et al, 1999). They are also expensive and, currently, appear to
have limited long-term positive effects (White & Pitts, 1998; Advisory Council on the Misuse of
Drugs, 2006).
Macleod (2007) Adv Psych Treat 13: 400-411
Incorrect
Marks for this submission: 0/1.
Question 9
Marks: 0/1
A man who drinks two bottles of vodka a day tries to stop by himself. He is brought into hospital with
ataxia, and confusion. The chances that he will develop Korsakoff's Psychosis are:
Choose one answer.
a. 100%
b. 50%
c. 20%
d. 10%
e. 80%
Approximately 80 to 90 percent of alcoholics with Wernicke's encephalopathy also develop Korsakoff's
psychosis, a chronic and debilitating syndrome characterized by persistent learning and memory
problems. Patients with Korsakoff's psychosis are forgetful and quickly frustrated and have difficulty
with walking and coordination
ALCOHOL'S DAMAGING EFFECTS ON THE BRAIN. Alcohol Alert. October 2004
Incorrect
Marks for this submission: 0/1.
Question 10
Marks: 0/1
A 17 year old girl is brought to A&E by ambulance. She appears fascinated by the chair by her bedside,
appears to be seeing visual illusions and is talking about the intense experience of hearing the colours
around her. Which of the following would be the most likely substance ingested?
Choose one answer.
a. Khat
b. Cocaine
c. Amphetamines
d. LSD
e. MDMA
There are two major classes of psychedelic hallucinogens : the indoleamines (e.g. LSD) and the
phenethylamines (e.g. mescaline). LSD is a drug that is taken orally, is rapidly absorbed and effects
begin within 1 hour and last for 12 hours. Effects are varied and centre around the distortion of reality.
Dependence does not develop to LSD and consequently withdrawal reactions do not occur. Several
moods may seem to coexist at the same time. In the longer term, users report 'flashbacks' which is a
recurrence of the effects of the drug without having taken any more. They are commonly precipitated
by dark environments, fatigue, or cannabis use.
Calvino (1998) American Journal of Nephrology 18(6):565-569. Aghajanian and Marek (1999)
Neuropsychopharmacology 21:16S-23S
Incorrect
Marks for this submission: 0/1.
Question 11
Marks: 0/1
The Project Match trial demonstrated that
Choose one answer.
a. Motivational Enhancement Therapy was significantly superior to 12-step approach
b. 12-step approach was significantly superior to Motivational Enhancement Therapy
c. there was no significant difference in outcome following Motivational Enhancement Therapy,
Cognitive Behavioural Therapy and 12-step approach
d. Cognitive Behavioural Therapy was not statistically different to placebo
e. 12-step approach was significantly superior to Cognitive Behavioural Therapy
Project Match is the largest ever study of psychological interventions for alcohol misuse, and allocated
17000 patients randomly to 3 outpatient treatments.
Project MATCH Research Group. (1993). Project MATCH: Rationale and methods for a multisite
clinical trial matching patients to alcoholism treatment. Alcoholism: Clinical and Experimental
Research, 17, 1130-1145.
Incorrect
Marks for this submission: 0/1.
Question 12
Marks: 0/1
Interventions with children who live in difficult circumstances should
Choose one answer.
a. wait until a crisis is reached and damage is apparent
b. focus solely on reducing risk factors
c. be made only by specialist child and adolescent psychiatrists
d. involve the professional making a very long-standing commitment to working with the child
and their family
e. start as early as possible in order to promote factors associated with greater resilience
there is clear evidence that some children are resilient, and that there are identifiable factors that seem
to be associated with both their protection and their resilience. There are two clear ways to promote
resilience : by working to reduce the risks to children (the fewer risk factors, the more likely it is that
children will not be negatively affected), and by working to increase protective processes and factors
(the more of them there are, the more likely it is that children will be resilient).
Velleman & Templeton (2007) Adv Psych Treat 13:79-89
Incorrect
Marks for this submission: 0/1.
Question 13
Marks: 0/1
A 19 year old tells you she takes MDMA. The least likely psychological consequence of this drug is:
Choose one answer.
a. Anxiety
b. Disinhibition
c. Increased friendliness
d. Perceptual disturbances
e. Increased desire to do mental tasks
The effects of MDMA include feelings of mental stimulation, emotional warmth, empathy toward
others, a general sense of well being, and decreased anxiety. In addition, users report enhanced sensory
perception as a hallmark of the MDMA experience
From National Institute on Drug Abuse, About.com
Incorrect
Marks for this submission: 0/1.
Question 14
Marks: 0/1
A 27 year old man presents with a acute-onset paranoid psychosis with first rank symptoms. Which of
the following substances its most likely to produce this picture, and may cause seizures and circulatory
collapse at toxic levels?
Choose one answer.
a. LSD
b. MDMA
c. Amphetamines
d. Khat
e. Cannabis
Amphetamine is a stimulant drug, and has many of its effects in common with cocaine. It is taken to
produce a 'high', but its side effects include agitation, sleeplessness, anorexia, hypertension and cardiac
arrhythmias. At toxic levels there may be seizures and circulatory collapse. Amphetamines produce a
paranoid psychosis clinically indistinguishable from schizophrenia. Like cocaine, sudden withdrawal of
the drug can lead to a 'crash', with symptoms of agitation, depression, insomnia and anorexia.
Curran et al (2004) BJ Psych 185:196-204
Incorrect
Marks for this submission: 0/1.
Question 15
Marks: 0/1
Which of the following biochemical effects of alcohol has been recognised?
Choose one answer.
a. Antagonism of serotonin neurotransmission
b. Craving may be due to dopamine release
c. Inhibition of GABA receptor complex
d. Dopamine is released in the nucleus basalis
e. Potentiation of NMDA receptor channels in glutamate receptor
Craving are thought to be due to striatal dopaminergic activity.
Heinz et al (2005) Am J Psych 162: 1515-1520
Incorrect
Marks for this submission: 0/1.
Question 16
Marks: 0/1
You are called to A&E to see someone who has taken PCP. Which of the following would the patient
be most likely to exhibit?
Choose one answer.
a. Loss of appetite
b. Visual hallucinations
c. Urinary incontinence
d. Bizarre postures
e. Depressed mood
Muscle contractions may cause uncoordinated movements and bizarre postures. When severe, the
muscle contractions can result in bone fracture or in kidney damage or failure as a consequence of
muscle cells breaking down. Very high doses of PCP can cause convulsions, coma, hyperthermia, and
death.
From National Institute on Drug Abuse, About.com
Incorrect
Marks for this submission: 0/1.
Question 17
Marks: 0/1
What kind of cannabis use has a strong genetic component
Choose one answer.
a. Moderate use
b. Light - moderate use
c. No genetic influence
d. Moderate - heavy use
e. Heavy use
Genetic risk factors have a moderate impact on the probability of ever using cannabis and a strong
impact on the liability to heavy use, abuse, and, probably, dependence. By contrast, the family and
social environment substantially influences risk of ever using cannabis but plays little role in the
probability of developing heavy cannabis use or abuse.
Kendler and Prescott. Cannabis Use, Abuse, and Dependence in a Population-Based Sample of Female
Twins, American Journal of Psychiatry, 1998
Incorrect
Marks for this submission: 0/1.
Question 18
Marks: 0/1
Disulfiram works by
Choose one answer.
a. blocking the action of aldehyde dehydrogenase enzyme
b. acts as a GABA agonist
c. inhibiting glutamate transmission
d. preventing the build-up of acetaldehyde
e. inhibiting alcohol dehydrogenase enzyme
Disulfiram acts as an aversive agent, by negative reinforcement, through blocking the aldehyde
dehydrogenase enzyme that normally metabolises acetaldehyde - this leads to accumulation of
acetaldehyde and unpleasant side effects such as flushing, headache, palpitations, nausea and vomiting,
even with small intake of alcohol. Larger alcohol intake can lead to hypotension and collapse.
Disulfiram should only be prescribed to patients who are physically healthy and understand the
possible adverse effects of drinking whilst on the drug.
Revision Notes in Psychiatry - Puri & Hall, 2nd Edition (2004) p348
Incorrect
Marks for this submission: 0/1.
Question 19
Marks: 0/1
Lofexidine, which is used to counter the symptoms of opiate withdrawal, works by which mechanism
of action?
Choose one answer.
a. alpha-2 receptor antagonism
b. 5-HT2 receptor antagonism
c. GABA receptor antagonism
d. beta-2 receptor agonism
e. alpa-2 receptor agonism
Clonidine and lofexidine are both centrally acting alpha-2 agonists. The alpha-2 receptor serves as an
autoreceptor in presynaptic nerve terminals where is regulates the release of noradrenaline. The
postulated mechanism of action is that these drugs stop the firing of noradrenergic neurones in the
locus coeruleus, so that the removal of opiates does not lead to rebound adrenergic firing (which is
believed to be responsible for many of the withdrawal symptoms).
Walsh et al (2003) Addiction 98(4): 427-439
Incorrect
Marks for this submission: 0/1.
Question 20
Marks: 0/1
In relation to cannabis use and public health, which ONE of the following statements is true?
Choose one answer.
a. preventing cannabis use may lead to reductions in population rates of psychosis.
b. if cannabis use does not cause psychosis then there is no public health justification for
preventing cannabis use
c. recent evidence suggests that cannabis use probably causes more harm to public health than
tobacco or alcohol use
d. there is good evidence that the prohibition of cannabis use is an effective strategy to reduce use
among young people
e. cannabis use appears to have dramatically increased since the recent reclassification of
cannabis under the UK Misuse of Drugs Act
Evidence of an association between cannabis use and psychosis has recently emerged. By normal
epidemiological conventions, evidence that this association has a causal basis is currently not strong.
However, cannabis may cause psychosis and this possibility presents a tantalising glimpse of a means
to effectively reduce the population burden of illnesses such as schizophrenia (McGrath & Saha, 2007).
Macleod (2007) Adv Psych Treat 13: 400-411
Incorrect
Marks for this submission: 0/1.
Question 21
Marks: 0/1
Which ONE of the following statements is true regarding parental substance misuse?
Choose one answer.
a. it affects children more if the misuser is the mother
b. it affects children more if the misuse takes place in the family home
c. it generally has an impact on young children, with adolescents escaping most ill-effects
d. it reduces the likelihood of violence in the family home
e. it generally affects only the misuser, and not their children
Many children affected by problem substance use within the family environment will reach the
attention of social services because of concerns regarding child protection (Forrester & Harwin, 2004).
The issues relate to both alcohol and drug misuse, but additional problems can arise when
the parent misuses illicit drugs. These include the illegal nature of drug misuse, the modes of ingestion,
the links to crime, the use of the family home for groups of people to take drugs (drug misuse is more
likely to be a home-based activity), and the even stronger links with poverty, unemployment and social
deprivation.
Velleman & Templeton (2007) Adv Psych Treat 13:79-89
Incorrect
Marks for this submission: 0/1.
Question 22
Marks: 0/1
Acamprosate is thought to reduce craving for alcohol by
Choose one answer.
a. inhibiting cholinergic transmission
b. inhibiting GABA-ergic transmission
c. functional antagonism of the glutamate NMDA receptor
d. facilitating serotonergic transmission
e. facilitating dopaminergic transmission
Acamprosate, in combination with psychosocial therapy, has been shown to be clinically effective in
maintaining abstinence in alcohol dependence. Current research suggests that its mechanism of action
involves functional antagonism of the ionotropic glutamate N-methyl-d-aspartate (NMDA) receptor.
However, direct interactions between acamprosate and the NMDA receptor are weak, and recent
findings suggest that acamprosate may modulate NMDA receptors via regulatory polyamine sites, or
that it may act directly on metabotropic glutamate receptors.
Littleton (2007) Journal of Addiction Medicine 1(3) : 115-125
Incorrect
Marks for this submission: 0/1.
Question 23
Marks: 0/1
Therapeutic skills needed to work with the children of substance misusers and the misusing parent
Choose one answer.
a. involve reassurance and risk reduction while referring on to CAMHS or a specialist substance
misuse team
b. require expertise in techniques such as motivational interviewing and relapse prevention
c. Therapeutic skills needed to work with the children of substance misusers and the misusing
parent
d. fundamentally differ for each group
e. take actions and maintain subsequent changes require further training in child psychotherapy
and/or family therapy
For work with anyone (adult or child) the clinician needs to: be warm, empathic and genuine; make a
therapeutic relationship; help clients to explore their difficulties; enable clients to set achievable goals;
empower clients to take action to reach these goals; stay with clients and help them to stabilise and
maintain changes.
Velleman & Templeton (2007) Adv Psych Treat 13:79-89
Incorrect
Marks for this submission: 0/1.
Question 24
Marks: 0/1
You see a 19 year old man in A&E with decreased consciousness, and is unable to give a history. There
are needle-marks on his arm. Which of the following clinical features would lead you to suspect opiate
overdose?
Choose one answer.
a. Diarrhoea
b. Tachypnoea
c. Stupor
d. Dilated pupils
e. Sweating
An opiate overdose can be a life-threatening emergency. Clinical features include stupor or coma,
pinpoint pupils, pallor, severe respiratory depression and pulmonary oedema.
Strang et al (2003) BMJ 326 : 959-960
Incorrect
Marks for this submission: 0/1.
Question 25
Marks: 0/1
A 22 year old woman, known to misuse alcohol, gives birth to a baby girl. Which of the following
features of the baby would lead you to suspect foetal alcohol syndrome?
Choose one answer.
a. Macrocephaly
b. High-set ears
c. Large birth weight
d. Absent philtrum
e. Large eyes
Features of foetal alcohol syndrome include low IQ, cardiac abnormalities, low-set ears, absent
philtrum, long thin upper lip, small nose, microcephaly, strabismus, poor growth.
Revision Notes in Psychiatry - Puri & Hall, 2nd Edition (2004) p338
Incorrect
Marks for this submission: 0/1.
Question 26
Marks: 0/1
Which ONE of the following is true regarding substance misuse in the UK?
Choose one answer.
a. 50% of all violent crimes are linked to drug misuse
b. in the 20-24 age group, survey results suggest that nearly 20% of all women and nearly 10% of
all men are drug dependent.
c. liver cirrhosis deaths are rising slower that they are in other western European countries
d. more people use alcohol hazardously than use drugs hazardously
e. more people have problems with drugs than with alcohol
Around 50% of all violent crimes and, at peak times, up to 70% of all admissions to accident and
emergency departments are linked to alcohol
misuse. The health effects of alcohol consumption in the UK can be seen in data showing that, between
the periods 1987-1991 and 1997-2001, the number of men dying from cirrhosis more than doubled in
Scotland and rose by over two-thirds in England and Wales. Deaths of women from cirrhosis increased
by almost half. These increases are the steepest in western Europe, and contrast with a decline in most
other countries, particularly those of southern Europe (Leon & McCambridge, 2006). Similarly, the
number of alcohol-related deaths in England and Wales, which rose throughout the 1980s and 1990s,
has continued to rise in more recent years.
Velleman & Templeton (2007) Adv Psych Treat 13:79-89
Incorrect
Marks for this submission: 0/1.
Question 27
Marks: 0/1
700ml of whisky (40%) contains how many units of alcohol?
Choose one answer.
a. 25
b. 28
c. 15
d. 3
e. 21
Units of alcohol can be calculated by volume (in litres) multiplied by ABC (Alcohol by Volume in %)
Revision Notes in Psychiatry - Puri & Hall, 1st Edition (1998) p226
Incorrect
Marks for this submission: 0/1.
Question 28
Marks: 0/1
With regard to cannabis use and psychotic illness in the UK population over the past 30 years, which
ONE of the following statements is true?
Choose one answer.
a. only a small minority of young people today are likely to have used cannabis.
b. rates of psychosis have clearly increased
c. increasing rates of cannabis use should inevitably have led to increasing rates of psychosis, if
cannabis use causes psychosis
d. rates of psychosis have clearly fallen
e. most evidence suggests that cannabis use by young people has increased
Since the late 1960s cannabis use has increased substantially in most high-income countries (Hickman
et al, 2007). The increase may now be levelling off, but this is hardly a reason for complacency.
Cannabis is now well established as the third most widely used psychoactive drug (after alcohol and
tobacco) in Europe, the USA and Australasia (Advisory Council on the Misuse of Drugs, 2006). In the
UK around half of adolescents
will use cannabis at least once and about a fifth of them will use it regularly (monthly or more
frequently) in young adulthood
Macleod (2007) Adv Psych Treat 13: 400-411
Incorrect
Marks for this submission: 0/1.
Question 29
Marks: 0/1
Which of the following is NOT a criteria for Alcohol Dependence?
Choose one answer.
a. compulsion
b. increased tolerance
c. neglect of other activities
d. damage to physical or mental health
e. withdrawal symptoms
The ICD-10 criteria for dependence include a strong desire or sense of compulsion to consume alcohol,
difficulty in controlling the intake or cessation of drinking, tolerance, withdrawal symptoms, use
despite knowledge of harm, preoccupation with alcohol use to exclusion of pleasurable activities.
World Health Organization. International Classification of Mental and Behavioural Disorders.
Diagnostic Criteria for Research (10th edn). Geneva: World Health Organization, 1993.
Incorrect
Marks for this submission: 0/1.
Question 30
Marks: 0/1
You are called to see a known opiate user in A&E. Which of the following symptoms would lead you
to suspect opioid withdrawal?
Choose one answer.
a. Pin-point pupils
b. Somnolence
c. Constipation
d. Abdominal cramps
e. Hallucinations
Symptoms of withdrawal from opiates include, but are not limited to, depression, leg cramps,
abdominal cramps, vomiting, diarrhoea, insomnia, and cravings for the drug itself.
Revision Notes in Psychiatry - Puri & Hall, 2nd Edition (2004) p353
Incorrect
Marks for this submission: 0/1.
Question 31
Marks: 0/1
Which of the following statements is NOT true regarding substance misuse and schizophrenia
Choose one answer.
a. Surveys of psychiatric disorders in the community have reported higher rates of substance
abuse disorders among persons with schizophrenia
b. Alcohol abuse and dependence are more common in the schizophrenic population than in the
general population
c. Rates of alcohol and stimulant use among patients with schizophrenia appear to have remained
stable over the last few decades
d. Cannabis use confers an overall twofold increase in the relative risk for later schizophrenia
e. Patients with schizophrenia are more likely to have used psychotomimetic drugs such as
amphetamines, cocaine, and hallucinogens than other psychiatric patients or normal controls
In case-control studies of cannabis and other psychoactive drug use among schizophrenic patients,
schizophrenic patients are more likely to have used psychotomimetic drugs such as amphetamines,
cocaine, and hallucinogens than other psychiatric patients or normal controls. The prevalence of
substance use in schizophrenic patients varies between studies but it is generally higher than
comparable figures in the general population. Rates of alcohol and stimulant use among schizophrenic
patients also appear to have increased over the past several decades. The controlled clinical studies
disagree about the correlates of substance abuse in schizophrenia. Most have found that young males
are over-represented among cannabis users, as in the general community. In some studies, substance
abusers have been reported to have an earlier onset of psychotic symptoms, a better premorbid
adjustment, more episodes of illness, and more hallucinations. But other well controlled studies have
failed to replicate some or all of these findings. Surveys of psychiatric disorders in the community have
reported higher rates of substance abuse disorders among persons with schizophrenia The ECA study
found an association between schizophrenia and alcohol and drug abuse and dependence. Nearly half of
the patients identified as schizophrenic in the ECA study had a diagnosis of substance abuse or
dependence.
Schneier FR, Siris SG. (1987) Journal of Nervous and Mental Disorders 175:641-652. Smith J, Hucker
S. (1994) British Journal of Psychiatry 165:13-21. Warner R et al. (1994) American Journal of
Orthopsychiatry 74:30-39.
Incorrect
Marks for this submission: 0/1.
Question 32
Marks: 0/1
You see a 56 year old man in A&E who appears dishevelled, malnourished and malodorous. Which of
the following signs would lead you to suspect Wernicke's encephalopathy?
Choose one answer.
a. Visual hallucinations
b. Hypertension
c. Amnesia
d. Ataxia
e. Confabulation
Wernicke-Korsakoff syndrome is a brain disorder involving loss of specific brain functions caused by a
thiamine deficiency . Dr Carl Wernicke, a Polish neurologist, described it in 1881 as a triad of acute
mental confusion, ataxia, and ophthalmoplegia.
Revision Notes in Psychiatry - Puri & Hall, 2nd Edition (2004) p343
Incorrect
Marks for this submission: 0/1.

EMIs
1
Marks: 0/3

Diagnosis – drug use.

A. Acute alcohol intoxication


B. Alcoholic hallucinosis
C. Amphetamine intoxication
D. Delirium tremens
E. Opiate withdrawal
F. Opiate addiction
G. LSD intoxication
H. Cannabis intoxication
I. Cocaine withdrawal
J. Cocaine intoxication

Choose ONE diagnosis that best fits each of the clinical scenarios given below:

A 20-year-old man is brought to casualty in the early hours of the morning after suddenly developing the
belief that his friends are conspiring to kill him. His pupils are dilated and he is sweating, agitated and
tachycardic
A 45-year-old man of no fixed abode is an inpatient on the medical ward. The nursing staff states that he is
confused and disorientated. He has been observed shouting and picking up invisible objects off the floor. At
times he appears terrified and is talking to imaginary people by his bed. He is restless, tremulous and
hypertensive
A young woman has been brought to casualty by the police, having been arrested for shoplifting. While in
custody she became unwell, complaining of severe abdominal cramps and pain in her muscles and joints. In
the casualty department she appears restless; she is sweating profusely and has ‘goose bumps’ on her skin

1. People intoxicated with amphetamines can develop acute psychotic episodes, which are
relatively short lived. Another clue to the diagnosis is the characteristic physical
symptoms.

2. Delirium tremens best fits this clinical picture. The core features are all present, including: clouding
of consciousness, bizarre affect, hallucinations, autonomic disturbance and tremor.
3. The clinical picture is one of opioid withdrawal. The woman will not have had access to drugs while
in custody and has developed withdrawal symptoms. Another clue is that she has been
arrested for shoplifting, which may be to finance her addiction.
Incorrect
Marks for this submission: 0/3.
Question 2
Marks: 0/3

DIAGNOSIS - SUBSTANCE MISUSE

A. Acute alcohol intoxication


B. Alcoholic hallucinosis
C. Delirium Tremens
D. HIV
E. Intracranial bleed
F. Korsakoff's psychosis
G. LSD intoxication
H. Opiate intoxication
I. Opiate withdrawal
J. Wernicke's encephalopathy

A 30-year-old man, dependent on alcohol, comes to the A and E. He has a wound in his head. He is homeless
and becomes agitated while in the department.

He experiences seeing squirrels crawling over the floor


He has 2 abscesses in groin and neck, he has small pupils and is bradycardic
He doesn’t recognise the left side of his body

1. C – The visual hallucinations are most suggestive of delirium tremens.


2. H – These features are suggestive of opiate overdose intoxication.
3. E – This hemi-neglect is most likely to have been caused by an intracranial bleed.

Incorrect
Marks for this submission: 0/3.
Question 3
Marks: 0/3
SUBSTANCE MISUSE – MANAGEMENT.

A. Buprenorphine
B. Benzodiazepines
C. Bupropion
D. Diamorphine
E. Loperamide
F. Lofexidine
G. Methadone
H. Naloxone
I. Naltrexone
J. NSAID

Lead in : For each of the following patients, select the most likely treatment.

A 40-year-old opiate dependent woman who wants to become abstinent but cannot tolerate withdrawal
symptoms.
A man of 35 who wants to stop using IV heroin but does not want methadone
At the clinic a 22-year-old known patient is found to have collapsed in the toilets with pinpoint pupils

Answers

1. F. Lofexidine is a alpha 2 agonist like clonidine used in the management of opiod withdrawal.
2. A – Buprenorphine, is used in substitution treatment of opiod dependence.
H – naloxone is the treatment of opiod overdoses
Incorrect
Marks for this submission: 0/3.
Question 4
Marks: 0/3

DIAGNOSIS – SUBSTANCE MISUSE.

A. Acute alcohol intoxication


B. Alcoholic hallucinosis
C. Amphetamine intoxication
D. Cannabis intoxication
E. Cocaine intoxication
F. Cocaine withdrawal
G. Delirium tremens
H. LSD intoxication
I. Opiate addiction
J. Opiate withdrawal
An 18-year-old man presents with a two month history of panic attacks, anxiety and vivid flashbacks. He has
recently become more withdrawn, and at times shown inappropriate behaviour. On examination he has a
conjunctival injection and a cough
A 45-year-old man of no fixed abode is an inpatient on the medical ward. The nursing staff state he is
confused and disorientated. He has been observed shouting and picking up invisible objects off the floor. At
times he appears terrified and is talking to imaginary people by his bed. He is restless, tremulous and
hypertensive
A 19-year-old man with a history of polysubstance abuse, is sweating profusely, appears to be agitated and
tachycardic. He claims to see the colour red while listening to his CDs.

1. D – Social withdrawal, change of behaviour, respiratory symptoms and red eyes are suggestive of
cannabis use.
2. G – This acute confusion and visual hallucinations in a dishevelled man are suggestive of delirium
tremens.
3. H – The physiological arousal and synaesthetic features suggest LSD use.

Incorrect
Marks for this submission: 0/3.
Question 5
Marks: 0/3

SUBSTANCE MISUSE – MANAGEMENT

A. Buprenorphine
B. Benzodiazepines
C. Bupropion
D. Diamorphine
E. Loperamide
F. Lofexidine
G. Methadone
H. Naloxone
I. Naltrexone
J. NSAID
For each of the following patients, select the most likely treatment.
A 32-year-old opiate dependent woman in the second trimester of her pregnancy, wants to stop using heroin,
but is scared of withdrawal symptoms.
A 45 year old man has recently finished detoxification from opiods. He is very motivated to stop using any
heroin, and seeks treatment to help him stop opiate use, but is not keen on any substitution treatment
A 22 year old man who was recently started on outpatient methadone treatment is found unconscious. He is
tachycardic and has pin point pupils on examination
1. G. Methadone is the only medication currently licensed for use in opioid detox or substitution treatment
in pregnancy.
2. I. Naltrexone which is an opioid antagonist has been used to prevent relapses in people who have
completed detox, and don’t want to be on substitution treatment.
H. Naloxone reverses the effects of heroin
Incorrect
Marks for this submission: 0/3.

Evidence Based Medicine


1
Marks: 0/1
The following are NOT considered to be steps of Evidence based practice
Choose one answer.
a. Formulating a question
b. Application of evidence
c. Searching for evidence
d. Critically appraising the evidence
e. Conducting a RCT, if evidence unavailable
The steps in Evidence based Psychiatry includes: Formulating a clinical question, Searching for
evidence, Critically appraising the validity and applicabiliy of the study, applying this on the patient
and finally auditing your practise.
Incorrect
Marks for this submission: 0/1.
Question 2
Marks: 0/1
Cochrane reviews constitute which grade of evidence?
Choose one answer.
a. III
b. Ib
c. IIa
d. IIb
e. Ia
Systematic reviews & Meta-analysis are considered as Type Ia Evidence. Cochrane reviews are robust
systematic reviews, hence are considered Type Ia evidence.
Incorrect
Marks for this submission: 0/1.
Question 3
Marks: 0/1
A Consultant Psychiatrist wanted to provide Evidence based prescribing for their patients. They saw a
young lady with bipolar depression, and searched for a credible RCT decide on the choice of treatment.
The next step for this consultant in their quest for Evidence based practice would be:
Choose one answer.
a. Providing the article for the lady to decide if she wanted the treatment
b. Audit the use of this treatment in the hospital.
c. Speak to colleagues about the value of the treatment advocated in the article
d. Decide on the validity and applicability of the study
e. Talk to patient user groups about the proposed treatment
The steps in Evidence based Psychiatry includes: Formulating a clinical question, Searching for
evidence, Critically appraising the validity and applicability of the study, applying this on the patient
and finally auditing your practice.
Incorrect
Marks for this submission: 0/1.
Question 4
Marks: 0/1
The database that provides systematic coverage of the psychological literature from the 1800s to the
present is
Choose one answer.
a. Psychlit
b. Embase
c. Psychinfo
d. Medline
e. Pubmed
PsycINFO is an abstract database that provides systematic coverage of the psychological literature from
the 1800s to the present. PsycINFO contains bibliographic citations, abstracts, cited references, and
descriptive information to help you find what you need across a wide variety of scholarly publications
in the behavioral and social sciences. Medline provides information from 1965, Psychlit from 1887 and
EMbase from 1947.
Incorrect
Marks for this submission: 0/1.
Question 5
Marks: 0/1
The best evidence for planning services is usually derived from
Choose one answer.
a. Meta analysis
b. Audits
c. Qualitative studies
d. Systematic Reviews
e. Cross sectional study
The most useful evidence in this planning exercise would have been Cochrane style quantitative
reviews of key parameters e.g.. prevalence, incidence, utilisation data and methods of service delivery
for specific population groups
Chipps et al, Planning an evidence based mental health service - is it possible?, Evid Action Int
Cochrane Colloq 8th 2000 Cape Town S Afr. 2000; 8
Incorrect
Marks for this submission: 0/1.
Question 6
Marks: 0/1
The following is true of practice of evidence based psychiatry
Choose one answer.
a. Cannot be done in the absence of Systematic reviews.
b. Includes assessing the scientific validity and clinical importance of evidence
c. Is only concerned with medical treatment
d. Recommends treatments based on evidence from RCTs
e. Is not impacted by a psychiatrists clinical experience
The steps in Evidence based Psychiatry includes: Formulating a clinical question, Searching for
evidence, Critically appraising the validity and applicabiliy of the study, applying this on the patient
and finally auditing your practise.
Incorrect
Marks for this submission: 0/1.
Question 7
Marks: 0/1
The following is True regarding searching for evidence during the process of EBM
Choose one answer.
a. A literature search is best preceded by a formulation of a four-part clinical question
b. Medline covers most of the published treatment studies in a given field
c. NICE guidelines are based on robust systematic reviews
d. Clinical guidelines are best derived from consensus meetings
e. When searching for evidence on treatment trials, looking for RCTs provide the best search
strategy
A literature search is best preceded by a four part clinical questions (PICO - Patient, Intervention,
Comparison & Outcome). Clinical guidelines are best derived from systematic reviews. Medline covers
about 40-60% of published literature, so it is best to search in multiple databases. NICE guidelines
while aiming to be based on robust evidence is still based on Consensus due to lack of available
evidence in many areas. When searching for best evidence for treatment trails, the best search strategy
would be to search for Systematic reviews.
Incorrect
Marks for this submission: 0/1.

Study Designs
1
Marks: 0/1
The following are true of case-control studies
Choose one answer.
a. Measures the exposure and the outcome prospectively
b. Suitable for rare exposures
c. Suitable for rare diseases
d. Can evaluate multiple outcomes of a single exposure
e. Less susceptible to recall bias
A case control study is an observational analytic comparison of subjects with and without a particular
disorder. Cases with a disease and controls without the disease are compared on the rates of previous
exposure to a measure of interest. These studies can be relatively easy, many exposures can be
investigated simultaneously and rare diseases can be examined. However, they are prone to many types
of bias and confounding, as the information on exposure is obtained retrospectively.
Critical Appraisal for Psychiatry. Lawrie SM, McIntosh AM & Rao S. 2000, P 26-27
Incorrect
Marks for this submission: 0/1.
Question 2
Marks: 0/1
Which of the following statements regarding RCT is True
Choose one answer.
a. Randomisation minimises measurement bias
b. Provides generalisable results
c. Proves effectiveness
d. Blinding creates two otherwise equal groups
e. Randomisation distributes confounders equally between groups
It is the process and concealment of randomisation that gives the RCT its status as the medical
equivalent of a scientific experiment. Randomisation provides two more or less identical groups, in
terms of both known and unknown confounders. Also concealed randomisation prevents the
investigators from filtering particular patients into certain treatment groups. Blinding patients and
doctors reduces the potential biases of a patient's placebo response.
Critical Appraisal for Psychiatry. Lawrie SM, McIntosh AM & Rao S. 2000, P 31-32
Incorrect
Marks for this submission: 0/1.
Question 3
Marks: 0/1
Which of the following statements about RCT is False
Choose one answer.
a. It is the best study design for generalising results to the population at large
b. Considered as the gold standard for evaluating treatment
c. Minimises the effect of confounders
d. The number of patients to be treated to get one additional benefit can be calculated
e. Reduces Bias by comparison of otherwise equal groups
It is the process and concealment of randomisation that gives the RCT its status as the medical
equivalent of a scientific experiment. Randomisation provides two more or less identical groups, in
terms of both known and unknown confounders. Also concealed randomisation prevents the
investigators from filtering particular patients into certain treatment groups. Blinding patients and
doctors reduces the potential biases of a patient's placebo response.
Critical Appraisal for Psychiatry. Lawrie SM, McIntosh AM & Rao S. 2000, P 31-32
Incorrect
Marks for this submission: 0/1.
Question 4
Marks: 0/1
An evaluation of a new screening questionnaire for depression in primary care is conducted. The most
important single feature of this questionnaire that would encourage you to use it is:
Choose one answer.
a. It is brief
b. It has a sensitivity of 80%
c. It is validated in different languages
d. It has a specificity of 60%
e. it has Positive Predictive Value of 20%
A screening instrument is selected on the basis of having a high sensitivity and specificity. In this
example, a sensitivity of 80% suggests that the screening instrument will correctly identify 80% of the
cases correctly which is a measure of utility of the test.
D G Altman and J M Bland. Statistics Notes: Diagnostic tests 1: sensitivity and specificity. BMJ Jun
1994; 308: 1552
Incorrect
Marks for this submission: 0/1.
Question 5
Marks: 0/1
The following is true regarding Experimental Clinical trials
Choose one answer.
a. Tend to overestimate the benefits of experimental treatments in the absence of control groups

b. Provides measure of effectiveness


c. Provides measures of efficiency
d. Randomisation is required to minimise bias
e. Are most reliable if conducted pragmatically
Experimental clinical trials are used to evaluate the effects of a new treatment or a novel way of
providing a service. The simplest clinical trial is open or uncontrolled. These habitually overestimate
the beneficial effects of an experimental treatment by an average of 20%. For example, several open
trials suggested that dialysis was useful in schizophrenia until a randomised controlled trial (RCT)
showed that it was not.
Critical Appraisal for Psychiatry. Lawrie SM, McIntosh AM & Rao S. 2000, P 30-31
Incorrect
Marks for this submission: 0/1.
Question 6
Marks: 0/1
A curve with plot of sensitivity versus 1-specificity on either axis is
Choose one answer.
a. ROC curve
b. Galbraith plot
c. normal curve
d. Lorenz curves
e. Nomogram
The receiver operating characteristic (ROC) curve is a plot of the true positive rate (sensitivity) of a test
versus its false-positive rate (1 - specificity ) for all possible cut points.
Receiver Operating Characteristic Curves and Their Use in Radiology, Nancy A.
Obuchowski,Statistical Concepts Series, Radiology 2003;229:3-8.)
Incorrect
Marks for this submission: 0/1.
Question 7
Marks: 0/1
Which of the following can be used to evaluate the possibility of publication bias in a systematic
review
Choose one answer.
a. Correlation
b. Funnel plot
c. The Q statistic
d. Kappa statistic
e. Confidence interval
Funnel plots were devised to evaluate the possibility of publication bias, but can also evaluate potential
sources of heterogeneity.
Critical Appraisal for Psychiatry. Lawrie SM, McIntosh AM & Rao S. 2000, P 81
Incorrect
Marks for this submission: 0/1.
Question 8
Marks: 0/1
A researcher is interested to understand why some people adapt so well to chronic illness whereas
many people find it difficult to cope. The study design best suited to answer their question would be
Choose one answer.
a. Case control study
b. Qualitative study
c. Cohort Study
d. Cross sectional survey
e. Randomised control study
Qualitative research is concerned with personal meanings, experiences, feelings, values and other types
of opinion. The data are usually gathered with semistructured individual interviews, in focus group
sessions or by simple observation. Such interviews or observations may not generalise to a wider
population, but may generate hypotheses for further testing.
Critical Appraisal for Psychiatry. Lawrie SM, McIntosh AM & Rao S. 2000, P 25
Incorrect
Marks for this submission: 0/1.
Question 9
Marks: 0/1
The cost of forgoing an established treatment whilst opting for a new treatment is termed as
Choose one answer.
a. Cost benefit
b. Cost effectiveness
c. Direct cost
d. Indirect cost
e. Opportunity cost
The opportunity cost of investing in a new intervention is best measured by the health benefits (life
years saved, quality adjusted life years (QALYs) gained) that could have been achieved had the money
been spent on the next best alternative intervention.
Stephen Palmer and James Raftery. Economics notes: Opportunity cost. BMJ 1999; 318: 1551 - 1552
Incorrect
Marks for this submission: 0/1.
Question 10
Marks: 0/1
Case reports and case series are types of which study?
Choose one answer.
a. Analytical observational studies
b. Experimental studies
c. Secondary research studies
d. Descriptive observational studies
e. Qualitative studies
Case reports & Series, Audits, Qualitative studies, sectional studies and Ecological studies are
descriptive observational studies
Critical Appraisal for Psychiatry. Lawrie SM, McIntosh AM & Rao S. 2000, P 19
Incorrect
Marks for this submission: 0/1.
Question 11
Marks: 0/1
Which of the following is most suitable to be studied using a Cohort study design
Choose one answer.
a. Link between birth order and schizophrenia
b. Link between previous contact with mental health services and suicide
c. Link between obstetric complications and schizophrenia
d. Link between Cannabis use and Psychosis
e. Link between first episode psychosis and social deprivation
In a typical analytic cohort study, subjects are classified by whether or not they have been exposed to a
suspected risk factor for a disease, and then followed up for a period of time to determine the
development of disease in each group.
Critical Appraisal for Psychiatry. Lawrie SM, McIntosh AM & Rao S. 2000, P 28
Incorrect
Marks for this submission: 0/1.
Question 12
Marks: 0/1
A hospital manager is interested to know if people who had committed suicide had contact with mental
health services prior to their suicide. The best study design to answer their questions would be
Choose one answer.
a. Randomised control study
b. Cohort Study
c. Case report
d. Ecological study
e. Case-control study
A case control study is an observational analytic comparison of subjects with and without a particular
disorder. Cases with a disease and controls without the disease are compared on the rates of previous
exposure to a measure of interest. These studies can be relatively easy, many exposures can be
investigated simultaneously and rare diseases can be examined. However, they are prone to many types
of bias and confounding, as the information on exposure is obtained retrospectively.
Critical Appraisal for Psychiatry. Lawrie SM, McIntosh AM & Rao S. 2000, P 26-27
Incorrect
Marks for this submission: 0/1.
Question 13
Marks: 0/1
This study design is given most weight in a meta-analysis
Choose one answer.
a. Case control study
b. Case reports
c. RCT
d. Cohort study
e. Cross sectional survey
In meta-analyses, larger studies, which are generally of higher quality and provide more precise
estimates of any effect than do smaller studies, are given greater weighting. Meta-analysis was devised
to deal with the results from RCTs.
Critical Appraisal for Psychiatry. Lawrie SM, McIntosh AM & Rao S. 2000, P 40-41
Incorrect
Marks for this submission: 0/1.
Question 14
Marks: 0/1
The following statements regarding Systematic review are true except
Choose one answer.
a. Facilitates assimilation of vast amounts of information in a short time
b. SRs can interpret heterogeneity
c. Much cheaper to conduct than a RCT
d. Most rigorous form of experimental study
e. Provides the best form of evidence
A systematic review gathers and cites all the evidence in a particular research field and is increasingly
important in all areas of medical research. Systematic reviews are time-consuming and a particular
concern is that missed studies may differ from those found. However, they are cheaper to conduct than
a clinical trial.
Critical Appraisal for Psychiatry. Lawrie SM, McIntosh AM & Rao S. 2000, P 38-39
Incorrect
Marks for this submission: 0/1.
Question 15
Marks: 0/1
Which of the following is a test of heterogeneity in Metaanalysis
Choose one answer.
a. t test
b. Kruskal Wallis test
c. Chi square test
d. ANOVA
e. MANCOVA
Chi squared test (?2), Cochran's Q test are statistical tests to detect Heterogeneity in SR. Galbraith plot
& Forest plot are graphical methods by which heterogeneity can be deduced.
John N S Matthews and Douglas G Altman. Statistics notes: Interaction 3: How to examine
heterogeneity. BMJ Oct 1996; 313: 862
Incorrect
Marks for this submission: 0/1.
Question 16
Marks: 0/1
A researcher wants to evaluate the use Magnetic resonance imaging as a screening test to identify cases
of Alzheimer's dementia. They would confirm the correct diagnosis of Alzheimer's dementia by post
mortem examination of the brain. Which of the following study designs would be the most appropriate
for such a study.
Choose one answer.
a. Delphi study
b. Pre post study
c. Case control study
d. Cross sectional study
e. Cohort study
Cross section studies are the most appropriate study design when trying to answer questions concerning
the value of screening or diagnostic tests and prevalence of a disease (i.e., the number of subjects with a
disease at a given point of time).
Incorrect
Marks for this submission: 0/1.
Question 17
Marks: 0/1
Descriptive observational studies are a class of clinical research studies. The following statements
regarding descriptive observational studies are True, EXCEPT
Choose one answer.
a. Cross-sectional studies require a control group
b. Qualitative studies do not require a control group
c. Case series can provide useful information on rare disorders
d. Case report findings can usually be attributable to chance
e. Correlational studies are prone to the ecological Fallacy
Cross sectional studies are most appropriate when trying to answer questions concerning the value of
screening or diagnostic tests and prevalence of a disease. They require a sample representative of the
population being studied, and do not require a control group.
Critical Appraisal for Psychiatry. Lawrie SM, McIntosh AM & Rao S. 2000, P 19, 26
Incorrect
Marks for this submission: 0/1.
Question 18
Marks: 0/1
A GP moves from rural Scotland to work in a inner city practice in London. They find that more young
people are presenting with psychotic symptoms compared to when they were in a practice in Scotland.
They feel that increased use of drugs in cities may be a reason for this, and would like to devise a study
to test their hypothesis. The study design best suite to answer this research question is answer
Choose one answer.
a. Randomised control study
b. Case control study
c. Cohort Study
d. Qualitative study
e. Cross sectional survey
In a typical analytic cohort study, subjects are classified by whether or not they have been exposed to a
suspected risk factor for a disease, and then followed up for a period of time to determine the
development of disease in each group.
Critical Appraisal for Psychiatry. Lawrie SM, McIntosh AM & Rao S. 2000, P 28
Incorrect
Marks for this submission: 0/1.
Question 19
Marks: 0/1
The most appropriate measure to explain about treatment response of a medication to a patient would
be
Choose one answer.
a. Number needed to harm
b. Odds ratio
c. Controlled event rate
d. Relative risk
e. Number needed to treat
The NNT is a useful measure of treatment response and is the number of patients that need to be treated
for one to benefit compared with a control.
Incorrect
Marks for this submission: 0/1.
Question 20
Marks: 0/1
The following statement regarding descriptive observational studies is true
Choose one answer.
a. Require a control group
b. Suitable to establish causality
c. Useful for hypothesis generation
d. Not prone for bias
e. High interrater reliability
Descriptive observational studies are generally conducted without a control group for comparison, they
are purely descriptive and are only suitable for hypothesis generation.
Critical Appraisal for Psychiatry. Lawrie SM, McIntosh AM & Rao S. 2000, P 19
Incorrect
Marks for this submission: 0/1.
Question 21
Marks: 0/1
The statement for reporting Meta analysis is provided by
Choose one answer.
a. COCHRANE
b. RESORT
c. CONSORT
d. MEDLINE
e. QUORUM
The Quality of Reporting of Meta analysis (QUORM) provides a statement for appropriate reporting of
meta analysis to ensure consistency across the studies reported. This is similar to the CONSORT
statement which has been provided for RCTs
Moher D, Cook DJ, Eastwood S, Olkin I, Rennie D, Stroup DF. Improving the quality of reports of
meta-analyses of randomised controlled trials: the QUORUM statement. Quality of reporting meta-
analysis. Lancet 1999; 354: 1896-1900
Incorrect
Marks for this submission: 0/1.
Question 22
Marks: 0/1
The following are weaknesses of cohort studies except
Choose one answer.
a. Lengthy & expensive
b. Non-participation bias
c. Change in exposure status
d. Loss to follow-up problems
e. Not suitable for studying rare exposures
Cohort studies can establish aetiology (causation) due the prospective nature of follow-up to identify
the development of disease. They can also be used to study multiple possible outcomes from a single
exposure. As they require large samples and are time-consuming, they are best suited to studying
relatively common diseases. Relatively rare disorders can, however, be suitable for cohort studies if
subjects are at enhanced risk for other reasons (eg those with a strong family history of schizophrenia)
or if an outcome is common among those exposed (i.e.. if the attributable risk percent is high).
Critical Appraisal for Psychiatry. Lawrie SM, McIntosh AM & Rao S. 2000, P 28-29
Incorrect
Marks for this submission: 0/1.
Question 23
Marks: 0/1
Grounded theory in qualitative research is strongly associated with which of the following processes
Choose one answer.
a. Deductive
b. Mutative
c. Inductive
d. Transductive
e. Conductive
Grounded theory is a complex iterative process. The research begins with the raising of generative
questions which help to guide the research but are not intended to be either static or confining. As the
researcher begins to gather data, core theoretical concept(s) are identified. Tentative linkages are
developed between the theoretical core concepts and the data. This early phase of the research tends to
be very open and can take months. Later on the researcher is more engaged in verification and
summary. The effort tends to evolve toward one core category that is central.
Glaser, B. (1992). Basics of grounded theory analysis. Mill Valley, CA: Sociology Press
Incorrect
Marks for this submission: 0/1.
Question 24
Marks: 0/1
The following are true of case control studies EXCEPT
Choose one answer.
a. Its prospective design avoids bias
b. quick and inexpensive
c. Suitable for rare outcomes
d. Can evaluate distant and multiple exposures
e. Useful if there is long time lag between exposure and outcome
A case control study is an observational analytic comparison of subjects with and without a particular
disorder. Cases with a disease and controls without the disease are compared on the rates of previous
exposure to a measure of interest. These studies can be relatively easy, many exposures can be
investigated simultaneously and rare diseases can be examined. However, they are prone to many types
of bias and confounding, as the information on exposure is obtained retrospectively.
Critical Appraisal for Psychiatry. Lawrie SM, McIntosh AM & Rao S. 2000, P 26-27
Incorrect
Marks for this submission: 0/1.
Question 25
Marks: 0/1
The following is true regarding Qualitative studies
Choose one answer.
a. Are useful to hypothesis testing
b. Are analytical observational studies
c. Are very reliable
d. Not prone for bias
e. Are useful to generate hypothesis
Qualitative research is concerned with personal meanings, experiences, feelings, values and other types
of opinion. The data are usually gathered with semi structured individual interviews, in focus group
sessions or by simple observation. Such interviews or observations may not generalise to a wider
population, but may generate hypotheses for further testing.
Critical Appraisal for Psychiatry. Lawrie SM, McIntosh AM & Rao S. 2000, P 25
Incorrect
Marks for this submission: 0/1.
Question 26
Marks: 0/1
The following are true statements about case-control studies except
Choose one answer.
a. quick and inexpensive
b. Provide information about causation between exposure and outcome
c. Suitable for rare outcomes
d. Require smaller sample size compared to cohort studies
e. Useful if there is long time lag between exposure and outcome
A case control study is an observational analytic comparison of subjects with and without a particular
disorder. Cases with a disease and controls without the disease are compared on the rates of previous
exposure to a measure of interest. These studies can be relatively easy, many exposures can be
investigated simultaneously and rare diseases can be examined. However, they are prone to many types
of bias and confounding, as the information on exposure is obtained retrospectively.
Critical Appraisal for Psychiatry. Lawrie SM, McIntosh AM & Rao S. 2000, P 26-27
Incorrect
Marks for this submission: 0/1.
Question 27
Marks: 0/1
Funnel plots are used to detect
Choose one answer.
a. Recall bias
b. Publication bias
c. Interviewer bias
d. Sensitivity bias
e. Language bias
Funnel plots are primarily used to detect publication bias.
Incorrect
Marks for this submission: 0/1.
Question 28
Marks: 0/1
The statement for reporting Randomised Controlled Trials is provided by
Choose one answer.
a. QUORUM
b. MEDLINE
c. CONSORT
d. RESORT
e. COCHRANE
CONSORT, which stands for Consolidated Standards of Reporting Trials is an evidence-based,
minimum set of recommendations for reporting RCTs. It offers a standard way for authors to prepare
reports of trial findings, facilitating their complete and transparent reporting, and aiding their critical
appraisal and interpretation.
Click here for reference
Incorrect
Marks for this submission: 0/1.
Question 29
Marks: 0/1
All the following are true about likelihood ratios except
Choose one answer.
a. Derived from sensitivity and specificity
b. They are affected by prevalence
c. Indicate changes in pre-test probability
d. Has no relation to post test probability
e. Allow better understanding of a diagnostic test
LRs are unaffected by prevalence and are constant regardless of disease prevalence.
Incorrect
Marks for this submission: 0/1.

Interpreting Results of Studies


1
Marks: 0/1
The following observation increases the possibility that an observed association is true EXCEPT
Choose one answer.
a. Specificity
b. Temporality
c. Strength of the association
d. Reverse causality
e. Consistency
Reverse causality is a particular problem for descriptive and case-control studies. This means that an
association between an exposure and a disease arises because the disease causes the exposure, rather
than vice-versa.
Critical Appraisal for Psychiatry. Lawrie SM, McIntosh AM & Rao S. 2000, P 43
Incorrect
Marks for this submission: 0/1.
Question 2
Marks: 0/1
The following statement regarding Confidence Interval are true EXCEPT
Choose one answer.
a. considered to be better estimate of significance of results
b. are a measure of precision of studies
c. Provide the same information as p values
d. The 95% CI is calculated as the effect 1.96 times SE
e. Lower the CI, higher the precision of studies
Confidence intervals are a measure of the dispersion of data. It gives the precision of a measure, as any
measure from any sample data set can only be regarded as an estimate of the true situation in the
population as a whole. The confidence interval can be defined as the range within which the true
measure actually lies, with a specific degree of assurance (usually 95%), based upon the estimate from
your data set. They are calculated approximately as the measure plus or minus 1.96 times the standard
error of the measure. The confidence interval gives all the information of the P value, plus the prevision
of any estimate.
Critical Appraisal for Psychiatry. Lawrie SM, McIntosh AM & Rao S. 2000, P 61
Incorrect
Marks for this submission: 0/1.
Question 3
Marks: 0/1
Which of the below increases the power of a study:
Choose one answer.
a. Comparing active treatment vs. active treatment
b. A scale with a lot of subscales
c. Increased heterogeneity
d. Smaller numbers of participants
e. Comparing active treatment vs placebo
The power of the study depends on the sample size, effect size, reliability of the measures used, and the
significance level adopted in the study. Effect size is the difference between the two groups being
compared, which would be maximum between active treatment and no treatment (placebo)
Incorrect
Marks for this submission: 0/1.
Question 4
Marks: 0/1
All the following are possible explanations for association between exposure and outcome EXCEPT
Choose one answer.
a. Chance
b. Reverse Causality
c. Causality
d. Validity
e. Bias
Validity is the extent to which a measure actually measures what it purports to. There are many
subtypes : face validity, concurrent validity, predictive validity.
Critical Appraisal for Psychiatry. Lawrie SM, McIntosh AM & Rao S. 2000, P 72
Incorrect
Marks for this submission: 0/1.
Question 5
Marks: 0/1
Quetiapine has been recently shown to induce significantly less weight gain than Olanzapine on the
Cochrane Database. A GP, wishing to replicate this finding in his small rural practice (following ethics
approval), randomly allocates 12 patients with schizophrenia into two groups. One group receives
Quetiapine for 3 months, the other Olanzapine. A statistical test of the mean weight gain between
groups produces a p-value of 0.09. This likely represents which type of error?
Choose one answer.
a. Bias
b. Confounding
c. Type I
d. Type III
e. Type II
A type error occurs when the null hypothesis (in this example, Null hypothesis would be no difference
between Quetiapine and Olanzapine in the weight gain they induce) is accepted when it should have
been rejected. The problem in this example would be the very small sample size.
Incorrect
Marks for this submission: 0/1.
Question 6
Marks: 0/1
Which of the following is an example of a Type II error,
Choose one answer.
a. A study involving 60 depressed patients with mild depression finds a big treatment effect for
St. Johns wort.
b. A study involving six patients with depression finds no effect of SSRIs on depression.
c. A study involving thirty depressed patients looking at effect of Venlafaxine using a thirty item
depression rating scale finds positive effect of venlafaxine on two items of the scale.
d. A study involving 30 patients with borderline personality disorder finds a strong association
between early childhood abuse and borderline personality disorder.
e. A study investigating association between coffee drinking and lung cancer in a study of 300
coffee drinkers, finds no association between coffee drinking and lung cancer.
A type error occurs when the null hypothesis (in this example, Null hypothesis would be
Antidepressants will have no benefit in depression) is accepted when it should have been rejected. The
problem in this example would be the very small sample size.
Incorrect
Marks for this submission: 0/1.
Question 7
Marks: 0/1
A researcher decides to investigate the link between childhood abuse and borderline personality
disorder. They decide to interview hospitalised borderline patients to look for past history of abuse. The
bias most likely to be introduced by such methodology is
Choose one answer.
a. Sampling bias
b. Recall bias
c. Interview bias
d. Response bias
e. Selection bias
Recall bias is a pernicious subject-introduced observation bias, particularly in case-control studies and
cross-sectional surveys. Subjects or informants are likely to search after meaning and identify possible
exposures.
Critical Appraisal for Psychiatry. Lawrie SM, McIntosh AM & Rao S. 2000, P 47
Incorrect
Marks for this submission: 0/1.
Question 8
Marks: 0/1
A confounder is defined as
Choose one answer.
a. A variable that inadvertently improves the power of the study
b. A variable that is related to the exposure and is an independent risk factor for the outcome of
the study
c. A variable introduced by the researcher to minimise biases in the study
d. A variable that at any stage of inference, tends to produce results or conclusions which differ
from the truth
e. A variable which is present on the causal pathway between the exposure and the outcome
A confounder is a variable that is related to the exposure (independent variable) and is an independent
risk factor for the outcome of the study (dependent variable)
Critical Appraisal for Psychiatry. Lawrie SM, McIntosh AM & Rao S. 2000, P 17
Incorrect
Marks for this submission: 0/1.
Question 9
Marks: 0/1
Which of the following is the least adequate method of randomisation?
Choose one answer.
a. Toss of a fair, unbiased coin.
b. Odd/even last digit of date of birth
c. Minimization
d. Permuted block randomisation
e. Simple randomisation by computer
The last digit of date of birth (0-9) being classified as odd/even would lead to imbalance of numbers
due to some months having 31st days (hence more odd numbers. This would lead to people would odd
numbered last digit of date of birth having a higher chance of being selected compared to people with
even numbered date of birth.
Incorrect
Marks for this submission: 0/1.
Question 10
Marks: 0/1
The following statements regarding Confounders are true except
Choose one answer.
a. Confounders are risk or protective factors for the outcome in question
b. Confounders are associated with both an exposure and a disease
c. Statistical techniques can be used to minimise the effects of confounding
d. Confounders can be controlled for in the design of a study
e. Confounder lies on the causal pathway between exposure and outcome
A confounder is a variable that is related to the exposure (independent variable) and is an independent
risk factor for the outcome of the study (dependent variable). A confounder can be either a risk or a
protective factor for the outcome in questions. Restriction, Matching, Randomisation & Statistical
techniques (Multivariate analysis) can be used to reduce the effect of confounding
Critical Appraisal for Psychiatry. Lawrie SM, McIntosh AM & Rao S. 2000, P 17
Incorrect
Marks for this submission: 0/1.
Question 11
Marks: 0/1
All the following measures can reduce the effect of the confounding factors EXCEPT
Choose one answer.
a. Restriction of the study sample
b. Multivariate analysis
c. Matching
d. Blinding
e. Randomisation
Blinding reduces the potential biases of a patient's placebo response and the doctor's sometimes
overzealous desire to find a good new treatment.
Critical Appraisal for Psychiatry. Lawrie SM, McIntosh AM & Rao S. 2000, P 32
Incorrect
Marks for this submission: 0/1.
Question 12
Marks: 0/1
You decide to investigate the link between completed suicide and social support. You interview friends
and family members of people who have committed suicide. The most likely bias introduced by you in
such a design would be:
Choose one answer.
a. Attrition bias
b. Response bias
c. Sampling bias
d. Recall bias
e. Interview bias
The bias in selecting subjects introduced by the researchers would be termed as sampling bias.
Incorrect
Marks for this submission: 0/1.
Question 13
Marks: 0/1
The p value associated with an effect represents:
Choose one answer.
a. The probability of there being an effect
b. The effect size
c. The lower bound of the 95% confidence interval
d. The probability of making a type two error
e. The probability of making a type one error
The probability of making a type I error is equal to P value and is termed as a. (usually set at 0.05)
Incorrect
Marks for this submission: 0/1.
Question 14
Marks: 0/1
Clozapine has been recently shown induce similar weight gain as Olanzapine at meta-analysis. A
psychiatrist in Papua New Guinea, wishing to test this finding, chooses 30 patients with schizophrenia
to receive Olanzapine for 3 months and 30 to receive Clozapine. A statistical test of the mean weight
gain between groups produces a p-value of 0.02. This likely represents which type of error?
Choose one answer.
a. Systematic error
b. Ecological fallacy
c. Type III
d. Type I
e. Type II
Meta-analysis would provide the best form of evidence. In this example, the true finding that the
researcher would have been expected to find is that there would be no difference in the weight gain
induced by Clozapine and Olanzapine. They have however found that there is a significant difference
between Clozapine and Olanzapine (p value is less than 0.05), and is likely to lead to rejecting a null
hypothesis which should have been accepting - Type I error.
Incorrect
Marks for this submission: 0/1.

Basic Statistics
1
Marks: 0/1
The following are measures of Central tendency
Choose one answer.
a. Mode
b. Range
c. Variance
d. Interquartile range
e. Standard deviation
The mean, mode and median are measures of central tendency.
Critical Appraisal for Psychiatry. Lawrie SM, McIntosh AM & Rao S. 2000, P 60
Incorrect
Marks for this submission: 0/1.
Question 2
Marks: 0/1
Which of the following is a discrete variable
Choose one answer.
a. Blood pressure
b. Years of education
c. Annual income
d. Age in years
e. Gender
A discrete, otherwise known as categorical, variable can only have set values (of whole numbers) and
can be subdivided into : binary, nominal, ordinal, interval or ratio data.
Critical Appraisal for Psychiatry. Lawrie SM, McIntosh AM & Rao S. 2000, P 56
Incorrect
Marks for this submission: 0/1.
Question 3
Marks: 0/1
The following are measures of Dispersion (Spread of the Data)
Choose one answer.
a. Mode
b. Percentile
c. Median
d. Mean
e. Standard deviation
The standard deviation is the spread of all observations around the mean, calculated as the square root
of the variance.
Critical Appraisal for Psychiatry. Lawrie SM, McIntosh AM & Rao S. 2000, P 60
Incorrect
Marks for this submission: 0/1.
Question 4
Marks: 0/1
The mean in the following sample is: 3, 6, 4, 11, 7, 5
Choose one answer.
a. 4
b. 5
c. 6
d. 7
e. 8
The mean is the sum of all the values divided by the number of values.
Critical Appraisal for Psychiatry. Lawrie SM, McIntosh AM & Rao S. 2000, P 60
Incorrect
Marks for this submission: 0/1.
Question 5
Marks: 0/1
The following is a measure of internal validity
Choose one answer.
a. Generalisability
b. relative risk
c. numbers needed to treat
d. reliability
e. Intention to treat
Reliability or accuracy is a measure of internal validity where as external validity is the generalisability
of results. The internal validity depends on the research design, research setting and the measures used,
whereas the external validity depends on the sample selection (ensuring representative sample for
generalisation to be achieved)
Godwin et al. Pragmatic controlled clinical trials in primary care: the struggle between external and
internal validity. BMC medical Research methodology,2003,3:28
Incorrect
Marks for this submission: 0/1.
Question 6
Marks: 0/1
The Mode in the following dataset is: 21, 22, 23, 22, 24, 25, 23, 22, 26
Choose one answer.
a. 24
b. 22
c. 23
d. 25
e. 21
The mode is the most common value
Critical Appraisal for Psychiatry. Lawrie SM, McIntosh AM & Rao S. 2000, P 60
Incorrect
Marks for this submission: 0/1.
Question 7
Marks: 0/1
The following data require nonparametric statistics
Choose one answer.
a. Measures on an interval scale
b. Data with a normal distribution
c. Continuous data
d. Measures on a ratio scale
e. Categorical data
For non-normally distributed variables, non-parametric statistics should be employed.
Critical Appraisal for Psychiatry. Lawrie SM, McIntosh AM & Rao S. 2000, P 69
Incorrect
Marks for this submission: 0/1.
Question 8
Marks: 0/1
The following is true for data recorded using ordinal scale of measurement
Choose one answer.
a. Are described using Median & range
b. Can be Analysed with Parametric statistics
c. Differences between points are equal across the scale
d. Have a true zero
e. Normally distributed
Ordinal data is where there is an order inherent in the measurement scale but this is not simply
quantifiable - for example, social classes I-V where the difference between I and II cannot be said to be
the same as the difference between IV and V.
Critical Appraisal for Psychiatry. Lawrie SM, McIntosh AM & Rao S. 2000, P 56
Incorrect
Marks for this submission: 0/1.
Question 9
Marks: 0/1
The shoe sizes of a group of participants were recorded. A shoe manufacturer can only produce one
size of shoe and has asked you for advice on which size shoe they should produce to sell the most
number of shoes. Which measure of central tendency would you use to help you give this advice?
Choose one answer.
a. Mean
b. Median
c. Mode
d. Range
e. Range
The mode is the most common value and will result in most sales.
Critical Appraisal for Psychiatry. Lawrie SM, McIntosh AM & Rao S. 2000, P 61
Incorrect
Marks for this submission: 0/1.
Question 10
Marks: 0/1
The following is true regarding reliability
Choose one answer.
a. Intra class coefficient can be used to quantify reliability
b. Is dependent on validity
c. Reliability seeks to find agreement between a theoretical concept and a specific measuring
device or procedure
d. Measured using Cronback's alpha
e. is defined as whether an instrument measures what it purports to measure
The reliability is the extent to which that measure is stable between raters (interrater reliability), within
one rater over time (intrarater reliability), between two halves of a scale (split-half scale reliability) or
between all the items in a scale (internal consistency). Reliability can be quantified as a correlation
coefficient or (where more than two raters are evaluated) the intraclass correlation coefficient.
Critical Appraisal for Psychiatry. Lawrie SM, McIntosh AM & Rao S. 2000, P 72
Incorrect
Marks for this submission: 0/1.
Question 11
Marks: 0/1
Cronbachs alpha measures which of the following
Choose one answer.
a. Split half reliability
b. Inter rater reliability
c. Internal consistency
d. Face validity
e. Test retest reliability
Internal consistency is a measure of validity and refers to whether the items on a scale are related to one
another. This is measured using Cronbachs alpha.
J Martin Bland and Douglas G Altman. Statistics notes: Cronbach's alpha. BMJ Feb 1997; 314: 572
Incorrect
Marks for this submission: 0/1.
Question 12
Marks: 0/1
The following increases the power of a study
Choose one answer.
a. Comparing active treatment vs. placebo
b. Use of self rated questionnaires
c. increased heterogeneity
d. smaller number of participants
e. Comparing active treatment vs. active treatment
The power of a study depends on sample size (large sample size - more power); Effect size (difference
between comparators, so more power when comparing active treatment vs. placebos); reliability of
measures used (observer rated measures more reliable than self-rated ones); and level of significance
adopted (p value significant at 95% etc.)
Companion to Psychiatric studies;6th ed.;168-169
Incorrect
Marks for this submission: 0/1.
Question 13
Marks: 0/1
Which of the following is an example of nominal data
Choose one answer.
a. Age in years
b. Marital status
c. Temperature in Celsius
d. Council tax bands
e. Intelligence quotient
Nominal data is where three or more unique categories bear no mathematical relation to each other.
Critical Appraisal for Psychiatry. Lawrie SM, McIntosh AM & Rao S. 2000, P 56
Incorrect
Marks for this submission: 0/1.
Question 14
Marks: 0/1
The variance of a sample is 4, the standard deviation of that sample would be
Choose one answer.
a. 32
b. 2
c. 64
d. 16
e. 8
The standard deviation is the spread of all observations around the mean, calculated as the square root
of the variance.
Critical Appraisal for Psychiatry. Lawrie SM, McIntosh AM & Rao S. 2000, P 60
Incorrect
Marks for this submission: 0/1.
Question 15
Marks: 0/1
The validity that seeks agreement between a theoretical concept and a specific scale is termed as
Choose one answer.
a. Criterion validity
b. Internal consistency
c. Construct validity
d. Face validity
e. Content validity
Construct validity refers to the degree to which inferences can legitimately be made from the
operationalizations in your study to the theoretical constructs on which those operationalizations were
based.
Critical Appraisal for Psychiatry. Lawrie SM, McIntosh AM & Rao S. 2000, P 72
Incorrect
Marks for this submission: 0/1.
Question 16
Marks: 0/1
The following is an example of Interval scale of measurement
Choose one answer.
a. Temperature in Celsius
b. Age in years
c. Council Tax bands
d. Gender
e. Temperature in Fahrenheit
Interval data is where the order is meaningful in that differences between the points are equal across the
scale, but zero is just a point on the scale without representing an absence of the characteristic.
Critical Appraisal for Psychiatry. Lawrie SM, McIntosh AM & Rao S. 2000, P 57
Incorrect
Marks for this submission: 0/1.
Question 17
Marks: 0/1
Select one correct statement from the following
Choose one answer.
a. The standard error is independent of the standard deviation of the sample
b. Mode is a measure of variation in the sample
c. The standard error varies with the size of the population
d. Standard deviation is a measure of precision of an estimate
e. In chronic diseases, the incidence is high relative to prevalence
The standard error reflects how much the mean and standard deviation would be likely to vary in other
samples from the same population. It uses the variablity of the sample measure as an estimate of the
true effect in the general population from which the sample was drawn.
Critical Appraisal for Psychiatry. Lawrie SM, McIntosh AM & Rao S. 2000, P 61
Incorrect
Marks for this submission: 0/1.

Critical Appraisal - Application


1
Marks: 0/1
Please read the precis below for the Questions 1-8 (Abuse in childhood and depression in adulthood
(Widom et al. Arch Gen Psychiatry. 2007;64:49-56))

Widom et al., (2007) wanted to determine if abused and neglected children were at elevated risk of
developing depression. They identified children who were identified to be substantiated cases of
physical and sexual abuse and neglect (before the age of 11 years) from January 1, 1967, to December
31, 1971 (n=676). Subjects were drawn from the records of county juvenile and adult criminal courts in
a metropolitan area. They found a group of nonabused and nonneglected children (n=520) who were
matched with the above group on the basis of age, sex, race and appropriate family class. Both the
groups were followed into young adulthood to determine the aim of their study.

Between October 20, 1989, and December 22, 1995, 2-hour in-person interviews were conducted with
each of the subjects, using the National Institute of Mental Health Diagnostic Interview Schedule,
Version III Revised, to determine DSM-III-R Major Depressive Disorder and other psychiatric
diagnoses. The interviewers were blind to the purpose of the study, to the participants' group
membership, and to the inclusion of an abused and/or neglected group. Similarly, the subjects were
blind to the purpose of the study and were told they had been selected to participate as part of a large
group of individuals who grew up in that area in the late 1960s and early 1970s.

Of the original sample of 1575 subjects (908 abused and/or neglected individuals and 667 controls),
1307 (83.0%) were located and 1196 (75.9%) were interviewed.

One of the results they found is in the table below.

Group Total Major depressive disorder


no. Lifetime Current
No. of RR (95% CI) No. of RR (95% CI)
subjects subjects
Control 520 108 NA 55 NA
Neglect 541 137 1.28(0.96 - 86 1.59(1.10-2.29)
1.70)
Physical 106 32 1.59(1.00 - 15 1.34(0.72-2.46)
abuse 2.52)
Sexual abuse 96 23 1.20(0.72- 11 1.09(0.55 - 2.18)
2.01)
Multiple 70 22 1.75(1.01- 10 1.41(0.68 - 2.91)
abuse 3.02)

What type of study design was used in this study?


Choose one answer.
a. Case Control study
b. Correlational study
c. Cross sectional study
d. Cohort study
e. Randomised controlled study
In this study the authors have identifed a group of children who were abused / neglected (exposure) and
a group of similar children who were not abused (non-exposed group) and followed them up for 11
years to see how many each group developed depression in adulthood (outcome). This is a prospective
cohort study. Companion to Psychiatric Studies, 7th edn, pg 171
Incorrect
Marks for this submission: 0/1.
Question 2
Marks: 0/1
In the above study, Abused & neglected children were matched to the control group on the basis of age,
race, gender, and family class. The authors did this to reduce which of the below:
Choose one answer.
a. Dropouts
b. Reverse causality
c. Random error
d. Confounders
e. Bias
Age, race, gender, and family class are associated with being abused and is independently associated
with depression, which makes them confounders. To reduce the effect of confounders, authors have
matched the exposed group with the non-exposed group on these confounders.
Critical Appraisal for Psychiatry by Lawrie, McIntosh and Rao, 2005, pg 48-49
Incorrect
Marks for this submission: 0/1.
Question 3
Marks: 0/1
The interviewers and the subjects were blind to the purpose of the study. This has been done to reduce
which of the below:
Choose one answer.
a. Confounders
b. Random error
c. Dropouts
d. Bias
e. Systematic error
Blinding of the interviewers was to prevent measurement bias. Blinding of the participants was to
prevent recall bias.
Critical Appraisal for Psychiatry by Lawrie, McIntosh and Rao, 2005, pg 32
Incorrect
Marks for this submission: 0/1.
Question 4
Marks: 0/1
In the study, the authors used the National Institute of Mental Health Diagnostic Interview Schedule, to
determine DSM-III-R Major Depressive Disorder and other psychiatric diagnoses. This would reduce
the possibility of which of the below.
Choose one answer.
a. Measurement bias
b. Selection bias
c. Publication bias
d. Surveillance bias
e. Recall Bias
Use of reliable measures is done to reduce measurement bias.
Page & Henderson (2008) EBMH NOTEBOOK
Appraising the evidence: what is measurement bias? Evdidence Based Mental Health 11:36-37
Incorrect
Marks for this submission: 0/1.
Question 5
Marks: 0/1
The authors were able to interview only 75% of the original sample. The ideal dropout rate which is
usually allowed for while accepting the results of such a study is:
Choose one answer.
a. 20%
b. 90%
c. 50%
d. 80%
e. 10%
Drop-outs are inevitable in cohort studies. A drop out of more than 20% of participants in a cohort
studies makes the results of the study questionable. Usually authors tend to compare people who drop-
out with people who do continue in the study to ensure that there is no difference between the two
groups.
Companion to Psychiatric Studies, 7th edn, pg 171
Incorrect
Marks for this submission: 0/1.
Question 6
Marks: 0/1
According to this study, which of the below factors in childhood is a risk factor for a current diagnosis
of major depressive disorder.
Choose one answer.
a. Neglect
b. None of the factors
c. Multiple abuse
d. Sexual abuse
e. Physical abuse
When you look at the table under the 'current diagnosis', you can see that for all the factors the relative
risk is over 1, but the confidence intervals for Neglect is the only one which does not cross one, which
makes it the significant predictor of current diagnosis of depression.
Companion to Psychiatric Studies, 7th edn, pg 192
Incorrect
Marks for this submission: 0/1.
Question 7
Marks: 0/1
Which of the below can we deduce from this study?
Choose one answer.
a. Being neglected in childhood is significantly associated with a lifetime diagnosis of MDD
b. Being physically abused in childhood is significantly associated with current diagnosis of
MDD
c. Being neglected in childhood may be a causative factor for a current diagnosis of MDD in
young adults.
d. Being subject to multiple abuses maybe a causative factor for a current diagnosis of MDD in
young adults.
e. Being sexually abused in childhood maybe a causative factor for major depression in later life

When you look at the table under the 'current diagnosis', you can see that for all the factors the relative
risk is over 1, but the confidence intervals for Neglect is the only one which does not cross one, which
makes it the significant predictor of current diagnosis of depression. By using this type of study the
authors are able to demonstrate a temporal sequence which is a key criteria for establishing causality.
Companion to Psychiatric Studies, 7th edn, pg 180
Incorrect
Marks for this submission: 0/1.
Question 8
Marks: 0/1
On the basis of this study, multiple abuse in childhood can be considered as a causative factor for
lifetime diagnosis of MDD due to which of the following reasons?
Choose one answer.
a. Dose-response relationship can be established
b. All confounders have been accounted for
c. Temporal relationship can be established
d. Biases have been minimised
e. Biological plausibility can be established
As the relative risk for "multiple abuse' is above 1, and the Confidence Intervals do not cross 1, it is a
significant predictor for Major Depressive Disorder in young adults. In this prospective cohort study, a
temporal relationship between Multiple abuse and MDD can be established.
Companion to Psychiatric Studies, 7th edn, pg 192
Incorrect
Marks for this submission: 0/1.
Question 9
Marks: 0/1
For questions 9 - 12 read the précis below: (Substance misuse & relapse of symptoms: Wade, D. et al.
The British Journal of Psychiatry 2006; 189: 229-234)

Wade et al., followed up 47 patients with first episode psychosis without substance misuse in
remission, and 51 first episode psychosis with substance misuse for a period of 15 months. They
plotted the percentage of people who had not had a relapse of symptoms vs. time (see figure below).
This type of plot is termed as
Choose one answer.
a. Receiver operator characteristic Curve
b. Funnel plot
c. Forest plot
d. Scatter plot
e. Kaplan-Meier survival curve
Kaplan-Meier survival curves (K-M curves) are used to represent outcomes that are times to an event.
K-M curves represent the proportion of the study population still surviving (or free of disease or some
other outcome) at successive times. KM curves for the intervention of interest and the comparator are
often represented on the same graph and a p-value can be calculated to determine the likelihood that
there is no difference between the two survival curves. As the number of subjects in each intervention
group decreases over time, the curves are more precise in the earlier periods (left hand side of the
survival curves) than later periods (right hand side of the survival curves). In order to account for this,
the relative risk (RR) of the event of interest over the entire study period needs to be weighted for the
number of subjects available over time (weighted RR is called the Hazard Ratio). Companion to
Psychiatric Studies, 7th edn, pg 198
Incorrect
Marks for this submission: 0/1.
Question 10
Marks: 0/1
From the above figure, the median time for relapse for patients with substance misuse is
Choose one answer.
a. Cannot be calculated
b. 250
c. 550
d. 350
e. 450
Median time to relapse is the time at which half the sample would have had the event of interest. In this
study, the time at which 50% of the substance misuse group would have had a relapse is ~ 350 days.
Critical Appraisal for Psychiatry by Lawrie, McIntosh and Rao, 2005, pg 78-79
Incorrect
Marks for this submission: 0/1.
Question 11
Marks: 0/1
Wade et al., followed up 47 patients with first episode psychosis without substance misuse in
remission, and 51 first episode psychosis with substance misuse for a period of 15 months. The authors
conducted a Cox regression analysis controlling for the effects of gender, age, psychotic disorder
diagnosis, duration of untreated psychosis and medication compliance, and found that the hazard ratio
for time to relapse in patients with substance misuse when compared to patients without substance
misuse to be 2.8 (95% CI 1.2-6.7, P=0.021). The authors have controlled for the above factors to
reduce the effect of
Choose one answer.
a. Confounders
b. Bias
c. Small sample size
d. Type II error
e. Type I error
Gender, age, psychotic disorder diagnosis, duration of untreated psychosis and medication compliance,
are associated with substance misuse and is independently associated with relapse of psychosis, which
makes them confounders. To reduce the effect of confounders, authors have used statistical tests to
control for these confounders.
Companion to Psychiatric Studies, 7th edn, pg 170 and 179 for more on confounding (for more on Cox
regression see Critical Appraisal for Psychiatry by Lawrie, McIntosh and Rao, 2005, pg 79)
Incorrect
Marks for this submission: 0/1.
Question 12
Marks: 0/1
From the above study, which of the following statements can be deduced
Choose one answer.
a. Substance misuse is not associated with time to relapse in patients with psychosis.
b. Substance misuse is associated with significantly shorter time to relapse in patients with
psychosis.
c. Time to relapse in patients who misuse substances are similar to patients who do not misuse
substance.
d. Substance misuse is associated with reduced relapse rates in patients with psychosis.
e. Substance misuse is significantly associated with longer time to relapse in patients with
psychosis.
From the above study, the hazard ratio for time to relapse in patients with substance misuse when
compared to patients without substance misuse to be 2.8 (95% CI 1.2-6.7, P=0.021). This means that
patients with substance misuse have a 2.8 times higher chance of earlier relapse compared to patients
without substance misuse, and this result is significant as the HR does not cross 1.
Companion to Psychiatric Studies, 7th edn, pg 192 for understanding confidence intervals; Critical
Appraisal for Psychiatry by Lawrie, McIntosh and Rao, 2005, pg 79
Incorrect
Marks for this submission: 0/1.
Question 13
Marks: 0/1
For questions 13 - 18 read the precis below:Depression & medical problems (Farmer A, et al., The
British Journal of Psychiatry 2008,192: 351-355).

Farmer et al., were interested to know the rates of physical disorders in people with recurrent
depression. 1546 participants with recurrent depression and 884 controls were interviewed by trained
graduate psychologists about the lifetime occurrence of 16 physical health diagnoses. Participants who
had experienced two or more episodes of major depression were recruited from psychiatric clinics &
hospitals (26%) and general practices (46%), and through self-help groups and media advertisement
(28%). Control participants were recruited through the UK general practice and were screened by
telephone interview.

All participants with depression were given a face-to-face semi-structured diagnostic interview, the
Schedules for the Clinical Assessment of Neuropsychiatry (SCAN) to establish the diagnosis of
recurrent depression Potential control group participants were interviewed by telephone using a
modified version of the Past History Schedule, and were included if there was no evidence of past or
present clinically significant psychiatric disorder
For the lifetime diagnosis of physical health disorders, a short structured interview was given in which
all participants in the cases and control groups were asked whether they had ever been treated by their
general practitioner for the below health problems: asthma, diabetes (insulin-dependent and non-
insulin-dependent), epilepsy, gastric ulcer, hypercholesterolaemia (elevated blood lipids), hypertension,
kidney disease, liver disease, myocardial infarction, osteoarthritis, osteoporosis, rheumatoid arthritis,
rhinitis or hay fever, stroke or thyroid disease. The interview was given face to face for the cases group
and by telephone interview for the control group.
In order to check the accuracy of self-report, the general practitioners of 30 representative participants
were asked to complete a checklist regarding the medical disorders listed above and asked to indicate
whether their patient had ever been treated for any of these disorders.

The results from this study are in the table below:

Medical disorder Depression vs. Control

Odds ratio (95% CI)


Epilepsy 3.06 (0.90 - 10.47)
Gastric Ulcer 4.31 (1.94 - 9.57)
Hypertension 2.20 (1.51 - 3.22)
Osteoarthritis 3.05 (1.83 - 5.08)
Rhinitis 3.29 (1.77 - 6.13)
Thyroid disease 2.78 (1.55 - 5.01)
Diabetes 2.06 (0.84 - 5.04)
Stroke 3.30 (0.97 - 11.50)

What type of study design was used in this study?


Choose one answer.
a. Correlational study
b. Cohort study
c. Randomised controlled study
d. Cross sectional study
e. Case Control study
In a case control study individuals with a particular condition (cases) are selected and compared with
individuals without the disease (controls). In this study the cases were individuals with 2 or more
episodes of depression whilst the controls were selected from GP practices. Companion to Psychiatric
Studies, 7th edn, pg 169
Incorrect
Marks for this submission: 0/1.
Question 14
Marks: 0/1
Participants who had experienced two or more episodes of major depression of at least moderate
severity were recruited from psychiatric clinics and hospitals (26%) and general practices (46%), and
through self-help groups and media advertisement (28%). This method of ascertaining cases would
have reduced which of the below
Choose one answer.
a. Measurement bias
b. Selection bias
c. Recall Bias
d. Surveillance bias
e. Publication bias
Selection bias occurs when the participants are unrepresentative of their group. Sampling cases from
three sources decreases the probability that the cases are unrepresentative.
Companion to Psychiatric Studies, 7th edn, pg 178
Incorrect
Marks for this submission: 0/1.
Question 15
Marks: 0/1
The authors have relied on self-reports from the subjects regarding presence of medical disorders. This
method of ascertaining physical disorders by the authors could have led to which of the below in this
study
Choose one answer.
a. Publication bias
b. Selection bias
c. Measurement bias
d. Surveillance bias
e. Recall bias
Measurement bias arises through the systematic misclassification of disease or exposure or both by the
researcher or by the subject. In this situation cases may be more likely to recall a medical illness than
controls.
Companion to Psychiatric Studies, 7th edn, pg 178
Incorrect
Marks for this submission: 0/1.
Question 16
Marks: 0/1
The authors have checked with some of the subjects' General Practitioner's for accuracy of the self-
report of medical disorders from the subjects. The statistical test that would best suited to assess degree
of agreement between self reports and GP reports would be:
Choose one answer.
a. Cronbach's alpha
b. Chi square
c. Pearson's
d. Spearmans rho
e. Cohen's kappa
Cohen's kappa is a measure of inter-rater reliability. A simple correlation is inadequate as one could
measure a very high correlation and not detect that one rater was consistently rating higher than
another.
Companion to Psychiatric Studies, 7th edn, pg 186
Incorrect
Marks for this submission: 0/1.
Question 17
Marks: 0/1
From the above study, which of the disorders is the best predictor for recurrent depression
Choose one answer.
a. Rhinitis
b. Ostearthritis
c. Stroke
d. Gastric ulcer
e. Hypertension
Gastric ulcer is the predictor with the highest odds ratio - this means that if an individual has gastric
ulcer it is estimated that they have around 4 times the odds of having recurrent depression.
Companion to Psychiatric Studies, 7th edn, pg 187
Incorrect
Marks for this submission: 0/1.
Question 18
Marks: 0/1
What cannot be deduced from this study?
Choose one answer.
a. Rhinitis is significantly associated with recurrent depression
b. Hypertension is a causative factor in the aetiology of recurrent depression
c. Gastric ulcer is significantly associated with recurrent depression
d. Epilepsy and recurrent depression do not have a significant association.
e. Thyroid disease is significantly associated with recurrent depression
Bradford Hill proposed some criteria for establishing causality - the most important of which is a
temporal sequence - this cannot be demonstrated in a case -control study. It was not shown that
hypertension preceded the depression.
Companion to Psychiatric Studies, 7th edn, pg 180
Incorrect
Marks for this submission: 0/1.
Question 19
Marks: 0/1
For questions 19 - 28 read the precis below: Aripiprazole in mania: Vieta et al., The British Journal of
Psychiatry (2005) 187: 235-242.

Vieta et al., compared aripiprazole and haloperidol in patients with bipolar I disorder experiencing
acute manic or mixed episodes. Patients were eligible for enrolment in the study if they were aged 18–
65 years, with a DSM–IV diagnosis of bipolar I disorder, receiving in-patient or out-patient treatment
for an acute manic or mixed episode. Exclusion criteria were the presence of rapid-cycling bipolar I
disorder; duration of the current manic episode of more than 4 weeks; proven substance misuse; patient
considered unresponsive to antipsychotics; patient at significant risk of suicide; recent treatment with a
long-acting antipsychotic, lithium or divalproate; use of psychotropic medications (other than
benzodiazepines) within 1 day of randomisation; fluoxetine treatment in the past 4 weeks; and previous
enrolment in an aripiprazole clinical study.

Patients (N=347) were randomised to receive either aripiprazole (15 - 30 mg) or haloperidol (10-15
mg), using a fixed randomisation schedule allocating patients between the two treatment arms in a 1:1
ratio. The primary outcome measure was the number of patients in response ( 50% improvement from
baseline in Young Mania Rating Scale score) and receiving therapy at week 12. Assessments were
made in a double blind manner, at baseline, days 4, 7, 10 and 14, then weekly until week 6 and every 2
weeks during weeks 6–12. Secondary efficacy measures included the time to discontinuation for any
reason. All efficacy analyses were performed on the last observation carried forward (LOCF) and
observed cases data-sets. Time to discontinuation was evaluated using the log rank test. The authors
used intention to treat analysis.

A total of 347 patients were randomised to medication (aripiprazole, n=175; haloperidol, n=172). 89
(50.9%) and 50 (29.1%) patients had completed 12 weeks of aripiprazole or haloperidol treatment,
respectively (P < 0.001; difference 21.8%, 95% CI 11.4–31.7).

At week 12, more patients taking aripiprazole (49.7%) were in response and receiving therapy
compared with those taking haloperidol (28.4%). Both aripiprazole and haloperidol treatment produced
marked improvements in mean YMRS total scores from baseline. At week 12, YMRS total scores
showed mean reductions of 19.9 with aripiprazole and 18.2 with haloperidol from baseline (LOCF
analysis; P=0.226). Extrapyramidal adverse events were more frequent with haloperidol than
aripiprazole group (62.7% v. 24.0%).

What type of study design was used in this study?


Choose one answer.
a. Ecological study
b. Systematic review & Meta-analysis
c. Randomised controlled study
d. Cross sectional study
e. Cross sectional study
The patients were randomised to either an experimental group (aripiprazole) or a control group
(haloperidol). This is a randomised controlled study. Companion to Psychiatric Studies, 7th edn, pg 173
Incorrect
Marks for this submission: 0/1.
Question 20
Marks: 0/1
The authors of this study have used stringent inclusion and exclusion criteria for selecting the sample.
This would affect which of the below factors related to the study results
Choose one answer.
a. Reduced generalisability
b. Increased generalisability
c. Improved power
d. Increased random errors
e. Reduced power
The more patients are excluded the less generalisable the trial is (that is the less likely the result will
apply to a typical patient). Trials with few exclusion criteria and therefore greater generalisability are
known as pragmatic trials
Companion to Psychiatric Studies, 7th edn, pg 173-174
Incorrect
Marks for this submission: 0/1.
Question 21
Marks: 0/1
The authors have randomised patients to receive either aripiprazole or haloperidol. The purpose of
doing this would be
Choose one answer.
a. To reduce random errors
b. To reduce the effect of confounders
c. To provide patients more choice
d. To reduce bias
e. To improve generalisability
Randomisation is an attempt to evenly distribute confounders (in this case confounders are other
reasons that treatment type for patient outcomes to be different). Some confounders might be known
and easily matched e.g. age and sex but there may be other unknown reasons why some patients will
have better outcomes and randomisation reduces the possibility that all patients with a good prognosis
will be in the same group.
Companion to Psychiatric Studies, 7th edn, pg 173
Incorrect
Marks for this submission: 0/1.
Question 22
Marks: 0/1
The study investigators and the patients were blind to the treatment that the patients were receiving
during the study. This was done to reduce which of the below
Choose one answer.
a. Power
b. Random error
c. Systematic error
d. Confounding
e. Bias
Blinding reduces the possibility that a patient who believes in the experimental drug has a placebo
response producing a reporting bias or the doctor has an observer bias in favour of the tested
compound.
Critical Appraisal for Psychiatry by Lawrie, McIntosh and Rao, 2005, pg 32
Incorrect
Marks for this submission: 0/1.
Question 23
Marks: 0/1
The drop-out rate of patients from both treatments after 12 weeks of treatment with either treatment is
Choose one answer.
a. 20%
b. 70%
c. 50%
d. 40%
e. 60%
We are given the number of patients who complete in each group and the number in each group at the
start - the overall completion rate is ((89+50)/(175+172))x100 = 40% - therefore the dropout rate is
100%-40%=60%
Companion to Psychiatric Studies, 7th edn, pg 175
Incorrect
Marks for this submission: 0/1.
Question 24
Marks: 0/1
The relative risk reduction at week 12 from using aripiprazole over haloperidol is
Choose one answer.
a. 90%
b. 75%
c. 50%
d. 21%
e. 40%
Relative risk reduction from using ariprazole is the (control event rate-experimental event rate)/control
event rate (0.284-0.497)/0.284 =- 0.75 (-75%). The risk of responding to aripirpazole is 75% greater
than the risk of repsonding to haloperidol.
Companion to Psychiatric Studies, 7th edn, pg 188
Incorrect
Marks for this submission: 0/1.
Question 25
Marks: 0/1
The number of additional patients to be treated with aripiprazole to achieve benefit in one patient
would be:
Choose one answer.
a. 10
b. 5
c. 3
d. 21
e. 1
This is the Number Need to Treat (NNT) which can be calculated as 1/Absolute Risk Reduction
(1/ARR). In this case 1/(0.497-0.284)
Companion to Psychiatric Studies, 7th edn, pg 188
Incorrect
Marks for this submission: 0/1.
Question 26
Marks: 0/1
The number of patients who need to be treated with haloperidol to cause extrapyramidal side effects in
one patient would be:
Choose one answer.
a. 100
b. 3
c. 33
d. 10
e. 6
This is the Number Needed to Harm (NNH) 1/Absolute Risk Reduction - in this case 1/(0.627-
0.24)=2.58. The NNH is always rounded up to give a whole number.
Critical Appraisal for Psychiatry by Lawrie, McIntosh and Rao, 2005, pg 129
Incorrect
Marks for this submission: 0/1.
Question 27
Marks: 0/1
From the above study, which of the following statements can be deduced:
Choose one answer.
a. Haloperidol and aripiprazole have similar incidence of extra pyramidal adverse effects
b. Aripiprazole and haloperidol have similar efficacy in the treatment of symptoms of mania
c. Patients on aripiprazole and haloperidol have similar number of responders at 12 weeks of
treatment
d. Patients on haloperidol and aripiprazole have similar effect on cognition
e. Patients on aripiprazole have similar drop-out rates as patients on haloperidol
The YMRS are similar and the p value is greater than 0.05 (P=0.226).
Companion to Psychiatric Studies, 7th edn, pg 191
Incorrect
Marks for this submission: 0/1.
Question 28
Marks: 0/1
The results of this study would lead you to conclude which of the below
Choose one answer.
a. Aripiprazole is associated with high dropout rates in patients with mania
b. Aripiprazole is a good alternative to haloperidol in treatment of mania
c. Aripiprazole should be used as a adjunct treatment with a mood stabiliser
d. Aripiprazole has more efficacy than haloperidol in the treatment of mania
e. Aripiprazole should replace haloperidol as a first line treatment of mania
Aripiprazole is a good alternative because it produces fewer EPSE, caused fewer drop outs and has a
similar efficacy.
Incorrect
Marks for this submission: 0/1.
Question 29
Marks: 0/1
For questions 29 - 46 read the precis below:Clozapine vs. standard antipsychotics. Moncrieff, J. The
British Journal of Psychiatry 2003;183:

The authors were interested to assess the evidence comparing clozapine with conventional
antipsychotics in the treatment of treatment resistant schizophrenia. The authors conducted an
electronic search using Medline and EMBASE from 1998 to April 2003 to identify trials conducted
using the keywords CLOZAPINE and RANDOMISED CONTROL TRIAL or CONTROLLED TRIAL
or RANDOMISED TRIAL. They identified nine trials that compared Clozapine with other
antipsychotic drugs in the treatment of schizophrenia. Individual study results were tabulated and
examined. The difference in symptom scores between the clozapine group and the comparison group at
the end of treatment as a percentage of the post-treatment score in the control group was calculated.

Meta-analysis was conducted to examine heterogeneity between studies, which is assessed by testing
the weighted variation of individual study results about the mean effect. The outcomes of individual
trials were converted to standardised mean differences (SMDs) to allow the results of studies using
different outcome measures to be combined. Intention-to-treat data were used if possible. The results
are presented in the plot below.

What type of study design was used in this study?


Choose one answer.
a. Cross sectional study
b. Cross sectional study
c. Ecological study
d. Randomised controlled study
e. Systematic review & Meta-analysis
A meta-analysis is the weighted average of the results from two or more studies and is often conducted
alongside a systematic review. Systematic reviews identify and cite studies in a pre-specified and
reliable way (including defined search strategies, inclusion & exclusion criteria etc). Companion to
Psychiatric Studies, 7th edn, pg 176 and 200
Incorrect
Marks for this submission: 0/1.
Question 30
Marks: 0/1
The authors have used multiple databases to identify relevant studies. This process would reduce which
of the following?
Choose one answer.
a. Selection bias
b. Language bias
c. Publication bias
d. Measurement bias
e. Recall bias
Using multiple databases helps to reduce selection bias by examining as many published studies as
possible. Medline is biased towards North American Studies whilst BIDS and Psychlit are biased
towards European ones.
Companion to Psychiatric Studies, 7th edn, pg 176
Incorrect
Marks for this submission: 0/1.
Question 31
Marks: 0/1
The authors have assessed the quality of the studies. Which of the below factors would not have been a
factor while assessing the quality of the studies
Choose one answer.
a. Prospective nature of studies
b. Generalisability
c. Funding by pharmaceutical company
d. Sample size
e. Standard error
Whilst pharmaceutical funding can influence how and whether results are published it would be
inappropriate to assume the quality of the study was influenced by this. Studies with a large sample,
prospective design, with good generalisability producing results with a small standard error are
indicative of a good quality study.
Companion to Psychiatric Studies, 7th edn, pg 177
Incorrect
Marks for this submission: 0/1.
Question 32
Marks: 0/1
The figure shown in this study is termed as
Choose one answer.
a. Galbraith plot
b. Stem Leaf plot
c. Forest plot
d. Receiver operating Curve
e. Funnel plot
The figure shown is a forest plot. The size of the squares represents the sample size, the midpoint
represents the effect size and the horizontal line represents the confidence interval.
Companion to Psychiatric Studies, 7th edn, pg 192-193
Incorrect
Marks for this submission: 0/1.
Question 33
Marks: 0/1
The number of trials which show a significant difference between Clozapine and Conventional
antipsychotic drugs is
Choose one answer.
a. 4
b. 8
c. 0
d. 6
e. 1
The number of trials which show a significant effect can be identified by counting the studies which
have a confidence interval(represented by the horizontal line) that does not cross zero.

Companion to Psychiatric Studies, 7th edn, pg 176 & 177


Incorrect
Marks for this submission: 0/1.
Question 34
Marks: 0/1
The two studies which showed the most significant difference between Clozapine and standard
antipsychotics is
Choose one answer.
a. Kumra & Klieser studies
b. Kane88 & Kane 01 studies
c. Kane88 & Rosenheck studies
d. Essock & Kane 01 studies
e. Rosenheck & Kane 01 studies
The study which is furthest from the line of no effect with the smallest confidence intervals has the
most significant difference.
Companion to Psychiatric Studies, 7th edn, pg 192-193
Incorrect
Marks for this submission: 0/1.
Question 35
Marks: 0/1
The number of trials which show a superior efficacy of Clozapine over the standard antipsychotics is
Choose one answer.
a. 6
b. 0
c. 4
d. 1
e. 8
There are four studies which have confidence intervals which do not cross the line of no difference and
have an effect which favours clozapine. Companion to Psychiatric Studies, 7th edn, pg 192-193
Incorrect
Marks for this submission: 0/1.
Question 36
Marks: 0/1
The number of trials which show a superior efficacy of standard antipsychotics over Clozapine is
Choose one answer.
a. 1
b. 6
c. 8
d. 0
e. 4
There are no studies which have confidence intervals which do not cross the line of no difference and
have an effect which favours conventional antipsychotics. Companion to Psychiatric Studies, 7th edn,
pg 192-193
Incorrect
Marks for this submission: 0/1.
Question 37
Marks: 0/1
From the plot above, choose the correct statement
Choose one answer.
a. Effect size of Hownitz study is larger than the effect size of the Kane 01 study
b. There is significant heterogeneity between studies
c. The effect size of the Rosenheck study was larger than the Effect size of the Hong Study
d. The confidence intervals for the Kenneth study was broad
e. The results of the Kumra study are not significant as the Confidence intervals cross unity
Heterogeneity can be seen on the forest plot as the confidence intervals do not all overlap.
Click here for reference
Incorrect
Marks for this submission: 0/1.
Question 38
Marks: 0/1
From the precis above, the conclusion that can be reached is
Choose one answer.
a. It is difficult to reach any conclusions due to the heterogeneity among the studies
b. Clozapine and Standard antipsychotics are of comparable efficacy in the treatment of Resistant
schizophrenia
c. Standard Antipsychotics are better than Clozapine in the treatment of Resistant schizophrenia.
d. Clozapine treatment is not helpful in the treatment of resistant schizophrenia.
e. Clozapine is more efficacious than Standard antipsychotics in the treatment of Resistant
schizophrenia
A forest plot which shows a high degree of heterogeneity must be interpreted with caution.
Click here for reference
Incorrect
Marks for this submission: 0/1.
Question 39
Marks: 0/1
The authors of the above study plotted the relative risk vs. the sample size of each of the studies they
had identified (see below).

This type of plot is termed as


Choose one answer.
a. A Funnel plot
b. A Galbraith plot
c. A receiver operator characteristic curve plot
d. A forest plot
e. A stem leaf plot
A funnel plot is a plot of the study effect size (in this case relative risk) against its precision (often SE
but in this case the number of patients). Companion to Psychiatric Studies, 7th edn, pg 201-202
Incorrect
Marks for this submission: 0/1.
Question 40
Marks: 0/1
This type of plot is used to determine which of the following
Choose one answer.
a. Selection bias
b. Sampling bias
c. Language bias
d. Measurement bias
e. Publication bias
A funnel plot determines publication bias - small studies should produce estimates of effect that vary
more widely, whilst large studies should produce estimates of effect size that converge on the true
effect size.
Critical Appraisal for Psychiatry by Lawrie, McIntosh and Rao, 2005, pg 81
Incorrect
Marks for this submission: 0/1.
Question 41
Marks: 0/1
From the graph above, which of the following can be deduced
Choose one answer.
a. There is publication bias as there is a lack of smaller negative studies
b. There is publication bias due to absence of larger negative studies
c. There is no publication bias due to a large number of studies included
d. There is no publication bias as a inverted funnel can be seen around the line of true effect
e. Not enough information available to determine publication bias
The presence of a publication bias can be inferred by the lack of small negative studies which mean
that a characteristic funnel shape has not been produced.
Critical Appraisal for Psychiatry by Lawrie, McIntosh and Rao, 2005, pg 81
Incorrect
Marks for this submission: 0/1.
Question 42
Marks: 0/1
Heterogeneity of studies included in such a study can be studied by the following methods except
Choose one answer.
a. Relative operator characteristic curve plot
b. Forest plot
c. Radial plot
d. Galbraith plot
e. Funnel plot
A Galbraith plot is also known as Galbraith's radial plot and is the same as a Radial plot. A
Galbraith/Radial plot, funnel plots and forest plots can demonstrate heterogeneity. Heterogeneity is a
sytematic difference in effects between studies beyond that expected by chance (it can also be
examined by calculating a Q statistic.
Critical Appraisal for Psychiatry by Lawrie, McIntosh and Rao, 2005, pg 81-82
Incorrect
Marks for this submission: 0/1.
Question 43
Marks: 0/1
The following statements are NOT true regarding Systematic review
Choose one answer.
a. Provides the best form of evidence
b. Facilitates assimilation of vast amounts of information in a short time
c. Most rigorous form of experimental study
d. SRs can interpret heterogeneity
e. Much cheaper to conduct than a RCT
A systematic review is not a form of experimental study. A systematic review aims to identify and cite
studies in a pre-specified and reliable way (including defined search strategies, inclusion & exclusion
criteria etc).
Companion to Psychiatric Studies, 7th edn, pg 166 and 176
Incorrect
Marks for this submission: 0/1.
Question 44
Marks: 0/1
Which of the following is not true about Systematic Review and Meta-analysis,
Choose one answer.
a. Is not helpful if carried out early in the hypothesis testing cycle.
b. Is independent of the quality of available studies.
c. Provides a summary effect size.
d. Is subject to the effects of publication bias.
e. Is affected by selection bias.
Systematic review and meta-analysis is highly dependent on the quality of the available studies.
Companion to Psychiatric Studies, 7th edn, pg 177
Incorrect
Marks for this submission: 0/1.
Question 45
Marks: 0/1
This study design is given most weight in a meta-analysis
Choose one answer.
a. Case reports
b. Cross sectional survey
c. Case control study
d. RCT
e. Cohort study
Meta-analysis of RCTs comes top in the classification of evidence levels.
Companion to Psychiatric Studies, 7th edn, pg 212
Incorrect
Marks for this submission: 0/1.
Question 46
Marks: 0/1
The following are NOT true of Forest plots,
Choose one answer.
a. The overall estimate from the meta-analysis and its confidence interval are put at the bottom,
represented as a diamond.
b. Each square shows the point estimate of the results of each study
c. The size of each square is independent of the weight assigned to each individual study.
d. The horizontal lines running through each square represent the confidence interval (95% CI)
for the study.
e. If the horizontal line crosses one, then there is no difference between the two groups being
compared.
The size of each square represents the study size.
Companion to Psychiatric Studies, 7th edn, pg 192
Incorrect
Marks for this submission: 0/1.
Question 47
Marks: 0/1
A new screening test for depression was developed to be used in General Practises. It was found to
have a sensitivity of 90% and a Likelihood ratio for negative result of 0.20. A Fagan‘s Nomogram has
been provided below.

The prevalence of depression in a community is around 10%. From the above precis, calculate the
probability of a person having depression when they test negative on the new screening test.
Choose one answer.
a. 20%
b. 10%
c. 2%
d. 5%
e. 0.02%
The pre-test probability is 10% (we know the prevalence in the community - a random person has a
10% chance of depression before we do the test). To use the nomogram draw a straight line from the
pre-test probability through the negative liklihood ratio of 0.2 to arrive at the post-test probability of
2%. Companion to Psychiatric Studies, 7th edn, pg 209
Incorrect
Marks for this submission: 0/1.
Question 48
Marks: 0/1
In the above example, the likelihood ratio for a positive result would be
Choose one answer.
a. 0.2
b. 4
c. 1.8
d. 6
e. 0.8
The likelihood ratio for a negative result is (1-sensitivity)/specificity. Knowing the LR- is 0.2 and the
sensitivity is 0.9 we can calculate a specificity of 0.5. The liklihood ratio for a postive result is
sensitivity/(1-speciificty) in this case 0.9/(1-0.5) =1.8
Companion to Psychiatric Studies, 7th edn, pg 209
Incorrect
Marks for this submission: 0/1.
Question 49
Marks: 0/1
A new scale for depression was compared with the Becks depression inventory. The cut-off scores to
diagnose depression using the new scale, and the value of sensitivity and specificity at each of the
values is provided in the graph below (Graph 1).
This type of graph is termed as which of the below:
Choose one answer.
a. A stem-leaf plot
b. A Funnel plot
c. A forest plot
d. A Galbraith plot
e. A Receiver Operator Characteristics curve
A Receiver Operator Characteristics curve plots the true positive rate (sensitivity) against the false
positive rate (1 minus speciificty). Companion to Psychiatric Studies, 7th edn, pg 710
Incorrect
Marks for this submission: 0/1.
Question 50
Marks: 0/1
From the above graph, choose the cut-off score as per the new depression scale, at which the best
compromise between sensitivity and specificity is obtained is
Choose one answer.
a. 5
b. 6
c. 7
d. 9
e. 8
After point 6 on the graph the small increases in sensitivity come with large increases in the false
positive rate. (In other circumstances, for example when the test is being used a screening test rather
than to give definitive diagnosis a higher false positive rate might be tolerated.)
Critical Appraisal for Psychiatry by Lawrie, McIntosh and Rao, 2005, pg 98-99
Incorrect
Marks for this submission: 0/1.

Statistical Tests
1
Marks: 0/1
In a study, each subject takes a test in two conditions, taking a different drug (A or B) before each one.
The data recorded is far from normally distributed, the test used to the compare scores between the two
conditions?
Choose one answer.
a. Mann-Whitney test
b. Kruskal Wallis test
c. McNemar test
d. Wilcoxon's Matched pairs test
e. Chi-squared test
Wilcoxon's matched pairs sign rank test is relatively free from assumptions (such as normally
distributed data) compared to its parametric equivalents and is suitable for paired or related samples.
Mann-Whitney is suitable for non-parametric data but is only for independent samples. T-tests assume
parametric data.
Critical Appraisal for Psychiatry. Lawrie SM, McIntosh AM & Rao S. 2000, P 66
Incorrect
Marks for this submission: 0/1.
Question 2
Marks: 0/1
The test to quantify degree of agreement between raters using a test with categorical variables is
Choose one answer.
a. Intra class correlation
b. Spearman's
c. Pearson's
d. Kappa
e. Chi-squared test
Kappa is a measure of agreement between two observers taking into account agreement that could
occur by chance (expected agreement).
Critical Appraisal for Psychiatry. Lawrie SM, McIntosh AM & Rao S. 2000, P 72
Incorrect
Marks for this submission: 0/1.
Question 3
Marks: 0/1
A study involves three groups of subjects whose weights and heights are measured. The test that
measures the weight and height difference between the three groups would be
Choose one answer.
a. Linear regression
b. Logistic regression
c. Kruskal Wallis test
d. t test
e. ANOVA
Analysis of variance (ANOVA) is a parametric test used to compare over two groups with normally
distributed data
Critical Appraisal for Psychiatry. Lawrie SM, McIntosh AM & Rao S. 2000, P 66
Incorrect
Marks for this submission: 0/1.
Question 4
Marks: 0/1
In a RCT, a group of patients were given a new drug for anxiety. They were asked to rate whether they
felt better or not at 2 weeks and subsequently at 4 weeks. The test to compare the difference in how
they felt at 4 weeks and 2 weeks is
Choose one answer.
a. Friedman test
b. McNemar test
c. Mann-Whitney U test
d. Chi-squared test
e. Kruskal Wallis test
MeNemar test is a binary test used to compare two paired groups
Critical Appraisal for Psychiatry. Lawrie SM, McIntosh AM & Rao S. 2000, P 66
Incorrect
Marks for this submission: 0/1.
Question 5
Marks: 0/1
In a study involving 38 patients with schizophrenia, it was found that 22 had ventricular enlargement,
whereas 16 did not. The statistical test most suited to identify the relationship between age at onset,
duration of illness, severity of illness and presence of ventricular enlargement is
Choose one answer.
a. McNemar test
b. ANOVA
c. Spearman's Rho
d. Regression analysis
e. Kruskal Wallis test
Regression analysis is a technique which examines the relation of a dependent variable (response
variable) to specified independent variables (explanatory variables). Regression analysis can be used as
a descriptive method of data analysis (such as curve fitting) without relying on any assumptions about
underlying processes generating the data. When paired with assumptions in the form of a statistical
model, regression can be used for prediction (including forecasting of time-series data), inference,
hypothesis testing, and modeling of causal relationships.
Critical Appraisal for Psychiatry. Lawrie SM, McIntosh AM & Rao S. 2000, P 76-77
Incorrect
Marks for this submission: 0/1.
Question 6
Marks: 0/1
What is the range of values for a correlation coefficient
Choose one answer.
a. 1 to infinity
b. o to infinity
c. (-infinity to +infinity)
d. (-infinity to 0)
e. (-1 to +1)
The strength or magnitude of the relationship between the two variables is expressed by a statistic
known as the CORRELATION COEFFICIENT, which varies from -1 (perfect negative relationship),
through 0 (no relationship) to +1 (perfect positive relationship). It is not possible to obtain a coefficient
more negative than -1 or greater than +1.
Incorrect
Marks for this submission: 0/1.
Question 7
Marks: 0/1
The following is true of Parametric tests
Choose one answer.
a. Can be applied on data collected on nominal & ordinal scales
b. Makes less rigorous assumptions about distribution of variables
c. Can be applied on data that is normally distributed
d. Makes less rigorous assumption about variances of comparison groups
e. Can be used for categorical data
Normally distributed data can be analysed with parametric statistics.
Critical Appraisal for Psychiatry. Lawrie SM, McIntosh AM & Rao S. 2000, P 57
Incorrect
Marks for this submission: 0/1.
Question 8
Marks: 0/1
Houses on two roads, one in a posh locality and one in a council estate, were compared for the amount
of rubbish they left for the council to collect each week. To make it easier, the council authorities
calculated the rubbish thrown as either small amounts, medium amounts or large amounts. The test to
measure the difference of rubbish thrown between houses in a posh area and the council estate is
Choose one answer.
a. ANOVA
b. Wilcoxon matched pairs test
c. Mann-Whitney U test
d. Chi-squared test
e. t test
Mann Whitney U test is a non-parametric test comparing two independent groups
Critical Appraisal for Psychiatry. Lawrie SM, McIntosh AM & Rao S. 2000, P 66
Incorrect
Marks for this submission: 0/1.
Question 9
Marks: 0/1
The following is a nonparametric test
Choose one answer.
a. Independent samples t test
b. Mann Whitney U test
c. Linear regression
d. Pearsons product moment correlation
e. ANOVA
Mann Whitney U test is a non-parametric test comparing two independent groups
Critical Appraisal for Psychiatry. Lawrie SM, McIntosh AM & Rao S. 2000, P 66
Incorrect
Marks for this submission: 0/1.
Question 10
Marks: 0/1
A researcher gives out 100 questionnaires and ask subjects ten questions about how sad certain things
make them. They are provided with a separate test to measure their stress level. The researcher is
interested to find mathematically the best equation to predict stress levels from the ten behaviours. The
statistical test they should be using would be
Choose one answer.
a. Logistic regression
b. Multiple linear regression
c. Linear Regression
d. t test
e. ANNOVA
Multiple linear regression gives the best predictive power from the final equation and describes the
relationship between the variables. Stepwise multiple linear regression produces a slightly simpler
equation (that is not statistically less predictive than that from multiple linear regression), and linear
regression can only be used when you have one independent (predictor) variable.
Critical Appraisal for Psychiatry. Lawrie SM, McIntosh AM & Rao S. 2000, P 76-77
Incorrect
Marks for this submission: 0/1.
Question 11
Marks: 0/1
A young girl is interested to find the correlation between the number of calories consumed per day and
the waist size. The Statistical test that will be of help would be
Choose one answer.
a. Intra class correlation
b. Pearson's
c. Kappa
d. Spearman's
e. ANOVA
Pearson's correlation coefficient is a correlation coefficient between two normally distributed
continuous variables.
Critical Appraisal for Psychiatry. Lawrie SM, McIntosh AM & Rao S. 2000, P 256
Incorrect
Marks for this submission: 0/1.
What type of study design was used in this study?
Choose one answer.
a. Cross sectional study
b. Cross sectional study
c. Ecological study
d. Randomised controlled study
e. Systematic review & Meta-analysis
A meta-analysis is the weighted average of the results from two or more studies and is often conducted
alongside a systematic review. Systematic reviews identify and cite studies in a pre-specified and reliable way
(including defined search strategies, inclusion & exclusion criteria etc). Companion to Psychiatric Studies, 7th
edn, pg 176 and 200
Incorrect
Marks for this submission: 0/1.
Question 30
Marks: 0/1
The authors have used multiple databases to identify relevant studies. This process would reduce which of the
following?
Choose one answer.
a. Selection bias
b. Language bias
c. Publication bias
d. Measurement bias
e. Recall bias
Using multiple databases helps to reduce selection bias by examining as many published studies as possible.
Medline is biased towards North American Studies whilst BIDS and Psychlit are biased towards European ones.
Companion to Psychiatric Studies, 7th edn, pg 176
Incorrect
Marks for this submission: 0/1.
Question 31
Marks: 0/1
The authors have assessed the quality of the studies. Which of the below factors would not have been a factor
while assessing the quality of the studies
Choose one answer.
a. Prospective nature of studies
b. Generalisability
c. Funding by pharmaceutical company
d. Sample size
e. Standard error
Whilst pharmaceutical funding can influence how and whether results are published it would be inappropriate
to assume the quality of the study was influenced by this. Studies with a large sample, prospective design, with
good generalisability producing results with a small standard error are indicative of a good quality study.
Companion to Psychiatric Studies, 7th edn, pg 177
Incorrect
Marks for this submission: 0/1.
Question 32
Marks: 0/1
The figure shown in this study is termed as
Choose one answer.
a. Galbraith plot
b. Stem Leaf plot
c. Forest plot
d. Receiver operating Curve
e. Funnel plot
The figure shown is a forest plot. The size of the squares represents the sample size, the midpoint represents
the effect size and the horizontal line represents the confidence interval.
Companion to Psychiatric Studies, 7th edn, pg 192-193
Incorrect
Marks for this submission: 0/1.
Question 33
Marks: 0/1
The number of trials which show a significant difference between Clozapine and Conventional antipsychotic
drugs is
Choose one answer.
a. 4
b. 8
c. 0
d. 6
e. 1
The number of trials which show a significant effect can be identified by counting the studies which have a
confidence interval(represented by the horizontal line) that does not cross zero.

Companion to Psychiatric Studies, 7th edn, pg 176 & 177


Incorrect
Marks for this submission: 0/1.
Question 34
Marks: 0/1
The two studies which showed the most significant difference between Clozapine and standard antipsychotics
is
Choose one answer.
a. Kumra & Klieser studies
b. Kane88 & Kane 01 studies
c. Kane88 & Rosenheck studies
d. Essock & Kane 01 studies
e. Rosenheck & Kane 01 studies
The study which is furthest from the line of no effect with the smallest confidence intervals has the most
significant difference.
Companion to Psychiatric Studies, 7th edn, pg 192-193
Incorrect
Marks for this submission: 0/1.
Question 35
Marks: 0/1
The number of trials which show a superior efficacy of Clozapine over the standard antipsychotics is
Choose one answer.
a. 6
b. 0
c. 4
d. 1
e. 8
There are four studies which have confidence intervals which do not cross the line of no difference and have an
effect which favours clozapine. Companion to Psychiatric Studies, 7th edn, pg 192-193
Incorrect
Marks for this submission: 0/1.
Question 36
Marks: 0/1
The number of trials which show a superior efficacy of standard antipsychotics over Clozapine is
Choose one answer.
a. 1
b. 6
c. 8
d. 0
e. 4
There are no studies which have confidence intervals which do not cross the line of no difference and have an
effect which favours conventional antipsychotics. Companion to Psychiatric Studies, 7th edn, pg 192-193
Incorrect
Marks for this submission: 0/1.
Question 37
Marks: 0/1
From the plot above, choose the correct statement
Choose one answer.
a. Effect size of Hownitz study is larger than the effect size of the Kane 01 study
b. There is significant heterogeneity between studies
c. The effect size of the Rosenheck study was larger than the Effect size of the Hong Study
d. The confidence intervals for the Kenneth study was broad
e. The results of the Kumra study are not significant as the Confidence intervals cross unity
Heterogeneity can be seen on the forest plot as the confidence intervals do not all overlap.
Click here for reference
Incorrect
Marks for this submission: 0/1.
Question 38
Marks: 0/1
From the precis above, the conclusion that can be reached is
Choose one answer.
a. It is difficult to reach any conclusions due to the heterogeneity among the studies
b. Clozapine and Standard antipsychotics are of comparable efficacy in the treatment of Resistant
schizophrenia
c. Standard Antipsychotics are better than Clozapine in the treatment of Resistant schizophrenia.
d. Clozapine treatment is not helpful in the treatment of resistant schizophrenia.
e. Clozapine is more efficacious than Standard antipsychotics in the treatment of Resistant schizophrenia

A forest plot which shows a high degree of heterogeneity must be interpreted with caution.
Click here for reference
Incorrect
Marks for this submission: 0/1.
Question 39
Marks: 0/1
The authors of the above study plotted the relative risk vs. the sample size of each of the studies they had
identified (see below).

EMIs
1
Marks: 0/3
Statistical tests

A. ANOVA
B. Chi-square test
C. Fischer‘s exact test
D. Friedman Test
E. Mann Whitney U test
F. McNemar test
G. Multiple regression
H. Paired t-test
I. t-test
J. Wilcoxon test

Select the most appropriate statistical tests for:

Comparing categorical data between two groups

Comparing parametric data between two independent samples with normal distribution.

Test to rule out effect of confounders on two continuous dependant variables

1.B, F – Chi-squared test (independent samples) & McNemar (paired samples) tests are two test
used to compare categorical data between two groups.
2.I – t test is parametric equivalent of chi-squared test.
3.G – A multivariate regression analysis would identify the effect of confounders.
Incorrect
Marks for this submission: 0/3.
Question 2
Marks: 0/5

Measurement

A. Interval scale
B. Mean
C. Median
D. Mode
E. Nominal scale
F. Ordinal scale
G. Range
H. Ratio scale
I. Standard deviation

Lead in: For each of the statements below, choose the most appropriate options from above.

You measure IQ scores of a group of medical students. You are interested in the IQ score that is most
frequently represented in this group

Your supervisor asks you to find how variable the IQ scores are in this group.
You want to divide the group into four categories of students: Highly intelligent, intelligent, not so
intelligent and borderline intelligent on the basis of their IQ scores. Choose One scale which would be
useful for this.

Your group is separated into males and females. Choose One scale which would be useful for this.

You are interested in the average IQ of this group, but do not want this average to be affected to
extreme IQ scores obtained by some students.

1. D - Mode is the most frequently occurring score in a distribution

2.G, I - Range and standard deviation provide information on dispersion in a sample

3.F - Ordinal scale would be useful for the ordered categories.

4.E - A nominal scale would be useful here

5.C - A median score would not be affected by the extreme values.

Incorrect
Marks for this submission: 0/5.
Question 3
Marks: 0/6

Cross sectional studies

Options.

A. 11%
B. 22%
C. 33%
D. 44%
E. 55%
F. 66%
G. 77%
H. 88%
I. 99%
Lead in: A study involved screening terminally ill people for depression with the question ‗Do you
think you are depressed?‘ A structured diagnostic interview also used to diagnose depression in the
same sample. The results of the study are in the table below. Calculate the below using the information
from the table.

Gold Standard Test


Screening Question Depressed Not Depressed
Yes to question 11 9
No to Question 14 40

What was the prevalence of depression in the sample?


What was the sensitivity of the screening?
What was the specificity of the screening?
What proportion of those who screened positive were depressed?
What proportion of those who were depressed screened positive?
What was the negative predictive value?

1. C. Prevalence = (a+c) / (a+b+c+d)  25 / 74 = 33.75%

2. D. Sensitivity = a / (a+c)  11/25 = 44%

3. H. Specificity = d / (b+d)  40 / 49 =82%

4. E. Positive predictive value = a / (a+b)  11/20 = 55%

5. D. This is sensitivity = 44%

6. G. Negative predictive value = d / (c+d) = 40 / 54 = 74%


Incorrect
Marks for this submission: 0/6.
Question 4
Marks: 0/3
A.Allocation Bias
B.Attrition Bias
C.Calculation Bias
D.Detection Bias
E.Interviewer Bias
F.Publication Bias
G.Recall Bias
H.Response Bias
I.Surveillance Bias

For each of the study types below, identify the ‗most common‘ bias/ biases associated with it.

Cohort study
Case-control study
Systematic Reviews
1.B, I – Attrition bias (loss to follow-up) and Surveillance bias (people with the exposure being
observed more closely for outcome compared to people without the exposure) are two
common biases associated with cohort studies.
2.E, G – Recall bias and Interview bias are two common biases associated with case-control
studies
3.F – Publication bias is the most often encountered bias in systematic reviews.
Incorrect
Marks for this submission: 0/3.
Question 5
Marks: 0/3
RCTs

A. Double Blinding
B. Intention To Treat analysis
C. Last Observation Carried Forward
D. Mean Imputation
E. Randomisation
F. Randomisation Concealment
G. Single Blinding
H. Survival Analysis
I. Triple Blinding
J. Worst Case Scenario Analysis

Which of the above options is most applicable to the statements below?

Strategy used to distribute confounders equally between groups.

Strategies related to analysis which aims to minimise measurement bias.

Three ways of carrying out an Intention to Treat Analysis.

1.E. Randomisation ensures that both known and unknown confounders are equally distributed between the
two groups.

2.B.Intention to treat analysis (where people randomised to a particular group are included in the same group
for final analysis) would account for attrition bias

3.C, D, J. The three ways for accounting for missing data would be LOCF, Mean imputation, & worse case
scenario analysis.
Incorrect
Marks for this submission: 0/3.
Question 6
Marks: 0/6
Study designs

A. Case-control
B. Cross over trial
C. Cross sectional survey
D. Ecological survey
E. N of 1 study
F. Open Label RCT
G. Parallel RCT
H. Prospective Cohort
J. Qualitative study
K. Systematic review

Which would be the most ethical (choose ONE) for the following research questions:

To look at the number of suicides following an episode of overdose.


To look at the number of children developing cardiac abnormalities in mothers on lithium.
To look at obstetric complications and the development of schizophrenia but you wish to avoid
attrition bias.
To simultaneously look for both exposure and outcome status whilst avoiding recall bias.
To compare a new drug for schizophrenia vs. placebo. The researchers wish to recruit fewer
participants but still have adequate power in the study.
To get patient‘s perspectives on the quality of care they receive from the mental health services.

1. A. As suicide is a rare outcome, a cohort study would be difficult to conduct, despite being the best
suited method. A case-control study looking at people who committed suicide and a control group
similar to this group, but without the people having suicide would be compared to see if there is any
difference in incidence of overdose in each of the groups would be the best suited in this condition.
2. H. A prospective cohort study is a research study that follows over time groups of individuals who are alike in
many ways but differ by a certain characteristic (for example, female nurses who smoke and those who do
not smoke) and compares them for a particular outcome (such as lung cancer).
3. A. A case control study involves identifying patients who have the outcome of interest (cases) and control
patients who do not have that same outcome, and looking back to see if they had the exposure of interest.
The exposure could be some environmental factor, a behavioural factor, or exposure to a drug or other
therapeutic intervention. A cohort study would have been best suited design, but it has problems with
attrition bias, which we are advised to avoid in this case.
4. C. Cross-sectional studies can be thought of as providing a "snapshot" of the frequency and characteristics of
a disease in a population at a particular point in time. This type of data can be used to assess the prevalence
of acute or chronic conditions in a population. However, since exposure and disease status are measured at
the same point in time, it may not always be possible to distinguish whether the exposure preceded or
followed the disease.
5. B. A cross over trial is a method of RCT, comparing 2 interventions in which patients are switched to the
alternative intervention after a specified period of time.
6. J. Qualitative methods aim to make sense of, or interpret, phenomena in terms of the meanings people bring to
them. Qualitative research may define preliminary questions which can then be addressed in quantitative
studies. A good qualitative study will address a clinical problem through a clearly formulated question and
using more than one research method (triangulation). Analysis of qualitative data can and should be done
using explicit, systematic, and reproducible methods.
Incorrect
Marks for this submission: 0/6.
Question 7
Marks: 0/3

STATISTICAL TESTS.

A. ANOVA.
B. Chi-Squared test

C. Independent t test

D. Kruskal Wallis test

E. Logistic regression

F. Mann-Whitney U test

G. McNemar test

H. Multiple regression

I. Paired t test

J. Pearson‘s product moment correlation

Lead in: A group of researchers conducted a RCT of fluoxetine vs. Sertraline in a group of depressed
patients using the Hamilton depression rating scale. Choose most appropriate test for each of the following
analysis they want to conduct.

They wanted to find if there were differences between single / non-single people in the fluoxetine
group compared to the sertraline group
They wanted to find the difference in HDRS scores between the two groups at baseline
They want to find if age and gender have any effect on improvement as defined by 50% reduction in
HDRS scores.
1. B, Chi-squared test to compare nominal data (single/non-single) between two groups (Fluoxetine group &
Sertraline group).
2. C. Independent t test to compare continuous data (age) between two groups as above. It is a procedure for
comparing the mean scores of two independent groups on a given quantitative variable.
3. E. Logistic regression to look if independent variables (age and gender) can predict dependent variable
(treatment effect). As the dependent variable is binary (improved / not improved) a logistic regression
rather than linear regression has been used.
Incorrect
Marks for this submission: 0/3.
Question 8
Marks: 0/5

Study designs:
A.Case control study
B. Cross sectional survey
C. Cluster randomized trial
D. Cross over randomized trial
E. Ecological Study
F. Economical analysis
G. Continuation open labelled trial
H. Parallel Randomized trial
I. Cohort Study
J. Qualitative Study
A researcher is interested to understand why some people adapt so well to chronic illness whereas
many people find it difficult to cope.
You are interested in the prevalence of depression and known risk factors in rural patients attending a
General practice.
A GP moves from rural Scotland to work in an inner city practice in London. They find that more
young people are presenting with psychotic symptoms compared to when they were in a practice in
Scotland. They feel that increased use of drugs in cities may be a reason for this, and would like to
devise a study to test their hypothesis
You are interested in investigating association between drug and alcohol use and a disease with an
estimated prevalence of 1/100,000.
A psychiatrist wants to establish whether a dialectical behaviour therapy helps in phobic anxiety in
comparison to standard behaviour therapy. They want to conduct this study across all the GP practices
in a city, with similar sample sizes in each practice.

1. J. Qualitative methods aim to make sense of, or interpret, phenomena in terms of the meanings
people bring to them. Qualitative research may define preliminary questions which can then be
addressed in quantitative studies. A good qualitative study will address a clinical problem
through a clearly formulated question and using more than one research method (triangulation).
Analysis of qualitative data can and should be done using explicit, systematic, and reproducible
methods
2. B. Cross-sectional studies can be thought of as providing a "snapshot" of the frequency and characteristics of
a disease in a population at a particular point in time. This type of data can be used to assess the
prevalence of acute or chronic conditions in a population. However, since exposure and disease status are
measured at the same point in time, it may not always be possible to distinguish whether the exposure
preceded or followed the disease.
3. I. A prospective cohort study is a research study that follows over time groups of individuals who are
alike in many ways but differ by a certain characteristic (for example, female nurses who smoke
and those who do not smoke) and compares them for a particular outcome (such as lung cancer)
4. A. As suicide is a rare outcome, a cohort study would be difficult to conduct, despite being the best
suited method. A case-control study looking at people who committed suicide and a control
group similar to this group, but without the people having suicide would be compared to see if
there is any difference in incidence of overdose in each of the groups would be the best suited in
this condition.
5. C. A cluster RCT would be appropriate in this situation
Incorrect
Marks for this submission: 0/5.
Question 9
Marks: 0/3

STATISTICAL TESTS.

A. ANCOVA
B. ANOVA
C. Chi square test
D. Fisher‘s exact test
E. Friedman test
F. Independent T test
G. Kruskal-Wallis test
H. Mantel Haenzel test
I. Multiple regression analysis
J. Paired T test

Select the most appropriate statistical test.


To compare 2 groups on a variable with normal distribution after adjusting for any confounding
variables.
To compare three groups of continuous variables which are shown to have skewed distribution.
To compare the difference in Blood pressure measurement taken at 3 weeks and 6 weeks, in a sample
of obese young men.

1. I. Multiple regression analysis would identify the effect of confounders on two or more variables

2. G. Kruskal-Wallis test would be the best choice for comparing three continuous variables with
skewed distributions

3. J. As BP (continuous variable) is being measured from the same group at two different time points, a
paired t test would be best suited to compare this data.

Incorrect
Marks for this submission: 0/3.
Question 10
Marks: 0/3

Study Designs

A. Audits
B. Case reports
C. Case series
D. Case-control study
E. Cohort study
F. Cross sectional studies.
G. Ecological studies
H. Qualitative study
I. Randomised control study
J. Systematic review and Meta analysis.

For each of the scenarios below identify the most appropriate study design to answer the research
question.

You are concerned about the recent increase in the use of amphetamines, and are worried that it may
be the reason for the recent rise in Psychosis in adolescents. The study design best suited to
investigate this relationship between amphetamine use and psychosis
2. You see a young gentleman with Mania and give him haloperidol to calm him down Two days later
you are told that he has developed neuroleptic malignant syndrome (NMS). You want to conduct a
study to identify risk factors that could to identification of people and risk and potentially avoid
antipsychotics in such people. The study design you ought to adopt to conduct this study is
There were reports that general practitioners (GPs) were writing more prescriptions for
antidepressants over the last few years. The psychotherapist in your hospital wants to verify these
claims to ensure that this was not the case. The study design best suited to answer this question is
1. E – Cohort studies are best type of studies to investigate questions on aetiology as it allows for
causality to be established. A RCT would be unethical in this case.
2. D – NMS is a rare event (Outcome), hence a Cohort study is not ideal. A case control study
would be the best to look for exposure (risk factors) in a rare outcome. A RCT would be
unethical in this case.
3. G – Ecological studies which consider differences between groups rather than individuals is
ideal for this question.
Incorrect
Marks for this submission: 0/3.
Question 11
Marks: 0/6

Randomised controlled studies:


A. 1
B. 1.5
C. 2
D. 3
E. 4
F. 11%
G. 22%
H. 33%
I. 44%
J. 50%
K. 66%
L. 77%

An RCT was conducted to identify the effect of CBT in the treatment of Agoraphobia compared to
placebo (control), and the results are presented in the table below:
Group Improved No improvement Total
CBT 60 30 90
Placebo 40 50 90

From the table above, calculate the following:

Control event rate


Experimental event rate
Relative benefit increase
Relative risk
Absolute Benefit increase
Number needed to treat
1. 44%
2. 66%
3. 50%
4. 1.5
5. 22%
6. 4
Incorrect
Marks for this submission: 0/6.
Question 12
Marks: 0/3
Study Designs

A. Audits
B. Case reports
C. Case series
D. Case-control study
E. Cohort study
F. Cross sectional studies.
G. Ecological studies
H. Qualitative study
I. Randomised control study
J. Systematic review and Meta analysis.

For each of the scenarios below identify the most appropriate study design to answer the research
question.

While discussing with a General Practitioner, you come to know that many patients who repeatedly
self-harm in their adolescence stop doing so after they reach the age of 24 years. You are interested to
know what factors lead them to stop harming themselves, to use these in patients who do not stop
harming themselves.

A patient with resistant schizophrenia comes to you seeking a prescription of omega 3 fatty acids for
treatment of their schizophrenia. You decide to look for evidence of omega 3 fatty acids in the
treatment of schizophrenia.

Your local NHS trust decides to implement a new guideline requiring all patients on Olanzapine to
have weekly blood glucose for the first three months after initiation of the drug. Your consultant, who
is also the Clinical Director is worried that all the consultants in the trust are not following this
guideline, and asks you to look into this.

1.H – Qualitative studies are used to collect data on people‘s experiences, feelings, values or
other types of opinion to generate hypothesis.
2.I & J – Systematic review & Meta-analysis if available provides the most robust evidence base
for any intervention, a single RCT would be the second best course of evidence for efficacy
of a treatment.
3.A – A clinical Audit measures current performance against established standards / criteria.
Incorrect
Marks for this submission: 0/3.
Question 13
Marks: 0/5
Calculations in critical appraisal

Options:

A. 0
B. 9
C. 1
D. 20
E. 4
F. 50
G. 5
H. 80
I. 6
J. 100
K. 8
L. Infinity

For each of the questions below, select the most appropriate number from the list above.
The usual upper limit of risk of type II error (expressed as a percentage) in power calculations for
randomised trials.
The odds of an event if it happens 90% of the time.
The ideal Number Needed to Treat.
The value which, if crossed, the 95% confidence interval of a Number Needed to Treat becomes non-
significant.
The sensitivity of a test, expressed as a percentage where 80 people were classified ―true positive‖ and
20 people were classified ―false negative‖
1. H - The power of a study is represented as 1-β, (1-0.2) = 80%
2. B - Odds = Probability / 1-probability. In this case, it would be 0.9/1-0.9 =9
3. C - This would be the ideal NNT.
4. L - Ideally, treatment with NNT less than 10 are not considered clinically important, but statistically,
if NNT crosses infinity, it becomes insignificant.
5. H - Sensitivity = true positive / (true positive+false negative) = 80%
Incorrect
Marks for this submission: 0/5.
Question 14
Marks: 0/5
Sources of Error

A. Attrition Bias
B. Confidence intervals
C. Confounding.
D. Measurement Bias
E. p value
F. Random error
G. Recall Bias
H. Selection Bias
I. Type I error.
J. Type II error.

Lead in: For each of the statements below, choose the most appropriate from the list above.

A RCT of olanzapine versus placebo found that there was no difference between the two in the
treatment of schizophrenia. You are surprised about the results, but on reading the study in detail, find
that despite appropriate methodology, the sample size was only six patients in each group. Choose
ONE which best explains the reason for this finding.

A study found a significant association between drinking coffee and increased incidence of myocardial
infarction. It was found that more people in the coffee drinkers group were smokers than in the control
group. Choose the most appropriate explanation for this finding.
A study finds that people with depression have higher cortisol levels. You want to be sure that this is
not a chance finding. Choose TWO which will provide you with this information.

4. A study was designed to find the association between borderline personality disorder and childhood
abuse. Patients were selected either after presenting to A & E with an overdose or from a therapeutic
community service. Controls were randomly selected from the general population. It was found that
patients presenting to A& E had higher rate of childhood abuse as opposed to patients from Therapeutic
community. Choose TWO reasons from above to explain this finding.

Two studies were conducted to assess impact of exercise on mild depression. One study which used the
Beck‘s depression Inventory found that exercise was beneficial in depression, whereas the study using
Hamilton Depression rating scale did not find any effect of exercise on depression. One reason for this
difference between the two studies could be

1.J, Type II error, i.e., false negative occurs when you accept a null hypothesis when it is false. Mainly
due to small sample sizes or large variance (noise)
2.C - A confounder is a variable that is related to the exposure (independent variable) and is an
independent risk factor for the outcome of the study (dependent variable)
3.B, E, - The role of chance finding is usually estimated using statistical techniques, p values and
confidence intervals.
4.G, H - Selection & recall bias could explain the findings.
5.D. BDI is a self-rated scale, whereas HDRS is an observer rated scale which could explain the
differences in the results of the two studies, i.e., measurement bias.

Antidepressants
1
Marks: 0/1

The duration of treatment of single episode of depression

Choose one answer.


Error! Not a valid embedded object. a. longer than 12 months
Error! Not a valid embedded object. b. 6 months after recovery
Error! Not a valid embedded object. c. at least 12 months
Error! Not a valid embedded object. d. 6 months from initiation
Error! Not a valid embedded object. e. none of the above
A single episode of depression should be treated for 6 months after recovery

Maudsley guidelines 2005-2006,8 edition,p-149


Incorrect
Marks for this submission: 0/1.
Question 2
Marks: 0/1

The following are best indicators for response to antidepressants

Choose one answer.


Error! Not a valid embedded
a. Family history of response and cost
object.
Error! Not a valid embedded b. A history of prior response to an agent and a family history of
object. preferential response
Error! Not a valid embedded
c. Side-effect profile and comorbid psychiatric symptoms
object.
Error! Not a valid embedded
d. all of the above
object.
Error! Not a valid embedded
e. Gender and sedating versus activating properties
object.
On average, all antidepressants have approximately equal efficacy. The best predictors of such a
response are a history of prior response to an agent and a family history of preferential response.

Comprehensive text book of psychiatry,8 the edition,p-1658


Incorrect
Marks for this submission: 0/1.
Question 3
Marks: 0/1

Classic and irreversible monoamine oxidase inhibitors (MAOIs) have been shown to be efficacious in
all of the following except

Choose one answer.


Error! Not a valid embedded object. a. Major depressive disorder
Error! Not a valid embedded object. b. Obsessive-compulsive disorder (OCD)
Error! Not a valid embedded object. c. "Atypical" depression
Error! Not a valid embedded object. d. Dysthymia
Error! Not a valid embedded object. e. Panic disorder
Studies clearly indicate that MAOIs can be effective in major depression, "atypical" depression, panic
disorder, and dysthymia. Although initial case reports suggested that MAOIs may be effective in OCD,
no double-blind studies have confirmed efficacy.

Maudsley guidelines 2005-2006,8 edition,p-140-1


Incorrect
Marks for this submission: 0/1.
Question 4
Marks: 0/1

The following treatment strategy would be the best choice for a patient with chronic symptoms of
depression with Alzheimer's disease

Choose one answer.


Error! Not a valid embedded object. a. discontinue antidementia drugs
Error! Not a valid embedded object. b. Haloperidol
Error! Not a valid embedded object. c. Citalopram
Error! Not a valid embedded object. d. imipramine
Error! Not a valid embedded object. e. maprotilline
Citalopram-treated patients with dementia complicated by depression scored significantly better than
placebo-treated patients on measures of orientation to time, recent memory, ability to increase tempo,
and fear-panic.

Nyth AL, Gottfries CG: The clinical efficacy of citalopram in treatment of emotional disturbances in
dementia disorders: a Nordic multicentre study. Br J Psychiatry 157:894–901, 1990
Incorrect
Marks for this submission: 0/1.
Question 5
Marks: 0/1

The first line of treatment for moderate -severe depression in the Uk is

Choose one answer.


Error! Not a valid embedded object. a. None of the above
Error! Not a valid embedded object. b. TCA's
Error! Not a valid embedded object. c. SSRI's
Error! Not a valid embedded object. d. Mirtrazepine
Error! Not a valid embedded object. e. Venlafaxine
SSRI's are as potent as TCA's and less likely to be discontinued

Nice guidelines for depression ,2004


Incorrect
Marks for this submission: 0/1.
Question 6
Marks: 0/1

The following are true about antidepressants in pregnancy and lactation except

Choose one answer.


Error! Not a valid a. SSRI's after 20 weeks gestation is associated with persistent
embedded object. pulmonary hypertension in neonates
Error! Not a valid
b. most TCA's have higher toxicity index than SSRI's
embedded object.
Error! Not a valid
c. all antidepressants have risk of withdrawal effects or toxicity
embedded object.
Error! Not a valid
d. paroxetine in I trimester is associated with foetal heart defects
embedded object.
Error! Not a valid e. In most cases the withdrawal effects in neonates are severe and
embedded object. persistent
In most cases the withdrawal effects are mild and self limiting

Nice guidelines for antenatal and postnatal mental health, April 2007
Incorrect
Marks for this submission: 0/1.
Question 7
Marks: 0/1
The following are recommended in the treatment of mild depression except

Choose one answer.


Error! Not a valid embedded object. a. computerised CBT
Error! Not a valid embedded object. b. exercise
Error! Not a valid embedded object. c. antidepressants
Error! Not a valid embedded object. d. guided self help
Error! Not a valid embedded object. e. individual CBT
Antidepressants are not recommended mild depression as the risk benefit ratio is poor

Nice guidelines for depression ,2004


Incorrect
Marks for this submission: 0/1.
Question 8
Marks: 0/1

The antidepressant of choice in patient with recent history of MI or unstable angina is

Choose one answer.


Error! Not a valid embedded object. a. Sertraline
Error! Not a valid embedded object. b. Venlafaxine
Error! Not a valid embedded object. c. Fluoxetine
Error! Not a valid embedded object. d. none of the above
Error! Not a valid embedded object. e. Mirtrazepine
Sertraline has the best evidence

Nice guidelines for depression ,2004


Incorrect
Marks for this submission: 0/1.
Question 9
Marks: 0/1

The following is false about treatment of depression with antidepressants and psychotherapy

Choose one answer.


Error! Not a valid
a. 55% patients responded to antidepressant alone
embedded object.
Error! Not a valid
b. 52% patients responded well to CBT alone
embedded object.
Error! Not a valid c. medication alone is effective in depression and preventing recurrences
embedded object.
Error! Not a valid d. recurrence rate was only 20% over 3 year period in patients with
embedded object. combined treatment with IPT and medication
Error! Not a valid e. 85% patients with combined CBT and medication had a positive
embedded object. response
Combined psychotherapy and antidepressant have a clear advantage in depression and preventing
recurrences

Synopsis of Psychiatry,9 the edition,p-969


Incorrect
Marks for this submission: 0/1.
Question 10
Marks: 0/1

The following drugs may produce a greater antidepressant response than selective serotonin reuptake
inhibitors (SSRIs)

Choose one answer.


Error! Not a valid embedded object. a. Mirtazapine
Error! Not a valid embedded object. b. Lithium
Error! Not a valid embedded object. c. Trazodone
Error! Not a valid embedded object. d. St. John‘s wort
Error! Not a valid embedded object. e. Venlafaxine
Results from meta-analyses of published and unpublished studies comparing venlafaxine with SSRIs
have provided further evidence that venlafaxine may produce a significantly greater antidepressant
response than fluoxetine and perhaps than SSRIs as a class (Einarson et al. 1999; Smith et al. 2002;
Thase et al. 2001).

Smith D, Dempster C, Glanville J, et al: Efficacy and tolerability of venlafaxine compared with
selective serotonin reuptake inhibitors and other antidepressants: a meta-analysis. Br J Psychiatry
180:396–404, 2002
Incorrect
Marks for this submission: 0/1.
Question 11
Marks: 0/1

The SSRI with least risk in pregnancy is

Choose one answer.


Error! Not a valid embedded object. a. citalopram
Error! Not a valid embedded object. b. Sertraline
Error! Not a valid embedded object. c. fluoxamine
Error! Not a valid embedded object. d. paroxetine
Error! Not a valid embedded object. e. Fluoxetine
Fluoxetine is the SSRI with lowest risk

Nice guidelines for antenatal and postnatal mental health, April 2007
Incorrect
Marks for this submission: 0/1.
Question 12
Marks: 0/1

The indications for sertraline include all except

Choose one answer.


Error! Not a valid embedded object. a. depression +anxiety
Error! Not a valid embedded object. b. OCD
Error! Not a valid embedded object. c. depression
Error! Not a valid embedded object. d. Bulimia
Error! Not a valid embedded object. e. PTSD in women
Fluoxetine has effectiveness in Bulimia

Maudsley guidelines 2005-2006,8 edition,p-138


Incorrect
Marks for this submission: 0/1.
Question 13
Marks: 0/1

The following is recommended by the NICE guidelines be considered in the initial treatment of severe
depression

Choose one answer.


Error! Not a valid embedded object. a. augmentation
Error! Not a valid embedded object. b. none of the above
Error! Not a valid embedded object. c. antidepressants alone
Error! Not a valid embedded object. d. antidepressants + individual CBT
Error! Not a valid embedded object. e. CBT alone
The combination of antidepressants and individual CBT should be considered in the initial management
as it is more cost effective than either on its own

Nice guidelines for depression ,2004


Incorrect
Marks for this submission: 0/1.
Question 14
Marks: 0/1

The best strategy after nonresponse to a first antidepressive agent is

Choose one answer.


Error! Not a valid embedded
a. Augment with thyroid hormone
object.
Error! Not a valid embedded
b. increase in dose above BNF limits
object.
Error! Not a valid embedded
c. Augment with lithium
object.
Error! Not a valid embedded d. Change to an alternative agent in a different chemical/mechanism
object. class
Error! Not a valid embedded
e. Augment with a stimulant
object.
In cases in which little or no response is seen, a new trial of an alternative agent should be considered.

Thase ME, Rush AJ, Howland RH, et al: Double-blind switch study of imipramine or sertraline
treatment of antidepressant-resistant chronic depression. Arch Gen Psychiatry 59: 233–239, 2002
Incorrect
Marks for this submission: 0/1.
Question 15
Marks: 0/1
Your Consultant suggests that you add Trazadone to an SSRI in the treatment of one of your
outpatients who is complaining of on-going symptoms. The symptom which trazadone is likely to help
with is

Choose one answer.


Error! Not a valid embedded object. a. memory difficulties
Error! Not a valid embedded object. b. fatigue
Error! Not a valid embedded object. c. sexual dysfunction
Error! Not a valid embedded object. d. insomnia
Error! Not a valid embedded object. e. appetite reduction
Trazadone is a phenylpiperazine and is a relatively weak serotonin reuptake inhibitor. One advantage it
has is that it is relatively safe in overdose. A side effect is that it causes sedation. It is this side effect
together with its antidepressant and anxioltic effects which make it an effective antidepressant for
people with insomnia. it can be usefully combined with SSRIs

Fundamentals of Clinical Psychopharmacology. Anderson IM & Reid IC. 2 edition,2004, p70


Incorrect
Marks for this submission: 0/1.
Question 16
Marks: 0/1

Moclobemide is licensed for

Choose one answer.


Error! Not a valid embedded object. a. OCD
Error! Not a valid embedded object. b. Social phobia
Error! Not a valid embedded object. c. Panic disorder
Error! Not a valid embedded object. d. PTSD
Error! Not a valid embedded object. e. none
Moclobemide is licensed for social phobia and depression

Maudsley guidelines 2005-2006,8 edition,p-138


Incorrect
Marks for this submission: 0/1.
Question 17
Marks: 0/1

If depression has not responded to 2 adequate trials of different antidepressants the following can be
considered

Choose one answer.


Error! Not a valid embedded object. a. Lithium augmentation
Error! Not a valid embedded object. b. All of the options stated
Error! Not a valid embedded object. c. None of the above
Error! Not a valid embedded object. d. Augmenting antidepressant with antidepressant
Error! Not a valid embedded object. e. Venlafaxine
All the above are options in the routine management of treatment resistant depression

Nice guidelines for depression ,2004


Incorrect
Marks for this submission: 0/1.
Question 18
Marks: 0/1

The most important factor affecting treatment response to antidepressant is

Choose one answer.


Error! Not a valid embedded object. a. common side effects
Error! Not a valid embedded object. b. non adherence
Error! Not a valid embedded object. c. Suicidal ideations
Error! Not a valid embedded object. d. Comorbid conditions
Error! Not a valid embedded object. e. Idiopathic side effects
The greatest threat to treatment response is nonadherence to an agent.

Comprehensive text book of psychiatry,8 the edition,p-1653


Incorrect
Marks for this submission: 0/1.
Question 19
Marks: 0/1

The following is true about the relation between antidepressants and suicide

Choose one answer.


Error! Not a valid a. Tricyclic antidepressants (TCAs) increase the risk of suicide only at
embedded object. doses three to five times the therapeutic dose
Error! Not a valid b. Bupropion is the safest of the second- and third-generation
embedded object. antidepressants when taken in overdose
Error! Not a valid
c. None of the above
embedded object.
Error! Not a valid
d. The phenomenon is likely not unique to any one antidepressant
embedded object.
Error! Not a valid
e. SSRIs have a narrow margin of safety
embedded object.
Meta-analyses (Kapur et al. 1992; Mann and Kapur 1991) and a later prospective study (Leon et al.
1999) showed that if any pro-suicide effect exists, it is extremely rare and not unique to any one
antidepressant.

Kapur S, Mieczkowski T, Mann JJ: Antidepressant medications and the relative risk of suicide attempt
and suicide. JAMA 268:3441–3445, 1992
Incorrect
Marks for this submission: 0/1.
Question 20
Marks: 0/1

Fluoxetine is licensed in the UK for all except

Choose one answer.


Error! Not a valid embedded object. a. Social phobia
Error! Not a valid embedded object. b. mixed anxiety depression
Error! Not a valid embedded object. c. Depression
Error! Not a valid embedded object. d. Bulimia nervosa
Error! Not a valid embedded object. e. OCD
Fluoxetine in not licensed for Social phobia

Maudsley guidelines 2005-2006,8 edition,p-138


Incorrect
Marks for this submission: 0/1.
Question 21
Marks: 0/1

Clomipramine is licensed in the UK for

Choose one answer.


Error! Not a valid embedded
a. All the options stated
object.
Error! Not a valid embedded
b. Depression
object.
Error! Not a valid embedded
c. Obsessional states
object.
Error! Not a valid embedded d. Adjunctive treatment of cataplexy associated with narcolepsy
object.
Error! Not a valid embedded
e. Phobias
object.
Clomipramine is used in all these

Maudsley guidelines 2005-2006,8 edition,p-136


Incorrect
Marks for this submission: 0/1.
Question 22
Marks: 0/1

The duration of maintainence treatment with antidepressants in patients with more than two depressive
episodes is

Choose one answer.


Error! Not a valid embedded object. a. 5 years
Error! Not a valid embedded object. b. none
Error! Not a valid embedded object. c. 1 year
Error! Not a valid embedded object. d. 6 months
Error! Not a valid embedded object. e. 2years
Patients with more than 2 depressive episodes should be advised to continue antidepressants for 2years

Nice guidelines for depression ,2004


Incorrect
Marks for this submission: 0/1.
Question 23
Marks: 0/1

Duloxetine may be beneficial in the following


Choose one answer.
Error! Not a valid embedded object. a. Depression
Error! Not a valid embedded object. b. None of the above
Error! Not a valid embedded object. c. Stress urinary incontinence
Error! Not a valid embedded object. d. All of the above
Error! Not a valid embedded object. e. Chronic pain
Duloxetine‘s efficacy in treating major depressive disorder has been demonstrated in six double-blind,
placebo-controlled or active-comparator studies (for a review, see Nemeroff et al. 2002). In addition,
duloxetine has been shown to benefit anxiety spectrum symptoms and chronic pain, producing
significant improvement on Ham-D17 Anxiety/Somatization subscale and Visual Analog Scale of pain
severity scores, respectively (Detke et al. 2002). Finally, initial results from an early evaluation of 553
women randomized to placebo or duloxetine indicated significant and dose-dependent decreases in
episode frequency of incontinence among duloxetine-treated women (Norton et al. 2002).

Nemeroff CB, Schatzberg AF, Goldstein DJ, et al: Duloxetine for the treatment of major depressive
disorder. Psychopharmacol Bull 36:106–132, 2002,Detke MJ, Lu Y, Goldstein DJ, et al: Duloxetine 60
mg once daily dosing versus placebo in the acute treatment of major depression. J Psychiatr Res
36:383–390, 2002 ,Norton P, Zinner N, Yalcin I, et al: Duloxetine Urinary Incontinence Study Group:
duloxetine versus placebo in the treatment of stress urinary incontinence. Am J Obstet Gynecol
187:40–48, 2002
Incorrect
Marks for this submission: 0/1.

Antipsychotics
1
Marks: 0/1

The next step in the pharmacological management of patient with Schizophrenia who has not
responded well to two different antipsychotics including one atypical antipsychotic is

Choose one answer.


Error! Not a valid embedded object. a. No medication
Error! Not a valid embedded object. b. Depot antipsychotics
Error! Not a valid embedded object. c. Clozapine
Error! Not a valid embedded object. d. Augmentation with lithium
Error! Not a valid embedded object. e. Combination antipsychotics
Clozapine is the treatment for treatment resistant schizophrenia which is non responsive to at least two
antipsychotics, of which one must have been an atypical.

NICE guidelines for the management of schizophrenia ,2002


Incorrect
Marks for this submission: 0/1.
Question 2
Marks: 0/1

All are true regarding choice of an antipsychotic for a patient with acute psychosis in schizophrenia
except
Choose one answer.
a. All of the second-generation (atypical) agents are more effective than the
Error! Not a valid
first-generation (typical) agents in the treatment of positive and negative
embedded object.
symptoms
Error! Not a valid b. It is acceptable to use a first-generation agent if a particular patient has
embedded object. responded to that agent in the past
Error! Not a valid c. The main advantage of second-generation agents is their overall lower risk
embedded object. of causing extra pyramidal symptoms and tardive dyskinesia
Error! Not a valid
d. Generally, second-generation (atypical) agents are preferred
embedded object.
Error! Not a valid e. Clozapine may be therapeutically superior to any other antipsychotic
embedded object. medications
I and II generation antipsychotics are equal in their efficacy

Nice guidelines for Schizophrenia,2002


Incorrect
Marks for this submission: 0/1.
Question 3
Marks: 0/1

The following treatments are helpful in reducing the risk of future relapses in Schizophrenia

Choose one answer.


Error! Not a valid embedded object. a. Antipsychotic medication
Error! Not a valid embedded object. b. Psychoeducational family therapy
Error! Not a valid embedded object. c. All the above
Error! Not a valid embedded object. d. Social skills training
Error! Not a valid embedded object. e. Group treatment involving families
All these are helpful in reducing he risk of future relapses in schizophrenia

Synopsis of Psychiatry,9 the edition,p-968-969


Incorrect
Marks for this submission: 0/1.
Question 4
Marks: 0/1

The antipsychotic that is effective for the treatment of nausea and intractable hiccups is

Choose one answer.


Error! Not a valid embedded object. a. Haloperidol
Error! Not a valid embedded object. b. Pimozide
Error! Not a valid embedded object. c. Molindone
Error! Not a valid embedded object. d. Perphenazine
Error! Not a valid embedded object. e. Chlorpromazine
The lower-potency antipsychotics exert a potent antiemetic effect through histamine1 receptor
antagonism.

BNF,9th edition,p-187-8
Incorrect
Marks for this submission: 0/1.
Question 5
Marks: 0/1

A patient with schizophrenia has been on 2 different atypical antipsychotics quetiapine and
aripiprazole. His concordance has been poor and he continues to be symptomatic. The next step in the
management is

Choose one answer.


Error! Not a valid embedded object. a. Change to another atypical
Error! Not a valid embedded object. b. Change to typical antipsychotic
Error! Not a valid embedded object. c. Psychological treatment
Error! Not a valid embedded object. d. Change to Clozapine
Error! Not a valid embedded object. e. Change to depot
In view of poor concordance the next helpful step is to consider depot.

Nice guidelines schizophrenia,2002


Incorrect
Marks for this submission: 0/1.
Question 6
Marks: 0/1

The following are true about Clozapine except

Choose one answer.


Error! Not a valid embedded object. a. very effective in actively and floridly psychotic
Error! Not a valid embedded object. b. reduces suicidality
Error! Not a valid embedded object. c. reduces aggression
Error! Not a valid embedded object. d. I line treatment of Schizophrenia
Error! Not a valid embedded object. e. effective in refractory cases
Clozapine is recommended after adequate trials of 2 different antipsychotics

Maudsley guidelines 2005-2006,8 edition,p-161


Incorrect
Marks for this submission: 0/1.
Question 7
Marks: 0/1

A patient is on Clozapine after adequate trials of conventional and atypical antipsychotics . But he
continues to be symptomatic , what is the next step in the treatment?

Choose one answer.


Error! Not a valid
a. Adding Lithium
embedded object.
Error! Not a valid
b. Stopping Clozapine
embedded object.
Error! Not a valid
c. Adding Omega 3
embedded object.
Error! Not a valid
d. Continue Clozapine montotherapy but with intermittent use of
embedded object.
another atypical antipsychotic
Error! Not a valid
e. Sulpiride can be added
embedded object.
In more difficult cases, augmentation of clozapine with benzamides (sulpiride, amisulpride) and anti-
epileptics (lamotrigine) shows some success. In extreme treatment resistance, a strategy is
recommended that combines the proven best drug for the particular patient and psychosocial
treatments.

Rob W. Kerwin and Anusha Bolonna, Management of clozapine-resistant schizophrenia , Advances in


Psychiatric Treatment (2005) 11: 101-106
Incorrect
Marks for this submission: 0/1.
Question 8
Marks: 0/1

The following second-generation antipsychotics is most effective for treatment-resistant schizophrenia

Choose one answer.


Error! Not a valid embedded object. a. Aripiprazole
Error! Not a valid embedded object. b. Quetiapine
Error! Not a valid embedded object. c. Risperidone
Error! Not a valid embedded object. d. Olanzapine
Error! Not a valid embedded object. e. Clozapine
Clozapine has superior efficacy to all other atypicals and licensed for refractory schizophrenia

Nice guidelines for Schizophrenia,2002


Incorrect
Marks for this submission: 0/1.
Question 9
Marks: 0/1

The following are options for augmenting clozapine except

Choose one answer.


Error! Not a valid embedded object. a. Pimozide
Error! Not a valid embedded object. b. Risperidone
Error! Not a valid embedded object. c. Omega 3 triglycerides
Error! Not a valid embedded object. d. Lamotrigine
Error! Not a valid embedded object. e. Sulpiride
Pimozide is not recommended due to cardiotoxicity

Maudsley guidelines 2005-2006,8 edition,p-49


Incorrect
Marks for this submission: 0/1.
Question 10
Marks: 0/1

The following medications may be useful in severe Tardive dyskinesia

Choose one answer.


Error! Not a valid embedded object. a. Clozapine
Error! Not a valid embedded object. b. l-Carnitine
Error! Not a valid embedded object. c. l-Serine
Error! Not a valid embedded object. d. Pimozide
Error! Not a valid embedded object. e. Haloperidol
Evidence suggests that clozapine, among the second-generation drugs, may be helpful in reducing
tardive dyskinesia symptoms

Maudsley guidelines 2005-2006,8 edition,p-78


Incorrect
Marks for this submission: 0/1.
Question 11
Marks: 0/1

Second-generation antipsychotics appear to demonstrate

Choose one answer.


Error! Not a valid embedded object. a. Improved memory
Error! Not a valid embedded object. b. All of the above
Error! Not a valid embedded object. c. Better verbal fluency
Error! Not a valid embedded object. d. Less weight gain than with first-generation antipsychotics
Error! Not a valid embedded object. e. Better overall clinical outcome
a meta-analysis of 15 studies indicated that second-generation antipsychotics have notable effects on
verbal fluency and executive function, with limited improvement in memory.

Bilder RM, Goldman RS, Volavka J, et al: Neurocognitive effects of clozapine, olanzapine,
risperidone, and haloperidol in patients with chronic schizophrenia or schizoaffective disorder. Am J
Psychiatry 159:1018–1028, 2002
Incorrect
Marks for this submission: 0/1.
Question 12
Marks: 0/1

After I episode of Schizophrenia antipsychotics should be continued for

Choose one answer.


Error! Not a valid embedded object. a. less than 6 months
Error! Not a valid embedded object. b. less than 1 yr
Error! Not a valid embedded object. c. none
Error! Not a valid embedded object. d. 1-2 years
Error! Not a valid embedded object. e. 6 months
Reccommended for 1-2 years

Maudsley guidelines 2005-2006,8 edition,p-44


Incorrect
Marks for this submission: 0/1.
Question 13
Marks: 0/1
After I episode of Schizophrenia antipsychotics should ideally be continued for

Choose one answer.


Error! Not a valid embedded object. a. 6 months
Error! Not a valid embedded object. b. less than 6 months
Error! Not a valid embedded object. c. less than 1 yr
Error! Not a valid embedded object. d. 1-2 years
Error! Not a valid embedded object. e. 3 months
The current recommendation is that antipsychotics should be continued for at least for 1-2 years after
the remission of the first episode.

Maudsley guidelines 2005-2006,8 edition,p-44


Incorrect
Marks for this submission: 0/1.
Question 14
Marks: 0/1

Antipsychotic which is best supported for use in patients with Tardive Dysknesia

Choose one answer.


Error! Not a valid embedded object. a. Clozapine
Error! Not a valid embedded object. b. Quetiapine
j
k
l
m
n c. Olanzapine
j
k
l
m
n d. Risperidone
j
k
l
m
n e. Aripiprazole
Clozapine has the best evidence

Maudsley guidelines 2005-2006,8 edition,p-78


Incorrect
Marks for this submission: 0/1.

Mood Stabilisers
1
Marks: 0/1

The following drugs have shown efficacy in the treatment of acute mania in except

Choose one answer.


Error! Not a valid embedded object. a. Carbamazepine
Error! Not a valid embedded object. b. Olanzepine
Error! Not a valid embedded object. c. Chlorpromazine
Error! Not a valid embedded object. d. Lamotrigine
Error! Not a valid embedded object. e. Semisodium Valproate
All except lamotrigine have been shown to be efficacious in treating acute mania . Lamotrigine may
help in depression associated with bipolar disorder .
Comprehensive text book of psychiatry,8 the edition,p-1679-80
Incorrect
Marks for this submission: 0/1.
Question 2
Marks: 0/1

Carbamazepine has been used in

Choose one answer.


Error! Not a valid embedded object. a. aggression
Error! Not a valid embedded object. b. All the above
Error! Not a valid embedded object. c. trigeminal neuralgia
Error! Not a valid embedded object. d. rapid cycling illness
Error! Not a valid embedded object. e. epilepsy
Carbamazepine has also bee used in alcohol withdrawal in high doses

Maudsley guidelines 2005-2006,8 edition,p-117


Incorrect
Marks for this submission: 0/1.
Question 3
Marks: 0/1

A patient develops nephrolithiasis after taking a mood stabilizer for treatment of their Bipolar disorder.
The drug most likely to have been used by this patient is

Choose one answer.


Error! Not a valid embedded object. a. Olanzapine
Error! Not a valid embedded object. b. Lithium
Error! Not a valid embedded object. c. Valproate
Error! Not a valid embedded object. d. Topiramate
Error! Not a valid embedded object. e. Carbamazepine
1.5% patients taking topiramate develop renal calculi. This is 10 times that associated with placebo.
The BNF recommends that patients on this medication are advised to maintain good hydration levels.

BNF, 52(2006)
Incorrect
Marks for this submission: 0/1.
Question 4
Marks: 0/1

Lithium is used in the prophylaxis and treatment of the following

Choose one answer.


Error! Not a valid embedded object. a. All the above
Error! Not a valid embedded object. b. Bipolar disorder
Error! Not a valid embedded object. c. hypomania
Error! Not a valid embedded object. d. mania
Error! Not a valid embedded object. e. recurrent depression
All these are indications for Lithium

Maudsley guidelines 2005-2006,8 edition,p-108


Incorrect
Marks for this submission: 0/1.
Question 5
Marks: 0/1

The following are true of Lithium except

Choose one answer.


Error! Not a valid embedded a. as effective as antidepressant in prophylaxis of recurrent
object. depression
Error! Not a valid embedded
b. reduces frequency and severity of relapses
object.
Error! Not a valid embedded
c. has utility as adjunct to antidepressant
object.
Error! Not a valid embedded
d. intermittent treatment may worsen the illness
object.
Error! Not a valid embedded
e. intermittent treatment has better outcome
object.
intermittent treatment may worsen the natural course of illness ,hence advised to use for 3 years

Maudsley guidelines 2005-2006,8 edition,p-111


Incorrect
Marks for this submission: 0/1.
Question 6
Marks: 0/1

Lithium is effective in

Choose one answer.


Error! Not a valid embedded
a. Treatment of obsessive-compulsive disorder
object.
Error! Not a valid embedded
b. Treatment of hyperthyroidism
object.
Error! Not a valid embedded
c. Antidepressant augmentation in unipolar depression
object.
Error! Not a valid embedded
d. Treatment of anorexia nervosa
object.
Error! Not a valid embedded e. Correction of syndrome of inappropriate antidiuretic hormone
object. secretion (SIADH
There has been evidence for lithium as augmenter to antidepressants

Comprehensive text book of psychiatry,8 the edition,p-3005


Incorrect
Marks for this submission: 0/1.
Question 7
Marks: 0/1
The following medications have been shown to have little to no efficacy in the treatment of acute mania
except

Choose one answer.


Error! Not a valid embedded object. a. Carbamazepine
Error! Not a valid embedded object. b. Clonazepam
Error! Not a valid embedded object. c. Lamotrigine
Error! Not a valid embedded object. d. Topiramate
Error! Not a valid embedded object. e. Gabapentin
carbamazepine is effective in acute mania

Maudsley guidelines 2005-2006,8 edition,p-122-3


Incorrect
Marks for this submission: 0/1.
Question 8
Marks: 0/1

All the following are recommended by NICE for the long term treatment of bipolar disorder except

Choose one answer.


Error! Not a valid embedded object. a. Depakote
Error! Not a valid embedded object. b. Olanzapine
Error! Not a valid embedded object. c. Lithium
Error! Not a valid embedded object. d. Risperidone
Error! Not a valid embedded object. e. Carbamazepine
NICE guidelines for Bipolar Disorder suggests :
l. Consider lithium, olanzapine or valproate for long-term treatment of bipolar disorder, depending on:
– response to previous treatments
– the relative risk, and precipitants, of manic versus depressive relapse
– physical risk factors, particularly renal disease, obesity and diabetes
– the patient‘s preference and history of adherence
– gender (valproate* should not normally be prescribed for women of child-bearing potential)
– a brief assessment of cognitive state if appropriate, for example, for older people.
lI. If the patient has frequent relapses, or continuing functional impairment:
– consider switching to a different prophylactic drug (lithium, olanzapine or valproate*)
– adding a second; possible combinations are lithium with valproate*, lithium with olanzapine,
valproate with olanzapine
– discuss with the patient (and document) the potential benefits and risks, and reasons for the
choice
– monitor closely clinical state, side effects and, where relevant, blood levels.
lII. If a combination of prophylactic agents proves ineffective, consider:
– consulting, or referring the patient to, a specialist
– prescribing lamotrigine* (especially if the patient has bipolar II disorder) or carbamazepine.

NICE guidance : Bipolar Disorder. July 2006.


Incorrect
Marks for this submission: 0/1.
Question 9
Marks: 0/1

The following mood stabilizer is not recommended in the prophylaxis of Bipolar affective disorder.
Choose one answer.
Error! Not a valid embedded object. a. Lithium
Error! Not a valid embedded object. b. Valproate
Error! Not a valid embedded object. c. Lamotrigine
Error! Not a valid embedded object. d. Topiramate
Error! Not a valid embedded object. e. Carbamazepine
Lithium ,Olanzepine,Valproate & Carbamazepine are recommended for prophylaxis of bipolar
disorders. Lamotrigine has been mentioned to have a role in Bipolar depression and its prophylaxis.
Topirimate is mentioned in the Maudsley guidelines (9th edn, pg 161) as having a role in acute mania.

NICE guidelines for bipolar disorders,2006


Incorrect
Marks for this submission: 0/1.
Question 10
Marks: 0/1

Lithium increases the level of which of the following

Choose one answer.


Error! Not a valid embedded object. a. Dopamine
Error! Not a valid embedded object. b. Norepinephrine
Error! Not a valid embedded object. c. none of the options
Error! Not a valid embedded object. d. Serotonin
Error! Not a valid embedded object. e. MAO
Lithium increases serotonin levels and can be one the factors that lead to serotonin syndrome in
patients who are treated with an antidepressant as well as lithium. Caution is advised.

Kaplan & saddock,Synopsis of Psychiatry,9th edition,p-567


Incorrect
Marks for this submission: 0/1.
Question 11
Marks: 0/1

All are true about lithium & semisodium valproate in the treatment of acute mania except

Choose one answer.


Error! Not a valid a. Both are efficacious for acute mania in comparison with placebo, but
embedded object. lithium is generally superior to divalproex in head-to-head trials
Error! Not a valid b. Semisodium Vaplroate works better in patients with previous multiple
embedded object. mood episodes
Error! Not a valid c. Both agents have better long-term efficacy in the treatment of acute mania
embedded object. in combination with other medications
Error! Not a valid d. Both lithium and semi sodium valproate can help with psychotic
embedded object. symptoms in acute mania
Error! Not a valid e. Semisodium Valproate works better in patients with depressive symptoms
embedded object.
Semisodium Valproate has superior efficacy compared with placebo and comparable efficacy
compared with lithium.
Maudsley guidelines ,2005-6,8 edition,p-108,Bowden CL, Brugger AM, Swann AC, et al: Efficacy of
divalproex vs lithium and placebo in the treatment of mania. JAMA 271:918–924, 1994
Incorrect
Marks for this submission: 0/1.
Question 12
Marks: 0/1

The choice of mood stabilizer, as recommended by NICE, in the long term management in a person
who has about 4 relapses in 1 year and has not responded well to monotherapy with lithium alone is

Choose one answer.


Error! Not a valid embedded object. a. Lithium and Valproate
Error! Not a valid embedded object. b. Lithium and fish oils
Error! Not a valid embedded object. c. Valproate
Error! Not a valid embedded object. d. Lamotrigine
Error! Not a valid embedded object. e. Carbamazepine
In rapid cyclers the first line of treatment suggested is a combination of Lithium and Valproate

NICE guidelines for Bipolar disorders,2006


Incorrect
Marks for this submission: 0/1.

Other Treatments
1
Marks: 0/1

Patients with obsessive-compulsive disorder (OCD), require optimal treatment with paroxetine
typically at

Choose one answer.


Error! Not a valid embedded object. a. None of the above
Error! Not a valid embedded object. b. 20 mg/day, 2–4 weeks until response
Error! Not a valid embedded object. c. 60 mg/day, 3–12 weeks until response
Error! Not a valid embedded object. d. 40 mg/day, less than 2 weeks until response
Error! Not a valid embedded object. e. 10 mg/day, 3–12 weeks until response
Typically, total daily doses of greater than or equal to 60 mg of paroxetine are required to optimally
treat OCD. Although patients characteristically respond to treatment within 3–4 weeks, clinical
improvement may not be discernible until 10–12 weeks; therefore, a standard drug trial of up to 12
weeks should be conducted before an alternative medication is considered

Rasmussen SA, Eisen JL, Pato MT: Current issues in the pharmacologic management of obsessive
compulsive disorder. J Clin Psychiatry 54 (suppl 6):4–9, 1993 [Erratum appears in J Clin Psychiatry
54(12):491, 1993]
Incorrect
Marks for this submission: 0/1.
Question 2
Marks: 0/1
The following augmentation strategies has been found to be effective in treatment-resistant OCD

Choose one answer.


Error! Not a valid embedded object. a. Addition of risperidone to a SSRI
Error! Not a valid embedded object. b. Addition of venlafaxine to buspirone
Error! Not a valid embedded object. c. Addition of lithium to risperidone
Error! Not a valid embedded object. d. Addition of valproate to clomipramine
Error! Not a valid embedded object. e. Addition of gabapentin to an SSRI
The following augmentation and/or combination strategies have been reported as offering benefit.
Olanzapine and risperidone have been added successfully in cases of partial SSRI response .

Saxena S, Wang D, Bystritsky A, et al: Risperidone augmentation of SRI treatment for refractory
obsessive-compulsive disorder. J Clin Psychiatry 57:303–306, 1996
Incorrect
Marks for this submission: 0/1.
Question 3
Marks: 0/1

Exposure treatment is useful in

Choose one answer.


Error! Not a valid embedded object. a. Social Phobia
Error! Not a valid embedded object. b. All the above
Error! Not a valid embedded object. c. Agoraphobia
Error! Not a valid embedded object. d. OCD
Error! Not a valid embedded object. e. Specific phobia
Exposure treatment is used in all these

Seminars in Psychotherapies,2007,P - 152


Incorrect
Marks for this submission: 0/1.
Question 4
Marks: 0/1

The tricyclic antidepressant (TCAs) which is most effective in the treatment of obsessive-compulsive
disorder (OCD)

Choose one answer.


Error! Not a valid embedded object. a. Amitriptyline
Error! Not a valid embedded object. b. Desipramine
Error! Not a valid embedded object. c. Doxepin
Error! Not a valid embedded object. d. Imipramine
Error! Not a valid embedded object. e. Clomipramine
Unlike depression, which responds to a variety of antidepressant agents, OCD appears to require
treatment with a serotonergic agent. Clomipramine, the most serotonergic of the TCAs is considered as
an effective treatment of OCD

Greist JH, Jefferson JW, Kobak KA, et al: Efficacy and tolerability of serotonin transport inhibitors in
obsessive-compulsive disorder: a meta-analysis. Arch Gen Psychiatry 52:53–60, 1995
Incorrect
Marks for this submission: 0/1.
Question 5
Marks: 0/1

The following agent is considered the first-line treatment for periodic limb movements in sleep (PLMS)
and restless legs syndrome (RLS)

Choose one answer.


Error! Not a valid embedded object. a. Lorazepam
Error! Not a valid embedded object. b. Selective serotonin reuptake inhibitors
Error! Not a valid embedded object. c. Zolpidem
Error! Not a valid embedded object. d. Clomipramine
Error! Not a valid embedded object. e. Ropinirole
Dopaminergic agents are currently used as first-line treatment of PLMS and RLS.Newer-generation
dopaminergic agents such as pramipexole and ropinirole also have been used successfully.

Hening W, Allen R, Earley C, et al: The treatment of restless legs syndrome and periodic limb
movement disorder: an American Academy of Sleep Medicine review. Sleep 22:970–999, 1999
Incorrect
Marks for this submission: 0/1.
Question 6
Marks: 0/1

The first line of management in a patient with Simple Phobia is

Choose one answer.


Error! Not a valid embedded object. a. Sensitisation treatment
Error! Not a valid embedded object. b. Flooding
Error! Not a valid embedded object. c. No treatment
Error! Not a valid embedded object. d. Graded exposure therapy
Error! Not a valid embedded object. e. Antidepressant medication
The Approach to simple phobia is a graded exposure therapy. This was first described by Van Os in a
series of patients who responded well to animal phobias.

American Psychiatric press Text book of Psychiatry ,3 rd Edn.p-1191


Incorrect
Marks for this submission: 0/1.
Question 7
Marks: 0/1

The following is true about treatment in OCD

Choose one answer.


Error! Not a valid a. Behaviour therapy alone is less effective than combined medication
embedded object. and psychological treatment
Error! Not a valid
b. SSRI's alone have a better outcome than combined treatment
embedded object.
Error! Not a valid
c. Clomipramine is preferred than SSRI's due to side effects
embedded object.
Error! Not a valid d. Behaviour therapy alone is as effective as combined medication and
embedded object. psychological treatment
Combining treatments appears to yield a better outcome than medication alone but not behaviour
therapy alone .

Synopsis of Psychiatry,9 the edition,p-970


Incorrect
Marks for this submission: 0/1.
Question 8
Marks: 0/1

Cognitive Analytic therapy may be useful in all except

Choose one answer.


Error! Not a valid embedded object. a. Substance misusers'
Error! Not a valid embedded object. b. Personality disorders
Error! Not a valid embedded object. c. Psychoses
Error! Not a valid embedded object. d. Improving concordance
Error! Not a valid embedded object. e. Neurotic Disorders
There is no evidence for use of CAT in the Psychoses

Seminars in Psychotherapies,2007,P - 136


Incorrect
Marks for this submission: 0/1.
Question 9
Marks: 0/1

The following treatments may relieve high-altitude insomnia

Choose one answer.


Error! Not a valid embedded object. a. None of the above
Error! Not a valid embedded object. b. Benzodiazepines
Error! Not a valid embedded object. c. Shifting of the sleep-cycle schedule
Error! Not a valid embedded object. d. Tricyclic antidepressants
Error! Not a valid embedded object. e. Acetazolamide
Acetazolamide (250 mg twice daily) can prevent or reduce symptoms of acute mountain sickness and
improve sleep at high altitudes

Roberts MJ: Acute mountain sickness—experience on the Roof of Africa expedition and military
implications. J R Army Med Corps 140:49–51, 1994
Incorrect
Marks for this submission: 0/1.
Question 10
Marks: 0/1

The following specific treatment can be used in the prevention of dementia in some one with mild
cognitive impairment

Choose one answer.


Error! Not a valid embedded object. a. HRT
Error! Not a valid embedded object. b. Vit E
Error! Not a valid embedded object. c. Aspirin
Error! Not a valid embedded object. d. None of the options
Error! Not a valid embedded object. e. Statins
There is no specific treatment for primary prevention of dementia.

Nice guidelines for dementia, Nov 2006


Incorrect
Marks for this submission: 0/1.

1
Marks: 0/1
The following feature is not associated with Retts syndrome
Choose one answer.
Error! Not a valid embedded object. a. Midline hand wringing or hand washing stereotypies are seen
Error! Not a valid embedded object. b. The child develops normally until 18 months
Error! Not a valid embedded object. c. Non progressive
Error! Not a valid embedded object. d. Developmental delays are seen early in the course of the illness
Error! Not a valid embedded object. e. Purposeful hand movements are lost
Retts synd is seen in girls, initially with normal development followed by regression and death by 30 yrs
Goodman & Scott, Child Psychiatry,2002,p-45
Incorrect
Marks for this submission: 0/1.
Question 2
Marks: 0/1
What is the best predictor of future sexual offences?
Choose one answer.
Error! Not a valid embedded object. a. Previous non-sexual offences
Error! Not a valid embedded object. b. Lack of victim empathy
Error! Not a valid embedded object. c. History of illicit drug use
Error! Not a valid embedded object. d. Mental illness
Error! Not a valid embedded object. e. sexual deviance
A meta-analysis of 82 recidivism studies (1,620 findings from 29,450 sexual offenders) identified deviant sexual
preferences and antisocial orientation as the major predictors of sexual recidivism for both adult and
adolescent sexual offenders. Many of the variables commonly addressed in sex offender treatment programs
(e.g., psychological distress, denial of sex crime, victim empathy, stated motivation for treatment) had little or
no relationship with sexual or violent recidivism. RK Hanson, Morton-Bourgon KE.. The characteristics of
persistent sexual offenders: a meta-analysis of recidivism studies. JOURNAL OF CONSULTING AND CLINICAL
PSYCHOLOGY, 2005 Dec;73(6):1154-63.
Incorrect
Marks for this submission: 0/1.
Question 3
Marks: 0/1
A 27 year old man has been diagnosed with simple schizophrenia . One of the following is not a feature of this
condition
Choose one answer.
Error! Not a valid embedded object. a. progressive deterioration
Error! Not a valid embedded object. b. Blunting of affect is seen
Error! Not a valid embedded object. c. increasing eccentricity
Error! Not a valid embedded object. d. Auditory hallucinations are less prominent
Error! Not a valid embedded object. e. Overt psychotic symptoms seen
In simple schizophrenia overt psychotic symptoms are absent. A disorder in which there is an insidious but
progressive development of oddities of conduct, inability to meet the demands of society, and decline in total
performance. The characteristic negative features of residual schizophrenia (e.g. blunting of affect and loss of
volition) develop without being preceded by any overt psychotic symptoms. It is a disorder in which there is an
insidious but progressive development of oddities of conduct, inability to meet the demands of society, and
decline in total performance. The characteristic negative features of residual schizophrenia (e.g. blunting of
affect and loss of volition) develop without being preceded by any overt psychotic symptoms.
Companion to psychiatric studies,6 the edn,p-378
Incorrect
Marks for this submission: 0/1.
Question 4
Marks: 0/1
The following is true regarding offending behaviour in patients with learning disabilities
Choose one answer.
Error! Not a valid embedded object. a. Fire setting is the most common offense
Error! Not a valid embedded object. b. People with LD are more likely to commit violent offences
Error! Not a valid embedded object. c. It is overrepresented by borderline and mild LD.
Error! Not a valid embedded object. d. Reoffending is less
Error! Not a valid embedded object. e. prevalence of learning disability in prisons was about 40%
Offending in LD is overrepresented in mild and borderline groups, they are more likely to reoffend and arson is
less common in LD offenders. The prevalence is about 9.4%
Hayes et al,The prevalence of intellectual disability in a major UK prison, British Journal of Learning Disabilities
Volume 35 Issue 3, Pages 162 - 167
Incorrect
Marks for this submission: 0/1.
Question 5
Marks: 0/1
A 25 yr old lady with Bipolar affective disorder who is planning pregnancy is worried about the teratogenic
effects of Lithium . The risk of Epstein's anomaly with Lithium is
Choose one answer.
Error! Not a valid embedded object. a. 1 in 30000
Error! Not a valid embedded object. b. 1 in 100
Error! Not a valid embedded object. c. 1 in 10000
Error! Not a valid embedded object. d. 1 in 1000
Error! Not a valid embedded object. e. 1 in 20000
The risk of cardiac abnormality in a pregnant woman on Lithium in the first trimester is 60/1000 (compared to 8
/ 1000 in the general population). The risk of Ebsteins anomaly is increased from 1 in 20,000 in the general
population to 10 in 20,000 in women taking Lithium.
NICE guidelines on Antenatal & Postnatal Mental Health 2007.
Incorrect
Marks for this submission: 0/1.
Question 6
Marks: 0/1
Tolerance is less likely to develop in the following signs / symptoms in opiate dependence syndrome
Choose one answer.
Error! Not a valid embedded object. a. Respiratory depression
Error! Not a valid embedded object. b. Euphoria
Error! Not a valid embedded object. c. Miosis
Error! Not a valid embedded object. d. Rhinorrhea
Error! Not a valid embedded object. e. Sedation
There is minimal or no tolerance to miosis, constipation and seizures.
Click here for reference
Incorrect
Marks for this submission: 0/1.
Question 7
Marks: 0/1
Empathy Skills are most likely to be delayed in
Choose one answer.
Error! Not a valid embedded object. a. A deaf child of deaf parents
Error! Not a valid embedded object. b. A hearing child of deaf parents
Error! Not a valid embedded object. c. A deaf child of hearing parents
Error! Not a valid embedded object. d. A hearing child adopted for 6 months by deaf parents
Error! Not a valid embedded object. e. A hearing child with one deaf parent
The role of Language acquisition in the development of Theory of mind can be deciphered by studying deaf
children with deaf parents and deaf children with hearing parents. Deaf parents are more sensitive to
communicative efforts of their infants, and thus help in early development of language compared to deaf
children with hearing parents. Deaf children outperform both deaf children of hearing parents and hearing
children on false-belief tasks, probably due to early exposure to visual perspective-taking (sign-language) and
develop theory of mind earlier. This shows that language has an important role to play in acquisition of Theory
of mind.
Candida C. Peterson & Michael Siegal ,Insights into Theory of Mind from Deafness and Autism, Mind &
Language
Volume 15 Issue 1, Pages 123 - 145. AND Schick B., et . Language and Theory of Mind: A Study of Deaf Children.
Child Development, March/April 2007, Volume 78, Number 2, Pages 376 - 396.
Incorrect
Marks for this submission: 0/1.
Question 8
Marks: 0/1
A 33 year old man with resistant schizophrenia on Clozapine (plasma levels of >0.80mg / L) develops a seizure.
The next step in his treatment would be
Choose one answer.
Error! Not a valid embedded object. a. Reduce the dose of Clozapine
Error! Not a valid embedded object. b. Add lamotrigine
Error! Not a valid embedded object. c. Add Sodium Valproate
Error! Not a valid embedded object. d. Add Carbamazepine
Error! Not a valid embedded object. e. Discontinue Clozapine
The recommended plasma level of Clozapine is 0.35 - 0.50 mg / L. In this patient, the plasma level of clozapine
is higher than this, so with the onset of the first seizure the first step would be to reduce the dose of Clozapine.
With the onset of second seizure, one would need to consider adding a anticonvulsant, preferably Sodium
valproate.
Wong J, Delva N. Clozapine-induced seizures: recognition and treatment. Can J Psychiatry. 2007 Jul;52(7):457-
63. The Maudsley prescribing guidelines. 9th Edition, 2007. Page no. 71.
Incorrect
Marks for this submission: 0/1.
Question 9
Marks: 0/1
The risk of developing schizophrenia in a child when both parents suffer from the illness is
Choose one answer.
Error! Not a valid embedded object. a. 36%
Error! Not a valid embedded object. b. 4.40%
Error! Not a valid embedded object. c. 26%
Error! Not a valid embedded object. d. 13.80%
Error! Not a valid embedded object. e. 12.30%
The risk of schizophrenia in children with both parents having schizophrenia is 36.6%
Shorter text book of psychiatry,4 edn,p-348
Incorrect
Marks for this submission: 0/1.
Question 10
Marks: 0/1
Which of the following is used as first line in the maintenance treatment for opiates
Choose one answer.
Error! Not a valid embedded object. a. Naltrexone
Error! Not a valid embedded object. b. Codeine
Error! Not a valid embedded object. c. Benzodiazepines
Error! Not a valid embedded object. d. Buprenorphine
Error! Not a valid embedded object. e. Oromorphine
Buprenorphine and methadone have been recommended as I line in the maintainence treatment of opiate
dependence. Naltrexone is also used in the maintainence treatment but to prevent relapse ,not as first line.
Nice guidelines - Drug Misuse - Methadone and Buprenorphine for managing opioid dependence, 2007
Incorrect
Marks for this submission: 0/1.
Question 11
Marks: 0/1
A child is diagnosed with infantile autism . The following is not a feature of this condition
Choose one answer.
Error! Not a valid embedded object. a. communication impairment
Error! Not a valid embedded object. b. late onset
Error! Not a valid embedded object. c. social impairment
Error! Not a valid embedded object. d. early onset
Error! Not a valid embedded object. e. restricted interests
Infantile autism is defined by the combination of four sets of features .
Child Psychiatry,Goodman and scott,2002,p-41
Incorrect
Marks for this submission: 0/1.
Question 12
Marks: 0/1
A 25 year old lady is diagnosed with pre menstrual dysphoria. The following has the best evidence of use in pre-
menstrual dysphoria
Choose one answer.
Error! Not a valid embedded object. a. Oil of evening primrose
Error! Not a valid embedded object. b. Bright light therapy
Error! Not a valid embedded object. c. Vitamin B6
Error! Not a valid embedded object. d. SSRIs
Error! Not a valid embedded object. e. Progesterone
The efficacy of SSRIs in the treatment of both physical and psychological symptoms of premenstrual syndrome
(Dimmock et al, 2000) has been confirmed and studies have shown that these drugs also reduce functional
impairment. The onset of improvement is more rapid than that in SSRI treatment of depressive disorder.
Carol A. Henshaw , PMS: diagnosis, aetiology, assessment and management ,Advances in Psychiatric Treatment
(2007) 13: 139-146.
Incorrect
Marks for this submission: 0/1.
Question 13
Marks: 0/1
The e most common reason for self harm in adolescents is
Choose one answer.
Error! Not a valid embedded object. a. Eating disorder
Error! Not a valid embedded object. b. Schoolwork
Error! Not a valid embedded object. c. Consequences of childhood sexual abuse
Error! Not a valid embedded object. d. Relationship problems with family
Error! Not a valid embedded object. e. Bereavement
The most common problem facing the young people involved relationship difficulties within the family,
followed by relationship with friends which was more common in girls.
Keith Hawton and Louise Harris, Deliberate self-harm by under-15-year-olds: characteristics, trends and
outcome, The journal of child psychiatry and psychology, Volume 49, Issue 4, Pages 441-448
Incorrect
Marks for this submission: 0/1.
Question 14
Marks: 0/1
You see a 8 year old child with hyperkinetic disorder and no other comorbidities. The first line of treatment
that you would consider taking into account the current evidence base is
Choose one answer.
Error! Not a valid embedded object. a. Atomoxetine
Error! Not a valid embedded object. b. Dexamphetamine
Error! Not a valid embedded object. c. Methylphenidate
Error! Not a valid embedded object. d. Imipramine
Error! Not a valid embedded object. e. Clonidine
NICE guideline recommend the use of methylphenidate as a first line medical treatment for ADHD with no
other co-morbidities. Some patients who do not respond to methylphenidate will respond to dexamphetamine
and vice versa. Dexamphetamine is licensed for use in children over 3 years for narcolepsy and as adjunct in the
management of refractory hyperkinetic states in children (under specialist supervision).
Nice guidelines for ADHD,2008
Incorrect
Marks for this submission: 0/1.
Question 15
Marks: 0/1
In psychodynamic psychotherapy the outcome is improved by the following modes of action except
Choose one answer.
Error! Not a valid embedded object. a. affirmation process
Error! Not a valid embedded object. b. judicious self disclosure
Error! Not a valid embedded object. c. fostering insight
Error! Not a valid embedded object. d. therapist keeps track of patients' daily activities
Error! Not a valid embedded object. e. therapeutic relationship
Fostering of insight and the therapeutic relationship are probably the primary mods and self disclosure and
affirmation are secondary strategies in psychodynamic psychotherapy
Oxford text book of psychotherapy,2007,p-10-11
Incorrect
Marks for this submission: 0/1.
Question 16
Marks: 0/1
A subcultural explanation rather than a biological one in learning disabilities is most accounted for by
Choose one answer.
Error! Not a valid embedded
a. by higher prevalence of LD in Asians
object.
Error! Not a valid embedded
b. Facial dysmorphologies
object.
Error! Not a valid embedded c. An even spread of LD across different socio-economic groups of the
object. population
Error! Not a valid embedded
d. LD in other members of the family
object.
Error! Not a valid embedded
e. Moderate LD
object.
Studies have reported higher prevalence of LD in the south Asian community than the rest. Cultural
competence in the delivery of care is being proposed to deal with this.
Hara,Learning disabilities and ethnicity: achieving cultural competence, Advances in Psychiatric Treatment,
2003,vol 9 166-176
Incorrect
Marks for this submission: 0/1.
Question 17
Marks: 0/1
A 36 year old man has low mood, anxiety, sleep disturbance and decreased energy. He has been treated with
Reboxitine 4mg BD without much improvement. On your assessment, you diagnose him to have Obsessive
compulsive disorder. The most appropriate next step in treatment would be
Choose one answer.
Error! Not a valid embedded object. a. Citalopram 20 mg
Error! Not a valid embedded object. b. Sertraline 150 mg
Error! Not a valid embedded object. c. Venlafaxine 75mg
Error! Not a valid embedded object. d. Mirtazepine 15mg
Error! Not a valid embedded object. e. Amitryptiline 150 mg
NICE guidelines on management of OCD have advocated that 'For adults with OCD, the initial pharmacological
treatment should be
one of the following SSRIs: fluoxetine, fluvoxamine, paroxetine, sertraline or citalopram'. They have advocated
that TCAs, SNRIs, Mirtazepine, antipsychotics as monotherapy should be avoided.
NICE Guidelines: Obsessive compulsive disorder, 2005
Incorrect
Marks for this submission: 0/1.
Question 18
Marks: 0/1
A 35 year old man with LD has been treated with Carbamazepine for localised seizures with secondary
generalisation. The following electrolyte abnormality is a side effect of Carbamazepine
Choose one answer.
Error! Not a valid embedded object. a. Hypernatremia
Error! Not a valid embedded object. b. Hypokalemia
Error! Not a valid embedded object. c. Hyperkalemia
Error! Not a valid embedded object. d. Hyponatremia
Error! Not a valid embedded object. e. Hyperchloremia
Hyponatremia is seen in 5-40% patients on Carbamazepine , more so with higher serum levels and older
individuals
New Oxford textbook of psychiatry,Vol2,p-1328
Incorrect
Marks for this submission: 0/1.
Question 19
Marks: 0/1
A 35 year old patient with learning disability complains of swallowing problem. This is a likely side effect of
Choose one answer.
Error! Not a valid embedded object. a. Naltrexone
Error! Not a valid embedded object. b. Sodium Valproate
Error! Not a valid embedded object. c. Lithium
Error! Not a valid embedded object. d. Clonazepam
Error! Not a valid embedded object. e. Fluoxetine
Dysphagia can be related to pancreatitis which is associated with Valproate.
Hinder & Perry, Sodium-Valproate-Induced Pancreatitis in a Man with Profound Intellectual Disability: the
Significance of Diagnostic Difficulties, Journal of Applied Research in Intellectual Disabilities, Volume 13, Issue 4,
Pages 292-297
Incorrect
Marks for this submission: 0/1.
Question 20
Marks: 0/1
Visual field defects are seen with which of the following medications?
Choose one answer.
Error! Not a valid embedded object. a. Levitiracetam
Error! Not a valid embedded object. b. Vigabatrin
Error! Not a valid embedded object. c. Sodium valproate
Error! Not a valid embedded object. d. Gabapentine
Error! Not a valid embedded object. e. Lamotigine
Vigabatrin is associated with bilateral visual field defects.
Wilton et al, Visual field defect associated with vigabatrin: observational cohort study ,BMJ 1999;319:1165-
1166 ( 30 October )
Incorrect
Marks for this submission: 0/1.
Question 21
Marks: 0/1
The following is not seen in a baby born to a 25 year old mother who has anorexia nervosa
Choose one answer.
Error! Not a valid embedded object. a. increased head circumference
Error! Not a valid embedded object. b. microcephaly
Error! Not a valid embedded object. c. lower birth weight
Error! Not a valid embedded object. d. Cleft palate
Error! Not a valid embedded object. e. small for gestational age
Pregnant women with past or active eating disorders are at greater risk of delivering infants with lower birth
weight, smaller head circumference, microcephaly, and small for gestational age babies.
Kouba et al, Pregnancy and Neonatal Outcomes in Women With Eating Disorders ,Obstetrics & Gynaecology
2005;105:255-260.
Incorrect
Marks for this submission: 0/1.
Question 22
Marks: 0/1
Exhibitionism is
Choose one answer.
Error! Not a valid embedded object. a. Included only in ICD 10
Error! Not a valid embedded object. b. Not included in both ICD and DSM
Error! Not a valid embedded object. c. Included in DSM and ICD
Error! Not a valid embedded object. d. Not a diagnosis
Error! Not a valid embedded object. e. Included only in DSM IV
Exhibitionism is included in both ICD 10 and DSM IV and is characterised by recurrent compulsive urges to
expose their genitals to another person
Oxford New Text book of Psychiatry, vol 1 p-901
Incorrect
Marks for this submission: 0/1.
Question 23
Marks: 0/1
The antidepressant of choice in treating moderate depression in children is
Choose one answer.
Error! Not a valid embedded object. a. Sertraline
Error! Not a valid embedded object. b. Fluoxetine
Error! Not a valid embedded object. c. Paroxetine
Error! Not a valid embedded object. d. Mirtrazepine
Error! Not a valid embedded object. e. Citalopram
Nice guidelines suggest that fluoxetine should be considered the first line in the treatment of depression in
children and young people.
Depression in Children & Young People. NICE guidleines, 2005.
Click here for reference
Incorrect
Marks for this submission: 0/1.
Question 24
Marks: 0/1
The herbal remedy for depression with most evidence in treatment of Major depression is
Choose one answer.
Error! Not a valid embedded object. a. Omega 3 fish oil
Error! Not a valid embedded object. b. Gingko Biloba
Error! Not a valid embedded object. c. Panax ginseng
Error! Not a valid embedded object. d. claviceps purpurea
Error! Not a valid embedded object. e. Hypericum perforatum
A recent Cochrane review suggests that the hypericum extracts tested in the included trials a) are superior to
placebo in patients with major depression; b) are similarly effective as standard antidepressants; c) and have
fewer side effects than standard antidepressants. The association of country of origin and precision with effects
sizes complicates the interpretation.
Linde K et al. St John's wort for Major depression. Cochrane Database of Systematic Reviews, Issue 4, 2008
Incorrect
Marks for this submission: 0/1.
Question 25
Marks: 0/1
Neuroimaging studies in patients with OCD show
Choose one answer.
Error! Not a valid embedded object. a. Reduced metabolism in caudate nuclei
Error! Not a valid embedded object. b. Hypermetabolism in left orbital gyrus and both caudate nuclei
Error! Not a valid embedded object. c. Asymmetries in blood flow in limbic system
Error! Not a valid embedded object. d. Hypermetabolism of parahippocampal gyrus
Error! Not a valid embedded object. e. Reduced metabolism in left orbital gyrus
PET studies have shown significant increases in metabolism in left orbital gyrus and both caudate nuclei which
also improved with drug treatment. OCD symptoms are mediated by hyperactivity in orbitofrontal-subcortical
circuits, perhaps due to an imbalance of tone between direct and indirect striato-pallidal pathways.
Organic Psychiatry,Lishman,3 edn,p-145
Incorrect
Marks for this submission: 0/1.
Question 26
Marks: 0/1
In Huntington's disease the mortality by suicide is increased by
Choose one answer.
Error! Not a valid embedded object. a. 2-4 times
Error! Not a valid embedded object. b. 25 times
Error! Not a valid embedded object. c. 30 times
Error! Not a valid embedded object. d. 10 times
Error! Not a valid embedded object. e. 20 times
It is well established that psychiatric symptoms are common in Huntingtons disease . In fact, completed suicide
has been reported to be as high as 13% in Huntingtons disease , reflecting a seven- to 12-fold increase from the
rate in the general population.
Paulsen et al,Critical Periods of Suicide Risk in Huntingtons Disease ,Am J Psychiatry 162:725-731, April 2005
Incorrect
Marks for this submission: 0/1.
Question 27
Marks: 0/1
The following psychotropic medication to be avoided in someone with renal impairment is
Choose one answer.
Error! Not a valid embedded object. a. Haloperidol
Error! Not a valid embedded object. b. Clozapine
Error! Not a valid embedded object. c. Amisulpiride
Error! Not a valid embedded object. d. Olanzapine
Error! Not a valid embedded object. e. Quetiapine
Amisulpiride is mainly excreted by the kidneys and has minor hepatic metabolism. The Maudsley guidelines
have advocated against the use of Sulpiride and Amisulpiride in some one with renal impairment. They advise
to use Haloperidol (if typical antipsychotics are to be used) and Olanzapine (Atypical Antipsychotics are to be
used).
The Maudsley prescribing guidelines. 9th Edition, 2007. Page no. 399. Bergemann et al. Plasma amisulpride
levels in schizophrenia or schizoaffective disorder ,European Neuropsychopharmacology Volume 14, Issue 3,
May 2004, Pages 245-250
Incorrect
Marks for this submission: 0/1.
Question 28
Marks: 0/1
The following antipsychotic when used prophylactically reduces the occurrence of post operative delirium
Choose one answer.
Error! Not a valid embedded object. a. Trifluoperazine
Error! Not a valid embedded object. b. Olanzapine
Error! Not a valid embedded object. c. Risperidone
Error! Not a valid embedded object. d. Quetiapine
Error! Not a valid embedded object. e. Haloperidol
Haloperidol used prophylactically has been associated with reduction in post operative delirium
Kaneko et al, Prophylactic administration of Haloperidol can reduce the occurrence of post operative delirium
in gastrointestinal surgery,Yonaga acta medica,1999;42;179-184
Incorrect
Marks for this submission: 0/1.
Question 29
Marks: 0/1
The following is most strongly associated with infanticide
Choose one answer.
Error! Not a valid embedded object. a. Maternal suicide risk
Error! Not a valid embedded object. b. Behavioural problems in other siblings
Error! Not a valid embedded object. c. Paternal suicide risk
Error! Not a valid embedded object. d. Paternal mental health problems
Error! Not a valid embedded object. e. Battering mother
Among infants aged over one day and under one year, the younger the child the more likely its death will be as
a consequence of a parent having lost his or her temper with some aspect of the infant's behaviour - 'battering
mother'.
Maureen Marks. Infanticide. Psychiatry Volume 5, Issue 1, 1 January 2006, Pages 13-15
Incorrect
Marks for this submission: 0/1.
Question 30
Marks: 0/1
The ICD 10 diagnosis for conversion disorder does not include
Choose one answer.
Error! Not a valid embedded a. There is correlation between the onset of symptoms and stressful
object. events
Error! Not a valid embedded
b. Resolution within weeks or months
object.
Error! Not a valid embedded
c. Evidence of psychological causation
object.
Error! Not a valid embedded
d. No evidence of physical disorder that might explain the symptoms
object.
Error! Not a valid embedded
e. Evidence of secondary gain
object.
Secondary gain and unconscious motivation are not included among the guidelines or criteria for diagnosis. The
common themes that are shared by dissociative or conversion disorders are a partial or complete loss of the
normal integration between memories of the past, awareness of identity and immediate sensations, and
control of bodily movements. All types of dissociative disorders tend to remit after a few weeks or months,
particularly if their onset is associated with a traumatic life event. More chronic disorders, particularly
paralyses and anaesthesias, may develop if the onset is associated with insoluble problems or interpersonal
difficulties. They are presumed to be psychogenic in origin, being associated closely in time with traumatic
events, insoluble and intolerable problems, or disturbed relationships. The symptoms often represent the
patient's concept of how a physical illness would be manifest. Medical examination and investigation do not
reveal the presence of any known physical or neurological
ICD handbook,2002,p-153
Incorrect
Marks for this submission: 0/1.
Question 31
Marks: 0/1
The following is true with regards to antidepressant induced sexual dysfunction?
Choose one answer.
Error! Not a valid embedded object. a. The incidence with SSRIs is low
Error! Not a valid embedded object. b. The incidence with mirtrazepine is low
Error! Not a valid embedded object. c. Tricyclics are responsible in 30%
Error! Not a valid embedded object. d. Moclobomide has high incidence
Error! Not a valid embedded object. e. Venlafaxine is a cause in 20%
The incidence of sexual dysfunction with SSRIs and venlafaxine is high, ranging from 58% to 73%, as compared
with serotonin-2 (5-HT2) blockers (nefazodone and mirtazapine), moclobemide, and amineptine.
Montejo et al,Incidence of sexual dysfunction associated with antidepressant agents: a prospective multicenter
study of 1022 outpatients. Spanish Working Group for the Study of Psychotropic-Related Sexual Dysfunction.J
Clin Psychiatry 2001; 62 10-21
Incorrect
Marks for this submission: 0/1.
Question 32
Marks: 0/1
A 29 year old man has been using drugs of recreation and asks you about the one which is less likely to have
physical and psychological dependence
Choose one answer.
Error! Not a valid embedded object. a. Amphetamine
Error! Not a valid embedded object. b. Cannabis
Error! Not a valid embedded object. c. Heroin
Error! Not a valid embedded object. d. Cocaine
Error! Not a valid embedded object. e. Speed
For much of 1970s cannabis was not regarded as a drug of dependence but studies have shown features of
dependence particularly in heavy users.
New Oxford Textbook of Psychiatry, vol 1,p-551
Incorrect
Marks for this submission: 0/1.
Question 33
Marks: 0/1
The Scale that is used to measure the severity of EPSE is
Choose one answer.
Error! Not a valid embedded object. a. AIMS
Error! Not a valid embedded object. b. Barnes Akathisia scale
Error! Not a valid embedded object. c. Simpson Angus scale
Error! Not a valid embedded object. d. BPRS
Error! Not a valid embedded object. e. MADRS
Simpson Angus Scale is used to measure severity of EPSE, AIMS is used measure Tardive dyskinesia SIMPSON &
ANGUS, A RATING SCALE FOR EXTRAPYRAMIDAL SIDE EFFECTS, Acta Psychiatrica Scandinavica Volume 45 Issue
S212, Pages 11 - 19 2007
Incorrect
Marks for this submission: 0/1.
Question 34
Marks: 0/1
The following is true in the treatment of ADHD
Choose one answer.
Error! Not a valid embedded object. a. Comorbid symptoms should not change the prescription
Error! Not a valid embedded object. b. Medication benefits only hyperactivity
Error! Not a valid embedded object. c. Atomoxetine is the first line in preschool children
Error! Not a valid embedded object. d. Avoid using antipsychotics in the treatment of ADHD in children
Error! Not a valid embedded object. e. Atomoxetine Can be given only as second line
Nice guidelines recommend to avoid the use of antipsychotics for ADHD in children and young people.
Nice guidelines for ADHD,2008
Incorrect
Marks for this submission: 0/1.
Question 35
Marks: 0/1
According to current evidence base, the treatment of choice for pathological crying is:
Choose one answer.
Error! Not a valid embedded object. a. Amitryptiline
Error! Not a valid embedded object. b. Duloxetine
Error! Not a valid embedded object. c. Mirtrazepine
Error! Not a valid embedded object. d. Reboxetine
Error! Not a valid embedded object. e. Venlafaxine
Amitryptilline has been effective in treating the symptoms of pathological crying and laughing at a dose of 75
mg.
Kate Jefferies ,The neuropsychiatry of multiple sclerosis ,Advances in Psychiatric Treatment (2006) 12: 214-220
Incorrect
Marks for this submission: 0/1.
Question 36
Marks: 0/1
The outcome of first episode psychosis in a young male is worse if the diagnosis is which of the following
Choose one answer.
Error! Not a valid embedded object. a. Schizophrenia
Error! Not a valid embedded object. b. Manic episode with psychosis
Error! Not a valid embedded object. c. acute transient psychosis
Error! Not a valid embedded object. d. Schizoaffective disorder
Error! Not a valid embedded object. e. drug induced psychosis
A recent large Systematic review has identified that the diagnosis of schizophrenia and treatment with
neuroleptics alone had a poor outcome. Predictors associated with better outcome domains were:
combination of pharmacotherapy and psychosocial therapy, and a developing country of origin.

MENEZES, et al. A systematic review of longitudinal outcome studies of first-episode psychosis, Psychological
Medicine. 36(10):1349-1362, October 2006
Incorrect
Marks for this submission: 0/1.
Question 37
Marks: 0/1
The antidepressant of choice used in the treatment of moderate depression in a patient who has Myocardial
infarction is
Choose one answer.
Error! Not a valid embedded object. a. Imipramine
Error! Not a valid embedded object. b. Citalopram
Error! Not a valid embedded object. c. Paroxetine
Error! Not a valid embedded object. d. Fluoxetine
Error! Not a valid embedded object. e. Sertraline
When initiating antidepressant treatment in patients with recent MI or unstable angina ,sertraline is the
treatment of choice and has the best evidence
Nice guidelines for depression,2004
Incorrect
Marks for this submission: 0/1.
Question 38
Marks: 0/1
An elderly gentleman has been diagnosed with delusional disorder. The following is true about delusional
disorder
Choose one answer.
Error! Not a valid embedded object. a. persecutory subtype was the commonest one
Error! Not a valid embedded object. b. persecutory delusions are seen in 25% of those with diagnosis
Error! Not a valid embedded object. c. Older age at onset being associated with somatic delusions
Error! Not a valid embedded object. d. Grandiose delusions were the most common type
Error! Not a valid embedded object. e. The most common type was somatic delusions
Persecutory delusions are the most common type in all populations , upto 60% followed by somatic in young
ages and grandiose delusions.
Yamada et al. Age at onset of delusional disorder is dependent on the delusional theme, Acta Psychiatrica
Scandinavica
Volume 97 Issue 2, Pages 122 - 124
Incorrect
Marks for this submission: 0/1.
Question 39
Marks: 0/1
A person is considered as Unfit to plead if
Choose one answer.
Error! Not a valid embedded object. a. They have severe anxiety disorder at the time of offence
Error! Not a valid embedded object. b. They cannot understand the charge against them
Error! Not a valid embedded object. c. They are thought disordered
Error! Not a valid embedded object. d. They have a personality disorder
Error! Not a valid embedded object. e. They have schizophrenia at the time of the offence
In order to be fit to plead the individual must be able to: Understand the charge (indictment) against them;
Understand the details of the evidence against them ; Adequately instruct his legal advisors; Follow the court
proceedings & Challenge a juror. Chiswick D, Cope R. Seminars in Forensic Psychiatry. Gaskell, 1995 p108-11
Incorrect
Marks for this submission: 0/1.
Question 40
Marks: 0/1
The risk of suicide after 1 year of self harming is
Choose one answer.
Error! Not a valid embedded object. a. 0.70%
Error! Not a valid embedded object. b. 2.50%
Error! Not a valid embedded object. c. 6%
Error! Not a valid embedded object. d. 4%
Error! Not a valid embedded object. e. 3%
The overall risk for all ages at 1 yr after DSH is 0.7% even though earlier studies reported 1% HAWTON et al,
Suicide following deliberate self-harm: long-term follow-up of patients who presented to a general hospital The
British Journal of Psychiatry (2003) 182: 537-542
Incorrect
Marks for this submission: 0/1.
Question 41
Marks: 0/1
A 30 year old lady has a diagnosis of postnatal depression. A self rated scale for severity of her depression is
Choose one answer.
Error! Not a valid embedded object. a. Zungs
Error! Not a valid embedded object. b. MADRS
Error! Not a valid embedded object. c. Edinburgh postnatal depression scale
Error! Not a valid embedded object. d. HAMD
Error! Not a valid embedded object. e. Becks Depression inventory
The Edinburgh post natal depression rating scale is a 10 item self administered scale used as a screening
instrument in postnatal mothers. It is not useful to rate the severity of depression. The Beck Depression
Inventory (BDI) is a 21-item self rated instrument which purports to measure presence and degree of
depression in adolescents and adults.
A.T. Beck, C.H. Ward, M. Mendelson, J. Mock, J. Erbaugh. An Inventory for Measuring Depression. Archives of
General Psychiatry. 1961. vol 4 pp 53-63
Incorrect
Marks for this submission: 0/1.
Question 42
Marks: 0/1
Which one of the following is recognised to cause schizophrenia like psychosis
Choose one answer.
Error! Not a valid embedded object. a. Obsessive compulsive disorder
Error! Not a valid embedded object. b. Huntington's disease
Error! Not a valid embedded object. c. Generalised anxiety disorder
Error! Not a valid embedded object. d. Heroin dependence
Error! Not a valid embedded object. e. Panic disorder
Huntington's disease can present with florid schizophrenic symptoms and can be present even before the true
diagnosis can become apparent.
Organic psychaitry,Lishman,3 edn,p-468
Incorrect
Marks for this submission: 0/1.
Question 43
Marks: 0/1
A 60 year old man is diagnosed with late onset schizophrenia . The following is less likely to be associated with
this condition
Choose one answer.
Error! Not a valid embedded object. a. auditory hallucinations are common
Error! Not a valid embedded object. b. 10-20% have only delusions
Error! Not a valid embedded object. c. Acute onset
Error! Not a valid embedded object. d. insidious onset
Error! Not a valid embedded object. e. Ist rank symptoms are seen in 30%
Late onset schizophrenia or paraphrenia is insidious in onset with auditory hallucinations being the most
common type , in 20% patients only delusions are seen
Companion to psychiatric studies,6 the edn,p-735
Incorrect
Marks for this submission: 0/1.
Question 44
Marks: 0/1
The most likely obstetric outcome in a young pregnant woman who suffers from anorexia nervosa is
Choose one answer.
Error! Not a valid embedded object. a. Caesarean section
Error! Not a valid embedded object. b. Post-term delivery
Error! Not a valid embedded object. c. Low Apgar scores
Error! Not a valid embedded object. d. Induction of labour
Error! Not a valid embedded object. e. Large for gestational age babies
Women with anorexia are more likely to deliver by caesarean section particularly if they are symptomatic
during the pregnancy .
Franko et al. Pregnancy complications and neonatal outcomes in women with eating disorders. Am J Psychiatry
2001;158:1461-6
Incorrect
Marks for this submission: 0/1.
Question 45
Marks: 0/1
In the treatment of hypochondriasis with CBT the following is applicable
Choose one answer.
Error! Not a valid embedded object. a. The delusions changed
Error! Not a valid embedded object. b. Repeated body checking is encouraged
Error! Not a valid embedded object. c. Modifying dysfunctional assumptions is not used
Error! Not a valid embedded object. d. Prevention of reassurance produced an improvement in symptoms
Error! Not a valid embedded object. e. The physical symptoms reduced
Studies have shown that exposure to illness cues and prevention of reassurance produced rapid improvement .
New Oxford textbook of psychiatry, vol 2,p-1103
Incorrect
Marks for this submission: 0/1.
Question 46
Marks: 0/1
The following are contraindications to the use of Zopiclone except
Choose one answer.
Error! Not a valid embedded object. a. Unstable myasthenia gravis
Error! Not a valid embedded object. b. Pregnancy
Error! Not a valid embedded object. c. Respiratory depression
Error! Not a valid embedded object. d. Sleep apnoea
Error! Not a valid embedded object. e. Breastfeeding
The use of benzodiazepines should be avoided in pregnancy but not contraindicated.
BNF,2008,p-790 & 182
Incorrect
Marks for this submission: 0/1.
Question 47
Marks: 0/1
A patient with schizophrenia becomes overweight after treatment with Olanzapine. What would be the best
alternative option in their treatment
Choose one answer.
Error! Not a valid embedded object. a. Aripiprazole
Error! Not a valid embedded object. b. Quetiapine
Error! Not a valid embedded object. c. Clozapine
Error! Not a valid embedded object. d. Amisulpiride
Error! Not a valid embedded object. e. Risperidone
Aripiprazole maybe a good alternative for patients who are experiencing weight gain on other antipsychotics as
it has less propensity to cause weight gain.
Sullivan G, Bienroth M, Jonesc M, Millar H, Ratna L & Taylor D 2007. Practical prescribing with aripiprazole in
schizophrenia: consensus recommendations of a UK multidisciplinary panel Current Medical Research and
Opinion 23(7) pp 1733-1744.
Incorrect
Marks for this submission: 0/1.
Question 48
Marks: 0/1
A 23 year old woman has features of hypomania and she is also breast feeding her baby. The best treatment
option for treatment of hypomania is
Choose one answer.
Error! Not a valid embedded object. a. Olanzapine
Error! Not a valid embedded object. b. Clozapine
Error! Not a valid embedded object. c. Lithium
Error! Not a valid embedded object. d. Lamotrigine
Error! Not a valid embedded object. e. Lorazepam
Antipsychotics apart from clozapine can be used in breastfeeding mothers but all the others mentioned here
should be avoided if possible
Nice guidelines, The management of bipolar disorder in adults, children and adolescents, in primary and
secondary care. 2006
Incorrect
Marks for this submission: 0/1.
Question 49
Marks: 0/1
You are counselling a mother who has one child with autism and is expecting her second. She wishes to know
the risk of autism in this second child. You say
Choose one answer.
Error! Not a valid embedded object. a. 10-20%
Error! Not a valid embedded object. b. 30-40%
Error! Not a valid embedded object. c. 40-50%
Error! Not a valid embedded object. d. 50-60%
Error! Not a valid embedded object. e. 2-3%
Studies have shown that the risk of autism in siblings is 2-3 %,which 50-100 fold increase over the population
rates.
New Oxford textbook of psychiatry,Vol2,p-1726
Incorrect
Marks for this submission: 0/1.
Question 50
Marks: 0/1
A 8 year old child is referred with enuresis .The main stay of treatment is
Choose one answer.
Error! Not a valid embedded object. a. Alarms
Error! Not a valid embedded object. b. Anticholinergics
Error! Not a valid embedded object. c. Amitryptilline
Error! Not a valid embedded object. d. Imipramine
Error! Not a valid embedded object. e. behaviour therapy
A simple monitoring and reward programme that includes a chart may be a n effective step before looking at
other options
Text book of Psychiatry,Hales,3 rd edition,p-1356
Incorrect
Marks for this submission: 0/1.
Question 51
Marks: 0/1
What treatment would you use for a patient with severe symptoms of pre-menstrual syndrome?
Choose one answer.
Error! Not a valid embedded object. a. Oligoacids
Error! Not a valid embedded object. b. Progesterone
Error! Not a valid embedded object. c. SSRI's
Error! Not a valid embedded object. d. Danazol
Error! Not a valid embedded object. e. Evening primrose oil
SSRI's have the strongest evidence for their effectiveness in the treatment of PMS.
Henshaw. PMS: Diagnosis, aetiology, assessment and management: REVISITING... PREMENSTRUAL SYNDROME.
Advan. Psychiatr. Treat., Mar 2007; 13: 139 - 146.
Incorrect
Marks for this submission: 0/1.
Question 52
Marks: 0/1
The best screening method for alcohol problems in primary care settings is
Choose one answer.
Error! Not a valid embedded object. a. MCV
Error! Not a valid embedded object. b. CAGE
Error! Not a valid embedded object. c. AUDIT
Error! Not a valid embedded object. d. GGT
Error! Not a valid embedded object. e. MAST

AUDIT and its derivatives are superior to CAGE for the detection of hazardous harmful drinking.

Parker et al. Diagnosis and Management of Alcogol use disorders. BMJ 2008;336:496-501

Incorrect
Marks for this submission: 0/1.
Question 53
Marks: 0/1
SIADH is more likely to be associated with
Choose one answer.
Error! Not a valid embedded object. a. Lamotrigine
Error! Not a valid embedded object. b. Lithium
Error! Not a valid embedded object. c. Olanzapine
Error! Not a valid embedded object. d. Carbamazepine
Error! Not a valid embedded object. e. Quetiapine
Carbamazepine is associated with SIADH in addition to antidepressants and antipsychotics
Chan, Drug-induced syndrome of inappropriate antidiuretic hormone secretion. Causes, diagnosis and
management, Drugs Aging. 1997 Jul;11(1):27-44.
Incorrect
Marks for this submission: 0/1.
Question 54
Marks: 0/1
The following is true in a lady with eating disorders
Choose one answer.
Error! Not a valid embedded a. The most common direction of movement is from bulimia to anorexia
object.
Error! Not a valid embedded
b. Movements from one type of disorder to another can occur once
object.
Error! Not a valid embedded c. The most common direction of movement is from anorexia to bulimia
object.
Error! Not a valid embedded
d. There is no movement from one type to another
object.
Error! Not a valid embedded
e. The criteria for anorexia and bulimia do not overlap
object.
50% of those with anorexia as primary diagnosis develop bulimia later. The change can occur on several
occasions.
Companion to psychiatric studies,6 the edn,p-510-11
Incorrect
Marks for this submission: 0/1.
Question 55
Marks: 0/1
When assessing a patient with the features of early dementia which of the following would suggest a sub-
cortical rather than a cortical cause?
Choose one answer.
Error! Not a valid embedded object. a. Mild aphasia
Error! Not a valid embedded object. b. Absence of dysarthria
Error! Not a valid embedded object. c. Normal speed of cognitive processes
Error! Not a valid embedded object. d. Euthymic mood
Error! Not a valid embedded object. e. Calculation preserved
Calculation is preserved until very late in subcortical dementia
Kaplan & Saddock's Synopsis of Psychiatry,p-334,10 the edn
Incorrect
Marks for this submission: 0/1.
Question 56
Marks: 0/1
In supranuclear palsy
Choose one answer.
Error! Not a valid embedded object. a. eyes are spared
Error! Not a valid embedded object. b. the onset is in the 4 the decade
Error! Not a valid embedded object. c. the patient is bed ridden in later stages due to rigidity
Error! Not a valid embedded object. d. the falls are usually forwards
Error! Not a valid embedded object. e. death occurs 15-20 years after onset
The onset is in the 6 the decade, with death 5-10 yrs after onset ,falls are typically backwards and eyes are
usually affected.
Lishmans text book of organic psychiatry,3 edn,p-666
Incorrect
Marks for this submission: 0/1.
Question 57
Marks: 0/1
A 35 year old appears to be experiencing symptoms associated with withdrawal of diazepam. The following
signs / symptoms are less likely to be seen
Choose one answer.
Error! Not a valid embedded object. a. Flu like symptoms
Error! Not a valid embedded object. b. Delirium
Error! Not a valid embedded object. c. Seizures
Error! Not a valid embedded object. d. Hypertension
Error! Not a valid embedded object. e. Insomnia
Postural hypotension and tachycardia are associated with diazepam withdrawal
Oxford textbook of psychiatry, vol 2,p-1288
Incorrect
Marks for this submission: 0/1.
Question 58
Marks: 0/1
An otherwise fit and intelligent 15 year old boy has features of a moderate depressive illness. The correct
treatment is
Choose one answer.
Error! Not a valid embedded object. a. SSRI
Error! Not a valid embedded object. b. CBT
Error! Not a valid embedded object. c. Venlafaxine
Error! Not a valid embedded object. d. TCA
Error! Not a valid embedded object. e. CBT and SSRI
There are major controversies about prescribing antidepressants in children and adolescents. TCAs,
Venlafaxine, and Paroxetine should not be used, and limits on use of SSRIs.
NICE recommends:
Mild depression: Antidepressant medication should not be used for the initial treatment of children and young
people with mild depression.
Moderate to severe depression: Children and young people with moderate to severe depression should be
offered, as a first-line treatment, a specific psychological therapy (individual cognitive behavioural therapy
[CBT], interpersonal therapy or shorter-term family therapy; it is suggested that this should be of at least 3
months duration). Antidepressant medication should not be offered to a child or young person with moderate
to severe depression except in combination with a concurrent psychological therapy. Specific arrangements
must be made for careful monitoring of adverse drug reactions, as well as for reviewing mental state and
general progress; for example, weekly contact.

Going by the NICE guidelines, the answer would CBT, however current emerging evidence from the TADS
studies indicates that in children and young people with moderate to sever depression, it is best to use a
combination of SSRIs and CBT.

Incorrect
Marks for this submission: 0/1.
Question 59
Marks: 0/1
The following have the best research evidence for improvement of cognitive functions in a person with Lewy
body dementia
Choose one answer.
Error! Not a valid embedded object. a. Rivastigmine
Error! Not a valid embedded object. b. Memantine
Error! Not a valid embedded object. c. Galantamine
Error! Not a valid embedded object. d. Ginkgo biloba
Error! Not a valid embedded object. e. Donezepil
Patients with dementia with Lewy bodies who suffer from behavioural disturbance or psychiatric problems may
benefit from rivastigmine if they tolerate it, but the evidence is weak
Wild R, Pettit T, Burns A. Cholinesterase inhibitors for dementia with Lewy bodies. Cochrane Database Syst Rev
2003;(3):CD003672.
Incorrect
Marks for this submission: 0/1.
Question 60
Marks: 0/1
A 30 year old patient with schizophrenia is discharged from the hospital on 6mg of Risperidone. They have
relapsed due to non-concordance to medication. The minimum effective dose of Risperdone in treating a
relapse of schizophrenia is
Choose one answer.
Error! Not a valid embedded object. a. 3mg
Error! Not a valid embedded object. b. 2mg
Error! Not a valid embedded object. c. 8mg
Error! Not a valid embedded object. d. 4mg
Error! Not a valid embedded object. e. 6mg
2mg is the minimum effective dose of risperidone for 1 st episode and 4mg for relapse of schizophrenia
The Maudsley prescribing guidelines. 9th Edition, 2007. Page no. 15
Incorrect
Marks for this submission: 0/1.
Question 61
Marks: 0/1
A 35 year old man attending therapy tells the therapist right at the end of the session that he has been abusing
his child
Choose one answer.
Error! Not a valid
a. To immediately clarify what they means by the term 'abusing'
embedded object.
Error! Not a valid b. To reassure her that everything said in therapy is confidential and that she
embedded object. should discuss the use in more detail.
Error! Not a valid
c. Change the subject
embedded object.
Error! Not a valid
d. To tell her that you have to report her to the authorities.
embedded object.
Error! Not a valid e. to ensure boundaries are maintained by ending the session on time and waiting
embedded object. until the next session to bring up the issue.
Ideally it is important for the therapist to ensure that boundaries are maintained and the session ends as per
schedule. However, in this case, as there is a potential risk of harm to others, and particularly for children, it is
the duty of the Therapist to clarify if there is cause for concern. There may be a need to make a possible
referral to Children protection agencies (at the therapist's discretion and after discussions with members of the
multi disciplinary team).
Zachary A. Individual Psychoanalytical Psychotherapy with perpetrators of sexual abuse. Advances in
Psychiatric Treatment 1998; 4:77-81 AND Norcross, J. C. & Guy, J. D. (2007). Leaving it at the office: A guide to
psychotherapist self care (pp. 93-113). London: Guilford Press.
Incorrect
Marks for this submission: 0/1.
Question 62
Marks: 0/1
The following psychotropic medication is contraindicated in a patient with Non Insulin dependent Diabetes
Mellitus who develops schizophrenia is
Choose one answer.
Error! Not a valid embedded object. a. Risperidone
Error! Not a valid embedded object. b. Olanzapine
Error! Not a valid embedded object. c. Aripiprazole
Error! Not a valid embedded object. d. Quetiapine
Error! Not a valid embedded object. e. Amisulpiride
Most antipsychotics are associated with impaired glucose tolerance and Diabetes Mellitus. The most implicated
is Clozapine, followed by Olanzapine, though there are case-reports of other antipsychotics being implicated.
The Maudsley guidelines cautiously recommend Ziprasidone, Aripiprazole, and Amisulpiride in those with a
history of or predisposition to Diabetes Mellitus.
The Maudsley prescribing guidelines. 9th Edition, 2007. Page no. 124.
Incorrect
Marks for this submission: 0/1.
Question 63
Marks: 0/1
In the treatment of opiate withdrawal in a person having low BP the following is given
Choose one answer.
Error! Not a valid embedded object. a. Buprenorphine
Error! Not a valid embedded object. b. Dihydrocodeine
Error! Not a valid embedded object. c. Naloxone
Error! Not a valid embedded object. d. Lofexidine
Error! Not a valid embedded object. e. Clonidine
Buprenorphine or methadone are suggested as the first line in the opioid detoxification. Lofexidine is less likely
to cause hypotension compared to clonidine hence this could be the choice if the patient prefers not to use
methadone or buprenorphine. NICE advocates that Clonidine or dihydrocodeine should not be routinely used
for opioid detoxification.
Nice guidelines Drug Misuse - Opioid detoxification, 2007
Incorrect
Marks for this submission: 0/1.
Question 64
Marks: 0/1
In a case of patient with Learning Disability with affective disorder, before starting lithium what are the
baseline investigation needed to be done:
Choose one answer.
Error! Not a valid embedded object. a. Urinalysis
b. EEG
Error! Not a valid embedded object. c. Chest X ray
Error! Not a valid embedded object. d. Lipid profile
Error! Not a valid embedded object. e. ECG
Before prescribing lithium cardiac, renal and thyroid function should be checked.
Maudsley prescribing guidelines 2005-6, 8 the edn
Incorrect
Marks for this submission: 0/1.
Question 65
Marks: 0/1
A 19 year old pregnant girl asks you for the best time for undergoing Opiate withdrawal during her pregnancy
Choose one answer.
Error! Not a valid embedded object. a. I trimester
Error! Not a valid embedded object. b. II trimester
Error! Not a valid embedded object. c. After delivery
Error! Not a valid embedded object. d. III trimester
Error! Not a valid embedded object. e. All 3 trimesters
The Orange book (Drug Misuse and Dependence - UK guidelines on clinical management - 2007) have
suggested that maintenance at a dose that stops or minimises illicit opiate use is the appropriate for ensuring
continuity of management of pregnancy and aftercare. If detoxification is requested, it is best done in the 2nd
trimester as there is a risk of spontaneous abortion in the 1st trimester and fetal stress, fetal distress and
stillbirth if detoxification is attempted in the 3rd trimester.
Click here for reference
Incorrect
Marks for this submission: 0/1.
Question 66
Marks: 0/1
Gastro intestinal symptoms of Opiate withdrawal include the following except:
Choose one answer.
Error! Not a valid embedded object. a. Diarrhoea
Error! Not a valid embedded object. b. Constipation
Error! Not a valid embedded object. c. Lacrimation
Error! Not a valid embedded object. d. Nausea
Error! Not a valid embedded object. e. Vomiting
The opioid withdrawal syndrome is characterized by rhinorrhea, sneezing, yawning, lacrimation, abdominal
cramping, leg cramping, piloerection (gooseflesh), nausea, vomiting, diarrhoea, and dilated pupils
""
Incorrect
Marks for this submission: 0/1.
Question 67
Marks: 0/1
An 80 year old lady admitted to general hospital as she was severely unwell .Two days later she began
complaining to her relatives that staff are not treating her well and poisoning her food .The likely diagnosis is
Choose one answer.
Error! Not a valid embedded object. a. Delusional disorder
Error! Not a valid embedded object. b. Drug Induced psychosis
Error! Not a valid embedded object. c. Delirium
Error! Not a valid embedded object. d. Schizophrenia
Error! Not a valid embedded object. e. Dementia
Acute onset of psychotic symptoms in the elderly particularly in patients on medical/surgical wards/icu it is
important to consider delirium as the most likely possibility.
Shorter text book of psychiatry,4 the edn,p-403
Incorrect
Marks for this submission: 0/1.
Question 68
Marks: 0/1
Containment is:
Choose one answer.
Error! Not a valid
a. Therapist's ability to keep the emotions elicited by the patient to himself
embedded object.
Error! Not a valid b. Ability of the therapist to modify and return the patient's difficult material in a
embedded object. way that he (she) can tolerate
Error! Not a valid
c. Patient's ability to deal with unpleasant emotions
embedded object.
Error! Not a valid
d. Patient's ability to deal with anger
embedded object.
Error! Not a valid
e. Therapist's ability to keep the actions elicited by the patient to himself
embedded object.
Container' refers to a thinking function that the projected element encounters (usually the group members'
projection). The therapist then uses the transference and the resulting counter-transference to understand
more about the group's needs and defences. (thus, in the therapeutic context, it refers to the therapist-patient
dynamics).
Text book of Psychotherapy in psychiatric practice,churchill livingstone,p-321
Incorrect
Marks for this submission: 0/1.
Question 69
Marks: 0/1
The prevalence of psychotic disorders in prisons compared to the general population is
Choose one answer.
Error! Not a valid embedded object. a. 10 times
Error! Not a valid embedded object. b. 40 times
Error! Not a valid embedded object. c. 30 times
Error! Not a valid embedded object. d. 50 times
Error! Not a valid embedded object. e. 20 times
The prevalence of psychotic disorders in prison is between 2-15% and approximately 20 times greater than that
for adults in the general population
New Oxford textbook of Psychaitry,Vol II,p-204
Incorrect
Marks for this submission: 0/1.
Question 70
Marks: 0/1
A 40 year farmer has a non itchy rash on his chest which is erythematous following tick bite. He also complains
of fatigue, seizures ,weakness and feeling low. The most likely diagnosis is
Choose one answer.
Error! Not a valid embedded object. a. Chronic fatigue syndrome
Error! Not a valid embedded object. b. Multiple sclerosis
Error! Not a valid embedded object. c. Dissociative disorder
Error! Not a valid embedded object. d. Lymes disease
Error! Not a valid embedded object. e. Depression
Lymes disease is caused by borrelia through tick bite has the above features. Complete recovery is the rule with
treatment.
Textbook of Organic psychiatry, Lischman, 3 edn,p-369
Incorrect
Marks for this submission: 0/1.
Question 71
Marks: 0/1
The outcome of first episode psychosis in a young male is worse if the diagnosis is
Choose one answer.
Error! Not a valid embedded object. a. affective psychosis
Error! Not a valid embedded object. b. drug induced psychosis
Error! Not a valid embedded object. c. Schizophrenia
Error! Not a valid embedded object. d. Mania with psychotic symptoms
Error! Not a valid embedded object. e. transient psychotic episode
A diagnosis of schizophrenia and treatment with typical neuroleptics is associated with a poor outcome
Menezes al, A systematic review of longitudinal outcome studies of first-episode psychosis, Psychological
Medicine. 36(10):1349-1362, October 2006.
Incorrect
Marks for this submission: 0/1.
Question 72
Marks: 0/1
NINCDS - ADRDA criteria for diagnosis of Alzheimer's disease does not include
Choose one answer.
Error! Not a valid embedded object. a. progressive worsening of memory and other cognitive functions
Error! Not a valid embedded object. b. no disturbance of consciousness
Error! Not a valid embedded object. c. onset between 40 -90 years
Error! Not a valid embedded object. d. Absence of systemic disorders
Error! Not a valid embedded object. e. deficits in more than one area of cognition
The criteria includes deficits in 2 or more areas of cognition with dementia established by clinical examination
and MMSE
Organic Psychiatry,Lishman,3 rd edn,p-439
Incorrect
Marks for this submission: 0/1.
Question 73
Marks: 0/1
The following psychotropic medication most likely to cause amenorrhoea is
Choose one answer.
Error! Not a valid embedded object. a. Olanzapine
Error! Not a valid embedded object. b. Quetiapine
Error! Not a valid embedded object. c. Aripiprazole
Error! Not a valid embedded object. d. Clozapine
Error! Not a valid embedded object. e. Amisulpiride
Plasma prolactin levels are increased during amisulpride therapy and amenorrhoea occurs in about 4% of
women.
""
Incorrect
Marks for this submission: 0/1.
Question 74
Marks: 0/1
Sexual dysfunction associated with SSRI is due to which of the following actions
Choose one answer.
Error! Not a valid embedded object. a. 5HT 1 stimulation
Error! Not a valid embedded object. b. 5 HT 2 Stimulation
Error! Not a valid embedded object. c. 5 HT 1 blockade
Error! Not a valid embedded object. d. 5HT3 blockade
Error! Not a valid embedded object. e. 5HT2 blockade
The sexual side effects of SSRIs are reported to be due to activation of 5HT2 receptors
David Baldwin and Andrew Mayers, Sexual side-effects of antidepressant and antipsychotic drugs Advances in
Psychiatric Treatment (2003) 9: 202-210
Incorrect
Marks for this submission: 0/1.
Question 75
Marks: 0/1
The following is not a diagnostic criteria for diagnosing chronic fatigue syndrome
Choose one answer.
Error! Not a valid embedded object. a. irregular, disturbed sleep
Error! Not a valid embedded object. b. excessive fatigue after mild exertion
Error! Not a valid embedded object. c. Pains and aches
Error! Not a valid embedded object. d. failure to recover from exertion after rest
Error! Not a valid embedded object. e. duration of symptoms >6 months
The diagnosis of CFS is made if the symptoms are present for 4 months and 3 months in adults and children
respectively with other diagnosis being excluded. The key symptoms include post-exertional fatigue or malaise
(typically delayed, for example by at least 24 hours, with slow recovery over several days) , cognitive
difficulties, sleep disturbance, chronic pain & ? palpitations in the absence of identified cardiac pathology.
Nice guidelines, Chronic fatigue syndrome / Myalgic encephalomyelitis (or encephalopathy); diagnosis and
management. 2007
Incorrect
Marks for this submission: 0/1.
Question 76
Marks: 0/1
A patient with Parkinson's disease becomes distressingly psychotic. Which one of the following is known is an
established treatment for this condition
Choose one answer.
Error! Not a valid embedded object. a. lorazepam
Error! Not a valid embedded object. b. Clozapine
Error! Not a valid embedded object. c. Amitryptilline
Error! Not a valid embedded object. d. Olanzapine
Error! Not a valid embedded object. e. Quetiapine
According to a 2007 meta-analysis, clozapine is the only atypical antipsychotic fully recommended for the
treatment of Parkinsons disease psychosis due to its demonstrated efficacy and tolerability
Laura B. Zahodne, and Hubert H. Fernandez. Course, Prognosis, and Management of Psychosis in Parkinsons
Disease: Are Current Treatments Really Effective? CNS Spectr. 2008;13(3 Suppl 4):26-33
Incorrect
Marks for this submission: 0/1.
Question 77
Marks: 0/1
A woman lost her mother recently. A feature which would take your diagnosis towards depression rather than
normal grief reaction
Choose one answer.
Error! Not a valid embedded object. a. Irritability
Error! Not a valid embedded object. b. Hearing the voice of the deceased
Error! Not a valid embedded object. c. Generalized guilt
Error! Not a valid embedded object. d. Severe insomnia
Error! Not a valid embedded object. e. Loss of appetite
Intense feelings of guilt not related to the bereavement, thoughts of suicide or a preoccupation with dying,
feelings of worthlessness, marked psychomotor retardation, prolonged or severe inability to function (not able
to work, socialise or enjoy any leisure activity), prolonged hallucinations of the deceased, or hallucinations
unrelated to the bereavement are all factors indicative of depression.
""
Incorrect
Marks for this submission: 0/1.
Question 78
Marks: 0/1
A 30 year old man has been suffering from schizophrenia asks you about the percentage of people who
discontinue medication during the first 18 months of treatment
Choose one answer.
Error! Not a valid embedded object. a. 25%
Error! Not a valid embedded object. b. 10%
Error! Not a valid embedded object. c. 75%
Error! Not a valid embedded object. d. 20%
Error! Not a valid embedded object. e. 50%
Medication discontinuation is a major problem in the treatment of first-episode patients; The CATIE study
found that 74 percent of patients discontinued the antipsychotic medication before 18 months (1061 of the
1432 patients who received at least one dose): 64 percent of those assigned to olanzapine, 75 percent of those
assigned to perphenazine, 82 percent of those assigned to quetiapine, 74 percent of those assigned to
risperidone, and 79 percent of those assigned to The figures are similar at one year.
Lieberman et al., Effectiveness of Antipsychotic Drugs in Patients with Chronic Schizophrenia. NEJM. 2005,
353:1209-1223
Incorrect
Marks for this submission: 0/1.
Question 79
Marks: 0/1
Which of the following is recommended in treatment of a adolescent girl with hypomania?
Choose one answer.
Error! Not a valid embedded object. a. Lithium
Error! Not a valid embedded object. b. Valproate
Error! Not a valid embedded object. c. Lamotrigine
Error! Not a valid embedded object. d. Olanzapine
Error! Not a valid embedded object. e. Carbamazepine
Atypical antipsychotics are recommended as first line treatment in adolescent girls . Medication that cause
weight gain and increased prolactin should be avoided, but of the choices Olanzapine though is associated with
weight gain, is the answer as the others are to be avoided.
Nice guidelines for BPAD,2006
Incorrect
Marks for this submission: 0/1.
Question 80
Marks: 0/1
What is the risk of a neonate developing neural tube defects if prescribing Valproate to their mother when
pregnant?
Choose one answer.
Error! Not a valid embedded object. a. 1 in 100000
Error! Not a valid embedded object. b. 1in 100
Error! Not a valid embedded object. c. 1 in 10 000
Error! Not a valid embedded object. d. 1 in 10
Error! Not a valid embedded object. e. 1 in 1000
The risk of fetal birth defects (particularly Neural tube defects) in a woman taking Sodium Valproate is 100-
200/10,000, i.e., 1-2 /100
NICE guidelines on Antenatal & Postnatal Mental Health 2007.
Incorrect
Marks for this submission: 0/1.
Question 81
Marks: 0/1
A 45 year old woman during the course of her therapy says that she is happy but the whole world is depressed.
She is displaying the following defence mechanism
Choose one answer.
Error! Not a valid embedded object. a. Selective abstraction
Error! Not a valid embedded object. b. Externalisation
Error! Not a valid embedded object. c. Overgeneralisation
Error! Not a valid embedded object. d. Projection
Error! Not a valid embedded object. e. Projective identification
The woman is using projection by attributing her subjective feelings onto an external force. This is different
from projective identification (Melanie Klien) which carries with it the added behavioural component of
reacting to the intolerable impulse projected outside self (provides a validation for the projection). Selective
abstraction is not a defense mechanism (instead, it is a CBT term). Externalisation and overgeneralisation are
types of cognitive distortion listed in the context of CBT (they are not listed by Freud or neo-Freudians as
'defense mechanisms'). Johnstone, E. C., Cunningham Owens, D. G., Lawrie, S. M., Sharpe, M., & Freeman, C. P.
L. (2004). Companion to psychiatric studies (7th ed., pp 311-314); Rycroft, C. (1995). A critical dictionary of
psychoanalysis. London: Penguin Books
Incorrect
Marks for this submission: 0/1.
Question 82
Marks: 0/1
The following is considered to be a higher risk for suicide
Choose one answer.
Error! Not a valid embedded object. a. 55 yr old ,married, social class V
Error! Not a valid embedded object. b. 16yrs old single, female unmarried, social class I
Error! Not a valid embedded object. c. 35yrs old female, unemployed, social class V
Error! Not a valid embedded object. d. 60yr old man, divorced, social class V
Error! Not a valid embedded object. e. 30yrs Male unmarried ,Social class I
Studies show that economic stressors such as unemployment and low income increase suicide risk more in
male than in female subjects .Studies have reported that single people are more likely to commit suicide
Qinet al, Suicide Risk in Relation to Socioeconomic, Demographic, Psychiatric, and Familial Factors: A National
Register-Based Study of All Suicides in Denmark, 1981-1997
Am J Psychiatry 160:765-772, April 2003
Incorrect
Marks for this submission: 0/1.
Question 83
Marks: 0/1
The following is uncommonly associated with PICA
Choose one answer.
Error! Not a valid embedded object. a. Socially deprived children
Error! Not a valid embedded object. b. Lead poisoning
Error! Not a valid embedded object. c. Toddler period transiently
Error! Not a valid embedded object. d. Learning disability
Error! Not a valid embedded object. e. Psychosis
Lead poisoning is a possible but uncommon danger from PICA
Companion to psychiatric studies,6 the edn,p-660
Incorrect
Marks for this submission: 0/1.
Question 84
Marks: 0/1
A woman who has had Herpes encephalitis develops severe carbohydrate craving ,weight gain and increased
sexual drive. What is the likely Diagnosis?
Choose one answer.
Error! Not a valid embedded object. a. Prader willi syndrome
Error! Not a valid embedded object. b. Hypothyroidism
Error! Not a valid embedded object. c. Kluver bucy syndrome
Error! Not a valid embedded object. d. Cushing's syndrome
Error! Not a valid embedded object. e. Diabetes mellitus
Kluver bucy syndrome is associated with hyperorality, overeating, hypersexuality, with involvement of frontal
and temporal lobes.
Organic psychiatry,Lishman,3 edn,p-433
Incorrect
Marks for this submission: 0/1.
Question 85
Marks: 0/1
LSD is detected in urine
Choose one answer.
Error! Not a valid embedded object. a. within 1 day
Error! Not a valid embedded object. b. 3-5 days
Error! Not a valid embedded object. c. 1-3 days
Error! Not a valid embedded object. d. 5-7 days
Error! Not a valid embedded object. e. after 1 day
LSD can be detected in urine not longer than 12-22 hours of use. Amphetamines / amphetamines, including
methylamphetamine and MDMA are detected upto 2 days. Codeine, dihydrocodeine, morphine, propoxyphene
is detected in urine for upto 48 hours, Cocaine for 2-3 days, Buprenorphine for upto 8 days, and methadone for
upto 7-9 days after last use.
Chromatography,by Erich Heftmann,6th edn ,p-101. & Drug Misuse and Dependence - UK guidelines on clinical
management - 2007, Page 30.
Incorrect
Marks for this submission: 0/1.
Question 86
Marks: 0/1
Smoking in a pregnant lady can lead to the following in the child
Choose one answer.
Error! Not a valid embedded object. a. Learning disability
Error! Not a valid embedded object. b. ADHD
Error! Not a valid embedded object. c. Autism
Error! Not a valid embedded object. d. Autistic spectrum disorder
Error! Not a valid embedded object. e. Mood disorders
Smoking in pregnancy is associated with ADHD, behaviour problems including conduct disorders and antisocial
behaviour in the child
Kate Langley, Peter A Holmans, Marianne BM van den Bree, and Anita Thapar, Effects of low birth weight,
maternal smoking in pregnancy and social class on the phenotypic manifestation of Attention Deficit
Hyperactivity Disorder and associated antisocial behaviour: investigation in a clinical sample. BMC psychiatry,
2007:7:26.
Incorrect
Marks for this submission: 0/1.
Question 87
Marks: 0/1
The most common psychiatric diagnosis associated with shoplifting is
Choose one answer.
Error! Not a valid embedded object. a. OCD
Error! Not a valid embedded object. b. Impulse control disorders
Error! Not a valid embedded object. c. Depression
Error! Not a valid embedded object. d. Substance misuse
Error! Not a valid embedded object. e. Eating disorders
Depression appears to be the most commonly associated diagnosis with shoplifting with studies reporting
almost 100 % lifetime prevalence of depression
New Oxford text book of psychiatry, Vol 1,p-980-981
Incorrect
Marks for this submission: 0/1.
Question 88
Marks: 0/1
The Female to Male ratio in hypothyroidism is
Choose one answer.
Error! Not a valid embedded object. a. 08:01
Error! Not a valid embedded object. b. 10:01
Error! Not a valid embedded object. c. 06:01
Error! Not a valid embedded object. d. 02:01
Error! Not a valid embedded object. e. 04:01
Hypothyroidism is commoner in females at ratio of 8:1 .
Organic psychiatry,Lishman,3 edn,p-512
Incorrect
Marks for this submission: 0/1.
Question 89
Marks: 0/1
According to Kernberg, the psychodynamic understanding of borderline personality disorder did not include
Choose one answer.
Error! Not a valid embedded object. a. Contradictory concept of self
Error! Not a valid embedded object. b. Low anxiety tolerance
Error! Not a valid embedded object. c. primitive defences
Error! Not a valid embedded object. d. Fully intact reality testing
Error! Not a valid embedded object. e. identity diffusion
As per Kernberg, Diagnoses of Borderline personality Organisation are based on three categories of criteria.
The first, and most important, category, comprises two signs:
the absence of psychosis (i.e., the ability to perceive reality accurately)
impaired ego integration - a diffuse and internally contradictory concept of self.
The second category is termed "nonspecific signs" and includes such things as low anxiety tolerance, poor
impulse control, and an undeveloped or poor ability to enjoy work or hobbies in a meaningful way.
The third category is the presence of "primitive defenses." The main one is splitting, in which a person or thing
is seen as all good or all bad. Other primitive defenses cited include magical thinking (beliefs that thoughts can
cause events), omnipotence, projection of unpleasant characteristics in the self onto others and projective
identification, a process where the borderline tries to elicit in others the feelings s/he is having.
Kernberg also includes as signs of BPO chao
Oxford textbook of psychotherapy,2007,p-292
Incorrect
Marks for this submission: 0/1.
Question 90
Marks: 0/1
A 40 year old man who is a heavy drinker does not recognise that he requires to change , does not accept his
problem drinking. Regarding his motivation to change he is in the stage of
Choose one answer.
Error! Not a valid embedded object. a. Action
Error! Not a valid embedded object. b. maintenance
Error! Not a valid embedded object. c. Contemplation
Error! Not a valid embedded object. d. Precontemplation
Error! Not a valid embedded object. e. Preparation
At the precontempaltion stage there is no intention to change and patients are not even aware of their
problem. Secret drinking is seen in this stage.
Handbook of Alcoholism,2000, Zernig et al, P-99-100
Incorrect
Marks for this submission: 0/1.
Question 91
Marks: 0/1
The mechanism of action of Donezepil is
Choose one answer.
Error! Not a valid embedded
a. NMDA antagonism
object.
Error! Not a valid embedded b. irreversible inhibition of the enzyme acetylcholinesterase and butryl
object. cholinesterase
Error! Not a valid embedded c. reversible inhibition of the enzyme acetylcholinesterase and butryl
object. cholinesterase
Error! Not a valid embedded
d. irreversible inhibition of the enzyme acetylcholinesterase
object.
Error! Not a valid embedded
e. reversible inhibition of the enzyme acetylcholinesterase
object.
Donezepil and Galantamine are reversible ACHE inhibitors while Rivastigmine is pseudo-reversible inhibitor of
both ACHE and BCHE. Memantine is a NMDA antagonist.
""
Incorrect
Marks for this submission: 0/1.
Question 92
Marks: 0/1
A 40 years old woman with diagnosis of BPAD, has had over the last few years four episodes of depression and
one episode of hypomania. Which of the following drugs should be used as maintenance
Choose one answer.
Error! Not a valid embedded object. a. Lamotrigine
Error! Not a valid embedded object. b. Valproate
Error! Not a valid embedded object. c. Olanzapine
Error! Not a valid embedded object. d. Carbamazepine
Error! Not a valid embedded object. e. Lithium
NICE guidelines suggest the use of long-term SSRI's, CBT, Quetiapine and Lamotrigine in the long term
treatment of recurrent depressive symptoms in Bipolar affective disorder.
Nice guidelines ,2006
Incorrect
Marks for this submission: 0/1.
Question 93
Marks: 0/1
In severe depression with comorbid personality disorder
Choose one answer.
Error! Not a valid embedded
a. CBT is better than antidepressant medication
object.
Error! Not a valid embedded b. Combination of antidepressants and cognitive therapy is more effective
object. than either
Error! Not a valid embedded
c. CBT has same results as IPT
object.
Error! Not a valid embedded
d. Antidepressant medication is better than CBT
object.
Error! Not a valid embedded
e. CBT is better than IPT
object.
Combined therapy was significantly more effective than pharmacotherapy for depressed patients with
personality disorders. For depressed patients without personality disorders, combined therapy was not more
effective than pharmacotherapy alone.
Fournier et al, Antidepressant medications v. cognitive therapy in people with depression with or without
personality disorder. The British Journal of Psychiatry, February 1, 2008; 192(2): 124 - 129. Kool S. et al.
Treatment of depressive disorder and comorbid personality pathology: combined therapy versus
pharmacotherapy. Tijdschr Psychiatry. 2007;49(6):361-72.
Incorrect
Marks for this submission: 0/1.
Question 94
Marks: 0/1
The least common feature of cushings syndrome is
Choose one answer.
Error! Not a valid embedded object. a. cognitive impairment
Error! Not a valid embedded object. b. hallucinations
Error! Not a valid embedded object. c. euphoria
Error! Not a valid embedded object. d. Delusions
Error! Not a valid embedded object. e. depression
In cushings syndrome 35% have depression,16% have cognitive impairment,9% have psychotic symptoms and 4
% have euphoria
Companion to psychiatric studies,6 edn,p-322
Incorrect
Marks for this submission: 0/1.
Question 95
Marks: 0/1
The last stage according to Prochaska and Diclemente model of change of behaviour is
Choose one answer.
Error! Not a valid embedded object. a. preparation
Error! Not a valid embedded object. b. Maintenance
Error! Not a valid embedded object. c. Precontemplation
Error! Not a valid embedded object. d. Action
Error! Not a valid embedded object. e. contemplation
In the last stage maintainence stage the new behaviours such as abstinence are perpetuated.
Handbook of Alcoholism,2000, Zernig et al, P-99-100
Incorrect
Marks for this submission: 0/1.
Question 96
Marks: 0/1
The lifetime prevalence of OCD is
Choose one answer.
Error! Not a valid embedded object. a. 1-3%
Error! Not a valid embedded object. b. 1-2%
Error! Not a valid embedded object. c. 4-5%
Error! Not a valid embedded object. d. 5-10%
Error! Not a valid embedded object. e. 2-3%
The life time prevalence varies from about 1 to 3 %, with some studies reporting the prevalence as 1-2%, while
others have reported it as 2-3%, so a safe answer would be 1-3%.
Hollander E. Obsessive-compulsive disorder: the hidden epidemic. J Clin Psychiatry 1997. 58 (Suppl 12): 3-6;
NICE guideline: Obsessive Compulsive disorder 2005
Incorrect
Marks for this submission: 0/1.
Question 97
Marks: 0/1
You see a 12 year old boy with Tourette's syndrome, who also has hyperkinetic disorder. The most effective
treatment for this boy would be
Choose one answer.
Error! Not a valid embedded object. a. Risperidone
Error! Not a valid embedded object. b. A TCA
Error! Not a valid embedded object. c. A NARI
Error! Not a valid embedded object. d. Pimoline
Error! Not a valid embedded object. e. A central ? agonist
NICE guidelines recommend the use of Methylphenidate or Atomoxetine (which is a NARI) in the treatment of
ADHD and comorbid Tics.
Nice guidelines for ADHD,2008
Incorrect
Marks for this submission: 0/1.
Question 98
Marks: 0/1
A curve with plot of sensitivity versus 1-specificity on either axis is
Choose one answer.
Error! Not a valid embedded object. a. ROC curve
Error! Not a valid embedded object. b. Galbraith plot
Error! Not a valid embedded object. c. Lorenz curves
Error! Not a valid embedded object. d. Nomogram
Error! Not a valid embedded object. e. normal curve
The receiver operating characteristic (ROC) curve is a plot of the true positive rate (sensitivity) of a test versus
its false-positive rate (1 - specificity ) for all possible cut points.
Receiver Operating Characteristic Curves and Their Use in Radiology, Nancy A. Obuchowski,Statistical Concepts
Series, Radiology 2003;229:3-8.)
Incorrect
Marks for this submission: 0/1.
Question 99
Marks: 0/1
The best evidence for planning services is usually derived from
Choose one answer.
Error! Not a valid embedded object. a. Systematic reviews
Error! Not a valid embedded object. b. Audits
Error! Not a valid embedded object. c. Crossectional studies
Error! Not a valid embedded object. d. Qualitative studies
Error! Not a valid embedded object. e. Meta analysis
The most useful evidence in this planning exercise would have been Cochrane style quantitative reviews of key
parameters e.g.. prevalence, incidence, utilisation data and methods of service delivery for specific population
groups
Chipps et al, Planning an evidence based mental health service - is it possible?, Evid Action Int Cochrane Colloq
8th 2000 Cape Town S Afr. 2000; 8
Incorrect
Marks for this submission: 0/1.
Question 100
Marks: 0/1
The following is a measure of internal validity
Choose one answer.
Error! Not a valid embedded object. a. numbers needed to treat
Error! Not a valid embedded object. b. relative risk
Error! Not a valid embedded object. c. Generalisability
Error! Not a valid embedded object. d. reliability
Error! Not a valid embedded object. e. Intention to treat
Reliability or accuracy is a measure of internal validity where as external validity is the generalisability of
results. The internal validity depends on the research design, research setting and the measures used, whereas
the external validity depends on the sample selection (ensuring representative sample for generalisation to be
achieved)
Godwin et al. Pragmatic controlled clinical trials in primary care: the struggle between external and internal
validity. BMC medical Research methodology,2003,3:28
Incorrect
Marks for this submission: 0/1.
Question 101
Marks: 0/1
The following increases the power of a study
Choose one answer.
Error! Not a valid embedded object. a. increased heterogeneity
Error! Not a valid embedded object. b. Comparing active treatment vs. active treatment
Error! Not a valid embedded object. c. smaller number of participants
Error! Not a valid embedded object. d. Comparing active treatment vs. placebo
Error! Not a valid embedded object. e. Use of self rated questionnaires
The power of a study depends on sample size (large sample size - more power); Effect size (difference between
comparators, so more power when comparing active treatment vs. placebos); reliability of measures used
(observer rated measures more reliable than self-rated ones); and level of significance adopted (p value
significant at 95% etc.)
Companion to Psychiatric studies;6th ed.;168-169
Incorrect
Marks for this submission: 0/1.
Question 102
Marks: 0/1
For questions 102 - 103, read the précis below:

There is a well established depression scale (A) with 100 items and a new scale (B) with 12 items has been
developed. Scale B only takes 10 minutes to administer whereas scale A takes around an hour. Both scales A
and B were administered to 100 depressed in-patients with depression and 100 controls. High scores on A
correlated with high scores on B (Spearman’s correlation r = 0.8 p<0.01)
From the above precis, choose the following which is true:
Choose one answer.
Error! Not a valid embedded object. a. A and B show good inter ratter reliability
Error! Not a valid embedded object. b. A is a good screening tool
Error! Not a valid embedded object. c. A and B are highly LINEARLY correlated
Error! Not a valid embedded object. d. B can replace A
Error! Not a valid embedded object. e. B is a good measure of depression.
From the information provided in the precis, we can only deduce that the two scales are highly correlated with
a correlation coefficient of 80% ""
Incorrect
Marks for this submission: 0/1.
Question 103
Marks: 0/1
From the above precis, choose the following which is true:
Choose one answer.
Error! Not a valid embedded object. a. Not enough information provided to assess reliability
Error! Not a valid embedded object. b. A is more valid than B
Error! Not a valid embedded object. c. Because A is longer than B it must be better
Error! Not a valid embedded object. d. B is more valid than A
Error! Not a valid embedded object. e. B is more reliable than A
To establish reliability, information on test-retest reliability, inter-rater reliability should have been provided.
To establish validity, information on concurrent, criterion, face or construct validity should have been provided.
""
Incorrect
Marks for this submission: 0/1.
Question 104
Marks: 0/1
The number of trials which show a significant difference between Clozapine and Conventional antipsychotic
drugs is
Choose one answer.
Error! Not a valid embedded object. a. 3
Error! Not a valid embedded object. b. 6
Error! Not a valid embedded object. c. 8
Error! Not a valid embedded object. d. 1
Error! Not a valid embedded object. e. 4
The studies Kane88, Klieser, Rosenheck and Kane01 have SMD (standard mean difference) and Confidence
intervals that do not cross the line of no difference (marked zero in this forest plot), and hence studies which
show significant difference between clozapine and standard neuroleptics. ""
Incorrect
Marks for this submission: 0/1.
Question 105
Marks: 0/1
The two studies which showed the most significant difference between Clozapine and standard antipsychotics
is
Choose one answer.
Error! Not a valid embedded object. a. Rosenheck & Kane 01 studies
Error! Not a valid embedded object. b. Kane88 & Rosenheck studies
Error! Not a valid embedded object. c. Kane88 & Kane 01 studies
Error! Not a valid embedded object. d. Kumra & Klieser studies
Error! Not a valid embedded object. e. Essock & Kane 01 studies
Kane88 & Rosenheck studies are the most significant due to the narrow confidence intervals (the horizontal
lines) and large sample sizes (size of the boxes).
""
Incorrect
Marks for this submission: 0/1.
Question 106
Marks: 0/1
For questions 104-110, read the précis below:

Please read this summary from Moncrieff, J. The British Journal of Psychiatry 2003;183:161-166.

The authors were interested to assess the evidence comparing clozapine with conventional antipsychotics in
the treatment of treatment resistant schizophrenia. The authors conducted an electronic search using Medline
and EMBASE from 1998 to April 2003 to identify trials conducted using the keywords CLOZAPINE and
RANDOMISED CONTROL TRIAL or CONTROLLED TRIAL or RANDOMISED TRIAL. They identified nine trials that
compared Clozapine with other antipsychotic drugs in the treatment of schizophrenia. Individual study results
were tabulated and examined. The difference in symptom scores between the clozapine group and the
comparison group at the end of treatment as a percentage of the post-treatment score in the control group
was calculated.

Meta-analysis was conducted to examine heterogeneity between studies, which is assessed by testing the
weighted variation of individual study results about the mean effect. The outcomes of individual trials were
converted to standardised mean differences (SMDs) to allow the results of studies using different outcome
measures to be combined. Intention-to-treat data were used if possible. The results are presented in the plot
below

The plot shown in the precis is referred as


Choose one answer.
Error! Not a valid embedded object. a. Forest plot
Error! Not a valid embedded object. b. Stem leaf plot
Error! Not a valid embedded object. c. Funnel plot
Error! Not a valid embedded object. d. Scatter plot
Error! Not a valid embedded object. e. Galbraith plot
A forest plot is used to provide a visual representation of the meta-analysis, which is a method used to
combine the results of different trials in order to obtain a quantified synthesis Click here for reference
Incorrect
Marks for this submission: 0/1.
Question 107
Marks: 0/1
The number of trials which show a superior efficacy of Clozapine over the standard antipsychotics is
Choose one answer.
Error! Not a valid embedded object. a. 1
Error! Not a valid embedded object. b. 6
Error! Not a valid embedded object. c. 4
Error! Not a valid embedded object. d. 8
The studies Kane88, Klieser, Rosenheck and Kane01 have SMD (standard mean difference) and Confidence
intervals that do not cross the line of no difference (marked zero in this forest plot), and hence studies which
show significant difference between clozapine and standard neuroleptics, and show that Clozapine is better
than conventional antipsychotics.
""
Incorrect
Marks for this submission: 0/1.
Question 108
Marks: 0/1
The number of trials which show a superior efficacy of standard antipsychotics over Clozapine is
Choose one answer.
Error! Not a valid embedded object. a. 8
Error! Not a valid embedded object. b. 1
Error! Not a valid embedded object. c. 4
Error! Not a valid embedded object. d. 6
Error! Not a valid embedded object. e. 0
Though the Essock study has SMD which is on the other side of the line of the no effect showing that
Conventional antipsychotics are better than Clozapine, the Confidence intervals cross the line of no effect,
hence it is not significant. ""
Incorrect
Marks for this submission: 0/1.
Question 109
Marks: 0/1
From the plot above, choose the correct statement
Choose one answer.
Error! Not a valid embedded a. The results of the Kumra study are not significant as the Confidence
object. intervals cross unity
Error! Not a valid embedded b. Effect size of Hownitz study is larger than the effect size of the Kane 01
object. study
c. The effect size of the Rosenheck study was larger than the Effect size of the
Hong Study
d. There is significant heterogeneity between studies
e. The confidence intervals for the Kenneth study was broad
In this forest plot, effect size is the standardised Men difference (position of the squares in relation to the line
of no effect - vertical line), and the confidence intervals are represented by the horizontal lines for each of the
squares. If confidence intervals of studies overlap with each other and the summary effect, then no
heterogeneity is present, i.e., if a straight line can be drawn touching the confidence intervals of each of the
studies then heterogeneity if not present.
""
Incorrect
Marks for this submission: 0/1.
Question 110
Marks: 0/1
From the precis above, the conclusion that can be reached is
Choose one answer.
a. Clozapine has more side effects than standard antipsychotics in the treatment of resistant
schizophrenia
b. Standard Antipsychotics are better than Clozapine in the treatment of Resistant schizophrenia
c. Clozapine is more efficacious than Standard antipsychotics in the treatment of Resistant schizophrenia

d. Clozapine and Standard antipsychotics are of comparable efficacy in the treatment of Resistant
schizophrenia
e. It is difficult to reach any conclusions due to the heterogeneity among the studies
Though the combined effect shows that CLozapine is more efficacious than standard antipsychotics, there is
Considerable heterogeneity between studies, which makes it difficult to consider this result as valid.
Incorrect
Marks for this submission: 0/1.
Question 111
Marks: 0/1
For questions 111-112, read the précis below:

A new screening test for depression was developed to be used in General Practises. It was found to have a
sensitivity of 90% and a Likelihood ratio for negative result of 0.20. A Fagan’s Nomogram has been provided
below.
The prevalence of depression in a community is around 10%. From the above precis, calculate the probability of
a person having depression when they test negative on the new screening test.
Choose one answer.
a. 0.02%
b. 2%
c. 5%
d. 10%
e. 20%
A post test probability can be calculated by two methods: Using a Nomogram or by calculating (see notes). The
prevalence is the pre-test probability. In the nomogram draw a line from the pre-test probability (10%) and the
Likelihood ratio (negative). Continue with that line till it meets the post-test probability line. This is the post-
test probability for the LR(negative). Click here for reference
Incorrect
Marks for this submission: 0/1.
Question 112
Marks: 0/1
In the above example, the likelihood ratio for a positive result would be
Choose one answer.
a. 4
b. 0.2
c. 6
d. 1.8
e. 0.8
LR(-) = (1- sensitivity) / Specificity. Give the LR (-ve) and sensitivity, we can calculate Specificity, and then using
that LR(+ve) = Sensitivity / (1-specificity)
http://www.bmj.com/cgi/content/full/329/7458/168?maxtoshow=&HITS=10&hits=10&RESULTFORMAT=&fullt
ext=Nomogram&searchid=1&FIRSTINDEX=0&resourcetype=HWCIT
Incorrect
Marks for this submission: 0/1.
Question 113
Marks: 0/3
Gender ratio

Options:
A. 1:1
B. 1:3
C. 1:6
D. 1:8
E. 1:10
F. 3:1
G. 4:1
H. 5:1
I. 6:1

Select the Male: Female ratios in the following conditions

Attention deficit and hyperactivity disorder


Autism
Bulimia
Answers.

1. F. 3:1. ADHD are more common in males at ratio of 3:1 (in community samples) whilst the ratio increases to
10:1 in clinic samples.
2. G. 4:1. A large review has found the M:F ratio of Autism to be 3.8 : 1 (or 4:1). Fombonne E. The epidemiology
of autism: a review.Psychol Med. 1999 Jul;29(4):769-86.
3. E. 1:10. Bulimia is more common in females with occurrence in males that of 1/10 th in females.

Incorrect
Marks for this submission: 0/3.
Question 114
Marks: 0/3
Gender ratios

A. 1:1
B. 1:2
C. 1:3
D. 1:5
E. 1:6
F. 1:4
G. 1:10
H. 2:1
I. 3:1
J. 4:1
Select the correct Female to Male ratios for the below:

Asperger's Syndrome
Adolescent depression
Tourette's syndrome
Answers:

1. F. 1:4. Asperger initially reported the condition only in boys but was eventually reported in girls.(Ehlers S et
al. The Epidemiology of Aspergers syndrome. J Child Psychol Psychiatry. 1993 Nov;34(8):1327-50.)

2. J. 4:1. During the adolescence the sex ratio changes progressively, girls outnumbering boys by up to 4:1 for
depression.

3. C. Tourette’s is more common in boys and onset is rare after 11.

Incorrect
Marks for this submission: 0/3.
Question 115
Marks: 0/3
Neuropsychiatry - Diagnosis

A. Alcohol withdrawal
B. Alcoholic delirium
C. Alcoholic hallucinosis
D. Complex partial seizures
E. Fahr’s syndrome
F. Metachromatic leucodystorphy
G. Neuroacanthosis
H. Post Herpetic encephalopathy
I. Systemic Lupus Erythematosis
A 44 year old homeless man is brought into A&E. he is agitated, psychotic and depressed. The ECG shows
decreased alpha activity and his gait is disturbed. MRI shows hypointensity of the striatum.
A 25 year old man recently returned from America shows aggressive behaviour. He recently developed
periods of decreased consciousness. His full blood count and liver functions are normal
A 30 year old woman has one sided facial nerve palsy and circumscribed lesions on both legs.
Answers:

1. B. DT can present as vivid hallucinations, delusions, profound confusion and inattention, agitation and
restlessness, sleeplessness, autonomic overactivity and fearful affect. Gait can be disturbed, EEG might show
reduced alpha activity and MRI could reflect changes in striatum.

2. H. Complex partial seizures cause impaired consciousness and arise from a single brain region. Impaired
consciousness implies decreased responsiveness and awareness of self and surroundings. During a complex
partial seizure, the patient may not communicate, respond to commands, or remember events that occurred.
Epilepsy of prolonged duration and brain dysfunction involving a broad area including the temporal and frontal
lobes may be associated with the occurrence of subacute postictal aggression.

3. H. Shingles is an acute infection caused by reactivation of latent varicella-zoster virus. After primary
chickenpox infection, the virus lies dormant in the dorsal root ganglia of the spinal cord. When reactivated, it
travels along the sensory nerve to affect one or more dermatomes, causing the characteristic shingles rash. It
can lead to complications like Ramsay Hunt syndrome characterised by unilateral facial palsy associated with
pain and herpetic blisters in the external auditory meatus due to involvement of the geniculate ganglion.
Incorrect
Marks for this submission: 0/3.
Question 116
Marks: 0/8
Research Methods

A. 2
B. 4
C. 5
D. 8
E. 10
F. 10%
G. 20%
H. 30%
I. 40%
J. 50%
K. 60%

A new antipsychotic drug, ‘Drug X’ has been introduced. A randomised controlled trial compared the efficacy of
new drug with chlorpromazine and also compared the side effect profile between the two. 500 patients were
in each arm receiving either drug X or chlorpromazine (CPZ). 200 patients showed clinical response in the drug
X-group. 300 patients recovered in the chlorpromazine group.

100 patients on new treatment X got extrapyramidal side effects while 50 patients in chlorpromazine group got
EPSEs.

Calculate the following:

EER for clinical response in drug-x group


CER for clinical response in chlorpromazine group
Absolute Risk Reduction using Drug X
Number needed to treat for Drug X
EER for EPSEs in drug-x group
CER for EPSEs in chlorpromazine group
Absolute risk reduction (or Increase) using Drug X
Number needed to harm
Answers.

1. I. Experimental event rate(EER)=events/subjects in experimental group=200/500=0.4 or 40%


2. K. Control event rate(CER)=events/subjects in control group=300/500=0.6 or 60%
3. G. Absolute Benefit Decrease (ARR)=|CER-EER|=0.2 or 20%
4. C. Number needed to treat=1/ARR=1/0.2=5
5. G. EER for EPSE’s in drug X group=100/500=0.2 or 20%
6. F. CER for EPSE’s in Chlorpromazine group=50/500=0.1 0r 10%
7. F. Absolute Risk Increase (ARI)= |EER-CER|=0.2-0.1=0.1 or 10%
8. E. Number needed to harm= 1/ ARI = 1/0.1 = 10.

Incorrect
Marks for this submission: 0/8.
Question 117
Marks: 0/7
Psychological treatment in group settings
A. Cohesiveness
B. Conditioning
C. Counter-dependence
D. Dependence
E. Fight-flight
F. Free floating discussion
G. Interpreting transference
H. Pairing
I. Universality
J. Vicarious learning

Lead in: From the options above choose:

Two curative factors in group therapy.

Three factors that hinder working in groups.

Two factors that are found in psychodynamic groups.

Answers

1. A, I. Yalom identified 11 curative factors in groups including cohesiveness (i.e. a sense of belongingness) and
universality (a sense that symptoms/problems are not specific to one group member, but shared by many
others) the other factors are: Installation of hope, imparting of information, interpersonal learning, imitation of
adaptive behaviour, catharsis, corrective recapitulation of the family group, altruism, guidance, existential
factors, and social skills.
2. D, E, H. Bion identified several factors that hinder group work including: dependence and fight-flight
response & Pairing. A group holds one of three basic assumptions, which may differ from the beliefs of the
individual members. The basic assumption of dependence arises from the group's anxious need to depend
absolutely on someone (usually the therapist) to protect the members, satisfy all their desires and solve their
problems. This corresponds to Klein’s depressive position. Collective belief in an enemy who can be dealt with
only by attack or retreat forms the basic assumption of fight-flight. This corresponds to Klein’s paranoid-
schizoid position. The basic assumption of pairing defines the notion that some future event or person will
come from outside to solve all problems. For example, there may be hope for the pairing of two individual
members of the group that could lead to the “birth” of a new solution for the group’s problems. This
corresponds to Klein and Freud’s oedipal fantasy.
3. F, G. Free-floating discussions (group-analytic equivalent of free-association) and Interpreting transference
(along with interpretation of defences and resistances, archaic and primordial experiences) are important in
the context of psychodynamic groups. Foulkes also listed ‘group-specific processes’ as important factors.

Incorrect
Marks for this submission: 0/7.
Question 118
Marks: 0/4
Sampling Techniques

A. Cluster sampling
B. Convenience sampling
C. Matched random
D. Progressive sampling
E. Quota Sampling
F. Simple random sampling
G. Snowball
H. Stratified random sampling
I. Systematic sampling

Select the most appropriate sampling technique for each of the following:

A study in which the participants from only selected areas are included

Participants in a study are recruited from odd numbered houses

The following types of sampling there is an equal opportunity for being selected

Answers:

1. A. Cluster sampling divides the population into groups, or clusters. A number of clusters are selected
randomly to represent the population, and then all units within selected clusters are included in the sample. No
units from non-selected clusters are included in the sample. They are represented by those from selected
clusters
2. I. In systematic sampling the participants are selected at regular intervals from a sampling frame. Using this
procedure each element in the population has a known and equal probability of selection.
3. F, I. In both systematic and simple random sampling there is an equal probability of being selected for every
person in the population.

Incorrect
Marks for this submission: 0/4.

January 2009
1
Marks: 0/1
The following is true regarding offending behavior in patients with learning disabilities
Choose one answer.
a. Fire setting is the most common offense
b. People with LD are more likely to commit violent offences

Error! Not a valid embedded


c. prevalence of learning disability in prisons was about 40%
object.
Error! Not a valid embedded
d. Reoffending is less
object.
Error! Not a valid embedded
e. It is overrepresented by borderline and mild LD.
object.
Offending in LD is overrepresented in mild and borderline groups, they are more likely to reoffend and
arson is less common in LD offenders. The prevalence is about 9.4%
Hayes et al., The prevalence of intellectual disability in a major UK prison, British Journal of Learning
Disabilities
Volume 35 Issue 3, Pages 162 - 167
Incorrect
Marks for this submission: 0/1.
Question 2
Marks: 0/1
A 36 year gentleman who is a heavy user of opioids currently undergoing detoxification develops low
blood pressure. Which of the following medications is the most likely cause of this?
Choose one answer.
Error! Not a valid embedded object. a. Buprenorphine
Error! Not a valid embedded object. b. Methadone
Error! Not a valid embedded object. c. Clonidine
Error! Not a valid embedded object. d. Buscopan
Error! Not a valid embedded object. e. Lorazepam
Clonidine works by stimulating a2 receptors in the brain which decreases cardiac output and peripheral
vascular resistance, lowering blood pressure. It has specificity towards the presynaptic alpha-2
receptors in the vasomotor center in the brainstem.
Keith Parker; Laurence Brunton; Goodman, Louis Sanford; Lazo, John S.; Gilman, Alfred (2006).
Goodman & Gilman's the pharmacological basis of therapeutics. New York: McGraw-Hill. pp. 854-
855
Incorrect
Marks for this submission: 0/1.
Question 3
Marks: 0/1
You see a 8 year old child with hyperkinetic disorder and no other comorbidities. The first line of
treatment that you would consider taking into account the current evidence base is
Choose one answer.
Error! Not a valid embedded object. a. Dexamphetamine
Error! Not a valid embedded object. b. Clonidine
Error! Not a valid embedded object. c. Atomoxetine
Error! Not a valid embedded object. d. Imipramine
Error! Not a valid embedded object. e. Methylphenidate
NICE guideline recommend the use of methylphenidate as a first line medical treatment for ADHD
with no other co-morbidities. Some patients who do not respond to methylphenidate will respond to
dexamphetamine and vice versa. Dexamphetamine is licensed for use in children over 3 years for
narcolepsy and as adjunct in the management of refractory hyperkinetic states in children (under
specialist supervision).
Nice guidelines for ADHD,2008
Incorrect
Marks for this submission: 0/1.
Question 4
Marks: 0/1
The following are correct about school refusal except
Choose one answer.
Error! Not a valid embedded
a. Parents are aware of the absence
object.
Error! Not a valid embedded
b. severe emotional distress about attending school
object.
Error! Not a valid embedded
c. Absence of significant anti social behavior
object.
Error! Not a valid embedded d. child often persuades parents to be allowed to stay at home
object.
Error! Not a valid embedded
e. Child does not comply with completing home work
object.
School refusal: Severe emotional distress about attending school; may include anxiety, temper
tantrums, depression, or somatic symptoms, Parents are aware of absence; child often tries to persuade
parents to allow him or her to stay home, Absence of significant antisocial behaviors such as juvenile
delinquency, During school hours, child usually stays home because it is considered a safe and secure
environment, Child expresses willingness to do schoolwork and complies with completing work at
home.
WANDA P. FREMONT. School Refusal in Children and Adolescents AFP Oct 2003
Incorrect
Marks for this submission: 0/1.
Question 5
Marks: 0/1
Shaping as used in behavior therapy for children is
Choose one answer.
Error! Not a valid embedded a. Differential positive reinforcement of successive
object. approximations
Error! Not a valid embedded
b. Use of Punishment to shape adaptive behaviors
object.
Error! Not a valid embedded c. Differential negative reinforcement of approximate behaviors
object.
Error! Not a valid embedded
d. Use of treats to get desired response
object.
Error! Not a valid embedded e. Reinforcing positive behaviors and ignoring negative behaviors
object.
Shaping is defined as Differential positive reinforcement of successive approximations. The successive
approximations reinforced are increasingly accurate approximations of a response desired. As training
progresses the trainer stops reinforcing the less accurate approximations.
""
Incorrect
Marks for this submission: 0/1.
Question 6
Marks: 0/1
A 34 year old man who is dependent on opioids is interested in replacement therapy to help become
abstinent from illicit opioid use. You would recommend which of the following medication?
Choose one answer.
Error! Not a valid embedded object. a. Lofexidine
Error! Not a valid embedded object. b. Dihydrocodeine
Error! Not a valid embedded object. c. Naltrexone
Error! Not a valid embedded object. d. Clonidine
Error! Not a valid embedded object. e. Buprenorphine
The medications approved for use in sustitution therapy for Opiod dependence is methadone or
buprenorphine.
""
Incorrect
Marks for this submission: 0/1.
Question 7
Marks: 0/1
Which of the following is not true of self injurious behaviour in individuals with learning disabilities
Choose one answer.
Error! Not a valid embedded object. a. higher in those with co-morbid epilepsy
Error! Not a valid embedded object. b. More in older adults
Error! Not a valid embedded object. c. more common in males
Error! Not a valid embedded object. d. high in those with hearing and visual impairment
Error! Not a valid embedded object. e. Inversely proportional to IQ
It is more common in young adults ,impairment with mobilty and communication and autism.
Collacot RA, Epidemiology of self injurious behaviour in adults with learning disabilites.BJPsych
1998;173:428-432
Incorrect
Marks for this submission: 0/1.
Question 8
Marks: 0/1
The genetic inheritance of Huntington's disease shows the following pattern
Choose one answer.
Error! Not a valid embedded object. a. single autosomal recessive
Error! Not a valid embedded object. b. single X linked recessive
Error! Not a valid embedded object. c. multiple autosomal recessive
Error! Not a valid embedded object. d. multiple autosomal dominant
Error! Not a valid embedded object. e. single autosomal dominant with high penetrance
The disease is proven to be associated with single autosomal dominant gene with a virtually 100%
manifestation and affects half of the offsprings
Lishman WA, Textbook of organic psychiatry p465-473
Incorrect
Marks for this submission: 0/1.
Question 9
Marks: 0/1
Which of the following is not a significant variable predictor of Sexual Offence Recidivism in
Offenders with Intellectual Disabilities
Choose one answer.
Error! Not a valid embedded object. a. allowances made by staff
Error! Not a valid embedded object. b. antisocial attitude
Error! Not a valid embedded object. c. poor relationship with mother
Error! Not a valid embedded object. d. acceptance of crime
Error! Not a valid embedded object. e. sexual abuse in childhood
Significant variables were allowances made by staff, antisocial attitude, poor relationship with mother,
denial of crime, sexual abuse in childhood, erratic attendance and poor response to treatment. Certain
variables, found significant in earlier studies, did not emerge in the current analysis. These predictors
accounted for around 53% of the variance for evidence of re-offending and around 74% of the variance
for suspicion of re-offending
William R. Lindsay, Susanne F. Elliot and Arlene Astell ,Predictors of Sexual Offence Recidivism in
Offenders with Intellectual Disabilities.Journal of Applied Research in Intellectual Disabilities 2004, 17
(4); 299 - 305
Incorrect
Marks for this submission: 0/1.
Question 10
Marks: 0/1
Which of the following drugs is most common cause of Syndrome of inapproprite antidiuretic hormone
secretion
Choose one answer.
Error! Not a valid embedded object. a. Lorazepam
Error! Not a valid embedded object. b. Risperidone
Error! Not a valid embedded object. c. Lamotrigine
Error! Not a valid embedded object. d. Olanzapine
Error! Not a valid embedded object. e. Citalopram
Most common cause is idiopathic however the drugs that can cause it include: - Exogenous
vasopressin, nonsteroidal anti-inflammatory drugs, nicotine, diuretics, chlorpropamide, Carbamazepine,
tricyclic antidepressants, SSRIs, vincristine, thioridazine, cyclophosphamide, clofibrate. Among
Psychotropics, SSRIs are the most implicated, though all antidepresants and antipsychotics are
associated with SIADH
Adrogue HJ, Madias NE. Hyponatremia. N Engl J Med. May 25 2000;342(21):1581-9
Incorrect
Marks for this submission: 0/1.
Question 11
Marks: 0/1
The following signs / symptoms are least likely to be associated with Opioid withdrawal
Choose one answer.
Error! Not a valid embedded object. a. Tachycardia
Error! Not a valid embedded object. b. Yawning
Error! Not a valid embedded object. c. Insomnia
Error! Not a valid embedded object. d. Weakness
Error! Not a valid embedded object. e. Pupillary constriction
Grade 1: Yawning, Sweating, Lacrimation (tearing), Rhinorrhea (runny nose) Grade 2: Mydriasis
(dilated pupils), Piloerection (goose bumps), Muscle twitching, Anorexia Grade 3: Insomnia, Increased
pulse, Increased respiratory rate, Elevated blood pressure, Abdominal cramps, Vomiting, Diarrhea,
Weakness
""
Incorrect
Marks for this submission: 0/1.
Question 12
Marks: 0/1
The most likely complication in a neonate born to a mother who took Paroxetine from the 5th month of
pregnancy is
Choose one answer.
Error! Not a valid embedded object. a. Ventricular septal defect
Error! Not a valid embedded object. b. Ebsteins anomaly
Error! Not a valid embedded object. c. Irritability
Error! Not a valid embedded object. d. Cleft lip
Error! Not a valid embedded object. e. Pyloric stenosis
Paroxetine if taken during the first Trimester leads to fetal heart defects and if taken later in pregnancy
lead to a withdrawal symptoms in the neonate
The Maudsley Prescribing guidelines, 9th edition, Page 368
Incorrect
Marks for this submission: 0/1.
Question 13
Marks: 0/1
According to NINCDS-ARDA criteria, A diagnosis of Alzheimer's dementia is unlikely if
Choose one answer.
Error! Not a valid embedded a. Onset is only after forty years of age
object.
Error! Not a valid embedded
b. plateaus in course of progression
object.
Error! Not a valid embedded
c. Progressive deterioration of specific cognitive function
object.
Error! Not a valid embedded d. there is gait disturbance at the onset or very early in the course
object.
Error! Not a valid embedded
e. Non specific EEG changes
object.
Diagnosis is unlikely if there is sudden apoplectic onset, focal neurological signs and incoordination
early in the course, seizures or gait disturbances at the onset or very early in the course
Heston et al ,1981,Dementia of Alzheimer's type .Clinical genetics, natural history and associated
conditions. Archives of general psychiatry 38, 1085-1090
Incorrect
Marks for this submission: 0/1.
Question 14
Marks: 0/1
You are counseling a mother who has one child with autism and is expecting her second. She wishes to
know the risk of autism in this second child. You say
Choose one answer.
Error! Not a valid embedded object. a. 40-50%
Error! Not a valid embedded object. b. 50-60%
Error! Not a valid embedded object. c. 10-20%
Error! Not a valid embedded object. d. 0-10%
Error! Not a valid embedded object. e. 30-40%
Studies have shown that the risk of autism in siblings is 2-3 %,which 50-100 fold increase over the
population rates.
New Oxford textbook of psychiatry,Vol2,p-1726
Incorrect
Marks for this submission: 0/1.
Question 15
Marks: 0/1
A woman lost her mother recently. A feature which would take your diagnosis towards depression
rather than normal grief reaction
Choose one answer.
Error! Not a valid embedded object. a. Irritability
Error! Not a valid embedded object. b. Loss of appetite
Error! Not a valid embedded object. c. Severe insomnia
Error! Not a valid embedded object. d. Hearing the voice of the deceased
Error! Not a valid embedded object. e. Generalized guilt
Intense feelings of guilt not related to the bereavement, thoughts of suicide or a preoccupation with
dying, feelings of worthlessness, marked psychomotor retardation, prolonged or severe inability to
function (not able to work, socialise or enjoy any leisure activity), prolonged hallucinations of the
deceased, or hallucinations unrelated to the bereavement are all factors indicative of depression.
""
Incorrect
Marks for this submission: 0/1.
Question 16
Marks: 0/1
The most common side-effect of Rivastigmine is
Choose one answer.
Error! Not a valid embedded object. a. Somnolence
Error! Not a valid embedded object. b. Nausea
Error! Not a valid embedded object. c. Sedation
Error! Not a valid embedded object. d. Dizziness
Error! Not a valid embedded object. e. Constipation
Excess cholinergic stimulation leads to GI side effects. Nausea is the most commonly reported adverse
event (11%-47%); vomiting, the next most common (10%-31%); diarrhea, next in order of frequency
(5%-19%); and anorexia, the least common (reported in 4%-17%). The package insert for rivastigmine
instructs clinicians to monitor patients for vomiting and weight loss when using this agent. Other
adverse events occurring more commonly with ChE-Is than placebo include insomnia, abnormal
dreams, incontinence, muscle cramps, bradycardia, syncope, and fatigue. ChE-Is are relatively
contraindicated in patients with bradycardia, sick-sinus syndrome, active peptic ulcer disease, or severe
asthma.
""
Incorrect
Marks for this submission: 0/1.
Question 17
Marks: 0/1
What rating scale would a health visitor use in a woman who had given birth recently to screen her for
possible symptoms of depression?
Choose one answer.
Error! Not a valid embedded object. a. EPDS
Error! Not a valid embedded object. b. MADRS
Error! Not a valid embedded object. c. BPRS
d. EAT
e. HAMD
The Edinburgh postnatal depression rating scale is a 10 item self report scale to screen for postnatal
depression in the community and is less sensitive to monitor response to treatment. The Morgan Russel
scale is a widely used measure of outcome for anorexia nervosa consisting of two scores: an average
outcome score and a general outcome score (possible total of 12). The average outcome score is based
on the outcome in five areas: nutritional status, menstrual function, mental state, sexual adjustment, and
socioeconomic status. The Zung Self-Rating Depression Scale is a 20-item self-report questionnaire
that is widely used as a screening tool, covering affective, psychological and somatic symptoms
associated with depression. The questionnaire takes about 10 minutes to complete, and items are
framed in terms of positive and negative statements.
Detection of postnatal depression: development of the 10- item Edinburgh Postnatal Depression Scale.
British Journal of Psychiatry, 150,782-876
Incorrect
Marks for this submission: 0/1.
Question 18
Marks: 0/1
The last stage according to Prochaska and Diclemente model of change of behavior is
Choose one answer.
a. Action
b. Precontemplation
c. preparation
d. Maintenance
e. contemplation
In the last stage maintenance stage the new behaviors such as abstinence are perpetuated.
Handbook of Alcoholism,2000, Zernig et al, P-99-100
Incorrect
Marks for this submission: 0/1.
Question 19
Marks: 0/1
A 40 year old man who is a heavy drinker does not recognise that he requires to change , does not
accept his problem drinking. Regarding his motivation to change he is in the stage of
Choose one answer.
a. Precontemplation
b. Contemplation
c. Preparation
d. maintenance
e. Action
At the precontempaltion stage there is no intention to change and patients are not even aware of their
problem. Secret drinking is seen in this stage.
Handbook of Alcoholism,2000, Zernig et al, P-99-100
Incorrect
Marks for this submission: 0/1.
Question 20
Marks: 0/1
According to current evidence base, the treatment of choice for pathological crying is:
Choose one answer.
a. Citalopram
b. Reboxetine
c. Venlafaxine
d. Duloxetine
e. Mirtrazepine
Pathological laughing and crying (PLC) is characterized by frequent, brief, intense paroxysms of
uncontrollable crying and/or laughing due to a neurological disorder. When sufficiently frequent and
severe, PLC may interfere with the performance of activities of daily living, interpersonal functioning,
or both, and is a source of distress for affected patients and their families. SSRIs are recommended as
first-line pharmacotherapy for this disorder. When SSRIs are ineffective or poorly tolerated, other
treatment options, including TCAs, noradrenergic reuptake inhibitors, novel antidepressants,
dopaminergic agents and uncompetitive NMDA receptor antagonists may be useful second-line
treatments. Current evidence suggests the use of Citalopram or Fluoxetine as first line drugs. There is
also evidence for TCAs like Amitryptline.
Pretzel et al. Pathological Laughing and Crying: Epidemiology, Pathophysiology and Treatment. CNS
Drugs. 22(7):531-545, 2008.
Incorrect
Marks for this submission: 0/1.
Question 21
Marks: 0/1
Neuroimaging studies in patients with OCD show
Choose one answer.
a. Reduced metabolism in caudate nuclei
b. Hypermetabolism in left orbital gyrus and both caudate nuclei
c. Asymmetries in blood flow in limbic system
d. Reduced metabolism in left orbital gyrus
e. Hypermetabolism of parahippocampal gyrus
PET studies have shown significant increases in metabolism in left orbital gyrus and both caudate
nuclei which also improved with drug treatment. OCD symptoms are mediated by hyperactivity in
orbitofrontal-subcortical circuits, perhaps due to an imbalance of tone between direct and indirect
striato-pallidal pathways.
Organic Psychiatry,Lishman,3 edn,p-145
Incorrect
Marks for this submission: 0/1.
Question 22
Marks: 0/1
NINCDS - ADRDA criteria for diagnosis of Alzheimer's disease does not include
Choose one answer.
a. deficits in two or more areas of cognition
b. no loss of consciousness
c. onset between 40 -90 years
d. progressive worsening of memory and other cognitive functions
e. Absence of systemic disorders
The criteria includes deficits in 2 or more areas of cognition along with dementia syndrome established
by clinical examination and MMSE
Organic Psychiatry,Lishman,3 rd edn,p-439
Incorrect
Marks for this submission: 0/1.
Question 23
Marks: 0/1
Which of the following is most common in delirium?
Choose one answer.
a. Delusions
b. Disturbed sleep wake cycle
c. Labile mood
d. Increased motor activity
e. Hallucinations
Common features of a delirious state are clouding of consciousness, altered sleep wake cycle,
agitation/overactive state or underactive/drowsy state and impaired cognition
Textbook of Organic Psychiatry
Incorrect
Marks for this submission: 0/1.
Question 24
Marks: 0/1
The mechanism of action of Rivastigmine is
Choose one answer.
a. irreversible inhibition of the enzyme acetylcholinesterase
b. NMDA antagonism
c. reversible inhibition of the enzyme acetylcholinesterase
d. reversible inhibition of the enzyme acetylcholinesterase and butryl cholinesterase
e. reversible inhibition of the enzyme acetylcholinesterase and butryl cholinesterase
Donezepil and Galantamine are reversible ACHE inhibitors while Rivastigmine is pseudo-reversible
inhibitor of both ACHE and BCHE. Memantine is a NMDA antagonist.
""
Incorrect
Marks for this submission: 0/1.
Question 25
Marks: 0/1
You see a 12 year old boy with Tourette's syndrome, who also has hyperkinetic disorder. The most
effective treatment for this boy would be
Choose one answer.
a. Pimoline
b. A NARI
c. A TCA
d. Risperidone
e. A central a agonist
NICE guidelines recommend the use of Methylphenidate or Atomoxetine (which is a NARI) in the
treatment of ADHD and comorbid Tics.
Nice guidelines for ADHD,2008
Incorrect
Marks for this submission: 0/1.
Question 26
Marks: 0/1
According to ICD-10 the following is not a feature of Dissociative disorder
Choose one answer.
a. Convincing association between onset and need
b. Amnesia can be too extensive to be explained by intentional simulation
c. Can present as loss of awareness of immediate surroundings
d. No temporal relationship between the onset and stressful events
e. Physical disorder may be present giving rise to other symptoms
ICD10 says that there are convincing associations in time between the onset of symptoms of the
disorder and stressful events, problems or needs.
ICD 10 WHO 1992
Incorrect
Marks for this submission: 0/1.
Question 27
Marks: 0/1
A 36 year old lady has been frequently seeing her dentist with pain and tingling sensation around her
jaw for the last two years. On each occasion, the examination did not reveal any abnormality. The most
likely diagnosis in this lady is
Choose one answer.
a. Delusional disorder
b. Dissociative disorder
c. Hypochondriasis
d. Somatoform disorder
e. Depression with psychotic symptoms
This scenario indicates a somatoform pain disorder wherein the predominant complaint is of persistent,
severe, and distressing pain, which cannot be explained fully by a physiological process or a physical
disorder, and which occurs in association with emotional conflict or psychosocial problems that are
sufficient to allow the conclusion that they are the main causative influences. The result is usually a
marked increase in support and attention, either personal or medical.
ICD-10 WHO 1992
Incorrect
Marks for this submission: 0/1.
Question 28
Marks: 0/1
Which of the following is the most common feature in delirium?
Choose one answer.
a. Hallucinations
b. Increased motor activity
c. Labile mood
d. Disturbed sleep wake cycle
e. Delusions
""
Gelder MG, Lopez-Ibor Jr, Andreasen N. New Textbook of Psychiatry. Oxford university press. Page.
1633
Incorrect
Marks for this submission: 0/1.
Question 29
Marks: 0/1
Which of the following psychotherapies utilises reciprocal role reversal
Choose one answer.
a. Moreno's pscyhodrama
b. Schema focused therapy
c. Dialectical behavioral therapy
d. Alums group therapy
e. Strategic family therapy
Two forms used in Psychodrama:1.Reciprocal (proper,classical,in situ) role reversal, based on social
psychology, is used mainly as an aid for dealing with people in the outer world, as a way of correcting
biased perceptions of other people and receiving feedback of oneself and as an interpersonal conflict
resolution technique.2. Representational(incomplete) role reversal, based on object relations theory, is
used more as an aid for the externalisation and interpolation of the inner world of one protagonist
Moreno, J.L. and Moreno, Z.T. (1959) Psychodrama, vol. 2, New York: Beacon House.
Incorrect
Marks for this submission: 0/1.
Question 30
Marks: 0/1
Asperger's syndrome differs from childhood autism in
Choose one answer.
a. Motor milestones
b. Reciprocal social interactions
c. Non-verbal communication
d. Restricted, repetitive interests
e. Language development
Unlike in autism, intellectual ability and syntactical speech are normal in Aspergers syndrome
Tom Berney. Asperger syndrome from childhood into adulthood. Advan. Psychiatr. Treat., Sep 2004;
10: 341 - 351
Incorrect
Marks for this submission: 0/1.
Question 31
Marks: 0/1
A 70 year old lady was admitted to general hospital being severely unwell.Two days later she began
complaining to her relatives that staff are not treating her well and poisoning her food. She also reports
seeing spiders crawling under the bed, most possibly placed there by the Nurses. the most likely
diagnosis is:
Choose one answer.
a. Delirium
b. Schizophrenia
c. Dissociative state
d. Delusional disorder
e. Dementia
Increasing old age, dementia, visual impairment and severe medical illness are important risk factors
for delirium. An acute onset, elderly and concurrent severe medical illness suggests delirium in this
case.
The prevention, diagnosis and management of delirium in older people. National Guidelines (June
2006). Clinical Effectiveness and Evaluation Unit. Royal College of Physicians
Incorrect
Marks for this submission: 0/1.
Question 32
Marks: 0/1
The following psychotropic medication is excreted mostly through the kidneys and is considered as
unsafe in a person with severe renal failure
Choose one answer.
a. Amisulpiride
b. Quetiapine
c. Clozapine
d. Olanzapine
e. Haloperidol
Amisulpiride is mainly excreted by the kidneys and has minor hepatic metabolism. The Maudsley
guidelines have advocated against the use of Sulpiride and Amisulpiride in some one with renal
impairment. They advise to use Haloperidol (if typical antipsychotics are to be used) and Olanzapine
(Atypical Antipsychotics are to be used).
The Maudsley prescribing guidelines. 9th Edition, 2007. Page no. 399. Bergemann et al. Plasma
amisulpride levels in schizophrenia or schizoaffective disorder ,European Neuropsychopharmacology
Volume 14, Issue 3, May 2004, Pages 245-250
Incorrect
Marks for this submission: 0/1.
Question 33
Marks: 0/1
The risk of alcoholism in children of Alcoholic parents compared to children of non-alcoholic parents
is
Choose one answer.
a. same
b. 6 times
c. 4 times
d. 2 times
e. 8 times
Though the most popularly reported risk of Alcoholism in Children of Alcoholic parents is 4 times the
risk in children of non-alcoholic parents, this does not adjust for risk of other illicit drug dependence. A
recent study which has adjusted for the risk of other illicit drug dependence has found the risk to be
around 2-3 times higher in children of acloholic parents compared to children of non-acloholic parents
Marmorstein et al. Alcohol and illicit drug dependence among parents: associations with offspring
externalizing disorders. Psychological Medicine (2009), 39:149-155
Incorrect
Marks for this submission: 0/1.
Question 34
Marks: 0/1
The ICD-10 diagnosis for conversion disorder includes the following
Choose one answer.
a. Resolution of symptoms within 6 months
b. Severe pain is a predominant symptom
c. Response to abreaction
d. Some clue that the person is feigning their symptoms
e. Subtle abnormality on neurological examination
The common themes that are shared by dissociative or conversion disorders are a partial or complete
loss of the normal integration between memories of the past, awareness of identity and immediate
sensations, and control of bodily movements. All types of dissociative disorders tend to remit after a
few weeks or months, particularly if their onset is associated with a traumatic life event. More chronic
disorders, particularly paralyses and anaesthesias, may develop if the onset is associated with insoluble
problems or interpersonal difficulties. There is evidence that the loss of function is an expression of
emotional conflicts or needs. The symptoms may develop in close relationship to psychological stress,
and often appear suddenly. Only disorders of physical functions normally under voluntary control and
loss of sensations are included here. Disorders involving pain and other complex physical sensations
mediated by the autonomic nervous system are classified under somatization disorder
ICD-10 WHO 1992
Incorrect
Marks for this submission: 0/1.
Question 35
Marks: 0/1
The first line of treatment of an 8 year old child with hyperkinetic disorders without any co-morbidities
but severe impairment is
Choose one answer.
a. Clonidine
b. Cognitive behavioral therapy
c. Methylphenidate
d. Atomoxetine
e. Dexamphetamine
Methylphenidate for ADHD without significant comorbidity, methylphenidate for ADHD with
comorbid conduct disorder, methylphenidate or atomoxetine in the presence of tics, Tourette's
syndrome, anxiety disorder, stimulant misuse or risk of stimulant diversion, atomoxetine if
methylphenidate has been tried and has been ineffective at the maximum tolerated, dose, or the child or
young person is intolerant to low or moderate doses of methylphenidate.
NICE Guidelines Sept 2008
Incorrect
Marks for this submission: 0/1.
Question 36
Marks: 0/1
The prevalence of psychotic disorders in prisons compared to the general population is
Choose one answer.
a. 20 times
b. 30 times
c. 10 times
d. 40 times
e. 50 times
The prevalence of psychotic disorders in prison is between 2-15% and approximately 20 times greater
than that for adults in the general population
New Oxford textbook of Psychaitry,Vol II,p-204
Incorrect
Marks for this submission: 0/1.
Question 37
Marks: 0/1
Which one of the following is recognised to cause schizophrenia like psychosis
Choose one answer.
a. Generalised anxiety disorder
b. Obsessive compulsive disorder
c. Huntington's disease
d. Heroin dependence
e. Panic disorder
Huntington's disease can present with florid schizophrenic symptoms and can be present even before
the true diagnosis can become apparent.
Organic psychaitry,Lishman,3 edn,p-468
Incorrect
Marks for this submission: 0/1.
Question 38
Marks: 0/1
Which of the following cognitive function is less impaired in patients with schizophrenia?
Choose one answer.
a. Slowed attention and processing
b. Sequencing
c. Visuospatial functions
d. Verbal memory
e. Immediate and delayed recall
Language, motor and visuospatial functions are less consistently impaired.
Bilder, R. M., Goodman, R. S., Robinson, D., et al (2000) Neuropsychology of first-episode
schizophrenia: initial characterisation and clinical correlates. American Journal of Psychiatry, 157, 549-
559
Incorrect
Marks for this submission: 0/1.
Question 39
Marks: 0/1
The Scale that is used to measure the severity of EPSE is
Choose one answer.
a. AIMS
b. Simpson Angus scale
c. BPRS
d. Barnes Akathisia scale
e. MADRS
Simpson Angus Scale is used to measure severity of EPSE, AIMS is used measure Tardive dyskinesia
SIMPSON & ANGUS, A RATING SCALE FOR EXTRAPYRAMIDAL SIDE EFFECTS, Acta
Psychiatrica Scandinavia
Volume 45 Issue S212, Pages 11 - 19
23 Aug 2007
Incorrect
Marks for this submission: 0/1.
Question 40
Marks: 0/1
A 63 year old inpatient has developed deviation of mouth and is drooling saliva from the corner of his
mouth. He is still able to frown his forehead.He has recently completed a course of antibiotics for a
middle ear infection. what is the likely underlying pathology
Choose one answer.
a. Hypoglossal
b. Facial nerve (lower motor neuron lesion)
c. Trigeminal (motor nuclei)
d. Trigeminal (sensory)
e. Facial nerve palsy (Upper motor neuron lesion)
In a LMN lesion the forehead is paralysed - the final common pathway to the muscles is destroyed;
whereas the upper facial muscles are partially spared in an UMN lesion because of alternative pathways
in the brainstem . Other useful lab investigations include audiogram, CT or MRI.Nerve conduction are
helpful.
""
Incorrect
Marks for this submission: 0/1.
Question 41
Marks: 0/1
Which of the following is now regarded as the standard screening test for alcohol use with a high
sensitivity and specificity?
Choose one answer.
a. AUDIT
b. ALC-M
c. CAGE
d. MAST
e. SAD-Q
Although they are not a substitute for a careful clinical interview, a range of self-administered
questionnaires can be used to screen for heavy drinking and alcohol-use disorders in clinical settings.
The shortest of the most widely-used instruments is the CAGE questionnaire, which is an acronym for
whether a patient has ever felt the need to Cut down on drinking; felt Annoyed when criticised about
alcohol use; felt Guilty about drinking, or ever needed an Eye-opener on awakening. Results vary
across different subgroups (with highest accuracy in men and white people). The cut-off score of two of
a possible four positive responses has a sensitivity of between 53% (in heavy drinkers) and 77% in
patients who have alcohol dependence, with specificities of 80% or higher. This short test might
operate best in medical and surgical settings, especially when combined with blood tests for heavy
drinking.
Another questionnaire is the ten-item version of the Michigan Alcohol Screening Test, in which five to
six affirmative answers indicate a possible alcohol-use disorder (with sensitivity and specificity of
about 80%), and seven indicates a probable alcohol-use disorder.
Alcohol use disorders indentification test (AUDIT) is the standard and is brief 10 question test that can
be administered by non-alcohol specialist. It has a sensitivity and specificity of 92% and 93%
respectively, although a lower sensitivity has been reported in women and elderly people.
Four of the AUDIT items are used for the shorter fast alcohol screening test (FAST), which has similar
accuracy to the full AUDIT test.
Another short instrument is the TWEAK questionnaire, which asks about Tolerance, Worry about
drinking by friends, Eye-opener drinks in the morning, Amnesia about drinking, and feeling the need to
Cut down.
1. Parker et al. Diagnosis and Management of Alcohol use disorders. BMJ 2008;336:496-501 2.Colin
Drummond & Hamid Ghodse. Use of investigations in the diagnosis and management of alcohol use
disorders, Advances in Psychiatric Treatment (1999), vol. 5, pp. 366-375 3.Chick, J., Badawy, A. &
Borg, S. (1993) Identification of excessive drinking and alcohol problems. Alcohol and Alcoholism,
suppl. 2, 121-125.
Incorrect
Marks for this submission: 0/1.
Question 42
Marks: 0/1
A subcultural explanation rather than a biological one in learning disabilities is most accounted for by:
Choose one answer.
a. An even spread of LD across different socio-economic groups of the population
b. Facial dysmorphologies
c. Moderate LD
d. Mild LD
e. Parents with a child with LD have lower IQ
IQ levels follow a normal distribution curve for IQ's above 70. For lower levels of IQ the curve is
skewed due to the addition of pathological (organic) causes of learning disability. 'Organic' LD has
known Perinatal or chromosomal causes, often mod-severe LD (IQ less than 50) e.g.. Down's
syndrome. 'Sub-cultural' LD is the normal variant on the lower end of the Gaussian distribution of
intelligence.

Click here for referenceClick here for reference


Incorrect
Marks for this submission: 0/1.
Question 43
Marks: 0/1
Which of the following is less commonly seen in Pediatric Autoimmune Neuropsychiatric disorder
associated with Streptococcal infection
Choose one answer.
a. Obsessions
b. Separation anxiety
c. Motor tics
d. Joint pains
e. Hallucinations
PANDAS, is an abbreviation for Pediatric Autoimmune Neuropsychiatric Disorders Associated with
Streptococcal Infections. The term is used to describe a subset of children who have Obsessive
Compulsive Disorder (OCD) and/or tic disorders such as Tourette's Syndrome, and in whom symptoms
worsen following strep. infections such as "Strep throat" and Scarlet Fever. In conjunction they may
have symptoms of ADHD, Sleep disturbances, bed wetting ,gross motor disturbances. In addition to
these symptoms, children may also become moody, irritable or show concerns about separating from
parents or loved ones. It has also been described as a subtype of childhood-onset OCD. The children
usually have dramatic, "overnight" onset of symptoms, including motor or vocal tics, obsessions,
and/or compulsions. In addition to these symptoms, children may also become moody, irritable or show
concerns about separating from parents or loved ones. This abrupt onset is generally preceded by a
Strep. throat infection.
Swedo and Pekar 2000,PANDAS: a new species of childhood-onset obsessive compulsive disorder
APA
Incorrect
Marks for this submission: 0/1.
Question 44
Marks: 0/1
The following is true regarding Acetylcholinesterase inhibitors
Choose one answer.
a. Galantamine does not have action on the nicotinic receptors
b. Gingo biloba has the same efficacy as Donepezil in the treatment of Alzheimer's dementia
c. Memantine is an agonist at NMDA receptors
d. Rivastigmine is given as a once daily dose
e. Donepezil is metabolised by 2D6 and 3A4 enzymes of the cytochrome P450 system
Potential for drug interactions with Donezepil and galantamine are present due to them being
metabolised by cytochromoes 2D6 and 3A4. Rivastigmine has no potential drug interaction
Maudsley prescribing guidelines, 9th Edition, Page 416
Incorrect
Marks for this submission: 0/1.
Question 45
Marks: 0/1
A 22 year old lady presents with severe OCD with obsessions of dirt and compulsion of washing. She
is currently neglecting herself and though she is eating one meal a day, she has not been drinking for
fears of contamination. The best treatment for her would be
Choose one answer.
a. SSRI
b. Inpatient admission
c. SNRI
d. Outpatient ECT
e. Outpatient CBT
The lady presents with risk which are difficult to manage on an outpatient basis and it would be most
appropriate to admit this lady, treat her with either antidepressants and CBT. It may be appropriate to
consider ECT if the risk of neglect are high
""
Incorrect
Marks for this submission: 0/1.
Question 46
Marks: 0/1
The following disorder is found only in girls.
Choose one answer.
a. Tuberous sclerosis
b. Rett's syndrome
c. Friedrich Ataxia
d. Beckwith-Wiedemann syndrome
e. Angelman syndrome
Retts syndrome (ICD10):apparently normal early development followed by partial or complete loss of
speech and of skills in locomotion and use of hands, together with deceleration in head growth, usually
onset between 7 and 24 months of age. Characterstic include loss of purposive hand movements, hand
wringing stereotypies and hyperventilation.
ICD-10, WHO
Incorrect
Marks for this submission: 0/1.
Question 47
Marks: 0/1
The following neonatal outcomes are seen in newborns born to anorexic mothers
Choose one answer.
a. Large for gestational age
b. Large head circumference
c. Low birth weight
d. Low APGAR score
e. Post term delivery
Women with past or current eating disorders were at increased risk of hyperemesis and deliver infants
with significantly lower birth weight and smaller head circumference as compared with controls. They
were also at greater risk of delivering infants with microcephaly and small for gestational age infants
Koubaa et al. Pregnancy and neonatal outcomes in women with eating disorders.Obstetric Gynaecol
2005; 105(2):255-60.
Incorrect
Marks for this submission: 0/1.
Question 48
Marks: 0/1
A 15 year old girl presents with Anorexia nervosa. The current best evidence for treatment is
Choose one answer.
a. Interpersonal therapy
b. Cognitive behavioral therapy
c. Fluoxetine
d. Group CBT
e. Family therapy
The NICE guidance suggests that Family interventions that directly address the eating disorder should
be offered to children and adolescents with anorexia nervosa.
Core interventions in the management of Anorexia nervosa, Bulimia nervosa and other related eating
disorders. NICE 2004.
Incorrect
Marks for this submission: 0/1.
Question 49
Marks: 0/1
The percentage of exhibitionists without a previous conviction for sexual offences is
Choose one answer.
a. 80%
b. 40%
c. 20%
d. 60%
e. 50%
Sugarman et al (1994) found that 26 percent had at least one conviction for a contact sex offence.
Sugarman et al. Dangerousness in Exhibitionists. Journal of Forensic Psychiatry & Psychology, 5 (2)
1994; 287 - 296
Incorrect
Marks for this submission: 0/1.
Question 50
Marks: 0/1
One of the most evidence based treatment for reducing Tics in Tourette Syndrome is
Choose one answer.
a. Clonidine
b. Haloperidol
c. Atomoxetine
d. Buspirone
e. Clonidine
Besides Haloperidol, Sulpiride and Ziprasidone are known to be effective in controlling tics in
Tourette. Evidence also exists for Risperidone, Pimozide and Olanzapine. Other pharmacological
treatments for tics include: -agonists such as clonidine and guanfacine; botulinum toxin; calcium
antagonists such as nifedipine, flunarizine and verapamil; nicotine; and the selective androgen receptor
antagonist, flutamide. All received some support in open use for decreasing tics
Dave Coghill Current issues in child and adolescent psychopharmacology. Part 2: Anxiety and
obsessive-compulsive disorders, autism, Tourette's and schizophrenia.Advances in Psychiatric
Treatment (2003) 9: 289-299
Incorrect
Marks for this submission: 0/1.
Question 51
Marks: 0/1
What treatment would you use for a patient with severe symptoms of pre-menstrual syndrome?
Choose one answer.
a. SSRI's
b. Oligoacids
c. Progesterone
d. Danazol
e. Evening primrose oil
SSRI's have the strongest evidence for their effectiveness in the treatment of PMS.
Henshaw. PMS: Diagnosis, aetiology, assessment and management: REVISITING...
PREMENSTRUAL SYNDROME. Advan. Psychiatr. Treat., Mar 2007; 13: 139 - 146. Click here for
reference
Incorrect
Marks for this submission: 0/1.
Question 52
Marks: 0/1
When assessing a patient with the features of early dementia which of the following would suggest a
sub-cortical rather than a cortical cause?
Choose one answer.
a. Euthymic mood
b. Calculation preserved
c. Normal speed of cognitive processes
d. Absence of dysarthria
e. Mild aphasia
Subcortical dementias are more likely to affect attention,motivation,emotionality and personality.
People with subcortical dementia often show early symptoms of depression, clumsiness, irritability or
apathy.
The college website - example MCQ!
Incorrect
Marks for this submission: 0/1.
Question 53
Marks: 0/1
Poor prognostic factors for Anorexia nervosa include all the following except
Choose one answer.
a. Young age of onset
Error! Not a valid embedded object. b. Social difficulties
Error! Not a valid embedded object. c. Poor relationship with family
Error! Not a valid embedded object. d. Low BMI
Error! Not a valid embedded object. e. Personality difficulties
Younger age and longer hospital stay at first hospitalisation was associated with better outcome, and
psychiatric and somatic comorbidity worsened the outcome
Papadopoulos FC et al. Excess mortality, causes of death and prognostic factors in anorexia nervosa.
The British Journal of Psychiatry (2009) 194: 10-17
Incorrect
Marks for this submission: 0/1.
Question 54
Marks: 0/1
The following route of cocaine use leads to a onset of action lasting for about 20-30 minutes
Choose one answer.
Error! Not a valid embedded object. a. Intravenous
Error! Not a valid embedded object. b. Intramuscular
Error! Not a valid embedded object. c. Oral
Error! Not a valid embedded object. d. Intranasal
Error! Not a valid embedded object. e. Inhalation
Orally ingested cocaine has a bioavailability of 30 - 40% and a relatively slow onset of action of
approximately 15 minutes. Subcutaneous or intramuscular injection is not favored today as the
vasoconstrictor effect of the drug prevents a rapid effect. Intravenous injection is favoured by some
groups and produces an intense euphoria in 30 seconds, after the injection and the effect lasting for 10-
20 minutes.
Intranasal cocaine use generally produces euphoria in 3 - 5 minutes, with the effect lasting for 15-30
minutes.
Inhalation (smoking) produces an intense high in approximately 8 seconds and the effect lasts for about
5-10 minutes.

Pedro Ruiz, Eric C Strain, John Langrod. Cocaine. IN The Substance Abuse Handbook. Lippincott
Williams and Wilkins. 2007 page 42.
Incorrect
Marks for this submission: 0/1.
Question 55
Marks: 0/1
The current best evidence for the treatment of dementia associated with Parkinson's disease is
Choose one answer.
Error! Not a valid embedded object. a. Donepezil
Error! Not a valid embedded object. b. Galantamine
Error! Not a valid embedded object. c. Memantine
Error! Not a valid embedded object. d. Atypical antipsychotics
Error! Not a valid embedded object. e. Rivastigmine
A recent Cochrane review found the most evidence for Rivastigmine with 15% of patients showing
some improvement in cognitive functioning and activities of daily living
Maidment I et al., Cholinestrase inhibitors for Parkinsons disease dementia. Cochrane Database Syst
Rev 2006.
Incorrect
Marks for this submission: 0/1.
Question 56
Marks: 0/1
A39 year old female patient has had recurrent depressive episodes in the past and one episode of
hypomania. The best medication to prevent relapse for her is:
Choose one answer.
Error! Not a valid embedded object. a. Lithium
Error! Not a valid embedded object. b. Olanzapine
Error! Not a valid embedded object. c. Carbamazepine
Error! Not a valid embedded object. d. Sodium valproate
Error! Not a valid embedded object. e. Lamotrigine
According to NICE guidelines Lithium, Valproate or Olanzapine should be first line for Bipolar
affective disorder. However there is increasing evidence to suggest that Lamotrigine has antidepressant
properties and can be used in patients who suffer from more depressive episodes in Bipolar affective
disorder, as well patients with Bipolar II disorder. The lady in this question appears to have Bipolar II
disorder, and hence Lamotrigine would be a good choice (please note: You could have chosen Lithium,
Olanzapine and Depakote without being completely wrong)
Bipolar disorder. NICE guidelines 2006
Incorrect
Marks for this submission: 0/1.
Question 57
Marks: 0/1
Which of the following treatments for multiple sclerosis is most likely to lead to depression?
Choose one answer.
Error! Not a valid embedded object. a. Baclofen
Error! Not a valid embedded object. b. Glatiramer acetate
Error! Not a valid embedded object. c. Steroids
Error! Not a valid embedded object. d. Amantadine
Error! Not a valid embedded object. e. Interferon-beta
Use of interferon beta is associated with depression and suicidal thoughts; these conditions are also
more common with MS. Any symptoms of depression should be immediately reported - patients should
be made aware that depression and/or suicidal thoughts may occur with interferon beta. Caution is also
advised in patients with a history of epilepsy and those with heart problems.
Expert Opin Drug Saf. 2007 May;6(3):279-88.
Incorrect
Marks for this submission: 0/1.
Question 58
Marks: 0/1
The scale used to screen for cognitive impairment in people with Downs syndrome is
Choose one answer.
Error! Not a valid embedded object. a. CAMCOG
Error! Not a valid embedded object. b. MMSE
Error! Not a valid embedded object. c. CAMDEX
Error! Not a valid embedded object. d. DMR
Error! Not a valid embedded object. e. Mental retardation scale
Alzheimer's dementia is quite common in people with Downs syndrome over the age of 35, though the
traditional screening instruments used in adults without downs syndrome may be less useful in
identifying this. The three dementia screening instruments that are currently in use among people with
intellectual disabilities, namely the Dementia Scale for Down Syndrome (DSDS; Gedye, 1995) and the
Dementia Questionnaire for Persons with Mental Retardation (DMR; Evenhuis, 1992, 1996), and the
Dementia Screening Questionnaire for Individuals with Intellectual Disabilities (DSQIID) (deb et al.,
2007)
Deb et al. Dementia Screening Questionnaire for Individuals with Intellectual Disabilities. The British
Journal of Psychiatry (2007) 190: 440-444.
Incorrect
Marks for this submission: 0/1.
Question 59
Marks: 0/1
You are seeing the parents of a 26 year old gentleman currently under your care. He has been
diagnosed with Schizophrenia and his parents want to know his prognosis. What type of schizophrenia
has the best prognosis out of the following
Choose one answer.
Error! Not a valid embedded object. a. Simple Schizophrenia
Error! Not a valid embedded object. b. Residual Schizophrenia
Error! Not a valid embedded object. c. Hebephrenic Schizophrenia
Error! Not a valid embedded object. d. Paranoid Schizophrenia
Error! Not a valid embedded object. e. Catatonic Schizophrenia
Factors associated with good prognosis are:paranoid type,female,married,abrupt onset,family history of
affective disorder,lack of negative and cognitive symptoms.
Lieberman et al 1993 Current opinion in Psychiatry
Incorrect
Marks for this submission: 0/1.
Question 60
Marks: 0/1
The antidepressant of choice used in the treatment of moderate depression in a patient who has
Myocardial infarction is
Choose one answer.
Error! Not a valid embedded object. a. Fluoxetine
Error! Not a valid embedded object. b. Imipramine
Error! Not a valid embedded object. c. Paroxetine
Error! Not a valid embedded object. d. Sertraline
Error! Not a valid embedded object. e. Citalopram
When initiating antidepressant treatment in patients with recent MI or unstable angina ,sertraline is the
treatment of choice and has the best evidence
Nice guidelines for depression,2004
Incorrect
Marks for this submission: 0/1.
Question 61
Marks: 0/1
Masses negative treatment is used in which of the following conditions
Choose one answer.
Error! Not a valid embedded object. a. Conduct disorder
Error! Not a valid embedded object. b. ADHD
Error! Not a valid embedded object. c. Tics in tourettes disorder
Error! Not a valid embedded object. d. Emotional disorders
Error! Not a valid embedded object. e. Enuresis
Massed negative practice (MNP) is a proposed treatment for the tics of Tourette syndrome in which the
individual with Tourette's "practices" tics continuously until a conditioned level of fatigue is reached. It
is based upon the Hullian learning theory, which holds that tics are "maladaptive habits that are
strengthened by repetition and can be replaced by the strengthening of more adaptive habits (i.e., not
having tics)
Woods DW, Himle MB, Conelea CA. Behavior therapy: other interventions for tic disorders. Adv
Neurol. 2006;99:234-40.
Incorrect
Marks for this submission: 0/1.
Question 62
Marks: 0/1
A 20 year old man presents to the A&E with PCP intoxication. The signs / symptoms that he can
present with include
Choose one answer.
Error! Not a valid embedded object. a. Increased pain sensitivity
Error! Not a valid embedded object. b. Hypotonia
Error! Not a valid embedded object. c. Paranoid ideas
Error! Not a valid embedded object. d. Ataxia
Error! Not a valid embedded object. e. Simplex hallucinations
The criteria for PCP intoxicationA. Recent use of phencyclidine (or a related substance). B. Clinically
significant maladaptive behavioral changes (e.g. belligerence, assaultativeness, impulsiveness,
unpredictability, psychomotor agitation, impaired judgment, or impaired social or occupational
functioning) that developed during, or shortly after, phencyclidine use. C. Within an hour (less when
smoked, "snorted," or used intravenously), two (or more) of the following signs: vertical or horizontal
nystagmus, hypertension or tachycardia, numbness or diminished responsiveness to pain, ataxia,
dysarthria, muscle rigidity, seizures or coma & hyperacusis
DSM IV. APA 1994
Incorrect
Marks for this submission: 0/1.
Question 63
Marks: 0/1
The following statements are valid in the use Automatism as a defense for murder
Choose one answer.
Error! Not a valid embedded
a. Murder should have ideally occurred within 2 hours of sleep
object.
Error! Not a valid embedded b. The person can partially recollect the act on the following morning
object.
Error! Not a valid embedded
c. The person recollects the act as if it had occurred in his dream
object.
Error! Not a valid embedded d. The person can clearly recollect the act in the morning after the
object. murder
Error! Not a valid embedded e. The person wakes up in the middle of the night screaming and in a
object. panic during the act
Automatism can be used as defence for a criminal charge. There are two types of automatisms: sane
and insane. A sane automatism must arise from an external factor (such as a head injury) whereas an
insane automatism arises from some 'disease of the mind', including mental illness, somnambulism
epilepsy etc.
Characteristics of automatism include:
involuntary (subject has no control over it)
inappropriate to the circumstances, lacking in judgment, out of character
may be complex, purposeful, co-ordinated, and directed
afterwards no recollection, or partial and confused memory
if organic, there must be corresponding disturbance of brain function
if psychogenic, behavior is appropriate to the psychopathology
there is a clear sensorium but complete or severe amnesia
Somnambulism (Sleep walking): A state of altered consciousness in which phenomena of sleep and
wakefulness are combined. During a sleepwalking episode the individual arises from bed, usually
during the first third of nocturnal sleep, and walks about, exhibiting low levels of awareness, reactivity,
and motor skill. Upon awakening, there is usually no recall of the event.
""
Incorrect
Marks for this submission: 0/1.
Question 64
Marks: 0/1
What is not true about Vareniciline tartarate
Choose one answer.
Error! Not a valid embedded object. a. Only contraindication is hypersensitivity to the drug
Error! Not a valid embedded object. b. Indicated for smoking cessation
Error! Not a valid embedded object. c. It is a nictoinic receptor anatgonist
Error! Not a valid embedded object. d. helps prevent 'slips' into full relapse
Error! Not a valid embedded object. e. Associated with nausea in nearly 30%
Introduced in UK in Dec 2006 for smoking cessation and is a partial agonist with high affinity to
nicotinic receptor
Jorenby et al JAMA 2006 ;296:56-63
Incorrect
Marks for this submission: 0/1.
Question 65
Marks: 0/1
A 41 years old pharmacist presents with a history of disabling anxiety symptoms. He has recently been
made redundant from his job due to certain concerns from his employer. Which of the following
symptom is likely to point towards a benzodiazepine withdrawal than a functional anxiety state?
Choose one answer.
Error! Not a valid embedded object. a. Dysphoria
Error! Not a valid embedded object. b. nightmares
Error! Not a valid embedded object. c. night sweats
Error! Not a valid embedded object. d. Sensory hypersensitivity
Error! Not a valid embedded object. e. palpitations
Symptoms Less Common in Anxiety States but relatively specific to Benzodiazepine withdrawal
include Perceptual disturbances, sense of movement, Depersonalisation, derealisation, Formication,
seizures, tinnitus, delirium & muscle twitches.
Nutt DJ. Benzodiazepine dependence: new insights from basic research. In: Hindmarch I, Beaumont G,
Brandon S, Leonard BE, eds. Benzodiazepines: Current Concepts. New York: John Wiley & Sons;
1990:19-41
Incorrect
Marks for this submission: 0/1.
Question 66
Marks: 0/1
The following disorder results due to deletion of the chromosome of maternal origin
Choose one answer.
Error! Not a valid embedded object. a. Prader-willi syndrome
Error! Not a valid embedded object. b. Tuberous sclerosis
Error! Not a valid embedded object. c. Retts syndrome
Error! Not a valid embedded object. d. Angleman syndrome
Error! Not a valid embedded object. e. Patau syndrome
Angelman syndrome is due to the micodeletion in the long arm of the maternally derived chromosome
15, characterised by ataxia, epilepsy, paroxysm of laughter, absence of speech, microcephaly, severe
LD, and behavior problems.
Oxford handbook of Psychiatry, 2005, Page 704
Incorrect
Marks for this submission: 0/1.
Question 67
Marks: 0/1
The sign / symptom least likely to be experienced by a person taking MDMA is
Choose one answer.
Error! Not a valid embedded object. a. Paranoia
Error! Not a valid embedded object. b. Anxiety
Error! Not a valid embedded object. c. Increased empathy
Error! Not a valid embedded object. d. Hallucinations
Error! Not a valid embedded object. e. Increased energy
Ectasy (MDMA) is a stimulant drug, which can lead to increase in energy levels, enhanced feelings of
calm, love, confidence and alertness. It can lead to anxiety, depression, paranioa & panic attacks. It is
not a hallucinagen and is not known to cause Hallucinations.
""
Incorrect
Marks for this submission: 0/1.
Question 68
Marks: 0/1
The most common symptom that appears on withdrawal from Benzodiazepines is
Choose one answer.
Error! Not a valid embedded object. a. Seizures
Error! Not a valid embedded object. b. Hallucinations
Error! Not a valid embedded object. c. Insomnia
Error! Not a valid embedded object. d. Dizziness
Error! Not a valid embedded object. e. Delirium
Physiological dependence on benzodiazepines is accompanied by a withdrawal syndrome which is
typically characterized by sleep disturbance, irritability, increased tension and anxiety, panic attacks,
hand tremor, sweating, difficulty in concentration, dry wretching and nausea, some weight loss,
palpitations, headache, muscular pain and stiffness and a host of perceptual changes. Withdrawal from
normal dosage benzodiazepine treatment can result in a number of symptomatic patterns. The most
common is a short-lived "rebound" anxiety and insomnia, coming on within 1-4 days of
discontinuation, depending on the half-life of the particular drug. The second pattern is the full-blown
withdrawal syndrome, usually lasting 10-14 days; finally, a third pattern may represent the return of
anxiety symptoms which then persist until some form of treatment is instituted. Seizures can develop in
about 75% of patients while a delirium can develop in 2/3rds of people withdrawing.
Petursson, H. The benzodiazepine withdrawal syndrome ADDICTION 1994, VOL 89; NUMBER 11,
pages 1455. Lader M. Rebound and withdrawal with benzodiazepine and non-benzodiazepine hypnotic
medication. In Clinical Pharmacology of Sleep (eds. S.R. Pandi-Perumal and J.M. Monti ). Springer.
2006
Incorrect
Marks for this submission: 0/1.
Question 69
Marks: 0/1
Which of the following is less likely an outcome of childhood onset conduct disorder
Choose one answer.
Error! Not a valid embedded object. a. have less number of children themselves
Error! Not a valid embedded object. b. Substance misuse
Error! Not a valid embedded object. c. violence against women and children
Error! Not a valid embedded object. d. Psycopathic personality traits
Error! Not a valid embedded object. e. Mental health problems
In a follow up study,the childhood-onset delinquents were the most elevated on psychopathic
personality traits, mental-health problems, substance dependence, numbers of children, financial
problems, work problems, and drug-related and violent crime, including violence against women and
children. The adolescent-onset delinquents at 26 years were less extreme but elevated on impulsive
personality traits, mental-health problems, substance dependence, financial problems, and property
offenses. A third group of men who had been aggressive as children but not very delinquent as
adolescents emerged as low-level chronic offenders who were anxious, depressed, socially isolated, and
had financial and work problems.
Noffitt TE et al Males on the life-course-persistent and adolescence-limited antisocial pathways:
follow-up at age 26 years Dev Psychopathol.2002 Winter;14(1):179-207.
Incorrect
Marks for this submission: 0/1.
Question 70
Marks: 0/1
In the treatment of opiate withdrawal in a person having low BP the following is given
Choose one answer.
Error! Not a valid embedded object. a. Buprenorphine
Error! Not a valid embedded object. b. Naloxone
Error! Not a valid embedded object. c. Clonidine
Error! Not a valid embedded object. d. Dihydrocodeine
Error! Not a valid embedded object. e. Lofexidine
Buprenorphine or methadone are suggested as the first line in the opioid detoxification. Lofexidine is
less likely to cause hypotension compared to clonidine hence this could be the choice if the patient
prefers not to use methadone or Buprenorphine. NICE advocates that Clonidine or dihydrocodeine
should not be routinely used for opioid detoxification.
Nice guidelines Drug Misuse - Opioid detoxification, 2007
Incorrect
Marks for this submission: 0/1.
Question 71
Marks: 0/1
Which of the following anticonvulsant has the strongest evidence to suggest that it can be used to
augment clozapine in treatment-resistant schizophrenia
Choose one answer.
Error! Not a valid embedded object. a. Semi sodium valproate
Error! Not a valid embedded object. b. Phenobarbital
Error! Not a valid embedded object. c. Ethosuximide
Error! Not a valid embedded object. d. Lamotrigine
Error! Not a valid embedded object. e. Gabapentin
Lamotrigine alongwith Sulpiride and Amisulpride has evidence to be used as augmenting agents after
partial response with clozapine.
Barnes, T. R. E., McEvedy, C. J. B. & Nelson, H. E. (1996) Management of treatment resistant
schizophrenia unresponsive to clozapine. British Journal of Psychiatry, 169 (suppl. 31), 31-40
Incorrect
Marks for this submission: 0/1.
Question 72
Marks: 0/1
A 15 year old boy presents with a moderate depressive illness. He is otherwise fit and intelligent and
does not have suicidal ideas or plans. The treatment of choice for this boy would be
Choose one answer.
Error! Not a valid embedded object. a. SSRI
Error! Not a valid embedded object. b. CBT & SSRI
Error! Not a valid embedded object. c. CBT
Error! Not a valid embedded object. d. Venlafaxine
Error! Not a valid embedded object. e. TCA
Major controversies about prescribing of antidepressants in children and adolescents. TCAs,
Venlafaxine, and Paroxetine should not be used, and limits on use of SSRIs NICE recommends: Mild
depression? Antidepressant medication should not be used for the initial treatment of children and
young people with mild depression. Moderate to severe depression ? Children and young people with
moderate to severe depression should be offered, as a first-line treatment, a specific psychological
therapy (individual cognitive behavioral therapy [CBT], interpersonal therapy or shorter-term family
therapy; it is suggested that this should be of at least 3 months' duration). ? Antidepressant medication
should not be offered to a child or young person with moderate to severe depression except in
combination with a concurrent psychological therapy. Specific arrangements must be made for careful
monitoring of adverse drug reactions, as well as for reviewing mental state and general progress; for
example, weekly contact with the child or young person and their parent's) or carer's) for the first 4
weeks of treatment. In the event that psychological therapies are declined, medication may still be
given, but as the young person will not be reviewed at psychological therapy sessions, the prescribing
doctor should closely monitor the child or young person's progress on a regular basis and focus
particularly on emergent adverse drug reactions. There is limited evidence for the treatment of
depression esp. antidepressants in children - except for fluoxetine December 2003 - the CSM advised
for fluoxetine only - the balance of risks and benefits favorable (However specialist may prescribe
other SSRIs)
NICE guidance, 2005
Incorrect
Marks for this submission: 0/1.
Question 73
Marks: 0/1
Which of the following is a false statement regarding functional enuresis in children
Choose one answer.
Error! Not a valid embedded object. a. Cannot occur as a complication of a psychiatric illness
Error! Not a valid embedded object. b. Male to female ratio of 3:1
Error! Not a valid embedded object. c. Maybe nocturnal or diurnal in nature
Error! Not a valid embedded object. d. Parental attitude has no aetiological role
Error! Not a valid embedded object. e. Part of Helman's triangle for predicting future violence
Besides genetic factors, rigid toilet training, negative or indifferent attitudes of parents and stressful
events leading to anxiety are etiological factors. Helmans triangle includes Bedwetting, firesetting and
cruelty to animals.
Nocturnal enuresis in the adolescent: a neglected problem. British Journal of Urology.
Incorrect
Marks for this submission: 0/1.
Question 74
Marks: 0/1
Which of the following has the least potential to cause psychological and physical addiction?
Choose one answer.
Error! Not a valid embedded object. a. Amphetamines
Error! Not a valid embedded object. b. Cocaine
Error! Not a valid embedded object. c. Cannabis
Error! Not a valid embedded object. d. Opiods
Error! Not a valid embedded object. e. LSD
Addiction can be described as Physiological (withdrawal symptoms & tolerance) or psychological
(compulsion to use, overwhelming involvement with the use of the drug, a willingness to go to great
lengths to secure it's continued supply, and a high tendency to relapse after it's withdrawal). A report by
the Strategy unit has identified Opiods and crack to have most addictive potential and LSD to have
minimal potential for psychological & physiological addiction Strategy Unit Drug Report:
Understanding the issues. May 2003.
Incorrect
Marks for this submission: 0/1.
Question 75
Marks: 0/1
Which of the following is a test of heterogeneity in Metaanalysis
Choose one answer.
Error! Not a valid embedded object. a. ANOVA
Error! Not a valid embedded object. b. MANCOVA
Error! Not a valid embedded object. c. t test
Error! Not a valid embedded object. d. Kruskal Wallis test
Error! Not a valid embedded object. e. Chi square test
Chi squared test (?2), Cochran's Q test are statistical tests to detect Heterogeneity in SR. Galbraith plot
& Forest plot are graphical methods by which heterogeneity can be deduced.
John N S Matthews and Douglas G Altman. Statistics notes: Interaction 3: How to examine
heterogeneity. BMJ Oct 1996; 313: 862
Incorrect
Marks for this submission: 0/1.
Question 76
Marks: 0/1
An evaluation of a new screening questionnaire for depression in primary care is conducted. The most
important single feature of this questionnaire that would encourage you to use it is:
Choose one answer.
Error! Not a valid embedded object. a. It is brief
Error! Not a valid embedded object. b. it has Positive Predictive Value of 20%
Error! Not a valid embedded object. c. It has a sensitivity of 80%
Error! Not a valid embedded object. d. It has a specificity of 60%
Error! Not a valid embedded object. e. It is validated in different languages
A screening instrument is selected on the basis of having a high sensitivity and specificity. In this
example, a sensitivity of 80% suggests that the screening instrument will correctly identify 80% of the
cases correctly which is a measure of utility of the test.
""
Incorrect
Marks for this submission: 0/1.
Question 77
Marks: 0/1
What is the range of values for a correlation coefficient
Choose one answer.
Error! Not a valid embedded object. a. (-infinity to 0)
Error! Not a valid embedded object. b. o to infinity
Error! Not a valid embedded object. c. (-1 to +1)
Error! Not a valid embedded object. d. 1 to infinity
Error! Not a valid embedded object. e. (-infinity to +infinity)
The strength or magnitude of the relationship between the two variables is expressed by a statistic
known as the CORRELATION COEFFICIENT, which varies from -1 (perfect negative relationship),
through 0 (no relationship) to +1 (perfect positive relationship). It is not possible to obtain a coefficient
more negative than -1 or greater than +1.
""
Incorrect
Marks for this submission: 0/1.
Question 78
Marks: 0/1
Expert opinions form what grade of Evidence
Choose one answer.
Error! Not a valid embedded object. a. B
Error! Not a valid embedded object. b. D
Error! Not a valid embedded object. c. E
Error! Not a valid embedded object. d. C
Error! Not a valid embedded object. e. A
Expert committee reports or opinions and/or clinical experiences of respected authorities (evidence
level IV) for Grade C of Evidence base as prescribed by NICE. This grading indicates that directly
applicable clinical studies of good quality are absent or not readily available. Systematic Reviews &
RCTs form Garde A, whilst studies without randomisation including cohort studies, case-control
studies and correlational studies form Grade B evidence.
""
Incorrect
Marks for this submission: 0/1.
Question 79
Marks: 0/1
Cronbachs alpha measures which of the following
Choose one answer.
Error! Not a valid embedded object. a. Test retest reliability
Error! Not a valid embedded object. b. Internal consistency
Error! Not a valid embedded object. c. Inter rater reliability
Error! Not a valid embedded object. d. Split half reliability
Error! Not a valid embedded object. e. Face validity
Internal consistency is a measure of validity and refers to whether the items on a scale are related to one
another. This is measured using Cronbach's alpha.
J Martin Bland and Douglas G Altman. Statistics notes: Cronbach's alpha. BMJ Feb 1997; 314: 572
Incorrect
Marks for this submission: 0/1.
Question 80
Marks: 0/1
The statement for reporting Meta analysis is provided by
Choose one answer.
Error! Not a valid embedded object. a. COCHRANE
Error! Not a valid embedded object. b. RESORT
Error! Not a valid embedded object. c. QUORUM
Error! Not a valid embedded object. d. CONSORT
Error! Not a valid embedded object. e. MEDLINE
The Quality of Reporting of Meta analysis provides a statement for appropriate reporting of meta
analysis to ensure consistency across the studies reported. This is similar to the CONSORT statement
which has been provided for RCTs
""
Incorrect
Marks for this submission: 0/1.
Question 81
Marks: 0/1
The most appropriate measure to explain about treatment response of a medication to a patient would
be
Choose one answer.
Error! Not a valid embedded object. a. Relative risk
Error! Not a valid embedded object. b. Controlled event rate
Error! Not a valid embedded object. c. Odds ratio
Error! Not a valid embedded object. d. Number needed to harm
Error! Not a valid embedded object. e. Number needed to treat
The NNT is a useful measure od treatment response and is the number of patients that need to be
treated for one to benefit compared with a control.
""
Incorrect
Marks for this submission: 0/1.
Question 82
Marks: 0/1
The database that provides systematic coverage of the psychological literature from the 1800s to the
present is
Choose one answer.
Error! Not a valid embedded object. a. Psychinfo
Error! Not a valid embedded object. b. Pubmed
Error! Not a valid embedded object. c. Psychlit
Error! Not a valid embedded object. d. Medline
Error! Not a valid embedded object. e. Embase
PsycINFO is an abstract database that provides systematic coverage of the psychological literature from
the 1800s to the present. PsycINFO contains bibliographic citations, abstracts, cited references, and
descriptive information to help you find what you need across a wide variety of scholarly publications
in the behavioral and social sciences. Medline provides information from 1965, Psychlit from 1887 and
EMbase from 1947.
""
Incorrect
Marks for this submission: 0/1.
Question 83
Marks: 0/1
The cost of forgoing an established treatment whilst opting for a new treatment is termed as
Choose one answer.
Error! Not a valid embedded object. a. Direct cost
Error! Not a valid embedded object. b. Opportunity cost
Error! Not a valid embedded object. c. Cost effectiveness
Error! Not a valid embedded object. d. Indirect cost
Error! Not a valid embedded object. e. Cost benefit
The opportunity cost of investing in a newintervention is best measured by the health benefits (life
years saved, quality adjusted life years (QALYs) gained) that could have been achieved had the money
been spent on the next best alternative intervention.
Stephen Palmer and James Raftery. Economics notes: Opportunity cost. BMJ 1999; 318: 1551 - 1552
Incorrect
Marks for this submission: 0/1.
Question 84
Marks: 0/1
A researcher wants to evaluate the use Magnetic resonance imaging as a screening test to identify cases
of Alzheimer's dementia. They would confirm the correct diagnosis of Alzheimer's dementia by post
mortem examination of the brain. Which of the following study designs would be the most appropriate
for such a study.
Choose one answer.
Error! Not a valid embedded object. a. Cross sectional study
Error! Not a valid embedded object. b. Cohort study
Error! Not a valid embedded object. c. Delphi study
Error! Not a valid embedded object. d. Case control study
Error! Not a valid embedded object. e. Pre post study
Cross section studies are the most appropriate study design when trying to answer questions concerning
the value of screening or diagnostic tests and prevalence of a disease (i.e., the number of subjects with a
disease at a given point of time).
""
Incorrect
Marks for this submission: 0/1.
Question 85
Marks: 0/1
All the following are true about likelihood ratios except
Choose one answer.
Error! Not a valid embedded object. a. They are affected by prevalence
Error! Not a valid embedded object. b. Has no relation to post test probability
Error! Not a valid embedded object. c. Allow better understanding of a diagnostic test
Error! Not a valid embedded object. d. Indicate changes in pre-test probability
Error! Not a valid embedded object. e. Derived from sensitivity and specificity
LRs are unaffected by prevalence and are constant regardless of disease prevalence.
""
Incorrect
Marks for this submission: 0/1.
Question 86
Marks: 0/1
Funnel plots are used to detect
Choose one answer.
Error! Not a valid embedded object. a. Language bias
Error! Not a valid embedded object. b. Publication bias
Error! Not a valid embedded object. c. Interviewer bias
Error! Not a valid embedded object. d. Recall bias
Error! Not a valid embedded object. e. Sensitivity bias
Funel plots are primarily used to detect publication bias.
""
Incorrect
Marks for this submission: 0/1.
Question 87
Marks: 0/1

Please read the following precis from an article from The British Journal of Psychiatry (2007) 190:
129-134 for questions 89-95

The authors of this study were interested to investigate whether patients with a history of schizophrenia
are at increased risk of hip fracture.

The GPRD was set up in the UK in 1987 and contains the computerised medical records of
approximately 5% of the UK population in primary care. Data recorded include prescription details,
clinical events, preventive care provided, specialist referrals, hospital admissions and their major
outcomes. All patients registered on the GPRD between 1 August 1987 and 22 November 1999 with a
recorded diagnosis of fractured neck of femur or ‗hip fracture‘ were identified. Two matching persons
without a recorded diagnosis of fractures were matched on age, gender, general practice and duration of
available GPRD data.

Variables included the patients‘ medical and psychiatric history, medication history and demographic
details, as well as lifestyle factors (alcohol consumption, smoking and body mass index). All recorded
diagnoses of schizophrenia were extracted and recorded on an ever/never basis. Ever having had a
prescription for a neuroleptic drug prior to or on the day of the first fracture was extracted and
recorded. Where comorbid disorders were examined all disorders under the main relevant ICD–9
heading were included e.g. ‗intestinal diseases‘ includes all diseases under this heading in ICD–9.

Variables that had been previously identified in the research literature as being significantly associated
with hip fracture were examined in univariate analyses and those that were significantly associated in
this analysis were selected for multivariate analysis. Conditional logistic regression was used to
calculate an unadjusted odds ratio for each selected variable.

There were 16 341 cases of hip fracture and 29 889 controls in this sample. The results of the study are
provided below in table 1:

Table 1 Adjusted odds ratios for hip fractures: General Practice Research Database 1987-1999 (n=36
330)

Exposure OR (95% CI)

Schizophrenia diagnosis 1.73 1.32-2.28

Previous falls 3.79 3.43-4.19

Overweight 0.55 0.50-0.62

Smoking 1.24 1.10-1.40

Inhaled corticosteroids 1.26 1.17-1.36

Injected corticosteroids 1.60 0.96-1.19

Alcohol use 1.60 1.33-1.92


Antipsychotic use 2.60 2.43 – 2.78

The type of study design used in this study is


Choose one answer.
Error! Not a valid embedded object. a. Cross sectional Survey
Error! Not a valid embedded object. b. Case Control
Error! Not a valid embedded object. c. retrospective Cohort
Error! Not a valid embedded object. d. Randomised controlled study
Error! Not a valid embedded object. e. Qualitative
This study is an example of a case-control study, where the cases are people with a recorded diagnosis
of fractures and the controls are people without a recorded diagnosis of fractures, and the comparisons
are being made with regards to various exposure variables including diagnosis of schizophrenia,
medication, use of steroids, etc. ""
Incorrect
Marks for this submission: 0/1.
Question 88
Marks: 0/1
From the above precis, Which of the following statements is the least true:
Choose one answer.
Error! Not a valid
a. The GP practices are likely to have computerised data
embedded object.
Error! Not a valid b. The sample is likely to be representative of the GP practices in the
embedded object. UK
Error! Not a valid c. The General Practice Research Database sample is a random sample
embedded object. from the general population
Error! Not a valid
d. The data in this database has been found to be accurate
embedded object.
Error! Not a valid
e. The diagnosis of schizophrenia has been validated
embedded object.
Reading through the actual paper, it is clear that the GPRD is representative of the general population,
though there is a likely bias towards larger practices which are more likely to have computerised data.
The diagnosis of schizophrenia has been validated across many studies, and data recorded in this
database has been found to be accurate by many quality assurance measures.
""
Incorrect
Marks for this submission: 0/1.
Question 89
Marks: 0/1
From the above precis, What was the most significant predictor of hip fractures?
Choose one answer.
Error! Not a valid embedded object. a. Alcohol use
Error! Not a valid embedded object. b. Previous falls
Error! Not a valid embedded object. c. Schizophrenia
Error! Not a valid embedded object. d. Weight
Error! Not a valid embedded object. e. Smoking
The Odds ratio and Confidence intervals for Previous falls is the highest, hence we can deduce that it is
the most significant predictor of hip fractures.
""
Incorrect
Marks for this submission: 0/1.
Question 90
Marks: 0/1
From the above precis, Which of the following predicts more hip fractures:
Choose one answer.
Error! Not a valid embedded object. a. Injected corticosteroids
Error! Not a valid embedded object. b. Inhaled corticosteroids
Error! Not a valid embedded object. c. Weight
Error! Not a valid embedded object. d. Smoking
Error! Not a valid embedded object. e. Schizophrenia
Among the variables in this question, Schizophrenia has the higher Odds Ratio and Cis, hence it is the
more likely to be associated with hip fractures.
""
Incorrect
Marks for this submission: 0/1.
Question 91
Marks: 0/1
From the above precis, What can we not deduce from this study?
Choose one answer.
Error! Not a valid embedded a. A diagnosis of schizophrenia is significantly associated with hip
object. fractures.
Error! Not a valid embedded
b. Smoking cause hip fractures.
object.
Error! Not a valid embedded
c. Antipsychotic use is significantly associated with hip fractures
object.
Error! Not a valid embedded d. Inhaled Corticosteroids are significantly associated with hip
object. fractures.
Error! Not a valid embedded
e. Being overweight is not associated with hip fractures
object.
The study design used here is a case-control study which is retrospective in nature, and at best can lead
to associations being established. A prospective study design like a cohort study or an RCT would help
in establishing causality.
""
Incorrect
Marks for this submission: 0/1.
Question 92
Marks: 0/1
According to this study, the next best step for someone who has had a fall would be to:
Choose one answer.
Error! Not a valid embedded object. a. Do a thorough falls risk assessment
Error! Not a valid embedded object. b. Check weight
Error! Not a valid embedded object. c. Provide them help with smoking
Error! Not a valid embedded object. d. Check alcohol use
Error! Not a valid embedded object. e. Check for schizophrenia
In this study the most significant predictor for hip fractures in history of previous falls, hence it would
be most appropriate to conduct a falls risk assessment to prevent further falls, and hence hip fractures
""
Incorrect
Marks for this submission: 0/1.
Question 93
Marks: 0/1
If the at risk elderly population in the UK is 55 million then calculate the number of people who have
had hip fractures in one year:
Choose one answer.
Error! Not a valid embedded object. a. 27000
Error! Not a valid embedded object. b. 10000
Error! Not a valid embedded object. c. 50000
Error! Not a valid embedded object. d. 60000
Error! Not a valid embedded object. e. 40000
In this study which represents 5% of the UK population, the incidence of hip fractures over a 12 year
period is 16341. The annual incidence is 16341 / 12 = 1362. The annual incidence for the whole UK
population of 55 million is 1362
20 = 27000.
Incorrect
Marks for this submission: 0/1.
Question 94
Marks: 0/1

For questions 96-98 read the following précis:

A screening test has been devised to diagnose bowel cancer. There is a new treatment that has recently
become available, which despite having good efficacy has also known to have significant side effects.
The different cut –offs on the new test to diagnose the cancer using this test has been plotted below.

Plot of sensitivity vs. 1-specificity of using various cut-off points of the new screening test.

What is the most ideal cut-off point to select patients with bowel cancer for treatment with the new
medication which has also found to have side effects
Choose one answer.
Error! Not a valid embedded object. a. 6
Error! Not a valid embedded object. b. 2
Error! Not a valid embedded object. c. 3
Error! Not a valid embedded object. d. 1
Error! Not a valid embedded object. e. 5
As the new treatment has side effects along with therapeutic effects, we need to ensure that the false
positives are less while the true positives are high. Though the cut-off of 5 seems to have a high true
positive, it also comes with a false positive rate of around 80%, which would not be appropriate in this
case. So a cut-off point of 3, which though has less true positive, also has lower false positives would
be the most appropriate to choose in this case. ""
Incorrect
Marks for this submission: 0/1.
Question 95
Marks: 0/1
The plot in this precis is termed as
Choose one answer.
Error! Not a valid embedded object. a. Funnel plot
Error! Not a valid embedded object. b. Forrest plot
Error! Not a valid embedded object. c. Receiver operating characteristic curve plot
Error! Not a valid embedded object. d. inverted funnel plot
Error! Not a valid embedded object. e. Stem plot
The plot of sensitivity (true positive) vs. 1-specificity (false positive) for different cut-off points is
termed as a ROC curve plot. D G Altman and J M Bland. Statistics Notes: Diagnostic tests 3: receiver
operating characteristic plots. BMJ Jul 1994; 309: 188
Incorrect
Marks for this submission: 0/1.
Question 96
Marks: 0/1
What is the cut-off point on the new test at which you will have the lowest false positive rate
Choose one answer.
Error! Not a valid embedded object. a. 2
Error! Not a valid embedded object. b. 3
Error! Not a valid embedded object. c. 5
Error! Not a valid embedded object. d. 6
Error! Not a valid embedded object. e. 1
False positive rate is nothing but 1-specificity, and in this plot, it is clear that the cut-off point of 1
would have the lowest false positive rates.
""
Incorrect
Marks for this submission: 0/1.
Question 97
Marks: 0/1

For questions 99-101 read the following précis:

A study (Pasco, J. A. et al. The British Journal of Psychiatry 2008;193:322-326) was conducted to
evaluate smoking as a risk factor for major depression, and the figure below is presented as results.
What is on the Y axis of the graph above
Choose one answer.
Error! Not a valid embedded object. a. Time to have a relapse of major depression
Error! Not a valid embedded object. b. Probability of not having Major depression
Error! Not a valid embedded object. c. Probability of improving from depression
Error! Not a valid embedded object. d. Time to develop depression
Error! Not a valid embedded object. e. Probability of having Major depression
This is a Kaplan Meier survival curve with the X axis showing time to depression in years and the Y
axis showing the probability of not having depression. ""
Incorrect
Marks for this submission: 0/1.
Question 98
Marks: 0/1
Which of the following statements can be inferred from the above figure
Choose one answer.
Error! Not a valid
a. The survival rates are similar for both the groups at year 4
embedded object.
Error! Not a valid b. All the people in the non-smoking group had not developed depression
embedded object. by year 10
Error! Not a valid c. There is a significant difference in the probability of developing
embedded object. depression between the two groups at year 2
Error! Not a valid
d. The rates of depression are similar across both the groups at year 6
embedded object.
Error! Not a valid e. All the people in the smoking group had developed depression by year
embedded object. 10
This is a Kaplan Meier survival curve with the X axis showing time to depression in years and the Y
axis showing the probability of not having depression. The values for the Y axis are slightly misleading
as the scaling used is depicting very small differences between corresponding values, hence we cannot
deduce differences between groups at year 2 or 6 easily. The two lines overlap at year 4, and it can
safely be deduced that probability of not having depression is similar at year 4
""
Incorrect
Marks for this submission: 0/1.
Question 99
Marks: 0/1
The following can be deduced from the above graph
Choose one answer.
Error! Not a valid a. At year 4, the probability of developing depression is higher in the
embedded object. smoking group compared to non-smoking group
Error! Not a valid b. At 10th year, the probability of survival is greater for non-smokers
embedded object. compared to smokers
Error! Not a valid c. At the 10th year the people in the smoking group had a 100%
embedded object. probability of developing depression
Error! Not a valid d. At year 7, the probability of developing depression is higher in the
embedded object. smoking group compared to non-smoking group
Error! Not a valid e. At 10th year, the probability of survival is greater for smokers compared
embedded object. to non-smokers
""
""
Incorrect
Marks for this submission: 0/1.
Question 100
Marks: 0/3
Epidemiology

A - 1:1
B - 1:3
C - 1:6
D - 1:8
E - 1:10
F - 3:1
G - 4:1
H - 5:1
I - 6:1
J - 7:1

Select the Male: Female ratios in the following conditions


Attention deficit and hyperactivity disorder
Autism
Bulimia
Answers

F
G
E

1. F. 3:1. ADHD is more common in males at ratio of 3:1 in community samples and 9:1 in clinic
samples.
2. G. 4:1. A large review of all prevalence studies has found that the M:F ratios for Autism is around
3.8 :1 (or 4:1) 3. E. 1:10. Bulimia is more common in females with occurrence in males t hat of 1/10 th
in females.

Incorrect
Marks for this submission: 0/3.
Question 101
Marks: 0/3
Child & Adolescent Psychiatry - Treatment

A - Clozapine
B - Diazepam
C - Family therapy
D - Lithium
E - Logotherapy
F - Methylphenidate
G - Parent training
H - Paroxetine
I - Risperidone
J - Sodium Valproate

Choose appropriate treatment for each of the following clinical presentations


A 4 year old child, disrespectful to teachers and hence banned from the nursery
A 17 year old, aggressive in all circumstances and to the point that parents have a court injunction
against him.
A 13 year old girl, aggressive, in trouble at school, diagnosed as hyperkinetic by psychologist
Answers

G
I
F

1.G - . NICE recommends group-based parent-training/education programmes in the management of


children with conduct disorders. Individual-based programmes are only recommended where the
family's needs are too complex for a group-based programme
2. I - Risperidone is the most widely prescribed drug for CD, even though there is no licensed drug for
the treatment of CD. It should therefore only be used in extreme cases.
3. F - NICE recommends methylphenidate as the first line drug for ADHD

Incorrect
Marks for this submission: 0/3.
Question 102
Marks: 0/3
Sampling Techniques

A - Cluster sampling
B - Convergent sampling
C - Matched random
D - Purposive sampling
E - Quota Sampling
F - Simple random sampling
G - Snowball
H - Stratified random sampling
I - Systematic sampling

Select the most appropriate sampling technique for each of the following:
A study in which the participants from only selected areas are included (Choose one)
Participants in a study are recruited from odd numbered houses (Choose one)
A researcher wishes to ascertain the experiences of people with personality disorders with the
therapeutic community service. (Choose one)
Answers

A
I
D

1. A Cluster sampling divides the population into groups, or clusters. A number of clusters are selected
randomly to represent the population, and then all units within selected clusters are included in the
sample. No units from non-selected clusters are included in the sample. They are represented by those
from selected clusters
2. I. In systematic sampling the participants are selected at regular intervals from a sampling frame.
Using this procedure each element in the population has a known and equal probability of selection.
3. D. In qualitative studies, mostly purposive sampling techniques are used to identify the sample
required to be studies.

""
Incorrect
Marks for this submission: 0/3.
Question 103
Marks: 0/4
Study designs

A - Case-control
B - Cross over trial
C - Cross sectional survey
D - Cohort study
E - N of 1 study
F - Open Label RCT
G - Parallel RCT
H - Prospective Cohort
I - Qualitative study
J - Systematic review

Which would be the most ethical (choose ONE) for the following research questions:
To identify the prevalence of cyclothymia in attendees to primary care surgeries.
To get mentally ill patient's perspectives on the quality of care they receive from the mental health
services.
To look at risk of psychotic disorders in illegal immigrants in the UK.
To compare a new drug for clozapine induced hypersalivation vs. placebo. The researchers wish to
recruit fewer participants but still have adequate power in the study.
Answers

C
I
D
B

1. C. Cross-sectional studies can be thought of as providing a "snapshot" of the frequency and


characteristics of a disease in a population at a particular point in time. This type of data can be used to
assess the prevalence of acute or chronic conditions in a population. However, since exposure and
disease status are measured at the same point in time, it may not always be possible to distinguish
whether the exposure preceded or followed the disease.
2. I. Qualitative methods aim to make sense of, or interpret, phenomena in terms of the meanings
people bring to them. Qualitative research may define preliminary questions that can then be addressed
in quantitative studies. A good qualitative study will address a clinical problem through a clearly
formulated question and using more than one research method (triangulation). Analysis of qualitative
data can and should be done using explicit, systematic, and reproducible methods.
3.D. A cohort study would be the best suited to identify if illegal immigration is a risk factor for
psychosis.
4. B. A cross over trial is a method of RCT, comparing 2 interventions in which patients are switched
to the alternative intervention after a specified period of time.

Incorrect
Marks for this submission: 0/4.
Question 104
Marks: 0/5
Cross sectional studies

A - 11%
B - 22%
C - 33%
D - 44%
E - 55%
F - 66%
G - 77%
H - 88%
I - 99%

A study involved screening terminally ill people for depression with the question 'Do you think you are
depressed?' A structured diagnostic interview using DSM IV also used to diagnose depression in the
same sample. The results of the study are in the table below. Calculate the below using the information
from the table.

Interview Positive Interview Negative


DSM Positive 11 9
DSM Negative 14 40

What was the sensitivity of the screening test?


What was the specificity of the screening test?
What was the positive predictive value of the test?
What proportion of those who were depressed screened positive?
What was the negative predictive value?
Answers
Please note that in the exam the table provided was contrary to the usual convention, which means
candidates who had learnt the formulae by rote would have had problems. If you do want to use
formulae, please ensure that the gold standard is on top and the screening test is on the left hand side of
the table. You can easily flip this table to get the desired table, which would look as below.

DSM positive DSM negative


Interview positive 11 14
Interview negative 9 40

It would be easy to do the calculations this way!

1.E.Sensitivity = a / (a+c) 11/20 = 55%


2. G. Specificity = d / (b+d) 40 / 54 =74%
3.D.Positive predictive value = a / (a+b) 11/25 = 44%
4.E.This is sensitivity = 44%
5.H.Negative predictive value = d / (c+d) = 40 / 49 = 81%
Incorrect
Marks for this submission: 0/5.
Question 105
Marks: 0/4
Calculations in Critical Appraisal

A-1
B-2
C-3
D-4
E - 10%
F - 15%
G - 23%
H - 33%
I - 43%
J - 55%

A systematic overview of evidence from randomised trials of continuing treatment with antidepressants
in patients with depressive disorders who have responded to acute treatment was conducted to evaluate
the need for continued treatment with antidepressants after treatment of an acute episode. The results
are provided in the table below: (JR Geddes, SM Carney and C Davies et al., Relapse prevention with
antidepressant drug treatment in depressive disorders: a systematic review, Lancet 361 (2003), pp. 653-
661).
From the table provided, calculate the following

Control event rate in the placebo group


Experimental event rate in the SSRI group
Overall Absolute Benefit increase
Number needed to treat for the MAO-I GROUP
Number needed to treat for the Antidepressant
Answers

I
F
G
A
D

1. I.1031/ 2505 --> 41%


2. F.156 / 1034 --> 15%
3. G.41-18 --> 23%
4. A. ABI = 86%-21% --> 75%. NNT = 1/0.75 1.33~ 1
5. D.ABI = 41 - 18 --> 23%, NNT = 1/0.23 = 4
Incorrect
Marks for this submission: 0/4.
Question 106
Marks: 0/3
Personality disorders

A - Anankastic personality disorder


B - Anxious-avoidant personality disorder
C - Dependent personality disorder
D - Dissocial personality disorder
E - Emotionally unstable - borderline type
F - Histrionic personality disorder
G - Narcissistic personality disorder
H - Paranoid personality disorder
I - Schizoid personality disorder
J - Schizotypal personality disorder

Choose ONE possible personality disorder for each of the following scenarios :
A 40 year old man has problems with his colleagues at work due to hi perception of work colleagues
being less competent than them, and hence being very hostile towards him. He has made repeated
complaints against his boss as they feel that their boss was being overtly critical of his perfect work,
this has however been investigated and found to be unsubstantiated. He had been in couple of
relationships that have ended due to their concerns about the fidelity of their partners.
A 40 year old stock broker was referred to the community alcohol team due to problems at work.
They are upset that they had to wait for up to half an hour before seeing the doctor and had shouted
twice at the receptionist. On seeing the doctor, they mention that they are possibly the best in the
business, which has upset some of the partners in their firm and had denied them the bonus that they
deserved. Their reason for drinking in excess in due their employers lack of appreciation of their
talents rather than any other reason. They are upset when the Doctor refuses to see them on a daily
basis, and feel that psychiatrists are 'failed doctors' and refuses to come back to the clinic.
A 24 year old woman been referred due to her difficulties in maintaining relationships and recurrent
depression and suicidal attempts. She initially forms a good relationship with her Community
psychiatric nurse, only to later lash out at the CPN in the CMHT for being neglected by them. She
then threatens to harm herself in front of the CPN, which causes intense frustration for the CPN.
Answers

H
G
E

1. H. Paranoid Personality disorder is characterized by excessive sensitivity to setbacks, unforgiveness


of insults; suspiciousness and a tendency to distort experience by misconstruing the neutral or friendly
actions of others as hostile or contemptuous; recurrent suspicions, without justification, regarding the
sexual fidelity of the spouse or sexual partner; and a combative and tenacious sense of personal rights.
There may be excessive self-importance, and there is often excessive self-reference.
2. G.Narcissistic personality disorder is a condition characterized by an inflated sense of self-
importance, need for admiration, extreme self-involvement, and lack of empathy for others. Individuals
with this disorder are usually arrogantly self-assured and confident. They expect to be noticed as
superior. Vulnerability in self-esteem makes individuals with this disorder very sensitive to criticism or
defeat. Although they may not show it outwardly, criticism may haunt these individuals and may leave
them feeling humiliated, degraded, hollow, and empty. They may react with disdain, rage, or defiant
counterattack. Their social life is often impaired due to problems derived from entitlement, the need for
admiration, and the relative disregard for the sensitivities of others. Though their excessive ambition
and confidence may lead to high achievement; performance may be disrupted due to intolerance of
criticism or defeat.
3. E. Emotionally unstable personality disorder is characterized by a definite tendency to act
impulsively and without consideration of the consequences; the mood is unpredictable and capricious.
There is a liability to outbursts of emotion and an incapacity to control the behavioural explosions.
There is a tendency to quarrelsome behaviour and to conflicts with others, especially when impulsive
acts are thwarted or censored. Two types may be distinguished: the impulsive type, characterized
predominantly by emotional instability and lack of impulse control, and the borderline type,
characterized in addition by disturbances in self-image, aims, and internal preferences, by chronic
feelings of emptiness, by intense and unstable interpersonal relationships, and by a tendency to self-
destructive behaviour, including suicide gestures and attempt

Incorrect
Marks for this submission: 0/3.
Question 107
Marks: 0/5
Errors in research

A - Alternate hypothesis
B - Bias
C - 95% Confidence Interval
D - Null Hypothesis
E - p value
F - Power
G - Reliability
H - Type I error
I - Type II error
J - 95% percentile

From the options listed above, choose the most appropriate option for the following.
The probability of rejecting the null hypothesis, when it is true
The probability of finding a difference if it existed
The range within which the true difference value lies 19 out of 20 times.
A probability of accepting a false null hypothesis
The probabilities of observing the study result in a random sample, given that the null hypothesis is
true.
Answers

H
F
C
I
E

1. H. This is the definition of Type I error A null hypothesis (for example, no difference between
groups) is rejected, while it should be accepted.

2. F. This is the definition of the power of a test. The power of a statistical test is the probability that the
test will reject a false null hypothesis (that it will not make a Type II error). As power increases, the
chances of a Type II error decrease.

3. C. A confidence interval gives an estimated range of values which is likely to include an unknown
population parameter (p value, mean difference etc.), the estimated range being calculated from a given
set of sample data.
4. I. This is a type II error, wherein a null hypothesis is accepted, while it should be rejected.
5. E. The p-value is the probability that your sample could have been drawn from the population(s)
being tested given the assumption that the null hypothesis is true

Incorrect
Marks for this submission: 0/5.
Question 108
Marks: 0/3
Reversible dementia

A - Carbimazole
B - Dexamethasone
C - Methotrexate
D - Neomycin
E - Surgical evacuation
F - Surgical shunt
G - Thiamine
H - Thyroxine
I - Vincristine
J - Vitamin B12

Choose the most appropriate treatment from the list of options above, for each of the scenarios below.
A 55 year old lady presents with a history of memory disturbance and early morning headaches which
tend to get better as the day progresses. On examination, there is facial nerve palsy. (Choose one)
A 60 year old lady presents with long standing memory problems, ataxia, nystagmus and asterixis.
(Choose One)
A 65 year old lady presents with mild cognitive impairment, urinary incontinence and ataxia (Choose
one)
Answers

E
G
F

1. E. This seems to be the most appropriate choice as the scenario seems to point towards a classical
case of brain tumour - Early morning headache, and focal neurological sign. It is important to bear in
mind that all early morning headache is not related to tumors - Please read -Larner AJ. Not all early
morning headaches are due to brain tumors. Practical Neurology 2009;9:80-84
2.G. This scenario is suggestive of a liver disease & Wernicke's encephalopathy secondary to
alcoholism. The treatment would be Thiamine.
3. F. This scenario is suggestive of Normal Presure Hydrocephalus triad of Urinary incontinence,
Ataxia and Memory difficulties. The treatment is usually a surgical shunt
Incorrect
Marks for this submission: 0/3.
Question 109
Marks: 0/3
Neuropsychiatry

A - Creudzfeldt Jacob disease


B - Huntington's disease
C - Meige's syndrome
D - Metachromatic leucodystrophy
E - Myoclonus
F - Neuroacanthosis
G - Parkinson' disease
H - Progressive supranuclear palsy
I - Sydenhams chorea
J - Tourette's disease

For the following questions below, choose the most appropriate diagnosis from above.
A 62 year old lady presents with difficulties due to frequent falls. On examination, she has a wide-
based shuffling gait with short steps, a vertical gaze palsy and micrographia.
A 15 year old girl presents with a halting gait, difficulty writing, and uncoordinated movements of her
limbs. She had a history of fever and sore throat six months ago.
A 40 year old man is brought by his wife, due to problems with their gait instability and loss of
memory. She mentions that he has become clumsy and has developed jerky movements of his
muscles.
Answers

H
I
A

1. H. Progressive supranuclear palsy (PSP), also known as Steele-Richardson-Olszewski syndrome, is a


neurodegenerative disease that affects cognition, eye movements, and posture. The exact cause is
unknown. The cardinal manifestations of PSP are supranuclear ophthalmoplegia; pseudobulbar palsy;
prominent neck dystonia; parkinsonism; behavioral, cognitive, and gait disturbances that cause
imbalance; and frequent falls
2. I. Sydenham chorea (SD) is a neurological disorder of childhood resulting from infection via Group
A beta-hemolytic streptococcus, the bacterium that causes rheumatic fever. Rapid, irregular, and
aimless involuntary movements of the arms and legs, trunk, and facial muscles characterize Syndenham
Chorea. It affects girls more often than boys and typically occurs between 5 and 15 years of age. Some
children will have a sore throat several weeks before the symptoms begin, but the disorder can also
strike up to 6 months after the fever or infection has cleared. Symptoms can appear gradually or all at
once, and may include uncoordinated movements, muscular weakness, stumbling and falling, slurred
speech, difficulty concentrating and writing, and emotional instability. The symptoms of SD can vary
from a halting gait and slight grimacing to involuntary movements that are frequent and severe enough
to be incapacitating. The random, writhing movements of chorea are caused by an auto-immune
reaction to the bacterium that interferes with the normal function of a part of the brain (the basal
ganglia) that controls motor movements.
3. A. Creutzfeldt-Jakob disease (CJD) is the best known of a group of diseases called prion disease,
which affects a form of protein found in the central nervous system and cause dementia. Early
symptoms include minor lapses of memory, mood changes and loss of interest. Within weeks, an
infected person may complain of clumsiness and feeling muddled, become unsteady in walking, and
exhibit slow or slurred speech. The symptoms progress to jerky movements, shakiness, stiffness of
limbs, incontinence and the loss of the ability to move or speak. By this stage, the person is unlikely to
be aware of their surroundings or disabilities. People affected by CJD usually die within six months of
early symptoms ? often from pneumonia. In a minority of patients, the disease may take two years to
run its course

Incorrect
Marks for this submission: 0/3.
Question 110
Marks: 0/4
Validity

A - Concurrent validity
B - Construct validity
C - Content validity
D - Criterion validity
E - Divergent validity
F - Face validity
G - Predictive validity

Choose ONE from the above that each of the following imaginary rating scales have:
Patients with an ICD-10 diagnosis of paranoid personality disorder have much higher scores on the
'paranoid subscale' of the MMPI than subjects without paranoid personality disorder.
Subjects with panic disorder are rated using a new scale, (Panic disorder scale) together with an older
well-validated scale. Scores on the two scales are highly correlated.
After an episode of deliberate self-harm, psychiatrists assess each patient using a new scale called
ADSH. All these patients are followed up for 6 months. Subjects with high scores on the ADSH were
much more likely to make further acts of deliberate self-harm than those with low scores.
A new self-report questionnaire for adult generalised anxiety disorder, the AGAD, has questions
covering in detail cognitive symptoms, metacognitive symptoms, behavioural and biological
symptoms.
Answers

D
A
G
C

1. D. Criterion validity also referred to as instrumental validity, is used to demonstrate the accuracy of a
measure by comparing it with another measure, which has been demonstrated to be valid or considered
as gold standard.
2. A. Concurrent validity is the agreement with another scale, which has itself been shown to be valid.
3. G. Predictive validity is the agreement between a present measurement and one in the future.
4. C. Content validity refers to the extent to which a scale covers all of the important subject matter.

Incorrect
Marks for this submission: 0/4.
Question 111
Marks: 0/3
Psychological treatment in group settings:

A. Cohesiveness
B. Conditioning
C. Counter-dependence
D. Dependence
E. Fight-flight
F. Free floating discussion
G. Interpreting transference
H. Pairing
I. Universality
J. Vicarious learning

Lead in: From the options above, Choose

Two curative factors in group therapy.


Three factors that hinder working in groups.

Two factors that are found in psychodynamic groups.

Answers

1. A & I. Yalom identified 11 curative factors in groups including cohesiveness (i.e. a sense of
belongingness) and universality (a sense that symptoms/problems are not specific to one group
member, but shared by many others) the other factors are: Installation of hope, imparting of
information, interpersonal learning, imitation of adaptive behavior, catharsis, corrective recapitulation
of the family group, altruism, guidance, existential factors, and social skills.

2. D, E, H. Bion identified several factors that hinder group work including: dependence and fight-
flight response & Pairing. A group holds one of three basic assumptions, which may differ from the
beliefs of the individual members. The basic assumption of dependence arises from the group's anxious
need to depend absolutely on someone (usually the therapist) to protect the members, satisfy all their
desires and solve their problems. This corresponds to Klein‘s depressive position. Collective belief in
an enemy who can be dealt with only by attack or retreat forms the basic assumption of fight-flight.
This corresponds to Klein‘s paranoid-schizoid position. The basic assumption of pairing defines the
notion that some future event or person will come from outside to solve all problems. For example,
there may be hope for the pairing of two individual members of the group that could lead to the ―birth‖
of a new solution for the group‘s problems. This corresponds to Klein and Freud‘s oedipal fantasy.

3. F, G. Free-floating discussions (group-analytic equivalent of free-association) and Interpreting


transference (along with interpretation of defences and resistances, archaic and primordial experiences)
are important in the context of psychodynamic groups. Foulkes also listed ‗group-specific processes‘ as
important factors.

Incorrect
Marks for this submission: 0/3.
Question 112
Marks: 0/3
Substance misuse presentations.

A. Acute alcohol intoxication


B. Alcoholic hallucinosis
C. Amphetamine intoxication
D. Benzodiazepine intoxication
E. Cannabis intoxication
F. Cocaine intoxication
G. Cocaine withdrawal
H. LSD intoxication
I. Opiate addiction
J. Opiate withdrawal

For each of the following patient presenting in the substance disorder clinic, select the most appropriate
diagnosis

An 18-year-old man presents to the A&E with vivid visual hallucinations, dilated pupils, and increased
appetite.
A 24 year old man presents with dilated pupils, reduced appetite, auditory hallucinations, insomnia and
irritability.

A 34 year old man presenting with slurred speech, incoordination of movements, and nystagmus

ANSWERS

1. E – Effects of cannabis intoxication include - Euphoria, relaxation, hallucinations, hilarity, increased


appetite and glazed eyes with large pupils.
2. C. Effects of amphetamine intoxication include - Increased alertness, pulse rate and blood pressure,
and feeling of high energy. Patients with amphetamine intoxication generally present with Enlarged
pupils, insomnia, loss of appetite and weight, irritability, and depression.
3. A, D. Acute alcohol and Benzodiazepine intoxication can present with similar signs including
incordination, slurring of speech, ocular signs, confusion etc.

Incorrect
Marks for this submission: 0/3.

August 2009
1
Marks: 0/1
The percentage of people in western Europe with a diagnosis of schizophrenia being treated in the
community, who smoke is ?
Choose one answer.
Error! Not a valid embedded object. a. 50%
Error! Not a valid embedded object. b. 40%
Error! Not a valid embedded object. c. 20%
Error! Not a valid embedded object. d. 60%
Error! Not a valid embedded object. e. 30%
Patients with schizophrenia consistently report higher rates of substance abuse than the general
population, notably with respect to licit (nicotine and alcohol) and illicit (cannabis and cocaine)
substances. 60-90% of patients with schizophrenia report smoking compared to 25 % of the general
population. Substance Use Disorders in Schizophrenia-Clinical Implications of
Comorbidity,Schizophrenia Bulletin 2009 35(3):469-472; doi:10.1093/schbul/sbp016 ,Nora D.
Volkow.
Incorrect
Marks for this submission: 0/1.
Question 2
Marks: 0/1
The following should be in the differential diagnosis for NMS
Choose one answer.
Error! Not a valid embedded object. a. Encephalitis
Error! Not a valid embedded object. b. Pulmonary embolism
Error! Not a valid embedded object. c. Hypothermia
Error! Not a valid embedded object. d. Opiate withdrawal
Error! Not a valid embedded object. e. Drug induced parkinsonism
Differentiating NMS from other neurological disorders can be very difficult. Some of the most
commonly mistaken diseases are: encephalitis, toxic encephalopathy, status epilepticus, heat stroke,
and malignant hyperthermia. Due to the comparative rarity of NMS, it is often overlooked and
immediate treatment for the syndrome is delayed. Drugs such as cocaine and amphetamine also result
in similar symptoms. Synopsis of Psychiatry, 10th edn, pg 996
Incorrect
Marks for this submission: 0/1.
Question 3
Marks: 0/1
Which of the following is a risk factor for NMS?
Choose one answer.
Error! Not a valid embedded object. a. Older patients
Error! Not a valid embedded object. b. Reduced core body temperature
Error! Not a valid embedded object. c. Dehydration
Error! Not a valid embedded object. d. Females affected more than males
Error! Not a valid embedded object. e. Depot more than oral
The risk factors for NMS include being male, younger and receiving depot rather than oral medication.
Synopsis of Psychiatry, 10th edn, pg 995
Incorrect
Marks for this submission: 0/1.
Question 4
Marks: 0/1
You are using a motivational interview approach on a patient with Hypochondriasis. Which of the
following is an approach that should be used?
Choose one answer.
Error! Not a valid embedded object. a. Colluding with the patient
Error! Not a valid embedded object. b. Making the patient feel that you understand them
Error! Not a valid embedded object. c. Making the patient feel that you believe them
Error! Not a valid embedded object. d. Attempt to amplify the ambivalence regarding change
Error! Not a valid embedded object. e. Re-attributing the symptoms
Motivational Interviewing is a client-centred approach to enhancing motivation for change. The main
strategy is to explore and amplify ambivalence regarding change rather than attempt to confront or
persuade the client. Amplifying resistance amplifies the person's cognitive dissonance, which thereby
motivates the person to try to minimize resistance, such as by developing a strong commitment to
engage in treatment to overcome his or her problems. A variety of well described techniques are used
to promote this process (e.g., rolling with resistance, developing discrepancy, and reinforcing self-
efficacy statements; see Miller & Rollnick, 2002). The goal is to help the patient to become his or her
own advocate for change by decreasing ambivalence and increasing
motivation.
McKay & Bouman (2008) Enhancing Cognitive-Behavioral Therapy for Monosymptomatic
Hypochondriasis With Motivational Interviewing: Three Case Illustrations. Journal of Cognitive
Psychotherapy: An International Quarterly 22(2):154-166
Incorrect
Marks for this submission: 0/1.
Question 5
Marks: 0/1
The risk of suicide after 1 year of self harming is
Choose one answer.
Error! Not a valid embedded object. a. 2-3%
Error! Not a valid embedded object. b. 0.5-2%
Error! Not a valid embedded object. c. 3-4%
Error! Not a valid embedded object. d. 0.1
Error! Not a valid embedded object. e. 4-5%
Owen et al in 2002 in their systematic review concluded that the risk of suicide following DSH as
between 0.5-2% after 1 year and above 5% after 9 years.
Companion to Psychiatric Studies, 7th edn, pg 678
Incorrect
Marks for this submission: 0/1.
Question 6
Marks: 0/1
To diagnose substance misuse a patient is assessed on criteria such as craving, tolerance symptoms,
withdrawal symptoms, exclusion of other activities etc. According to the ICD-10 how many of them
does the patient need to have to receive a diagnosis of dependency?
Choose one answer.
Error! Not a valid embedded object. a. 5
Error! Not a valid embedded object. b. 3
Error! Not a valid embedded object. c. 1
Error! Not a valid embedded object. d. 2
Error! Not a valid embedded object. e. 4
In ICD 10 diagnosis of dependence should be made if three or more of the following have been
experienced or exhibited at some time during the last year.
Shorter Oxford Textbook of Psychiatry, 5th edn, pg431
Incorrect
Marks for this submission: 0/1.
Question 7
Marks: 0/1
A 35 year old appears to be experiencing symptoms associated with withdrawal of diazepam. The
following is not seen
Choose one answer.
Error! Not a valid embedded object. a. Hypertension
Error! Not a valid embedded object. b. Delirium
Error! Not a valid embedded object. c. Seizures
Error! Not a valid embedded object. d. Insomnia
Error! Not a valid embedded object. e. Flu like symptoms
Postural hypotension and tachycardia are associated with diazepam withdrawal
New Oxford Textbook of Psychiatry, Vol 2, pg 1288
Incorrect
Marks for this submission: 0/1.
Question 8
Marks: 0/1
A 25 year old man is brought to you by his parents as has become more withdrawn. He describes
pseudohallucinations and quasi-psychotic symptoms. He presents as being quite eccentric. Which is the
most likely diagnosis?
Choose one answer.
Error! Not a valid embedded object. a. Schizoid PD
Error! Not a valid embedded object. b. Paranoid PD
Error! Not a valid embedded object. c. Schizophrenia
Error! Not a valid embedded object. d. Schizotypal PD
Error! Not a valid embedded object. e. Avoidant PD
Persons with Schizoptypal PD have poor interpersonal relationships and under stress can decompensate
and have psychotic symptoms.
Synopsis of Psychiatry, 10th edn, pg 797
Incorrect
Marks for this submission: 0/1.
Question 9
Marks: 0/1
A 30 year old man has been suffering from schizophrenia asks you about the percentage of people who
discontinue medication during the first year of treatment
Choose one answer.
Error! Not a valid embedded object. a. 26%
Error! Not a valid embedded object. b. 50%
Error! Not a valid embedded object. c. 75%
Error! Not a valid embedded object. d. 20%
Error! Not a valid embedded object. e. 10%
Medication discontinuation is a major problem in the treatment of first-episode patients; 26% of
patients stopped antipsychotics during the first year of treatment and 30% stopped antipsychotics
during maintenance treatment following the first relapse. The CATIE study found that around 75% of
patients discontinued treatment within 18 months, which seems to be a reasonable estimate. However,
since the college question is within 12 months, the best evidence seems to be 26% within a year.

Robinson et al (2002) Predictors of medication discontinuation by patients with first-episode


schizophrenia and schizoaffective disorder, Schizophrenia Research 57(2-3): 209-219.
Lieberman, J.A., et al. (2005). Effectiveness of Antipsychotic Drugs in Patients with Chronic
Schizophrenia. New England Journal of Medicine, (353), p.1209-1223.

Incorrect
Marks for this submission: 0/1.
Question 10
Marks: 0/1
Which of the following is recommended in treatment of an adolescent girl with hypomania?
Choose one answer.
Error! Not a valid embedded object. a. Lamotrigine
Error! Not a valid embedded object. b. Olanzapine
Error! Not a valid embedded object. c. Carbamazepine
Error! Not a valid embedded object. d. Lithium
Error! Not a valid embedded object. e. Valproate
Atypical antipsychotics are recommended as first line treatment in adolescent girls. Medication that
cause weight gain and increased prolactin should be avoided, but of the choices Olanzapine though is
associated with weight gain is the answer as the others are avoided
NICE guidelines for BPAD, 2006
Incorrect
Marks for this submission: 0/1.
Question 11
Marks: 0/1
Which of the following type of memory is most likely to be damaged in Wernike-Korsakoff syndrome?
Choose one answer.
Error! Not a valid embedded object. a. Semantic
Error! Not a valid embedded object. b. Procedural
Error! Not a valid embedded object. c. Episodic
Error! Not a valid embedded object. d. Working
Error! Not a valid embedded object. e. Praxis
Episodic memory is usually although not exclusively affected in the amnestic syndrome where as
semantic memory is particularly damaged in semantic dementia involving focal temporal lobe atrophy.
Lishman's Organic Psychiatry, 4th edn, pg36
Incorrect
Marks for this submission: 0/1.
Question 12
Marks: 0/1
A 23 year old woman has features of hypomania and she is also breast feeding her baby. The best
treatment option for treatment of hypomania is
Choose one answer.
Error! Not a valid embedded object. a. Olanzapine
Error! Not a valid embedded object. b. Lorazepam
Error! Not a valid embedded object. c. Clozapine
Error! Not a valid embedded object. d. Lamotrigine
Error! Not a valid embedded object. e. Lithium
Antipsychotics other than clozapine can be used in breastfeeding mothers but all the others mentioned
here should be avoided if possible.
NICE Guidelines, 2006
Incorrect
Marks for this submission: 0/1.
Question 13
Marks: 0/1
A 27 year old man has been diagnosed with simple schizophrenia . This is not a feature of this
condition
Choose one answer.
Error! Not a valid embedded object. a. Progressive deterioration
Error! Not a valid embedded object. b. Simple schizophrenia is included in ICD 10
Error! Not a valid embedded object. c. Overt psychotic symptoms seen
Error! Not a valid embedded object. d. Absence of overt psychotic symptoms
Error! Not a valid embedded object. e. Increasing eccentricity
In simple schizophrenia overt psychotic symptoms are absent
Companion to Psychiatric Studies, 7th edn, pg 397
Incorrect
Marks for this submission: 0/1.
Question 14
Marks: 0/1
Which of the following is an actuarial risk assessment instrument to assess risk of sexual offending
Choose one answer.
Error! Not a valid embedded object. a. PCL-R
Error! Not a valid embedded object. b. SORAG
Error! Not a valid embedded object. c. SARA
Error! Not a valid embedded object. d. VRAG
Error! Not a valid embedded object. e. HCR 20
The Hare Psychopathology Checklist Revised (PCL-R), Historical Clinical risk (HCR-20), and
Violence risk appraisal guide are actuarial risk instruments for violent recidivism. Spousal Assault risk
assessment guide (SARA) is an instrument to assess risk of assault by spouse, and the Sex offender risk
appraisal guide is to assess risk of sexual recidivism.
Click here for reference
Incorrect
Marks for this submission: 0/1.
Question 15
Marks: 0/1
Which of the following would you find in Prader Willi syndrome?
Choose one answer.
Error! Not a valid embedded object. a. Psychotic symptoms are absent
Error! Not a valid embedded object. b. Incessant skin picking
Error! Not a valid embedded object. c. Compulsive behaviour and severe mutilation
Error! Not a valid embedded object. d. Severe and frequent self-harm
Error! Not a valid embedded object. e. Mood disorders are common
Skin picking is seen at some time in 70% of people with Prader-Willi syndrome. Psychotic symptoms
are common (100% of older adults with uniparental disomic origin of Prader Willi syndrome were
psychotic with hallucinations).I139
Companion to Psychiatric Studies, 7th edn, pg 557
Incorrect
Marks for this submission: 0/1.
Question 16
Marks: 0/1
When assessing a patient with the features of early dementia which of the following would suggest a
sub-cortical rather than a cortical cause?
Choose one answer.
Error! Not a valid embedded object. a. Normal speed of cognitive processes
Error! Not a valid embedded object. b. Euthymic mood
Error! Not a valid embedded object. c. Absence of dysarthria
Error! Not a valid embedded object. d. Mild aphasia
Error! Not a valid embedded object. e. Calculation is preserved
Subcortical dementias are characterised by: no aphasia, dysarthria, visuospatial abilities damaged,
calculation preserved till end, apathetic personality, depressed mood, co-ordination damaged, motor
speed slowed + chorea and tics
Organic Psychiatry,Lishman,3 rd Edition P 452
Incorrect
Marks for this submission: 0/1.
Question 17
Marks: 0/1
The following is a measure of internal validity
Choose one answer.
Error! Not a valid embedded object. a. Reliability
Error! Not a valid embedded object. b. Relative risk
Error! Not a valid embedded object. c. Generalisability
Error! Not a valid embedded object. d. Intention to treat
Error! Not a valid embedded object. e. Numbers needed to treat
Reliability or accuracy is a measure of internal validity where as extrenal validity is the generalisability
of results
Godwin et al,Pragmatic controlled clinical trials in primary care;the struggle between external and
internal validity,BMC medical Resarch methodology,2003,3:28
Incorrect
Marks for this submission: 0/1.
Question 18
Marks: 0/1
What treatment would you use for a patient with severe symptoms of pre-menstrual syndrome
Choose one answer.
Error! Not a valid embedded object. a. Progesterone
Error! Not a valid embedded object. b. Fluoxetine
Error! Not a valid embedded object. c. Danazol
Error! Not a valid embedded object. d. Oligoacids
Error! Not a valid embedded object. e. Evening primrose oil
SSRIs have the strongest evidence for their effectiveness in the treatment of PMS.
Henshaw (2007) PMS: Diagnosis, aetiology, assessment and management: REVISITING...
PREMENSTRUAL SYNDROME. Advan. Psychiatr. Treat.13: 139 - 146.
Incorrect
Marks for this submission: 0/1.
Question 19
Marks: 0/1
A 14 year old boy is brought by his parents because of recurrent intrusive thoughts about
contamination. He recognises the thoughts as 'silly' but has started to develop rituals to cope with them.
Which of the following medications would be most appropriate?
Choose one answer.
Error! Not a valid embedded object. a. Clomipramine
Error! Not a valid embedded object. b. Citalopram
Error! Not a valid embedded object. c. Sertraline
Error! Not a valid embedded object. d. Fluoxetine
Error! Not a valid embedded object. e. Paroxetine
Sertraline is the first choice as it is licensed for use in OCD in Children and Adolescents. The others are
not recommended in child or adolescent patients
BNF, 2009
Incorrect
Marks for this submission: 0/1.
Question 20
Marks: 0/1
A young man comes to your outpatient clinic concerned that he may develop problem drinking. His
father and his aunt were heavy drinkers and were dependant on alcohol. What are is the increase in risk
of him becoming alcohol dependant.
Choose one answer.
Error! Not a valid embedded object. a. 4 fold
Error! Not a valid embedded object. b. 3 fold
Error! Not a valid embedded object. c. 6 fold
Error! Not a valid embedded object. d. 5 fold
Error! Not a valid embedded object. e. 2 fold
The riskof alcohol dependence in an individual with a first degree relative in increased twofold. There
is also an increased risk of antisocial PD, drug dependence, anxiety and mood disorder.
Nurnberger et al (2004) A Family Study of Alcohol Dependence: Coaggregation of Multiple disorders
in Relatives of Alcohol Dependent Probands. Arch Gen Psychiatry 61(12):1246-1256
Incorrect
Marks for this submission: 0/1.
Question 21
Marks: 0/1
Following is an example of Nominal data
Choose one answer.
Error! Not a valid embedded object. a. BDI score
Error! Not a valid embedded object. b. Gender
Error! Not a valid embedded object. c. Intelligent quotient
Error! Not a valid embedded object. d. Weight in kg
Error! Not a valid embedded object. e. Temperature in Celsius
These variables, also called "attribute variables" or "categorical variables," classify observations into a
small number of categories. A good rule of thumb is that an individual observation of a nominal
variable is usually a word, not a number. Examples of nominal variables include sex (the possible
values are male or female), genotype (values are AA, Aa, or aa), or ankle condition (values are normal,
sprained, torn ligament, or broken). Nominal variables are often used to divide individuals up into
classes, so that other variables may be compared among the classes. In the comparison of head width in
male vs. female isopods, the isopods are classified by sex, a nominal variable, and the measurement
variable head width is compared between the sexes.

Birmingham Course Notes, Paper III statistics, research, epidemiology, pg 39


Incorrect
Marks for this submission: 0/1.
Question 22
Marks: 0/1
According to Kernberg the psychodynamic understanding of borderline personality disorder did not
include
Choose one answer.
Error! Not a valid embedded object. a. Partially intact reality testing
Error! Not a valid embedded object. b. Characteristic object relations
Error! Not a valid embedded object. c. Identity diffusion
Error! Not a valid embedded object. d. Fully intact reality testing
Error! Not a valid embedded object. e. Primitive defences
The intrapsychic features pointed by Kernberg included partially intact reality testing, that is vulnerable
to alterations and failures because of aggression
Oxford Textbook of Psychotherapy, 2007, pg 292
Incorrect
Marks for this submission: 0/1.
Question 23
Marks: 0/1
The following is true regarding pharmacokinetics in the elderly:
Choose one answer.
Error! Not a valid embedded object. a. Drug interactions are common
Error! Not a valid embedded object. b. Volume of distribution reduced
Error! Not a valid embedded object. c. Decrease in body fat
Error! Not a valid embedded object. d. No change in renal function with age
Error! Not a valid embedded object. e. No change in hepatic function with age
In older people there is gradual reduction in renal & hepatic function with increase in body fat hence
leading to increased distribution of psychotropics.
Companion to Psychiatric Studies, 7th edn, pg 624
Incorrect
Marks for this submission: 0/1.
Question 24
Marks: 0/1
In a positive skew graph, which of the following is true?
Choose one answer.
Error! Not a valid embedded object. a. Mean greater than the mode
Error! Not a valid embedded object. b. Median greater than the mean
Error! Not a valid embedded object. c. Mean greater than the median
Error! Not a valid embedded object. d. Mean is equal to median & mode
Error! Not a valid embedded object. e. Mean of 1
The mean is higher than the median in positively (right) skewed distributions and lower than the
median in negatively (left) skewed distributions.
Companion to Psychiatric Studies, 7th edn, pg 186
Incorrect
Marks for this submission: 0/1.
Question 25
Marks: 0/1
The following would suggest severe lithium toxicity
Choose one answer.
Error! Not a valid embedded object. a. Hyporeflexia
Error! Not a valid embedded object. b. Nausea
Error! Not a valid embedded object. c. Blurred vision
Error! Not a valid embedded object. d. Convulsions
Error! Not a valid embedded object. e. Nystagmus
Severe lithium intoxication occurs at levels > than 2.5 meq /L and can cause generalised convulsions,
oliguria, renal failure and death. More moderate lithium intoxication causes nausea, blurred vision,
nystagmus and hypERreflexia
Synopsis of Psychiatry, 10th edn, pg 1060
Incorrect
Marks for this submission: 0/1.
Question 26
Marks: 0/1
Tolerance doesn't or is less likely to develop in the following in opiate dependence syndrome
Choose one answer.
Error! Not a valid embedded object. a. Miosis
Error! Not a valid embedded object. b. Respiratory depression
Error! Not a valid embedded object. c. Euphoria
Error! Not a valid embedded object. d. Sedation
Error! Not a valid embedded object. e. Constipation
Tolerance to miosis occurs at a much slower rate.
Seminars in Clinical Psychopharmacology by King, 2nd Edn, pg 504
Incorrect
Marks for this submission: 0/1.
Question 27
Marks: 0/1
Which method of randomisation should be avoided?
Choose one answer.
Error! Not a valid embedded object. a. Flip of a unbiased coin
Error! Not a valid embedded object. b. Odd/even number of birth dates
Error! Not a valid embedded object. c. Computer generated randomisation
Error! Not a valid embedded object. d. Simple randomisation
Error! Not a valid embedded object. e. Double blind randomisation
By assigning patients with odd / even number dates the two groups might differ significantly affecting
the results. To avoid this treatments should be assigned randomly.
Birmingham Course Notes, Paper III statistics, research, epidemiology, pg 23
Incorrect
Marks for this submission: 0/1.
Question 28
Marks: 0/1
A mother asks you what is the chances of her 12 year old son with conduct disorder going on to
develop antisocial personality disorder?
Choose one answer.
Error! Not a valid embedded object. a. 44%
Error! Not a valid embedded object. b. 11%
Error! Not a valid embedded object. c. 55%
Error! Not a valid embedded object. d. 33%
Error! Not a valid embedded object. e. 22%
A study conducted on a nationally representative sample from the National Epidemiologic Survey on
Alcohol and Related Conditions (NESARC) found that Approximately 75% of those with Conduct
disorder also met criteria for Antisocial personality disorder, contradicting the views from previous
studies that only about 40% of conduct disorder go on to have ASPD. Gelhorn H L. et al. DSM-IV
Conduct disorder criteria as predictors of antisocial personality disorder. Compr Psychiatry. 2007;
48(6): 529-538.
Incorrect
Marks for this submission: 0/1.
Question 29
Marks: 0/1
Which of the following is the characteristic feature in transient global amnesia?
Choose one answer.
Error! Not a valid embedded object. a. Abnormal blood test
Error! Not a valid embedded object. b. Imapired recall
Error! Not a valid embedded object. c. Retrograde memory loss
Error! Not a valid embedded object. d. Transient loss of short term memory
Error! Not a valid embedded object. e. Retained ability to retain new Information
Transient global amnesia manifests with a paroxysmal, transient loss of memory function. Immediate
recall ability is preserved, as is remote memory; however, patients experience striking loss of memory
for recent events and an impaired ability to retain new information. In some cases, the degree of
retrograde memory loss is mild.
Lishman's Organic Psychiatry, 4th edn, pg 33
Incorrect
Marks for this submission: 0/1.
Question 30
Marks: 0/1
A Psychiatrist has been asked to do a court report on a 34 year old charged with stalking. The most
common delusion associated with stalking is
Choose one answer.
Error! Not a valid embedded object. a. Erotomania
Error! Not a valid embedded object. b. Persecutory
Error! Not a valid embedded object. c. Jealousy
Error! Not a valid embedded object. d. Garndiosity
Error! Not a valid embedded object. e. Referential
The delusional conviction of being loved by someone who is identified but unattainable, usually by
reason of their social class or importance, is the basis of erotomania or de Clerambault's syndrome.
Technically, this represents a persistent delusional disorder than being part of schizophrenia.This forms
the majority of the sample of stalkers.
Kamphius J.S, & Emmelkamp, PMJ. Stalking - a contemporary challenge for the forensic & clinical
psychiatry. British Journal of Psychiatry, 2000, 176, 206-209.
Incorrect
Marks for this submission: 0/1.
Question 31
Marks: 0/1
In the following series of numbers 1, 2, 3, 7, 2, 5 . The following is correct
Choose one answer.
Error! Not a valid embedded object. a. cannot comment on mean, median or mode
Error! Not a valid embedded object. b. 2 is mode
Error! Not a valid embedded object. c. 3 is median
Error! Not a valid embedded object. d. 2 is median
Error! Not a valid embedded object. e. 2 is mean
The mode is the most frequent value. [The median is the middle value in a ranked list (in this case 2.5).
The mean is the sum of all observations dIvided by the number of observations (in this case 3.33).]
www.medicine.ox.ac.uk/bandolier/booth/glossary/mean.html
Incorrect
Marks for this submission: 0/1.
Question 32
Marks: 0/1
A patient who has recently completed an inpatient detoxification from opiates would like to be
prescribed something that will help to prevent relapse. Which of the following would be the most
appropriate?
Choose one answer.
Error! Not a valid embedded object. a. Methadone liquid
Error! Not a valid embedded object. b. Naltrexone
Error! Not a valid embedded object. c. Buprenorphine
Error! Not a valid embedded object. d. Naloxone
Error! Not a valid embedded object. e. Lofexidine
Buprenorphine and methadone have been recommended as Ist line in the maintenance treatment of
opiate dependence. Naltrexone is also used in the maintenance treatment but to prevent relapse, not as
first line.
NICE guidelines, 2007
Incorrect
Marks for this submission: 0/1.
Question 33
Marks: 0/1
Which of the following is not metabolised by CYP2D6?
Choose one answer.
Error! Not a valid embedded object. a. Paroxetine
Error! Not a valid embedded object. b. Citalopram
Error! Not a valid embedded object. c. Reboxetine
Error! Not a valid embedded object. d. Duloxetine
Error! Not a valid embedded object. e. Clomipramine
The noradrenaline-selective antidepressant reboxetine in vitro is a weak inhibitor of both cytochrome
P450 (CYP) 2D6 and CYP3A4.
Kuhn et al (2007) Reboxetine and cytochrome P450--comparison with paroxetine treatment in
humans.Int J Clin Pharmacol Ther. 45(1):36-46. & Synopsis of Psychiatry, 10th edn, pg 1085
Incorrect
Marks for this submission: 0/1.
Question 34
Marks: 0/1
Autism is most likely to be associated with which of the following:
Choose one answer.
Error! Not a valid embedded object. a. ADHD
Error! Not a valid embedded object. b. Asperger's syndrome
Error! Not a valid embedded object. c. Gilles De La Tourette
Error! Not a valid embedded object. d. Conduct disorder
Error! Not a valid embedded object. e. Williams syndrome
Hyperactivity is common with autism
Goodman & Scott, Child Psychiatry, 2002, pg 43
Incorrect
Marks for this submission: 0/1.
Question 35
Marks: 0/1
The following could commonly lead to oral contraceptive failure
Choose one answer.
Error! Not a valid embedded object. a. Lithium
Error! Not a valid embedded object. b. Olanzapine
Error! Not a valid embedded object. c. Carbamazepine
Error! Not a valid embedded object. d. Clozapine
Error! Not a valid embedded object. e. Sodium Valproate
Carbamazepine is a potent inducer of hepatic cytochrome P450 enzymes and is metabolised by
CYP3A4. Patients receiving contraception should either receive a preparation containing less than 50
micrograms of oestrogen or use a non hormonal method.
Maudsley Prescribing Guidelines, 2009, 9th edn, pg 136
Incorrect
Marks for this submission: 0/1.
Question 36
Marks: 0/1
Apperceptive agnosia is the inability to
Choose one answer.
Error! Not a valid embedded object. a. Recognise objects as whole
Error! Not a valid embedded object. b. Identify objects
Error! Not a valid embedded object. c. Describe objects that are seen
Error! Not a valid embedded object. d. Name objects on tactile contact
Error! Not a valid embedded object. e. Copy drawings
Apperceptive agnosics cannot copy objects or drawings unless they do so lavishly and very slowly,
often producing frequent errors.
Lishman's Organic Psychiatry, 4th edn, pg 64
Incorrect
Marks for this submission: 0/1.
Question 37
Marks: 0/1
A 40 year old man who is a heavy drinker does not recognise that he requires to change and does not
accept his problem drinking in alcohol. Regarding his motivation to change he is in the stage of
Choose one answer.
Error! Not a valid embedded object. a. Action
Error! Not a valid embedded object. b. Contemplation
Error! Not a valid embedded object. c. Precontemplation
Error! Not a valid embedded object. d. Preparation
Error! Not a valid embedded object. e. Maintenance
At the precontemplation stage there is no intention to change and patients are not even aware of their
problem. Secret drinking is seen in this stage.
Handbook of Alcoholism, 2000, Zernig et al, pg 99-100
Incorrect
Marks for this submission: 0/1.
Question 38
Marks: 0/1
The following is commonly seen in a patient with anorexia nervosa
Choose one answer.
Error! Not a valid embedded object. a. Hypertension & Tachycardia
Error! Not a valid embedded object. b. Hypotension & tachycardia
Error! Not a valid embedded object. c. Hypotension & bradycardia
Error! Not a valid embedded object. d. Ventricular arrhythymias
Error! Not a valid embedded object. e. Hypertension & bradycardia
In a community sample of 214 women with anorexia nervosa, the common medical findings included;
anemia, 38.6%; leukocytopenia, 34.4%; hyponatremia, 19.7%; hypokalemia, 19.7%; bradycardia,
41.3%; hypotension, 16.1%; hypothermia, 22.4%; elevation of alanine aminotransferase concentration,
12.2%; osteopenia, 51.7%; osteoporosis, 34.6%; and primary amenorrhea, 14.8%.
Miller et al. Common Findings in outpatients with Anorexia Nervosa. Arch Intern Med. 2005;165:561-
566.
Incorrect
Marks for this submission: 0/1.
Question 39
Marks: 0/1
In interpersonal therapy, which is the most appropriate stance for the therapist to take?
Choose one answer.
Error! Not a valid embedded object. a. To act as the patient's advocate
Error! Not a valid embedded object. b. Transference-based stance
Error! Not a valid embedded object. c. Stance based on friendship
Error! Not a valid embedded object. d. Passive role
Error! Not a valid embedded object. e. Neutral stance
The therapeutic role of a therapist in IPT is to be a patients advocate not neutral, be active not passive,
therapeutic relationship is not interpreted as transference and is not a friendship.
Interpersonal therapy for depression, Klerman, Basic Books, 1984, pg 73-76
Incorrect
Marks for this submission: 0/1.
Question 40
Marks: 0/1
Common side effects of Cyproterone acetate inlcudes
Choose one answer.
Error! Not a valid embedded object. a. Decrased libido
Error! Not a valid embedded object. b. Liver problems
Error! Not a valid embedded object. c. Rash
Error! Not a valid embedded object. d. Hypersentivity
Error! Not a valid embedded object. e. Galactorrhea
The commonest side effects (affecting 1 in 10) include decreased libido, reduced sperm production,
impotence and reduced volume of sperm.
Click here for reference
Incorrect
Marks for this submission: 0/1.
Question 41
Marks: 0/1
The most common side effect of Rivastigmine is
Choose one answer.
Error! Not a valid embedded object. a. Nausea
Error! Not a valid embedded object. b. Constipation
Error! Not a valid embedded object. c. Sedation
Error! Not a valid embedded object. d. Agitation
Error! Not a valid embedded object. e. Abdominal pain
The most common adverse effects associated with Rivastigmine are nausea, vomitting, dizziness,
headache, diarrhoea, abdominal pain, anorexia , fatigue, and somnolence
Synopsis of Psychiatry, 10th edn, pg 1034
Incorrect
Marks for this submission: 0/1.
Question 42
Marks: 0/1
In PANSS which of the following is considered to be a positive symptom?
Choose one answer.
Error! Not a valid embedded object. a. Social withdrawal
Error! Not a valid embedded object. b. Conceptual disorganisation
Error! Not a valid embedded object. c. Apathy
Error! Not a valid embedded object. d. Affective blunting
Error! Not a valid embedded object. e. Somatic passivity
The items on the positive scale include Delusions, Conceptual disorganization, Hallucinatory behavior,
Excitement, Grandiosity, Suspiciousness and Hostility.
Kay et al. (1987) The Positive and Negative Syndrome Scale (PANSS) for Schizophrenia.
Schizophrenia Bulletin 13(2): 261-76
Incorrect
Marks for this submission: 0/1.
Question 43
Marks: 0/1
What is the most likely diagnosis that an in-patient with HIV is most likely to have?
Choose one answer.
Error! Not a valid embedded object. a. Anxiety
Error! Not a valid embedded object. b. Depression
Error! Not a valid embedded object. c. Dementia
Error! Not a valid embedded object. d. Delirium
Error! Not a valid embedded object. e. Psychosis
The commonest CNS manifestations in patients with HIV disease has emerged as HIV associated
dementia.
Lishman's Organic Psychiatry, 4th edn, pg 408
Incorrect
Marks for this submission: 0/1.
Question 44
Marks: 0/1
In autism the following is true
Choose one answer.
Error! Not a valid embedded
a. Hereditability over 90%
object.
Error! Not a valid embedded
b. Perinatal complications not associated with autism
object.
Error! Not a valid embedded
c. Not associated with Fragile X syndrome
object.
Error! Not a valid embedded d. Rates have been found to be higher in patients exposed to the
object. MMR vaccine
Error! Not a valid embedded
e. Monozygotic concordance rate less than 10%
object.
Early studies of twins estimated the heritability of autism to be more than 90%; in other words, that
90% of the differences between autistic and non-autistic individuals is due to genetic effects.
[Monozygotic concordance is >90%. Autism is seen in around 25% of male patients with fragile X and
6% of females. Perinatal complications are associated with autism (there is a higher prevalence of
complications, although note that this may not be causal).]
Rutter (2000) Genetic Studies of Autism: From the 1970s into the Millenium. Journal of Abnormal
Child Psychology. 28(1) 3-14 and Abrahams BS, Geschwind DH (2008) Advances in autism genetics:
on the threshold of a new neurobiology. Nat Rev Genet 9 (5): 341-55.
Incorrect
Marks for this submission: 0/1.
Question 45
Marks: 0/1
Which one of the following is recognised to cause schizophrenia like psychosis?
Choose one answer.
Error! Not a valid embedded object. a. Huntington's disease
Error! Not a valid embedded object. b. Panic disorder
Error! Not a valid embedded object. c. OCD
Error! Not a valid embedded object. d. Generalised anxiety disorder
Error! Not a valid embedded object. e. Heroin dependance
Huntington's disease can present with florid schizophrenic symptoms and can be present even before
the true diagnosis can become apparent.
Lishman's Organic Psychiatry, 4th Edn, pg 580
Incorrect
Marks for this submission: 0/1.
Question 46
Marks: 0/1
A journalists asks about the probability of an offender released from medium-secure units going on to
commit a further offence?
Choose one answer.
Error! Not a valid embedded object. a. 35%
Error! Not a valid embedded object. b. 5%
Error! Not a valid embedded object. c. 15%
Error! Not a valid embedded object. d. 55%
Error! Not a valid embedded object. e. 25%
Though the rate of reoffending amongst people released from a secure units varies from 7% to 50%, the
definite answer is from the Sainsbury Center for Mental health from the data from England and Wales,
the rates are actually around 7%. Max Rutherford and Sean Duggan. Forensic mental health services:
facts and ?gures on current provision. Forensic Practice, 10 (4), 2008, 4-10.
Incorrect
Marks for this submission: 0/1.
Question 47
Marks: 0/1
In an RCT 50 patients are allocated placebo and 50 are allocated to a treatment group. However 6
patients refuse to take the active treatment. Which of the following is true?
Choose one answer.
Error! Not a valid a. Patients who refused drug treatment should be excluded from the
embedded object. trial
Error! Not a valid
b. The patients who refused should be analysed as separate group
embedded object.
Error! Not a valid c. Patients who refused drug treatment should be included in the
embedded object. treatment arm during analysis
Error! Not a valid d. Patients who refused active treatment should be included in the
embedded object. placebo arm during analysis
Error! Not a valid
e. The results of this study cannot be analysed
embedded object.
All trials have patients / subjects who drop out. If dropouts are ignored, the results of the analysis may
not provide the true picture. Results should usually be reported on an intention-to-treat (ITT) basis,
where everyone who was randomised to a particula rgroup is included in the same group for final
analysis. It avoids bias associated with non random loss of participants.Common methods of ITT
include last observation carried forward (LOCF), worst case scenario (assume dropouts in treatment
arm have worst outcome and in control group have best outcome), and mean imputation.
Click here for reference
Incorrect
Marks for this submission: 0/1.
Question 48
Marks: 0/1
Smoking does not affect the metabolism of which of the following medication?
Choose one answer.
Error! Not a valid embedded object. a. Lithium
Error! Not a valid embedded object. b. Haloperidol
Error! Not a valid embedded object. c. Clozapine
Error! Not a valid embedded object. d. Fluvoxamine
Error! Not a valid embedded object. e. Olanzapine
Tobacco smoke contains aromatic hydrocarbons that induce the hepatic enzyme CYP1A2 and hence
reduces the level of psychotropics in the plasma. Lithium is predominantly excreted by the kidneys and
is not metabolised.
Maudsley Prescribing Guidelines, 2009, 9th edn, pg 492-3
Incorrect
Marks for this submission: 0/1.
Question 49
Marks: 0/1
Which of the following is the most common neurological side effect of fluoxetine?
Choose one answer.
Error! Not a valid embedded object. a. Hallucinations
Error! Not a valid embedded object. b. Blurred vision
Error! Not a valid embedded object. c. Peripheral neuropathy
Error! Not a valid embedded object. d. Mania
Error! Not a valid embedded object. e. Tremor
Other common adverse effects are dry mouth, nervousness, anxiety, headache, insomnia, and dizziness.
BNF, 2009, pg 213
Incorrect
Marks for this submission: 0/1.
Question 50
Marks: 0/1
A patient is seen in clinic with yawning, dysphoria and dilated pupils. What is the most likely
diagnosis?
Choose one answer.
Error! Not a valid embedded object. a. Cannabis intoxication
Error! Not a valid embedded object. b. Opiate intoxication
Error! Not a valid embedded object. c. Amphetamine intoxication
Error! Not a valid embedded object. d. Heroin intoxication
Error! Not a valid embedded object. e. Benzodiazepine intoxication
Amphetamine causes non life threatening effects such as pupilllary dilation, dysphoria and yawning in
addition to life threatening effects (MI, severe hypertension, ischaemic collitis).
Synopsis of Psychiatry, 10th edn, pg 410
Incorrect
Marks for this submission: 0/1.
Question 51
Marks: 0/1
A drug is considered to be teratogenic if it raises the level of birth abnormalities above the level of
expected prevalence. What is the level of expected prevalence?
Choose one answer.
Error! Not a valid embedded object. a. 2-4%
Error! Not a valid embedded object. b. 4-6%
Error! Not a valid embedded object. c. 7-9%
Error! Not a valid embedded object. d. 10-12%
Error! Not a valid embedded object. e. 0-2%
The UK national teratology information service suggest that the average risk of congenital
abnormalities is 2-3% this increases to 10% in a woman taking lithium during pregnancy.
Companion to Psychiatric Studies, 7th edn, pg 750
Incorrect
Marks for this submission: 0/1.
Question 52
Marks: 0/1
Which of the following personality disorders is most commonly seen in patients over the age of 60 who
commit suicide?
Choose one answer.
Error! Not a valid embedded object. a. Histrionic
Error! Not a valid embedded object. b. Schizoid
Error! Not a valid embedded object. c. Paranoid
Error! Not a valid embedded object. d. Dependant
Error! Not a valid embedded object. e. Anxious
The main results of a case control study looking into suicides in older people found that 77% of the
suicide sample had a psychiatric disorder at the time of death, most often depression (63%). Personality
disorder or personality trait accentuation was present in 44%, with anankastic or anxious traits the most
frequent. Depression, personality disorder, and personality trait accentuation emerged as predictors of
suicide in the case-control analysis. The conclusion was that personality factors, as well as depression,
are important risk factors for suicide in older people.
Harwood et al. (2001) Psychiatric disorder and personality factors associated with suicide in older
people: A descriptive and case-control study. International Journal of Geriatric Psychiatry 16(2): 155-
165
Incorrect
Marks for this submission: 0/1.
Question 53
Marks: 0/1
Patient experienced lithium toxicity: all the following are associated with lihtium toxicity except
Choose one answer.
Error! Not a valid embedded object. a. Fluoxetine
Error! Not a valid embedded object. b. Thiazides
Error! Not a valid embedded object. c. Captopril
Error! Not a valid embedded object. d. Aminophylline
Error! Not a valid embedded object. e. NSAID
The following drugs can increase the risk of Lithium toxicity including ACE inhibitors, Diuretics,
NSAIDS, Carbamazepine, SSRIs.
Maudsley Prescribing Guidelines, 2009, 9th edn, pg 132-3
Incorrect
Marks for this submission: 0/1.
Question 54
Marks: 0/1
The following is a feature of Progressive supranuclear palsy:
Choose one answer.
Error! Not a valid embedded object. a. Onset in the 4th decade
Error! Not a valid embedded object. b. Neurofibrillary tangles in the cortex
Error! Not a valid embedded object. c. Frequent falling forwards
Error! Not a valid embedded object. d. Lateral gaze problems are an early feature
Error! Not a valid embedded object. e. Bradyphrenia common
In supranuclear palsy the onset is usually in the 6th decade; ocular symptoms affect the vertical plane
involving downward gaze; there are frequent backwards falls and neurofibrillary tangles are seen in the
basal ganglia, brainstem and cerebellar nuclei.
Lishman's Organic Psychiatry, 4th edn, pg 776
Incorrect
Marks for this submission: 0/1.
Question 55
Marks: 0/1
The ideal value for kappa is
Choose one answer.
Error! Not a valid embedded object. a. 0
Error! Not a valid embedded object. b. -1
Error! Not a valid embedded object. c. -0.5
Error! Not a valid embedded object. d. 0.5
Error! Not a valid embedded object. e. 1
Kappa is a measure of inter rater reliability. If the raters are in complete agreement then ? = 1. If there
is no agreement among the raters (other than what would be expected by chance) then ? = 0.
Companion to Psychiatric Studies, 7th edn, pg 189
Incorrect
Marks for this submission: 0/1.
Question 56
Marks: 0/1
A mother asks you which category of schizophrenia has the 'best' prognosis:
Choose one answer.
Error! Not a valid embedded object. a. Residual schizophrenia
Error! Not a valid embedded object. b. Simple schizophrenia
Error! Not a valid embedded object. c. Hebephrenic schizophrenia
Error! Not a valid embedded object. d. Catatonic schizophrenia
Error! Not a valid embedded object. e. Paranoid schizophrenia
Hebephrenic and catatonic forms tend to have the worst prognosis and the paranoid form the best.
Companion to Psychiatric Studies, 7th edn, pg 397
Incorrect
Marks for this submission: 0/1.
Question 57
Marks: 0/1
Which of the following is adjusted for the age of the population?
Choose one answer.
Error! Not a valid embedded object. a. MMR
Error! Not a valid embedded object. b. SMR
Error! Not a valid embedded object. c. Crude mortality rate
Error! Not a valid embedded object. d. Mortality rate
Error! Not a valid embedded object. e. Perinatal rate
Standardized mortality ratio (indirect age adjustment) tells how many persons, per thousand of the
population, will die in a given year and what the causes of death will be. Thus, for each age group, the
number of deaths expected is calculated, and these numbers are totalled. The numbers of deaths that
were actually observed in that population are also calculated and totalled. The ratio of the total number
of deaths actually observed to the total number of deaths expected, if the population of interest had had
the mortality experience of the known population, is then calculated. This ratio is called the
standardized mortality ratio (SMR). The SMR is defined as follows: SMR = (Observed no. of deaths
per year)/(Expected no. of deaths per year).

Gordis, Leon (December 6, 2004). Epidemiology. Saunders; 3 edition. pp.64-65.


Incorrect
Marks for this submission: 0/1.
Question 58
Marks: 0/1
One of the following is less accurate than others
Choose one answer.
Error! Not a valid embedded
a. Recommended treatment in drug induced psychosis
object.
Error! Not a valid embedded
b. Recommended treatment in depression
object.
Error! Not a valid embedded
c. Recommended treatment in paranoid schizophrenia
object.
Error! Not a valid embedded d. Unilateral ECT causes more memory problems than B/L ECT
object.
Error! Not a valid embedded
e. Recommended treatment in acute mania
object.
ECT has evidence of effectiveness in depression, schizophrenia, mania, catatonia, drug induced
psychosis( L Dopa etc) states in patients with Parkinson's disease, epileptic psychosis.NICE
recommends the use of ECT in severe depressive illness, catatonia & a prolonged or severe manic
episode. B/L ECT has been associated with a higher incidence of memory problems. NICE guidelines
for ECT, 2003
Incorrect
Marks for this submission: 0/1.
Question 59
Marks: 0/1
An infarct in which region leads to an increased rate of depression:
Choose one answer.
Error! Not a valid embedded object. a. Medulla
Error! Not a valid embedded object. b. Occipital lobe
Error! Not a valid embedded object. c. Basal Ganglia
Error! Not a valid embedded object. d. Cerebellum
Error! Not a valid embedded object. e. Parietal lobe
MRI-documented infarcts in hippocampus, basal ganglia and frontal areas have been classically
associated with post stroke depression.
Frodl et al. (2004) Hippocampal and amygdala changes in patients with major depressive disorder and
healthy controls during a 1-year follow-up. J Clin Psychiatry. 65:492-499.
Incorrect
Marks for this submission: 0/1.
Question 60
Marks: 0/1
Which of the below increases the power of a study:
Choose one answer.
Error! Not a valid embedded object. a. Smaller numbers of participants
Error! Not a valid embedded object. b. Increased heterogeneity
Error! Not a valid embedded object. c. Comparing active treatment vs placebo
Error! Not a valid embedded object. d. Comparing active treatment vs. active treatment
Error! Not a valid embedded object. e. A scale with a lot of subscales
The power of the study depends on the sample size, effect size, reliability of the measures used, and the
significance level adopted in the study. Effect size is the difference between the two groups being
compared, which would be maximum between active treatment and no treatment (placebo)
Incorrect
Marks for this submission: 0/1.
Question 61
Marks: 0/1
Smoking in a pregnant lady can lead to the following in the child
Choose one answer.
Error! Not a valid embedded object. a. Learning disability
Error! Not a valid embedded object. b. Autistic spectrum disorder
Error! Not a valid embedded object. c. Mood disorders
Error! Not a valid embedded object. d. Autism
Error! Not a valid embedded object. e. ADHD
Smoking in pregnancy is associated with ADHD and behaviour problems including conduct disorders
and antisocial behaviour in the child
Langley K et al. (2007) Effects of low birth weight, maternal smoking in pregnancy and social class on
the phenotypic manifestation of Attention Deficit Hyperactivity Disorder and associated antisocial
behaviour: investigation in a clinical sample. BMC Psychiatry 7:26
Incorrect
Marks for this submission: 0/1.
Question 62
Marks: 0/1
An anorexic patient presents with shortness of breath and fainting episodes. Which of the following
would account for this?
Choose one answer.
Error! Not a valid embedded object. a. Pulmonary embolism
Error! Not a valid embedded object. b. Anaemia
Error! Not a valid embedded object. c. Cardiomyopathy
Error! Not a valid embedded object. d. Ist degree heart block
Error! Not a valid embedded object. e. Cardiac arrhythmias
Anaemia is a common haematological finding in Anorexia and can manifest with shortness of breath
and fainting spells.
Oxford Handbook of Clinical Medicine, 7th edn, pg 310 and Companion to Psychiatric Studies, 7th
edn, pg 492
Incorrect
Marks for this submission: 0/1.
Question 63
Marks: 0/1
A woman lost her mother recently. A feature which would take your diagnosis towards depression
rather than normal grief reaction
Choose one answer.
Error! Not a valid embedded object. a. Generalized guilt
Error! Not a valid embedded object. b. Loss of appetite
Error! Not a valid embedded object. c. Severe insomnia
Error! Not a valid embedded object. d. Global loss of self esteem
Error! Not a valid embedded object. e. Auditory hallucinations
The symptoms of retardation and global loss of self esteem are seldom present in uncomplicated grief
Shorter Oxford Textbook of Psychiatry, 5th edn, pg 170
Incorrect
Marks for this submission: 0/1.
Question 64
Marks: 0/1
A 16 year old boy is referred to you because he is found drunk in school just before he was about to
give a presentation. He has stopped going out with his friends and has recently refused to go out for a
meal with his family members. What is the most likely diagnosis?
Choose one answer.
Error! Not a valid embedded object. a. Separation anxiety
Error! Not a valid embedded object. b. Adjustment disorder
Error! Not a valid embedded object. c. Social phobia
Error! Not a valid embedded object. d. Simple phobia
Error! Not a valid embedded object. e. Agoraphobia
The essence of social phobia is persistent fear of social situations in which embarrassment occurs. It
can lead to excessive alcohol intake and alcohol dependence syndrome.
Companion to Psychiatric Studies, 7th edn, pg 470-1
Incorrect
Marks for this submission: 0/1.
Question 65
Marks: 0/1
Which of the following would you see abnormal language development in?
Choose one answer.
Error! Not a valid embedded object. a. Aspergers
Error! Not a valid embedded object. b. Autism
Error! Not a valid embedded object. c. Tourettes syndrome
Error! Not a valid embedded object. d. ADHD
Error! Not a valid embedded object. e. Conduct disorder
Deficits in language development and difficulty using language to communicate ideas are among the
principal criteria for diagnosing autistic disorder. Language is not affected in the other choices
mentioned.
Synopsis of Psychiatry, 10th edn, pg 1194
Incorrect
Marks for this submission: 0/1.
Question 66
Marks: 0/1
People with emotionally unstable borderline personality disorder are in
Choose one answer.
Error! Not a valid embedded object. a. False self position
Error! Not a valid embedded object. b. Paranoid-schizoid position
Error! Not a valid embedded object. c. Klein's depressive position
Error! Not a valid embedded object. d. Schizoid position
Error! Not a valid embedded object. e. Oedipal stage
Kernberg suggested that patients with BPD internalize early pathologic object relations. The use of
primitive defense mechanisms (which individuals without BPD outgrow during normal development)
maintains these early pathologic object relations. Kernberg hypothesized that, in the early stages of
development, the infant experiences the maternal figure in 2 contradictory ways: The first is the good
mother, who provides for, loves, and remains close. The second is the hateful, depriving mother, who
unpredictably punishes and abandons the child. The result is intense anxiety, which leads to the
borderline defense of splitting.
Borderline Personality Disorder, Lubit & Finley-Belgrad, Borderline Personality Disorder: eMedicine
Psychiatry
Incorrect
Marks for this submission: 0/1.
Question 67
Marks: 0/1
The following gives an association of the rate of the disease development
Choose one answer.
Error! Not a valid embedded object. a. Standardised mortality rate
Error! Not a valid embedded object. b. Crude death rate
Error! Not a valid embedded object. c. Prevalence rate
Error! Not a valid embedded object. d. Incidence rate
Error! Not a valid embedded object. e. Mortality rate
Incidence or inception rate of a disease refers to the rate of new cases arising
Understanding clinical papers ,David Bowers,House & David Owens,p-89
Incorrect
Marks for this submission: 0/1.
Question 68
Marks: 0/1
Which of the following is true regarding Fragile X?
Choose one answer.
Error! Not a valid embedded object. a. IQ improves with age
Error! Not a valid embedded object. b. IQ is stable in adult patients
Error! Not a valid embedded object. c. Severity depends on the number of CGG repeats
Error! Not a valid embedded object. d. Cognitive deterioration is a feature
Error! Not a valid embedded object. e. IQ correlates with physical features
Cognitive decline is a feature of fragile X syndrome. IQ declines with age and CGG counts of greater
than 200 leads to fragile X syndrome
Companion to Psychiatric Studies, 7th edn, pg 549
Incorrect
Marks for this submission: 0/1.
Question 69
Marks: 0/1
You see a patient with dementia, who also has a peripheral neuropathy. A tumour in which of the
following could cause this presentation?
Choose one answer.
Error! Not a valid embedded object. a. Brain
Error! Not a valid embedded object. b. Colon
Error! Not a valid embedded object. c. Lungs
Error! Not a valid embedded object. d. Liver
Error! Not a valid embedded object. e. Kidneys
Cancers of the lung can cause peripheral neuropathies,confusion, cerebellar syndrome and
polymyositis.
Oxford Handbook of Clinical Medicine, 7th edn, pg162
Incorrect
Marks for this submission: 0/1.
Question 70
Marks: 0/1
Which of the following drugs is the most common cause of Syndrome of Inappropriate Antidiuretic
Hormone Secretion
Choose one answer.
Error! Not a valid embedded object. a. Citalopram
Error! Not a valid embedded object. b. Risperidone
Error! Not a valid embedded object. c. Quetiapine
Error! Not a valid embedded object. d. Lithium
Error! Not a valid embedded object. e. Olanzapine
SSRI antidepressants have been reported to be associated with hyponatraemia than other
antidepressants and psychotropics .
Maudsley Prescribing Guidelines, 2009, 9th edn, pg193
Incorrect
Marks for this submission: 0/1.
Question 71
Marks: 0/1
An elderly gentle man has been diagnosed with delusional disorder. The following is true about
delusional disorder
Choose one answer.
Error! Not a valid embedded a. Persecutory delusions are seen in 25% of those with diagnosis
object.
Error! Not a valid embedded
b. The most common type is somatic delusions
object.
Error! Not a valid embedded
c. Grandiose delusions are the most common type
object.
Error! Not a valid embedded
d. Older age at onset is associated with somatic delusions
object.
Error! Not a valid embedded
e. A persecutory subtype is the commonest one
object.
Persecutory delusions are the most common type in all populations, (up to 60%) followed by somatic in
young ages and then grandiose dellusions.
Yamada et al (1998) Age at onset of delusional disorder is dependent on the delusional theme. Acta
Psychiatrica Scandinavica. 97(2):122 - 124
Incorrect
Marks for this submission: 0/1.
Question 72
Marks: 0/1
Containment is best described as the
Choose one answer.
Error! Not a valid
a. Patient's ability to deal with anger
embedded object.
Error! Not a valid
b. Patient's ability to deal with unpleasant emotions
embedded object.
Error! Not a valid c. Therapist's ability to keep the emotions elicited by the patient to
embedded object. himself
Error! Not a valid d. Therapist's ability to keep the actions elicited by the patient to himself
embedded object.
Error! Not a valid e. Ability of the therapist to modify and return the patient's difficult
embedded object. material in a way that he (she) can tolerate
Containment in psychotherapy is the ability of the therapist to interact with the patient in order to help
him understand what he is experiencing in an acceptable way to the patient.
Textbook of Psychotherapy in Psychiatric Practice, Holmes, Churchill Livingstone, pg 321
Incorrect
Marks for this submission: 0/1.
Question 73
Marks: 0/1
The most commonly used antidepressant in a patient with post-stroke depression
Choose one answer.
Error! Not a valid embedded object. a. Citalopram
Error! Not a valid embedded object. b. Sertraline
Error! Not a valid embedded object. c. Paroxetine
Error! Not a valid embedded object. d. Venlafaxine
Error! Not a valid embedded object. e. Fluoxetine
There are more studies showing efficacy of fluoxetine in post stroke depression. There is evidence for
citalopram as well as TCAs.
Whytea E & Mulsant B (2002) Post stroke depression: epidemiology, pathophysiology, and biological
treatment Biological Psychiatry 52(3): 253-264
Incorrect
Marks for this submission: 0/1.
Question 74
Marks: 0/1
A patient with Parkinson's disease becomes distressingly psychotic, which of the following can be
used.
Choose one answer.
Error! Not a valid embedded object. a. Clozapine
Error! Not a valid embedded object. b. Quetiapine
Error! Not a valid embedded object. c. Lorazepam
Error! Not a valid embedded object. d. Olanzapine
Error! Not a valid embedded object. e. Amitryptilline
According to a 2007 meta-analysis, clozapine is the only atypical antipsychotic fully recommended for
the treatment of Parkinson's disease psychosis due to its demonstrated efficacy and tolerability. Though
there were intial open label trials that showed good response with Quetiapine, subsequent randomised
trials have not replicated these, hence the only available evidence is for low dose Clozapine.

Low-dose clozapine for the treatment of drug-induced psychosis in Parkinson's disease. The Parkinson
Study Group. N Engl J Med. 1999 Mar 11;340(10):757-63.
Zahodne LB, and Fernandez H (2008) Course, Prognosis, and Management of Psychosis in Parkinson's
Disease: Are Current Treatments Really Effective? CNS Spectr. 13(3 Suppl 4):26-33.
Rabbey et al. Effect of quetiapine in psychotic Parkinson's disease
patients: A double-blind labeled study of 3 months' duration.
Movement Disorders 2007, 22 (3), Pages 313 - 318
Incorrect
Marks for this submission: 0/1.
Question 75
Marks: 0/1
A man saw his friend seriously injured in a car he was driving. He now avoids driving to work that
way. What is the most likely diagnosis?
Choose one answer.
Error! Not a valid embedded object. a. Adjustment reaction
Error! Not a valid embedded object. b. Malingering
Error! Not a valid embedded object. c. PTSD
Error! Not a valid embedded object. d. Panic disorder
Error! Not a valid embedded object. e. Agoraphobia
The patient has symptoms of PTSD mainly in the form of avoidance behaviour which is one of the 3
clusters of symptom seen in PTSD. In adjustment reactions the stress is less catastrophic and the
psychological symptoms are less specific.
Companion to Psychiatric Studies, 7th edn, pg 477
Incorrect
Marks for this submission: 0/1.
Question 76
Marks: 0/1
Which correlation co-efficient demonstrates an inverse relationship between two variables?
Choose one answer.
Error! Not a valid embedded object. a. 0.5
Error! Not a valid embedded object. b. 1
Error! Not a valid embedded object. c. -1
Error! Not a valid embedded object. d. -0.5
Error! Not a valid embedded object. e. 0
The strength or magnitude of the relationship between the two variables is expressed by a statistic
known as the CORRELATION COEFFICIENT, which varies from -1 (perfect negative relationship or
inverse correlation ), through 0 (no relationship) to +1 (perfect positive relationship). Intuitive
biostatistics, Harvey Motulsky, pg 185
Incorrect
Marks for this submission: 0/1.
Question 77
Marks: 0/1
The lifetime prevalence of OCD is
Choose one answer.
Error! Not a valid embedded object. a. 2-3%
Error! Not a valid embedded object. b. 4-5%
Error! Not a valid embedded object. c. 5-10%
Error! Not a valid embedded object. d. 1-3%
Error! Not a valid embedded object. e. 1-2%
The life time prevalence varies from about 1 to 3 % in studies
New Oxford Textbook of Psychiatry, Vol 1, pg 823
Incorrect
Marks for this submission: 0/1.
Question 78
Marks: 0/1
What is the risk of neural tube defects with a pregnant lady taking sodium valproate?
Choose one answer.
Error! Not a valid embedded object. a. 1 in 50
Error! Not a valid embedded object. b. 1 in 100
Error! Not a valid embedded object. c. 1 in 1000
Error! Not a valid embedded object. d. 1 in 10000
Error! Not a valid embedded object. e. 1 in 10
The risk of neural tube defects induced by valproate is about 1-4% of all women who take valproate
during the first trimester of pregnancy.
Synopsis of Psychiatry, 10th edn, pg 1114
Incorrect
Marks for this submission: 0/1.
Question 79
Marks: 0/1
A 34 year old woman with a diagnosis of schizophrenia also complains of apathy and fatigue,
accompanied by abdominal bloating and constipation. Which of the following is most likely to cause
this?
Choose one answer.
Error! Not a valid embedded object. a. Hypoglycaemia
Error! Not a valid embedded object. b. Hypercalcaemia
Error! Not a valid embedded object. c. Hyponatraemia
Error! Not a valid embedded object. d. Hypocalcaemia
Error! Not a valid embedded object. e. Hyperglycaemia
Hypercalcaemia is associated with STONES (kidney), Abdominal GROANS (abdo pain &
constipation) and psychic MOANS (depression & fatigue).
Oxford Handbook of Clinical Medicine, 7th edn, pg 672
Incorrect
Marks for this submission: 0/1.
Question 80
Marks: 0/1
You would like to treat a 36 year gentleman with an alpha 2 agonist: which of the following you are
going to start as the patient has tendency to suffer low blood pressure?
Choose one answer.
Error! Not a valid embedded object. a. Methadone Liquid
Error! Not a valid embedded object. b. Clonidine
Error! Not a valid embedded object. c. Lofexidine
Error! Not a valid embedded object. d. Buprenorphine
Error! Not a valid embedded object. e. Naltrexone
Buprenorphine or methadone are suggested as the first line in the treatment. Only lofexedine and
clonidine are alpha 2 agonists. Lofexidine is less likely to cause hypotension compared to clonidine
hence this could be the choice if the patient prefers not to use methadone or buprenorphine.
NICE guidelines, 2007
Incorrect
Marks for this submission: 0/1.
Question 81
Marks: 0/1
Visual field defects are seen with
Choose one answer.
Error! Not a valid embedded object. a. Levetiracetam
Error! Not a valid embedded object. b. Sodium Valproate
Error! Not a valid embedded object. c. Vigabatrin
Error! Not a valid embedded object. d. Gabapentin
Error! Not a valid embedded object. e. Lamotrigine
Vigabatrin is associated with bilateral visual field defects.
Wilton et al. (1999) Visual field defect associated with vigabatrin: observational cohort study. BMJ
319:1165-1166
Incorrect
Marks for this submission: 0/1.
Question 82
Marks: 0/1
A woman who has had Herpes Encephalitis develops severe carbohydrate craving, weight gain and
increased sexual drive. What is the likely diagnosis?
Choose one answer.
Error! Not a valid embedded object. a. Cushing's syndrome
Error! Not a valid embedded object. b. Hypothyroidism
Error! Not a valid embedded object. c. Prader Willi syndrome
Error! Not a valid embedded object. d. Kluver Bucy syndrome
Error! Not a valid embedded object. e. Diabetes mellitus
Kluver Bucy syndrome is associated with overeating and hypersexuality (as well as placidity and
hyperoralisty). Prader Willi is a genetic disorder associated with overeating but not hypersexuality.
Cushing's syndrome and hypothyroidism are associated with weight gain but not hypersexuality.
Textbook of Clinical Neuropsychiatry, Moore, 2nd Edn, pg 127
Incorrect
Marks for this submission: 0/1.
Question 83
Marks: 0/1
The risk of heart failure in patients with coronary artery disease and depression compared to those with
coronary artery disease without depression is
Choose one answer.
Error! Not a valid embedded object. a. 2:1
Error! Not a valid embedded object. b. 3:1
Error! Not a valid embedded object. c. 5:1
Error! Not a valid embedded object. d. 1:5
Error! Not a valid embedded object. e. 4:1
The risk of Heart failure in patients with Coronary artery disease and depression is 16.4 per 100 and the
risk of HF in patients with CAD disease and without depression is 3.6 per 100. May et al. Depression
After Coronary Artery Disease Is Associated With Heart Failure J Am Coll Cardiol, 2009; 53:1440-
1447
Incorrect
Marks for this submission: 0/1.
Question 84
Marks: 0/1
The last stage according to Prochaska and Diclemente model of change of behaviour is
Choose one answer.
Error! Not a valid embedded object. a. Action
Error! Not a valid embedded object. b. Preparation
Error! Not a valid embedded object. c. Contemplation
Error! Not a valid embedded object. d. Precontemplation
Error! Not a valid embedded object. e. Maintenance
In the last stage maintenance stage the new behaviours such as abstinence are perpetuated.
Handbook of Alcoholism, 2000, Zernig et al, pg 99-100
Incorrect
Marks for this submission: 0/1.
Question 85
Marks: 0/1
A 19 year old pregnant girl asks you for the best time for undergoing opiate withdrawal during her
pregnancy
Choose one answer.
Error! Not a valid embedded object. a. All 3 trimesters
Error! Not a valid embedded object. b. III trimester
Error! Not a valid embedded object. c. I trimester
Error! Not a valid embedded object. d. II trimester
Error! Not a valid embedded object. e. After delivery
Opiate withdrawal can cause foetal distress leading to miscarrage, termination and abortions. Hence the
overall aim is to start methadone early in pregnancy and continue for the whole pregnancy.
Day E and George S (2005) Management of drug misuse in pregnancy Advances in Psychiatric
Treatment 11: 253-261
Incorrect
Marks for this submission: 0/1.
Question 86
Marks: 0/1
A patient with moderate depression has been given a trial of fluoxetine, and has shown a partial
response, with symptoms of decreased sleep still persisting. Which of the following medication would
be a useful strategy for augmentation?
Choose one answer.
Error! Not a valid embedded object. a. Mianserin
Error! Not a valid embedded object. b. Lithium
Error! Not a valid embedded object. c. Risperidone
Error! Not a valid embedded object. d. Aripiprizole
Error! Not a valid embedded object. e. Mirtazepine
The current NICE guidance (Oct 2009) advises using Lithium, Mirtazepine, Mianserin, Antipsychotics
for augmenting antidepressants in treatment of depression in adults. In this case of a person with sleep
problems, it may be useful to use Mirtazepine as an augmenting agent.
Depression in Adults. NICE 2009.
Incorrect
Marks for this submission: 0/1.
Question 87
Marks: 0/1
The following is more liekly to beseen in an anorexic patient
Choose one answer.
Error! Not a valid embedded object. a. Low GH
Error! Not a valid embedded object. b. Low T3
Error! Not a valid embedded object. c. Low cortisol
Error! Not a valid embedded object. d. Raised oestrogen
Error! Not a valid embedded object. e. High white cell count
In anorexia there is anaemia, leucopenia, thrombocytopenia, raised GH & cortisol, low oestrogen &
progesterone.
Companion to Psychiatric Studies, 7th edn, pg 492
Incorrect
Marks for this submission: 0/1.
Question 88
Marks: 0/1
For questions 88-90, please read the precis below:

Association of smoking and depression has been studied using an Australian sample
(Pasco, J. A., et al (2008) Tobacco smoking as a risk factor for major depressive disorder: population-
based study. The British Journal of Psychiatry, 193, 322-326).
A population of 1500 women in an electoral area in Australia participated in a 10 year longitudinal
study conducted primarily to find factors associated with osteoporosis since 1995. Among these, 1043
women underwent psychiatric assessment in the year 2004 (5 years ago). 835 out of this 1043 had
longitudinal psychiatric data available, based on which 164 were excluded as they had past episodes of
depression. Remaining 671 women aged 20–84 years with no history of major depressive disorder at
baseline recorded in 1995 were eligible for inclusion. From the baseline, 51 patients had developed de
novo major depressive disorder and 620 remained major depressive disorder-free during follow-up.
Participants were classified as smokers if they were current smokers at baseline (initial recruitment in
1995); otherwise they were classified as non-smokers.

What type of graph is shown above?


Choose one answer.
Error! Not a valid embedded object. a. Kaplan Meier graph
HTMLCONTROL
Forms.HTML:Option.1 |Error! Not a b. Forest plot
valid embedded object.}
HTMLCONTROL
Forms.HTML:Option.1 |Error! Not a c. Galbraith plot
valid embedded object.}
HTMLCONTROL
Forms.HTML:Option.1 |Error! Not a d. Funnel plot
valid embedded object.}
HTMLCONTROL
Forms.HTML:Option.1 |Error! Not a e. Stem leaf plot
valid embedded object.}
his is a graph of the Survival function against time using the Kaplan-Meier Curve, which plots time
on the x-axis and the proportion surviving (without the outcome) on the y-axis prior to conducting any
analysis.
Incorrect
Marks for this submission: 0/1.
Question 89
Marks: 0/1
In the above graph, values along y axis refer to
Choose one answer.
a. Probability of being
Error! Not a valid embedded object.
depressed
HTMLCONTROL Forms.HTML:Option.1 |Error! Not a valid b. Time to onset of major
embedded object.} depression
HTMLCONTROL Forms.HTML:Option.1 |Error! Not a valid c. Probability of being a
embedded object.} smoker
HTMLCONTROL Forms.HTML:Option.1 |Error! Not a valid d. Probability of not being
embedded object.} depressed
HTMLCONTROL Forms.HTML:Option.1 |Error! Not a valid e. Probability of being a non
embedded object.} smoker
his is a Kaplan Meier survival curve with the X axis showing time to depression in years and the Y
axis showing the probability of not having depression.
Incorrect
Marks for this submission: 0/1.
Question 90
Marks: 0/1
When do the two groups (smokers and non-smokers) have similar probability of developing depression
Choose one answer.
Error! Not a valid embedded object. a. 8-10 years
HTMLCONTROL Forms.HTML:Option.1 |Error! Not a valid embedded object.} b. 2-4 years
HTMLCONTROL Forms.HTML:Option.1 |Error! Not a valid embedded object.} c. 9-10 years
HTMLCONTROL Forms.HTML:Option.1 |Error! Not a valid embedded object.} d. 4-6 years
HTMLCONTROL Forms.HTML:Option.1 |Error! Not a valid embedded object.} e. 6-8 years
he two suvival curves are close to each other between 2-4 years, but then diverge indicating different
probabilities between the two groups.
Incorrect
Marks for this submission: 0/1.
Question 91
Marks: 0/1
For the questions 91-94, please read the precis below:

A study compared shared care records with standard care for patients with psychosis. Patients were
randomised to either have shared care records or standard care, and their symptoms were assessed
using the Brief Psychiatric rating scale (BPRS) at 3, 6 months and 1 yr follow-up. The results are
provided in the graph below.

Dotted - standard care; Plain - shared care


What type of graph is shown above?
Choose one answer.
Error! Not a valid embedded object. a. Forest plot
HTMLCONTROL Forms.HTML:Option.1 |Error! Not a valid embedded
b. Stem leaf plot
object.}
HTMLCONTROL Forms.HTML:Option.1 |Error! Not a valid embedded
c. Galbraith plot
object.}
HTMLCONTROL Forms.HTML:Option.1 |Error! Not a valid embedded d. Box & whisker
object.} plot
HTMLCONTROL Forms.HTML:Option.1 |Error! Not a valid embedded
e. Funnel plot
object.}
n descriptive statistics, a box plot or box-and-whisker diagram or plot is a convenient way of
graphically depicting groups of numerical data through their five-number summaries: the smallest
observation (sample minimum), lower quartile (Q1), median (Q2), upper quartile (Q3), and largest
observation (sample maximum).
Incorrect
Marks for this submission: 0/1.
Question 92
Marks: 0/1
In the figure, the box represents
Choose one answer.
Error! Not a valid embedded object. a. Range
HTMLCONTROL Forms.HTML:Option.1 |Error! Not a valid embedded
b. Standard deviation
object.}
HTMLCONTROL Forms.HTML:Option.1 |Error! Not a valid embedded c. Confidence
object.} intervals
HTMLCONTROL Forms.HTML:Option.1 |Error! Not a valid embedded
d. The quartiles
object.}
HTMLCONTROL Forms.HTML:Option.1 |Error! Not a valid embedded
e. Median
object.}
he box represents quartiles, i.e., the values between the upper quartile and the lower quartile.
Click here for reference
Incorrect
Marks for this submission: 0/1.
Question 93
Marks: 0/1
In the figure, the vertical line coming from the box represents
Choose one answer.
Error! Not a valid embedded object. a. Standard deviation
HTMLCONTROL Forms.HTML:Option.1 |Error! Not a valid embedded
b. Range
object.}
HTMLCONTROL Forms.HTML:Option.1 |Error! Not a valid embedded c. Confidence
object.} intervals
HTMLCONTROL Forms.HTML:Option.1 |Error! Not a valid embedded
d. Mode
object.}
HTMLCONTROL Forms.HTML:Option.1 |Error! Not a valid embedded
e. Median
object.}
he vertical lines coming from the box connects the lowest and highest value for that group. i.e., the
range
Click here for reference
Incorrect
Marks for this submission: 0/1.
Question 94
Marks: 0/1
From this precis, what can be deduced
Choose one answer.
a. Shared care is better than standard care
Error! Not a valid embedded object.
at 12 months
HTMLCONTROL Forms.HTML:Option.1 |Error! Not b. Shared care is better than standard care
a valid embedded object.} at 6 months
HTMLCONTROL Forms.HTML:Option.1 |Error! Not c. It is not possible to deduce fro the
a valid embedded object.} information provided
HTMLCONTROL Forms.HTML:Option.1 |Error! Not d. There is no difference between shared
a valid embedded object.} care and standard care
HTMLCONTROL Forms.HTML:Option.1 |Error! Not e. Standard care is better than shared care
a valid embedded object.} at 12 months
box plot is a method of displaying data, and part of descriptive statistics. Care should be taken when
making interpretation from the boxplot, as interpretive statistics are required for this.
Incorrect
Marks for this submission: 0/1.
Question 95
Marks: 0/1
For questions 95-100, please read the precis below:

Purandare et al. Suicide in dementia: 9-year national clinical survey in England and Wales.The British
Journal of Psychiatry.2009; 194: 175-180.

The study was carried out as part of the National Confidential Inquiry into Suicide and Homicide by
people with Mental Illness, to assess the factors associated with suicide in dementia. Information on all
deaths between 1 April 1996 until 31 December 2004 in England and Wales receiving a suicide or an
open verdict at coroner's inquest was obtained from the Office for National Statistics. Identifying
details on each individual who had died by suicide were submitted to the main hospital and community
NHS trusts who provided mental health services to people living in the deceased's district of residence.
When NHS records showed that contact had occurred in the 12 months before the suicide, the person
became an `Inquiry case'.

For each Inquiry case, the consultant psychiatrist was sent a questionnaire and asked to complete it
based on their knowledge of the patient, psychiatry case notes, and in discussion with other members of
the mental health team. The questionnaire consisted of sections covering social/demographic
characteristics, clinical history, details of suicide, aspects of care, details of final contact with mental
health services and clinicians' views on suicide prevention. Clinicians were asked for an ICD–10
primary diagnosis and also any ICD–10 secondary diagnoses. 95% of questionnaires were returned by
their psychiatrist.

Based on the information provided by the responsible consultant psychiatrist, they identified all
patients with an ICD–10 diagnosis of dementia. A total of 118 patients with dementia were identified.
For each individual with dementia, they extracted four gender-matched suicide cases of patients
without a diagnosis of dementia nearest in age to the dementia patients (n=472). This selection was
masked, i.e. the researchers were unaware of the behavioural, clinical and care characteristics of these
controls.
Over the 9-year study period, there were 45012 deaths by suicide, in which 11375 cases (that included
data on primary diagnosis), a psychiatric diagnosis was recorded. The cause of suicide is provided in
the table below.

Cause of Dementia Group Control group OR (CI)


death (n =118) (n= 472)

N (%) N (%)
Self-poisoning 30 (28) 153 (33) 0.88 (0.54 – 1.42)
Drowning 21 (19) 54 (12) 1.55 (0.90 – 2.65)
Hanging 19 (17) 142 (30) 0.45 (0.26 – 0.78)
Jumping 9 (8) 26 (6) 0.90 (0.43 – 1.86)
(height /
moving
vehicle)
Suffocation 6 (6) 35 (8) 0.75 (0.30 – 1.88)
Cutting or 5 (5) 12 (3) 1.51 (0.54 – 4.25)
stabbing
The strength of the study design used in this study is
Choose one answer.
Error! Not a valid embedded object. a. Good for rare outcomes
HTMLCONTROL Forms.HTML:Option.1 |Error! Not a b. Can evaluate multiple outcomes of a
valid embedded object.} single exposure
HTMLCONTROL Forms.HTML:Option.1 |Error! Not a
c. Good for rare exposures
valid embedded object.}
HTMLCONTROL Forms.HTML:Option.1 |Error! Not a
d. Less susceptible to recall bias
valid embedded object.}
HTMLCONTROL Forms.HTML:Option.1 |Error! Not a e. Measures the exposure and the
valid embedded object.} outcome prospectively
case control study is an observational analytic comparison of subjects with and without a particular
disorder. Cases with a disease and controls without the disease are compared on the rates of previous
exposure to a measure of interest. These studies can be relatively easy, many exposures can be
investigated simultaneously and rare diseases can be examined. However, they are prone to many types
of bias and confounding, as the information on exposure is obtained retrospectively. Critical Appraisal
for Psychiatry. Lawrie SM, McIntosh AM & Rao S. 2000, P 26-27
Incorrect
Marks for this submission: 0/1.
Question 96
Marks: 0/1
The particular strength of the methodology used in this study is
Choose one answer.
Error! Not a valid embedded object. a. The control group selected for this study
HTMLCONTROL Forms.HTML:Option.1 |Error! b. Psychiatrist being blind to the purpose of
Not a valid embedded object.} the study
HTMLCONTROL Forms.HTML:Option.1 |Error! c. The high percentage of forms returned by
Not a valid embedded object.} the Psychiatrist
HTMLCONTROL Forms.HTML:Option.1 |Error! d. The validated diagnosis of dementia used
Not a valid embedded object.} in this study
HTMLCONTROL Forms.HTML:Option.1 |Error! e. All the suicides during the specificed time
Not a valid embedded object.} period were included in this study
hough this study tried to include all dementia patients who committed suicide, it has only included
patients who were in contact with mental services, hence excluding many with dementia not in contact
with MH services. The ideal control group would have been patients with dementia who did not
commit suicide, hence the control group selected for this study were less than ideal. The diagnosis of
dementia was retrospective, and not done using validated scales. the psychiatrist was not clind to the
purpose of the study
Incorrect
Marks for this submission: 0/1.
Question 97
Marks: 0/1
Controls were more likely to be selected if
Choose one answer.
Error! Not a valid embedded object. a. Age matched healthy vlounteers
HTMLCONTROL Forms.HTML:Option.1 |Error! b. Patients with mental illness who did not
Not a valid embedded object.} complete suicide
HTMLCONTROL Forms.HTML:Option.1 |Error! c. Patients with a mental illness other than
Not a valid embedded object.} dementia who completed suicide
HTMLCONTROL Forms.HTML:Option.1 |Error! d. People with no mental illness who
Not a valid embedded object.} completed suicide
HTMLCONTROL Forms.HTML:Option.1 |Error! e. patients with dementia who did not
Not a valid embedded object.} complete suicide
he control group had to be similar to the cases except that the cases had dementia. So the control
group would be age, and gender matched patients who commited suicide, but did not have a diagnosis
of dementia (had other diagnosis though).
Incorrect
Marks for this submission: 0/1.
Question 98
Marks: 0/1
From the table above, the commonest method used by patients with dementia to commit suicide is
Choose one answer.
a. Self-poisoning
Error! Not a valid embedded object.

HTMLCONTROL Forms.HTML:Option.1 |Error! Not a valid embedded


b. Hanging
object.}
HTMLCONTROL Forms.HTML:Option.1 |Error! Not a valid embedded
c. Jumping
object.}
HTMLCONTROL Forms.HTML:Option.1 |Error! Not a valid embedded
d. Cutting
object.}
HTMLCONTROL Forms.HTML:Option.1 |Error! Not a valid embedded
e. Drowning
object.}
s per the table, self-poisoning is the commonest method of suicide in the dementia group, as well as
the control group
Incorrect
Marks for this submission: 0/1.
Question 99
Marks: 0/1
Which method is significantly less common in patients than in controls?
Choose one answer.
Error! Not a valid embedded object. a. Hanging
HTMLCONTROL Forms.HTML:Option.1 |Error! Not a valid
b. self-poisoning
embedded object.}
HTMLCONTROL Forms.HTML:Option.1 |Error! Not a valid
c. Jumping
embedded object.}
HTMLCONTROL Forms.HTML:Option.1 |Error! Not a valid d. Cannot be deduced from
embedded object.} this table
HTMLCONTROL Forms.HTML:Option.1 |Error! Not a valid
e. stabbing
embedded object.}
anging was used significantly less often by those in the dementia group compared with controls (the
odds ratio is less than 1, and the confidence intervals do not cross 1, which makes the reults significant)
Incorrect
Marks for this submission: 0/1.
Question 100
Marks: 0/1
If the population in the UK is 55 million then calculate the annual incidence of suicide over the study
period
Choose one answer.
a. 8 per thousand
Error! Not a valid embedded object.
population
HTMLCONTROL Forms.HTML:Option.1 |Error! Not a valid b. 2 per thousand
embedded object.} population
HTMLCONTROL Forms.HTML:Option.1 |Error! Not a valid c. 1 per thousand
embedded object.} population
HTMLCONTROL Forms.HTML:Option.1 |Error! Not a valid d. 3 per thousand
embedded object.} population
HTMLCONTROL Forms.HTML:Option.1 |Error! Not a valid e. 5 per thousand
embedded object.} population
he number of suicides during the nine year study period is 45012, and the population is 55 million.
The annual incidence rate of suicide is (45012/5500000)/12 =0.0009 or= 1 per thosand population.
Incorrect
Marks for this submission: 0/1.
Question 101
Marks: 0/6
Screening
Terminally ill patients in a palliative care service are screened for depression with the question ‘Do
you think you are depressed?’ This response is then compared to a structured diagnostic interview
used to diagnose depression (gold standard).

Lead in: Please see the following table and answer each question underneath using the choices below.
Each option may be used once, more than once or not at all.

Screening Question Structured diagnostic interview


Depression No depression Total
diagnosed diagnosed
Depression 11 9 20
diagnosed
No depression 14 40 54
diagnosed
Total 25 49 74

A - 11%
B - 22%
C - 33%
D - 44%
E - 55%
F - 66%
G - 74%
H - 77%
I - 78%
J - greater than 80%
K - 99%

What was the prevalence of depression in the sample?


What was the sensitivity of the screening?
What was the specificity of the screening?
What proportion of those who were screened positive was depressed?
What proportion of those who were depressed screened positive?
What was the negative predictive value?
Answers

1. C - The prevalence is the number of individuals with a disease in a population at a particular pint
(point prevalence) or over a period of time (period prevalence).

2. D - 44% (11/25). Sensitivity (True Positive): Proportion of patients testing positive out of all patients
out with the disease. It is the Probability that an individual with the disease will have a positive test.

3. J - 82% (40/49).Specificity (True Negative Rate): Proportion of patients testing negative out of all
patients without the disease. It is the Probability that an individual without the disease will have a
negative test.

4. E - 55%(11/20). Positive LR: The odds (likelihood) of having the disease if the test result is positive.

5. D - 44 %( 11/25)

6. G - 74% (40/54). Negative predictive value: Measures the Proportion of patients without the disease
out of all the patients testing negative. Probability that an individual with a negative test will not have
the disease (Given a negative test, what is the probability of not having the disease).

Incorrect
Marks for this submission: 0/6.
Question 102
Marks: 0/3
Reversible Dementia

A - Carbimazole
B - Dexamethasone
C - Methotrexate
D - Neomycin
E - Surgical evacuation
F - Surgical shunt
G - Thiamine
H - Thyroxine
I - Vincristine
J - Vitamin B12

Choose the most appropriate treatment from the list of options above, for each of the scenarios
below. Each option may be used once, more than once or not at all.

A 55 year old lady presents with a history of memory disturbance and early morning headaches which
tend to get better as the day progresses. On examination, there is facial nerve palsy. (Choose one).
A 45 year old lady presents with long standing Alcohol problems, ataxia, nystagmus and asterixis.
(Choose One).
A 65 year old lady presents with mild cognitive impairment, urinary incontinence and ataxia (Choose
one).
Answers

1. E - This seems to be the most appropriate choice as the scenario seems to point towards a classical
case of brain tumour - Early morning headache, and focal neurological sign. It is important to bear in
mind that all early morning headache is not related to tumors - Please read -Larner AJ. Not all early
morning headaches are due to brain tumors.
Practical Neurology 2009; 9:80-84.
2. G - This scenario is suggestive of a liver disease & Wernicke's encephalopathy secondary to
alcoholism. The treatment would be Thiamine.

3. F - This scenario is suggestive of Normal Presure Hydrocephalus triad of Urinary incontinence,


Ataxia and Memory difficulties. The treatment is usually a surgical shunt.

Incorrect
Marks for this submission: 0/3.
Question 103
Marks: 0/3
Rating scales in Dementia

A - ACE
B - ADAS-Cog
C - CAMDEX
D - CAPE
E - CPI
F - GDS
G - HONOS
H - MMSE

Choose ONE most appropriate scales for the following in the diagnosis of dementia. Each
option may be used once, more than once or not at all.

Used for differential diagnosis


Used to assess the progression of dementia with treatment
Used as a screening test for dementia
Answers

1. C - The AGECAT (Automated Geriatric Examination for Computer Assisted Taxonomy) and
CAMDEX (Cambridge Mental Disorders of the Elderly Examination) cover all the major diagnosis of
Old Age Psychiatry.

2. D - CAPE (Clifton Assessment Procedure for the Elderly) and REPDS (The Revised Elderly Persons
Disability Scale ) do give some estimate of severity of general impairment after diagnosis has been
carried out.

3. H - MMSE (Mini Mental State Examination), ACE (Addenbrooke's Cognitive Examination) and
information orientation section of the Clifton Assessment Procedure for the Elderly (CAPE) provide
general rating of mental impairment that can be used as screening but not diagnostic tests.

Incorrect
Marks for this submission: 0/3.
Question 104
Marks: 0/8
Research Methods

A-2
B-4
C-5
D-8
E - 10
F - 10%
G - 20%
H - 30%
I - 40%
J - 50%
K - 60%
A randomised controlled trial comparing the efficacy of Xanomeline (cholinergic M1 selective agonist)
against chlorpromazine (comparative control) was conducted last year. This trial also reported the
safety and side effect profile of the two drugs. 500 patients were in each arm receiving either
Xanomeline or chlorpromazine (CPZ). 200 patients showed clinical response in th e Xanomeline
group. 300 patients recovered in the chlorpromazine group.100 patients on Xanomeline
developed extra pyramidal side effects while 50 patients in chlorpromazine group developed
EPSEs.

Calculate the following: Each option may be used once, more than once or not at all.

EER for clinical response in Xanomeline group


CER for clinical response in chlorpromazine group
Absolute Risk Reduction using Xanomeline
Number needed to treat for Xanomeline
EER for EPSEs in Xanomeline group
CER for EPSEs in chlorpromazine group
Absolute risk reduction (or Increase) using Xanomeline
Number needed to harm
Answers

1. I - Experimental event rate (EER) =events/subjects in experimental group=200/500=0.4 or 40%

2. K - Control event rate (CER) =events/subjects in control group=300/500=0.6 or 60%

3. G - Absolute Benefit Decrease (ARR) =|CER-EER|=0.2 or 20%

4. C - Number needed to treat=1/ARR=1/0.2=5

5. G - EER for EPSE's in drug X group=100/500=0.2 or 20%

6. F - CER for EPSEs in Chlorpromazine group=50/500=0.1 0r 10%

7. F - Absolute Risk Increase (ARI) = |EER-CER|=0.2-0.1=0.1 or 10%

8. E -

Incorrect
Marks for this submission: 0/8.
Question 105
Marks: 0/4
Movement disorders in psychiatry

A - Alcohol withdrawal
B - Creutzfeldt Jakob disease
C - Essential myoclonus
D - Essential tremors
E - Fahr's syndrome
F - Frontal meningioma
G - Huntington's disease
H - Meige's syndrome
I - Neuroacanthocytosis
J - Parkinson's disease
K - Supranuclear palsy
L - Sydenham's chorea

For each of the following clinical conditions below, match the most appropriate diagnosis from the
above list. Each option may be used once, more than once or not at all.

A 65 year old lady presents with short strides, short stepping gait and progressively small and
cramped handwriting when tested. She also has a vertical gaze problem on examination.
A 15 year old boy develops non-repetitive sudden movements of his arms. He has no past history of
similar problems, and the family has no degenerative illnesses. The only medical history seems to be
an episode of pharyngitis nearly 6 months ago.
A 45 years old man has a history of progressive memory loss. His wife has observed him having
sudden muscle movements on a number of occasions as if experiencing an electric shock
momentarily.
A 37 year old woman has repeated involuntary blinking and chin-thrusting. She is cognitively intact
and has no other movement problems. She constantly uses chewing gums to conceal her facial
movements.
Answers

1. K - Progressive supranuclear palsy (PSP), also known as Steele-Richardson-Olszewski syndrome, is


a neurodegenerative disease that affects cognition, eye movements, and posture. The exact cause is
unknown. The cardinal manifestations of PSP are supranuclear ophthalmoplegia; pseudo bulbar palsy;
prominent neck dystonia; Parkinsonism; behavioral, cognitive, and gait disturbances that cause
imbalance; and frequent falls

2. L - Sydenham chorea (SD) is a neurological disorder of childhood resulting from infection via Group
A beta-hemolytic streptococcus, the bacterium that causes rheumatic fever. Rapid, irregular, and
aimless involuntary movements of the arms and legs, trunk, and facial muscles characterize Sydenham
Chorea. It affects girls more often than boys and typically occurs between 5 and 15 years of age. Some
children will have a sore throat several weeks before the symptoms begin, but the disorder can also
strike up to 6 months after the fever or infection has cleared. Symptoms can appear gradually or all at
once, and may include uncoordinated movements, muscular weakness, stumbling and falling, slurred
speech, difficulty concentrating and writing, and emotional instability. The symptoms of SD can vary
from a halting gait and slight grimacing to involuntary movements that are frequent and severe enough
to be incapacitating. The random, writhing movements of chorea are caused by an auto-immune
reaction to the bacterium that interferes with the normal function of a part of the brain (the basal
ganglia) that controls motor movements.

3. B - Creutzfeldt-Jakob disease (CJD) is the best known of a group of diseases called prion disease,
which affects a form of protein found in the central nervous system and cause dementia. Early
symptoms include minor lapses of memory, mood changes and loss of interest. Within weeks, an
infected person may have clumsiness and feeling muddled, become unsteady in walking, and exhibit
slow or slurred speech. The symptoms progress to jerky movements, shakiness, and stiffness of limbs,
incontinence and the loss of the ability to move or speak. By this stage, the person is unlikely to be
aware of their surroundings or disabilities. People affected by CJD usually die within six months of
onset often from pneumonia. In a minority of patients, the disease may take two years to run its course.

4. H - Henri Meige described in 1904 what is now commonly called oral facial dystonia. There is some
variation in what has been described since in what has been called Meige's syndrome, but in all
descriptions there is blinking and chin thrusting. Some patients have lip pursing or tongue movements
and, for a few, the movements spread into the shoulders. The cause remains obscure, treatment less
than ideal, and frustration is a major factor as it is with blepharospasm. Patients may chew gum, whistle
or touch their face in an effort to lessen the movements.

Incorrect
Marks for this submission: 0/4.
Question 106
Marks: 0/3
Personality disorders

A - Anankastic personality disorder


B - Anxious-avoidant personality disorder
C - Dependent personality disorder
D - Dissocial personality disorder
E - Emotionally unstable - borderline type
F - Histrionic personality disorder
G - Narcissistic personality disorder
H - Paranoid personality disorder
I - Schizoid personality disorder
J - Schizotypal personality disorder

Choose ONE possible personality disorder for each of the following scenarios: Each option may
be used once, more than once or not at all.

A 23 year old woman brings her boyfriend who always asks her before he does anything which is
causing some irritation to her. He always tries to please his colleagues by accepting to do things even
though he may struggle with overload of work.
A young lady with too much make up and has a seductive way of dressing. She is very emotional
when criticised.
A 40 year old stock broker was referred to the community alcohol team due to problems at work. He
is upset that he had to wait for up to half an hour before seeing the doctor and had shouted twice at the
receptionist. On seeing the doctor, he mentions that he is possibly the best in the business, which has
upset some of the partners in their firm and had denied them the bonus that they deserved. His reason
for drinking in excess is due his employer's lack of appreciation of his talents rather than any other
reason. He is upset when the Doctor refuses to see him on a daily basis, and feel that psychiatrists are
'failed doctors' and refuses to come back to the clinic.
Answers

1. C - This is suggestive of Dependent PD. The prevalence is 0.7%. They are passive, avoiding
responsibility/decision-making, subservient to others' needs. Aetiology - fixation at oral stage (Freud),
childhood deprivation.

2. F - This is suggestive of Histrionic PD. Excessive emotionality and attention seeking, dramatic,
colourful, and extroverted behaviour. Egocentric, dependent, and demanding interpersonal
relationships. Aetiology - genetic aspect to histrionic personality traits, possible different manifestation
of dissocial genotype, associated with childhood deprivation/trauma.

3. G - Narcissistic personality disorder is a condition characterized by an inflated sense of self-


importance, need for admiration, extreme self-involvement, and lack of empathy for others. Individuals
with this disorder are usually arrogantly self-assured and confident. They expect to be noticed as
superior. Vulnerability in self-esteem makes individuals with this disorder very sensitive to criticism or
defeat. Although they may not show it outwardly, criticism may haunt these individuals and may leave
them feeling humiliated, degraded, hollow, and empty. They may react with disdain, rage, or defiant
counterattack. Their social life is often impaired due to problems derived from entitlement, the need for
admiration, and the relative disregard for the sensitivities of others. Though their excessive ambition
and confidence may lead to high achievement; performance may be disrupted due to intolerance of
criticism or defeat.

Incorrect
Marks for this submission: 0/3.
Question 107
Marks: 0/4
Sampling methods

A - Cluster sampling
B - Convenience sampling
C - Multi-stage sampling
D - Progressive sampling
E - Quota sampling
F - Simple random sampling
G - Snowball sampling
H - Stratified random sampling
I - Systematic sampling

Choose one option for each question below. Each option may be used once, more than once or not at
all.

A researcher is interested in measuring dimensional factors of PTSD in illegal immigrants. He could


not find a reliable sampling frame for recruitment.
A researcher is interested in studying the lifestyle factors influencing metabolic syndrome in patients
who receive antipsychotic prescriptions. He classifies the eligible sample into inpatients, outpatients
and CMHT clients before recruiting for his study, in order to balance for the effect of one's
environment on metabolic syndrome.
A research team is interested in studying the prevalence of depression across all primary care
attendees in UK. But due to the scope of the task, they decide to randomly pick representative primary
care authorities and survey all patients in the selected regions.
A research nurse wants to study the experience of some mentally ill parents with respect to the
demands of their teenage children. She designs a qualitative study method to carry out this work.
Answers

1. G - Snowball sampling is used to reach target population where the sampling units are difficult to
identify. Each identified member of the target population is asked to identify other sampling units who
belong to the same target population. This technique can be used in the study of AIDS sufferers or a
study into drug addiction or study of illegal immigrants.
(Applied Business Statistics: Methods and Applications,By Trevor Wegnerp-215)
2. H - Stratified Random Sampling. Where the population embraces a number of distinct categories, the
frame can be organized by these categories into separate "strata." Each stratum is then sampled as an
independent sub-population, out of which individual elements can be randomly selected.

3. A - Cluster sampling. Sometimes it is cheaper to 'cluster' the sample in some way e.g. by selecting
respondents from certain areas only, or certain time-periods only. (Nearly all samples are in some sense
'clustered' in time - although this is rarely taken into account in the analysis.)
4. B - Convenience sampling. The 3 broad approaches for qualitative studies are convenience sampling,
Judgment sampling and theoretical sampling.

Incorrect
Marks for this submission: 0/4.
Question 108
Marks: 0/3
Substance misuse in pregnancy

A - Foetal Alcohol Syndrome


B - Neonatal Alcohol Withdrawal
C - Neonatal Amphetamine withdrawal
D - Neonatal Cannabis withdrawal
E - Neonatal Cocaine intoxication
F - Neonatal Nicotine withdrawals
G - Neonatal Opiate Withdrawal

Choose the most appropriate condition for the following. Each option may be used once, more than
once or not at all.

A baby born with facial abnormalities.


High pitched cry and irritability in baby following birth.
Some mild irritability and changes in tone are seen.
Answers

1. A - Children with Fetal Alcohol Syndrome have Facial anomalies, Growth retardation and abnormal
CNS development.

2. G - Neonatal withdrawal from heroin in the new born can cause high pitched crying, seizures, coma,
irritability, tremor, sneezing and diarrhoea.

3. C - Neonatal withdrawal from Amphetamines is associated with irritability, hypotonic, respiratory


distress in the new born.
(Companion to Psychiatric Studies, 7 edn, p746)
Incorrect
Marks for this submission: 0/3.
Question 109
Marks: 0/3
Differential diagnoses for eating disorders

A - Anorexia Nervosa
B - Bulimia Nervosa
C - Coeliac disease
D - Cushing's syndrome
E - Eating disorder NOS
F - Hyperthyroidism
G - Hypothyroidism

Choose the most appropriate diagnosis in the following scenarios. Each option may be used once,
more than once or not at all.
A 20 year old woman is brought to you by dominating parents. She has been loosing weight and her
mother is concerned about this. Her father feels that she is trying to push her mother away because she
refuses to put the heating on in the house, as she says she is very sensitive to it.
A 16 year old girl is brought in by her parents who are concerned that she counts the calories in
everything she eats and exercises excessively. Her BMI is 16.5.
A young girl is brought to you by her parents because they are concerned about her recent weight loss
and also that she has fatty foul-smelling stools. You notice on examination that she has mouth ulcers
and angular stomatitis.
Answers

1. F - Hyperthyroidism. Major clinical signs include weight loss (often accompanied by an increased
appetite), anxiety, intolerance to heat, hair loss, weakness, hyperactivity, irritability, apathy, depression,
polyuria, polydipsia, delirium, tremor, and sweating. Additionally, patients may present with a variety
of symptoms such as palpitations and arrhythmias (notably atrial fibrillation), shortness of breath
(dyspnea), loss of libido, nausea, vomiting, and diarrhoea. Long term untreated hyperthyroidism can
lead to osteoporosis. In the elderly, these classical symptoms may not be present.

2. A - Anorexia Nervosa. Anorexia nervosa is a psychiatric illness that describes an eating disorder
characterized by extremely low body weight and body image distortion with an obsessive fear of
gaining weight. Individuals with anorexia nervosa are known to control body weight commonly
through the means of voluntary starvation, calorie counting, excessive exercise, or other weight control
measures such as diet pills or diuretic drugs.

3. C - Coeliac's disease. The diarrhoea characteristic of coeliac disease is pale, voluminous and
malodorous. Abdominal pain and cramping, bloatedness with abdominal distension (thought to be due
to fermentative production of bowel gas), angular stomatitis and mouth ulcers may be present.

Incorrect
Marks for this submission: 0/3.
Question 110
Marks: 0/3
Addiction Psychiatry

A - Abstinence
B - Action
C - Alcohol dependence syndrome.
D - Contemplation
E - Maintenance
F - Pre-contemplation
G - Preparation
H - Relapse

Choose the most appropriate stage of motivational interviewing based on Prochaska and
DiClemente’s Stages of Change Model. Each option may be used once, more than once or not at all.

A 40 year old man with Alcohol Dependence Syndrome doesn't want to stop drinking alcohol.
A 51 year old woman with Alcoholic liver disease wants to stop drinking alcohol on her birthday in 2
weeks time.
A 57 year old man has not any alcohol for the past 15 years after having had a history of significant
alcohol problems prior to that.
Answers
1. F -

2. G -

3. H -

Stage of Change Characteristics Techniques


Pre-contemplation Not currently considering Validate lack of readiness
change: "Ignorance is bliss" Clarify: decision is theirs
Encourage re-evaluation of
current behavior
Encourage self-exploration, not
action
Explain and personalize the risk
Contemplation Ambivalent about change: Validate lack of readiness
"Sitting on the fence" Clarify: decision is theirs
Not considering change Encourage evaluation of pros and
within the next month cons of behavior change
Identify and promote new,
positive outcome expectations
Preparation Some experience with Identify and assist in problem
change and are trying to solving re: obstacles
change: "Testing the waters" Help patient identify social
Planning to act within support
1month Verify that patient has underlying
skills for behavior change
Encourage small initial steps
Action Practicing new behavior for Focus on restructuring cues and
3-6 months social support
Bolster self-efficacy for dealing
with obstacles
Combat feelings of loss and
reiterate long-term benefits
Maintenance Continued commitment to Plan for follow-up support
sustaining new behavior Reinforce internal rewards
Post-6 months to 5 years Discuss coping with relapse
Relapse Resumption of old Evaluate trigger for relapse
behaviors: "Fall from grace" Reassess motivation and barriers
Plan stronger coping strategies

Incorrect
Marks for this submission: 0/3.
Question 111
Marks: 0/3
Management in Child & Adolescent Psychiatry

A - Atomoxetine
B - Citalopram
C - Family therapy
D - Lithium
E - Mirtazepine
F - Parental skills training
G - Risperidone
H - Venlafaxine

Choose the most appropriate treatment option in the following. Each option may be used once,
more than once or not at all.
A 4 year girl who is very aggressive and disruptive in school. She has hit one of the other pupils.
An 8 year old boy presents with repeated eye blinking, grimacing, using swear words repeatedly and
also obscene gestures.
A 16 year old girl presents with low mood, anhedonhia and suicidal ideations.
Answers

1. F - Parents skills training is indicated here as positive parenting is known to have good outcomes.

2. G - Low dose antipsychotics have been shown to be effective in severe Tourettes syndrome.

3. B - SSRIs have been suggested as the first line of treatment of antidepressant medication in
adolescent depression who prefer medication.

Incorrect
Marks for this submission: 0/3.
Question 112
Marks: 0/3
Side effects

A - B blockers
B - Cimetidine
C - Digitalis
D - Isoniazid
E - L Dopa
F - Methyldopa
G - Neomycin
H - Rifampicin

Choose the most likely cause for each of the following: Each of the options can be used once,
more than once or not at all.

A patient with colour visual problems


A patient develops low mood and poor sleep
May cause mania and delirium
Answers

1. C - The entire digitalis plant is toxic (including the roots and seeds), although the leaves of the upper
stem are particularly potent, with just a nibble being enough to potentially cause death. Early symptoms
of ingestion include nausea, vomiting, diarrhea, abdominal pain, wild hallucinations, delirium, and
severe headache. Depending on the severity of the toxicosis the victim may later suffer irregular and
slow pulse, tremors, various cerebral disturbances, especially of a visual nature (unusual colour visions
with objects appearing yellowish to green and blue halos around lights), convulsions, and deadly
disturbances of the heart.
2. A - Though most of the medications above are known to cause depression, beta blockers have been
consistently associated with depression.

3. E - L Dopa can cause psychiatric symptoms including hypomania and psychosis. These are dose
limiting side effects.

Incorrect
Marks for this submission: 0/3.
Question 113
Marks: 0/4
Neuroimaging

A - SPECT scan showing loss of functional dopaminergic neuron terminals in the hypothalamus
B - SPECT scans showing hypo perfusion of the frontal lobe
C - MRI showing focal white matter lesions
D - MRI with medial temporal lobe atrophy
E - LP with raised white cell count
F - SPECT scan showing hypoperfusion of occipital area.
G - A CT scan showing enlarged ventricles

Choose the most appropriate investigative finding in the following scenarios of dementia
patients. Each of the options may be used once, more than once or not at all.

Patient with depression and intermittent leg numbness


Patient with slow onset dementia in multiple cognitive domains
Patient with change in personality and becoming disinhibited
Patient suffering from visual hallucinations made worse by neuroleptics
Answers

1. C - The features are suggestive of Multiple Sclerosis and it is now apparent that almost all patients
with clinically definite MS will show discrete white matter abnormalities.

2. D - The features are suggestive of Alzheimer's disease and medial temporal lobe atrophy may be
diagnostic of early AD and also has prognostic value.

3. B - These are features of FrontoTemporal Dementia in which functional imaging shows reduced
blood flow or metabolism selectively in the anterior regions of the brain.

4. F - This is suggestive of Lewy Body Dementia. Objective and quantitative Cerebral blood flow
measurement in the medial occipital lobe using SPECT may be useful in the clinical differentiation of
DLB and AD.

Incorrect
Marks for this submission: 0/4.
Question 114
Marks: 0/3
Antidepressants

A - Citalopram
B - Clomipramine
C - Mirtazapine
D - Moclobemide
E - Reboxetine
F - Sertraline
G - Trazadone
H - Venlafaxine

Which of the above would you use next in the following scenarios? Each option may be used once,
more than once or not at all.

You see a middle-aged woman who you have been treating for depression with duloxetine. However
she confides in you that she is having intrusive thoughts of an aggressive and sexual nature. (Choose
1)
You're treating a young woman with post-partum depression with fluoxetine. She has not improved in
the last 8 weeks. She is not currently breastfeeding. (Choose 1)
You're treating a man with citalopram. He has not responded well and his most problematic symptom
is severe insomnia. (Choose 1)
Answers

1. F - Sertraline, Paroxetine and Clomipramine can be used for Obsessions and ruminations of OCD,
though combination of an SNRI and TCA would not be advisable, hence sertraline would be the most
appropriate choice in this scenario

2. E - NICE guidelines recommend the use of another SSRI, mirtazepine, reboxetine or lofepramine. If
the patient was breastfeeding another SSRI or TCA could be the options as they have the best evidence.

3. C - Mirtazepine can be considered due to its sedative effects by acting on H1 receptors. Trazodone
can also used and has sedative actions.

Incorrect
Marks for this submission: 0/3.
Question 115
Marks: 0/5
Research and Statistics

A - Audit
B - Case control study
C - Cohort study
D - Cross-sectional study
E - Qualitative study
F - Randomised control study

A drug X is added to the hospital formulary and you would like to investigate how it will be used
against an older drug Y. Choose the most appropriate option for the below scenarios from the list
above. Each option may be used once, more than once or not at all.

You want to compare the effectiveness of Drug X versus drug Y


You want to find out if people taking drug X have more side effects than people taking drug Y
You want to find out about the experiences of patients taking drug X vs Drug Y.
Your consultant wants to find out how many patients are currently being prescribed drug X and how
many are being prescribed drug Y.
A guideline is published to suggest that Drug X should be used as first line and your consultant wants
to know whether this guideline is being followed.
Answers

1. F - RCTs are used to study the effectiveness and safety profile including its most frequently
occurring side-effects. RCTs have the best evidence in clinical trials.

2. B - Prospective Cohort studies are best suited to assess side effect profiles of medication, as RCTs
may be considered unethical for this purpose.

3. E - Patients experiences can be studied using a qualitative study.

4. D - Cross sectional study gives the no of patients currently on the medication or at the time of the
study.

5. A - Studying as to whether current practice is adherent with local/national guidelines is audit.

Incorrect
Marks for this submission: 0/5.
Question 116
Marks: 0/3
Group Therapy

A. Cohesiveness
B. Conditioning
C. Counter-dependence
D. Dependence
E. Fight-flight
F. Free floating discussion
G. Interpreting transference
H. Pairing
I. Universality
J. Vicarious learning

Lead in: Each option may be used once, more than once or not at all. From the options above, choose

Interpersonal ―Curative factors‖ in a group.

Bion‘s factors which affect group therapy

Factors seen facilitating communication in a psychodynamic group

1. A, I

Yalom identified 11 curative factors in groups including cohesiveness (i.e. a sense of belongingness)
and universality (a sense that symptoms/problems are not specific to one group member, but shared by
many others) the other factors are: Installation of hope, imparting of information, interpersonal
learning, imitation of adaptive behavior, catharsis, corrective recapitulation of the family group,
altruism, guidance, existential factors, and social skills. (see also Introduction to Psychotherapy by
Bateman (former chief examiner), Brown and Pedder. (2000) Routledge pg 135-136)

2. D, E

Bion identified several factors that hinder group work including: dependence and fight-flight response
& pairing. A group holds one of three basic assumptions, which may differ from the beliefs of the
individual members.

The basic assumption of dependence arises from the group's anxious need to depend absolutely on
someone (usually the therapist) to protect the members, satisfy all their desires and solve their
problems. This corresponds to Klein‘s depressive position.

Collective belief in an enemy who can be dealt with only by attack or retreat forms the basic
assumption of fight-flight. This corresponds to Klein‘s paranoid-schizoid position.

The basic assumption of pairing defines the notion that some future event or person will come from
outside to solve all problems. For example, there may be hope for the pairing of two individual
members of the group that could lead to the ―birth‖ of a new solution for the group‘s problems. This
corresponds to Klein and Freud‘s Oedipal fantasy.

(see also Introduction to Psychotherapy by Bateman (former chief examiner), Brown and Pedder.
(2000) Routledge pg 124)

3. F, G.

Free-floating discussions (group-analytic equivalent of free-association) and Interpreting transference


(along with interpretation of defences and resistances, archaic and primordial experiences) are
important in the context of psychodynamic groups. Foulkes also listed ‗group-specific processes‘ as
important factors.

Incorrect
Marks for this submission: 0/3.
Question 117
Marks: 0/4
Developmental syndromes

Options

A. Ataxic gait
B. Compulsive eating
C. Highly sociable
D. Large head, large ears & macro-orchidism
E. Midline repetitive movements
F. Self mutilation
G. Shy and reserved
H. Skin picking

Lead in: Choose the most appropriate features of the following syndromes. Each option may be used
once, more than once or not at all.

Angelman syndrome

William syndrome
Lesch Nyhan Syndrome

Fragile X syndrome

Answers:

1. A
In Angelman syndrome (aka happy puppet) 1; 15000 live births there is microcephaly, prominent jaw,
paroxysmal laughing in addition to tongue thrusting and ataxic gait.

2. G.
Children with Williams‘s syndrome are highly sociable but also susceptible to significant social
anxiety.

3. F
A striking feature of Lesch Nyhan Syndrome is self-mutilating behaviors, characterized by lip and
finger biting, which begin in the second year of life. Neurological symptoms include facial grimacing,
involuntary writhing, and repetitive movements of the arms and legs.

4. D, G
The common features of Fragile X syndrome are macro orchidism, elongated face, large ears and
shyness with social anxiety.

Incorrect
Marks for this submission: 0/4.
Question 118
Marks: 0/4
Qualitative Studies

Options:

A. Data saturation
B. Ethnographic methods
C. Focus groups
D. Grounded Theory
E. Investigator triangulation
F. Purposive sampling
G. Semi-structured interviews
H. Snowball sampling
I. Triangulation

Lead in: You want to do a study on attitudes to psychiatrists in different teams. Choose the most
appropriate answers. Each of the options can be used once, more than once or not at all.

You want to collect information about the opinion of people from different professional backgrounds

How will you know that you have enough data?

You decide that you want to get the opinions of social workers and nurses and analyse them separately
You decide that you need an independent person to review the analysis of your data.

Answers:

1. D, F
Grounded theory (GT) is a systematic qualitative research methodology in the social sciences
emphasizing generation of theory from data in the process of conducting research.It is a research
method that operates almost in a reverse fashion from traditional research and at first may appear to be
in contradiction of the scientific method. Rather than beginning by researching and developing a
hypothesis, the first step is data collection, through a variety of methods. From the data collected, the
key points are marked with a series of codes, which are extracted from the text. The codes are grouped
into similar concepts in order to make them more workable. From these concepts, categories are
formed, which are the basis for the creation of a theory, or a reverse engineered hypothesis. This
contradicts the traditional model of research, where the researcher chooses a theoretical framework, and
only then applies this model to the studied phenomenon.

The consecutive or random selection of participants that is common in quantitative research is replaced
by purposive sampling in qualitative research. Sampling aims to cover a range of potentially relevant
social phenomena and perspectives from an appropriate array of data sources. Selection criteria often
evolve over the course of analysis, and investigators return repeatedly to the data to explore new cases
or new angles. Purposive sampling might aim to represent any of the following: typical cases, unusual
cases, critical cases, politically important cases, or cases with connections to other cases (i.e., "snowball
sampling").Least compelling is the pursuit of merely convenient cases that are most easily accessed.
Nevertheless, many qualitative studies do rely on convenience sampling to some extent (e.g., for
pragmatic reasons, study participants may only be those who speak the same language as the
investigators or only individuals who are willing to be interviewed). Readers of qualitative studies
should look for sound reasoning describing and justifying the participant selection strategies.

2. A

Data saturation is the situation in which the data has been heard before. For example, a researcher
might interview 30 people that all had the same experience, if the researcher only interviews a few
people, their personal nuances and characteristics will colour the findings. On the other hand, there is
no need to continue interviewing people once the researcher finds that further interviews are not adding
to the findings or repeating what was already found in the previous interviews.

3. I.

Collecting information from different sources is triangulation and is used to corroborate data.

4. E

Investigator triangulation requires more than one investigator to collect and analyze the raw data, such
that the findings emerge through consensus between investigators. An investigative team best
accomplishes this.

Incorrect
Marks for this submission: 0/4.
Question 119
Marks: 0/3
Rating scales in Psychiatry
Options:

A. Abnormal inveoluntary rating scale


B. Becks Depression Inventory
C. Edinburgh post natal depression rating scale
D. General health questionnaire
E. Hamilton depression rating scale
F. Montogmery Asberg depression rating scale
G. PANSS
H. SANS
I. Simpson angus scale
J. UKU side effect rating scale

Lead in: Choose the most appropriate rating scale from above for each of the scenaios below. Each of
the options may be used once, more than once or not at all.

Two observer rated scales to monitor response to treatment in a women with a 6 week old baby
diagnosed to have depression.

One scale to screen a post partum women for depression

One scale to assess extrapyramidal side effects.

Answers:

1. E, F. HAM-D and MADRS are two observer rating scales used to monitor changes in depression

2. C. Though both BDI and EPDRS can be used to screen for depression, the Edinburgh postnatal
depression rating scale has been specifically designed to screen for depression in the post natal period.

3. I. Simpson angus scale is used to assess EPSEs.

Incorrect
Marks for this submission: 0/3.
Question 120
Marks: 0/3
Substance misuse presentations.

A. Acute alcohol intoxication


B. Alcoholic hallucinosis
C. Amphetamine intoxication
D. Benzodiazepine intoxication
E. Cannabis intoxication
F. Cocaine intoxication
G. Cocaine withdrawal
H. LSD intoxication
I. Opiate addiction
J. Opiate withdrawal

For each of the following patient presenting in the substance disorder clinic, select the most appropriate
diagnosis. Each of the options can be used once, more than once or not at all.

An 18-year-old man presents to the A&E with vivid visual hallucinations, dilated pupils, and increased
appetite.

A 24 year old man presents with dilated pupils, reduced appetite, auditory hallucinations, insomnia and
irritability.

A 34 year old man presenting with slurred speech, incoordination of movements, and nystagmus

ANSWERS

1. E – Effects of cannabis intoxication include - Euphoria, relaxation, hallucinations, hilarity, increased


appetite and glazed eyes with large pupils.
2. C. Effects of amphetamine intoxication include - Increased alertness, pulse rate and blood pressure,
and feeling of high energy. Patients with amphetamine intoxication generally present with Enlarged
pupils, insomnia, loss of appetite and weight, irritability, and depression.
3. A, D. Acute alcohol and Benzodiazepine intoxication can present with similar signs including
incordination, slurring of speech, ocular signs, confusion etc.

Incorrect
Marks for this submission: 0/3.

January 2010
1
Marks: 0/1
The following disorder results due to deletion of the chromosome of maternal origin
Choose one answer.
a. Retts syndrome
b. Prader-willi syndrome
c. Angleman syndrome
d. Tuberous sclerosis
e. Patau syndrome
Angelman syndrome is due to the micodeletion in the long arm of the maternally derived chromosome
15, characterised by ataxia, epilepsy, paroxysm of laughter, absence of speech, microcephaly, severe
LD, and behavior problems.

Oxford handbook of Psychiatry, 2005, Page 704

This question was taken from the January 2010 paper


Incorrect
Marks for this submission: 0/1.
Question 2
Marks: 0/1
An infarct in which region leads to an increased rate of depression:
Choose one answer.
a. Basal Ganglia
b. Parietal lobe
c. Cerebellum
d. Occipital lobe
e. Medulla
MRI-documented infarcts in hippocampus, basal ganglia and frontal areas have been classically
associated with post stroke depression.

Frodl et al. (2004) Hippocampal and amygdala changes in patients with major depressive disorder and
healthy controls during a 1-year follow-up. J Clin Psychiatry. 65:492-499.

This question was taken from the January 2010 paper


Incorrect
Marks for this submission: 0/1.
Question 3
Marks: 0/1
The common side effect of clonidine does not include
Choose one answer.
a. hypotension
b. constipation
c. depression
d. sedation
e. dry mouth
Uncommon CNS side effects of clonidine include insomnia,anxiety,depression,night mares and
hallucinations.
Synopsis of Psychiatry,10 edn,p-999

This question was taken from the January 2010 paper


Incorrect
Marks for this submission: 0/1.
Question 4
Marks: 0/1
The proportion of pateints with brain tumours presenting with psychiatric symptoms include
Choose one answer.
a. 40%
b. 60%
c. 10%
d. 20%
e. 30%
Two of the larger series of cerebral tumor patients report a percentage of 78 % & 52%.
Lishman's Organic Psychiatry, 4th edn, pg 283

This question was taken from the January 2010 paper


Incorrect
Marks for this submission: 0/1.
Question 5
Marks: 0/1
Tests for measuring continuous variables does not include
Choose one answer.
a. Logistic regression
b. t-tests
c. Linear regression
d. ANOVA
e. Pearsons correlation
There are two general classes of statistical tests: those based on binomial theory and those based on
normal theory. Chi-square and logistic regression deal with binomial theory or binomial distributions ie
discrete variables. T-tests, ANOVA, correlation, and regression deal with normal theory ie continuous
variables.

This question was taken from the January 2010 paper


Incorrect
Marks for this submission: 0/1.
Question 6
Marks: 0/1
Which of the following personality disorders is most commonly seen in patients over the age of 60 who
commit suicide?
Choose one answer.
a. Schizoid
b. Histrionic
c. Dependant
d. Anxious
e. Paranoid
The main results of a case control study looking into suicides in older people found that 77% of the
suicide sample had a psychiatric disorder at the time of death, most often depression (63%). Personality
disorder or personality trait accentuation was present in 44%, with anankastic or anxious traits the most
frequent. Depression, personality disorder, and personality trait accentuation emerged as predictors of
suicide in the case-control analysis. The conclusion was that personality factors, as well as depression,
are important risk factors for suicide in older people.

Harwood et al. (2001) Psychiatric disorder and personality factors associated with suicide in older
people: A descriptive and case-control study. International Journal of Geriatric Psychiatry 16(2): 155-
165

This question was taken from the January 2010 paper


Incorrect
Marks for this submission: 0/1.
Question 7
Marks: 0/1
The following is not a scale for depression
Choose one answer.
a. MADRS
b. YBOCS
c. EPDS
d. Caroll rating scale
e. HAMD
The YBOCS was designed in order to provide a rating scale which would be a measure of the severity
of symptoms of OCD.
Rating scales,Freeman & Graham,p-30

This question was taken from the January 2010 paper


Incorrect
Marks for this submission: 0/1.
Question 8
Marks: 0/1
The risk of suicide after 1 year of self harming is
Choose one answer.
a. 1 in 1000
b. 1 in 40
c. 1 in 50
d. 1 in 100
e. 1 in 10
Owen et al in 2002 in their systematic review concluded that the risk of suicide following DSH as
between 0.5-2% after 1 year and above 5% after 9 years.
Companion to Psychiatric Studies, 7th edn, pg 678

This question was taken from the January 2010 paper


Incorrect
Marks for this submission: 0/1.
Question 9
Marks: 0/1
Clozapine is indicated if there is no response to
Choose one answer.
a. 1 typical antipsychotic
b. 2 typical antipsychotics
c. 1 atypical antipsychotic
d. 3 typical antipschotics
e. 1 typical and 1 atypical antipschotic
Offer clozapine to people with schizophrenia whose illness has not responded adequately to treatment
despite the sequential use of adequate doses of at least two different antipsychotic drugs. At least one of
the drugs should be a non-clozapine second-generation antipsychotic.
Nice guidelines for Schizophrenia,March 2009

This question was taken from the January 2010 paper


Incorrect
Marks for this submission: 0/1.
Question 10
Marks: 0/1
You are treating a patient with depression by fluoxetine without response.Subsequently there is a
partial response to amitriptyline. The next next line of treatment is
Choose one answer.
a. ECT
b. SSRI's
c. Duloxetine
d. CBT
e. Venlafaxine
CBT along with antidepressants increases the effectiveness of tretament of depression. Increasing the
dose and augmentation are other options that can be considered.
NICE guidelines for depression, 2009

This question was taken from the January 2010 paper


Incorrect
Marks for this submission: 0/1.
Question 11
Marks: 0/1
The last stage according to Prochaska and Diclemente model of change of behavior is
Choose one answer.
a. preparation
b. Action
c. Maintenance
d. contemplation
e. Precontemplation
In the last stage (maintenance stage) the new behaviors such as abstinence are perpetuated.

Handbook of Alcoholism,2000, Zernig et al, P-99-100

This question was taken from the January 2010 paper


Incorrect
Marks for this submission: 0/1.
Question 12
Marks: 0/1
Following is an example of Nominal data
Choose one answer.
a. Weight in kg
b. BDI score
c. Intelligent quotient
d. Temperature in Celsius
e. Gender
These variables, also called "attribute variables" or "categorical variables," classify observations into a
small number of categories. A good rule of thumb is that an individual observation of a nominal
variable is usually a word, not a number. Examples of nominal variables include sex (the possible
values are male or female), genotype (values are AA, Aa, or aa), or ankle condition (values are normal,
sprained, torn ligament, or broken). Nominal variables are often used to divide individuals up into
classes, so that other variables may be compared among the classes. In the comparison of head width in
male vs. female isopods, the isopods are classified by sex, a nominal variable, and the measurement
variable head width is compared between the sexes.

This question was taken from the January 2010 paper


Incorrect
Marks for this submission: 0/1.
Question 13
Marks: 0/1
The mechanism of action of Modafinil includes
Choose one answer.
a. Alpha 1 adrenergic antagonism
b. dopamine reuptake inhibition
c. dopaminergic antagonism
d. dopaminergic agonism
e. Alpha 1 adrenergic agonism
Modafinil activates hypocretin producing neurons thus useful in Narcolepsy. Modafinil is reported to
act by alpha 1 agonism accounting for its alertness.
Kaplan & saddock,10 edn,p-1099

This question was taken from the January 2010 paper


Incorrect
Marks for this submission: 0/1.
Question 14
Marks: 0/1
You are using a motivational interview approach on a patient with Hypochondriasis. Which of the
following is an approach that should be used?
Choose one answer.
a. Re-attributing the symptoms
b. Colluding with the patient
c. Making the patient feel that you understand them
d. Attempt to amplify the ambivalence regarding change
e. Making the patient feel that you believe them
Motivational Interviewing is a client-centred approach to enhancing motivation for change. The main
strategy is to explore and amplify ambivalence regarding change rather than attempt to confront or
persuade the client. Amplifying resistance amplifies the person's cognitive dissonance, which thereby
motivates the person to try to minimize resistance, such as by developing a strong commitment to
engage in treatment to overcome his or her problems. A variety of well described techniques are used
to promote this process (e.g., rolling with resistance, developing discrepancy, and reinforcing self-
efficacy statements; see Miller & Rollnick, 2002). The goal is to help the patient to become his or her
own advocate for change by decreasing ambivalence and increasing
motivation.

McKay & Bouman (2008) Enhancing Cognitive-Behavioral Therapy for Monosymptomatic


Hypochondriasis With Motivational Interviewing: Three Case Illustrations. Journal of Cognitive
Psychotherapy: An International Quarterly 22(2):154-166

This question was taken from the January 2010 paper


Incorrect
Marks for this submission: 0/1.
Question 15
Marks: 0/1
The most common side effect that you would discuss with a patient when prescribing cyproterone
acetate is?
Choose one answer.
a. Insomnia
b. Depression
c. Sedation
d. Weight gain
e. Nightmares
The side effects of cyproterone acetate is gynaecomastia, thromboembolic phenomena, fatigue and
depression
Companion to psychiatric studies,7 edn,p-765

This question was taken from the January 2010 paper


Incorrect
Marks for this submission: 0/1.
Question 16
Marks: 0/1
The most common neurological side effect of Fluoxetine is
Choose one answer.
a. Seizure
b. Tremors
c. Insomnia
d. Sedation
e. Headache
Fluoxetine is the SSRI that is most likely to cause headache.The incidence is 18-20%
Kaplan & Saddock,Synopsis of Psychiatry,10 edn,p-1088

This question was taken from the January 2010 paper


Incorrect
Marks for this submission: 0/1.
Question 17
Marks: 0/1
The database that provides systematic coverage of the psychological literature from the 1800s to the
present is
Choose one answer.
a. Medline
b. Psychinfo
c. Pubmed
d. Psychlit
e. EMBASE
PsycINFO is an abstract database that provides systematic coverage of the psychological literature from
the 1800s to the present. PsycINFO contains bibliographic citations, abstracts, cited references, and
descriptive information to help you find what you need across a wide variety of scholarly publications
in the behavioral and social sciences. Medline provides information from 1965, Psychlit from 1887 and
EMbase from 1947.
J Martin Bland and Douglas G Altman. Statistics notes: Cronbach's alpha. BMJ Feb 1997; 314: 572

This question was taken from the January 2010 paper


Incorrect
Marks for this submission: 0/1.
Question 18
Marks: 0/1
The test that is used in patients who are diagnosed with mild dementia in order to assess their ability to
drive is
Choose one answer.
a. MMSE
b. Clock drawing test
c. Driving Assessment
d. ACE
e. CAMCOG
A special driving assessment (commonly referred to as Mobilty Driving Test) is used to determine the
driver's competency and ability in the areas of concentration, perception, attention, and/or judgment.
The test could be appropriate if a medical diagnosis of mild dementia is given.
Click here for reference

This question was taken from the January 2010 paper


Incorrect
Marks for this submission: 0/1.
Question 19
Marks: 0/1
A 30 year old man has been suffering from schizophrenia asks you about the percentage of people who
discontinue medication during the first 18 months of treatment
Choose one answer.
a. 25%
b. 50%
c. 20%
d. 10%
e. 75%
Medication discontinuation is a major problem in the treatment of first-episode patients; The CATIE
study found that 74 percent of patients discontinued the antipsychotic medication before 18 months
(1061 of the 1432 patients who received at least one dose): 64 percent of those assigned to olanzapine,
75 percent of those assigned to perphenazine, 82 percent of those assigned to quetiapine, 74 percent of
those assigned to risperidone, and 79 percent of those assigned to The figures are similar at one year.
Lieberman et al., Effectiveness of Antipsychotic Drugs in Patients with Chronic Schizophrenia. NEJM.
2005, 353:1209-1223
This question was taken from the January 2010 paper
Incorrect
Marks for this submission: 0/1.
Question 20
Marks: 0/1
You are seeing the parents of a 26 year old gentleman currently under your care. He has been
diagnosed with Schizophrenia and his parents want to know his prognosis. What type of schizophrenia
has the best prognosis out of the following
Choose one answer.
a. Paranoid schizophrenia
b. Simple schizophrenia
c. Catatonic schizophrenia
d. Hebephrenic schizophrenia
e. Residual schizophrenia
Factors associated with good prognosis are:paranoid type,female,married,abrupt onset,family history of
affective disorder,lack of negative and cognitive symptoms. Lieberman et al 1993 Current opinion in
Psychiatry This question was taken from the January 2010 paper
Incorrect
Marks for this submission: 0/1.
Question 21
Marks: 0/1
The following is more accurate about rates of receidivism in adolscents as compared to adults
Choose one answer.
a. adolscents had higher rate of receidivism for non sexual offences
b. adolscents had a higher rate of receidivism for sexual offences as adults
c. adolscents had a rate of receidivism for sexual offences that is similar to adults
d. adolscents had a lower rate of receidivism for sexual offences as adults
e. receidivism rates for non sexual offences were similar in adults
The juveniles adjudicated for a sexual offence had a significantly higher rate of receidivism for sexual
offences as adults than did the comparison group but rearrest rates for sexual offences were low in both
groups.The juvenile non sexual offenders had higher rates of adult receidivism for all other types of
offences.
Adolsecnt sexual offenders grown up-Recidivism in yound adulthood,Sipe et al,Criminal justice and
behaviour 1998,25,109

This question was taken from the January 2010 paper


Incorrect
Marks for this submission: 0/1.
Question 22
Marks: 0/1
Which correlation co-efficient demonstrates an inverse relationship between two variables?
Choose one answer.
a. 1
b. -0.5
c. 0.5
d. 0
e. -1
The strength or magnitude of the relationship between the two variables is expressed by a statistic
known as the CORRELATION COEFFICIENT, which varies from -1 (perfect negative relationship or
inverse correlation ), through 0 (no relationship) to +1 (perfect positive relationship).
Intuitive biostatistics, Harvey Motulsky, pg 185

This question was taken from the January 2010 paper


Incorrect
Marks for this submission: 0/1.
Question 23
Marks: 0/1
Which of the following cognitive function is less impaired in patients with schizophrenia?
Choose one answer.
a. Verbal memory
b. Immediate and delayed recall
c. Sequencing
d. Slowed attention and processing
e. Visuospatial functions
Language, motor and visuospatial functions are less consistently impaired.
Bilder, R. M., Goodman, R. S., Robinson, D., et al (2000) Neuropsychology of first-episode
schizophrenia: initial characterisation and clinical correlates. American Journal of Psychiatry, 157, 549-
559

This question was taken from the January 2010 paper


Incorrect
Marks for this submission: 0/1.
Question 24
Marks: 0/1
Which of the following is not a significant variable predictor of Sexual Offence Recidivism in
Offenders with Intellectual Disabilities
Choose one answer.
a. poor relationship with mother
b. allowances made by staff
c. sexual abuse in childhood
d. antisocial attitude
e. acceptance of crime
Significant variables were allowances made by staff, antisocial attitude, poor relationship with mother,
denial of crime, sexual abuse in childhood, erratic attendance and poor response to treatment. Certain
variables, found significant in earlier studies, did not emerge in the current analysis. These predictors
accounted for around 53% of the variance for evidence of re-offending and around 74% of the variance
for suspicion of re-offending
William R. Lindsay, Susanne F. Elliot and Arlene Astell ,Predictors of Sexual Offence Recidivism in
Offenders with Intellectual Disabilities.Journal of Applied Research in Intellectual Disabilities 2004, 17
(4); 299 - 305

This question was taken from the January 2010 paper


Incorrect
Marks for this submission: 0/1.
Question 25
Marks: 0/1
The following is commonly seen in a patient with anorexia nervosa
Choose one answer.
a. Hypotension & bradycardia
b. Hypertension & Tachycardia
c. Ventricular arrhythymias
d. Hypotension & tachycardia
e. Hypertension & bradycardia
In a community sample of 214 women with anorexia nervosa, the common medical findings included;
anemia, 38.6%; leukocytopenia, 34.4%; hyponatremia, 19.7%; hypokalemia, 19.7%; bradycardia,
41.3%; hypotension, 16.1%; hypothermia, 22.4%; elevation of alanine aminotransferase concentration,
12.2%; osteopenia, 51.7%; osteoporosis, 34.6%; and primary amenorrhea, 14.8%.

Miller et al. Common Findings in outpatients with Anorexia Nervosa. Arch Intern Med. 2005;165:561-
566.

This question was taken from the January 2010 paper


Incorrect
Marks for this submission: 0/1.
Question 26
Marks: 0/1
Autism is most likely to be associated with which of the following:
Choose one answer.
a. Williams syndrome
b. Asperger's syndrome
c. Conduct disorder
d. ADHD
e. Gilles De La Tourette
Hyperactivity is common with autism
Goodman & Scott, Child Psychiatry, 2002, pg 43

This question was taken from the January 2010 paper


Incorrect
Marks for this submission: 0/1.
Question 27
Marks: 0/1
Which of the following is an actuarial risk assessment instrument to assess risk of sexual offending
Choose one answer.
a. VRAG
b. HCR 20
c. SARA
d. PCL-R
e. SORAG
The Hare Psychopathology Checklist Revised (PCL-R), Historical Clinical risk (HCR-20), and
Violence risk appraisal guide are actuarial risk instruments for violent recidivism. Spousal Assault risk
assessment guide (SARA) is an instrument to assess risk of assault by spouse, and the Sex offender risk
appraisal guide is to assess risk of sexual recidivism.
Forensic Psychiatry - Risk Assessment Instruments

This question was taken from the January 2010 paper


Incorrect
Marks for this submission: 0/1.
Question 28
Marks: 0/1
The antidepressant of choice in the treatment of depression in a patient with cerebral palsy is
Choose one answer.
a. MAOI
b. Citalopram
c. Fluoxetine
d. TCA
e. SNRI
Fluoxetine is licenced for use in children & adolscents. There are case reports of use of fluoxetne in
treating depression in cerebral palsy.
Cerebral palsy and juvenile-onset bipolar disorder. A preliminary report, European Child & Adolescent
Psychiatry, 11:134-137 (2002) DOI 10.1007/s00787-002-0235-9 - CASE REPORT

This question was taken from the January 2010 paper


Incorrect
Marks for this submission: 0/1.
Question 29
Marks: 0/1
The ideal value for kappa is
Choose one answer.
a. 0
b. 1
c. -0.5
d. 0.5
e. -1
Kappa is a measure of inter rater reliability. If the raters are in complete agreement then Kappa = 1. If
there is no agreement among the raters (other than what would be expected by chance) then Kappa = 0.
Companion to Psychiatric Studies, 7th edn, pg 189.

This question was taken from the January 2010 paper


Incorrect
Marks for this submission: 0/1.
Question 30
Marks: 0/1
A 36 year old man with opioid dependence wishes to have opioid detoxification, he has tendency to
suffer from low blood pressure and he is not keen on using opiates. Which of the following would you
use?
Choose one answer.
a. Methadone Liquid
b. Lofexidine
c. Clonidine
d. buprenorphine
e. naltrexone
Buprenorphine or methadone are suggested as the first line in the treatment but are opiates. Lofexedine
and clonidine are alpha 2 agonists, Lofexidine is less likely to cause hypotension compared to clonidine
hence this could be the choice if the patient prefers not to use opiates.
NICE guidelines, 2007

This question was taken from the January 2010 paper


Incorrect
Marks for this submission: 0/1.
Question 31
Marks: 0/1
The preferred type of sampling used to recruit participants in a study involving drug users is
Choose one answer.
a. Systematic sampling
b. Stratified sampling
c. Snow ball sampling
d. Simple random sampling
e. Cluster sampling
In sociology and statistics research, snowball sampling is a technique for developing a research sample
where existing study subjects recruit future subjects from among their acquaintances. Thus the sample
group appears to grow like a rolling snowball. As the sample builds up, enough data is gathered to be
useful for research. This sampling technique is often used in hidden populations which are difficult for
researchers to access; example populations would be drug users or prostitutes. As sample members are
not selected from a sampling frame, snowball samples are subject to numerous biases. For example,
people who have many friends are more likely to be recruited into the sample
Goodman, L.A. (1961). "Snowball sampling". Annals of Mathematical Statistics 32: 148-170.

This question was taken from the January 2010 paper


Incorrect
Marks for this submission: 0/1.
Question 32
Marks: 0/1
The following is helpful in differentiating dissociative seizures from true seizures
Choose one answer.
a. absence of pelvic thrusting
b. duration less than 2 minutes
c. duration over 2 minutes
d. severe tongue biting
e. acute onset
Dissociative seizures are usually more than 2 minutes, gradula in onset,with occasional pelvic thrusting
which is rare in epileptic seizures,and severe tongue biting is usually very rare in dissociative seizures.
Lishman's Organic Psychiatry, 4th edn, pg 359

This question was taken from the January 2010 paper


Incorrect
Marks for this submission: 0/1.
Question 33
Marks: 0/1
Which of the following can cause both hyperglycaemia and dyslipidaemia
Choose one answer.
a. Haloperidol
b. Olanzapine.
c. Risperidone.
d. Aripriprazole
e. Amisulpiride.
Estimated rankings of the atypical agents, based on the available literature, show that the relative risk
of glucose intolerance or diabetes mellitus, hyperlipidaemia and hyperleptinaemia is highest for
clozapine and olanzapine, moderately high for quetiapine, rather low for risperidone and lowest for
ziprasidone.
Adverse Metabolic Effects Associated with Atypical Antipsychotics: Literature Review and Clinical
Implications ,Drugs, Volume 64, Number 7, 2004 , pp. 701-723(23)

This question was taken from the January 2010 paper


Incorrect
Marks for this submission: 0/1.
Question 34
Marks: 0/1
A woman lost her mother recently. A feature which would suggest depression rather than a grief
reaction are:
Choose one answer.
a. Loss of appetite
b. Global loss of self esteem
c. Auditory hallucinations
d. Severe insomnia
e. Generalized guilt
The symptoms of retardation and global loss of self esteem are seldom present in uncomplicated grief
Shorter Oxford Textbook of Psychiatry, 5th edn, pg 170

This question was taken from the January 2010 paper


Incorrect
Marks for this submission: 0/1.
Question 35
Marks: 0/1
The risk of heart failure in patients with coronary artery disease and depression compared to those with
coronary artery disease without depression is
Choose one answer.
a. 2:1
b. 4:1
c. 3:1
d. 1:5
e. 5:1
The risk of Heart failure in patients with Coronary artery disease and depression is 16.4 per 100 and the
risk of HF in patients with CAD disease and without depression is 3.6 per 100. May et al. Depression
After Coronary Artery Disease Is Associated With Heart Failure J Am Coll Cardiol, 2009; 53:1440-
1447 This question was taken from the January 2010 paper
Incorrect
Marks for this submission: 0/1.
Question 36
Marks: 0/1
Empathy Skills are most likely to be delayed in
Choose one answer.
a. A deaf child of hearing parents
b. A hearing child of deaf parents
c. A hearing child with one deaf parent
d. A deaf child of deaf parents
e. A hearing child adopted for 6 months by deaf parents
The role of Language acquisition in the development of Theory of mind can be deciphered by studying
deaf children with deaf parents and deaf children with hearing parents. Deaf parents are more sensitive
to communicative efforts of their infants, and thus help in early development of language compared to
deaf children with hearing parents. Deaf children outperform both deaf children of hearing parents and
hearing children on false-belief tasks, probably due to early exposure to visual perspective-taking (sign-
language) and develop theory of mind earlier. This shows that language has an important role to play in
acquisition of Theory of mind.

Candida C. Peterson & Michael Siegal ,Insights into Theory of Mind from Deafness and Autism, Mind
& Language
Volume 15 Issue 1, Pages 123 - 145. AND Schick B., et . Language and Theory of Mind: A Study of
Deaf Children. Child Development, March/April 2007, Volume 78, Number 2, Pages 376 - 396.

This question was taken from the January 2010 paper


Incorrect
Marks for this submission: 0/1.
Question 37
Marks: 0/1
The mechanism of Psychotropic related QT prolongation is
Choose one answer.
a. Enhancing cardiac potassium
b. Blocking cardiac sodium
c. blocking cardiac potassium
d. blocking cardiac calcium
e. Blocking non cardiac potassium
Antipsychotics appear to share the ability to antagonize the rapid component of the delayed rectifier
IKr which results in variable lengthening of the action potential, delayed repolarization and QT
prolongation. Taylor DM. Antipsychotics and QT prolongation.Acta Psychiatr Scand 2003: 107: 85-95.

This question was taken from the January 2010 paper


Incorrect
Marks for this submission: 0/1.
Question 38
Marks: 0/1
To diagnose schizotypal disorder according to ICD-10, the following criteria needs to be fulfilled.
Choose one answer.
a. Paranoid delusions
b. Duration of at least 2 years
c. Delusional perception
d. Passivity phenomenon
e. Thought broadcast
Schizotypal disorder is characterized by eccentric behaviour and anomalies of thinking and affect
which resemble those seen in schizophrenia, though no definite and characteristic schizophrenic
anomalies occur at any stage. The symptoms may include a cold or inappropriate affect; anhedonia;
odd or eccentric behaviour; a tendency to social withdrawal; paranoid or bizarre ideas not amounting to
true delusions; obsessive ruminations; thought disorder and perceptual disturbances; occasional
transient quasi-psychotic episodes with intense illusions, auditory or other hallucinations, and delusion-
like ideas, usually occurring without external provocation. There is no definite onset and evolution and
course are usually those of a personality disorder. The typical features listed above should have been
present, continuously or episodically, for at least 2 years.
ICD - 10, 1992.

This question was taken from the January 2010 paper


Incorrect
Marks for this submission: 0/1.
Question 39
Marks: 0/1
A patient who has recently completed an inpatient detoxification from opiates would like to be
prescribed something that will help to prevent relapse. Which of the following would be the most
appropriate?
Choose one answer.
a. Naloxone
b. Lofexidine
c. Methadone liquid
d. Naltrexone
e. Buprenorphine
Buprenorphine and methadone have been recommended as Ist line in the maintenance treatment of
opiate dependence. Naltrexone is used in the abstinence based treatments to prevent relapse, not as first
line. Due to shorter half life and parenteral route Naloxone is not a practical option.
NICE guidelines, 2007

This question was taken from the January 2010 paper


Incorrect
Marks for this submission: 0/1.
Question 40
Marks: 0/1
The following is a feature of Progressive supranuclear palsy:
Choose one answer.
a. Lateral gaze problems are an early feature
b. Frequent falling forwards
c. Neurofibrillary tangles in the cortex
d. Onset in the 4th decade
e. Bradyphrenia common
In supranuclear palsy the onset is usually in the 6th decade; ocular symptoms affect the vertical plane
involving downward gaze; there are frequent backwards falls and neurofibrillary tangles are seen in the
basal ganglia, brainstem and cerebellar nuclei.
Lishman's Organic Psychiatry, 4th edn, pg 776

This question was taken from the January 2010 paper


Incorrect
Marks for this submission: 0/1.
Question 41
Marks: 0/1
Concerning Naltrexone, which of the following is true;
Choose one answer.
a. There are no risks with discontinuation of Naltreoxone
b. Motivation of patient is not needed
c. Naltrexone is better than methadone in relapse prevention
d. Its not cost effective treatment for opioid dependence
e. Evidence for the effectiveness of naltrexone as a treatment for relapse prevention in Opioids
misusers has not been conclusive
Evidence of effectiveness of Naltrexone as a treatment for relapse prevention in opioid dependence has
been inconclusive. Individuals in abstinence programmes need to be highly motivated, are to be fully
informed of the potential adverse effects and benefits of treatment. Naltrexone treatment has been
found by NICE to be a cost effective treatment strategy in aiding abstinence from opiates.
Maudsley guidleines,10 edn, p-314

This question was taken from the January 2010 paper


Incorrect
Marks for this submission: 0/1.
Question 42
Marks: 0/1
Which of the following would you see abnormal language development in?
Choose one answer.
a. Conduct disorder
b. ADHD
c. Autism
d. Tourettes syndrome
e. Aspergers
Deficits in language development and difficulty using language to communicate ideas are among the
principal criteria for diagnosing autistic disorder. Language is not affected in the other choices
mentioned.

Synopsis of Psychiatry, 10th edn, pg 1194

This question was taken from the January 2010 paper


Incorrect
Marks for this submission: 0/1.
Question 43
Marks: 0/1
The following clinical symptom is more suggestive of seratonin syndrome rather than Lithium toxicity
Choose one answer.
a. Seizures
b. Diarhhoea
c. autonomic instability
d. Circulatory failure
e. Hyperreflexia
The GI symptoms of Lithium toxicity inlcudes vomitting,abdominal pain and dryness of mouth.
Kaplan & Saddock,Synopsis of Psychiatry,10 edn,p-1060 &1089

This question was taken from the January 2010 paper


Incorrect
Marks for this submission: 0/1.
Question 44
Marks: 0/1
A 23 year old woman has features of hypomania and she is also breast feeding her baby. The best
treatment option for treatment of hypomania is
Choose one answer.
a. Lithium
b. Olanzepine
c. Lorazepam
d. Lamotrigine
e. Clozapine
Antipsychotics apart from clozapine can be used in breastfeeding mothers but all the others mentioned
here should be avoided if possible.
NICE guidelines, The management of bipolar disorder in adults, children and adolescents, in primary
and secondary care. 2006

This question was taken from the January 2010 paper


Incorrect
Marks for this submission: 0/1.
Question 45
Marks: 0/1
An 18 year old male presents with diaphoresis, agitation, tachycardia and dilated pupils. What is the
most likely diagnosis?
Choose one answer.
a. Benzodiazepine intoxication
b. Amphetamine intoxication
c. Barbiturate intoxication
d. Heroin intoxication
e. Cannabis intoxication
Amphetamine causes non life threatening effects such as pupilllary dilation, excessive sweating,
agitation and tachycardia in addition to life threatening effects (MI, severe hypertension, ischaemic
collitis).

Synopsis of Psychiatry, 10th edn, pg 410

This question was taken from the January 2010 paper


Incorrect
Marks for this submission: 0/1.
Question 46
Marks: 0/1
Smoking in a pregnant lady can lead to the following in the child
Choose one answer.
a. Autistic spectrum disorder
b. Mood disorders
c. ADHD
d. Learning disability
e. Autism
Smoking in pregnancy is associated with ADHD and behaviour problems including conduct disorders
and antisocial behaviour in the child
Langley K et al. (2007) Effects of low birth weight, maternal smoking in pregnancy and social class on
the phenotypic manifestation of Attention Deficit Hyperactivity Disorder and associated antisocial
behaviour: investigation in a clinical sample. BMC Psychiatry 7:26

This question was taken from the January 2010 paper


Incorrect
Marks for this submission: 0/1.
Question 47
Marks: 0/1
The symptom that is suggestive of hepatic encephalopathy in a pateint in the ICU is
Choose one answer.
a. Asterixis
b. Confusion
c. hallucinations
d. Abnormal LFTs
e. Ataxia
Hepatic encephalopathy is suggested by Asterixis and can occur in chronic liver failure and acute
fulminant hepatic failure. Asterixis ia an abnormal tremor consisting of involuntary jerking movements,
especially in the hands also called flapping tremor.
Harrisons principles of Internal medicine ,vol 2,15 edn,p-2495.

This question was taken from the January 2010 paper


Incorrect
Marks for this submission: 0/1.
Question 48
Marks: 0/1
What does PECO means?
Choose one answer.
a. patient, expert , compulsion, option
b. patient, exposure, comparison, outcome.
c. project, experience,compel, outing
d. protect, experience, compel, option
e. prevent, expert, compulsion, options
The acronym PICO or PECO provides precision and focus to identfy problems in health care delivery.
It helps in decision making. PICO or PECO stands for P-patient or population,E-exposure or I-
intervention,C-comparison & O-outcome
Evidence-based healthcare and public health: how to make decisions ,By John Armstrong Muir Gray,
Muir Gray, Erica Ison,p-343,2008

This question was taken from the January 2010 paper


Incorrect
Marks for this submission: 0/1.
Question 49
Marks: 0/1
Whath is the standard error of mean if population is 16 and standard deviation is 8
Choose one answer.
a. 8
b. 16
c. 10
d. 4
e. 2
The standard error of the mean (SEM) is the standard deviation of the sample mean estimate of a
population mean. (It can also be viewed as the standard deviation of the error in the sample mean
relative to the true mean, since the sample mean is an unbiased estimator.) SEM is usually estimated by
the sample estimate of the population standard deviation (sample standard deviation) divided by the
square root of the sample size (assuming statistical independence of the values in the sample): where s
is the sample standard deviation (i.e., the sample based estimate of the standard deviation of the
population), and n is the size (number of observations) of the sample.

Everitt, B.S. (2003) The Cambridge Dictionary of Statistics, CUP. ISBN 0-521-81099-x

This question was taken from the January 2010 paper


Incorrect
Marks for this submission: 0/1.
Question 50
Marks: 0/1
What is the best predictor of future sexual offences?
Choose one answer.
a. Sexual deviance
b. Previous non-sexual offences
c. Mental illness
d. Lack of victim empathy
e. History of illicit drug use
A meta-analysis of 82 recidivism studies (1,620 findings from 29,450 sexual offenders) identified
deviant sexual preferences and antisocial orientation as the major predictors of sexual recidivism for
both adult and adolescent sexual offenders. Many of the variables commonly addressed in sex offender
treatment programs (e.g., psychological distress, denial of sex crime, victim empathy, stated motivation
for treatment) had little or no relationship with sexual or violent recidivism.
RK Hanson, Morton-Bourgon KE.. The characteristics of persistent sexual offenders: a meta-analysis
of recidivism studies. JOURNAL OF CONSULTING AND CLINICAL PSYCHOLOGY, 2005
Dec;73(6):1154-63.

This question was taken from the January 2010 paper


Incorrect
Marks for this submission: 0/1.
Question 51
Marks: 0/1
The following is scale is used to assess for depression in patients with dementia
Choose one answer.
a. The cornell scale
b. MADRS
c. Becks depression inventory
d. Carroll rating scale
e. HAMD
The Cornell Scale for Depression in Dementia (CSDD) was specifically developed to assess signs and
symptoms of major depression in patients with dementia. Because some of these patients may give
unreliable reports, the CSDD uses a comprehensive interviewing approach that derives information
from the patient and the informant.
Click here for reference

This question was taken from the January 2010 paper


Incorrect
Marks for this submission: 0/1.
Question 52
Marks: 0/1
The following predisposes to CNS toxicity when used with Lithium
Choose one answer.
a. Quetiapine
b. Carbamazepine
c. SSRI
d. Ziprasisidone
e. Aspirin
Dopamine receptor antagonist antipsychotic medication ,carbamazepine,clonazepam,NSAIDs
,Diuretics increase the risk of neurotoxicity with Lithium.
Kaplan & Saddock,Synopsis of Psychiatry,10 edn,p-1060
This question was taken from the January 2010 paper
Incorrect
Marks for this submission: 0/1.
Question 53
Marks: 0/1
The following is seen in somnambulism
Choose one answer.
a. Complete amnesia for the transpired event
b. full recall of transpired event
c. Attempt at arousal result in immediate return to full awareness
d. attacks are not purposeful
e. partial recall of transpired event
The attcaks in somnambulism usually lasts for less than 10 minutes and attempts at arousal result in
gradual return to full awareness, often with marked disorientation and sleep drunkenness. There is
complete amnesia for what has transpired.
Lishman's Organic Psychiatry, 4th edn, pg 837

This question was taken from the January 2010 paper


Incorrect
Marks for this submission: 0/1.
Question 54
Marks: 0/1
The following is a late manifestation of HIV dementia
Choose one answer.
a. social withdrawal
b. psychomotor retardation
c. lethargy
d. confusional state
e. Akinetic mutism
The final stages of HIV dementia are marked by severe global dementia ,akinetic mutism, incontinence,
paraplegia and myoclonus.
Lishmans Organic psychiatry,4 edn,p-410

This question was taken from the January 2010 paper


Incorrect
Marks for this submission: 0/1.
Question 55
Marks: 0/1
What treatment would you use for a patient with severe symptoms of pre-menstrual syndrome
Choose one answer.
a. Oligoacids
b. Progesterone
c. Danazol
d. Evening primrose oil
e. Fluoxetine
SSRIs have the strongest evidence for their effectiveness in the treatment of PMS.

Henshaw (2007) PMS: Diagnosis, aetiology, assessment and management: REVISITING...


PREMENSTRUAL SYNDROME. Advan. Psychiatr. Treat.13: 139 - 146.

This question was taken from the January 2010 paper


Incorrect
Marks for this submission: 0/1.
Question 56
Marks: 0/1
The common depressive symptoms in pateints with learning disability does not include
Choose one answer.
a. Weight gain
b. self injury
c. suicidal thoughts
d. hyperactivity
e. irritability
Suicidal thoughts and acts occur mainly in those with mild LD and overall they are not frequently
reported .
Companion to psychiatric studies,7 edn,p-566

This question was taken from the January 2010 paper


Incorrect
Marks for this submission: 0/1.
Question 57
Marks: 0/1
A 29 year old man has been using drugs of recreation and asks you about the one which is less likely to
have dependence
Choose one answer.
a. Heroin
b. Cocaine
c. Ecstasy
d. Amphetamine
e. Cannabis
For much of 1970s cannabis was not regarded as a drug of dependence but studies have shown features
of dependence particularly in heavy users. However compared to other illicit drugs cannabis has least
physical dependence.
New Oxford Textbook of Psychiatry, vol 1,p-551

This question was taken from the January 2010 paper


Incorrect
Marks for this submission: 0/1.
Question 58
Marks: 0/1
The hierarchy of evidence for treatment studies is
Choose one answer.
a. Cohort study, Case control study, case series
b. Expert opinion, case series, case control stuidies
c. RCT, Non RCT, metanalysis
d. Metanalysis, RCT, Cohort studies
e. cohort study, expert opinion, case series.
Level I is Evidence obtained from a single randomised controlled trial or a metaanalysis of randomised
controlled trials -grade A. At least one randomised controlled trial as part of a body of literature of
overall good quality and consistency addressing the specific recommendation (evidence level I)
without extrapolation
Hierarchy of evidence and grading of recommendations. Thorax 2004 59: i13-i14

This question was taken from the January 2010 paper


Incorrect
Marks for this submission: 0/1.
Question 59
Marks: 0/1
A researcher wants to evaluate the use Magnetic resonance imaging as a screening test to identify cases
of Alzheimer's dementia. They would confirm the correct diagnosis of Alzheimer's dementia by post
mortem examination of the brain. Which of the following study designs would be the most appropriate
for such a study.
Choose one answer.
a. Pre post study
b. Cross sectional study
c. Case control study
d. Cohort study
e. Delphi study
Cross section studies are the most appropriate study design when trying to answer questions concerning
the value of screening or diagnostic tests and prevalence of a disease (i.e., the number of subjects with a
disease at a given point of time).
Stephen Palmer and James Raftery. Economics notes: Opportunity cost. BMJ 1999; 318: 1551 - 1552

This question was taken from the January 2010 paper


Incorrect
Marks for this submission: 0/1.
Question 60
Marks: 0/1
Containment is best described as
Choose one answer.
a. Therapist's ability to keep the emotions elicited by the patient to himself
b. Patient's ability to deal with anger
c. Patient's ability to deal with unpleasant emotions
d. Ability of the therapist to modify and return the patient's difficult material in a way that he
(she) can tolerate
e. Therapist's ability to keep the actions elicited by the patient to himself
Containment refers to a thinking function that the projected element encounters (usually the group
members' projection). The therapist then uses the transference and the resulting counter-transference to
understand more about the group's needs and defences. (thus, in the therapeutic context, it refers to the
therapist-patient dynamics).
Text book of Psychotherapy in psychiatric practice,churchill livingstone,p-321

This question was taken from the January 2010 paper


Incorrect
Marks for this submission: 0/1.
Question 61
Marks: 0/1
The percentage of women suffering domestic abuse is
Choose one answer.
a. 25%
b. 40%
c. 5%
d. 10%
e. 30%
In a questionnaire survey of 1035 women attending east London general practices the prevalence of
physical abuse in the past year from a partner or expartner ranges was 17% and lifetime prevalence
40%.
Richardson J, Coid J, Petruckevitch A, Chung WS, Moorey S, Feder G:
Identifying domestic violence: cross sectional study in primary care. BMJ 2002, 324:274-278.

This question was taken from the January 2010 paper


Incorrect
Marks for this submission: 0/1.
Question 62
Marks: 0/1
The least common feature of Cushings syndrome is
Choose one answer.
a. Euphoria
b. Hallucinations
c. Delusions
d. Depression
e. Cognitive impairment
In cushings syndrome 35% have depression,16% have cognitive impairment,9% have psychotic
symptoms and 4 % have euphoria Companion to psychiatric studies,6 edn,p-322 This question was
taken from the January 2010 paper
Incorrect
Marks for this submission: 0/1.
Question 63
Marks: 0/1
The most common side effect of Rivastigmine is
Choose one answer.
a. seizure
b. diarrhoea
c. Nausea
d. Sedation
e. Agitation
The most common adverse effects associated with Rivastigmine are nausea, vomitting, dizziness,
headache, diarrhoea, abdominal pain, anorexia , fatigue, and somnolence
Synopsis of Psychiatry, 10th edn, pg 1034

This question was taken from the January 2010 paper


Incorrect
Marks for this submission: 0/1.
Question 64
Marks: 0/1
People with emotionally unstable borderline personality disorder are in
Choose one answer.
a. Paranoid-schizoid position
b. Oedipal stage
c. False self position
d. Schizoid position
e. Klein's depressive position
Kernberg suggested that patients with BPD internalize early pathologic object relations. The use of
primitive defense mechanisms (which individuals without BPD outgrow during normal development)
maintains these early pathologic object relations. Kernberg hypothesized that, in the early stages of
development, the infant experiences the maternal figure in 2 contradictory ways: The first is the good
mother, who provides for, loves, and remains close. The second is the hateful, depriving mother, who
unpredictably punishes and abandons the child. The result is intense anxiety, which leads to the
borderline defense of splitting.
Borderline Personality Disorder, Lubit & Finley-Belgrad, Borderline Personality Disorder: eMedicine
Psychiatry

This question was taken from the January 2010 paper


Incorrect
Marks for this submission: 0/1.
Question 65
Marks: 0/1
The antidepressant of choice used in the treatment of moderate depression in a patient who has
Myocardial infarction is
Choose one answer.
a. Citalopram
b. Paroxetine
c. Imipramine
d. Fluoxetine
e. Sertraline
When initiating antidepressant treatment in patients with recent MI or unstable angina, Sertraline is the
treatment of choice and has the best evidence Nice guidelines for depression,2004 (Though this is not
mentioned in NICE guidance 2009), Maudsley Prescribing guidelines, 2009.

This question was taken from the January 2010 paper


Incorrect
Marks for this submission: 0/1.
Question 66
Marks: 0/1
Which of the following psychotherapies utilises reciprocal role reversal
Choose one answer.
a. Dialectical behavioral therapy
b. Schema focused therapy
c. Alums group therapy
d. Strategic family therapy
e. Moreno's pscyhodrama
Two forms used in Psychodrama:1.Reciprocal (proper,classical,in situ) role reversal, based on social
psychology, is used mainly as an aid for dealing with people in the outer world, as a way of correcting
biased perceptions of other people and receiving feedback of oneself and as an interpersonal conflict
resolution technique.2. Representational(incomplete) role reversal, based on object relations theory, is
used more as an aid for the externalisation and interpolation of the inner world of one protagonist

Moreno, J.L. and Moreno, Z.T. (1959) Psychodrama, vol. 2, New York: Beacon House.

This question was taken from the January 2010 paper


Incorrect
Marks for this submission: 0/1.
Question 67
Marks: 0/1
The sign or symptom least likely to be experienced by a person taking MDMA is
Choose one answer.
a. Increased energy
b. Paranoia
c. Anxiety
d. Increased empathy
e. Hallucinations
Ectasy (MDMA) is a stimulant drug, which can lead to increase in energy levels, enhanced feelings of
calm, love, confidence and alertness. It can lead to anxiety, depression, paranioa & panic attacks. It is
not a hallucinagen and is not known to cause Hallucinations.

This question was taken from the January 2010 paper


Incorrect
Marks for this submission: 0/1.
Question 68
Marks: 0/1
A patient sustained fracture of facial bones from RTA is very anxious in A&E. After how long does his
anxiety start to diminish to make diagnosis of acute stress reaction.
Choose one answer.
a. 1month
b. 1 hour
c. 1 day
d. 1 year
e. 1 week
An acute stress reaction defined in ICD 10 usually occurs within minutes of the triggering event and
resolves rapidly,for example within few hours if the stressor is removed and within 1-3 days even if it
is not.
Companion to psychitric studies, 7 edn, p-476

This question was taken from the January 2010 paper


Incorrect
Marks for this submission: 0/1.
Question 69
Marks: 0/1
An elderly gentleman has been diagnosed with delusional disorder. The following is true about
delusional disorder
Choose one answer.
a. A persecutory subtype is the commonest one
b. Persecutory delusions are seen in 25% of those with diagnosis
c. Grandiose delusions are the most common type
d. The most common type is somatic delusions
e. Older age at onset is associated with somatic delusions
Persecutory delusions are the most common type in all populations, (up to 60%) followed by somatic in
young ages and then grandiose delusions.

Yamada et al (1998) Age at onset of delusional disorder is dependent on the delusional theme. Acta
Psychiatrica Scandinavica. 97(2):122 - 124

This question was taken from the January 2010 paper


Incorrect
Marks for this submission: 0/1.
Question 70
Marks: 0/1
In a positive skew graph, which of the following is true?
Choose one answer.
a. Mean of 1
b. Mean is equal to median & mode
c. Median greater than the mean
d. Mean greater than the median
e. Mean greater than the mode
The mean is higher than the median in positively (right) skewed distributions and lower than the
median in negatively (left) skewed distributions.
Companion to Psychiatric Studies, 7th edn, pg 186

This question was taken from the January 2010 paper


Incorrect
Marks for this submission: 0/1.
Question 71
Marks: 0/1
The most commonly used antidepressant in a patient with post-stroke depression
Choose one answer.
a. Paroxetine
b. Citalopram
c. Fluoxetine
d. Venlafaxine
e. Mirtazapine
The Cochrane review has identified that antidepressants particularly SSRIs are more effective than
psychotherapy in the treatment of depression after a stroke. Paolucci (2008) has provided a detailed
review of the intervention studies in the treatment of post stroke depression. There are more studies
which show that fluoxetine at doses of 20mg per day is more beneficial in improving mood and motor
performance. There are two studies which show that Citalopram is beneficial, with less side effects.
Studies show improvement with TCAs including nortriptyline, desipramine etc. but the drop out rates
are very high due to side effects.
Paolucci P. Epidemiology and treatment of post-stroke depression. Neuropsychiatr Dis Treat. 2008;
4(1): 145-154. Hackett ML, Anderson CS, House A, Xia J. Interventions for treating depression after
stroke. Cochrane Database Syst Rev. 2008;(4):CD003437. Review.

This question was taken from the January 2010 paper


Incorrect
Marks for this submission: 0/1.
Question 72
Marks: 0/1
Which of the following is the characteristic feature in transient global amnesia?
Choose one answer.
a. Abnormal blood test
b. Retrograde memory loss
c. Retained ability to retain new Information
d. Impaired recall
e. Transient loss of short term memory
Transient global amnesia manifests with a paroxysmal, transient loss of memory function. Immediate
recall ability is preserved, as is remote memory; however, patients experience striking loss of memory
for recent events and an impaired ability to retain new information. In some cases, the degree of
retrograde memory loss is mild.

Lishman's Organic Psychiatry, 4th edn, pg 33

This question was taken from the January 2010 paper


Incorrect
Marks for this submission: 0/1.
Question 73
Marks: 0/1
According to Kernberg the psychodynamic understanding of borderline personality disorder did not
include
Choose one answer.
a. Characteristic object relations
b. Partially intact reality testing
c. Identity diffusion
d. Primitive defences
e. Fully intact reality testing
The intrapsychic features pointed by Kernberg included partially intact reality testing, that is vulnerable
to alterations and failures because of aggression
Oxford Textbook of Psychotherapy, 2007, pg 292

This question was taken from the January 2010 paper


Incorrect
Marks for this submission: 0/1.
Question 74
Marks: 0/1
Please read the following precis for questions 74-76:

The following chart shows the distribution of weight of patients on atypical antipsychotic medication.

What type of graph is shown above


Choose one answer.
a. Galbraith plot
b. Stem leaf plot
c. Box & whisker plot
d. Forest plot
e. Funnel plot
In descriptive statistics, a box plot or box-and-whisker diagram or plot is a convenient way of
graphically depicting groups of numerical data through their five-number summaries: the smallest
observation (sample minimum), lower quartile (Q1), median (Q2), upper quartile (Q3), and largest
observation (sample maximum). This question was taken from the January 2010 paper
Incorrect
Marks for this submission: 0/1.
Question 75
Marks: 0/1
In the figure, the box represents
Choose one answer.
a. Confidence intervals
b. Range
c. Median
d. Standard deviation
e. Interquartile Range
The box represents inter quartile, i.e., the values between the upper quartile and the lower quartile.
Click here for reference

This question was taken from the January 2010 paper


Incorrect
Marks for this submission: 0/1.
Question 76
Marks: 0/1
In the figure, the horizontal line within the box represents
Choose one answer.
a. Confidence intervals
b. Range
c. Median
d. Mode
e. Standard deviation
The central line within the box represents the middle value of the sample i.e., Median
Click here for reference

This question was taken from the January 2010 paper


Incorrect
Marks for this submission: 0/1.
Question 77
Marks: 0/1
Please read the following precis for questions 77-81.

Charman T et al., Efficacy of three screening instruments in the identification of autistic-


spectrum disorders. The British Journal of Psychiatry (2007) 191: 554-559.

The authors‘ aim was to compare the Social Communication Questionnaire (SCQ), the Social
Responsiveness Scale (SRS) and the Children's Communication Checklist (CCC) in screening to
estimate screen characteristics for a realistic target population.

As part of the larger SNAP study of the prevalence of autistic-spectrum disorders within a total
population cohort of 56 946 children born between 1 July 1990 and 31 December 1991 all those with a
current clinical diagnosis of pervasive developmental disorder (n=255) or considered at risk of having
the undetected disorder by virtue of having a statement of special educational needs (n=1515) were
screened using the SCQ. A total of 1066 SCQs were returned completed (return rate 60.2%); 31
families declined further participation, leaving 1035 (return rate 58.5%) who returned the SCQ and
opted in for further assessments.

A stratified subsample (by coincidence, also n=255) received a comprehensive diagnostic assessment,
including standardised clinical observation (the Autism Diagnostic Observation Schedule - Generic
(ADOS-G), parent-interview assessments of autistic symptoms (ADI-R), language and IQ tests,
evaluation of psychiatric comorbidities and a medical examination. The team used ICD-10 research
diagnostic criteria to derive a clinical consensus diagnosis of childhood autism, other autistic-spectrum
disorders or no autistic-spectrum disorder.

The authors collated the following data for their subsample of 119 children: IQ; severity of symptoms
of autism, measured by ADI-R and ADOS-G algorithm total scores; a total count of ICD-10 symptoms
(0-12), systematically completed as part of the diagnostic review process of every case; parent and
teacher reports of emotional and behavioural problems; and adaptive behaviour, assessed using the
Vineland Adaptive Behavior Scales (VABS). Parents of a subsample of children (n=119) completed
both the SRS and the CCC in addition to the SCQ.

This study found that SCQ had a sensitivity of 0.86 and specificity of 0.78. The SRS had sensitivity of
0.78 and specificity of 0.67. The CCC had a sensitivity of 0.93 and specificity of 0.46.

The study design used in this study is


Choose one answer.
a. Case control study
b. Cohort study
c. Randomised control study
d. Cross sectional study
e. Meta analysis
In this study, a selected sample is subjected to undergo many diagnostic (ADI- R, ADOS-G & RDC of
ICD-10) and screening tests (SCQ, CCC & SRS) at the same time to establish the effectiveness of
screening instruments in screening for Autism. This is a cross sectional study. This question was taken
from the January 2010 paper
Incorrect
Marks for this submission: 0/1.
Question 78
Marks: 0/1
The screening test with the best sensitivity is
Choose one answer.
a. SCQ
b. ADI-R
c. SRS
d. ADOS-G
e. CCC
This study found that SCQ had a sensitivity of 0.86 and specificity of 0.78. The SRS had sensitivity of
0.78 and specificity of 0.67. The CCC had a sensitivity of 0.93 and specificity of 0.46, which indicates
that CCC had a very high sensitivity of 93%, though it had a low specificity of 46%.

This question was taken from the January 2010 paper


Incorrect
Marks for this submission: 0/1.
Question 79
Marks: 0/1
The screening test with the best specificity is
Choose one answer.
a. SCQ
b. ADOS-G
c. ADI-R
d. CCC
e. SRS
This study found that SCQ had a sensitivity of 0.86 and specificity of 0.78. The SRS had sensitivity of
0.78 and specificity of 0.67. The CCC had a sensitivity of 0.93 and specificity of 0.46, which indicates
that SCQ had a very high specificity of 78% among the three.

This question was taken from the January 2010 paper


Incorrect
Marks for this submission: 0/1.
Question 80
Marks: 0/1
Based on this data, which test would you recommend to screen for Autism spectrum disorders
Choose one answer.
a. ADI-R
b. SRS
c. ADOS-G
d. SCQ
e. CCC
This study found that SCQ had a sensitivity of 0.86 and specificity of 0.78. The SRS had sensitivity of
0.78 and specificity of 0.67. The CCC had a sensitivity of 0.93 and specificity of 0.46. SCQ seems to
have the right balance between finding the true positive (sensitivity) and true negatives (specificity)
among the three screening tests. Hence it would be the best to be recommended.

This question was taken from the January 2010 paper


Incorrect
Marks for this submission: 0/1.
Question 81
Marks: 0/1
The gold standard test used in this study is
Choose one answer.
a. SCQ
b. ICD-10
c. ADI-R
d. CCC
e. ADOS-G
The authors have used ADI-R & ADOS-G as diagnostic instruments, and then have used this data to
diagnose Autism as per the ICD-10 Research diagnostic Criteria. Hence ICD-10 is the gold standard
used in this study.
This question was taken from the January 2010 paper
Incorrect
Marks for this submission: 0/1.
Question 82
Marks: 0/4
Movement disorders in psychiatry

A - Alcohol withdrawal
B - Creutzfeldt Jakob disease
C - Essential myoclonus
D - Essential tremors
E - Fahr's syndrome
F - Frontal meningioma
G - Huntington's disease
H - Meige's syndrome
I - Neuroacanthocytosis
J - Parkinson's disease
K - Supranuclear palsy
L - Sydenham's chorea

For each of the following clinical conditions below, match the most appropriate diagnosis from the
above list. Each option may be used once, more than once or not at all.
A 65 year old lady presents with short strides, short stepping gait and and progressively small
cramped handwriting when tested. She also has a vertical gaze problem on examination.
A 15 year old boy develops non-repetitive sudden movements of his arms. He has no past history of
similar problems, and the family has no degenerative illnesses. The only medical history seems to be
an episode of pharyngitis nearly 6 months ago.
A 45 years old man has a history of progressive memory loss. His wife has observed him having
sudden muscle movements on a number of occasions as if experiencing an electric shock
momentarily.
A 37 year old woman has repeated involuntary blinking and chin-thrusting. She is cognitively intact
and has no other movement problems. She constantly uses chewing gums to conceal her facial
movements.
Answers

1. K - Progressive supranuclear palsy (PSP), also known as Steele-Richardson-Olszewski syndrome, is


a neurodegenerative disease that affects cognition, eye movements, and posture. The exact cause is
unknown. The cardinal manifestations of PSP are supranuclear ophthalmoplegia, pseudo bulbar palsy,
prominent neck dystonia, Parkinsonism; behavioral, cognitive, and gait disturbances that cause
imbalance, and frequent falls.

2. L - Sydenham chorea (SD) is a neurological disorder of childhood resulting from infection via Group
A beta-hemolytic streptococcus, the bacterium that causes rheumatic fever. Rapid, irregular, and
aimless involuntary movements of the arms and legs, trunk, and facial muscles characterize Sydenham
Chorea. It affects girls more often than boys and typically occurs between 5 and 15 years of age. Some
children will have a sore throat several weeks before the symptoms begin, but the disorder can also
strike up to 6 months after the fever or infection has cleared. Symptoms can appear gradually or all at
once, and may include uncoordinated movements, muscular weakness, stumbling and falling, slurred
speech, difficulty concentrating and writing, and emotional instability. The symptoms of SD can vary
from a halting gait and slight grimacing to involuntary movements that are frequent and severe enough
to be incapacitating. The random, writhing movements of chorea are caused by an auto-immune
reaction to the bacterium that interferes with the normal function of a part of the brain (the basal
ganglia) that controls motor movements.

3. B - Creutzfeldt-Jakob disease (CJD) is the best known of a group of diseases called prion disease,
which affects a form of protein found in the central nervous system and cause dementia. Early
symptoms include minor lapses of memory, mood changes and loss of interest. Within weeks, an
infected person may have clumsiness and feeling muddled, become unsteady in walking, and exhibit
slow or slurred speech. The symptoms progress to jerky movements, shakiness, and stiffness of limbs,
incontinence and the loss of the ability to move or speak. By this stage, the person is unlikely to be
aware of their surroundings or disabilities. People affected by CJD usually die within six months of
onset often from pneumonia. In a minority of patients, the disease may take about two years to run its
course.

4. H - Henri Meige described in 1904 what is now commonly called oral facial dystonia. There is some
variation in what has been described since in what has been called Meige's syndrome, but in all
descriptions there is blinking and chin thrusting. Some patients have lip pursing or tongue movements
and, for a few, the movements spread into the shoulders. The cause remains obscure, treatment less
than ideal, and frustration is a major factor as it is with blepharospasm. Patients may chew gum, whistle
or touch their face in an effort to lessen the movements.

Incorrect
Marks for this submission: 0/4.
Question 83
Marks: 0/6
Screening

A - 11%
B - 22%
C - 33%
D - 44%
E - 55%
F - 66%
G - 74%
H - 77%
I - 78%
J - greater than 80%
K - 99%

Terminally ill patients in a palliative care service are screened for depression with the question Do you
think you are depressed? This response is then compared to a structured diagnostic interview used to
diagnose depression (gold standard). Please see the following table and answer each question
underneath using the choices below. Each option may be used once, more than once or not at all.

Structured diagnostic interview

Depression diagnosed No depression Total


diagnosed
Depression diagnosed 11 9 20
Screening
Question No depression 14 40 54
diagnosed
Total 25 49 74

What was the prevalence of depression in the sample?


What was the sensitivity of the screening?
What was the specificity of the screening?
What proportion of those who were screened positive was depressed?
What proportion of those who were depressed screened positive?
What was the negative predictive value?
Answers

1. C - The prevalence is the number of individuals with a disease in a population at a particular point
(point prevalence) or over a period of time (period prevalence). (a+c/Grand Total; 25/74=33%).

2. D - 44% (a/a+c; 11/25). Sensitivity (True Positive): Proportion of patients testing positive out of all
patients out with the disease. It is the probability that an individual with the disease will have a positive
test.

3. J - 82% (d/b+d; 40/49).Specificity (True Negative Rate): Proportion of patients testing negative out
of all patients without the disease. It is the Probability that an individual without the disease will have a
negative test.

4. E - 55%(11/20). Positive LR: The odds (likelihood) of having the disease if the test result is positive.

5. D - 44 %( 11/25).

6. G - 74% (d/c+d; 40/54). Negative predictive value: Measures the Proportion of patients without the
disease out of all the patients testing negative. Probability that an individual with a negative test will
not have the disease (Given a negative test, what is the probability of not having the disease).

Incorrect
Marks for this submission: 0/6.
Question 84
Marks: 0/5
Epidemiology

A - 1:1
B - 1:3
C - 1:6
D - 1:8
E - 1:10
F - 3:1
G - 4:1
H - 5:1
I - 6:1
J - 7:1
K - 8:1

Select the Male: Female ratios in the following conditions:


Attention deficit and hyperactivity disorder
Autism
Bulimia
Aspergers syndrome
Prepubertal depression
Answers

1. F - 3:1. ADHD is more common in males at ratio of 3:1 in community samples and 9:1 in clinic
samples.

2. G - 4:1. A large review of all prevalence studies has found that the M:F ratios for Autism is around
4:1.

3. E - 1:10. Bulimia is more common in females with occurrence in males that of 1/10 th in females.

4. K - 8:1. With regard to gender differences, Aspergers appears to be much more common in boys. Dr.
Asperger's first patients were all boys, but girls have been diagnosed with Aspergers since the 1980s.
One Swedish study found the male/female ratio to be 4:1; however, the World Health Organization's
ICD-10 classification gives the male to female ratio as 8 to 1.

5. A - 1:1. Depression in school age children is about the same in boys as in girls with some surveys
indicating a slightly increased rate among boys. The rate of depression in adolescent females is double
the rate in adolescent males.

Incorrect
Marks for this submission: 0/5.
Question 85
Marks: 0/4
Neuroimaging

A - A CT scan showing enlarged ventricles.


B - LP with raised white cell count.
C - MRI showing focal white matter lesions.
D - MRI with medial temporal lobe atrophy.
E - SPECT scan showing hypo perfusion of occipital area.
F - SPECT scan showing hypo perfusion of the frontal lobe.
G - SPECT scan showing loss of functional dopaminergic neuron terminals in the hypothalamus.

Choose the most appropriate investigative finding in the following scenarios of dementia patients. Each
of the options may be used once, more than once or not at all.
Patient with depression and intermittent leg numbness.
Patient with slow onset dementia in multiple cognitive domains.
Patient with change in personality and becoming disinhibited.
Patient suffering from visual hallucinations made worse by neuroleptics.
Answers

1. C - The features are suggestive of Multiple Sclerosis and it is now apparent that almost all patients
with clinically definite MS will show discrete white matter abnormalities.

2. D - The features are suggestive of Alzheimer's disease and medial temporal lobe atrophy may be
diagnostic of early AD and also has prognostic value.
3. F - These are features of FrontoTemporal Dementia in which functional imaging shows reduced
blood flow or metabolism selectively in the anterior regions of the brain.

4. E - This is suggestive of Lewy Body Dementia. Objective and quantitative Cerebral blood flow
measurement in the medial occipital lobe using SPECT may be useful in the clinical differentiation of
DLB and AD.

Incorrect
Marks for this submission: 0/4.
Question 86
Marks: 0/3
Differential diagnoses for eating disorders

A - Anorexia Nervosa
B - Bulimia Nervosa
C - Coeliac disease
D - Cushing's syndrome
E - Eating disorder NOS
F - Hyperthyroidism
G - Hypothyroidism

Choose the most appropriate diagnosis in the following scenarios. Each option may be used once, more
than once or not at all.
A 20 year old woman is brought to you by dominating parents. She has been loosing weight and her
mother is concerned about this. Her father feels that she is trying to push her mother away because she
refuses to put the heating on in the house, as she says she is very sensitive to it.
A 16 year old girl is brought in by her parents who are concerned that she counts the calories in
everything she eats and exercises excessively. Her BMI is 16.5.
A young girl is brought to you by her parents because they are concerned about her recent weight loss
and also that she has fatty foul-smelling stools. You notice on examination that she has mouth ulcers
and angular stomatitis.
Answers

1. F - Hyperthyroidism. Major clinical signs include weight loss (often accompanied by an increased
appetite), anxiety, intolerance to heat, hair loss, weakness, hyperactivity, irritability, apathy, depression,
polyuria, polydipsia, delirium, tremor, and sweating. Additionally, patients may present with a variety
of symptoms such as palpitations and arrhythmias (notably atrial fibrillation), shortness of breath
(dyspnea), loss of libido, nausea, vomiting, and diarrhoea. Long term untreated hyperthyroidism can
lead to osteoporosis. In the elderly, these classical symptoms may not be present.

2. A - Anorexia Nervosa. Anorexia nervosa is a psychiatric illness that describes an eating disorder
characterized by extremely low body weight and body image distortion with an obsessive fear of
gaining weight. Individuals with anorexia nervosa are known to control body weight commonly
through the means of voluntary starvation, calorie counting, excessive exercise, or other weight control
measures such as diet pills or diuretic drugs.

3. C - Coeliac's disease. The diarrhoea characteristic of coeliac disease is pale, voluminous and
malodorous. Abdominal pain and cramping, bloatedness with abdominal distension (thought to be due
to fermentative production of bowel gas), angular stomatitis and mouth ulcers may be present.

Incorrect
Marks for this submission: 0/3.
Question 87
Marks: 0/3
Diagnosis in Child & Adolescent Psychiatry

A - ADHD
B - ASD
C - Conduct disorder
D - Depression
E - Learning difficulties
F - OCD
G - ODD
H - Tourettes syndrome

Choose the most appropriate diagnosis in the following scenarios. Each option may be used once, more
than once or not at all.
A 11 yr child very disruptive doesnt let you speak during assessment and also finds it difficult to sit in
one place.
A 9 yr old boy attends your clinic but doesnt touch anything in your office, likes food cooked in
microwave only because thats how germs are killed and finds it difficult to complete assignments at
school.
A 8 yr old boy brought to your clinic is reported to have tantrums in response to limits on his
behaviour and also continues to engage in unacceptable behaviour as though he is trying provoke his
mother.
Answers

1. A - ADHD. The principal signs are inattention, impulsivity and hyperactivity at least in two different
settings.

2. F - OCD. The most common obsessions in children are fears of contamination, exposure to dirt,
germs or disease. Typical compulsions are cleaning, checking, counting and repetitive behaviours.

3. G - Oppositional defiant behaviour:. Children with ODD often argue with adults, lose their temper
and are angry, resentful and easily annoyed by others.

Incorrect
Marks for this submission: 0/3.
Question 88
Marks: 0/3
Management in Child & Adolescent Psychiatry

A - Atomoxetine
B - Family therapy
C - Fluoxetine
D - Lithium
E - Mirtazepine
F - Parental skills training
G - Risperidone
H - Venlafaxine
I - Do nothing

Choose the most appropriate treatment option in the following. Each option may be used once, more
than once or not at all.
A young girl who is very aggressive and disruptive in school. She has hurt her cat by throwing it on
the wall.
An 8 year old boy presents with auditory hallucinations, thought broadcast and delusions of reference
through TV and magazines.
A 16 year old girl presents with low mood, anhedonhia, self harm behaviour and suicidal ideations.
Answers

1. F - Parents skills training is indicated here as positive parenting is known to have good outcomes.

2. G - Atypical antipsychotics, serotonin dopamine antagonists are current first line treatment for child
& adolescents with schizophrenia. Childhood onset schizophrenia is a rare and severe form of
schizophrenia characterized by onset of psychotic symptoms by the age of 12 yrs.

3. C - SSRIs have been suggested as the first line of treatment of antidepressant medication in
adolescent depression who prefer medication. Fluoxetine has best evidence base and is licensed for use
in children.

Incorrect
Marks for this submission: 0/3.
Question 89
Marks: 0/4
Sexual dysfunction

A - Childhood sexual abuse


B - Fluoxetine
C - Mirtrazepine
D - Performance anxiety
E - Sertraline
F - Trazodone

Choose the most appropriate option in the following. Each option may be used once, more than once or
not at all.
Priapism (Choose one)
Vaginismus (Choose one)
Premature ejaculation (Choose one)
Decreased libido (Choose two)
Answers

1. F - Trazodone. Trazodone causes priapism; prolonged erection in the absence of sexual stimuli, 1 in
10,000 men.

2. A - Childhood sexual abuse. Vaginismus is the involuntary muscle contraction of the outer third of
the vagina that interferes with penile penetration and intercourse, this condition is associated with
history of sexual abuse.

3. D - Performance anxiety. Premature ejaculation is when a man ejaculates before or immediately after
entering the vagina. One of the common cause for this is their concern for partner satisfaction.

4. B,E - All SSRIs are associated with sexual dysfunction with reduced sexual libido being the most
common problem.
Incorrect
Marks for this submission: 0/4.
Question 90
Marks: 0/3
Substance misuse presentations

A - Acute alcohol intoxication


B - Alcoholic hallucinosis
C - Amphetamine intoxication
D - Benzodiazepine intoxication
E - Cannabis intoxication
F - Cocaine intoxication
G - Cocaine withdrawal
H - LSD intoxication
I - Opiate addiction
J - Opiate withdrawal

For each of the following patient presenting in the substance disorder clinic, select the most appropriate
diagnosis.
An 18-year-old man presents to the A&E with (consider the symptoms profile as these can also
happen in stimulant intoxication), dilated pupils, and increased appetite. (Choose one)
A 24 year old man presents with dilated pupils, reduced appetite, auditory hallucinations, insomnia
and irritability. (Choose one)
A 34 year old man presenting with slurred speech, incoordination of movements, and nystagmus
(Choose two)
Answers

1. E - Effects of cannabis intoxication include - Euphoria, relaxation,ilarity, increased appetite and


glazed eyes with large pupils.

2. C - Effects of amphetamine intoxication include - Increased alertness, pulse rate and blood pressure,
and feeling of high energy. Patients with amphetamine intoxication generally present with enlarged
pupils, insomnia, loss of appetite and weight, irritability, and depression.

3. A,D - Acute alcohol and Benzodiazepine intoxication can present with similar signs including inco-
ordination, slurring of speech, ocular signs, confusion etc

Incorrect
Marks for this submission: 0/3.
Question 91
Marks: 0/4
Teratogenic side effects of Psychotropics in pregnancy

A - Carbamazepine
B - Clozapine
C - Fluoxetine
D - Lamotrigine
E - Lithium
F - Tricyclic antidepressants
G - Valproate
H - Venlafaxine
For each of the following congenital defects, choose the most appropriate medication which is
implicated . Each option may be used once, more than once or not at all.
Neural tube defects in foetus.
Atrial and Ventricular septal defects in foetus.
Abnormally placed tricuspid valve in foetus.
Persistent pulmonary hypertension of the new born.
Answers

1. G - Both Valproate and Carbamazepine have been associated with spina bifida. Valproate confers a
higher risk than CBZ.

2. E - Lithium use during pregnancy has a well known association with the cardiac malformation
Ebsteins anomaly at 1:1000.The risk to foetus is maximum during 2-6 weeks after conception before
many women know that they are pregnant. The risk of ASD & VSD is also increased with Lithium.

3. E - Lithium use during pregnancy has a well known association with the cardiac malformation
Ebsteins anomaly at 1:1000.The risk to foetus is maximum during 2-6 weeks after conception before
many women know that they are pregnant. The risk of ASD & VSD is also increased with Lithium.

4. C - When taken in late pregnancy, SSRIs may increase the risk of persistent pulmonary hypertension
of the new born.

Incorrect
Marks for this submission: 0/4.
Question 92
Marks: 0/4
Diagnosis-Hoarding behaviour

A - Autism
B - Collectionism
C - Dementia
D - Depression
E - Diogenes syndrome
F - OCD
G - Schizophrenia

Choose the most appropriate diagnosis in the following. Each option may be used once, more than once
or not at all.
A 50 year old man has history of hoarding newspapers & magazines, inability to discard and feels
distressed if he cant hoard.
A 60 year old lady with a recent history of hoarding rubbish, change in personality and inability to
learn new information.
A 15 year old boy has been collecting rare stamps and has organized these into albums for display.
A 66 year old man living on his own in his mansion has hoarded his mansion with rubbish, refuses
any help, keeps away from his family & neighbours presents with gross self neglect.
Answers

1. F - Compulsive hoarding is a form of OCD associated with excessive hoarding of possessions,


inability to discard possessions, cluttering and feels distressed if they cant hoard objects and associated
with other obsessions as well as compulsions.
Lahera et al,. Differential diagnosis of hoarding behaviors. Actas Esp Psiquiatr 2006;34(6):403-407.
2. C - OF the options provided, this scenario is more suggestive of early onset dementia.
Lahera et al,. Differential diagnosis of hoarding behaviors. Actas Esp Psiquiatr 2006;34(6):403-407.
3. B - Collectionism is a normal phenomenon seen in children and adults with organized activity where
the collected objects are kept in an order not just to hoard them.
Lahera et al,. Differential diagnosis of hoarding behaviors. Actas Esp Psiquiatr 2006;34(6):403-407.
4. E - Diogenes syndrome is a combination of gross self neglect, social withdrawal, hoarding and
refusal of help. More common after 65 yrs and associated with psychiatric diagnosis.
Lahera et al,. Differential diagnosis of hoarding behaviors. Actas Esp Psiquiatr 2006;34(6):403-407.
Incorrect
Marks for this submission: 0/4.
Question 93
Marks: 0/4
Sexual dysfunction

Options

A. Trazodone
B. Sertraline
C. Performance anxiety
D. Childhood sexual abuse
E. Fluoxetine
F. Mirtrazepine

Choose the most appropriate option in the following. Each option may be used once, more than once or
not at all.

Priapism

Vaginismus

Premature ejaculation

Decreased libido

Answers:

1. A - Trazodone. Trazodone causes priapism; prolonged erection in the absence of sexual stimuli, 1 in
10,000 men.

2. D - Childhood sexual abuse. Vaginismus is the involuntary muscle contraction of the outer third of
the vagina that interferes with penile penetration and intercourse, this condition is associated with
history of sexual abuse.

3. C - Performance anxiety. Premature ejaculation is when a man ejaculates before or immediately after
entering the vagina. One of the common cause for this is their concern for partner satisfaction.

4. B, E - All SSRI, s are associated with sexual dysfunction with reduced sexual libido being the most
common problem.

Incorrect
Marks for this submission: 0/4.
Question 94
Marks: 0/3
Substance misuse presentations

Options

A. Acute alcohol intoxication


B. Alcoholic hallucinosis
C. Amphetamine intoxication
D. Benzodiazepine intoxication
E. Cannabis intoxication
F. Cocaine intoxication
G. Cocaine withdrawal
H. LSD intoxication
I. Opiate addiction
J. Opiate withdrawal

For each of the following patient presenting in the substance disorder clinic, select the most appropriate
diagnosis.

An 18-year-old man presents to the A&E with (consider the symptoms profile as these can also happen
in stimulant intoxication), dilated pupils, and increased appetite

A 24 year old man presents with dilated pupils, reduced appetite, auditory hallucinations, insomnia and
irritability.

A 34 year old man presenting with slurred speech, incoordination of movements, and nystagmus

alamal complex

Answers

1. E. Effects of cannabis intoxication include - Euphoria, relaxation,ilarity, increased appetite and


glazed eyes with large pupils.
2. C. Effects of amphetamine intoxication include - Increased alertness, pulse rate and blood pressure,
and feeling of high energy. Patients with amphetamine intoxication generally present with Enlarged
pupils, insomnia, loss of appetite and weight, irritability, and depression.
3. A, D. Acute alcohol and Benzodiazepine intoxication can present with similar signs including inco-
ordination, slurring of speech, ocular signs, confusion etc

Incorrect
Marks for this submission: 0/3.

October 2010 (NEW!)


Review of attempt 3
Started on Sunday, 3 July 2011, 11:06 PM
Completed on Sunday, 3 July 2011, 11:07 PM
Time taken 51 secs
Marks 0/155
Grade 0 out of a maximum of 10 (0%)
Question 1
Marks: 0/1
A old lady has been admitted to a medical ward with a 48 hour history of sudden onset distressing
visual hallucinations, what is the most likely diagnosis?
Choose one answer.
Error! Not a valid embedded object. a. Bipolar affective disorder
Error! Not a valid embedded object. b. Delirium
Error! Not a valid embedded object. c. Depression
Error! Not a valid embedded object. d. Dementia
Error! Not a valid embedded object. e. Schizophrenia
Delirium is a common and severe neuropsychiatric syndrome with core features of acute onset and
fluctuating course, attentional deficits and generalized severe disorganization of behavior. It typically
involves other cognitive deficits, changes in arousal (hyperactive, hypoactive, or mixed), perceptual
deficits, altered sleep-wake cycle, and psychotic features such as hallucinations and delusions. Flaherty
JH, Rudolph J, Shay K, et al. (2007). "Delirium is a serious and under-recognized problem: why
assessment of mental status should be the sixth vital sign". J Am Med Dir Assoc 8 (5): 2735
Incorrect
Marks for this submission: 0/1.
Question 2
Marks: 0/1
The scale used to screen for cognitive impairment in people with Downs syndrome is
Choose one answer.
Error! Not a valid embedded object. a. MMSE
Error! Not a valid embedded object. b. CAMCOG
Error! Not a valid embedded object. c. Mental retardation scale
Error! Not a valid embedded object. d. DMR
Error! Not a valid embedded object. e. CAMDEX
Alzheimer's dementia is quite common in people with Downs syndrome over the age of 35, though the
traditional screening instruments used in adults without downs syndrome may be less useful in
identifying this. The three dementia screening instruments that are currently in use among people with
intellectual disabilities, namely the Dementia Scale for Down Syndrome (DSDS; Gedye, 1995) and the
Dementia Questionnaire for Persons with Mental Retardation (DMR; Evenhuis, 1992, 1996), and the
Dementia Screening Questionnaire for Individuals with Intellectual Disabilities (DSQIID) (Deb et al.,
2007) Deb et al. Dementia Screening Questionnaire for Individuals with Intellectual Disabilities. The
British Journal of Psychiatry (2007) 190: 440-444.
Incorrect
Marks for this submission: 0/1.
Question 3
Marks: 0/1
The following is not seen in Lenox Gastaut syndrome
Choose one answer.
Error! Not a valid a. Diffuse slow spike and wave discharges against the background with
embedded object. paroxysms of fats activity
Error! Not a valid
b. Multiple seizure types
embedded object.
Error! Not a valid
c. Intellectual impairment
embedded object.
Error! Not a valid
d. There is a strong genetic component in the aetilogy
embedded object.
Error! Not a valid
e. Onset between 1 and 8 years of age
embedded object.
A strong genetic component with a family hsitory of epilepsy is seen in upto 1/3 rd of affected children
in Epilepsy with myoclonic seizures Lishmans organic psychiatry, 4 edn,p-318
Incorrect
Marks for this submission: 0/1.
Question 4
Marks: 0/1
A 68-year old man with a previous diagnosis of depression has been recently diagnosed with
Parkinsons disease. His wife complains that in the last 2 weeks he has been sending her sexually-
explicit messages, and accusing her of having an affair. What is the most likely drug to have caused
this?
Choose one answer.
Error! Not a valid embedded object. a. Propranolol
Error! Not a valid embedded object. b. Carbimazole
Error! Not a valid embedded object. c. Ropinirole
Error! Not a valid embedded object. d. Dosulepin
Error! Not a valid embedded object. e. Doxepin
Ropinirole acts as a D2, D3, and D4 dopamine receptor agonist with highest affinity for D3 used in the
treatment of parkinsons disease. Ropinirole can cause nausea, dizziness, hallucinations, orthostatic
hypotension, and sudden sleep attacks during the daytime. Rarer and more unusual side effects specific
to D3-preferring agonists such as ropinirole and pramipexole can include hypersexuality and
compulsive gambling, even in patients without a prior history of these behaviours Bostwick JM,
Hecksel KA, Stevens SR, Bower JH, Ahlskog JE. Frequency of new-onset pathologic compulsive
gambling or hypersexuality after drug treatment of idiopathic Parkinson disease. Mayo Clinic
Proceedings. 2009 84(4):310-6.
Incorrect
Marks for this submission: 0/1.
Question 5
Marks: 0/1
What is the lifetime risk of suicide in alcohol dependence?
Choose one answer.
Error! Not a valid embedded object. a. 2%
Error! Not a valid embedded object. b. 1%
Error! Not a valid embedded object. c. 10%
Error! Not a valid embedded object. d. 5%
Error! Not a valid embedded object. e. 75%
The lifetime risk of suicide among individuals with alcohol dependence treated in out-patient and in-
patient settings was 2.2% and 3.4%, respectively. Nonetheless, individuals with alcohol dependence
have a 60120 times greater suicide risk than the non-psychiatrically-ill population. Sher, L. Alcohol
Consumption and Suicide. QJM 2006, 99 (1): 57-61.
Incorrect
Marks for this submission: 0/1.
Question 6
Marks: 0/1
A 39 year old lady has had many recurrent depressive episodes in the past and one episode of
hypomania. The best medication to prevent relapse for her is:
Choose one answer.
Error! Not a valid embedded object. a. Sodium valproate
Error! Not a valid embedded object. b. Lamotrigine
Error! Not a valid embedded object. c. Carbamezapine
Error! Not a valid embedded object. d. Olanzapine
Error! Not a valid embedded object. e. Lithium
According to NICE guidelines Lithium, Valproate or Olanzapine should be first line for Bipolar
affective disorder. However there is increasing evidence to suggest that Lamotrigine has antidepressant
properties and can be used in patients who suffer from more depressive episodes in Bipolar affective
disorder, as well patients with Bipolar II disorder. The lady in this question appears to have Bipolar II
disorder, and hence Lamotrigine would be a good choice (please note: You could have chosen Lithium,
Olanzapine and Depakote without being completely wrong) Bipolar disorder. NICE guidelines 2006
Incorrect
Marks for this submission: 0/1.
Question 7
Marks: 0/1
A 38-year old lady with multiple sclerosis, presents to her GP with depression. Her GP is concerned,
and asks you what medication may have caused her depression?
Choose one answer.
Error! Not a valid embedded object. a. Anti-epileptics
Error! Not a valid embedded object. b. Mitoxantrone
Error! Not a valid embedded object. c. Interferon beta
Error! Not a valid embedded object. d. Natalizumab
Error! Not a valid embedded object. e. Corticosteroids
A recent meta analysis of 9 studies of Beta intereferon in MS has found no difference between Beta
inteferon and placebo in causing depression as a side effect. Corticosteroids are associated with
depression in several studies. Nikfar S. et al. A meta-analysis of the efficacy and tolerability of
interferon- in multiple sclerosis, overall and by drug and disease type.Clin Ther. 2010;32(11):1871-88.
Incorrect
Marks for this submission: 0/1.
Question 8
Marks: 0/1
A drug A is compared with placebo in the treatment of a condition and outcome used is the time to
hospital admission. Which is the best statistic to use to compare the two interventions?
Choose one answer.
Error! Not a valid embedded object. a. Logrank
Error! Not a valid embedded object. b. Chi-square
Error! Not a valid embedded object. c. ANOVA
Error! Not a valid embedded object. d. Cox's proportional hazard
Error! Not a valid embedded object. e. McNemar
In statistics, the logrank test (sometimes called the MantelCox test) is a hypothesis test to compare the
survival distributions of two samples. It is a nonparametric test and appropriate to use when the data are
right skewed and censored (technically, the censoring must be non-informative). It is widely used in
clinical trials to establish the efficacy of a new treatment compared to a control treatment when the
measurement is the time to event (such as the time from initial treatment to institutionalisation). The
logrank test can also be viewed as a time stratified CochranMantelHaenszel test. KaplanMeier provides
a method for estimating the survival curve, the log rank test provides a statistical comparison of two
groups, and Cox's proportional hazards model allows additional covariates to be included.
Incorrect
Marks for this submission: 0/1.
Question 9
Marks: 0/1
The most common side effect of Rivastigmine is
Choose one answer.
Error! Not a valid embedded object. a. Agitation
Error! Not a valid embedded object. b. Hypertension
Error! Not a valid embedded object. c. Nausea
Error! Not a valid embedded object. d. Diarrhoea
Error! Not a valid embedded object. e. Seizure
The most common adverse effects associated with Rivastigmine are nausea, vomitting, dizziness,
headache, diarrhoea, abdominal pain, anorexia , fatigue, and somnolence Synopsis of Psychiatry, 10th
edn, pg 1034
Incorrect
Marks for this submission: 0/1.
Question 10
Marks: 0/1
You are asked to write a court report for a man convicted of a sexual assault, which of the following is
most likely to increase his risk of subsequent offending?
Choose one answer.
Error! Not a valid embedded object. a. Drug dependence
Error! Not a valid embedded object. b. Diagnosis of schizophrenia
Error! Not a valid embedded object. c. Alcohol dependence
Error! Not a valid embedded object. d. Previous conviction for non-sexual assault
Error! Not a valid embedded object. e. Previous conviction of sexual assault
A meta-analysis of 82 recidivism studies (1,620 findings from 29,450 sexual offenders) identified
deviant sexual preferences and antisocial orientation as the major predictors of sexual recidivism for
both adult and adolescent sexual offenders. Many of the variables commonly addressed in sex offender
treatment programs (e.g., psychological distress, denial of sex crime, victim empathy, stated motivation
for treatment) had little or no relationship with sexual or violent recidivism. RK Hanson, Morton-
Bourgon KE.. The characteristics of persistent sexual offenders: a meta-analysis of recidivism studies.
JOURNAL OF CONSULTING AND CLINICAL PSYCHOLOGY, 2005;73(6):1154-63.
Incorrect
Marks for this submission: 0/1.
Question 11
Marks: 0/1
Publication bias can be detected by using a
Choose one answer.
Error! Not a valid embedded object. a. Galbraith plot
Error! Not a valid embedded object. b. Gaussian curve
Error! Not a valid embedded object. c. Funnel plot
Error! Not a valid embedded object. d. Forest plot
Error! Not a valid embedded object. e. Log-rank test
. A funnel plot is a scatterplot of treatment effect against a measure of study size. It is used primarily as
a visual aid to detecting bias or systematic heterogeneity. A symmetric inverted funnel shape arises
from a well-behaved data set, in which publication bias is unlikely. An asymmetric funnel indicates a
relationship between treatment effect and study size. This suggests the possibility of either publication
bias or a systematic difference between smaller and larger studies (small study effects). Asymmetry can
also arise from use of an inappropriate effect measure. Whatever the cause, an asymmetric funnel plot
leads to doubts over the appropriateness of a simple meta-analysis and suggests that there needs to be
investigation of possible causes. R. J. Light, D. B. Pillemer (1984). Summing up: The Science of
Reviewing Research. Cambridge, Massachusetts.: Harvard University Press.
Incorrect
Marks for this submission: 0/1.
Question 12
Marks: 0/1
The following are correct about school refusal except
Choose one answer.
Error! Not a valid embedded
a. Parents are aware of the absence
object.
Error! Not a valid embedded
b. Child does not comply with completing home work
object.
Error! Not a valid embedded
c. Severe emotional distress about attending school
object.
Error! Not a valid embedded
d. Absence of significant anti social behavior
object.
Error! Not a valid embedded e. Child often persuades parents to be allowed to stay at home
object.
School refusal: Severe emotional distress about attending school; may include anxiety, temper
tantrums, depression, or somatic symptoms, Parents are aware of absence; child often tries to persuade
parents to allow him or her to stay home, Absence of significant antisocial behaviors such as juvenile
delinquency, During school hours, child usually stays home because it is considered a safe and secure
environment, Child expresses willingness to do schoolwork and complies with completing work at
home. WANDA P. FREMONT. School Refusal in Children and Adolescents AFP Oct 2003
Incorrect
Marks for this submission: 0/1.
Question 13
Marks: 0/1
A couple have one child with Down's syndrome. Which of the following type in the child would
increase the chance of their second child having Down's?
Choose one answer.
Error! Not a valid embedded object. a. Disomy 21
Error! Not a valid embedded object. b. Mosaicism
Error! Not a valid embedded object. c. Trisomy 21
Error! Not a valid embedded object. d. Trisomy 23
Error! Not a valid embedded object. e. Translocation at chromosome 21
About 95% of cases of Down syndrome are due to trisomy 21 and the risk of recurrence in a
subsequent child is about 1 in 100. The remaining 5% of cases are attributable either to translocation
involving chromosome 21 or to mosaicism. The translocation is often inherited and the risk of
recurrence is about 1 in 10. Shorter Oxford Textbook of Psychiatry. 5th Edn. 2006. pp 718
Incorrect
Marks for this submission: 0/1.
Question 14
Marks: 0/1
The treatment of choice in childhood PTSD is
Choose one answer.
Error! Not a valid embedded object. a. Trauma focused CBT
Error! Not a valid embedded object. b. EMDR
Error! Not a valid embedded object. c. Debreifing
Error! Not a valid embedded object. d. Psychotherapy
Error! Not a valid embedded object. e. SSRI
Several RCTs have provided evidence for the efficacy of trauma focused cognitive behaviour therapy
in the treatment of PTSD in children PTSD. NICE 2005
Incorrect
Marks for this submission: 0/1.
Question 15
Marks: 0/1
The most common genetic change leading to Downs syndrome is
Choose one answer.
Error! Not a valid embedded object. a. Ring chromosome
Error! Not a valid embedded object. b. Trisomy 23
Error! Not a valid embedded object. c. Trisomy 21
Error! Not a valid embedded object. d. Translocation
Error! Not a valid embedded object. e. Mosaicism
Trisomy 21 is the cause of approximately 95% of observed Down syndromes, with 88% coming from
nondisjunction in the maternal gamete and 8% coming from nondisjunction in the paternal
gamete.Mosaicism is the cause of 12% of the observed Down syndromes.Translocation Down
syndrome is often referred to as familial Down syndrome. It is the cause of 23% of observed cases of
Down syndrome Click here for reference
Incorrect
Marks for this submission: 0/1.
Question 16
Marks: 0/1
What treatment would you use for a patient with severe symptoms of pre-menstrual syndrome
Choose one answer.
Error! Not a valid embedded object. a. Danazol
Error! Not a valid embedded object. b. Primrose oil
Error! Not a valid embedded object. c. Fluoxetine
Error! Not a valid embedded object. d. Lithium
Error! Not a valid embedded object. e. Progesterone
SSRIs have the strongest evidence for their effectiveness in the treatment of PMS. Henshaw (2007)
PMS: Diagnosis, aetiology, assessment and management: REVISITING... PREMENSTRUAL
SYNDROME. Advan. Psychiatr. Treat.13: 139 - 146.
Incorrect
Marks for this submission: 0/1.
Question 17
Marks: 0/1
Which is the treatment of choice in childhood obsessive compulsive disorder?
Choose one answer.
Error! Not a valid embedded object. a. Citalopram
Error! Not a valid embedded object. b. CBT
Error! Not a valid embedded object. c. Paroxetine
Error! Not a valid embedded object. d. Fluoxetine
Error! Not a valid embedded object. e. Sertraline
Although the evidence-base is small for psychological treatment, clinical consensus recommends the
use of psychological treatment as first-line in young people with OCD. Among medication SSRI are
preferred; fluoxetine, sertraline and fluvoxamine can be used in children. NICE clinical guideline 31,
Obsessive-compulsive disorder, 2006, p 169. & BNF 59, Mar 2010, pp 232-235. & Maudsley
guidelines, 10th Edn, pp 260-262.
Incorrect
Marks for this submission: 0/1.
Question 18
Marks: 0/1
The following is true regarding pharmacokinetics in the elderly:
Choose one answer.
Error! Not a valid embedded object. a. There is 25% reduction in GFR in 80-year olds.
Error! Not a valid embedded object. b. The distribution of lipid soluble drugs is increased
Error! Not a valid embedded object. c. No change in hepatic function with age
Error! Not a valid embedded object. d. The rate os absorption if slower
Error! Not a valid embedded object. e. The absorption is less than in younger adults
The pharmacokinetics in the elderly is: Absorption: There is reduced gut motility, leading to drugs
being absorbed slowly ? slower onset of action. The amount of drugs absorbed is similar to young
adults. Distribution: Half-life of drugs is generally increased in older people as they have an increase in
body fat, less body water and less albumin than young adults. This also leads to increased concentration
of drugs at site of action, and more free form available (active form) necessitating use of lower dosages
than younger adults. Metabolism: Though the liver size is reduced, there is no significant reduction in
the metabolic capacity. Excretion: Renal function is reduced by 35% by the age of 65; and 50% by the
age of 80. This leads to reduced capacity to excrete metabolites of the drugs, and in some cases, the
drugs itself (lithium & Sulpiride). A reduction in dosage is required for this reason.
Incorrect
Marks for this submission: 0/1.
Question 19
Marks: 0/1
What is the risk of recurrence of postpartum psychosis in subsequent pregnancies?
Choose one answer.
Error! Not a valid embedded object. a. 50%
Error! Not a valid embedded object. b. 1%
Error! Not a valid embedded object. c. 10%
Error! Not a valid embedded object. d. 25%
Error! Not a valid embedded object. e. 5%
Postpartum psychoses are unusually acute, rapidly reaching a climax of severity. The onset is usually
between 2 and 14 days after delivery. Puerperal recurrences occur after 20 to 25 per cent of subsequent
pregnancies. Non-puerperal recurrences are also common. New Oxford Textbook of Psychiatry,
Volume 2, 2000, p 1203.
Incorrect
Marks for this submission: 0/1.
Question 20
Marks: 0/1
Which is the most suitable alpha 2 agonist for a patient with opioid dependence with tendency to have
low blood pressure?
Choose one answer.
Error! Not a valid embedded object. a. clonidine
Error! Not a valid embedded object. b. Methadone
Error! Not a valid embedded object. c. Lofexidine
Error! Not a valid embedded object. d. Bupropion
Error! Not a valid embedded object. e. Naloxone
Clonidine and lofexidine are both centrally acting alpha-2 agonists. The postulated mechanism of
action is that these drugs stop the firing of noradrenergic neurones in the locus coeruleus, so that the
removal of opiates does not lead to rebound adrenergic firing (which is believed to be responsible for
many of the withdrawal symptoms). Lofexidine has a lower propensity to cause hypotension. Walsh et
al.., Evaluation of the effects of lofexidine and clonidine on naloxone-precipitated withdrawal in
opioid-dependent humans. Addiction, 2003, 98(4), pp 427-439.
Incorrect
Marks for this submission: 0/1.
Question 21
Marks: 0/1
Which of the following may worsen psoriasis?
Choose one answer.
Error! Not a valid embedded object. a. Valproate
Error! Not a valid embedded object. b. Clonazepam
Error! Not a valid embedded object. c. Chlorpromazine
Error! Not a valid embedded object. d. Lamotrigine
Error! Not a valid embedded object. e. Lithium
Some skin conditions such as psoriasis and acne can be aggravated by lithium therapy. Maudsley
Prescribing Guidelines, 10th Edn. P 129.
Incorrect
Marks for this submission: 0/1.
Question 22
Marks: 0/1
A woman with breast cancer has developed moderate depression. She is on tamoxifen treatment. Which
antidepressant would you avoid in this lady?
Choose one answer.
Error! Not a valid embedded object. a. Amitryptiline
Error! Not a valid embedded object. b. Venlafaxine
Error! Not a valid embedded object. c. Fluoxetine
Error! Not a valid embedded object. d. Moclobomide
Error! Not a valid embedded object. e. Mirtazepine
In several studies, concurrent use of tamoxifen with the potent CYP2D6-inhibitor antidepressants
paroxetine and fluoxetine, was associated with a significant reduction in circulating endoxifen levels
(active tamoxifen metabolite) in some women. Based on current research, the psychotropic medications
which are the strongest CYP2D6 inhibitors include paroxetine, fluoxetine, buproprion, duloxetine,
while sertraline, escitalopram, and doxepin are moderate inhibitors, and venlafaxine is a weak inhibitor.
Jin et al. CYP2D6 genotype, antidepressant use, and tamoxifen metabolism during adjuvant breast
cancer treatment, J Natl Cancer Inst. 2005;97(1):30-9.
Incorrect
Marks for this submission: 0/1.
Question 23
Marks: 0/1
The diagnosis with most heritability among thw following is?
Choose one answer.
Error! Not a valid embedded object. a. ADHD
Error! Not a valid embedded object. b. School refusal
Error! Not a valid embedded object. c. Oppositional defiant disorder
Error! Not a valid embedded object. d. Conduct disorder
Error! Not a valid embedded object. e. Childhood OCD
The disorders with high heritability ie >70% include Autism, ADHD, Schizophrenia and BPAD. The
second group with moderate heritability ie 30-60% include unipolar depression,ODD,conduct disorders
and anxiety disorders. Child and adolescent Psychiatry By Michael Rutter, Eric A. Taylor,p-190
Incorrect
Marks for this submission: 0/1.
Question 24
Marks: 0/1
Smoking during pregnancy can cause the following in the child
Choose one answer.
Error! Not a valid embedded object. a. Premature delivery
Error! Not a valid embedded object. b. SIDS
Error! Not a valid embedded object. c. Cranial anomalies
Error! Not a valid embedded object. d. Neurobehavioral sequalae
Error! Not a valid embedded object. e. A syndrome similar to fetal alcohol syndrome
Nicotine is associated with miscarriage, IUGR and premature delivery Companion to psychiatric
studies,8 edn,p-788
Incorrect
Marks for this submission: 0/1.
Question 25
Marks: 0/1
A young man comes to your outpatient clinic concerned that he may develop problem drinking. His
father and his aunt were heavy drinkers and were dependant on alcohol. What are is the increase in risk
of him becoming alcohol dependant.
Choose one answer.
Error! Not a valid embedded object. a. 6 fold
Error! Not a valid embedded object. b. 5 fold
Error! Not a valid embedded object. c. 2 fold
Error! Not a valid embedded object. d. 3 fold
Error! Not a valid embedded object. e. 4 fold
The riskof alcohol dependence in an individual with a first degree relative in increased twofold. There
is also an increased risk of antisocial PD, drug dependence, anxiety and mood disorder. Nurnberger et
al (2004) A Family Study of Alcohol Dependence: Coaggregation of Multiple disorders in Relatives of
Alcohol Dependent Probands. Arch Gen Psychiatry 61(12):1246-1256
Incorrect
Marks for this submission: 0/1.
Question 26
Marks: 0/1
Which drug has the best evidence to treat acute mania?
Choose one answer.
Error! Not a valid embedded object. a. Valproate
Error! Not a valid embedded object. b. Lorazepam
Error! Not a valid embedded object. c. Lithium
Error! Not a valid embedded object. d. Lamotrigine
Error! Not a valid embedded object. e. Carbamazepine
Only lithium, olanzapine, quetiapine, risperidone and valproate semisodium are licensed for the
treatment of acute mania in the UK. Lithium probably has the best evidence in the treatment of acute
mania. Bipolar disorder. NICE guidelines 2006
Incorrect
Marks for this submission: 0/1.
Question 27
Marks: 0/1
Subcultural causes for mild LD does not include
Choose one answer.
Error! Not a valid embedded object. a. Low income
Error! Not a valid embedded object. b. Social class 4 or 5
Error! Not a valid embedded object. c. Room overcrowding
Error! Not a valid embedded object. d. High social class
Error! Not a valid embedded object. e. large sibship size
Higher rates of mild LD in social class IV & V, especially with large sibship, overcrowding, maternal
educational level & poverty point to sub-cultural causes' of learning disability . Companion to
psychiatric studies, 8th edn,p-545
Incorrect
Marks for this submission: 0/1.
Question 28
Marks: 0/1
You are seeing the parents of a 26 year old gentleman currently under your care. He has been
diagnosed with Schizophrenia and his parents want to know his prognosis. What type of schizophrenia
has the best prognosis out of the following
Choose one answer.
Error! Not a valid embedded object. a. Paranoid schizophrenia
Error! Not a valid embedded object. b. Hebephrenic schizophrenia
Error! Not a valid embedded object. c. Catatonic schizophrenia
Error! Not a valid embedded object. d. Simple schizophrenia
Error! Not a valid embedded object. e. Residual schizophrenia
Factors associated with good prognosis are:paranoid type,female,married,abrupt onset,family history of
affective disorder,lack of negative and cognitive symptoms. Companion to Psychiatric Studies, 8th edn,
pg 399.
Incorrect
Marks for this submission: 0/1.
Question 29
Marks: 0/1
Which of the following medications could cause a decrease in plasma lithium level?
Choose one answer.
Error! Not a valid embedded object. a. Non-steroidal anti-inflammatory drugs
Error! Not a valid embedded object. b. Theophylline
Error! Not a valid embedded object. c. ACE inhibitors
Error! Not a valid embedded object. d. Angiotensin-II receptor anatagonists
Error! Not a valid embedded object. e. SSRI
Due to narrow therapeutic index of lithium, awareness of drug interactions is important. Theophylline
and Sodium bicarbonate increase lithium levels. Diuretics, NSAID, ACE inhibitors, Angiotensin-II
receptor antagonists and certain antibiotics can increase lithium levels. Lithium can enhance levels of
SSRI, antipsychotics, carbamazepine, phenytoin and calcium channel blockers, and can lead to
increased side effects and toxicity by these agents. Shorter Oxford Textbook of Psychiatry. 5th Edn.
2006. pp 559-560
Incorrect
Marks for this submission: 0/1.
Question 30
Marks: 0/1
Neuroimaging studies in patients with OCD show
Choose one answer.
Error! Not a valid embedded object. a. Hypometabolism in the parietal cortex
Error! Not a valid embedded object. b. Hypermetabolism of parahippocampal gyrus
Error! Not a valid embedded object. c. Reduced metabolism in caudate nuclei
Error! Not a valid embedded object. d. Hypermetabolism in Orbitofrontal cortex
Error! Not a valid embedded object. e. Hypometabolism in the orbitofrontal cortex
PET studies have shown significant increases in metabolism in left orbital gyrus and both caudate
nuclei which also improved with drug treatment. OCD symptoms are mediated by hyperactivity in
orbitofrontal-subcortical circuits, perhaps due to an imbalance of tone between direct and indirect
striato-pallidal pathways. Companion to Psychiatric Studies, 8th edn, pg 89.
Incorrect
Marks for this submission: 0/1.
Question 31
Marks: 0/1
Which of the following is most commonly seen in Progressive Supranuclear Palsy?
Choose one answer.
Error! Not a valid embedded object. a. Falling forward
Error! Not a valid embedded object. b. Cognitive impairment is uncommon
Error! Not a valid embedded object. c. Bradyphrenia
Error! Not a valid embedded object. d. Onset in 4th decade
Error! Not a valid embedded object. e. Neurofibrillary tangles in cortex
PSP is one of the Parkinson's + syndrome charecterised by supranuclear paralysis of external ocular
movements (especially in the vertical plane and involving downward gaze), pseudobulbar palsy,
dystonic rigidity, cognitive impairment (in up to 80% of patients and most striking cognitive change is
bradyphrenia, i.e.., slowing of response) and signs of pyramidal tract and cerebellar dysfunction. A
tendency to fall backwards is charecteristic. The estimated prevalence is 5-6 per 100000 with onset
usually in the sixth decade. The pathology shows cell loss, neurofibrillary tangles, gliosis and
demyelination particularly affecting the basal ganglia, brainstem and cerebellar nuclei. Lishman's
Organic Psychiatry. 4th Edn. 2009. pp 776-777
Incorrect
Marks for this submission: 0/1.
Question 32
Marks: 0/1
A 70-year old woman whose sister was diagnosed with Alzheimers wants to know the odds of her
getting the disorder relevant to the normal population
Choose one answer.
Error! Not a valid embedded object. a. 3 times higher
Error! Not a valid embedded object. b. 5 times higher
Error! Not a valid embedded object. c. Similar risk
Error! Not a valid embedded object. d. 9 times higher
Error! Not a valid embedded object. e. 7 times higher
The risk of Alzheimer's disease is raised 3-4 fold in first degree relatives. However in the average
lifespan only a third of this risk is realised. Liddell & Lovestone (2001) Genetic risk of Alzheimer's
disease: advising relatives The British Journal of Psychiatry 178: 7-11 available online at Click here for
reference
Incorrect
Marks for this submission: 0/1.
Question 33
Marks: 0/1
In prader willi syndrome the following is commonly seen
Choose one answer.
Error! Not a valid embedded object. a. Depression
Error! Not a valid embedded object. b. Compulsive behaviour-overeating
Error! Not a valid embedded object. c. Severe self mutilation
Error! Not a valid embedded object. d. Self picking
Error! Not a valid embedded object. e. Severe Learning disability
Compulsive behaviour in the form of overeating is the most frequent feature and is usually associated
with borderline or moderate LD Kaplan & Saddock,10 edn,p-1148,synopsis of psychiatry
Incorrect
Marks for this submission: 0/1.
Question 34
Marks: 0/1
A pregant woman who is dependent on heroin is keen to undergo detoxification. Which of the
following is most apporpriate?
Choose one answer.
Error! Not a valid embedded object. a. Lofexidine
Error! Not a valid embedded object. b. Clonidine
Error! Not a valid embedded object. c. Methadone
Error! Not a valid embedded object. d. Naltrexone
Error! Not a valid embedded object. e. Bupropion
Methadone maintenance results in improved maternal and fetal health and should be offered to opioid
dependent pregnant women. However, for women who are opioid dependent during pregnancy and
prefer detoxification, this should only be undertaken with caution. Detoxification should be avoided in
the first trimester, is preferred in the second and used with caution in third. Methadone is the best
known substitute pharmacotherapy in pregnancy and will usually be the first choice. Lingford-Hughes,
Evidence-based guidelines for the pharmacological management of substance misuse, addiction and
comorbidity: recommendations from the British Association for Psychopharmacology. Journal of
Psychopharmacology. 18(3): 2004, pp 293335.
Incorrect
Marks for this submission: 0/1.
Question 35
Marks: 0/1
What is the term used to describe the loss of productivity to patients and employers when patients had
to seek treatment?
Choose one answer.
Error! Not a valid embedded object. a. Direct costs
Error! Not a valid embedded object. b. Indirect costs
Error! Not a valid embedded object. c. Clinical costs
Error! Not a valid embedded object. d. Intangible costs
Error! Not a valid embedded object. e. QALY
The indirect costs include the work days lost and also include the value of unpaid work.The direct costs
include bording,lodging in hosiptal,drugs,dressings,investigations,staff salaries etc. The intangible costs
include the pain ,suffering and social stigma How to read a paper,Trisha Greenlagh,e 2 edn,p-154
Incorrect
Marks for this submission: 0/1.
Question 36
Marks: 0/1
The cost of forgoing an established treatment whilst opting for a new treatment is termed as
Choose one answer.
Error! Not a valid embedded object. a. Cost benefit
Error! Not a valid embedded object. b. Cost effectiveness
Error! Not a valid embedded object. c. Indirect cost
Error! Not a valid embedded object. d. Opportunity cost
Error! Not a valid embedded object. e. Direct cost
The opportunity cost of investing in a newintervention is best measured by the health benefits (life
years saved, quality adjusted life years (QALYs) gained) that could have been achieved had the money
been spent on the next best alternative intervention. Stephen Palmer and James Raftery. Economics
notes: Opportunity cost. BMJ 1999; 318: 1551 - 1552
Incorrect
Marks for this submission: 0/1.
Question 37
Marks: 0/1
A depressed patient is on Venlafaxine 225mg and Mirtazapine 45mg but is not improving, which of the
following do you do?
Choose one answer.
Error! Not a valid embedded object. a. Stop Venlafaxine and augment with lithium
Error! Not a valid embedded object. b. Stop both and start another antidepressant
Error! Not a valid embedded object. c. Stop mirtazapine and augment with lithium
Error! Not a valid embedded object. d. ECT
Error! Not a valid embedded object. e. Augment with lithium and continue
NICE has recommended that ECT is considered for acute treatment of severe depression that is life-
threatening and when a rapid response is required, or when other treatments have failed. One of the
comprehensive studies on the pharmacotherapy of refractory depression, the Sequenced Treatment
Alternatives to Relieve Depression (STAR-D) places the venlafaxine+mirtazapine combination in stage
4 (in stage 1 to 4 in a in a step wise fashion) and if there is no response to such a combination then
pharmacotherapy could be considered ineffective and ECT considered as a treatment option. NICE
clinical guideline 90. Depression: The treatment and management of depression in adults, 2009. and
Maudsley Prescribing Guidelines, 10th Edn, pp 177-179.
Incorrect
Marks for this submission: 0/1.
Question 38
Marks: 0/1
A 29 year old man has been using drugs of recreation and asks you about the one which is less likely to
have dependence?
Choose one answer.
Error! Not a valid embedded object. a. Cannabis
Error! Not a valid embedded object. b. LSD
Error! Not a valid embedded object. c. Amphetamines
Error! Not a valid embedded object. d. Cocaine
Error! Not a valid embedded object. e. Ecstacy
For much of 1970s cannabis was not regarded as a drug of dependence but studies have shown features
of dependence particularly in heavy users. However compared to other illicit drugs cannabis has less
physical dependence. Opiods and crack to have most addictive potential and LSD to have minimal
potential for psychological & physiological addiction. Companion to Psychiatric Studies, 8th edition,
Page 381.
Incorrect
Marks for this submission: 0/1.
Question 39
Marks: 0/1
A 36 year old man with moderate depression has been given a trial of fluoxetine, and has shown a
partial response, with symptoms of decreased sleep still persisting. Which of the following medication
would be a useful strategy for augmentation?
Choose one answer.
Error! Not a valid embedded object. a. Risperidone
Error! Not a valid embedded object. b. Mianserin
Error! Not a valid embedded object. c. Mirtazepine
Error! Not a valid embedded object. d. Aripiprizole
Error! Not a valid embedded object. e. Lithium
The current NICE guidance (Oct 2009) advises using Lithium, Mirtazepine, Mianserin, Antipsychotics
for augmenting antidepressants in treatment of depression in adults. In this case of a person with sleep
problems, it may be useful to use Mirtazepine as an augmenting agent. Depression in Adults. NICE
2009.
Incorrect
Marks for this submission: 0/1.
Question 40
Marks: 0/1
A pregnant woman who had a previous depressive episode is now euthymic and not on any
psychotropic medications. What is her risk of depressive relapse in the postnatal period?
Choose one answer.
Error! Not a valid embedded object. a. 40%
Error! Not a valid embedded object. b. 50%
Error! Not a valid embedded object. c. 20%
Error! Not a valid embedded object. d. 30%
Error! Not a valid embedded object. e. 10%
Previous history of depression is one of the strongest predictors of Postnatal depression, with studies
the chance of a woman with previous history of depression developing postnatal depression being 1:3.
Bloch M et al., Risk Factors for early postpartum depressive symptoms. General Hospital Psychiatry
2006:28(1):3-8
Incorrect
Marks for this submission: 0/1.
Question 41
Marks: 0/1
The following is true regarding learning disability and PTSD
Choose one answer.
Error! Not a valid embedded a. People with Learning disability not affected by traumatic
object. events
Error! Not a valid embedded
b. Accidents are the commonly experienced traumatic event
object.
Error! Not a valid embedded
c. People with learning disability overdiagnosed with PTSD
object.
Error! Not a valid embedded d. Sexual assault is the commonly experienced precipitating
object. trauma
Error! Not a valid embedded
e. OCD is the common presentation after traumatic events
object.
There has been only one reported major study of adults with learning disability diagnosed to have
PTSD (Ryan, 1994). This was of a clinic population of 51 adults and showed that people with learning
disability develop PTSD at a rate comparable to the able population when exposed to trauma. Each
person had suffered at least two types of trauma. That most frequently experienced was sexual abuse by
multiple perpetrators (commonly starting in childhood), physical abuse or life-threatening neglect
committed with some other active abuse or trauma. Post-traumatic stress disorder in people with
learning disability ,Jane McCarthy ,Advances in Psychiatric Treatment (2001) 7: 163-169
Incorrect
Marks for this submission: 0/1.
Question 42
Marks: 0/1
A patient who has recently completed an inpatient detoxification from opiates would like to be
prescribed something that will help to prevent relapse. Which of the following has the best evidence
base for maintainence treatment for this person.
Choose one answer.
Error! Not a valid embedded object. a. Naltrexone
Error! Not a valid embedded object. b. Suboxone
Error! Not a valid embedded object. c. Naloxone
Error! Not a valid embedded object. d. Lofexidine
Error! Not a valid embedded object. e. Buprenorphine
Buprenorphine and Methadone have been recommended as Ist line in the maintenance treatment of
opiate dependence. Naltrexone is used in the abstinence based treatments to prevent relapse, not as first
line. Due to shorter half life and parenteral route Naloxone is not a practical option. NICE guidelines,
2007
Incorrect
Marks for this submission: 0/1.
Question 43
Marks: 0/1
The pattern of inheritance of Klinefelters syndrome
Choose one answer.
Error! Not a valid embedded object. a. X-linked dominant
Error! Not a valid embedded object. b. Sporadic
Error! Not a valid embedded object. c. Autosomal recessive
Error! Not a valid embedded object. d. X-linked recessive
Error! Not a valid embedded object. e. Autosomal dominant
Klinefelter syndrome is not inherited; it usually occurs as a random event during the formation of
reproductive cells (eggs and sperm). An error in cell division called nondisjunction results in a
reproductive cell with an abnormal number of chromosomes. For example, an egg or sperm cell may
gain one or more extra copies of the X chromosome as a result of nondisjunction. If one of these
atypical reproductive cells contributes to the genetic makeup of a child, the child will have one or more
extra X chromosomes in each of the body's cells. Click here for reference
Incorrect
Marks for this submission: 0/1.
Question 44
Marks: 0/1
Which of the following would be the most appropriate next step for someone who has been
unsuccesfully treated with both buprenorphine and methadone?
Choose one answer.
Error! Not a valid embedded object. a. Optimise oral Methadone dose
Error! Not a valid embedded object. b. IV Buprenorphine
Error! Not a valid embedded object. c. Avoid opiates
Error! Not a valid embedded object. d. IV Methadone
Error! Not a valid embedded object. e. IV Heroin
Injectable diacetylmorphine (diamorphine or heroin) was more effective than oral methadone, but
because of risks of overdose and seizures, prompt medical intervention should be available in settings
where it is available. Methadone should remain the treatment of choice for the majority of patients.
However injectable heroin can be an option if adequate doses of methadone and buprenorphine do not
give the desired results. Brissette S, Marsh DC, et al. Diacetylmorphine versus methadone for the
treatment of opioid addiction. N Engl J Med, 2009, 361, pp 777-786.
Incorrect
Marks for this submission: 0/1.
Question 45
Marks: 0/1
The following is true with regards to Autism.
Choose one answer.
Error! Not a valid embedded
a. Hereitability over 90%
object.
Error! Not a valid embedded
b. Not associated with Fragile X syndrome
object.
Error! Not a valid embedded c. Rates have been found to be higher in patients exposed to the
object. MMR vaccine
Error! Not a valid embedded
d. Perinatal complications not associated with autism
object.
Error! Not a valid embedded
e. Monozygotic concordance rate less than 10%
object.
Early studies of twins estimated the heritability of autism to be more than 90%; in other words, that
90% of the differences between autistic and non-autistic individuals is due to genetic effects.
[Monozygotic concordance is >90%. Autism is seen in around 25% of male patients with fragile X and
6% of females. Perinatal complications are associated with autism (there is a higher prevalence of
complications, although note that this may not be causal).] Rutter (2000) Genetic Studies of Autism:
From the 1970s into the Millenium. Journal of Abnormal Child Psychology. 28(1) 3-14 and Abrahams
BS, Geschwind DH (2008) Advances in autism genetics: on the threshold of a new neurobiology. Nat
Rev Genet 9 (5): 341-55.
Incorrect
Marks for this submission: 0/1.
Question 46
Marks: 0/1
According to NINCDS-ARDA criteria, A diagnosis of Alzheimer's dementia is unlikely if
Choose one answer.
Error! Not a valid embedded object. a. Progressive deterioration of specific cognitive function
Error! Not a valid embedded object. b. No loss of consciousness
Error! Not a valid embedded object. c. Plateaux in the course of progression
Error! Not a valid embedded object. d. There is gait disturbance at the onset of illness
Error! Not a valid embedded object. e. Onset is only after forty years of age
The National Institute of Neurological & Communicable diseases and Stroke-Alzheimer's disease and
related disorders association (NINCDS-ARDA) have provided criteria for the clinical diagnosis of
Alzhemier's disease. Diagnosis of AD is unlikely if there is sudden onset, focal neurological signs and
incoordination early in the course, seizures or gait disturbances at the onset or very early in the course
of the illness. Companion to Psychiatric Studies 8th Edn, p658.
Incorrect
Marks for this submission: 0/1.
Question 47
Marks: 0/1
A depressed woman is prescribed a tricyclic antidepressant. Which of the following is is more likely to
tolerate?
Choose one answer.
Error! Not a valid embedded object. a. Lofepramine
Error! Not a valid embedded object. b. Imipramine
Error! Not a valid embedded object. c. Nortriptyline
Error! Not a valid embedded object. d. Dosulpin
Error! Not a valid embedded object. e. Amitriptyline
Lofepramine has demonstrated a low incidence of cardiac toxicity. Lofepramine has lower incidence of
side effects and is less dangerous in over dosage but is infrequently associated with hepatic toxicity.
Newer TCAs (lofepramine and dothiepin) have been shown to have a greater tolerability than older
TCAs. Both dothiepin and lofepramine showed lower discontinuous ratios for side effects compared to
older TCAs, but there was no difference in efficacy. The discontinuation ratio of newer TCAs was
31.4% compared with 39.0% for older TCAs.It has been demonstrated that the efficacies are both the
same for lofepramine and amitriptyline but patients treated with lofepramine experienced less side
effects compared to amitriptyline.Both the groups investigated had high drop out rates, but lofepramine
had lower drop out rates Department of Health. Prescribing figures. London: Department of Health;
2002.
Incorrect
Marks for this submission: 0/1.
Question 48
Marks: 0/1
The mechanism of action of Modafinil includes
Choose one answer.
Error! Not a valid embedded object. a. Dopamine reuptake inhibition
Error! Not a valid embedded object. b. Dopaminergic agonism
Error! Not a valid embedded object. c. Alpha 1 adrenergic antagonism
Error! Not a valid embedded object. d. Alpha 1 adrenergic agonism
Error! Not a valid embedded object. e. Dopaminergic antagonism
Modafinil activates hypocretin producing neurons thus useful in Narcolepsy. Modafinil is reported to
act by alpha 1 agonism accounting for its alertness. Kaplan & saddock,10 edn,p-1099
Incorrect
Marks for this submission: 0/1.
Question 49
Marks: 0/1
Which instrument is used for assessing psychopathy?
Choose one answer.
Error! Not a valid embedded object. a. Static-99
Error! Not a valid embedded object. b. HCR-20
Error! Not a valid embedded object. c. SARA
Error! Not a valid embedded object. d. PCL-R
Error! Not a valid embedded object. e. VRAG
The most widely used measure of psychopathy is the Hare Psychopathy Checklist. Hare PCL-R has
gradually come to be used to assess likely future recidivism and violent offending. It is a 20-item rating
scale, scored on the basis of both semi-structured interview and collateral information. It has been
validated for use in adult male correctional and forensic psychiatric samples. Over recent years,
research has shown that it is a relatively good predictor of violence across diverse populations.The
Hare Psychopathology Checklist Revised (PCL-R), Historical Clinical risk (HCR-20), and Violence
risk appraisal guide are actuarial risk instruments for violent recidivism. Spousal Assault risk
assessment guide (SARA) is an instrument to assess risk of assault by spouse, and the Sex offender risk
appraisal guide is to assess risk of sexual recidivism. Dale, et al., Forensic Mental Health: Issues in
Practice. Elsevier Publishing. 2005. p 172.
Incorrect
Marks for this submission: 0/1.
Question 50
Marks: 0/1
Autism is most likely to be associated with which of the following:
Choose one answer.
Error! Not a valid embedded object. a. Asperger's syndrome
Error! Not a valid embedded object. b. Williams syndrome
Error! Not a valid embedded object. c. Gilles De La Tourette
Error! Not a valid embedded object. d. ADHD
Error! Not a valid embedded object. e. Conduct disorder
Hyperactivity is common with autism Goodman & Scott, Child Psychiatry, 2002, pg 43
Incorrect
Marks for this submission: 0/1.
Question 51
Marks: 0/1
The following is a feature of advanced heroin withdrawal in a young man you see in A&E
Choose one answer.
Error! Not a valid embedded object. a. Increased respiration
Error! Not a valid embedded object. b. Headache
Error! Not a valid embedded object. c. Dilated pupils
Error! Not a valid embedded object. d. Increased lacrimation
Error! Not a valid embedded object. e. Agitation
Opioid withdrawal reactions are very uncomfortable but are not life threatening. Symptoms usually
start within 12 hours of last heroin usage and within 30 hours of last methadone exposure. Early
symptoms of withdrawal include: Agitation, Anxiety, Muscle aches, Increased lacrimation, Insomnia,
Runny nose, Sweating, Yawning. Late symptoms of withdrawal include: Abdominal cramping,
Diarrhea, Dilated pupils, Goose bumps, Nausea & Vomiting Revision Notes in Psychiatry - Puri &
Hall, 2nd Edition (2004) p353
Incorrect
Marks for this submission: 0/1.
Question 52
Marks: 0/1
A patient has been recently diagnosed with Vascular dementia. A past hx of cerebrovacular disease in
this patient would be most evident by which of the following?
Choose one answer.
Error! Not a valid embedded object. a. History of psychotic features
Error! Not a valid embedded object. b. History of epilepsy
Error! Not a valid embedded object. c. History of slurred speech
Error! Not a valid embedded object. d. History of low mood
Error! Not a valid embedded object. e. History of anxiety disorders
History of CVD is suggested by slurred speech, incontinence, ataxia, falls , weakness etc. Organic
psychiatry,Lishman,4 edn,p-568
Incorrect
Marks for this submission: 0/1.
Question 53
Marks: 0/1
The ideal value for kappa is
Choose one answer.
Error! Not a valid embedded object. a. 0
Error! Not a valid embedded object. b. 1
Error! Not a valid embedded object. c. 0.5
Error! Not a valid embedded object. d. -1
Error! Not a valid embedded object. e. -0.5
Kappa is a measure of inter rater reliability. If the raters are in complete agreement then Kappa = 1. If
there is no agreement among the raters (other than what would be expected by chance) then Kappa = 0.
Companion to Psychiatric Studies, 8th edn, pg 159.
Incorrect
Marks for this submission: 0/1.
Question 54
Marks: 0/1
Which of the following is not a significant variable predictor of Sexual Offence Recidivism in
Offenders with Intellectual Disabilities
Choose one answer.
Error! Not a valid embedded object. a. Acceptance of crime
Error! Not a valid embedded object. b. Sexual abuse in childhood
Error! Not a valid embedded object. c. Poor relationship with mother
Error! Not a valid embedded object. d. Allowances made by staff
Error! Not a valid embedded object. e. Antisocial attitude
Significant variables were allowances made by staff, antisocial attitude, poor relationship with mother,
denial of crime, sexual abuse in childhood, erratic attendance and poor response to treatment. Certain
variables, found significant in earlier studies, did not emerge in the current analysis. These predictors
accounted for around 53% of the variance for evidence of re-offending and around 74% of the variance
for suspicion of re-offending William R. Lindsay, Susanne F. Elliot and Arlene Astell ,Predictors of
Sexual Offence Recidivism in Offenders with Intellectual Disabilities. Journal of Applied Research in
Intellectual Disabilities 2004, 17 (4); 299 - 305
Incorrect
Marks for this submission: 0/1.
Question 55
Marks: 0/1
The following is true about cocaine users
Choose one answer.
Error! Not a valid embedded a. Mood disorders usually precedes the onset of cocaine related
object. disorders
Error! Not a valid embedded
b. Alcoholism is the commonest lifetime comorbidity
object.
Error! Not a valid embedded
c. Schizophrenia is one of the commonest comorbidties
object.
Error! Not a valid embedded
d. Cocaine use is on the rise
object.
Error! Not a valid embedded e. Anxiety disorders commonly following the onset of cocaine
object. related disorders
Current cocaine use is on the decline with anxiety disorders, ADHD and antisocial PD preceding the
development of cocaine related disorders.
Incorrect
Marks for this submission: 0/1.
Question 56
Marks: 0/1
Which of the following psychotherapies utilises reciprocal role reversal
Choose one answer.
Error! Not a valid embedded object. a. Dialectical Behavioural Therapy
Error! Not a valid embedded object. b. Dialectical behavioral therapy
Error! Not a valid embedded object. c. Energy Psychology
Error! Not a valid embedded object. d. Alums group therapy
Error! Not a valid embedded object. e. Moreno's pscyhodrama
Two forms used in Psychodrama:1.Reciprocal (proper,classical,in situ) role reversal, based on social
psychology, is used mainly as an aid for dealing with people in the outer world, as a way of correcting
biased perceptions of other people and receiving feedback of oneself and as an interpersonal conflict
resolution technique.2. Representational(incomplete) role reversal, based on object relations theory, is
used more as an aid for the externalisation and interpolation of the inner world of one protagonist
Moreno, J.L. and Moreno, Z.T. (1959) Psychodrama, vol. 2, New York: Beacon House.
Incorrect
Marks for this submission: 0/1.
Question 57
Marks: 0/1
In a survey done by community team on patients in the community the response rates were very low .
The response rates can be improved by
Choose one answer.
Error! Not a valid embedded object. a. By doing home visits and interviewing patients
Error! Not a valid embedded object. b. Response incentives
Error! Not a valid embedded object. c. Sending the questionaire to a large population sample
Error! Not a valid embedded object. d. Using all the questionaires in English
Error! Not a valid embedded object. e. Using semi structured interviews
Studies have shown that response rates can be improved by using incentives. The response rates have
been between 15-50% with incentives. Its important to look into the possibility of bias in respones due
to the incentive. Measuring and improving patient satisfaction By Patrick J. Shelton,p-169
Incorrect
Marks for this submission: 0/1.
Question 58
Marks: 0/1
The pattern of inheritance of Picks disease.
Choose one answer.
Error! Not a valid embedded object. a. X-linked dominant
Error! Not a valid embedded object. b. Autosomal dominant
Error! Not a valid embedded object. c. X-linked recessive
Error! Not a valid embedded object. d. Autosomal recessive
Error! Not a valid embedded object. e. Environmental
An autosomal dominant mode of inheritance is found in 1027% of all Pick's dementia patients. Seelar
H et al. Clinical, genetic and pathological heterogeneity of frontotemporal dementia: a review. J Neurol
Neurosurg Psychiatry. 2010
Incorrect
Marks for this submission: 0/1.
Question 59
Marks: 0/1
Which of the following represent 'acting out' during therapy in an individual with antisocial personality
disorder?
Choose one answer.
Error! Not a valid embedded a. Praising the therapist in the session when actually angry with the
object. therapist
Error! Not a valid embedded
b. Avoiding painful discussion in session
object.
Error! Not a valid embedded
c. Missing sessions
object.
Error! Not a valid embedded
d. Late for session
object.
Error! Not a valid embedded
e. Leaving the session suddenly and getting drunk
object.
Acting out is enacting an unconscious wish or fantasy impulsively as a way of avoiding painful affect.
Acting out is characteristic of Psychopathy and Behavior Disorders and reduces the accessibility of
these conditions to psychoanalysis. A patient is said to be acting out if he engages in activity which can
be interpreted as a substitute for remembering past events. The essence of the concept is the
replacement of thought by action and it implies that either (a) the impulse being acted out has never
acquired verbal representation, or (b) the impulse is too intense to be dischargeable in words, or (c) that
the patient lacks the capacity of inhibition. There may commonly be sabotage of the therapeutic setting
in a variety of ways if management is insufficient, e.g., there may be gross misuse of drugs or alcohol.
Oxford Textbook of Psychotherapy. 2005. pp 5, 269-270.
Incorrect
Marks for this submission: 0/1.
Question 60
Marks: 0/1
The database that provides systematic coverage of the psychological literature from the 1800s to the
present is
Choose one answer.
Error! Not a valid embedded object. a. Pubmed
Error! Not a valid embedded object. b. Medline
Error! Not a valid embedded object. c. Psychinfo
Error! Not a valid embedded object. d. EMBASE
Error! Not a valid embedded object. e. Psychlit
PsycINFO is an abstract database that provides systematic coverage of the psychological literature from
the 1800s to the present. PsycINFO contains bibliographic citations, abstracts, cited references, and
descriptive information to help you find what you need across a wide variety of scholarly publications
in the behavioral and social sciences. Medline provides information from 1965, Psychlit from 1887 and
EMbase from 1947. J Martin Bland and Douglas G Altman. Statistics notes: Cronbach's alpha. BMJ
Feb 1997; 314: 572
Incorrect
Marks for this submission: 0/1.
Question 61
Marks: 0/1
The most common symptom of delirium is
Choose one answer.
Error! Not a valid embedded object. a. Sleep wake cycle disturbances
Error! Not a valid embedded object. b. Retardation
Error! Not a valid embedded object. c. Agitation
Error! Not a valid embedded object. d. Audiotry hallucinations
Error! Not a valid embedded object. e. Persecutory delusions
In a group of 100 patients the symptoms and their frequency of occurrence is provided:
Neuropsychiatric & Behavioral symptoms: Sleepwake cycle disturbance 97%, Perceptual disturbances
and hallucinations 50% , Delusions 31%, Lability of affect 53%, Language 57%, Thought process
abnormalities 54%, Motor agitation 62% & Motor retardation 62 %Cognitive - Orientation 76 ,
Attention 97, Short-term memory 88 , Long-term memory 89 , Visuospatial ability 87% Meagher DJ et
al. Phenomenology of delirium. Assessment of 100 adult cases using standardised measures. The
British Journal of Psychiatry (2007) 190: 135-141.
Incorrect
Marks for this submission: 0/1.
Question 62
Marks: 0/1
What is mode of inheritance of PICK'S disease
Choose one answer.
Error! Not a valid embedded object. a. X linked dominant
Error! Not a valid embedded object. b. X linked recessive
Error! Not a valid embedded object. c. Mitochondrial inheritance
Error! Not a valid embedded object. d. Autosomal recessive
Error! Not a valid embedded object. e. Autosomal dominant
There is at present no solid proof of an autosomal dominant inheritance in the majority of studies of
Picks disease. A linkage to chromosome 17q2122 has been found in 13 families with an autosomal
dominantly inherited Frontotemporal dementia with with parkinsonism (FTDP-17). The FTDP-17 locus
has been mapped to a region where the tau gene also lies. Pick's disease is charecterized by three-repeat
tau, whereas four-repeat tau is charecteristic of progressive supranuclear palsy and corticobasal
degeneration. New Oxford Textbook of Psychiatry, Volume 1, 2000, p 402 & Shorter Oxford Textbook
of Psychiatry, 5th Edn, 2006, pp 342-343.
Incorrect
Marks for this submission: 0/1.
Question 63
Marks: 0/1
What proportion of sexual offenders have no prior history of criminal offence ?
Choose one answer.
Error! Not a valid embedded object. a. 50%
Error! Not a valid embedded object. b. 20%
Error! Not a valid embedded object. c. 80%
Error! Not a valid embedded object. d. 5%
Error! Not a valid embedded object. e. 10%
A home office report has found that 79% of those serving their sentence for an offence against a child
and 75% whose victim had been an adult had no previous conviction for a sexual offence. Of those who
had at least one prior sexual conviction, 42% of both groups had more than one. A third of those
imprisoned for an offence against a child outside their own family had a history of known sexual
offending, compared with only 14% of those who had victimised solely family members. Click here for
reference
Incorrect
Marks for this submission: 0/1.
Question 64
Marks: 0/1
A 29 year old man who is on a Quetiapine 600 mg has QTc 500ms + T wave morphology on routine
ECG. What woud be the next step of management?
Choose one answer.
Error! Not a valid embedded
a. Repeat ECG after 2 weeks
object.
Error! Not a valid embedded b. Reduce dose of quetiapine and repeat ECG with an option of
object. referring to cardiology.
Error! Not a valid embedded
c. No intervention so long as the patient is asymptomatic.
object.
Error! Not a valid embedded d. Cross-taper in order to start risperidone and repeat ECG in 2 weeks.
object.
Error! Not a valid embedded
e. Stop quetiapine and then add risperidone.
object.
Antipsychotics are known to block cardiac potassium channels, leading to prolongation of QT intervals,
which can cause ventricular arrhythmias and sudden cardiac death. QTc of >440ms in men and >470
ms in women is considered to be significant. QTc of more than 500ms is associated with definite risk of
arrhythmia and hence the causative drug should be stopped and switched to a drug of lower effect,
preferably Aripripizole. Maudsley Prescribing Guidelines, 10th Edn.
Incorrect
Marks for this submission: 0/1.
Question 65
Marks: 0/1
A young woman comes to see you one month after the death of her sister. She describes low mood,
poor sleep and that she is hearing her sister's voice. What is the most likely diagnosis?
Choose one answer.
Error! Not a valid embedded object. a. Depressive Episode
Error! Not a valid embedded object. b. Stage III of greaf reaction
Error! Not a valid embedded object. c. Abnormal grief
Error! Not a valid embedded object. d. Stage II of grief reaction
Error! Not a valid embedded object. e. Stage I of grief reaction
Grief is a continuous process but for clarity is described in 3 stages. Stage I last from hours to days and
denial is common. Stage II last from weeks to about 6 months and can have sadness, poor sleep,
diminished apetite, illusions, hearing the dead persons voice and social withdrawal (all symptoms that
could be seen in a depression). In stage III the symptoms resolve. Grief is considered abnormal if it is
unusually intense, prolonged, delayed, inhibited or distorted. Shorter Oxford Textbook of Psychiatry.
5th Edn. 2006. pp 168-169.
Incorrect
Marks for this submission: 0/1.
Question 66
Marks: 0/1
Which one of the following would be the most commonly seen symptom in velo-cardio facial
syndrome?
Choose one answer.
Error! Not a valid embedded object. a. Severe MR
Error! Not a valid embedded object. b. Moderate to severe LD
Error! Not a valid embedded object. c. Specific learning disorders
Error! Not a valid embedded object. d. Mild to Moderate LD
Error! Not a valid embedded object. e. No LD
LD is usually mild to moderate in VCFS, but in some there no Learning disability, rather specific
learning disorders. Companion to Psychiatric Studies, 8TH Edn, 2010, p 570.
Incorrect
Marks for this submission: 0/1.
Question 67
Marks: 0/1
A man has stopped drinking alcohol for about three months having previously been dependent. Which
medication will be best to help keep him abstinent?
Choose one answer.
Error! Not a valid embedded object. a. Clonidine
Error! Not a valid embedded object. b. Methadone
Error! Not a valid embedded object. c. SSRIs
Error! Not a valid embedded object. d. Disulfiram
Error! Not a valid embedded object. e. Acamprosate
Acamprosate and naltrexone can be used to improve abstinence rates (total and cumulative, reduced
days drinking, greater time to relapse, improved treatment retention and craving). There is no consistent
evidence to suggest which types of patient will respond. We recommend that acamprosate and
naltrexone be considered as treatment options for patients attempting to maintain abstinence from
alcohol. Disulfiram is also effective if intake is supervised. Disulfiram can be offered as a treatment
option for patients who intend to maintain abstinence, and for whom there are no contraindications.
Acamprosate is preferred as Disulfiram intake needs to be supervised. Naltrexone is not licenced in
alcohol dependence in UK. Lingford-Hughes, Evidence-based guidelines for the pharmacological
management of substance misuse, addiction and comorbidity: recommendations from the British
Association for Psychopharmacology. Journal of Psychopharmacology. 18(3): 2004, pp 293335
Incorrect
Marks for this submission: 0/1.
Question 68
Marks: 0/1
A 20 year old man presents to the A&E with Phencyclidine intoxication. The signs / symptoms that he
can present with include
Choose one answer.
Error! Not a valid embedded object. a. Ataxia
Error! Not a valid embedded object. b. Simplex hallucinations
Error! Not a valid embedded object. c. Paranoid ideas
Error! Not a valid embedded object. d. Hypotonia
Error! Not a valid embedded object. e. Increased pain sensitivity
The criteria for PCP intoxication -A. Recent use of phencyclidine (or a related substance). B. Clinically
significant maladaptive behavioral changes (e.g. belligerence, assaultativeness, impulsiveness,
unpredictability, psychomotor agitation, impaired judgment, or impaired social or occupational
functioning) that developed during, or shortly after, phencyclidine use. C. Within an hour (less when
smoked, "snorted," or used intravenously), two (or more) of the following signs: vertical or horizontal
nystagmus, hypertension or tachycardia, numbness or diminished responsiveness to pain, ataxia,
dysarthria, muscle rigidity, seizures or coma & hyperacusis DSM IV. APA 1994
Incorrect
Marks for this submission: 0/1.
Question 69
Marks: 0/1
A middle aged man is suffering from depression and has been prescribed fluoxetine by gp, which has
made some improvement in his mood, but having problems with sleep. Which antidepressant would
you prescribe?
Choose one answer.
Error! Not a valid embedded object. a. Mirtrazepine
Error! Not a valid embedded object. b. Trazodone
Error! Not a valid embedded object. c. Sertraline
Error! Not a valid embedded object. d. Paroxetine
Error! Not a valid embedded object. e. Amitriptyline
Mirtrazepine is the drug of choice in this case because of its sedative action and is an effective
alternative to SSRI's as II line Rx in depression Nice guidance for depression 2009
Incorrect
Marks for this submission: 0/1.
Question 70
Marks: 0/1
The most appropriate measure to explain about treatment response of a medication to a patient would
be
Choose one answer.
Error! Not a valid embedded object. a. Odds ratio
Error! Not a valid embedded object. b. Number needed to treat
Error! Not a valid embedded object. c. Effectiveness
Error! Not a valid embedded object. d. Number needed to harm
Error! Not a valid embedded object. e. Efficacy
The NNT is a useful measure of treatment response and is the number of patients that need to be treated
for one to benefit compared with a control. Click here for reference
Incorrect
Marks for this submission: 0/1.
Question 71
Marks: 0/1
The rate of relapse of mood disorder in the post natal period in a woman with a preexisting diagnosis of
Bipolar disorder is
Choose one answer.
Error! Not a valid embedded object. a. 50%
Error! Not a valid embedded object. b. 30%
Error! Not a valid embedded object. c. 20%
Error! Not a valid embedded object. d. 80%
Error! Not a valid embedded object. e. 10%
Pre-existing BPAD is one of the greatest risk factors for puerperal psychosis .Recent studies estimate
that over 25-50% of women with bipolar disorder will experience relapsae in the first 6 months of the
post natal period if not taking mood stabilizers. Companion to psychiatric studies, 8 edn,p-790.
Incorrect
Marks for this submission: 0/1.
Question 72
Marks: 0/1
Regarding Aggression in ADHD, which of the following is not correct
Choose one answer.
Error! Not a valid embedded
a. It is hereditary
object.
Error! Not a valid embedded
b. Dggression is due to adverse intrafamily relationships
object.
Error! Not a valid embedded
c. 75% of children with ADHD have aggression
object.
Error! Not a valid embedded
d. Aggression is not a core feature of ADHD
object.
Error! Not a valid embedded e. Aggression can be a response to reactions from school staff &
object. peers.,
About 75% of children with ADHD show behavioral symptoms of aggression and deviance fairly
conssitently.These behaviours are generally associated with adverse family dynamics, school situations
where school personal show adverse reactions to behaviour characteristics of ADHD Kaplan &
Saddock,10 edn,p-1208, Synopsis of psychiatry
Incorrect
Marks for this submission: 0/1.
Question 73
Marks: 0/1
The kappa is used to measure
Choose one answer.
Error! Not a valid embedded object. a. Internal consistency
Error! Not a valid embedded object. b. Test-retest reliability
Error! Not a valid embedded object. c. Intra rater relaibility
Error! Not a valid embedded object. d. Accuracy of a test
Error! Not a valid embedded object. e. Inter-rater reliability
Cohen's kappa coefficient is a statistical measure of inter-rater agreement or inter-annotator
agreement[1] for qualitative (categorical) items. It is generally thought to be a more robust measure
than simple percent agreement calculation since ? takes into account the agreement occurring by
chance. Companion to Psychiatric Studies, 8th edn, pg 159.
Incorrect
Marks for this submission: 0/1.
Question 74
Marks: 0/1
The last stage according to Prochaska and Diclemente model of change of behavior is
Choose one answer.
Error! Not a valid embedded object. a. Preparation
Error! Not a valid embedded object. b. Maintenance
Error! Not a valid embedded object. c. Action
Error! Not a valid embedded object. d. Contemplation
Error! Not a valid embedded object. e. Precontemplation
In the last stage (maintenance stage) the new behaviors such as abstinence are perpetuated. Zernig et
al.., Handbook of Alcoholism, 2000, pp 99-100.
Incorrect
Marks for this submission: 0/1.
Question 75
Marks: 0/1
A 15 year old boy presents with a moderate depressive illness. He is otherwise fit and intelligent and
does not have suicidal ideas or plans. The treatment of choice for this boy would be
Choose one answer.
Error! Not a valid embedded object. a. SSRI
Error! Not a valid embedded object. b. CBT & SSRI
Error! Not a valid embedded object. c. Cognitive behaviour therapy
Error! Not a valid embedded object. d. Tricyclic antidepressants
Error! Not a valid embedded object. e. Dialectical behaviour therapy
Major controversies about prescribing of antidepressants in children and adolescents. TCAs,
Venlafaxine, and Paroxetine should not be used, and limits on use of SSRIs NICE recommends: Mild
depression- Antidepressant medication should not be used for the initial treatment of children and
young people with mild depression. Moderate to severe depression - Children and young people with
moderate to severe depression should be offered, as a first-line treatment, a specific psychological
therapy (individual cognitive behavioral therapy [CBT], interpersonal therapy or shorter-term family
therapy; it is suggested that this should be of at least 3 months' duration). Antidepressant medication
should not be offered to a child or young person with moderate to severe depression except in
combination with a concurrent psychological therapy. Specific arrangements must be made for careful
monitoring of adverse drug reactions, as well as for reviewing mental state and general progress; for
example, weekly contact with the child or young person and their parent's) or carer's) for the first 4
weeks of treatment. In the event that psychological therapies are declined, medication may still be
given, but as the young person will not be reviewed at psychological therapy sessions, the prescribing
doctor should closely monitor the child or young person's progress on a regular basis and focus
particularly on emergent adverse drug reactions. There is limited evidence for the treatment of
depression esp. antidepressants in children - except for fluoxetine December 2003 - the CSM advised
for fluoxetine only - the balance of risks and benefits favorable Nice guidance 2006
Incorrect
Marks for this submission: 0/1.
Question 76
Marks: 0/1
You are counseling a mother who has one child with autism and is expecting her second. She wishes to
know the risk of autism in this second child. You say
Choose one answer.
Error! Not a valid embedded object. a. 11-20%
Error! Not a valid embedded object. b. 41-50%
Error! Not a valid embedded object. c. 1-10%
Error! Not a valid embedded object. d. 21-30%
Error! Not a valid embedded object. e. 31-40%
Studies have shown that the risk of autism in siblings is 2-3 %,which 50-100 fold increase over the
population rates. New Oxford textbook of psychiatry,Vol2,p-1726
Incorrect
Marks for this submission: 0/1.
Question 77
Marks: 0/1
The following is true with regards to dementia drugs:
Choose one answer.
Error! Not a valid embedded
a. Memantine is an NMDA-agonist
object.
Error! Not a valid embedded
b. Donepezil is metabolised by CYP2D6, and CYP3A4
object.
Error! Not a valid embedded c. Donepezil is more likely to cause GI side effects
object.
Error! Not a valid embedded d. Rivastigmine blocks the actions of acetylcholine esterase only
object.
Error! Not a valid embedded
e. Rivastigmine can be given once daily
object.
Donepezil undergoes extensive metabolism via both CYP2D6 and 3A4 isozymes. Donepezil 's
favourable side effect profile appears to corelate with its lack of inhibition of cholineesterases in the GI
tract. Synopsis of Psychiatry, 10th edn, pg 1034
Incorrect
Marks for this submission: 0/1.
Question 78
Marks: 0/1
A measure of the odds of an event happening in one group compared to the odds of the same event
happening in another group is termed as
Choose one answer.
Error! Not a valid embedded object. a. Odds
Error! Not a valid embedded object. b. Risk ratio
Error! Not a valid embedded object. c. Hazard ratio
Error! Not a valid embedded object. d. Odds ratio
Error! Not a valid embedded object. e. Relative risk
In a case control study one only calculate the odds ratio, i.e, the odds that those with the disease have
been exposed to the risk factor to those without the disease exposed to the risk factor. David Bowers,
Medical Statistics from Scratch, Wiley, 2002, p 92
Incorrect
Marks for this submission: 0/1.
Question 79
Marks: 0/1
The following is true regarding Fragile X syndrome
Choose one answer.
Error! Not a valid embedded
a. Inherited in autosomal dominant pattern.
object.
Error! Not a valid embedded b. Consistent with a finding of 26 repeats of the CAG sequence.
object.
Error! Not a valid embedded c. Length of sequence correlates with the amount of cognitive
object. decline.
Error! Not a valid embedded
d. Almost all people have mild to moderate learning difficulty
object.
Error! Not a valid embedded
e. There is no progression in terms of IQ with increasing age.
object.
Fragile X syndrome is a X linked dominant condition that causes a range of developmental problems
including learning disabilities and cognitive impairment. Usually, males are more severely affected by
this disorder than females. Mutations in the FMR1 gene cause fragile X syndrome. The FMR1 gene
provides instructions for making a protein called fragile X mental retardation 1 protein, whose function
is not fully understood. Nearly all cases of fragile X syndrome are caused by a mutation in which a
DNA segment, known as the CGG triplet repeat, is expanded within the FMR1 gene. Normally, this
DNA segment is repeated from 5 to about 40 times. In people with fragile X syndrome, however, the
CGG segment is repeated more than 200 times. Males and females with 55 to 200 repeats of the CGG
segment are said to have an FMR1 gene premutation. Most people with a premutation are intellectually
normal. Click here for reference
Incorrect
Marks for this submission: 0/1.
Question 80
Marks: 0/1
Which of the following is most suitable instrument for differential diagnosis with particular reference
to dementia in the elderly?
Choose one answer.
Error! Not a valid embedded object. a. MMSE
Error! Not a valid embedded object. b. CAMDEX
Error! Not a valid embedded object. c. SCAN
Error! Not a valid embedded object. d. CSDD
Error! Not a valid embedded object. e. SCID
Cambridge Mental Disorder of the Elderly Examination (CAMDEX) is a standardised instrument for
the diagnosis of mental disorder in the elderly with special reference to the early detection of dementia.
Mini Mental State Examination (MMSE) is a screening instrument for cognitive impairment, Cornell
Scale for Depression in Dementia (CSDD) is for detection of depression in those with dementia.
Structured Clinical Interview for DSM-IV (SCID) may not be suitable in this age group. Roth M et al..,
CAMDEX. A standardised instrument for the diagnosis of mental disorder in the elderly with special
reference to the early detection of dementia. The British Journal of Psychiatry, 149, 1986, pp 698-709.
Incorrect
Marks for this submission: 0/1.
Question 81
Marks: 0/1
The antidepressant of choice used in the treatment of moderate depression in a patient who has
Myocardial infarction is
Choose one answer.
Error! Not a valid embedded object. a. Citalopram
Error! Not a valid embedded object. b. Trazadone
Error! Not a valid embedded object. c. Fluoxetine
Error! Not a valid embedded object. d. Amitriptyline
Error! Not a valid embedded object. e. Sertraline
When initiating antidepressant treatment in patients with recent MI or unstable angina, Sertraline is the
treatment of choice and has the best evidence Nice guidelines for depression,2004 (Though this is not
mentioned in NICE guidance 2009), Maudsley Prescribing guidelines, 2009.
Incorrect
Marks for this submission: 0/1.
Question 82
Marks: 0/1
The following statements are valid in the use Automatism as a defense for murder
Choose one answer.
Error! Not a valid embedded a. The person can partially recollect the act on the following morning
object.
Error! Not a valid embedded
b. The person recollects the act as if it had occurred in his dream
object.
Error! Not a valid embedded
c. Murder should have ideally occurred within 2 hours of sleep
object.
Error! Not a valid embedded d. The person can clearly recollect the act in the morning after the
object. murder
Error! Not a valid embedded e. The person wakes up in the middle of the night screaming and in a
object. panic during the act
Automatism can be used as defence for a criminal charge. There are two types of automatisms: sane
and insane. A sane automatism must arise from an external factor (such as a head injury) whereas an
insane automatism arises from some 'disease of the mind', including mental illness, somnambulism
epilepsy etc. Characteristics of automatism include:involuntary (subject has no control over
it),inappropriate to the circumstances, lacking in judgment, out of character may be complex,
purposeful, co-ordinated, and directed afterwards no recollection, or partial and confused memory if
organic, there must be corresponding disturbance of brain function if psychogenic, behavior is
appropriate to the psychopathology there is a clear sensorium but complete or severe amnesia Hart, H.
L. A. "Punishment and Responsibility: Essays in the Philosophy of Law". (1968) Oxford University
Press. ISBN 0-19-825181-5
Incorrect
Marks for this submission: 0/1.
Question 83
Marks: 0/1
A mother has recently given birth to a baby, but she had been taking paroxetine throughout her
pregnancy. Which of the following is more liekly to be seen in the baby?
Choose one answer.
Error! Not a valid embedded object. a. Hypotonia
Error! Not a valid embedded object. b. VSD
Error! Not a valid embedded object. c. Neonatal bleeding
Error! Not a valid embedded object. d. Neural tube defects
Error! Not a valid embedded object. e. Ebsteins anomaly
A recent meta-analysis has reported an increased prevalence of combined cardiac defects with first
trimester paroxetine use, particularly Ventricular septal defects. There are other studies which have
provided conflicting results, and hence this paper should be viewed with caution, given its limitations.
First trimester paroxetine use and the prevalence of congenital, specifically cardiac, defects: a meta-
analysis of epidemiological studies. Birth Defects Res A Clin Mol Teratol. 2010;88(3):159-70
Incorrect
Marks for this submission: 0/1.
Question 84
Marks: 0/1
Which rating scale would you use to assess the response to antidepressants in a woman who has
recently given birth and is depressed?
Choose one answer.
Error! Not a valid embedded object. a. EPDS
Error! Not a valid embedded object. b. HAMD.
Error! Not a valid embedded object. c. YBOCS
Error! Not a valid embedded object. d. PANNS
Error! Not a valid embedded object. e. MADRS
EPDS is a scale commonly used to screen for post natal natal depression but has been used in studies to
monitor the response to antidepressants as well. Click here for reference
Incorrect
Marks for this submission: 0/1.
Question 85
Marks: 0/1
The following is true regarding offending in learning disabilities
Choose one answer.
Error! Not a valid embedded a. The majority of those convicted have mild to moderate LD
object.
Error! Not a valid embedded b. Conviction for arson leads to a fixed prison sentence
object.
Error! Not a valid embedded
c. Offending is more likely in severe LD
object.
Error! Not a valid embedded
d. Violent sexual offending are the most common
object.
Error! Not a valid embedded
e. Fire setting is the most common offence
object.
Crimes of personal violence are low in frequency, sex offences and arson are over represented in LD
population Hodgins Crim Justice and Behaviour 24 432-454
Incorrect
Marks for this submission: 0/1.
Question 86
Marks: 0/1
A curve with plot of sensitivity versus 1-specificity on either axis is
Choose one answer.
Error! Not a valid embedded object. a. Normal curve
Error! Not a valid embedded object. b. Galbraith plot
Error! Not a valid embedded object. c. Lorenz curves
Error! Not a valid embedded object. d. ROC curve
Error! Not a valid embedded object. e. Nomogram
The receiver operating characteristic (ROC) curve is a plot of the true positive rate (sensitivity) of a test
versus its false-positive rate (1 - specificity ) for all possible cut points. Receiver Operating
Characteristic Curves and Their Use in Radiology, Nancy A. Obuchowski,Statistical Concepts Series,
Radiology 2003;229:3-8.
Incorrect
Marks for this submission: 0/1.
Question 87
Marks: 0/1
A young male, seen by you, admits to using heroin twice per week, to settle him, due to the death of his
child a few months ago due to sudden infant death syndrome. He currently is checking doors 3-4 times
to keep his family safe, if he resists checking, he becomes irritable and anxious. Which of the following
psychological interventions would benefit him?
Choose one answer.
Error! Not a valid embedded object. a. Cognitive restructuring
Error! Not a valid embedded object. b. Relaxation training
Error! Not a valid embedded object. c. Dairy keeping
Error! Not a valid embedded object. d. Activity scheduling
Error! Not a valid embedded object. e. Exposure and response prevention
It appears that in this scenario a recent significant life event has led to an anxiety disorder, with
obsessive compulsive features, and possible opiate abuse to deal with the anxiety. The Obsessive
compulsive symptoms and opiate misuse appear to be related to the traumatic event of death of his
child, and hence Cognitive restructing would be beneficial to him rather than Exposure and response
prevention.
Incorrect
Marks for this submission: 0/1.
Question 88
Marks: 0/1
Which correlation co-efficient demonstrates an inverse relationship between two variables?
Choose one answer.
a. 1
Error! Not a valid embedded object. b. -1
Error! Not a valid embedded object. c. 0.5
Error! Not a valid embedded object. d. -0.5
Error! Not a valid embedded object. e. 0
The strength or magnitude of the relationship between the two variables is expressed by a statistic
known as the CORRELATION COEFFICIENT, which varies from -1 (perfect negative relationship or
inverse correlation ), through 0 (no relationship) to +1 (perfect positive relationship). Intuitive
biostatistics, Harvey Motulsky, pg 185
Incorrect
Marks for this submission: 0/1.
Question 89
Marks: 0/1
A 27 year old man presents with diaphoresis, agitation, tachycardia and dilated pupils. Which illicit
substance is he likely to be intoxicated with?
Choose one answer.
Error! Not a valid embedded object. a. Amphetamine intoxication
Error! Not a valid embedded object. b. Cannabis intoxication
Error! Not a valid embedded object. c. Heroin intoxication
Error! Not a valid embedded object. d. Barbiturate intoxication
Error! Not a valid embedded object. e. Benzodiazepine intoxication
Amphetamines are probably the most frequently used stimulant in the UK, about 22% of 16-29 year
olds have used amphetamines at least once and 2% in the last 1 year. The acute effects of drug use
include over-talkativeness, overactivity, insomnia, mydriasis, tachycardia and hypertension. With
higher doses there can be cardiac arrythmias, circulatory collapse, seizures and coma. Acute adverse
psychological effects include dysphoria, irritability, insomnia and confusion. Shorter Oxford Textbook
of Psychiatry. 5th Edn. 2006. pp 463-464
Incorrect
Marks for this submission: 0/1.
Question 90
Marks: 0/1
Which of the following is a false statement regarding functional enuresis in children
Choose one answer.
Error! Not a valid embedded object. a. Cannot occur as a complication of a psychiatric illness
Error! Not a valid embedded object. b. Parental attitude has no aetiological role
Error! Not a valid embedded object. c. Male to female ratio of 3:1
Error! Not a valid embedded object. d. Maybe nocturnal or diurnal in nature
Error! Not a valid embedded object. e. Part of Helman's triangle for predicting future violence
Besides genetic factors, rigid toilet training, negative or indifferent attitudes of parents and stressful
events leading to anxiety are etiological factors. Helmans triangle includes Bedwetting, firesetting and
cruelty to animals. Nocturnal enuresis in the adolescent: a neglected problem. British Journal of
Urology.
Incorrect
Marks for this submission: 0/1.
Question 91
Marks: 0/1
A woman with Bipolar affective disorder is breastfeeding and would like to know about the safest
medication to use. Your answer would be?
Choose one answer.
Error! Not a valid embedded object. a. Lithium
Error! Not a valid embedded object. b. Lorazepam
Error! Not a valid embedded object. c. Lamotrigine
Error! Not a valid embedded object. d. Valproate
Error! Not a valid embedded object. e. Carbamazepine
If taking lithium, benzodiazepines, or lamotrigine the woman should be advised not to breastfeed. If
she wishes to continue breast feeding then should be offered a prophylactic agent that can be used when
breastfeeding an antipsychotic should be the first choice (but not clozapine). If a mood stabiliser is to
be used then Valproate and Carbamzepine are the best options, with Valproate being the first choice.
NICE guidelines, The management of bipolar disorder in adults, children and adolescents, in primary
and secondary care. 2006
Incorrect
Marks for this submission: 0/1.
Question 92
Marks: 0/1
An elderly gentleman admitted under your care has abnormal blood tests showing high calcium levels,
which on the following can effect these results?
Choose one answer.
Error! Not a valid embedded object. a. Increased PTH
Error! Not a valid embedded object. b. High Potassium
Error! Not a valid embedded object. c. Reduced sodium
Error! Not a valid embedded object. d. Reduced Carbonate
Error! Not a valid embedded object. e. Reduced urea
Primary hyperparathyroidism is a disorder of calcium homeostasis that occurs most commonly in older
adults. Resultant hypercalcemia may be accompanied by neuropsychiatric symptoms, ranging from
mild depression and cognitive changes to extreme agitation and psychosis. A high level of serum
calcium, in combination with elevated levels of parathyroid hormone, is strong evidence for the
diagnosis, which is strengthened by a 99mTc-sestamibi scan suggesting an adenoma, and confirmed by
surgical neck exploration. New Onset of Neuropsychiatric Symptoms in the Elderly: Possible Primary
Hyperparathyroidism ,Lea C. Watson, M.D., and Christine E. Marx, M.A., M.D. Psychosomatics
43:413-417, October 2002
Incorrect
Marks for this submission: 0/1.
Question 93
Marks: 0/1
Massed negative treatment is used in which of the following conditions
Choose one answer.
a. ADHD
b. Enuresis
c. Tics in tourettes disorder
d. Conduct disorder
e. Emotional disorders
Massed negative practice (MNP) is a proposed treatment for the tics of Tourette syndrome in which the
individual with Tourette's "practices" tics continuously until a conditioned level of fatigue is reached. It
is based upon the Hullian learning theory, which holds that tics are "maladaptive habits that are
strengthened by repetition and can be replaced by the strengthening of more adaptive habits (i.e., not
having tics) Woods DW, Himle MB, Conelea CA. Behavior therapy: other interventions for tic
disorders. Adv Neurol. 2006;99:234-40.
Incorrect
Marks for this submission: 0/1.
Question 94
Marks: 0/1
The current best evidence for the treatment of dementia associated with Parkinson's disease is
Choose one answer.
a. Donepizil
b. Clozapine
c. Memantine
d. Galantamine
e. Rivastigmine
A recent Cochrane review found the most evidence for Rivastigmine with 15% of patients showing
some improvement in cognitive functioning and activities of daily living Maidment I et al.,
Cholinestrase inhibitors for Parkinsons disease dementia. Cochrane Database Syst Rev 2006.
Incorrect
Marks for this submission: 0/1.
Question 95
Marks: 0/1
The following is true with regards to women with postnatal depression
Choose one answer.
a. More common with assisted deliveries
b. Occurs 2 6 days within giving birth
c. More common in older women
d. Past history of depression increases the risk of post natal depression
e. Associated with lower social class
The risk factors for post natal depression include past history of depression,psychological problems
during pregnancy,poor social support & marital relationship,recent life adverse events and severe baby
blues I.Brockington. Postpartum psychiatric disorders. The Lancet,2004 363(9405): 303-310
Incorrect
Marks for this submission: 0/1.
Question 96
Marks: 0/1
Staccato speech is not a feature of
Choose one answer.
a. Multiple sclerosis
b. Amyotrophic lateral sclerosis
c. Neoplastic lesions of cerebellum
d. Charcots triad
e. Vascular lesions of cerebellum
In Staccato speech the individual syllables and words are produced in a truncated or scanning fashion
with absence of normal modulation of pitch seen in cerebellar dysarthria that includes all the causes
mentioned. Charcots triad is a combination od scanning dysarthria,nystagmus and intention tremor.
Differential diagnosis in internal medicine: from symptom to diagnosis By Walter Siegenthaler,2007,p-
102
Incorrect
Marks for this submission: 0/1.
Question 97
Marks: 0/1
Which of the following are crucial to make a diagnosis of NMS according to DSM-IV TR?
Choose one answer.
a. Autonomic instability and fever
b. Fever and rigidity
c. Rigidity and raised CK
d. Autonomic instability and rigidity
e. Fever and raised CK
The occurance of severe muscle rigidity and elevated temperature associated with the use of
neuroleptic medication as well as 2 or more of the following; Diaphoresis, dysphagia, tremor,
incontinence, changes in level of consciousness ranging from confusion to coma, mutism, tachycardia,
elevated or labile blood pressure, leukocytosis, laboratory evidence of muscle injury. DSM IV-TR
Manual, Published by American Psychiatry Association.
Incorrect
Marks for this submission: 0/1.
Question 98
Marks: 0/1
You want to find out the opinions of service users about some aspect of their care, which of the
following are best to use for this purpose?
Choose one answer.
a. Focus groups
b. Direct observation
c. Structured interviews
d. Indepth interviews
e. Semi-structured interviews
Data collection in Qualitative studies could be done using three methods: 1. Participant observation are
methods by which an individual or individuals gather firsthand data on programs, processes, or
behaviors being studied. Observations can be useful during both the formative and summative phases
of evaluation. They provide evaluators with an opportunity to collect data on a wide range of behaviors,
to capture a great variety of interactions, and to openly explore the evaluation topic. 2. Interviews:
nterviews provide very different data from observations: they allow the capture the perspectives of
participants, staff, and others associated with a project. The use of interviews as a data collection
method begins with the assumption that the participants perspectives are meaningful, knowable, and
able to be made explicit, and that their perspectives affect the success of the project. Two types of
interviews are structured interviews and indepth interviews. In-depth interviews are optimal for
collecting data on individualspersonal histories, per- spectives, and experiences, particularly when
sensitive topics are being explored. 3. Focus groups combine elements of both interviewing and
participant observation. The focus group session is, indeed, an interview not a discussion group,
problem-solving session, or decision-making group. Focus groups are useful for identifying and
defining problems in project implementation; identifying project strengths, weaknesses, and
recommendations, assisting with interpretation of quantitative findings; obtaining perceptions of project
outcomes and impacts; and generating new ideas. Click here for reference
Incorrect
Marks for this submission: 0/1.
Question 99
Marks: 0/1
For questions 97– 100, see the precis below:

Charman T et al., Efficacy of three screening instruments in the identification of autistic-


spectrum disorders. The British Journal of Psychiatry (2007) 191: 554-559.

The authors‘ aim was to compare the Social Communication Questionnaire (SCQ), the Social
Responsiveness Scale (SRS) and the Children's Communication Checklist (CCC) in screening for
autistic-spectrum disorder. The findings are published in the figure below.
What does the x axis represent?
Choose one answer.
a. True positive rate
b. True negative rate
c. False negative rate
d. None of the options
e. False positive rate
A receiver operating characteristic plot is obtained by calculating the sensitivity and specificity of
every observed data value and plotting sensitivity against 1 - specificity. In the ROC curves, the X axis
usually represents 1-specificity, which is the false positive rate. The Y axis represents sensitivity which
is the true positive rate. Altman & Bland. Statistics Notes: Diagnostic tests 3: receiver operating
characteristic plots BMJ 1994; 309 : 188
Incorrect
Marks for this submission: 0/1.
Question 100
Marks: 0/1
What does the Y axis represent?
Choose one answer.
a. True positive rate
b. None of the options
c. False negative rate
d. False positive rate
e. True negative rate
A receiver operating characteristic plot is obtained by calculating the sensitivity and specificity of
every observed data value and plotting sensitivity against 1 - specificity. In the ROC curves, the X axis
usually represents 1-specificity, which is the false positive rate. The Y axis represents sensitivity which
is the true positive rate. Altman & Bland. Statistics Notes: Diagnostic tests 3: receiver operating
characteristic plots BMJ 1994; 309 : 188
Incorrect
Marks for this submission: 0/1.
Question 101
Marks: 0/1
Which study would provide for such a result?
Choose one answer.
a. Cross sectional study
b. Correlational study
c. Cohort study
d. Case control study
e. Randomised controlled study
Cross section studies are the most appropriate study design when trying to answer questions concerning
the value of screening or diagnostic tests and prevalence of a disease (i.e., the number of subjects with a
disease at a given point of time).
Incorrect
Marks for this submission: 0/1.
Question 102
Marks: 0/1
Which of the following is the most effective for screening for autistic spectrum disorder?
Choose one answer.
a. CCC
b. SRS
c. None of the options
d. SCQ
e. Not enough information provided
A receiver operating characteristic plot is particularly useful when comparing two or more measures. A
test with a curve that lies wholly above the curve of another will be clearly better at diagnostic
accuracy. In this ROC, SCQ has a curve which is above the other two curves, hence is better. Altman &
Bland. Statistics Notes: Diagnostic tests 3: receiver operating characteristic plots BMJ 1994; 309 : 188
Incorrect
Marks for this submission: 0/1.
Question 103
Marks: 0/4
Addiction treatment

Options:
A. Acalmprosate
B. Antabuse
C. Anti psychotics
D. Benzodiazapines
E. Buprenorphine
F. Community detox
G. Motivational interviewing.
H. Naloxone
I. Naltrexone
J. SSRI

Choose the most appropriate option in the following scenarios. Each option may be used once or more
than once or not at all.

A 22 year old man with problem drinking has presented to GP with gastritis. He is unsure if he wants to
stop drinking but is willing to link drinking to his physical problems

A 37 year old an with alcohol dependence expresses desire to stop drinking, but has reported his
inability to do due withdrawals, paranoia and depression.

A 40 year old male, been off alcohol for 3 months what you would prescribe to maintain abstinence
Answers:

1. G. It appears this man is in the contemplation stage of change and may be helped by motivational
interviewing.
2. C, F. Alcohol induced psychotic disorder is associated with resolution of symptoms following
abstinence but may need antipsychotic medication, in addition to community detox.
3. A. Acamprosate is ideal treatment of choice in this person.

Incorrect
Marks for this submission: 0/4.
Question 104
Marks: 0/5
Cross sectional studies

A - 11%
B - 22%
C - 33%
D - 44%
E - 55%
F - 66%
G - 77%
H - 88%
I - 99%
J - 2%
K - 10%

A study involved screening terminally ill people for depression with the question Do you think you are
depressed?. A structured diagnostic interview using DSM IV also used to diagnose depression in the
same sample. The results of the study are in the table below. Calculate the below using the information
from the table. Each option may be used once, more than once or not at all.

Interview positive Interview negative


DSM positive 11 9

DSM negative 14 40

What is the probability that an individual with the disease will be interview positive?
What is the probability that an individual without the disease will be interview negative?
What proportion of people with a positive test actually have the disorder?
What proportion of people with a negative test will not have the disease?
What is the probability of having the disease if the test result is positive?
Answers

Please note that in the exam the table provided was contrary to the usual convention, which means
candidates who had learnt the formulae by rote would have had problems. If you do want to use
formulae, please ensure that the gold standard is on top and the screening test is on the left hand side of
the table. You can easily flip this table to get the desired table, which would look as below
DSM IV positive DSM IV negative
Interview positive 11 14
Interview negative 9 40

1. E - This is sensitivity = a / (a+c) 11/20 = 55%

2. G - This is specificity = d / (b+d) 40 / 54 =74%

3. D - This is positive predictive value = a / (a+b) 11/25 = 44%

4. H - Negative predictive value = d / (c+d) = 40 / 49 = 81%

5. J - Likelihood ratio = Sensitivity / (1-specificity) = 0.55 /(1-0.74) 2.11

Incorrect
Marks for this submission: 0/5.
Question 105
Marks: 0/5
Developmental Syndromes

A - Angelmans syndrome
B - Downs syndrome
C - Fragile X Syndrome
D - Huntingtons syndrome
E - Lesch Nyhan Syndrome
F - Prader-Willi syndrome
G - Retts syndrome.
H - Robinson-Turbey syndrome
I - Williams syndrome

Choose the most appropriate clinical syndrome in the following scenarios. Each option may be selected
once, more than once or not all.
The condition associated with maternally derived chromosome.
This condition occurs in females predominantly.
The clinical picture of hyperphagia, self-mutilation is associated with the condition.
The most common genetic cause of learning disability.
The disorder inherited through mutation in the HPRT gene.
Answers

1. A - Angelman syndrome is usually due to a micro deletion on chromosome 15 at the point 15q11-13
but on a maternally derived chromosome but the same in a paternally derived chromosome is
associated with Prader Willi syndrome.

2. G - Retts syndrome is a x linked dominant genetic disorder that occurs predominantly in females.

3. F - Hyperphagia and self injury in the form of skin picking is a feature of Prader Willi syndrome.

4. B - Downs syndrome is the most common cause of LD followed by Fragile X syndrome and
phenyketonuria.
5. E - Lesch Nyhan syndrome is a x linked recessive condition caused by a defect in the hypoxanthine
guanine phosphoribosyl transferase.

Incorrect
Marks for this submission: 0/5.
Question 106
Marks: 0/6
Errors & Measures in research-critical appraisal

A - 95th PERCENTILE.
B - 95% confidence interval
C - IQR
D - MEAN
E - MEDIAN
F - MODE
G - Power
H - P-value
I - Standard error of the mean
J - Standard deviation

Choose the most appropriate term from the list of options above. Each option may be used once, more
than once or not at all.
This measure includes 95% of values.
Uncertainty of the measured mean.
A range within which the true value of the study lies with some degree of certainty.
The probability of rejecting a null hypothesis when it is true.
A measure of dispersion (spread of data) of the study.
The ability of a test to reject a null hypothesis when it is false.
Answers

1. A - 95 percentile, a percentile (or centile) is the value of a variable below which a certain percent of
observations fall. So the 95th percentile is the value (or score) below which 95 percent of the
observations may be found.

2. I - The standard error of the mean (i.e., of using the sample mean as a method of estimating the
population mean) is the standard deviation of those sample means over all possible samples (of a given
size) drawn from the population.

3. B - 95% confidence interval. A confidence interval gives an estimated range of values which is likely
to include an unknown population parameter (p value, mean difference etc.), the estimated range being
calculated from a given set of sample data.

4. H - Type I error is defined as a null hypothesis (for example, no difference between groups) being
rejected, while it should be accepted. The probability of a type I error is equal to the significance level
or the P value.

5. J - Measures of central tendency describe the "average" of a data set that includes mean, median and
mode. Another important quality to measure is the "spread" of a data set that includes range, standard
deviation and variance.
6. G - . This refers to the power of the test, the ability not to make type 2 errors. Type 2 errors are made
when the null hypothesis is not rejected when it is actually false.

Incorrect
Marks for this submission: 0/6.
Question 107
Marks: 0/3
Gait disorders

A - Creudzfeldt Jacob disease


B - Huntingtons disease
C - Meiges syndrome
D - Metachromatic leucodystrophy
E - Myoclonus
F - Neuroacanthosis
G - Parkinsons disease
H - Progressive supranuclear palsy
I - Sydenham's chorea
J - Tourettes disease

For the following questions, choose the most appropriate diagnosis from above. Each option may be
used once, more than once or not at all.
A 62-year-old lady presents with difficulties due to frequent falls. On examination, she has a wide-
based shuffling gait with short steps, a vertical gaze palsy and micrographia.
A 15-year-old girl presents with a halting gait, difficulty writing, and uncoordinated movements of
her limbs. She had a history of fever and sore throat six months ago.
A 40-year-old man is brought by his wife, due to problems with their gait instability and loss of
memory. She mentions that he has become clumsy and has developed jerky movements of his
muscles.
Answers

1. H - Progressive supranuclear palsy (PSP), also known as Steele-Richardson-Olszewski syndrome, is


a neurodegenerative disease that affects cognition, eye movements, and posture. The exact cause is
unknown. The cardinal manifestations of PSP are supranuclear ophthalmoplegia; pseudobulbar palsy;
prominent neck dystonia; parkinsonism; behavioural, cognitive, and gait disturbances that cause
imbalance; and frequent falls.

2. I - Sydenham chorea (SD) is a neurological disorder of childhood resulting from infection via Group
A beta-hemolytic streptococcus, the bacterium that causes rheumatic fever. Rapid, irregular and aimless
involuntary movements of the arms and legs, trunk and facial muscles characterize Syndenham chorea.
It affects girls more often than boys and typically occurs between 5 and 15 years of age. Some children
will have a sore throat several weeks before the symptoms begin but the disorder can also strike up to 6
months after the fever or infection has cleared. Symptoms can appear gradually or all at once, and may
include uncoordinated movements, muscular weakness, stumbling and falling, slurred speech, difficulty
concentrating and writing and emotional instability. The symptoms of SD can vary from a halting gait
and slight grimacing to involuntary movements that are frequent and severe enough to be
incapacitating. The random, writhing movements of chorea are caused by an auto-immune reaction to
the bacterium that interferes with the normal function of a part of the brain (the basal ganglia) that
controls motor movements.

3. A - Creutzfeldt-Jakob disease (CJD) is the best known of a group of diseases called prion disease,
which affects a form of protein found in the central nervous system and causes dementia. Early
symptoms include minor lapses of memory, mood changes and loss of interest. Within weeks, an
infected person may complain of clumsiness and feeling muddled, become unsteady in walking, and
exhibit slow or slurred speech. The symptoms progress to jerky movements, shakiness, stiffness of
limbs, incontinence and the loss of the ability to move or speak. By this stage, the person is unlikely to
be aware of their surroundings or disabilities. People affected by CJD usually die within six months of
early symptoms - often from pneumonia. In a minority of patients, the disease may take two years to
run its course.

Incorrect
Marks for this submission: 0/3.
Question 108
Marks: 0/3
Group Therapy

A. Cohesiveness
B. Conditioning
C. Counter-dependence
D. Dependence
E. Fight-flight
F. Free floating discussion
G. Interpreting transference
H. Pairing
I. Universality
J. Vicarious learning

Lead in: Each option may be used once, more than once or not at all. From the options above, choose

Interpersonal ―Curative factors‖ in a group

Bion‘s factors which affect group therapy

Factors seen facilitating communication in a psychodynamic group

Answers

1. A, I

Yalom identified 11 curative factors in groups including cohesiveness (i.e. a sense of belongingness)
and universality (a sense that symptoms/problems are not specific to one group member, but shared by
many others) the other factors are: Installation of hope, imparting of information, interpersonal
learning, imitation of adaptive behavior, catharsis, corrective recapitulation of the family group,
altruism, guidance, existential factors, and social skills. (see also Introduction to Psychotherapy by
Bateman (former chief examiner), Brown and Pedder. (2000) Routledge pg 135-136)

2. D, E

Bion identified several factors that hinder group work including: dependence and fight-flight response
& pairing. A group holds one of three basic assumptions, which may differ from the beliefs of the
individual members.

The basic assumption of dependence arises from the group's anxious need to depend absolutely on
someone (usually the therapist) to protect the members, satisfy all their desires and solve their
problems. This corresponds to Klein‘s depressive position.

Collective belief in an enemy who can be dealt with only by attack or retreat forms the basic
assumption of fight-flight. This corresponds to Klein‘s paranoid-schizoid position.

The basic assumption of pairing defines the notion that some future event or person will come from
outside to solve all problems. For example, there may be hope for the pairing of two individual
members of the group that could lead to the ―birth‖ of a new solution for the group‘s problems. This
corresponds to Klein and Freud‘s Oedipal fantasy.

(see also Introduction to Psychotherapy by Bateman (former chief examiner), Brown and Pedder.
(2000) Routledge pg 124)

3. F, G.

Free-floating discussions (group-analytic equivalent of free-association) and Interpreting transference


(along with interpretation of defences and resistances, archaic and primordial experiences) are
important in the context of psychodynamic groups. Foulkes also listed ‗group-specific processes‘ as
important factors.

Incorrect
Marks for this submission: 0/3.
Question 109
Marks: 0/5
Male to female ratios

A - 1:1
B - 1:2
C - 1:5
D - 1:9
E - 1:10
F - 2:1
G - 3:1
H - 4:1
I - 5:1
J - 8:1

Choose the correct male to female ratio in the following conditions. Each option may be used once,
more than once or not at all.
Autism
ADHD
Bulimia Nervosa
Aspergers syndrome
Prepubertal depression
Answers

1. H - In autism the overall the male: female ratio was 4:1 but the ratio tends to be higher in those with
learning disability.

2. G - For ADHD the ratio of males to females in community samples is approximately 3:1.
3. D - The great majority of patients with bulimia nervosa are female and most are in their twenties,
80% have previous history of anorexia nervosa which has female:male ratio of 9:1.

4. I - There does appear to be a male predominance in the order of 9 to 1; similar sex ratios are
observed in autism not associated with mental retardation.

5. A - In contrast to earlier childhood, when psychiatric disorder (mood disorder) is more common
among boys, the adolescent period shows a shift towards an equal gender ratio in early adolescence
followed by a subsequent female preponderance in late adolescence and adult life.

Incorrect
Marks for this submission: 0/5.
Question 110
Marks: 0/3
Management of disruptive behaviour in children

A - Cognitive behaviour therapy


B - Family therapy
C - Methylphenidate
D - Mirtazepine
E - No Intervention
F - Parental skills training
G - Psychoeducation
H - Systemic therapy

Choose the treatment most suitable from the above options. Each option may be used once, more than
once or not at all.

A 4 year old child is repeatedly disrespectful and rude to teachers at nursery and hence banned from the
nursery

A 17 year old male who is aggressive towards his parents when they don‘t give him money, parents are
reluctantly considering an injunction order due to persistent aggression

An 8-year old boy who presents with over activity and poor attention span that is noticed by school
teachers and you diagnose him as having a hyperkinetic disorder

Answers

1. F - Oppositional defiant disorder (ODD) is characteristically seen in children below the age of 9 or
10 years. It is defined by the presence of markedly defiant, disobedient, provocative behaviour and by
the absence of more severe dissocial or aggressive acts that violate the law or the rights of others.
Positive parenting within the framework of reinforcement theory including provision of information,
attention and approval following pro-social action and withholding of these following inappropriate
actions helps reduce the problem behaviours.

2. B - Conduct disorders are characterized by a repetitive and persistent pattern of dissocial, aggressive,
or defiant conduct. Conduct disorder can be confined to the family context and this variety can be
treated with family therapy. Home based family therapy provides a means of working intensively with
severely disturbed children and adolescents. Such intervention can reduce the risk of institution of care
or escalation of the situation.

3. B, C. Methyphenidate is the stimulant medication that‘s considered to be the drug of choice for
ADHD. Atomoxetine is the non stimulant nor epinephrine uptake inhibitor that is also licensed for
ADHD in children. If atomoxetine is not mentioned as one of the choice then perhaps family therapy
may be an option
Incorrect
Marks for this submission: 0/3.
Question 111
Marks: 0/3
Personality Disorders

A - Anankastic personality disorder


B - Anxious-avoidant personality disorder
C - Dependent personality disorder
D - Dissocial personality disorder
E - Emotionally unstable borderline type
F - Histrionic personality disorder
G - Narcissistic personality disorder
H - Paranoid personality disorder
I - Schizoid personality disorder
J - Schizotypal personality disorder

Choose ONE possible personality disorder for each of the following scenarios: Each option may be
used once, more than once or not at all.
A 55-year-old well-groomed gentleman came to your clinic to see a specialist. He has four previous
failed relationships, and all his sons and daughters are successful and high profile. He claims to have a
lot of money invested on the stock market. He complained that he fell out with his bosses in two
previous jobs, and that is why he was fired on each occasion. He feels that people were jealous of his
capabilities.
A 49-year-old gentleman came to see you in clinic and told you that he doesnt trust women and that is
why he never had any long-term relationships. He also disregards the powers that be, and thinks they
let him down on several occasions. He said other people think that he is odd, and he sometimes got
into arguments that have turned physical.
A 24-year-old lady came to see you in the outpatient clinic and told you about her relationship with
her boyfriend. She noted that he was perfect and she moved in with him because she fell out with her
family who were horrible to her. She had feelings of emptiness but since being with him, she felt
much better. In the next appointment, she tells you she fell out with her boyfriend, and started feeling
depressed and suicidal, but moved in with a new boyfriend who is wonderful and makes her happy.
She tells you you are the only one I can open up to.
Answers

1. G - Narcissistic personality disorder is a condition characterized by an inflated sense of self-


importance, need for admiration, extreme self-involvement, and lack of empathy for others. Individuals
with this disorder are usually arrogantly self-assured and confident. They expect to be noticed as
superior. Vulnerability in self-esteem makes individuals with this disorder very sensitive to criticism or
defeat. Although they may not show it outwardly, criticism may haunt these individuals and may leave
them feeling humiliated, degraded, hollow and empty. They may react with disdain, rage or defiant
counterattack. Their social life is often impaired due to problems derived from entitlement, the need for
admiration and the relative disregard for the sensitivities of others. Though their excessive ambition
and confidence may lead to high achievement; performance may be disrupted due to intolerance of
criticism or defeat.
2. H - Paranoid personality disorder is generally characterized by having a long-standing pattern of
pervasive distrust and suspiciousness of others. A person with paranoid personality disorder will nearly
always believe that other people's motives are suspect or even malevolent. Individuals with Paranoid
Personality Disorder are generally difficult to get along with and often have problems with close
relationships. Their excessive suspiciousness and hostility may be expressed in overt
argumentativeness, aggression, in recurrent complaining, or by quiet, apparently hostile aloofness.

3. E - Emotionally unstable personality disorder borderline type is a personality disorder described as a


prolonged disturbance of personality function in a person characterized by depth and variability of
moods. The disorder typically involves unusual levels of instability in mood; black and white thinking,
or splitting; the disorder often manifests itself in idealization and devaluation episodes, as well as
chaotic and unstable interpersonal relationships, self-image, identity, and behavior; as well as a
disturbance in the individual's sense of self.

Incorrect
Marks for this submission: 0/3.
Question 112
Marks: 0/3
Rating scales in Psychiatry

Options:

A. Abnormal inveoluntary rating scale


B. Becks Depression Inventory
C. Edinburgh post natal depression rating scale
D. General health questionnaire
E. Hamilton depression rating scale
F. Montogmery Asberg depression rating scale
G. PANSS
H. SANS
I. Simpson angus scale
J. UKU side effect rating scale

Lead in: Choose the most appropriate rating scale from above for each of the scenarios below. Each of
the options may be used once, more than once or not at all.

Two observer rated scales to monitor response to treatment in a women with a 6 week old baby
diagnosed to have depression.

One scale for the use of health visitors to screen post partum women for depression.

One scale to assess extrapyramidal side effects.

Answers:

1. E, F. HAM-D and MADRS are two observer rating scales used to monitor changes in depression

2. C. Though both BDI and EPDRS can be used to screen for depression, the Edinburgh postnatal
depression rating scale has been specifically designed to screen for depression in the post natal period.

3. I. Simpson angus scale is used to assess EPSEs.


Incorrect
Marks for this submission: 0/3.
Question 113
Marks: 0/3
Searching for articles

A-""
B - ()
C-[]
D-*
E-#
F - AND
G - NEXT
H - NOT
I - OR

You are trying to search for research articles using PUBMED. For each of the following scenarios,
choose the correct term from above. Each option may be used once, more than once or not at all.
You want to search for all articles on similar topics?
You want the exact study or the closest match of the relevant to your search terms?
You want to find all articles that begin with particular text?
Answers

1. I - The Boolean operators AND, OR, NOT can be used to combine search terms in PubMed. In
PubMed, Boolean operators must be entered in uppercase letters. OR is used to retrieve a set in which
each citation contains at least one of the search terms. Use OR when you want to pull together articles
on similar topics.

2. F - AND is used to retrieve a set in which each citation contains all search terms, and is useful when
you are looking for the article with the closest match to your search terms.

3. D - Truncation can be used when you want PubMed to find all terms that begin with a given text
string. Truncation is represented by the asterisk (*), sometimes referred to as a "wildcard." For
example, let's say you are trying to search all terms that have the root, mimic. If you search mimic*
PubMed will retrieve words such as mimic, mimics, mimicking etc

Reference: Please read the tutorial in Pubmed following this link

Incorrect
Marks for this submission: 0/3.
Question 114
Marks: 0/4
Stages of change

A - Abstinence
B - Action
C - Alcohol dependence syndrome.
D - Contemplation
E - Maintenance
F - Pre-contemplation
G - Preparation
H - Relapse

Choose the most appropriate stage of motivational interviewing based on Prochaska and DiClementes
Stages of Change Model. Each option may be used once, more than once or not at all.
A 40-year-old man with Alcohol Dependence Syndrome doesn't want to stop drinking alcohol as he
does not feel that his drinking is a problem.
A 51-year-old woman with Alcoholic liver disease wants to stop drinking alcohol on her birthday in 2
weeks time.
A 57-year-old man who has not drunk any alcohol for the past 15 years after having had a history of
significant alcohol problems prior to that.
A 35-year-old man with a history suggestive of problematic drinking is not sure about stopping
drinking but is weighing up his options and can recognize the effects of alcohol on him.
Answers

1. F

2. G

3. E

4. D

Stage of Change Characteristics Techniques


Pre- Not currently considering Validate lack of readiness
contemplation change: "Ignorance is Clarify: decision is theirs
bliss" Encourage re-evaluation of
current behavior
Encourage self-exploration, not
action
Explain and personalize the
risk
Contemplation Ambivalent about change: Validate lack of readiness
"Sitting on the fence" Clarify: decision is theirs
Not considering change Encourage evaluation of pros
within the next month and cons of behavior change
Identify and promote new,
positive outcome expectations
Preparation Some experience with Identify and assist in problem
change and are trying to solving re: obstacles
change: "Testing the Help patient identify social
waters" support
Planning to act within Verify that patient has
1month underlying skills for behavior
change
Encourage small initial steps
Action Practicing new behavior Focus on restructuring cues
for and social support
3-6 months Bolster self-efficacy for dealing
with obstacles
Combat feelings of loss and
reiterate long-term benefits
Maintenance Continued commitment to Plan for follow-up support
sustaining new behavior Reinforce internal rewards
Post-6 months to 5 years Discuss coping with relapse
Relapse Resumption of old Evaluate trigger for relapse
behaviors: "Fall from Reassess motivation and
grace" barriers
Plan stronger coping strategies

Incorrect
Marks for this submission: 0/4.
Question 115
Marks: 0/3
Statistical Tests

A - ANCOVA
B - ANOVA
C - Chi square test
D - Fishers exact test
E - Friedman test
F - Independent T test
G - Kruskal-Wallis test
H - Mantel Haenzel test
I - Multiple regression analysis
J - Paired T test

Select the most appropriate statistical test for the following. Each option may be used once, more than
once or not at all.
To compare 2 groups on a variable with normal distribution after adjusting for any confounding
variables.
To compare three groups of continuous variables which are shown to have skewed distribution.
To compare the difference in Blood pressure measurement taken at 3 weeks and 6 weeks, in a sample
of obese young men.
Answers

1. I - Multiple regression analysis would identify the effect of confounders on two or more continuous
variables with normal distribution.
Trisha Greenhalgh, How to read a paper: Statistics for the non-statistician. I: Different types of data
need different statistical tests. British Medical Journal, 1997, (315), pp 364-366.
2. G - Kruskal-Wallis test would be the best choice for comparing three continuous variables with
skewed distributions. If the distribution were normal, then ANOVA would be the statistical test of
choice.
Trisha Greenhalgh, How to read a paper: Statistics for the non-statistician. I: Different types of data
need different statistical tests. British Medical Journal, 1997, (315), pp 364-366.
3. J - As BP (continuous variable) is being measured from the same group at two different time points,
a paired t test would be best suited to compare this data.
Trisha Greenhalgh, How to read a paper: Statistics for the non-statistician. I: Different types of data
need different statistical tests. British Medical Journal, 1997, (315), pp 364-366.
Incorrect
Marks for this submission: 0/3.
Question 116
Marks: 0/3
Treatment of reversible dementia

A - Carbimazole
B - Dexamethasone
C - Beta Interferon
D - Neomycin
E - Surgical evacuation
F - Surgical shunt
G - Thiamine
H - Thyroxine
I - Vincristine
J - Vitamin B12

Choose the most appropriate treatment from the list of options above, for each of the scenarios below.
Each of the option may be used once, more than once or not at all.
A 55-year-old lady presents with a history of memory disturbance and early morning headaches
which tend to get better as the day progresses. On examination, there is facial nerve palsy. (Choose
one).
A 60-year-old lady presents with long standing memory problems, ataxia, nystagmus and asterixis.
(Choose one).
A 65-year-old lady presents with mild cognitive impairment, urinary incontinence and ataxia. (Choose
one).
Answers

1. E - This seems to be the most appropriate choice as the scenario seems to point towards a classical
case of brain tumour Early morning headache, and focal neurological sign. It is important to bear in
mind that all early morning headache is not related to tumors Please read -Larner AJ. Not all early
morning headaches are due to brain tumors. Practical Neurology 2009;9:80-84.

2. G - This scenario is suggestive of a liver disease & Wernickes encephalopathy secondary to


alcoholism. The treatment would be Thiamine.

3. F - This scenario is suggestive of Normal Presure Hydrocephalus triad of Urinary incontinence,


Ataxia and Memory difficulties. The treatment is usually a surgical shunt incontinence, Ataxia and
Memory difficulties. The treatment is usually a surgical shunt.

Das könnte Ihnen auch gefallen